• Shuffle
    Toggle On
    Toggle Off
  • Alphabetize
    Toggle On
    Toggle Off
  • Front First
    Toggle On
    Toggle Off
  • Both Sides
    Toggle On
    Toggle Off
  • Read
    Toggle On
    Toggle Off
Reading...
Front

Card Range To Study

through

image

Play button

image

Play button

image

Progress

1/1001

Click to flip

Use LEFT and RIGHT arrow keys to navigate between flashcards;

Use UP and DOWN arrow keys to flip the card;

H to show hint;

A reads text to speech;

1001 Cards in this Set

  • Front
  • Back
1. The driving force of the ventilator (Datex-Ohmeda 7000, 7810, 7100, and 7900) on the anesthesia workstation is accomplished with A. Compressed oxygen B. Compressed air C. Electricity alone D. Electricity and compressed oxygen
1. (A) The control mechanism of standard anesthesia ventilators, such as the Ohmeda 7000, uses compressed oxygen (100%) to compress the ventilator bellows and electric power for the timing circuits. Some ventilators (e.g., North American Dräger AV-E and AV-2+) use a Venturi device, which mixes oxygen and air. Still other ventilators use sophisticated digital controls that allow advanced ventilation modes. These ventilators use an electric stepper motor attached to a piston (Miller: Miller’s Anesthesia, ed 8, p 757; Ehrenwerth: Anesthesia Equipment: Principles and Applications, ed 2, pp 160–161; Miller: Basics of Anesthesia, ed 6, pp 208–209).
2. Select the correct statement regarding color Doppler imaging. A. It is a form of M-mode echocardiography B. The technology is based on continuous wave Doppler C. By convention, motion toward the Doppler is red and motion away from the Doppler is blue D. Two ultrasound crystals are used: one for transmission of the ultrasound signal and one for reception of the returning wave
2. (C) Continuous wave Doppler—Continuous wave Doppler uses two dedicated ultrasound crystals, one for continuous transmission and a second for continuous reception of ultrasound signals. This permits measurement of very high frequency Doppler shifts or velocities. The “cost” is that this technique receives a continuous signal along the entire length of the ultrasound beam. It is used for measuring very high velocities (e.g., as seen in aortic stenosis). Also, continuous wave Doppler cannot spatially locate the source of high velocity (e.g., differentiate a mitral regurgitation velocity from aortic stenosis; both are systolic velocities). Pulsed Doppler—In contrast to continuous wave Doppler, which records the signal along the entire length of the ultrasound beam, pulsed wave Doppler permits sampling of blood flow velocities from a specific region. This modality is particularly useful for assessing the relatively low velocity flows associated with transmitral or transtricuspid blood flow, pulmonary venous flow, and left atrial appendage flow or for confirming the location of eccentric jets of aortic insufficiency or mitral regurgitation. To permit this, a pulse of ultrasound is transmitted, and then the receiver “listens” during a subsequent interval defined by the distance from the transmitter and the sample site. This transducer mode of transmitwait- receive is repeated at an interval termed the pulse-repetition frequency (PRF). The PRF is therefore depth dependent, being greater for near regions and lower for distant or deeper regions. The distance from the transmitter to the region of interest is called the sample volume, and the width and length of the sample volume are varied by adjusting the length of the transducer “receive” interval. In contrast to continuous wave Doppler, which is sometimes performed without two-dimensional guidance, pulsed Doppler is always performed with two-dimensional guidance to determine the sample volume position. Because pulsed wave Doppler echo repeatedly samples the returning signal, there is a maximum limit to the frequency shift or velocity that can be measured unambiguously. Correct identification of the frequency of an ultrasound waveform requires sampling at least twice per wavelength. Thus, the maximum detectable frequency shift, or Nyquist limit, is one half the PRF. If the velocity of interest exceeds the Nyquist limit, “wraparound” of the signal occurs, first into the reverse channel and then back to the forward channel; this is known as aliasing (Miller: Basics of Anesthesia, ed 6, pp 325–327).
3. When the pressure gauge on a size “E” compressedgas cylinder containing N2O begins to fall from its previous constant pressure of 750 psi, approximately how many liters of gas will remain in the cylinder? A. 200 L B. 400 L C. 600 L D. Cannot be calculated
3. (B) The pressure gauge on a size “E” compressed-gas cylinder containing liquid N2O shows 750 psi when it is full and will continue to register 750 psi until approximately three fourths of the N2O has left the cylinder (i.e., liquid N2O has all been vaporized). A full cylinder of N2O contains 1590 L. Therefore, when 400 L of gas remain in the cylinder, the pressure within the cylinder will begin to fall (Miller: Basics of Anesthesia, ed 6, p 201; Butterworth: Morgan & Mikhail’s Clinical Anesthesiology, ed 5, pp 12–13).
4. What percent desflurane is present in the vaporizing chamber of a desflurane vaporizer (pressurized to 1500 mm Hg and heated to 23° C)? A. Nearly 100% B. 85% C. 65% D. 45%
4. (D) Desflurane is unique among the current commonly used volatile anesthetics because of its high vapor pressure of 664 mm Hg. Because of the high vapor pressure, the vaporizer is pressurized to 1500 mm Hg and electrically heated to 23° C to give more predicable concentrations: 664/1500 = about 44%. If desflurane were used at 1 atmosphere, the concentration would be about 88% (Barash: Clinical Anesthesia, ed 7, pp 666–668; Miller: Basics of Anesthesia, ed 6, pp 202–203; Butterworth: Morgan & Mikhail’s Clinical Anesthesiology, ed 5, pp 60–64).
5. If the internal diameter of an intravenous catheter were doubled, flow through the catheter would be A. Decreased by a factor of 2 B. Decreased by a factor of 4 C. Increased by a factor of 8 D. Increased by a factor of 16
5. (D) Factors that influence the rate of laminar flow of a substance through a tube are described by the Hagen- Poiseuille law of friction. The mathematic expression of the Hagen-Poiseuille law of friction is as follows: ˙ (� P) V = �r4 Anesthesia Equipment and Physics 11 where ˙V is the flow of the substance, r is the radius of the tube, ΔP is the pressure gradient down the tube, L is the length of the tube, and μ is the viscosity of the substance. Note that the rate of laminar flow is proportional to the radius of the tube to the fourth power. If the diameter of an intravenous catheter is doubled, flow would increase by a factor of two raised to the fourth power (i.e., a factor of 16) (Ehrenwerth: Anesthesia Equipment: Principles and Applications, ed 2, pp 377–378).
6. A size “E” compressed-gas cylinder completely filled with N2O contains how many liters? A. 1160 L B. 1470 L C. 1590 L D. 1640 L
6. (C) The World Health Organization requires that compressed-gas cylinders containing N2O for medical use be painted blue. Size “E” compressed-gas cylinders completely filled with liquid N2O contain approximately 1590 L of gas. See table from Explanation 10 (Miller: Basics of Anesthesia, ed 6, p 201; Butterworth: Morgan & Mikhail’s Clinical Anesthesiology, ed 5, p 12).
7. Which of the following methods can be used to detect all leaks in the low-pressure circuit of all contemporary anesthesia machines? A. Negative-pressure leak test B. Common gas outlet occlusion test C. Traditional positive-pressure leak test D. None of the above
7. (D) Anesthesia machines should be checked each day before their use. For most machines, three parts are checked before use: calibration for the oxygen analyzer, the low-pressure circuit leak test, and the circle system. Many consider the low-pressure circuit the area most vulnerable for problems because it is more subject to leaks. Leaks in this part of the machine have been associated with intraoperative awareness (e.g., loose vaporizer filling caps) and hypoxia. To test the low-pressure part of the machine, several tests have been used. For the positive-pressure test, positive pressure is applied to the circuit by depressing the oxygen flush button and occluding the Y-piece of the circle system (which is connected to the endotracheal tube or the anesthesia mask during anesthetic administration) and looking for positive pressure detected by the airway pressure gauge. A leak in the low-pressure part of the machine or the circle system will be demonstrated by a decrease in airway pressure. With many newer machines, a check valve is positioned downstream from the flowmeters (rotameters) and vaporizers but upstream from the oxygen flush valve, which would not permit the positive pressure from the circle system to flow back to the low-pressure circuit. In these machines with the check valve, the positive-pressure reading will fall only with a leak in the circle part, but a leak in the low-pressure circuit of the anesthesia machine will not be detected. In 1993, use of the U.S. Food and Drug Administration universal negative-pressure leak test was encouraged, whereby the machine master switch and the flow valves are turned off, and a suction bulb is collapsed and attached to the common or fresh gas outlet of the machine. If the bulb stays fully collapsed for at least 10 seconds, a leak did not exist (this needs to be repeated for each vaporizer, each one opened at a time). Of course, when the test is completed, the fresh gas hose is reconnected to the circle system. Because machines continue to be developed and to differ from one another, you should be familiar with each manufacturer’s machine preoperative checklist. For example, the negative-pressure leak test is recommended for Ohmeda Unitrol, Ohmeda 30/70, Ohmeda Modulus I, Ohmeda Modulus II and II plus, Ohmeda Excel series, Ohmeda CD, and Datex-Ohmeda Aestiva. The Dräger Narkomed 2A, 2B, 2C, 3, 4, and GS require a positive-pressure leak test. The Fabius GS, Narkomed 6000, and Datex-Ohmeda S5/ADU have self-tests (Butterworth: Morgan & Mikhail’s Clinical Anesthesiology, ed 5, pp 83–85; Miller: Miller’s Anesthesia, ed 8, pp 752–755). Check valve Oxygen flush Machine outlet valve Suction bulb –65 cm Suction bulb Leak Machine outlet Check valve Oxygen flush valve 0 cm Negative Pressure Leak Test 12 Part 1 Basic Sciences
8. Which of the following valves prevents transfilling between compressed-gas cylinders? A. Fail-safe valve B. Check valve C. Pressure-sensor shutoff valve D. Adjustable pressure-limiting valve
8. (B) Check valves permit only unidirectional flow of gases. These valves prevent retrograde flow of gases from the anesthesia machine or the transfer of gas from a compressed-gas cylinder at high pressure into a container at a lower pressure. Thus, these unidirectional valves will allow an empty compressed-gas cylinder to be exchanged for a full one during operation of the anesthesia machine with minimal loss of gas. The adjustable pressure-limiting valve is a synonym for a pop-off valve. A fail-safe valve is a synonym for a pressure-sensor shutoff valve. The purpose of a fail-safe valve is to discontinue the flow of N2O (or proportionally reduce it) if the O2 pressure within the anesthesia machine falls below 30 psi (Miller: Miller’s Anesthesia, ed 8, p 756).
9. The expression that for a fixed mass of gas at constant temperature, the product of pressure and volume is constant is known as A. Graham’s law B. Charles’ law C. Boyle’s law D. Dalton’s law
9. (C) Boyle’s law states that for a fixed mass of gas at a constant temperature, the product of pressure and volume is constant. This concept can be used to estimate the volume of gas remaining in a compressedgas cylinder by measuring the pressure within the cylinder (Ehrenwerth: Anesthesia Equipment: Principles and Applications, ed 2, p 4).
10. The pressure gauge on a size “E” compressed-gas cylinder containing O2 reads 1600 psi. How long could O2 be delivered from this cylinder at a rate of 2 L/min? A. 90 minutes B. 140 minutes C. 250 minutes D. 320 minutes
10. (C) U.S. manufacturers require that all compressed-gas cylinders containing O2 for medical use be painted green. A compressed-gas cylinder completely filled with O2 has a pressure of approximately 2000 psi and contains approximately 625 L of gas. According to Boyle’s law, the volume of gas remaining in a closed container can be estimated by measuring the pressure within the container. Therefore, when the pressure gauge on a compressed-gas cylinder containing O2 shows a pressure of 1600 psi, the cylinder contains 500 L of O2. At a gas flow of 2 L/min, O2 could be delivered from the cylinder for approximately 250 minutes (Ehrenwerth: Anesthesia Equipment: Principles and Applications, ed 2, p 4; Butterworth: Morgan & Mikhail’s Clinical Anesthesiology, ed 5, pp 10–12). CHARACTERISTICS OF COMPRESSED GASES STORED IN “E” SIZE CYLINDERS THAT MAY BE ATTACHED TO THE ANESTHESIA MACHINE Characteristics Oxygen N2O CO2 Air Cylinder color Green* Blue Gray Yellow* Physical state in cylinder Gas Liquid and gas Liquid and gas Gas Cylinder contents (L) 625 1590 1590 625 Cylinder weight empty (kg) 5.90 5.90 5.90 5.90 Cylinder weight full (kg) 6.76 8.80 8.90 Cylinder pressure full (psi) 2000 750 838 1800 *The World Health Organization specifies that cylinders containing oxygen for medical use be painted white, but manufacturers in the United States use green. Likewise, the international color for air is white and black, whereas cylinders in the United States are color-coded yellow. From Miller RD: Basics of Anesthesia, ed 6, Philadelphia, Saunders, 2011, p 201, Table 15-2.
11. A 25-year-old healthy patient is anesthetized for a femoral hernia repair. Anesthesia is maintained with isoflurane and N2O 50% in O2, and the patient’s lungs are mechanically ventilated. Suddenly, the “low-arterial saturation” warning signal on the pulse oximeter gives an alarm. After the patient is disconnected from the anesthesia machine, he undergoes ventilation with an Ambu bag with 100% O2 without difficulty, and the arterial saturation quickly improves. During inspection of your anesthesia equipment, you notice that the bobbin in the O2 rotameter is not rotating. This most likely indicates A. Flow of O2 through the O2 rotameter B. No flow of O2 through the O2 rotameter C. A leak in the O2 rotameter below the bobbin D. A leak in the O2 rotameter above the bobbin
11. (B) Given the description of the problem, no flow of O2 through the O2 rotameter is the correct choice. In a normally functioning rotameter, gas flows between the rim of the bobbin and the wall of the Thorpe tube, causing the bobbin to rotate. If the bobbin is rotating, you can be certain that gas is flowing through the rotameter and that the bobbin is not stuck (Ehrenwerth: Anesthesia Equipment: Principles and Applications, ed 2, pp 43–45). Anesthesia Equipment and Physics 13 N2O Oxygen supply failure alarm Pipeline pressure Cylinder gauge pressure gauge N2O cylinder supply N2O pipeline supply N2O O2 O2 Check valve “Fail-safe” valve Flowmeters Low-pressure circuit Flow-control valve Pressure regulator Calibrated vaporizers Check valve (or internal to vaporizer) Second stage O2 pressure regulator O2 pipeline supply O2 cylinder supply Oxygen flush valve Machine outlet (common gas outlet)
12. The O2 pressure-sensor shutoff valve requires what O2 pressure to remain open and allow N2O to flow into the N2O rotameter? A. 10 psi B. 30 psi C. 50 psi D. 100 psi
12. (B) Fail-safe valve is a synonym for pressure-sensor shutoff valve. The purpose of the fail-safe valve is to prevent the delivery of hypoxic gas mixtures from the anesthesia machine to the patient resulting from failure of the O2 supply. Most modern anesthesia machines, however, would not allow a hypoxic mixture, because the knob controlling the N2O is linked to the O2 knob. When the O2 pressure within the anesthesia machine decreases below 30 psi, this valve discontinues the flow of N2O or proportionally decreases the flow of all gases. It is important to realize that this valve will not prevent the delivery of hypoxic gas mixtures or pure N2O when the O2 rotameter is off, because the O2 pressure within the circuits of the anesthesia machine is maintained by an open O2 compressed-gas cylinder or a central supply source. Under these circumstances, an O2 analyzer will be needed to detect the delivery of a hypoxic gas mixture (Ehrenwerth: Anesthesia Equipment: Principles and Applications, ed 2, pp 37–40; Miller: Basics of Anesthesia, ed 6, pp 199–200).
13. A 78-year-old patient is anesthetized for resection of a liver tumor. After induction and tracheal intubation, a 20-gauge arterial line is placed and connected to a transducer that is located 20 cm below the level of the heart. The system is zeroed at the stopcock located at the wrist while the patient’s arm is stretched out on an arm board. How will the arterial line pressure compare with the true blood pressure (BP)? A. It will be 20 mm Hg higher B. It will be 15 mm Hg higher C. It will be the same D. It will be 15 mm Hg lower
13. (C) It is important to zero the electromechanical transducer system with the reference point at the approximate level of the heart. This will eliminate the effect of the fluid column of the transducer system on the arterial BP reading of the system. In this question, the system was zeroed at the stopcock, which was located at the patient’s wrist (approximate level of the ventricle). The BP expressed by the arterial line will therefore be accurate, provided the stopcock remains at the wrist and the transducer is not moved once zeroed. Raising the arm (e.g., 15 cm) decreases the BP at the wrist but increases the pressure on the transducer by the same amount (i.e., the vertical tubing length is now 15 cm H2O higher than before) (Ehrenwerth: Anesthesia Equipment: Principles and Applications, ed 2, pp 276–278; Miller: Miller’s Anesthesia, ed 8, pp 1354–1355).
14. The second-stage O2 pressure regulator delivers a constant O2 pressure to the rotameters of A. 4 psi B. 8 psi C. 16 psi D. 32 psi
14. (C) O2 and N2O enter the anesthesia machine from a central supply source or compressed-gas cylinders at pressures as high as 2200 psi (O2) and 750 psi (N2O). First-stage pressure regulators reduce these pressures to approximately 45 psi. Before entering the rotameters, second-stage O2 pressure regulators further reduce the pressure to approximately 14 to 16 psi (Miller: Miller’s Anesthesia, ed 8, p 761). 14 Part 1 Basic Sciences
15. The highest trace concentration of N2O allowed in the operating room (OR) atmosphere by the National Institute for Occupational Safety and Health (NIOSH) is A. 1 part per million (ppm) B. 5 ppm C. 25 ppm D. 50 ppm
15. (C) NIOSH sets guidelines and issues recommendations concerning the control of waste anesthetic gases. NIOSH mandates that the highest trace concentration of N2O contamination of the OR atmosphere should be less than 25 ppm. In dental facilities where N2O is used without volatile anesthetics, NIOSH permits up to 50 ppm (Butterworth: Morgan & Mikhail’s Clinical Anesthesiology, ed 5, p 81).
16. A sevoflurane vaporizer will deliver an accurate concentration of an unknown volatile anesthetic if the latter shares which property with sevoflurane? A. Molecular weight B. Oil/gas partition coefficient C. Vapor pressure D. Blood/gas partition coefficient
16. (C) Agent-specific vaporizers, such as the Sevotec (sevoflurane) vaporizer, are designed for each volatile anesthetic. However, volatile anesthetics with identical saturated vapor pressures can be used interchangeably, with accurate delivery of the volatile anesthetic. Although halothane is no longer used in the United States, that vaporizer, for example, may still be used in developing countries for administration of isoflurane (Butterworth: Morgan & Mikhail’s Clinical Anesthesiology, ed 5, pp 61–63; Ehrenwerth: Anesthesia Equipment: Principles and Applications, ed 2, pp 72–73). VAPOR PRESSURES Agent Vapor Pressure mm Hg at 20° C Halothane 243 Sevoflurane 160 Isoflurane 240 Desflurane 669
17. A 58-year-old patient has severe shortness of breath and “wheezing.” On examination, the patient is found to have inspiratory and expiratory stridor. Further evaluation reveals marked extrinsic compression of the midtrachea by a tumor. The type of airflow at the point of obstruction within the trachea is A. Laminar flow B. Turbulent flow C. Undulant flow D. Stenotic flow
17. (B) Turbulent flow occurs when gas flows through a region of severe constriction such as that described in this question. Laminar flow occurs when gas flows down parallel-sided tubes at a rate less than critical velocity. When the gas flow exceeds the critical velocity, it becomes turbulent (Butterworth: Morgan & Mikhail’s Clinical Anesthesiology, ed 5, pp 488–489).
18. Concerning the patient in Question 17, administration of 70% helium in O2 instead of 100% O2 will decrease the resistance to airflow through the stenotic region within the trachea because A. Helium decreases the viscosity of the gas mixture B. Helium decreases the friction coefficient of the gas mixture C. Helium decreases the density of the gas mixture D. Helium increases the Reynolds number of the gas mixture
18. (C) During turbulent flow, the resistance to gas flow is directly proportional to the density of the gas mixture. Substituting helium for oxygen will decrease the density of the gas mixture, thereby decreasing the resistance to gas flow (as much as threefold) through the region of constriction (Butterworth: Morgan & Mikhail’s Clinical Anesthesiology, ed 5, pp 498–499, 1286–1287; Ehrenwerth: Anesthesia Equipment: Principles and Applications, ed 2, pp 230–234).
19. A 56-year-old patient is brought to the OR for elective replacement of a stenotic aortic valve. An awake 20-gauge arterial catheter is placed into the right radial artery and is then connected to a transducer located at the same level as the patient’s left ventricle. The entire system is zeroed at the transducer. Several seconds later, the patient raises both arms into the air until his right wrist is 20 cm above his heart. As he is doing this the BP on the monitor reads 120/80 mm Hg. What would this patient’s true BP be at this time? A. 140/100 mm Hg B. 135/95 mm Hg C. 120/80 mm Hg D. 105/65 mm Hg Anesthesia Equipment and Physics 3
19. (C) Modern electronic BP monitors are designed to interface with electromechanical transducer systems. These systems do not require extensive technical skill on the part of the anesthesia provider for accurate use. A static zeroing of the system is built into most modern electronic monitors. Thus, after the zeroing procedure is accomplished, the system is ready for operation. The system should be zeroed with the reference point of the transducer at the approximate level of the aortic root, eliminating the effect of the fluid column of the system on arterial BP readings (Ehrenwerth: Anesthesia Equipment: Principles and Applications, ed 2, pp 276–278).
20. An admixture of room air in the waste gas disposal system during an appendectomy in a paralyzed, mechanically ventilated patient under general volatile anesthesia can best be explained by which mechanism of entry? A. Positive-pressure relief valve B. Negative-pressure relief valve C. Soda lime canister D. Ventilator bellows
20. (B) Waste gas disposal systems, also called scavenging systems, are designed to decrease pollution in the OR by anesthetic gases. These scavenging systems can be passive (waste gases flow from the anesthesia machine to a ventilation system on their own) or active (anesthesia machine is connected to a vacuum system, then to the ventilation system). Positive-pressure relief valves open if there is an obstruction between the anesthesia machine and the disposal system, which would then leak the gas into the OR. A leak in the soda lime canisters would also vent to the OR. Given that most ventilator bellows are powered by oxygen, a leak in the bellows will not add air to the evacuation system. The negative-pressure relief valve is used in active systems and will entrap room air if the pressure in the system is less than −0.5 cm H2O (Miller: Miller’s Anesthesia, ed 8, p 802; Miller: Basics of Anesthesia, ed 6, pp 212; Ehrenwerth: Anesthesia Equipment: Principles and Applications, ed 2, pp 101–103).
21. The relationship between intra-alveolar pressure, surface tension, and the radius of an alveolus is described by A. Graham’s law B. Beer’s law C. Bernoulli’s law D. Laplace’s law
21. (D) The relationship between intra-alveolar pressure, surface tension, and the radius of alveoli is described by Laplace’s law for a sphere, which states that the surface tension of the sphere is directly proportional to the radius of the sphere and pressure within the sphere. With regard to pulmonary alveoli, the mathematic expression of Laplace’s law is as follows: T = (1/2) PR Anesthesia Equipment and Physics 15 where T is the surface tension, P is the intra-alveolar pressure, and R is the radius of the alveolus. In pulmonary alveoli, surface tension is produced by a liquid film lining the alveoli. This occurs because the attractive forces between the molecules of the liquid film are much greater than the attractive forces between the liquid film and gas. Thus, the surface area of the liquid tends to become as small as possible, which could collapse the alveoli (Butterworth: Morgan & Mikhail’s Clinical Anesthesiology, ed 5, pp 493–494; Miller: Miller’s Anesthesia, ed 8, p 475).
22. Currently, the commonly used vaporizers (e.g., GEDatex- Ohmeda Tec 4, Tec 5, Tec 7; Dräger Vapor 19.n and 2000 series) are described as having all of the following features EXCEPT A. Agent specificity B. Variable bypass C. Bubble through D. Temperature compensated
22. (C) Because volatile anesthetics have different vapor pressures, the vaporizers are agent specific. Vaporizers are described as having variable bypass, which means that some of the total fresh gas flow (usually less than 20%) is diverted into the vaporizing chamber, and the rest bypasses the vaporizer. Tipping the vaporizers (which should not occur) may cause some of the liquid to enter the bypass circuit, leading to a high concentration of anesthetic being delivered to the patient. The gas that enters the vaporizer flows over (does not bubble through) the volatile anesthetic. The older (now obsolete) Copper Kettle and Vern-Trol vaporizers were not agent specific, and oxygen (with a separate flowmeter) was bubbled through the volatile anesthetic; then, the combination of oxygen with volatile gas was diluted with the fresh gas flow (oxygen, air, N2O) and administered to the patient. Because vaporization changes with temperature, modern vaporizers are designed to maintain a constant concentration over clinically used temperatures (20° C-35° C) (Barash: Clinical Anesthesia, ed 7, pp 661–672; Miller: Basics of Anesthesia, ed 6, pp 202–203; Butterworth: Morgan & Mikhail’s Clinical Anesthesiology, ed 5, pp 60–64).
23. For any given concentration of volatile anesthetic, the splitting ratio is dependent on which of the following characteristics of that volatile anesthetic? A. Vapor pressure B. Molecular weight C. Specific heat D. Minimum alveolar concentration (MAC) at 1 atmosphere
23. (A) Vaporizers can be categorized into variable-bypass and measured-flow vaporizers. Measured-flow vaporizers (nonconcentration calibrated vaporizers) include the obsolete Copper Kettle and Vernitrol vaporizers. With measured-flow vaporizers, the flow of oxygen is selected on a separate flowmeter to pass into the vaporizing chamber, from which the anesthetic vapor emerges at its saturated vapor pressure. By contrast, in variable-bypass vaporizers, the total gas flow is split between a variable bypass and the vaporizer chamber containing the anesthetic agent. The ratio of these two flows is called the splitting ratio. The splitting ratio depends on the anesthetic agent, the temperature, the chosen vapor concentration set to be delivered to the patient, and the saturated vapor pressure of the anesthetic (Ehrenwerth: Anesthesia Equipment: Principles and Applications, ed 2, pp 68–71).
24. A mechanical ventilator (e.g., Ohmeda 7000) is set to deliver a tidal volume (VT) of 500 mL at a rate of 10 breaths/min and an inspiratory-to-expiratory (I:E) ratio of 1:2. The fresh gas flow into the breathing circuit is 6 L/min. In a patient with normal total pulmonary compliance, the actual VT delivered to the patient would be A. 500 mL B. 600 mL C. 700 mL D. 800 mL
24. (C) The contribution of the fresh gas flow from the anesthesia machine to the patient’s VT should be considered when setting the VT of a mechanical ventilator. Because the ventilator pressure-relief valve is closed during inspiration, both the gas from the ventilator bellows and the fresh gas flow will be delivered to the patient’s breathing circuit. In this question, the fresh gas flow is 6 L/min, or 100 mL/sec (6000 mL/60 sec). Each breath lasts 6 seconds (60 sec/10 breaths), with inspiration lasting 2 seconds (I:E ratio = 1:2). Under these conditions, the 500 VT delivered to the patient by the mechanical ventilator will be augmented by approximately 200 mL. In some ventilators, such as the Ohmeda 7900, VT is controlled for the fresh gas flow rate in such a manner that the delivered VT is always the same as the dial setting (Butterworth: Morgan & Mikhail’s Clinical Anesthesiology, ed 5, pp 79–81).
25. In reference to Question 24, if the ventilator rate were decreased from 10 to 6 breaths/min, the approximate VT delivered to the patient would be A. 600 mL B. 700 mL C. 800 mL D. 900 mL
25. (C) The ventilator rate is decreased from 10 to 6 breaths/min. Thus, each breath will last 10 seconds (60 sec/6 breaths), with inspiration lasting approximately 3.3 seconds (I:E ratio = 1:2) (i.e., 3.3 seconds × 100 mL/sec). Under these conditions, the actual VT delivered to the patient by the mechanical ventilator will be 830 mL (500 mL + 330 mL) (Butterworth: Morgan & Mikhail’s Clinical Anesthesiology, ed 5, pp 79–81).
26. A 65-year-old patient is mechanically ventilated in the intensive care unit (ICU) after an open nephrectomy. How far should the suction catheter be inserted into the endotracheal tube for suctioning? A. To the midlevel of the endotracheal tube B. To the tip of the endotracheal tube C. Just proximal to the carina D. Past the carina
26 (B) Endotracheal tubes frequently become partially or completely occluded with secretions. Periodic suctioning of the endotracheal tube in the ICU assures patency of the artificial airway. There are hazards, however, of endotracheal tube suctioning. They include mucosal trauma, cardiac dysrhythmias, hypoxia, increased intracranial pressure, colonization of the distal airway, and psychologic trauma to the patient. To reduce the possibility of colonization of the distal airway it is prudent to keep the suction catheter within the endotracheal tube during suctioning. Pushing the suctioning catheter beyond the distal limits of the endotracheal tube also may produce suctioning trauma to the tracheal tissue (Tobin: Principles and Practices of Mechanical Ventilation, ed 3, p 1223).
27. If the anesthesia machine is discovered Monday morning to have run with 5 L/min of oxygen all weekend long, the most reasonable course of action before administering the next anesthetic would be to A. Administer 100% oxygen for the first hour of the next case B. Place humidifier in line with the expiratory limb C. Avoid use of sevoflurane D. Change the CO2 absorbent
27. (D) CO can be generated when volatile anesthetics are exposed to CO2 absorbers that contain NaOH or KOH (e.g., soda lime) and have sometimes produced carboxyhemoglobin levels of 35%. Factors that are involved 16 Part 1 Basic Sciences in the production of CO and formation of carboxyhemoglobin include (1) the specific volatile anesthetic used (desflurane ≥ enflurane > isoflurane ≫ sevoflurane = halothane), (2) high concentrations of volatile anesthetic (more CO is generated at higher volatile concentrations), (3) high temperatures (more CO is generated at higher temperatures), (4) low fresh gas flows, and especially (5) dry soda lime (dry granules produce more CO than do hydrated granules). Soda lime contains 15% water by weight, and only when it gets dehydrated to below 1.4% will appreciable amounts of CO be formed. Many of the reported cases of patients experiencing elevated carboxyhemoglobin levels occurred on Monday mornings, when the fresh gas flow on the anesthesia circuit was not turned off and high anesthetic fresh gas flows (>5 L/min) for prolonged periods of time (e.g., >48 hours) occurred. Because of some resistance of the inspiratory valve, retrograde flow through the CO2 absorber (which hastens the drying of the soda lime) will develop, especially if the breathing bag is absent, the Y-piece of the circuit is occluded, and the adjustable pressure-limiting valve is open. Whenever you are uncertain as to the dryness of the CO2 absorber, especially when the fresh gas flow was not turned off the anesthesia machine for an extended or indeterminate period of time, the CO2 absorber should be changed. This CO production occurs with soda lime and occurred more so with Baralyme (which is no longer available), but it does not occur with Amsorb Plus or DrägerSorb Free (which contains calcium chloride and calcium hydroxide and no NaOH or KOH) (Barash: Clinical Anesthesia, ed 7, p 676; Miller: Basics of Anesthesia, ed 6, pp 212–215; Miller: Miller’s Anesthesia, ed 8, pp 789–792).
28. According to NIOSH regulations, the highest concentration of volatile anesthetic contamination allowed in the OR atmosphere when administered in conjunction with N2O is A. 0.5 ppm B. 2 ppm C. 5 ppm D. 25 ppm
28. (A) NIOSH mandates that the highest trace concentration of volatile anesthetic contamination of the OR atmosphere when administered in conjunction with N2O is 0.5 ppm (Butterworth: Morgan & Mikhail’s Clinical Anesthesiology, ed 5, p 81).
29. The device on anesthesia machines that most reliably detects delivery of hypoxic gas mixtures is the A. Fail-safe valve B. O2 analyzer C. Second-stage O2 pressure regulator D. Proportion-limiting control system
29. (B) The O2 analyzer is the last line of defense against the inadvertent delivery of hypoxic gas mixtures. It should be located in the inspiratory (not expiratory) limb of the patient’s breathing circuit to provide maximum safety. Because the O2 concentration in the fresh-gas supply line may be different from that of the patient’s breathing circuit, the O2 analyzer should not be located in the fresh-gas supply line (Ehrenwerth: Anesthesia Equipment: Principles and Applications, ed 2, pp 209–210).
30. A ventilator pressure-relief valve stuck in the closed position can result in A. Barotrauma B. Hypoventilation C. Hyperventilation D. Low breathing circuit pressure
30. (A) The ventilator pressure-relief valve (also called the spill valve) is pressure controlled via pilot tubing that communicates with the ventilator bellows chamber. As pressure within the bellows chamber increases during the inspiratory phase of the ventilator cycle, the pressure is transmitted via the pilot tubing to close the pressure-relief valve, thus making the patient’s breathing circuit “gas tight.” This valve should open during the expiratory phase of the ventilator cycle to allow the release of excess gas from the patient’s breathing circuit into the waste-gas scavenging circuit after the bellows has fully expanded. If the ventilator pressure-relief valve were to stick in the closed position, there would be a rapid buildup of pressure within the circle system that would be readily transmitted to the patient. Barotrauma to the patient’s lungs would result if this situation were to continue unrecognized (Butterworth: Morgan & Mikhail’s Clinical Anesthesiology, ed 5, pp 34, 79–80). Anesthesia Equipment and Physics 17
31. A mixture of 1% isoflurane, 70% N2O, and 30% O2 is administered to a patient for 30 minutes. The expired isoflurane concentration measured is 1%. N2O is shut off, and a mixture of 30% O2 and 70% N2 with 1% isoflurane is administered. The expired isoflurane concentration measured 1 minute after the start of this new mixture is 2.3%. The best explanation for this observation is A. Intermittent back pressure (pumping effect) B. Diffusion hypoxia C. Concentration effect D. Effect of N2O solubility in isoflurane 4 Part 1 Basic Sciences 50 0 5 PCO2 (mm Hg) 10 15
31. (D) Vaporizer output can be affected by the composition of the carrier gas used to vaporize the volatile agent in the vaporizing chamber, especially when N2O is either initiated or discontinued. This observation can be explained by the solubility of N2O in the volatile agent. When N2O and oxygen enter the vaporizing chamber, a portion of the N2O dissolves in the liquid agent. Thus, the vaporizer output transiently decreases. Conversely, when N2O is withdrawn as part of the carrier gas, the N2O dissolved in the volatile agent comes out of solution, thereby transiently increasing the vaporizer output (Miller: Miller’s Anesthesia, ed 8, pp 769–771).
32. The capnogram waveform above represents which of the following situations? A. Kinked endotracheal tube B. Bronchospasm C. Incompetent inspiratory valve D. Incompetent expiratory valve
32. (D) The capnogram can provide a variety of information, such as verification of exhaled CO2 after tracheal intubation, estimation of the differences in Paco2 and Petco2, abnormalities of ventilation, and hypercapnia or hypocapnia. The four phases of the capnogram are inspiratory baseline, expiratory upstroke, expiratory plateau, and inspiratory downstroke. The shape of the capnogram can be used to recognize and diagnose a variety of potentially adverse circumstances. Under normal conditions, the inspiratory baseline should be 0, indicating that there is no rebreathing of CO2 with a normal functioning circle breathing system. If the inspiratory baseline is elevated above 0, there is rebreathing of CO2. If this occurs, the differential diagnosis should include an incompetent expiratory valve, exhausted CO2 absorbent, or gas channeling through the CO2 absorbent. However, the inspiratory baseline may be elevated when the inspiratory valve is incompetent (e.g., there may be a slanted inspiratory downstroke). The expiratory upstroke occurs when the fresh gas from the anatomic dead space is quickly replaced by CO2-rich alveolar gas. Under normal conditions, the upstroke should be steep; however, it may become slanted during partial airway obstruction, if a sidestream analyzer is sampling gas too slowly, or if the response time of the capnograph is too slow for the patient’s respiratory rate. Partial obstruction may be the result of an obstruction in the breathing system (e.g., by a kinked endotracheal tube) or in the patient’s airway (e.g., chronic obstructive pulmonary disease or acute bronchospasm). The expiratory plateau is normally characterized by a slow but shallow progressive increase in CO2 concentration. This occurs because of imperfect matching of ventilation and perfusion in all lung units. Partial obstruction of gas flow either in the breathing system or in the patient’s airways may cause a prolonged increase in the slope of the expiratory plateau, which may continue rising until the next inspiratory downstroke begins. The inspiratory downstroke is caused by the rapid influx of fresh gas, which washes the CO2 away from the CO2 sensing or sampling site. Under normal conditions, the inspiratory downstroke is very steep. The causes of a slanted or blunted inspiratory downstroke include an incompetent inspiratory valve, slow mechanical inspiration, slow gas sampling, and partial CO2 rebreathing (Ehrenwerth: Anesthesia Equipment: Principles and Applications, ed 2, p 248).
33. Select the FALSE statement. A. If a Magill forceps is used for a nasotracheal intubation, the right nares is preferable for insertion of the nasotracheal tube B. Extension of the neck can convert an endotracheal intubation to an endobronchial intubation C. Bucking signifies the return of the coughing reflex D. Postintubation pharyngitis is more likely to occur in female patients
33. (B) The complications of tracheal intubation can be divided into those associated with direct laryngoscopy and intubation of the trachea, tracheal tube placement, and extubation of the trachea. The most frequent complication associated with direct laryngoscopy and tracheal intubation is dental trauma. If a tooth is dislodged and not found, radiographs of the chest and abdomen should be taken to determine whether the tooth has passed through the glottic opening into the lungs. Should dental trauma occur, immediate consultation with a dentist is indicated. Other complications of direct laryngoscopy and tracheal intubation include hypertension, tachycardia, cardiac dysrhythmias, and aspiration of gastric contents. The most common complication that occurs while the endotracheal tube is in place is inadvertent endobronchial intubation. Flexion, not extension, of the neck or a change from the supine position to the head-down position can shift the carina upward, which may convert a midtracheal tube placement into a bronchial intubation. Extension of the neck can cause cephalad displacement of the tube into the pharynx. Lateral rotation of the head can displace the distal end of the endotracheal tube approximately 0.7 cm away from the carina. The complications associated with extubation of the trachea can be immediate or delayed; of the immediate complications associated with extubation of the trachea, the two most serious are laryngospasm and aspiration of gastric contents. Laryngospasm is most likely to occur in patients who are lightly anesthetized at the time of extubation. If laryngospasm occurs, positive-pressure bag and mask ventilation with 100% O2 and forward displacement of the mandible may be sufficient treatment. However, if laryngospasm persists, succinylcholine should be administered intravenously or intramuscularly. Pharyngitis is another frequent complication after extubation of the trachea. It occurs most commonly in female individuals, presumably because of the thinner mucosal covering over the posterior vocal cords in comparision with male individuals. This complication usually does not require treatment and spontaneously resolves in 48 to 72 hours. Delayed complications associated with extubation of the trachea include laryngeal ulcerations, tracheitis, tracheal stenosis, vocal cord paralysis, and arytenoid cartilage dislocation (Miller: Miller’s Anesthesia, ed 8, p 1655). 18 Part 1 Basic Sciences
34. Gas from an N2O compressed-gas cylinder enters the anesthesia machine through a pressure regulator that reduces the pressure to A. 60 psi B. 45 psi C. 30 psi D. 15 psi
34. (B) Gas leaving a compressed-gas cylinder is directed through a pressure-reducing valve, which lowers the pressure within the metal tubing of the anesthesia machine to 45 to 55 psi (Ehrenwerth: Anesthesia Equipment: Principles and Applications, ed 2, pp 27–34).
35. Eye protection for OR staff is needed when laser surgery is performed. Clear wraparound goggles or glasses are adequate with which kind of laser? A. Argon laser B. Nd:YAG (neodymium:yttrium-aluminum-garnet) laser C. CO2 laser D. None of the above
35. (C) CO2 lasers can cause serious corneal injury, whereas argon, Nd:YAG, ruby, or potassium titanyl phosphate lasers can burn the retina. Use of the incorrect filter provides no protection! Clear glass or plastic lenses are opaque for CO2 laser light and are adequate protection for this beam (contact lenses are not adequate protection). For argon or krypton laser light, amber-orange filters are used. For Nd:YAG laser light, special green-tinted filters are used. For potassium titanyl phosphate:Nd:YAG laser light, red filters are used (Miller: Miller’s Anesthesia, ed 8, pp 2328–2331).
36. Which of the following systems prevents attachment of gas-administering equipment to the wrong type of gas line? A. Pin index safety system B. Diameter index safety system C. Fail-safe system D. Proportion-limiting control system
36. (B) The diameter index safety system prevents incorrect connections of medical gas lines. This system consists of two concentric and specific bores in the body of one connection, which correspond to two concentric and specific shoulders on the nipple of the other connection (Ehrenwerth: Anesthesia Equipment: Principles and Applications, ed 2, pp 20, 27–28).
37. A patient with aortic stenosis is scheduled for laparoscopic cholecystectomy. Preoperative echocardiography demonstrated a peak velocity of 4 m/sec across the aortic valve. If her BP was 130/80 mm Hg, what was the peak pressure in the left ventricle? A. 145 mm Hg B. 160 mm Hg C. 194 mm Hg D. 225 mm Hg
37. (C) The modified Bernoulli equation defines the pressure drop (or gradient) across an obstruction, narrowing, or stenosis as follows: ΔP = 4V2 Where ΔP is the pressure gradient; V is the measured velocity across the stenosis using Doppler echocardiography. In this example, ΔP = 4 × 42 = 64. The peak pressure in the left ventricle is 130 + 64 = 194 mm Hg (Kaplan: Kaplan’s Cardiac Anesthesia: The Echo Era, ed 6, pp 315–382).
38. The dial of an isoflurane-specific, variable bypass, temperature-compensated, flowover, out-of-circuit vaporizer (i.e., modern vaporizer) is set on 2%, and the infrared spectrometer measures 2% isoflurane vapor from the common gas outlet. The flowmeter is set at a rate of 700 mL/min during this measurement. The output measurements are repeated with the flowmeter set at 100 mL/min and 15 L/min (vapor dial still set on 2%). How will these two measurements compare with the first measurement taken? A. Output will be less than 2% in both cases B. Output will be greater than 2% in both cases C. Output will be 2% at 100 mL/min O2 flow and less than 2% at 15 L/min flow D. Output will be less than 2% at 100 mL/min and 2% at 15 L/min
38. (A) The output of the vaporizer will be lower at flow rates less than 250 mL/min because there is insufficient pressure to advance the molecules of the volatile agent upward. At extremely high carrier gas flow rates (>15 L/min), there is insufficient mixing in the vaporizing chamber (Miller: Miller’s Anesthesia, ed 8, pp 777–778).
39. Which of the following would result in the greatest decrease in the arterial hemoglobin saturation (Spo2) value measured by the dual-wavelength pulse oximeter? A. Intravenous injection of indigo carmine B. Intravenous injection of indocyanine green C. Intravenous injection of methylene blue D. Elevation of bilirubin
39. (C) Pulse oximeters estimate arterial hemoglobin saturation (Sao2) by measuring the amount of light transmitted through a pulsatile vascular tissue bed. Pulse oximeters measure the alternating current component of light absorbance at each of two wavelengths (660 and 940 nm) and then divide this measurement by the corresponding direct current component. Then the ratio (R) of the two absorbance measurements is determined by the following equation: R AC660 / DC660 AC940 / DC940 = Using an empiric calibration curve that relates arterial hemoglobin saturation to R, the actual arterial hemoglobin saturation is calculated. Based on the physical principles outlined above, the sources of error in Spo2 readings can be easily predicted. Pulse oximeters can function accurately when only two hemoglobin species, oxyhemoglobin and reduced hemoglobin, are present. If any light-absorbing species other than oxyhemoglobin and reduced hemoglobin are present, the pulse oximeter measurements will be inaccurate. Fetal hemoglobin has a minimal effect on the accuracy of pulse oximetry because the extinction coefficients for fetal hemoglobin at the two wavelengths used by pulse oximetry are very similar to the corresponding values for adult hemoglobin. In addition to abnormal hemoglobins, any substance present in the blood that absorbs light at either 660 or 940 nm, such as intravenous dyes used for diagnostic purposes, will affect the value of R, making accurate measurements of the pulse oximeter impossible. These dyes include methylene blue and indigo carmine. Methylene blue has the greatest effect on Sao2 measurements because the extinction coefficient is so similar to that of oxyhemoglobin (Ehrenwerth: Anesthesia Equipment: Principles and Applications, ed 2, pp 261–262). Anesthesia Equipment and Physics 19
40. Each of the following statements concerning nonelectronic conventional flowmeters (also called rotameters) is true EXCEPT A. Rotation of the bobbin within the Thorpe tube is important for accurate function B. The Thorpe tube increases in diameter from bottom to top C. Its accuracy is affected by changes in temperature and atmospheric pressure D. The rotameters for N2O and CO2 are interchangeable
40. (D) Rotameters consist of a vertically positioned tapered tube that is smallest in diameter at the bottom (Thorpe tube). Gas enters at the bottom of the Thorpe tube and elevates a bobbin or float, which comes to rest when gravity on the float is balanced by the fall in pressure across the float. The rate of gas flow through the tube depends on the pressure drop along the length of the tube, the resistance to gas flow through the tube, and the physical properties (density and viscosity) of the gas. Because few gases have the same density and viscosity, rotameters cannot be used interchangeably (Barash: Clinical Anesthesia, ed 7, pp 655–657; Ehrenwerth: Anesthesia Equipment: Principles and Applications, ed 2, pp 43–45).
41. Which of the following combinations would result in delivery of a lower-than-expected concentration of volatile anesthetic to the patient? A. Sevoflurane vaporizer filled with desflurane B. Isoflurane vaporizer filled with sevoflurane C. Sevoflurane vaporizer filled with isoflurane D. All of the above would result in less than the dialed concentration Anesthesia Equipment and Physics 5
41. (B) Saturated vapor pressures depend on the physical properties of the liquid and the temperature. Vapor pressures are independent of barometric pressure. At 20° C the vapor pressures of halothane (243 mm Hg) and isoflurane (240 mm Hg) are similar, and at 1 atmosphere the concentration in the vaporizer for these drugs is 240/760, or about 32%. Similarly, the vapor pressure for sevoflurane (160 mm Hg) and enflurane (172 mm Hg) are similar, and at 1 atmosphere the concentration in the vaporizer for these drugs is 160/760, or about 21%. If desflurane (vapor pressure of 669 mm Hg) is placed in a 1-atmosphere pressure vaporizer, the concentration would be 669/760 = 88%. Because the bypass flow is adjusted for each vaporizer, putting a volatile anesthetic with a higher saturated vapor pressure would lead to a higher-than-expected concentration of anesthetic delivered from the vaporizer, whereas putting a drug with a lower saturated vapor pressure would lead to a lower-than-expected concentration of drug delivered from the vaporizer (Barash: Clinical Anesthesia, ed 7, pp 661–672). VAPOR PRESSURE AND MINIMUM ALVEOLAR CONCENTRATION Halothane Enflurane Sevoflurane Isoflurane Desflurane Methoxyflurane Vapor pressure 20° C mm Hg 243 172 160 240 669 23 MAC 30-55 yr 0.75 1.63 1.8 1.17 6.6 0.16 MAC, minimum alveolar concentration.
42. At high altitudes, the flow of a gas through a rotameter will be A. Greater than expected B. Less than expected C. Less than expected at high flows but greater than expected at low flows D. Greater than expected at high flows but accurate at low flows
42. (D) Gas density decreases with increasing altitude (i.e., the density of a gas is directly proportional to atmospheric pressure). Atmospheric pressure will influence the function of rotameters because the accurate function of rotameters is influenced by the physical properties of the gas, such as density and viscosity. The magnitude of this influence, however, depends on the rate of gas flow. At low gas flows, the pattern of gas flow is laminar. Atmospheric pressure will have little effect on the accurate function of rotameters at low gas flows because laminar gas flow is influenced by gas viscosity (which is minimally affected by atmospheric pressure), not by gas density. However, at high gas flows, the gas flow pattern is turbulent and is influenced by gas density. At high altitudes (i.e., low atmospheric pressure), the gas flow through the rotameter will be greater than expected at high flows but accurate at low flows (Ehrenwerth: Anesthesia Equipment: Principles and Applications, ed 2, pp 43–45, 230–231).
43. A patient presents for knee arthroscopy and tells his anesthesiologist that he has a VDD pacemaker. Select the true statement regarding this pacemaker. A. It senses and paces only the ventricle B. It paces only the ventricle C. Its response to a sensed event is always inhibition D. It is not useful in a patient with atrioventricular (AV) nodal block
43. (B) Pacemakers have a three- to five-letter code that describes the pacemaker type and function. Given that the purpose of the pacemaker is to send electric current to the heart, the first letter identifies the chamber(s) paced: A for atrial, V for ventricle, and D for dual chamber (A + V). The second letter identifies the chamber where endogenous current is sensed: A,V, D, and O for none sensed. The third letter describes the response to sensing: O for none, I for inhibited, T for triggered, and D for dual (I + T). The fourth letter describes programmability or rate modulation: O for none and R for rate modulation (i.e., faster heart rate with exercise). The fifth letter describes multisite pacing (more important in dilated heart chambers): A, V or D (A + V), or O. A VDD pacemaker is used for patients with AV node dysfunction but intact sinus node activity (Miller: Miller’s Anesthesia, ed 8, pp 1467–1468).
44. All of the following would result in less trace gas pollution of the OR atmosphere EXCEPT A. Use of a high gas flow in a circular system B. Tight mask seal during mask induction C. Use of a scavenging system D. Allow patient to breathe 100% O2 as long as possible before extubation
44. (A) Although controversial, it is thought that chronic exposure to low concentrations of volatile anesthetics may constitute a health hazard to OR personnel. Therefore, removal of trace concentrations of volatile anesthetic gases from the OR atmosphere with a scavenging system and steps to reduce and control gas leakage into the environment are required. High-pressure system leakage of volatile anesthetic gases into the OR atmosphere occurs when gas escapes from compressed-gas cylinders attached to the anesthetic machine (e.g., faulty yokes) or from tubing delivering these gases to the anesthesia machine from a central supply source. The most common cause of low-pressure leakage of anesthetic gases into the OR atmosphere is the escape of gases from 20 Part 1 Basic Sciences sites located between the flowmeters of the anesthesia machine and the patient, such as a poor mask seal. The use of high gas flows in a circle system will not reduce trace gas contamination of the OR atmosphere. In fact, this could contribute to the contamination if there is a leak in the circle system (Miller: Miller’s Anesthesia, ed 8, pp 3232–3234).
45. The greatest source for contamination of the OR atmosphere is leakage of volatile anesthetics A. Around the anesthesia mask B. At the vaporizer C. At the CO2 absorber D. At the endotracheal tube
45. (A) Although there is insufficient evidence that chronic exposure to low concentrations of inhaled anesthetics may pose a health hazard to those in the OR, precautions are made to decrease the pollution of inhalation anesthetics there. This includes ventilating the room adequately (air in the OR should be exchanged at least 15 times an hour), maintenance of anesthetic system scavenging systems to remove anesthetic vapors, and a tight anesthetic seal with no leakage of gas into the OR atmosphere. Although periodic equipment maintenance should be performed to make sure the anesthetic equipment is operating properly, leakage around an improperly sealed face mask as well as the face mask not applied to the face during airway manipulations (placement of an airway) poses the greatest risk of OR contamination from inhaled anesthetics (Barash: Clinical Anesthesia, ed 7, pp 62–64; Miller: Basics of Anesthesia, ed 6, pp 211–212; Ehrenwerth: Anesthesia Equipment: Principles and Applications, ed 2, pp 130–145; Miller: Miller’s Anesthesia, ed 8, pp 3232–3234).
46. Uptake of sevoflurane from the lungs during the first minute of general anesthesia is 50 mL. How much sevoflurane would be taken up from the lungs between the 16th and 36th minutes? A. 25 mL B. 50 mL C. 100 mL D. 500 mL
46. (C) The amount of volatile anesthetic taken up by the patient in the first minute is equal to the amount taken up between the squares of any two consecutive minutes (square root of time equation). Thus, if 50 mL is taken up in the first minute, 50 mL will be taken up between the first (1 squared) and fourth (2 squared) minutes. Similarly, between the fourth and ninth minutes (2 squared and 3 squared), another 50 mL will be absorbed. In this example, we are looking for the uptake between the 16th (4 squared) and 36th (6 squared) minutes, which would be 2 consecutive minutes squared, or 2 × 50 mL = 100 mL (Miller: Miller’s Anesthesia, ed 8, pp 650–651).
47. Which of the drugs below would have the LEAST impact on somatosensory evoked potentials (SSEPs) monitoring in a 15-year-old patient undergoing scoliosis surgery? A. Midazolam B. Propofol C. Isoflurane D. Vecuronium
47. (D) In evaluating SSEPs, one looks at both the amplitude or voltage of the recorded response wave and the latency (time measured from the stimulus to the onset or peak of the response wave). A decrease in amplitude (>50%) and/or an increase in latency (>10%) is usually clinically significant. These changes may reflect hypoperfusion, neural ischemia, temperature changes, or drug effects. All of the volatile anesthetics and the barbiturates cause a decrease in amplitude as well as an increase in latency. Propofol affects both latency and amplitude and, like other intravenous agents, has a significantly less effect than “equipotent” doses of volatile anesthetics. Etomidate causes an increase in latency and an increase in amplitude. Midazolam decreases the amplitude but has little effect on latency. Opioids cause small and not clinically significant increases in latency and a decrease in amplitude of the SSEPs. Muscle relaxants have no effect on SSEPs (Miller: Miller’s Anesthesia, ed 8, pp 1514–1517; Miller: Basics of Anesthesia, ed 6, pp 505–506).
48. Which of the following is NOT found in the lowpressure circuit on an anesthesia machine? A. Oxygen supply failure alarm B. Flowmeters C. Vaporizers D. Vaporizer check valve
48. (A) The anesthesia machine, now more properly called the anesthesia workstation, has two main pressure circuits. The higher-pressure circuits consist of the gas supply from the pipelines and tanks, all piping, pressure gauges, pressure reduction regulators, check valves (which prevent backward gas flow), the oxygen pressuresensor shutoff valve (also called the oxygen failure cutoff or fail-safe valve), the oxygen supply failure alarm, and the oxygen flush valve—or, simplistically, everything up to the gas flow control valves and the machine common gas outlet. The low-pressure circuit starts with and includes the gas flow control valves, flowmeters, vaporizers, and vaporizer check valve and goes to the machine common gas outlet. See also figure for explanation to Question 12 (Barash: Clinical Anesthesia, ed 7, pp 641–650; Miller: Basics of Anesthesia, ed 6, pp 198–204).
49. Frost develops on the outside of an N2O compressedgas cylinder during general anesthesia. This phenomenon indicates that A. The saturated vapor pressure of N2O within the cylinder is rapidly increasing B. The cylinder is almost empty C. There is a rapid transfer of heat to the cylinder D. The flow of N2O from the cylinder into the anesthesia machine is rapid
49. (D) Vaporization of a liquid requires the transfer of heat from the objects in contact with the liquid (e.g., the metal cylinder and surrounding atmosphere). For this reason, at high gas flows, atmospheric water will condense as frost on the outside of compressed-gas cylinders (Butterworth: Morgan & Mikhail’s Clinical Anesthesiology, ed 5, pp 12–13; Miller: Basics of Anesthesia, ed 6, p 201).
50. The LEAST reliable site for central temperature monitoring is the A. Pulmonary artery B. Skin on the forehead C. Distal third of the esophagus D. Nasopharynx
50. (B) Temperature measurements of the pulmonary artery, esophagus, axilla, nasopharynx, and tympanic membrane correlate with central temperature in patients undergoing noncardiac surgery. Skin temperature does not reflect central temperature and does not warn adequately of malignant hyperthermia or excessive hypothermia (Butterworth: Morgan & Mikhail’s Clinical Anesthesiology, ed 5, p 137; Miller: Miller’s Anesthesia, ed 8, pp 1643–1644).
51. Of the following medical lasers, which laser light penetrates tissues the most? A. Argon laser B. Helium–neon laser (He–Ne) C. Nd:YAG (neodymium:yttrium-aluminum-garnet) laser D. CO2 laser
51. (C) Laser refers to Light Amplification by the Stimulated Emission of Radiation. Laser light differs from ordinary light in three main ways. First, laser light is monochromic (possesses one wavelength or color). Second, laser Anesthesia Equipment and Physics 21 light is coherent (the photons oscillate in the same phase). Third, laser light is collimated (exists in a narrow parallel beam). Visible light has a wide spectrum of wavelengths in the 385- to 760-nm range. Argon laser light, which can penetrate tissues to a depth of 0.05 to 2.0 mm, is either blue (wavelength 488 nm) or green (wavelength 514 nm) and is often used for vascular pigmented lesions because it is intensively absorbed by hemoglobin. Helium–neon laser light is red, has a frequency of 632 nm, and is often used as an aiming beam because it has very low power and presents no significant danger to OR personnel. Nd:YAG laser light is the most powerful medical laser and can penetrate tissues from 2 to 6 mm. Nd:YAG laser light is in the near infrared range, with a wavelength of 1064 nm, has general uses (e.g., prostate surgery, laryngeal papillomatosis, coagulation), and can be used with fiberoptics. CO2 laser light is in the far infrared range, with a long wavelength of 10,600 nm. Because CO2 laser light penetrates tissues poorly, it can vaporize superficial tissues with little damage to underlying cells (Barash: Clinical Anesthesia, ed 7, pp 212–214; Butterworth: Morgan & Mikhail’s Clinical Anesthesiology, ed 5, pp 776–777; Miller: Miller’s Anesthesia, ed 8, pp 2598–2601).
52. The reason Heliox (70% helium and 30% oxygen) is more desirable than a mixture of 70% nitrogen and 30% oxygen for a spontaneously breathing patient with tracheal stenosis is that A. Helium has a lower density than nitrogen B. Helium is a smaller molecule than O2 C. Absorption atelectasis is decreased D. Helium has a lower critical velocity for turbulent flow than does O2
52. (A) Normal gas flow is laminar within the trachea, but with tracheal stenosis, airflow is more turbulent. Resistance during turbulent flow depends on gas density, and helium has a lower gas density than nitrogen. Thus, there is less work of breathing when helium is substituted for nitrogen. Remember, though: the higher the concentration of helium, the lower the concentration of oxygen (Miller: Miller’s Anesthesia, ed 8, p 2545).
53. The maximum Fio2 that can be delivered by a nasal cannula is A. 0.30 B. 0.35 C. 0.40 D. 0.45
53. (D) The Fio2 delivered to patients from low-flow systems (e.g., nasal prongs) is determined by the size of the O2 reservoir, the O2 flow, and the patient’s breathing pattern. As a rule of thumb, assuming a normal breathing pattern, the Fio2 delivered by nasal prongs increases by approximately 0.04 for each L/min increase in O2 flow up to a maximal Fio2 of approximately 0.45 (at an O2 flow of 6 L/min). In general, the larger the patient’s VT or the faster the respiratory rate, the lower the Fio2 for a given O2 flow (Butterworth: Morgan & Mikhail’s Clinical Anesthesiology, ed 5, pp 1282–1283).
54. General anesthesia is administered to an otherwise healthy 38-year-old patient undergoing repair of a right inguinal hernia. During mechanical ventilation, the anesthesiologist notices that the scavenging system reservoir bag is distended during inspiration. The most likely cause of this is A. An incompetent pressure-relief valve in the mechanical ventilator B. An incompetent pressure-relief valve in the patient’s breathing circuit C. An incompetent inspiratory unidirectional valve in the patient’s breathing circuit D. An incompetent expiratory unidirectional valve in the patient’s breathing circuit 6 Part 1 Basic Sciences
54. (A) APL valve Ventilator relief valve Frequency Flow Volume Intake valve 22 Part 1 Basic Sciences In a closed scavenging system interface, the reservoir bag should expand during expiration and contract during inspiration. During the inspiratory phase of mechanical ventilation, the ventilator pressure-relief valve closes, thereby directing the gas inside the ventilator bellows into the patient’s breathing circuit. If the ventilator pressure-relief valve is incompetent, there will be a direct communication between the patient’s breathing circuit and the scavenging circuit. This will result in delivery of part of the mechanical ventilator VT directly to the scavenging circuit, causing the reservoir bag to inflate during the inspiratory phase of the ventilator cycle (Ehrenwerth: Anesthesia Equipment: Principles and Applications, ed 2, pp 130–132).
55. Which color of nail polish would have the greatest effect on the accuracy of dual-wavelength pulse oximeters? A. Red B. Yellow C. Blue D. Green
55. (C) The accurate function of dual-wavelength pulse oximeters is altered by nail polish. Because blue nail polish has a peak absorbance similar to that of adult deoxygenated hemoglobin (near 660 nm), it has the greatest effect on the Spo2 reading. Nail polish causes an artifactual and fixed decrease in the Spo2 reading as shown by these devices. Turning the finger probe 90 degrees and having the light shining sidewise through the finger is useful when there is nail polish on the patient’s fingernails (Miller: Miller’s Anesthesia ed 8, p 1547).
56. The minimum macroshock current required to elicit ventricular fibrillation is A. 1 mA B. 10 mA C. 100 mA D. 500 mA
56. (C) Leakage electric currents less than 1 mA are imperceptible to touch. The minimal ventricular fibrillation threshold of current applied to the skin is about 100 mA. If the current bypasses the high resistance of the skin and is applied directly to the heart via pacemaker, central line, etc. (microshock), currents as low as 100 μA (0.1 mA) may be fatal. Because of this, the American National Standards Institute has set the maximum leakage of electric current allowed through electrodes or catheters in contact with the heart at 10 μA (Barash: Clinical Anesthesia, ed 7, p 192; Butterworth: Morgan & Mikhail’s Clinical Anesthesiology, ed 5, p 17; Miller: Miller’s Anesthesia, ed 8, p 3226).
57. The line isolation monitor A. Prevents microshock B. Prevents macroshock C. Provides electric isolation in the OR D. Sounds an alarm when grounding occurs in the OR
57. (D) The line isolation monitor gives an alarm when grounding occurs in the OR or when the maximum current that a short circuit could cause exceeds 2 to 5 mA. The line isolation monitor is purely a monitor and does not interrupt electric current. Therefore, the line isolation monitor will not prevent microshock or macroshock (Brunner: Electricity, Safety, and the Patient, ed 1, p 304; Miller: Miller’s Anesthesia, ed 8, pp 3221–3223).
58. Kinking or occlusion of the transfer tubing from the patient’s breathing circuit to the closed scavenging system interface can result in A. Barotrauma B. Hypoventilation C. Hypoxia D. Hyperventilation
58. (A) A scavenging system with a closed interface is one in which there is communication with the atmosphere through positive-pressure and negative-pressure relief valves. The positive-pressure relief valve will prevent transmission of excessive pressure buildup to the patient’s breathing circuit, even if there is an obstruction distal to the interface or if the system is not connected to wall suction. However, obstruction of the transfer tubing from the patient’s breathing circuit to the scavenging circuit is proximal to the interface. This will isolate the patient’s breathing circuit from the positive-pressure relief valve of the scavenging system interface. Should this occur, barotrauma to the patient’s lungs can result (Ehrenwerth: Anesthesia Equipment: Principles and Applications, ed 2, pp 130–137).
59. The reason a patient is not burned by the return of energy from the patient to the ESU (electrosurgical unit, Bovie) is that A. The coagulation side of this circuit is positive relative to the ground side B. Resistance in the patient’s body attenuates the energy C. The exit current density is much less D. The overall energy delivered is too small to cause burns
59. (C) Electrocautery units, or electrosurgical units (ESUs), were invented by Professor W. T. Bovie and were first used in 1926. They operate by generating ultra-high frequency (0.1-3 MHz) alternating electric Anesthesia Equipment and Physics 23 currents and are commonly used today for cutting and coagulating tissue. Whenever a current passes through a resistance such as tissue, heat is generated and is inversely proportional to the surface area through which the current passes. At the point of entry to the body from the small active electrode or cautery tip, a fair amount of heat is generated. For the current to complete its circuit, the return electrode plate or dispersive pad (incorrectly but commonly called the ground pad) has a large surface area, where very little heat develops. The dispersive pad should be as close as is reasonable to the site of surgery. If the current from the ESU passes through an artificial cardiac pacemaker, the pacemaker may misinterpret the current as cardiac activity and may not pace, which is why a magnet placed over the pacemaker will turn off the pacemaker sensor, putting the pacemaker in the asynchronous mode, and should be available (if the pacemaker’s sensory mode is not turned off preoperatively). In addition, automatic implantable cardioverter-defibrillators (AICDs) may misinterpret the electric activity as ventricular fibrillation and defibrillate the patient. AICDs should be turned off before use of an ESU (Barash: Clinical Anesthesia ed 7, pp 204–206; Butterworth: Morgan & Mikhail’s Clinical Anesthesiology, ed 5, pp 19–22).
60. Select the FALSE statement regarding noninvasive arterial BP monitoring devices. A. If the width of the BP cuff is too narrow, the measured BP will be falsely lowered B. The width of the BP cuff should be 40% of the circumference of the patient’s arm C. If the BP cuff is wrapped around the arm too loosely, the measured BP will be falsely elevated D. Frequent cycling of automated BP monitoring devices can result in edema distal to the cuff
60. (A) Automated noninvasive BP (ANIBP) devices provide consistent and reliable arterial BP measurements. Variations in the cuff pressure resulting from arterial pulsations during cuff deflation are sensed by the device and are used to calculate mean arterial pressure. Then, values for systolic and diastolic pressures are derived from formulas that use the rate of change of the arterial pressure pulsations and the mean arterial pressure (oscillometric principle). This method provides accurate measurements of arterial BP in neonates, infants, children, and adults. The main advantage of ANIBP devices is that they free the anesthesia provider to perform other duties required for optimal anesthesia care. Additionally, these devices provide alarm systems to draw attention to extreme BP values, and they have the capacity to transfer data to automated trending devices or recorders. Improper use of these devices can lead to erroneous measurements and complications. The width of the BP cuff should be approximately 40% of the circumference of the patient’s arm. If the BP cuff is too narrow or if the BP cuff is wrapped too loosely around the arm, the BP measurement by the device will be falsely elevated. Frequent BP measurements can result in edema of the extremity distal to the cuff. For this reason, cycling of these devices should not be more frequent than every 1 to 3 minutes. Other complications associated with improper use of ANIBP devices include ulnar nerve paresthesia, superficial thrombophlebitis, and compartment syndrome. Fortunately, these complications are rare (Butterworth: Morgan & Mikhail’s Clinical Anesthesiology, ed 5, pp 88–91; Miller: Basics of Anesthesia, ed 6, pp 321–322; Miller: Miller’s Anesthesia, ed 8, pp 1347–1348).
61. When electrocardiogram (EKG) electrodes are placed for a patient undergoing a magnetic resonance imaging (MRI) scan, which of the following is true? A. Electrodes should be as close as possible and in the periphery of the magnetic field B. Electrodes should be as close as possible and in the center of the magnetic field C. Placement of electrodes relative to field is not important as long as they are far apart D. EKG cannot be monitored during an MRI scan
61. (B) EKG monitoring is often not used during MRI scans because artifacts are very common (abnormalities in T waves and ST waves), and heating of the wires during the scan would potentially burn the patient. However, EKG can be used if the electrodes are placed close together and toward the center of the magnetic field and the wires are insulated from the patient’s skin and straight. In addition, the wires should not be wound together in loops (because this can induce heating of the wires), and worn or frayed wires should not be used (Barash: Clinical Anesthesia, ed 7, p 884; Miller: Miller’s Anesthesia, ed 8, p 2655).
62. The pressure gauge of a size “E” compressed-gas cylinder containing air shows a pressure of 1000 psi. Approximately how long could air be delivered from this cylinder at the rate of 10 L/min? A. 10 minutes B. 20 minutes C. 30 minutes D. 40 minutes
62. (C) A size “E” compressed-gas cylinder completely filled with air contains 625 L and will show a pressure gauge reading of 2000 psi. Therefore, a cylinder with a pressure gauge reading of 1000 psi is half-full, containing approximately 325 L of air. A half-full size “E” compressed-gas cylinder containing air can be used for approximately 30 minutes at a flow rate of 10 L/min (see definition of Boyle’s law, Question 9) (Butterworth: Morgan & Mikhail’s Clinical Anesthesiology, ed 5, pp 10–12; Miller: Basics of Anesthesia, ed 6, pp 199–201).
63. The most frequent cause of mechanical failure of the anesthesia delivery system to deliver adequate O2 to the patient is A. Attachment of the wrong compressed-gas cylinder to the O2 yoke B. Improperly assembled O2 rotameter C. Fresh-gas line disconnection from the anesthesia machine to the in-line hosing D. Disconnection of the O2 supply system from the patient
63. (D) Failure to oxygenate patients adequately is an important cause of anesthesia-related morbidity and mortality. All of the choices listed in this question are potential causes of inadequate delivery of O2 to the patient; however, the most frequent cause is inadvertent disconnection of the O2 supply system from the patient (e.g., disconnection of the patient’s breathing circuit from the endotracheal tube) (Ehrenwerth: Anesthesia Equipment: Principles and Applications, ed 2, p 121; Butterworth: Morgan & Mikhail’s Clinical Anesthesiology, ed 5, pp 43–47).
64. The esophageal detector device A. Uses a negative-pressure bulb B. Is especially useful in children younger than 1 year of age C. Requires a cardiac output to function appropriately D. Is reliable in morbidly obese patients and parturients
64. (A) The esophageal detector device (EDD) is essentially a bulb that is first compressed and then attached to the endotracheal tube after the tube is inserted into the patient. The pressure generated 24 Part 1 Basic Sciences is about –40 cm of water. If the endotracheal tube is placed in the esophagus, then the negative pressure will collapse the esophagus, and the bulb will not inflate. If the endotracheal tube is in the trachea, then the air from the lung will enable the bulb to inflate (usually in a few seconds, but sometimes more than 30 seconds). A syringe that has a negative pressure applied to it has also been used. Although initial studies were very positive about the use of the EDD, more recent studies show that up to 30% of correctly placed endotracheal tubes in adults may be removed because the EDD has suggested esophageal placement. Misleading results have been noted in patients with morbid obesity, late pregnancy, status asthmaticus, and copious endotracheal secretion, wherein the trachea tends to collapse. Its use in children younger than 1 year of age has shown poor sensitivity and poor specificity. Although a cardiac output is needed to get CO2 to the lungs for a CO2 gas analyzer to function, a cardiac output is not needed for an EDD (Miller: Miller’s Anesthesia, ed 8, p 1654).
65. The reason CO2 measured by capnometer is less than the arterial Paco2 value measured simultaneously is A. Use of ion-specific electrode for blood gas determination B. Alveolar capillary gradient C. One-way values D. Alveolar dead space
65. (D) The capnometer measures the CO2 concentration of respiratory gases. Today this is most commonly performed by infrared absorption using a sidestream gas sample. The sampling tube should be connected as close as possible to the patient’s airway. The difference between the end-tidal CO2 (Etco2) and the arterial CO2 (Paco2) is typically 5 to 10 mm Hg and is due to alveolar dead space ventilation. Because nonperfused alveoli do not contribute to gas exchange, any condition that increases alveolar dead space ventilation (i.e., reduces pulmonary blood flow, as by pulmonary embolism or cardiac arrest) will increase dead space ventilation and the Etco2-to-Paco2 difference. Conditions that increase pulmonary shunt result in minimal changes in the Paco2–Etco2 gradient. CO2 diffuses rapidly across the capillary-alveolar membrane (Barash: Clinical Anesthesia, ed 7, pp 704–706; Miller: Miller’s Anesthesia, ed 8, pp 1551–1553).
66. Which of the following arrangements of rotameters on the anesthesia machine manifold is safest with leftto- right gas flow? A. O2, CO2, N2O, air B. CO2, O2, N2O, air C. Air, CO2, O2, N2O D. Air, CO2, N2O, O2 Anesthesia Equipment and Physics 7
66. (D) The last gas added to a gas mixture should always be O2. This arrangement is the safest because it ensures that leaks proximal to the O2 inflow cannot result in the delivery of a hypoxic gas mixture to the patient. With this arrangement (O2 added last), leaks distal to the O2 inflow will result in a decreased volume of gas, but the Fio2 of anesthesia will not be reduced (Miller: Basics of Anesthesia, ed 6, pp 201–202; Ehrenwerth: Anesthesia Equipment: Principles and Applications, ed 2, pp 43–45).
67. A Datex-Ohmeda Tec 4 vaporizer is tipped over while being attached to the anesthesia machine but is placed upright and installed. The soonest it can be safely used is A. After 30 minutes of flushing with dial set to “off” B. After 6 hours of flushing with dial set to “off” C. After 30 minutes with dial turned on D. Immediately
67. (C) Most modern Datex-Ohmeda Tec or North American Dräger Vapor vaporizers (except desflurane) are variable-bypass, flow-over vaporizers. This means that the gas that flows through the vaporizers is split into two parts, depending on the concentration selected. The gas goes through either the bypass chamber on the top of the vaporizer or the vaporizing chamber on the bottom of the vaporizer. If the vaporizer is tipped, which might happen when a filled vaporizer is switched out or moved from one machine to another machine, part of the anesthetic liquid in the vaporizing chamber may get into the bypass chamber. This could result in a much higher concentration of gas than that dialed. With the Datex-Ohmeda Tec 4 or the North American Dräger Vapor 19.1 series, it is recommended to flush the vaporizer at high flows with the vaporizer set at a low concentration until the output shows no excessive agent (this usually takes 20-30 minutes). The Dräger Vapor 2000 series has a transport (T) dial setting. This setting isolates the bypass from the vaporizer chamber. The Aladin cassette vaporizer does not have a bypass flow chamber and has no tipping hazard (Miller: Miller’s Anesthesia, ed 8, p 771).
68. In the event of misfilling, what percent sevoflurane would be delivered from an isoflurane vaporizer set at 1%? A. 0.6% B. 0.8% C. 1.0% D. 1.2%
68. (A) Accurate delivery of volatile anesthetic concentration is dependent on filling the agent-specific vaporizer with the appropriate (volatile) agent. Differences in anesthetic potencies further necessitate this requirement. Each agent-specific vaporizer uses a splitting ratio that determines the portion of the fresh gas that is directed through the vaporizing chamber versus that which travels through the bypass chamber. VAPOR PRESSURE, ANESTHETIC VAPOR PRESSURE, AND SPLITTING RATIO Halothane Sevoflurane Isoflurane Enflurane Vapor pressure at 20° C 243 mm Hg 160 mm Hg 240 mm Hg 172 mm Hg VP/(BP−VP) 0.47 0.27 0.47 0.29 Splitting ratio for 1% vapor 1:47 1:27 1:47 1:29 BP, blood pressure; VP, vapor pressure. Anesthesia Equipment and Physics 25 The table shows the calculation (fraction) that when multiplied by the quantity of fresh gas traversing the vaporizing chamber (affluent fresh gas in mL/min) will yield the output (mL/min) of anesthetic vapor in the effluent gas. When this fraction is multiplied by 100, it equals the splitting ratio for 1% for the given volatile agent. For example, when the isoflurane vaporizer is set to deliver 1% isoflurane, one part of fresh gas is passed through the vaporizing chamber while 47 parts travel through the bypass chamber. One can determine on inspection that when a less soluble volatile agent like sevoflurane (or the obsolete volatile agent enflurane, for the sake of example) is placed into an isoflurane (or halothane) vaporizer, the output in volume percent will be less than expected; how much less can be determined by simply comparing their splitting ratios 27/47 or 0.6. Halothane and enflurane are no longer used in the United States, but old halothane and enflurane vaporizers can be (and are) used elsewhere in the world to accurately deliver isoflurane and sevoflurane, respectively (Ehrenwerth: Anesthesia Equipment: Principles and Applications, ed 2, pp 72–73).
69. How long would a vaporizer (filled with 150 mL volatile) deliver 2% isoflurane if total flow is set at 4.0 L/min? A. 2 hours B. 4 hours C. 6 hours D. 8 hours
69. (C) Two percent of 4 L/min will be 80 mL of isoflurane per minute. VAPOR PRESSURE PER MILLILITER OF LIQUID Halothane Enflurane Isoflurane Sevoflurane Desflurane mL vapor per mL liquid at 20° C 226 196 195 182 207 Given that 1 mL of isoflurane liquid yields 195 mL of anesthetic vapor and by applying the calculation (195 mL vapor/1 mL liquid isoflurane) × (150 mL isoflurane liquid) = 29,250 mL isoflurane vapor, it follows that (29,250 mL ÷ 80 mL/min = 365 minutes). Three hundred sixty-five minutes is around 6 hours (Ehrenwerth: Anesthesia Equipment: Principles and Applications, ed 2, pp 65–70).
70. Raising the frequency of an ultrasound transducer used for line placement or regional anesthesia (e.g., from 3 MHz to 10 MHz) will result in A. Higher penetration of tissue with lower resolution B. Higher penetration of tissue with higher resolution C. Lower penetration of tissue with higher resolution D. Higher resolution with no change in tissue penetration
70. (C) The human ear can perceive sound in the range of 20 Hz to 20 kHz. Frequencies above 20 kHz, inaudible to humans, are ultrasonic frequencies (ultra = Latin for “beyond” or “on the far side of”). In regional anesthesia, ultrasound is used for imaging in the frequency range of 2.5 to 10 MHz. Wavelength is inversely proportional to frequency (i.e., λ = C/f [λ = wavelength, C = velocity of sound through tissue or 1540 m/sec, f = frequency]). Wavelength in millimeters can be calculated by dividing 1.54 by the Doppler frequency in megahertz. Penetration into tissue is 200 to 400 times wavelength, and resolution is twice the wavelength. Therefore, a frequency of 3 MHz (wavelength 0.51 mm) would have a resolution of 1 mm and a penetration of up to 100 to 200 mm (10-20 cm), whereas 10 MHz (wavelength 0.15 mm) corresponds to a resolution of 0.3 mm but a penetration depth of no more than 60 to 120 mm (6-12 cm) (Miller: Miller’s Anesthesia, ed 8, pp 1398–1405; Butterworth: Morgan & Mikhail’s Clinical Anesthesiology, ed 5, p 979).
71. The fundamental difference between microshock and macroshock is related to A. Location of shock B. Duration C. Voltage D. Lethality
71. (A) Microshock refers to electric shock located in or near the heart. A current as low as 100 μA passing through the heart can produce ventricular fibrillation. Pacemaker electrodes, central venous catheters, pulmonary artery catheters, and other devices in the heart are necessary prerequisites for microshock. Because the line isolation monitor has a threshold of 2 mA (2000 μA) for alarming, it will not protect against microshock (Miller: Miller’s Anesthesia, ed 8, p 3226).
72. Intraoperative awareness under general anesthesia can be eliminated by closely monitoring A. Electroencephalogram B. BP/heart rate C. Bispectral index (BIS) D. None of the above
72. (D) Intraoperative awareness or recall during general anesthesia is rare (overall incidence is 0.2%, for obstetrics 0.4%, for cardiac 1%-1.5%) except for major trauma, which has a reported incidence as high as 43%. With the electroencephalogram, trends can be identified with changes in the depth of anesthesia; however, the sensitivity and specificity of the available trends are such that none serve as a sole indicator of anesthesia depth. Although using the bispectral index monitor may reduce the risk of recall, it, like the other listed signs as well as patient movement, does not totally eliminate recall (Miller: Miller’s Anesthesia, ed 8, pp 1527–1528).
73. A mechanically ventilated patient is transported from the OR to the ICU using a portable ventilator that consumes 2 L/min of oxygen to run the mechanically controlled valves and drive the ventilator. The transport cart is equipped with an “E” cylinder with a gauge pressure of 2000 psi. The patient receives a VT of 500 mL at a rate of 10 breaths/min. If the ventilator requires 200 psi to operate, how long could the patient be mechanically ventilated? A. 20 minutes B. 40 minutes C. 60 minutes D. 80 minutes
73. (D) The minute ventilation is 5 L (0.5 L per breath at 10 breaths/min) and 2 L/min to drive the ventilator for a total O2 consumption of 7 L/min. A full oxygen “E” cylinder contains 625 L. Ninety percent of the volume of the cylinder (≈560 L) can be delivered before the ventilator can no longer be driven. At a rate of 7 L/min, this supply would last about 80 minutes (Miller: Basics of Anesthesia, ed 6, pp 201, 209; Ehrenwerth: Anesthesia Equipment: Principles and Applications, ed 2, pp 29–33, 37; Butterworth: Morgan & Mikhail’s Clinical Anesthesiology, ed 5, pp 10–11). 26 Part 1 Basic Sciences
74. A 135-kg man is ventilated at a rate of 14 breaths/min with a VT of 600 mL and positive end-expiratory pressure (PEEP) of 5 cm H2O during a laparoscopic banding procedure. Peak airway pressure is 50 cm H2O, and the patient is fully relaxed with a nondepolarizing neuromuscular blocking agent. How can peak airway pressure be reduced without a loss of alveolar ventilation? A. Increase the inspiratory flow rate B. Take off PEEP C. Reduce the I:E ratio (e.g., change from 1:3 to 1:2) D. Decrease VT to 300 and increase rate to 28
74. (C) After eliminating reversible causes of high peak airway pressures (e.g., occlusion of the endotracheal tube, mainstem intubation, or bronchospasm), adjusting the ventilator can reduce the peak airway pressure. Increasing the inspiratory flow rate would cause the airway pressures to go up faster and would produce higher peak airway pressures. Removing PEEP would lower peak pressure at the expense of alveolar ventilation. Changing the I:E ratio from 1:3 to 1:2 will permit 8% (25% inspiratory time to 33% inspiratory time) more time for the VT to be administered and will result in lower airway pressures. Decreasing the VT to 300 and increasing the rate to 28 would give the same minute ventilation but not the same alveolar ventilation. Recall that alveolar ventilation equals (frequency) times (VT minus dead space), and because dead space is the same (about 2 mL/kg ideal weight), alveolar ventilation would be reduced, in this case to a dangerously low level. Another option is to change from volume-cycled to pressure-cycled ventilation, which produces a more constant pressure over time instead of the peaked pressures seen with fixed VT ventilation (Barash: Clinical Anesthesia, ed 7, pp 1593–1596; Miller: Miller’s Anesthesia, ed 8, pp 3064–3074).
75. The pressure and volume per minute delivered from the central hospital oxygen supply are A. 2100 psi and 650 L/min B. 1600 psi and 100 L/min C. 75 psi and 100 L/min D. 50 psi and 50 L/min
75. (D) The central hospital oxygen supply to the ORs is designed to give enough pressure and oxygen flow to run the three oxygen components of the anesthesia machine (patient fresh gas flow, anesthesia ventilator, and oxygen flush valve). The oxygen flowmeter on the anesthesia machine is designed to run at an oxygen pressure of 50 psi, and for emergency purposes the oxygen flush valve delivers oxygen at 35 to 75 L/min (Miller: Basics of Anesthesia, ed 6, pp 199–201).
76. During normal laminar airflow, resistance is dependent on which characteristic of oxygen? A. Density B. Viscosity C. Molecular weight D. Temperature
76. (B) Within the respiratory system, both laminar and turbulent flows exist. At low flow rates, the respiratory flow tends to be laminar, like a series of concentric tubes that slide over one another with the center tubes flowing faster than the more peripheral tubes. Laminar flow is usually inaudible and is dependent on gas viscosity. Turbulent flow tends to be faster, is audible, and is dependent on gas density. Gas density can be decreased by using a mixture of helium with oxygen (Butterworth: Morgan & Mikhail’s Clinical Anesthesiology, ed 5, pp 54–56).
77. If the oxygen cylinder were being used as the source of oxygen at a remote anesthetizing location and the oxygen flush valve on an anesthesia machine were pressed and held down, as during an emergency situation, each of the items below would be bypassed during 100% oxygen delivery EXCEPT A. O2 flowmeter B. First-stage regulator C. Vaporizer check valve D. Vaporizers 8 Part 1 Basic Sciences
77. (B) Anesthesia workstations have high-pressure, intermediate-pressure, and low-pressure circuits (see figure in the explanation for Question 12). The high-pressure circuit is from the oxygen cylinder to the oxygen pressure regulator (first-stage regulator), which takes the oxygen pressure from a high of 2200 psi to 45 psi. The intermediate-pressure circuit consists of the pipeline pressure of about 50 to 55 psi and goes to the second-stage regulator, which then lowers the pressure to 14 to 26 psi (depending on the machine). The low-pressure circuit then consists of the flow tubes, vaporizer manifold, vaporizers, and vaporizer check valve to the common gas outlet. The oxygen flush valve is in the intermediate-pressure circuit and bypasses the low-pressure circuit (Ehrenwerth: Anesthesia Equipment: Principles and Applications, ed 2, pp 34–36; Miller: Basics of Anesthesia, ed 6, p 200).
78. After induction and intubation with confirmation of tracheal placement, the O2 saturation begins to fall. The O2 analyzer shows 4% inspired oxygen. The oxygen line pressure is 65 psi. The O2 tank on the back of the anesthesia machine has a pressure of 2100 psi and is turned on. The oxygen saturation continues to fall. The next step should be to A. Exchange the tank B. Replace pulse oximeter probe C. Disconnect O2 line from hospital source D. Extubate and start mask ventilation
78. (C) Two major problems should be noted in this case. The first obvious problem is the inspired oxygen concentration of 4%, a concentration that is not possible if the gases going to the machine are appropriate unless the oxygen analyzer is faulty. Given the dire consequences of a hypoxic gas mixture, one must assume the oxygen analyzer is correct and work on the premise that the O2 pipeline is supplying a gas other than oxygen. Second, the oxygen line pressure is 65 psi. The pipeline pressures are normally around 50 to 55 psi, whereas the pressure from the oxygen cylinder, if the cylinder is turned on, is reduced to 45 psi. For the oxygen tank to deliver oxygen to the patient, the pipeline pressure needs to be less than 45 psi, which in this case will occur only when the pipeline is disconnected. Although we rarely think of problems with hospital gas lines, a survey of more than 200 hospitals showed about 33% had problems with the pipelines. The most common pipeline problems were low pressure, followed by high pressure and, very rarely, crossed gas lines (Ehrenwerth: Anesthesia Equipment: Principles and Applications, ed 2, p 34; Miller: Miller’s Anesthesia, ed 8, p 756).
79. The correct location for placement of the V5 lead is A. Midclavicular line, third intercostal space B. Anterior axillary line, fourth intercostal space C. Midclavicular line, fifth intercostal space D. Anterior axillary line, fifth intercostal space
79. (D) There are many ways to monitor the electric activity of the heart. The five-electrode system using one lead for each limb and the fifth lead for the precordium is commonly used in the OR. The precordial lead placed in the V5 position (anterior axillary line in the fifth intercostal space) gives the V5 tracing, which, combined with the standard lead II, are the most common tracings used to look for myocardial ischemia (Miller: Miller’s Anesthesia, ed 8, pp 1429–1434).
80. The diameter index safety system refers to the interface between A. Pipeline source and anesthesia machine B. Gas cylinders and anesthesia machine C. Vaporizers and refilling connectors attached to bottles of volatile anesthetics D. Both pipeline and gas cylinders interface with anesthesia machine
80. (A) See also Question 36. The diameter index safety system provides threaded, noninterchangeable connections for medical gas pipelines through the hospital as well as to the anesthesia machine. The pin index safety system has two metal pins in different arrangements around the yoke on the back of anesthesia machines, Anesthesia Equipment and Physics 27 with each arrangement for a specific gas cylinder. Vaporizers often have keyed fillers that attach to the bottle of anesthetic and the vaporizer. Vaporizers not equipped with keyed fillers occasionally have been misfilled with the wrong anesthetic liquid (Butterworth: Morgan & Mikhail’s Clinical Anesthesiology, ed 5, pp 49–50).
81. Each of the following is cited as an advantage of calcium hydroxide lime (Amsorb Plus, Drägersorb) over soda lime EXCEPT A. Compound A is not formed B. CO is not formed C. More absorptive capacity per 100 g of granules D. It does not contain NaOH or KOH
81. (C) Calcium hydroxide lime does not contain the monovalent hydroxide bases that are present in soda lime (namely, NaOH and KOH). Sevoflurane in the presence of NaOH or KOH is degraded to trace amounts of compound A, which is nephrotoxic to rats at high concentrations. Soda lime normally contains about 13% to 15% water, but if the soda lime is desiccated (water content <5%—which has occurred if the machine is not used for a while and the fresh gas flow is left on) and is exposed to current volatile anesthetics (isoflurane, sevoflurane, and especially desflurane), CO can be produced. Neither compound A nor CO is formed when calcium hydroxide lime is used. With soda lime and calcium hydroxide lime, the indicator dye changes from white to purple as the granules become exhausted. The two major disadvantages of calcium hydroxide lime are the expense and the fact that its absorptive capacity is about half that of soda lime (10.2 L of CO2/100 g of calcium hydroxide lime versus 26 L of CO2/100 g of soda lime) (Miller: Miller’s Anesthesia, ed 8, pp 787–789; Butterworth: Morgan & Mikhail’s Clinical Anesthesiology, ed 5, pp 36–38; Miller: Basics of Anesthesia, ed 6, pp 212–214).
82. 160 80 0 1 second ART 166/56 (82) NIBP 126/63 (84) The arrows in the figure above indicate A. Respiratory variation B. An underdamped signal C. An overdamped signal D. Atrial fibrillation
82. (B) The aim of direct invasive monitoring is to give continuous arterial BPs that are similar to the intermittent noninvasive arterial BPs from a cuff, as well as to give a port for arterial blood samples. The displayed signal reflects the actual pressure and the distortions from the measuring system (i.e., the catheter, tubing, stopcocks, and amplifier). Although the signal is usually accurate, at times we see an underdamped or an overdamped signal. In an underdamped signal, as in this case, exaggerated readings are noted (widened pulse pressure). In an overdamped signal, readings are diminished (narrowed pulse pressure). However, the mean BP tends to be accurate in both underdamped and overdamped signals (Miller: Miller’s Anesthesia, ed 8, pp 1347–1359).
83. During a laparoscopic cholecystectomy, exhaled CO2 is 6%, but inhaled CO2 is 1%. Which explanation could NOT account for rebreathing CO2? A. Channeling through soda lime B. Faulty expiratory valve C. Exhausted soda lime D. Absorption of CO2 through peritoneum
83. (D) Rebreathing of expired gases (e.g., stuck open expiratory or inspiratory valves), faulty removal of CO2 from the CO2 absorber (e.g., exhausted CO2 absorber, channeling through a CO2 absorber, or having the CO2 absorber bypassed—an option in some older anesthetic machines), or addition of CO2 from a gas supply (rarely done with current anesthetic machines) can all increase inspired CO2. The absorption of CO2 during laparoscopic surgery when CO2 is used as the abdominal distending gas will increase absorption of CO2 but will not cause an increase in inspired CO2 (Miller: Miller’s Anesthesia, ed 8, pp 1551–1559; Butterworth: Morgan & Mikhail’s Clinical Anesthesiology, ed 5, p 42).
84. Helium A. Black B. Brown C. Blue D. Gray
84. (B) Medical gas cylinders are color coded, but the colors may differ from one country to another. In the United States, if there is a combination of two gases, the tank would have both corresponding colors; for example, a tank containing oxygen and helium would be green and brown. The only exception to the mixed gas color scheme is O2 and N2 in the proportion of 19.5% to 23.5% O2 mixed with N2, which is solid yellow (air) (Ehrenwerth: Anesthesia Equipment: Principles and Applications, ed 2, p 7). GAS COLOR CODES Gas United States International Air Yellow White and black CO2 Gray Gray Helium Brown Brown Nitrogen Black Black N2O Blue Blue Oxygen Green White Data from Ehrenwerth J, Eisenkraft JB, Berry JM: Anesthesia Equipment: Principles and Applications, ed 2, Philadelphia, Saunders, 2013. 28 Part 1 Basic Sciences
85. Nitrogen A. Black B. Brown C. Blue D. Gray
85. (A) Medical gas cylinders are color coded, but the colors may differ from one country to another. In the United States, if there is a combination of two gases, the tank would have both corresponding colors; for example, a tank containing oxygen and helium would be green and brown. The only exception to the mixed gas color scheme is O2 and N2 in the proportion of 19.5% to 23.5% O2 mixed with N2, which is solid yellow (air) (Ehrenwerth: Anesthesia Equipment: Principles and Applications, ed 2, p 7). GAS COLOR CODES Gas United States International Air Yellow White and black CO2 Gray Gray Helium Brown Brown Nitrogen Black Black N2O Blue Blue Oxygen Green White Data from Ehrenwerth J, Eisenkraft JB, Berry JM: Anesthesia Equipment: Principles and Applications, ed 2, Philadelphia, Saunders, 2013. 28 Part 1 Basic Sciences
86. CO2 A. Black B. Brown C. Blue D. Gray
86. (D) Medical gas cylinders are color coded, but the colors may differ from one country to another. In the United States, if there is a combination of two gases, the tank would have both corresponding colors; for example, a tank containing oxygen and helium would be green and brown. The only exception to the mixed gas color scheme is O2 and N2 in the proportion of 19.5% to 23.5% O2 mixed with N2, which is solid yellow (air) (Ehrenwerth: Anesthesia Equipment: Principles and Applications, ed 2, p 7). GAS COLOR CODES Gas United States International Air Yellow White and black CO2 Gray Gray Helium Brown Brown Nitrogen Black Black N2O Blue Blue Oxygen Green White Data from Ehrenwerth J, Eisenkraft JB, Berry JM: Anesthesia Equipment: Principles and Applications, ed 2, Philadelphia, Saunders, 2013. 28 Part 1 Basic Sciences
87. Best for spontaneous ventilation. Match the figures below with the correct numbered statement. Each lettered figure may be selected once, more than once, or not at all. A B C D E F FGF FGF FGF FGF FGF FGF
87. (A) There are six different types of Mapleson breathing circuits (designated A through F). These circuits vary in arrangement of the fresh-gas-flow inlet, tubing, mask, reservoir bag, and unidirectional expiratory valve. These systems are lightweight, portable, and easy to clean; they offer low resistance to breathing, and, because of high fresh gas inflows, they prevent rebreathing of exhaled gases. In addition, with these breathing circuits, the concentration of volatile anesthetic gases and O2 delivered to the patient can be accurately estimated. The reservoir bag enables the anesthesia provider to provide assisted or controlled ventilation of the lungs. The unidirectional expiratory valve functions to direct fresh gas into the patient and exhaled gases out of the circuit. In the Mapleson A breathing circuit, the unidirectional expiratory valve is near the patient, and the fresh-gas-flow inlet is proximal to the reservoir bag. This arrangement is the most efficient for elimination of CO2 during spontaneous breathing. However, because the unidirectional expiratory valve must be tightened to permit production of positive airway pressure when the gas reservoir bag is manually compressed, this breathing circuit is less efficient in preventing rebreathing of CO2 during assisted or controlled ventilation of the lungs. The structure of the Mapleson D breathing circuit is similar to that of the Mapleson A breathing circuit except that the positions of the fresh-gasflow inlet and the unidirectional expiratory valve are reversed. The placement of the fresh-gas-flow inlet near the patient produces efficient elimination of CO2, regardless of whether the patient is breathing spontaneously or with controlled ventilation. The Bain anesthesia breathing circuit is a coaxial version of the Mapleson D breathing circuit except that the fresh gas enters through a narrow tube within the corrugated expiratory limb of the circuit. The Jackson-Rees breathing circuit is a modification of the Mapleson E breathing circuit and is called a Mapleson F breathing circuit. In the Jackson-Rees breathing circuit, the adjustable unidirectional expiratory valve is incorporated into the reservoir bag, and the fresh-gas-flow inlet is close to the patient. This arrangement offers the advantage of ease of instituting assisted or controlled ventilation of the lungs, as well as monitoring ventilation by movement of the reservoir bag during spontaneous breathing (Ehrenwerth: Anesthesia Equipment: Principles and Applications, ed 2, pp 109–117; Miller: Miller’s Anesthesia, ed 8, pp 780–781). 29
88. Best for controlled ventilation. Match the figures below with the correct numbered statement. Each lettered figure may be selected once, more than once, or not at all. A B C D E F FGF FGF FGF FGF FGF FGF
88. (D) There are six different types of Mapleson breathing circuits (designated A through F). These circuits vary in arrangement of the fresh-gas-flow inlet, tubing, mask, reservoir bag, and unidirectional expiratory valve. These systems are lightweight, portable, and easy to clean; they offer low resistance to breathing, and, because of high fresh gas inflows, they prevent rebreathing of exhaled gases. In addition, with these breathing circuits, the concentration of volatile anesthetic gases and O2 delivered to the patient can be accurately estimated. The reservoir bag enables the anesthesia provider to provide assisted or controlled ventilation of the lungs. The unidirectional expiratory valve functions to direct fresh gas into the patient and exhaled gases out of the circuit. In the Mapleson A breathing circuit, the unidirectional expiratory valve is near the patient, and the fresh-gas-flow inlet is proximal to the reservoir bag. This arrangement is the most efficient for elimination of CO2 during spontaneous breathing. However, because the unidirectional expiratory valve must be tightened to permit production of positive airway pressure when the gas reservoir bag is manually compressed, this breathing circuit is less efficient in preventing rebreathing of CO2 during assisted or controlled ventilation of the lungs. The structure of the Mapleson D breathing circuit is similar to that of the Mapleson A breathing circuit except that the positions of the fresh-gasflow inlet and the unidirectional expiratory valve are reversed. The placement of the fresh-gas-flow inlet near the patient produces efficient elimination of CO2, regardless of whether the patient is breathing spontaneously or with controlled ventilation. The Bain anesthesia breathing circuit is a coaxial version of the Mapleson D breathing circuit except that the fresh gas enters through a narrow tube within the corrugated expiratory limb of the circuit. The Jackson-Rees breathing circuit is a modification of the Mapleson E breathing circuit and is called a Mapleson F breathing circuit. In the Jackson-Rees breathing circuit, the adjustable unidirectional expiratory valve is incorporated into the reservoir bag, and the fresh-gas-flow inlet is close to the patient. This arrangement offers the advantage of ease of instituting assisted or controlled ventilation of the lungs, as well as monitoring ventilation by movement of the reservoir bag during spontaneous breathing (Ehrenwerth: Anesthesia Equipment: Principles and Applications, ed 2, pp 109–117; Miller: Miller’s Anesthesia, ed 8, pp 780–781). 29
89. Bain system is modification of. Match the figures below with the correct numbered statement. Each lettered figure may be selected once, more than once, or not at all. A B C D E F FGF FGF FGF FGF FGF FGF
89. (D) There are six different types of Mapleson breathing circuits (designated A through F). These circuits vary in arrangement of the fresh-gas-flow inlet, tubing, mask, reservoir bag, and unidirectional expiratory valve. These systems are lightweight, portable, and easy to clean; they offer low resistance to breathing, and, because of high fresh gas inflows, they prevent rebreathing of exhaled gases. In addition, with these breathing circuits, the concentration of volatile anesthetic gases and O2 delivered to the patient can be accurately estimated. The reservoir bag enables the anesthesia provider to provide assisted or controlled ventilation of the lungs. The unidirectional expiratory valve functions to direct fresh gas into the patient and exhaled gases out of the circuit. In the Mapleson A breathing circuit, the unidirectional expiratory valve is near the patient, and the fresh-gas-flow inlet is proximal to the reservoir bag. This arrangement is the most efficient for elimination of CO2 during spontaneous breathing. However, because the unidirectional expiratory valve must be tightened to permit production of positive airway pressure when the gas reservoir bag is manually compressed, this breathing circuit is less efficient in preventing rebreathing of CO2 during assisted or controlled ventilation of the lungs. The structure of the Mapleson D breathing circuit is similar to that of the Mapleson A breathing circuit except that the positions of the fresh-gasflow inlet and the unidirectional expiratory valve are reversed. The placement of the fresh-gas-flow inlet near the patient produces efficient elimination of CO2, regardless of whether the patient is breathing spontaneously or with controlled ventilation. The Bain anesthesia breathing circuit is a coaxial version of the Mapleson D breathing circuit except that the fresh gas enters through a narrow tube within the corrugated expiratory limb of the circuit. The Jackson-Rees breathing circuit is a modification of the Mapleson E breathing circuit and is called a Mapleson F breathing circuit. In the Jackson-Rees breathing circuit, the adjustable unidirectional expiratory valve is incorporated into the reservoir bag, and the fresh-gas-flow inlet is close to the patient. This arrangement offers the advantage of ease of instituting assisted or controlled ventilation of the lungs, as well as monitoring ventilation by movement of the reservoir bag during spontaneous breathing (Ehrenwerth: Anesthesia Equipment: Principles and Applications, ed 2, pp 109–117; Miller: Miller’s Anesthesia, ed 8, pp 780–781). 29
90. Jackson-Rees system. Match the figures below with the correct numbered statement. Each lettered figure may be selected once, more than once, or not at all. A B C D E F FGF FGF FGF FGF FGF FGF
90. (F) There are six different types of Mapleson breathing circuits (designated A through F). These circuits vary in arrangement of the fresh-gas-flow inlet, tubing, mask, reservoir bag, and unidirectional expiratory valve. These systems are lightweight, portable, and easy to clean; they offer low resistance to breathing, and, because of high fresh gas inflows, they prevent rebreathing of exhaled gases. In addition, with these breathing circuits, the concentration of volatile anesthetic gases and O2 delivered to the patient can be accurately estimated. The reservoir bag enables the anesthesia provider to provide assisted or controlled ventilation of the lungs. The unidirectional expiratory valve functions to direct fresh gas into the patient and exhaled gases out of the circuit. In the Mapleson A breathing circuit, the unidirectional expiratory valve is near the patient, and the fresh-gas-flow inlet is proximal to the reservoir bag. This arrangement is the most efficient for elimination of CO2 during spontaneous breathing. However, because the unidirectional expiratory valve must be tightened to permit production of positive airway pressure when the gas reservoir bag is manually compressed, this breathing circuit is less efficient in preventing rebreathing of CO2 during assisted or controlled ventilation of the lungs. The structure of the Mapleson D breathing circuit is similar to that of the Mapleson A breathing circuit except that the positions of the fresh-gasflow inlet and the unidirectional expiratory valve are reversed. The placement of the fresh-gas-flow inlet near the patient produces efficient elimination of CO2, regardless of whether the patient is breathing spontaneously or with controlled ventilation. The Bain anesthesia breathing circuit is a coaxial version of the Mapleson D breathing circuit except that the fresh gas enters through a narrow tube within the corrugated expiratory limb of the circuit. The Jackson-Rees breathing circuit is a modification of the Mapleson E breathing circuit and is called a Mapleson F breathing circuit. In the Jackson-Rees breathing circuit, the adjustable unidirectional expiratory valve is incorporated into the reservoir bag, and the fresh-gas-flow inlet is close to the patient. This arrangement offers the advantage of ease of instituting assisted or controlled ventilation of the lungs, as well as monitoring ventilation by movement of the reservoir bag during spontaneous breathing (Ehrenwerth: Anesthesia Equipment: Principles and Applications, ed 2, pp 109–117; Miller: Miller’s Anesthesia, ed 8, pp 780–781). 29
91. A 29-year-old man is admitted to the intensive care unit (ICU) after a drug overdose. The patient is placed on a ventilator with a set tidal volume (Vt) of 750 mL at a rate of 10 breaths/min. The patient is making no inspiratory effort. The measured minute ventilation is 6 L and the peak airway pressure is 30 cm H2O. What is the compression factor for this ventilator delivery circuit? A. 2 mL/(cm H2O) B. 3 mL/(cm H2O) C. 4 mL/(cm H2O) D. 5 mL/(cm H2O)
91. (D) A volume-cycled ventilator set to deliver a volume of 750 mL at a rate of 10/min would deliver a minute ventilation of 7.5 L. The measured minute ventilation, however, is only 6 L; therefore, 1.5 L must be absorbed by the breathing circuit. This volume is known as the compression volume. If one divides the volume by 10 (number of breaths/min), then one determines the compression volume/breath. This number (mL) can be further divided by the peak inflation pressure (cm H2O) to determine the actual compression factor, which in this case is 5 mL/(cm H2O) (Miller: Basics of Anesthesia, ed 6, p 208; Ehrenwerth: Anesthesia Equipment Principles and Applications, p 364). Compression volume = (V V )/ Respiratory rate Peak airway pressure (cm H O) delivered measured 2 ˙ − ˙ = 5 mL/(cm H2O)
92. A 62-year-old man is brought to the ICU after elective repair of an abdominal aortic aneurysm. His vital signs are stable, but he requires a sodium nitroprusside infusion at a rate of 10 μg/kg/min to keep the systolic blood pressure below 110 mm Hg. The Sao2 is 98% with controlled ventilation at 12 breaths/min and an Fio2 of 0.60. After 3 days, his Sao2 decreases to 85% on the pulse oximeter. Chest x-ray film and results of physical examination are unchanged. Which of the following would most likely account for this desaturation? A. Cyanide toxicity B. Thiocyanate toxicity C. Methemoglobinemia D. Thiosulfate toxicity
92. (C) The metabolism of nitroprusside in the body requires the conversion of oxyhemoglobin (Fe++) to methemoglobin (Fe+++). The presence of sufficient quantities of methemoglobin in the blood will cause the pulse oximeter to read 85% saturation regardless of the true arterial saturation. Cyanide toxicity is also a possibility in any patient who is receiving nitroprusside. Cyanide toxicity should be suspected when the patient develops metabolic acidosis or becomes resistant to the hypotensive effects of this drug despite a sufficient infusion rate. This can be confirmed by measuring the mixed venous Pao2, which would be elevated in the presence of cyanide toxicity. Thiocyanate toxicity is also a potential hazard of nitroprusside administration in patients with renal failure. Patients suffering from thiocyanate toxicity display nausea, mental confusion, and skeletal-muscle weakness (Miller: Miller’s Anesthesia, ed 8, pp 1545, 2228; Brunton: Goodman & Gilman’s The Pharmacological Basis of Therapeutics, ed 12, pp 782–783).
93. Maximizing which of the following lung parameters is the most important factor in prevention of postoperative pulmonary complications? A. Tidal volume (Vt) B. Inspiratory reserve volume C. Vital capacity D. Functional residual capacity (FRC)
93. (D) (Please see diagram and table for explanation with Question 102.) FRC is composed of expiratory reserve volume plus residual volume. It is essential to maximize FRC in the postoperative period to ensure that it will be greater than closing volume. Closing volume is that lung volume at which small-airway closure begins to occur. Maximizing FRC, therefore, reduces atelectasis and lessens the incidence of arterial hypoxemia and pneumonia. Maneuvers aimed at increasing FRC include early ambulation, incentive spirometry, deep breathing, and intermittent positive-pressure breathing (Barash: Clinical Anesthesia, ed 7, p 279).
94. An 83-year-old woman is admitted to the ICU after coronary artery surgery. A pulmonary artery catheter is in place and yields the following data: central venous pressure (CVP) 5 mm Hg, cardiac output (CO) 4.0 L/min, mean arterial pressure (MAP) 90 mm Hg, mean pulmonary artery pressure (PAP) 20 mm Hg, pulmonary artery occlusion pressure (PAOP) 12 mm Hg, and heart rate 90. Calculate this patient’s pulmonary vascular resistance (PVR). A. 40 dyne-sec/cm5 B. 80 dyne-sec/cm5 C. 160 dyne-sec/cm5 D. 200 dyne-sec/cm5
94. (C) PVR = (PAP PAOP) CO mean� � 80 where PVR is the pulmonary vascular resistance, PAPmean is the mean pulmonary artery pressure, PAOP is the mean pulmonary capillary occlusion pressure, and CO is the cardiac output. PVR = (20� ) � 80 = 160 dyne-sec/cm5 12 4 The normal range for PVR is 50 to 150 dyne-sec/cm5 (Miller: Miller’s Anesthesia, ed 8, pp 1460–1461).
95. A 72-year-old man with a history of myocardial infarction 12 months earlier is scheduled to undergo elective repair of a 6-cm abdominal aortic aneurysm under general anesthesia. When would this patient be at highest risk for another myocardial infarction? A. During placement of the aortic cross-clamp B. Upon release of the aortic cross-clamp C. 24 hours postoperatively D. On the third postoperative day
95. (D) For reasons that are not fully understood, patients who have sustained a myocardial infarction and subsequently undergo surgery are most likely to have another infarction on the third postoperative day (Miller: Respiratory Physiology and Critical Care Medicine 37
96. Calculate the body mass index (BMI) of a man 200 cm (6 feet 6 inches) tall who weighs 100 kg (220 lb). A. 20 B. 25 C. 30 D. 35
96. (B) Calculation of BMI for adults (>20 years of age) can help identify patients who are underweight (BMI <18.5), normal weight (BMI 18.5-24.9), overweight (BMI 25-29.9), class 1 obesity (BMI 30-34.9), class 2 obesity (BMI 35-39.9), class 3 obesity (BMI 40-49.9), and the superobese (BMI >50). BMI = mass (kg) (Height)2 (meters ) BMI = 100 25 (2)2 = All major organ systems are affected as a consequence of obesity. The greatest concerns for the anesthesiologist are, however, related to the heart and lungs. Cardiac output must increase about 0.1 L/min for each extra kilogram of adipose tissue. As a consequence, obese patients frequently are hypertensive, and many ultimately develop cardiomegaly and left-sided heart failure. FRC is reduced in obese patients, and management of the airway often can be difficult (Miller: Miller’s Anesthesia, ed 8, pp 2200–2201).
97. The normal FEV1/FVC ratio is A. 0.95 B. 0.80 C. 0.60 D. 0.50
97. (B) The forced expiratory volume in 1 second (FEV1) is the total volume of air that can be exhaled in the first second. Normal healthy adults can exhale approximately 75% to 85% of their forced vital capacity (FVC) in the first second, 94% in 2 seconds, and 97% in 3 seconds. Therefore, the normal FEV1/FVC ratio is 0.75 or higher. In the presence of obstructive airway disease, the FEV1/FVC ratio less than 70% reflects mild obstruction, less than 60% moderate obstruction, and less than 50% severe obstruction. This ratio can be used to determine the severity of obstructive airway disease and to monitor the efficacy of bronchodilator therapy (Barash: Clinical Anesthesia, ed 7, p 279).
98. Direct current (DC) cardioversion is not useful and, therefore, NOT indicated in an unstable patient with which of the following? A. Supraventricular tachycardia in a patient with Wolff-Parkinson-White syndrome B. Atrial flutter C. Multifocal atrial tachycardia (MAT) D. New-onset atrial fibrillation
98. (C) MAT is a non-reentrant, ectopic atrial rhythm often seen in patients with chronic obstructive pulmonary disease (COPD). It is frequently confused with atrial fibrillation but, in contrast to atrial fibrillation, atrial flutter, and paroxysmal supraventricular tachycardia, DC cardioversion is ineffective in converting it to normal sinus rhythm. Ectopic atrial tachydysrhythmias are not amenable to cardioversion because they lack the re-entrant mechanism, which is necessary for successful termination with electrical counter shock (Miller: Miller’s Anesthesia, ed 8, pp 3191–3193).
99. During the first minute of apnea, the Paco2 will rise A. 2 mm Hg/min B. 4 mm Hg/min C. 6 mm Hg/min D. 8 mm Hg/min
99. (C) During apnea, the Paco2 will increase approximately 6 mm Hg during the first minute and then 3 to 4 mm Hg each minute thereafter (Miller: Basics of Anesthesia, ed 6, p 61).
100. Potential complications associated with total parenteral nutrition (TPN) include all of the following EXCEPT A. Ketoacidosis B. Hyperglycemia C. Hypoglycemia D. Hypophosphatemia
100. (A) TPN therapy is associated with numerous potential complications. Blood sugars need to be carefully monitored because hyperglycemia may develop due to the high glucose load and require treatment with insulin, and hypoglycemia may develop if TPN is abruptly stopped (i.e., infusion turned off or mechanical obstruction in the IV tubing). Other complications include electrolyte disturbances (e.g., hypokalemia, hypophosphatemia, hypomagnesemia, hypocalcemia), volume overload, catheter-related sepsis, renal and hepatic dysfunction, thrombosis of the central veins, and nonketotic hyperosmolar coma. Increased work of breathing is related to increased production of CO2 most frequently due to overfeeding. Acidosis in these patients is hyperchloremic metabolic acidosis resulting from formation of HCl during metabolism of amino acids. Ketoacidosis is not associated with TPN therapy (Hines: Stoelting’s Anesthesia and Co-Existing Disease, ed 6, p 331).
101. O2 requirement for a 70-kg adult is A. 150 mL/min B. 250 mL/min C. 350 mL/min D. 450 mL/min
101. (B) The O2 requirement for an adult is 3 to 4 mL/kg/min. The O2 requirement for a newborn is 7 to 9 mL/ kg/min. Alveolar ventilation (Va) in neonates is double that of adults to help meet their increased O2 requirements. This increase in Va is achieved primarily by an increase in respiratory rate as Vt is similar to that of adults (i.e., 7 mL/kg). Although CO2 production also is increased in neonates, the elevated Va maintains the Paco2 near 38 to 40 mm Hg (Barash: Clinical Anesthesia, ed 7, pp 1181–1182).
102. The FRC is composed of the A. Expiratory reserve volume and residual volume B. Inspiratory reserve volume and residual volume C. Inspiratory capacity and vital capacity D. Expiratory capacity and Vt
102. (A) A comprehensive understanding of respiratory physiology is important for understanding the effects of both regional and general anesthesia on respiratory mechanics and pulmonary gas exchange. The volume of gas remaining in the lungs after a normal expiration is called the functional residual capacity. The volume of gas remaining in the lungs after a maximal expiration is called the residual volume. The difference between these two volumes is called the expiratory reserve volume. Therefore, the FRC is composed of the expiratory reserve volume and residual volume (Barash: Clinical Anesthesia, ed 7, pp 278–279; Stoelting: Pharmacology and Physiology in Anesthetic Practice, ed 4, pp 776–777). 38 Part 1 Basic Sciences LUNG VOLUMES AND CAPACITIES Measurement Abbreviation Normal Adult Value Tidal volume Vt 500 mL (6-8 mL/kg) Inspiratory reserve volume IRV 3000 mL Expiratory reserve volume ERV 1200 mL Residual volume RV 1200 mL Inspiratory capacity IC 3500 mL Functional residual capacity FRC 2400 mL Vital capacity VC 4500 mL (60-70 mL/kg) Forced exhaled volume in 1 sec FEV1 80% Total lung capacity TLC 5900 mL 1200 2400 3600 4800 6000 0 Lung volume (mL) RV ERV VT IRV IC FRC VC TLC
103. Which of the following statements correctly defines the relationship between minute ventilation ( ˙V E ), dead space ventilation ( ˙V D), and Paco2? A. If ˙V E is constant and ˙V D increases, then Paco2 will increase B. If ˙V E is constant and ˙V D increases, then Paco2 will decrease C. If ˙V D is constant and ˙V E increases, then Paco2 will increase D. If ˙V D is constant and ˙V E decreases, then Paco2 will decrease
103. (A) The volume of gas in the conducting airways of the lungs (and not available for gas exchange) is called the anatomic dead space. The volume of gas in ventilated alveoli that are unperfused (and not available for gas exchange) is called the functional dead space. The anatomic dead space together with the functional dead space is called the physiologic dead space. Physiologic dead space ventilation (Vd) can be calculated by the Bohr dead space equation, which is mathematically expressed as follows: V /V = (Pa P ) Pa D T CO ECO CO 2 2 2 − where Vd/Vt is the ratio of Vd to Vt, and a and e represent arterial and mixed expired, respectively. Of the choices given, only the first is correct. A large increase in Vd will result in an increase in Paco2 (Barash: Clinical Anesthesia, ed 7, pp 275–277; West: Respiratory Physiology, ed 9, pp 19–21; Miller: Miller’s Anesthesia, ed 8, pp 446–447).
104. A 22-year-old patient who sustained a closed head injury is brought to the operating room (OR) from the ICU for placement of a dural bolt. Hemoglobin has been stable at 15 g/dL. Blood gas analysis immediately before induction reveals a Pao2 of 120 mm Hg and an arterial saturation of 100%. After induction, the Pao2 rises to 150 mm Hg and the saturation remains the same. How has the oxygen content of this patient’s blood changed? A. It has increased by 10% B. It has increased by 5% C. It has increased by less than 1% D. Cannot be determined without Paco2
104. (C) The oxygen content of blood can be calculated with the following formula: O2 content = 1.39 × [Hgb] × arterial saturation + (0.003 × Pao2) First oxygen content = (1.39 × 15 × 1.0) + 0.003 × 120 = 21.21 mL / dL Second oxygen content = (1.39 × 15 × 1.0) + 0.003 × 150 = 21.30 mL/dL The difference in the oxygen content is 0.09 mL/dL. This represents a change of 0.42% (Miller: Basics of Anesthesia, ed 6, p 57 ). Respiratory Physiology and Critical Care Medicine 39
105. Inhalation of CO2 increases ˙V E by A. 0.5 to 1 L/min/mm Hg increase in Paco2 B. 2 to 3 L/min/mm Hg increase in Paco2 C. 3 to 5 L/min/mm Hg increase in Paco2 D. 5 to 10 L/min/mm Hg increase in Paco2
105. (B) The degree of ventilatory depression caused by volatile anesthetics can be assessed by measuring resting Paco2, the ventilatory response to hypercarbia, and the ventilatory response to hypoxemia. Of these techniques, the resting Paco2 is the most frequently used index. However, measuring the effects of increased Paco2 on ventilation is the most sensitive method of quantifying the effects of drugs on ventilation. In awake unanesthetized humans, inhalation of CO2 increases minute ventilation ( ˙V E) by approximately 2 to 3 L/min/mm Hg increase in Paco2. Using this technique, halothane, isoflurane, desflurane-O2, desflurane-N2O, and N2O cause a dose-dependent depression of the ventilation (Miller: Basics of Anesthesia, ed 6, pp 93–94).
106. What is the O2 content of whole blood if the hemoglobin concentration is 10 g/dL, the Pao2 is 60 mm Hg, and the Sao2 is 90%? A. 10 mL/dL B. 12.5 mL/dL C. 15 mL/dL D. 17.5 mL/dL
106. (B) (See also explanation to Question 104.) The amount of O2 in blood (O2 content) is the sum of the amount of O2 dissolved in plasma and the amount of O2 combined with hemoglobin. The amount of O2 dissolved in plasma is directly proportional to the product of the blood/gas solubility coefficient of O2 (0.003) and Pao2. The amount of O2 bound to hemoglobin is directly related to the fraction of hemoglobin that is saturated. One gram of hemoglobin can bind 1.39 mL of O2. The mathematical expression of O2 content is as follows: O2 content = 1.39 × [Hgb] × SaO2 + (0.003 × PaO2) where [Hgb] is the hemoglobin concentration (g/dL), Sao2 is the fraction of hemoglobin saturated with O2, and (0.003 × Pao2) is the amount of O2 dissolved in plasma. In this case (1.39 × 10 × 0.9) + (0.003 × 60) = 12.51 + 0.18 = 12.69 or approximately 13 mL/dL (Miller: Basics of Anesthesia, ed 6, p 57 ).
107. Each of the following will cause erroneous readings by dual-wavelength pulse oximeters EXCEPT A. Carboxyhemoglobin B. Methylene blue C. Fetal hemoglobin D. Methemoglobin
107. (C) The presence of hemoglobin species other than oxyhemoglobin can cause erroneous readings by dualwavelength pulse oximeters. Hemoglobin species such as carboxyhemoglobin and methemoglobin, dyes such as methylene blue and indocyanine green, and some colors of nail polish will cause erroneous readings. Because the absorption spectrum of fetal hemoglobin is similar to that of adult oxyhemoglobin, fetal hemoglobin does not significantly affect the accuracy of these types of pulse oximeters. High levels of bilirubin have no significant effect on the accuracy of dual-wavelength pulse oximeters but may cause falsely low readings by nonpulsatile oximeters (Miller: Miller’s Anesthesia, ed 8, pp 1545–1547).
108. �0.5 0 1 2 3 4 5 6 7 8 9 10 0 0.5 1.0 1.5 2.0 2.5 3.0 3.5 Pressure (kPa) D C B A Volume (L) In the diagram above, curve “D” represents A. Emphysema B. Chronic bronchitis C. Normal lungs D. Fibrotic lungs
108. (D) This graph depicts lung volumes as a function of pressure or compliance; one kPa is roughly equal to 10 cm H2O. Curve A shows an enormous volume with a small pressure (i.e., emphysema). Curve B depicts chronic bronchitis or asthma. The compliance curve is roughly the same as the normal lung, curve C, but volumes have increased. Curve D depicts stiff noncompliant lungs as seen with fibrosis or ARDS (Miller: Miller’s Anesthesia, ed 8, pp 447–448).
109. The P50 for normal adult hemoglobin is approximately A. 15 mm Hg B. 25 mm Hg C. 35 mm Hg D. 45 mm Hg Respiratory Physiology and Critical Care Medicine 31
109. (B) P50 is the Pao2 required to produce 50% saturation of hemoglobin. The P50 for adult hemoglobin at a pH of 7.4 and body temperature of 37° C is 26 mm Hg (Stoelting: Pharmacology and Physiology in Anesthetic Practice, ed 4, pp 788–789; Miller: Basics of Anesthesia, ed 6, p 56).
110. During a normal Vt (500-mL) breath, the transpulmonary pressure increases from 0 to 5 cm H2O. The product of transpulmonary pressure and Vt is 2500 cm H2O-mL. This expression of the pressurevolume relationship during breathing determines what parameter of respiratory mechanics? A. Lung compliance B. Airway resistance C. Pulmonary elastance D. Work of breathing
110. (D) The work of breathing is defined as the product of transpulmonary pressure and Vt. The work of breathing is related to two factors: the work required to overcome the elastic forces of the lungs, and the work required to overcome airflow or frictional resistances of the airways (Barash: Clinical Anesthesia, ed 7, pp 266–268; Miller: Miller’s Anesthesia, ed 8, p 1563).
111. An oximetric pulmonary artery catheter is placed in a 69-year-old man who is undergoing surgical repair of an abdominal aortic aneurysm under general anesthesia. Before the aortic cross-clamp is placed, the mixed venous O2 saturation decreases from 75% to 60%. Each of the following could account for the decrease in mixed venous O2 saturation EXCEPT A. Hypovolemia B. Bleeding C. Congestive heart failure D. Sepsis
111. (D) The normal mixed venous O2 saturation is 75%. Physiologic factors that affect mixed venous O2 saturation include hemoglobin concentration, arterial Pao2, cardiac output, and O2 consumption. Anemia, hypoxia, decreased cardiac output, and increased O2 consumption decrease mixed venous O2 saturation. During sepsis with adequate volume resuscitation, the cardiac output is increased and maldistribution of perfusion (distributive shock) results in an elevated mixed-venous O2 saturation. Mixed venous O2 saturation (Svo2) is related to a number of factors, as shown in this equation: Sv V 13.9 Q Hgb O O O 2 2 = Sa − 2 ˙ ˙ ( ( where Hgb is hemoglobin concentration, 13.9 is a constant (O2 combining power of Hgb [mL/10 g]), ˙Q is cardiac output, and ˙V O2 is the oxygen consumption (Miller: Miller’s Anesthesia, ed 8, pp 1386–1387 ). 40 Part 1 Basic Sciences
112. The normal vital capacity for a 70-kg man is A. 1 L B. 2 L C. 5 L D. 7 L
112. (C) The volume of gas exhaled during a maximum expiration is the vital capacity. In a normal healthy adult, the vital capacity is 60 to 70 mL/kg. In a 70-kg patient, the vital capacity is approximately 5 L (Stoelting: Pharmacology and Physiology in Anesthetic Practice, ed 4, p 776; Barash: Clinical Anesthesia, ed 7, p 278).
113. A 32-year-old man is found unconscious by the fire department in a room where he has inhaled 0.1% carbon monoxide for a prolonged period. His respiratory rate is 42 breaths/min, but he is not cyanotic. Carbon monoxide has increased this patient’s minute ventilation by which of the following mechanisms? A. Shifting the O2 hemoglobin dissociation curve to the left B. Increasing CO2 production C. Causing lactic acidosis D. Decreasing Pao2
113. (C) Carbon monoxide inhalation is the most common immediate cause of death from fire. Carbon monoxide binds to hemoglobin with an affinity 200 times greater than that of oxygen. For this reason, very small concentrations of carbon monoxide can greatly reduce the oxygen-carrying capacity of blood. In spite of this, the arterial Pao2 often is normal. Because the carotid bodies respond to arterial Pao2, there would not be an increase in minute ventilation until tissue hypoxia was sufficient to produce lactic acidosis (Hines: Stoelting’s Anesthesia and Co-Existing Disease, ed 6, pp 554–555; Miller: Miller’s Anesthesia, ed 8, pp 2679–2680; West: Respiratory Physiology, ed 9, pp 80–82).
114. An acute increase in Paco2 of 10 mm Hg will result in a decrease in pH of A. 0.01 pH unit B. 0.02 pH unit C. 0.04 pH unit D. 0.08 pH unit
114. (D) Respiratory acidosis is present when the Paco2 exceeds 44 mm Hg. Respiratory acidosis is caused by decreased elimination of CO2 by the lungs (i.e., hypoventilation) or increased metabolic production of CO2. An acute increase in Paco2 of 10 mm Hg will result in a decrease in pH of approximately 0.08 pH unit. The acidosis of arterial blood will stimulate ventilation via the carotid bodies, and the acidosis of cerebrospinal fluid will stimulate ventilation via the medullary chemoreceptors located in the fourth cerebral ventricle. Volatile anesthetics greatly attenuate the carotid body–mediated and aortic body–mediated ventilatory responses to arterial acidosis, but they have little effect on the medullary chemoreceptor– mediated ventilatory response to cerebrospinal fluid acidosis (Miller: Basics of Anesthesia, ed 6, pp 339–340, 343).
115. You are taking care of a patient in shock in the ICU, and, after adequate fluid resuscitation, you decide to add a vasoactive medication. Each of the following initial infusion rates is correct EXCEPT A. Dopamine 2 to 10 μg/kg/min B. Norepinephrine 0.1 to 0.5 μg/kg/min C. Vasopressin 0.01 to 0.04 units/kg/min D. None of the above; they all are reasonable starting doses
115. (C) Dopamine can be infused at low doses (2-5 μg/kg/min), moderate doses (5-10 μg/kg/min), or high doses (10-20 μg/kg/min). Many feel that if dopamine is needed at rates greater than 10 μg/kg/min, one should use epinephrine or norepinephrine infusions instead. Epinephrine and norepinephrine infusion rates are commonly started at 0.1 to 0.5 μg/kg/min. Although many cardiovascular drugs are based on a μg/kg/min dose, vasopressin is not. The starting vasopressin dose is 0.01 to 0.04 unit/min (American Heart Association: 2010 American Heart Association Guidelines for Cardiopulmonary Resuscitation and Emergency Cardiovascular Care Science, pp S774–S775; Kaplan: Cardiac Anesthesia, ed 6, pp 1000, 1034–1035; Miller: Basics of Anesthesia, ed 6, pp 675–676).
116. A 44-year-old patient is hyperventilated to a Paco2 of 24 mm Hg for 48 hours. What [HCO3 −] would you expect (normal [HCO3 −] is 24 mEq/L)? A. 10 mEq/L B. 12 mEq/L C. 14 mEq/L D. 16 mEq/L
116. (D) Respiratory alkalosis is present when the Paco2 is less than 36 mm Hg. There are three compensatory mechanisms responsible for attenuating the increase in pH that accompanies respiratory alkalosis. First, there is an immediate shift in the equilibrium of the [HCO3 −] buffer system, which results in the production of CO2. Second, alkalosis stimulates the activity of phosphofructokinase, which increases glycolysis and the production of pyruvate and lactic acid. Third, there is a decrease in reabsorption of [HCO3 −] by the proximal and distal renal tubules. These three compensatory mechanisms result in a maximum decrease in [HCO3 −] of approximately 5 mEq/L for every 10 mm Hg decrease in Paco2 less than 40 mm Hg (Miller: Basics of Anesthesia, ed 6, p 340; Butterworth: Morgan & Mikhail’s Clinical Anesthesia, ed 5, pp 1154–1155).
117. The diagram below depicts which mode of ventilation? 30 20 10 0 A. Spontaneous ventilation B. Controlled ventilation C. Assisted ventilation D. Assisted/controlled ventilation
117. (D) Mechanical ventilation of the lungs can be accomplished by various modes. These modes are categorized as controlled, assisted, assisted/controlled, controlled with positive end-expiratory pressure (PEEP), and assisted/controlled using intermittent mandatory ventilation (IMV). Assisted/controlled modes of mechanical ventilation are used in patients when the muscles of respiration require rest because minimal breathing efforts are required. With the assisted/controlled mode of ventilation, positive-pressure ventilation is triggered by small breathing efforts produced by the patient. The airway pressure tracing shown is typical of that of a patient requiring assisted/controlled ventilation (Hines: Stoelting’s Anesthesia and Co-Existing Disease, ed 6, pp 207–208). Respiratory Physiology and Critical Care Medicine 41
118. A 35-year-old morbidly obese patient is discharged after gastric bypass surgery. She is readmitted 4 days later after she falls and twists her ankle. She is noted in the emergency room (ER) to be in atrial fibrillation and is hypotensive but only complains of leg pain. She is admitted to the hospital, and temperature on admission is 38.6° C and heart rate is 105 beats/min. The next step in management of her dysrhythmia should be A. Ibutilide B. Procainamide C. Echocardiographic study D. DC cardioversion
118. (C) The first step in evaluating any patient with a tachycardia is to determine if the patient is hemodynamically stable or unstable (serious signs or symptoms are chest pain or congestive heart failure due to the tachycardia). In the unstable patient, DC cardioversion should be performed for rapid heart rate control regardless of the duration of atrial fibrillation. In this case, where the patient is reasonably stable, the three major goals in the management of atrial fibrillation should be considered. These goals are control of ventricular rate, assessment of anticoagulation needs, and conversion to sinus rhythm. In addition, the underlying cause of atrial fibrillation should be sought and treated. Because this patient is febrile and may be dehydrated, an intravenous (IV) line for fluid resuscitation should be initiated. Because we do not know when atrial fibrillation developed (after 48 hours, embolic events may occur with conversion to sinus rhythm), it would be best not to convert the atrial fibrillation to sinus rhythm using either ibutilide or procainamide until the patient is adequately anticoagulated. Adequate anticoagulation should usually be therapeutic for at least 3 weeks. In marginal cases where the duration of atrial fibrillation is uncertain, cardiac consultation and transesophageal echocardiography to exclude atrial thrombus should be performed before cardioversion. This patient should undergo cardiac echocardiographic study to look for intra-atrial thrombus and to determine the ejection fraction (EF) of the ventricle. After adequate hydration, rate control could be improved with calcium channel blockers or β-blockers in patients with preserved left ventricular function (EF > 40%) or with digoxin, diltiazem, or amiodarone if EF is less than 40% (2010 AHA Guidelines for CPR and Emergency Cardiovascular Care: Circulation 122 (Suppl 3) S750–S756). 42 Part 1 Basic Sciences
119. The P50 of sickle cell hemoglobin is A. 19 mm Hg B. 26 mm Hg C. 31 mm Hg D. 35 mm Hg
119. (C) A P50 less than 26 mm Hg defines a leftward shift of the oxyhemoglobin dissociation curve. This means that at any given Pao2, hemoglobin has a higher affinity for O2. A P50 greater than 26 mm Hg describes a rightward shift of the oxyhemoglobin dissociation curve. This means that at any given Pao2, hemoglobin has a lower affinity for O2. Conditions that cause a rightward shift of the oxyhemoglobin dissociation curve are metabolic and include respiratory acidosis, hyperthermia, increased erythrocyte 2,3-diphosphoglycerate (2,3-DPG) content, pregnancy, and abnormal hemoglobins, such as sickle cell hemoglobin or thalassemia. Alkalosis, hypothermia, fetal hemoglobin, abnormal hemoglobin species, such as carboxyhemoglobin, methemoglobin, and sulfhemoglobin, and decreased erythrocyte 2,3-DPG content will cause a leftward shift of the oxyhemoglobin dissociation curve. Also see explanation to Question 109 (Miller: Miller’s Anesthesia, ed 8, p 1843; West: Respiratory Physiology, ed 9, pp 79–82).
120. Data from the ARDS network trial (ARDSNet) showed increased mortality from A. Atelectrauma B. Volutrauma C. Barotrauma D. Inhaled nitric oxide 32 Part 1 Basic Sciences
120. (B) Adult respiratory distress disorder (ARDS) was first reported in adults in 1967 and is associated with decreased lung compliance. Initial therapies for ARDS included mechanical ventilation with tidal volumes of 10 to 15 mL/kg with rates to achieve a normal pH and Paco2. In 2000, the National Institutes of Health (NIH) ARDS Network (ARDSNet) trial noted a reduction in mortality for patients with ARDS who were ventilated with low tidal volumes (6 mL/kg predicted body weight [PBW]—mortality rate of 31%) compared to traditional tidal volumes (12 mL/kg PBW—mortality rate of 40%). It was felt that the larger tidal volumes caused overdistention of the alveoli (i.e., produced volume trauma or volutrauma). This increased alveolar volume resulted in mechanical injury and a systemic inflammatory response. It was felt that the stretch and not the pressure (barotrauma) caused the release of the inflammatory cytokinins into the circulation. Because the lower tidal volumes used were associated with an elevation of arterial CO2 and lower arterial oxygen levels, the term “permissive hypercapnia and hypoxemia” was used. Patients with ARDS also develop atelectasis. Recruitment maneuvers (sustained breaths of increased airway pressures) were used to re-expand atelectatic alveoli to avoid atelectrauma. However, results with the recruitment breaths showed only a transient increase in oxygenation and no change in mortality. Another respiratory technique proposed included the use of inhaled nitrous oxide (iNO) that can improve ventilation-perfusion mismatch and improve oxygenation. Randomized controlled studies have shown only limited effectiveness with no overall improvement in mortality or duration of ventilation. Further studies are looking at iNO for specific conditions (e.g., severe pulmonary hypertension, right ventricular failure refractory hypoxemia) (Miller: Basics of Anesthesia, ed 6, p 669; Miller: Miller’s Anesthesia, ed 8, pp 3040–3044, 3078–3079).
121. Which of the following is the correct mathematical expression of Fick’s law of diffusion of a gas through a lipid membrane ( ˙V = rate of diffusion, D = diffusion coefficient of the gas, A = area of the membrane, P1 − P2 = transmembrane partial pressure gradient of the gas, T = thickness of the membrane)? A. V D A T P1 P2 ˙ = × × − B. V = A T D (P1 − P2 ) ˙ × C. V = D A(P P ) T ˙ × 1 2 − D. V = D T(P P ) A ˙ × 1 2 −
121. (C) The rate at which a gas diffuses through a lipid membrane is directly proportional to the area of the membrane, the transmembrane partial pressure gradient of the gas, and the diffusion coefficient of the gas, and it is inversely proportional to the thickness of the membrane. The diffusion coefficient of the gas is directly proportional to the square root of gas solubility and is inversely proportional to the square root of the molecular weight of the gas. This is known as Fick’s law of diffusion (Barash: Clinical Anesthesia, ed 7, p 1147).
122. Each of the following is decreased in elderly patients compared with their younger counterparts EXCEPT A. Closing volume B. FEV1 C. Ventilatory response to hypercarbia D. Vital capacity
122. (A) Aging is associated with reduced ventilatory volumes and capacities, and decreased efficiency of pulmonary gas exchange. These changes are caused by progressive stiffening of cartilage and replacement of elastic tissue in the intercostal and intervertebral areas, which decreases compliance of the thoracic cage. In addition, progressive kyphosis or scoliosis produces upward and anterior rotation of the ribs and sternum, which further restricts chest wall expansion during inspiration. With aging, the FRC, residual volume, and closing volume are increased, whereas the vital capacity, total lung capacity, maximum breathing capacity, FEV1, and ventilatory response to hypercarbia and hypoxemia are reduced. In addition, age-related changes in lung parenchyma, alveolar surface area, and diminished pulmonary capillary bed density cause ventilation/perfusion mismatch, which decreases resting Pao2 (Miller: Basics of Anesthesia, ed 6, pp 571–572; Hines: Stoelting’s Anesthesia and Co-Existing Disease, ed 6, p 644).
123. Calculate the Vd/Vt ratio (physiologic dead space ventilation) based on the following data: Paco2 45 mm Hg, mixed expired CO2 tension (Peco2) 30 mm Hg. A. 0.1 B. 0.2 C. 0.3 D. 0.4
123. (C) Physiologic dead space ventilation can be estimated using the Bohr equation (described in the explanation to Question 103): mm Hg mm Hg mm Hg mm Hg mm Hg VD/VT = 45 − 30 = 45 15 45 = 0.33 (Barash: Clinical Anesthesia, ed 7, pp 276–277; West: Respiratory Physiology, ed 9, pp 19–21; Miller: Miller’s Anesthesia, ed 8, pp 446–447). Respiratory Physiology and Critical Care Medicine 43
124. Which of the following statements concerning the distribution of O2 and CO2 in the upright lungs is TRUE? A. Pao2 is greater at the apex than at the base B. Paco2 is greater at the apex than at the base C. Both Pao2 and Paco2 are greater at the apex than at the base D. Both Pao2 and Paco2 are greater at the base than at the apex
124. (A) The ventilation/perfusion ratio is greater at the apex of the lungs than at the base of the lungs. Thus, dependent regions of the lungs are hypoxic and hypercarbic compared to the nondependent regions. Also see explanation to Question 132 (Miller: Miller’s Anesthesia, ed 8, pp 451–454; West: Respiratory Physiology, ed 9, pp 21–22, 44–46).
125. Which of the following acid-base disturbances is the least well-compensated? A. Metabolic alkalosis B. Respiratory alkalosis C. Increased anion gap metabolic acidosis D. Normal anion gap metabolic acidosis
125. (A) The degree to which a person can hypoventilate to compensate for metabolic alkalosis is limited; hence, this is the least well-compensated acid-base disturbance. Respiratory compensation for metabolic alkalosis is rarely more than 75% complete. Hypoventilation to a Paco2 greater than 55 mm Hg is the maximum respiratory compensation for metabolic alkalosis. A Paco2 greater than 55 mm Hg most likely reflects concomitant respiratory acidosis (Miller: Basics of Anesthesia, ed 6, p 342).
126. What is the (calculated) Pao2 of a patient on room air in Denver, Colorado? (Assume a barometric pressure of 630 mm Hg, respiratory quotient of 0.8, and Paco2 of 34 mm Hg.) A. 80 mm Hg B. 90 mm Hg C. 100 mm Hg D. 110 mm Hg
126. (A) Pao2 can be estimated using the alveolar gas equation, which is given as follows: PaO (PB )F Pa R IO CO2 2 = − 47 2 − where Pb is the barometric pressure (mm Hg), Fio2 is the fraction of inspired O2, Paco2 is the arterial CO2 tension (mm Hg), and R is the respiratory quotient (Barash: Clinical Anesthesia, ed 7, p 277; West: Respiratory Physiology, ed 9, p 59).
127. A venous blood sample from which of the following sites would correlate most reliably with Pao2 and Paco2? A. Jugular vein B. Subclavian vein C. Antecubital vein D. Vein on posterior surface of a warmed hand
127. (D) When arterial sampling is not possible, “arterialized” venous blood can be used to estimate ABG tensions. Because blood in the veins on the back of the hands has very little O2 extracted, the O2 content in this blood best approximates the O2 content in a sample of blood obtained from an artery (Stoelting: Basics of Anesthesia, ed 5, p 324).
128. Which of the following pulmonary function tests is LEAST dependent on patient effort? A. Forced expiratory volume in 1 second (FEV1) B. Forced vital capacity (FVC) C. FEF 800 to 1200 D. FEF 25% to 75%
128. (D) Pulmonary function tests can be divided into those that assess ventilatory capacity and those that assess pulmonary gas exchange. The simplest test to assess ventilatory capacity is the FEV1/FVC ratio. Other tests to assess ventilatory capacity include the maximum midexpiratory flow (FEF 25%-75%), MVV, and flow-volume curves. The most significant disadvantage of these tests is that they are dependent on patient effort. However, because the FEF 25% to 75% is obtained from the midexpiratory portion of the flow-volume loop, it is least dependent on patient effort. Also see explanation to Question 97 (Barash: Clinical Anesthesia, ed 7, p 279).
129. A 33-year-old woman with 20% carboxyhemoglobin is brought to the ER for treatment of smoke inhalation. Which of the following is LEAST consistent with a diagnosis of carbon monoxide poisoning? A. Cyanosis B. Pao2 105 mm Hg, oxygen saturation 80% on initial room air arterial blood gases (ABGs) C. 98% oxygen saturation on dual-wavelength pulse oximeter D. Oxyhemoglobin dissociation curve shifted far to the left
129. (A) Carbon monoxide binds to hemoglobin with an affinity greater than 200 times that of oxygen. This stabilizes the oxygen–hemoglobin complex and hinders release of oxygen to the tissues, leading to a leftward shift of the oxyhemoglobin dissociation curve. The diagnosis is suggested when there is a low oxygen hemoglobin saturation in the face of a normal Pao2. The two-wave pulse oximeter cannot distinguish oxyhemoglobin from carboxyhemoglobin so that a normal oxyhemoglobin saturation would be observed in the presence of high concentrations of carboxyhemoglobin. Carbon monoxide poisoning is not associated with cyanosis. See also explanations for Questions 113 and 140 (Hines: Stoelting’s Anesthesia and Co-Existing Disease, ed 6, pp 554–555; Miller: Miller’s Anesthesia, ed 8, pp 2679–2680).
130. The Pao2 − Pao2 of a patient breathing 100% O2 is 240 mm Hg. The estimated fraction of the cardiac output shunted past the lungs without exposure to ventilated alveoli (i.e., transpulmonary shunt) is A. 5% B. 12% C. 17% D. 20%
130. (B) The fraction of total cardiac output that traverses the pulmonary circulation without participating in gas exchange is called the transpulmonary shunt. It can be calculated exactly by the equation: ˙ ˙Q s/QT = CaO Cc� CvO O O 2 2 Cc� − − 2 2 where Cc′, Ca, and CvO2 stand for the content of oxygen in the alveolar capillary, artery, and mixed venous samples, respectively. This information is not provided in the question; however, the alveolar-toarterial partial pressure of oxygen difference is using high inspired oxygen concentrations. The alveolar to arterial oxygen difference can be used to estimate venous admixture, most commonly transpulmonary shunt. For every increase in alveolar-arterial O2 of 20 mm Hg, there is an increase in shunt fraction of 1% of the cardiac output. In the example, 240/20 = 12 and the transpulmonary shunt can be estimated at 12% (Miller: Miller’s Anesthesia, ed 44 Part 1 Basic Sciences
131. Each of the following will alter the position or slope of the CO2-ventilatory response curve EXCEPT A. Hypoxemia B. Fentanyl C. N2O D. Ketamine
131. (D) Measuring the ventilatory response to increased Paco2 is a sensitive method for quantifying the effects of drugs on ventilation. In general, all volatile anesthetics (including N2O), narcotics, benzodiazepines, and barbiturates depress the ventilatory response to increased Paco2 in a dose-dependent manner. The magnitude of ventilatory depression by volatile anesthetics is greater in patients with COPD than in healthy patients. Arterial blood gases (ABGs) may need to be monitored during recovery from general anesthesia in patients with COPD. Ketamine causes minimal respiratory depression. Typically, respiratory rate is decreased only 2 to 3 breaths/min and the ventilatory response to changes in Paco2 is maintained during ketamine anesthesia. Also see explanation to Question 105 (Miller: Basics of Anesthesia, ed 6, pp 63–64, 93–94, 110; Miller: Miller’s Anesthesia, ed 8, pp 691–693).
132. Which of the following statements concerning the distribution of alveolar ventilation (˙V A) in the upright lungs is TRUE? A. The distribution of ˙V A is not affected by body posture B. Alveoli at the apex of the lungs (nondependent alveoli) are better ventilated than those at the base C. All areas of the lungs are ventilated equally D. Alveoli at the base of the lungs (dependent alveoli) are better ventilated than those at the apex Respiratory Physiology and Critical Care Medicine 33
132. (D) (See also explantation to Question 124.) The orientation of the lungs relative to gravity has a profound effect on efficiency of pulmonary gas exchange. Because alveoli in dependent regions of the lungs expand more per unit change in transpulmonary pressure (i.e., are more compliant) than alveoli in nondependent regions of the lungs, ˙V A increases from the top to the bottom of the lungs. Because pulmonary blood flow increases more from the top to the bottom of the lungs than does ˙V A, the ventilation/ perfusion ratio is high in nondependent regions of the lungs and is low in dependent regions of the lungs. Therefore, in the upright lungs, the Pao2 and pH are greater at the apex, whereas the Paco2 is greater at the base (Miller: Miller’s Anesthesia, ed 8, pp 451–454; West: Respiratory Physiology, ed 9, pp 21–22, 44–46).
133. In the resting adult, what percentage of total body O2 consumption is due to the work of breathing? A. 2% B. 5% C. 10% D. 20%
133. (A) The work required to overcome the elastic recoil of the lungs and thorax, along with airflow or frictional resistances of the airways, contributes to the work of breathing. When the respiratory rate or airway resistance is high or pulmonary or chest wall compliance is reduced, a large amount of energy is spent overcoming the work of breathing. In the healthy resting adult, only 1% to 3% of total O2 consumption is used for the work of breathing at rest, but up to 50% may be needed in patients with pulmonary disease. Also see explanation to question 110 (Miller: Miller’s Anesthesia, ed 8, p 1563).
134. The anatomic dead space in a 70-kg man is A. 50 mL B. 150 mL C. 250 mL D. 500 mL
134. (B) The conducting airways (trachea, right and left mainstem bronchi, and lobar and segmental bronchi) do not contain alveoli and, therefore, do not take part in pulmonary gas exchange. These structures constitute the anatomic dead space. In the adult, the anatomic dead space is approximately 2 mL/kg. The anatomic dead space increases during inspiration because of the traction exerted on the conducting airways by the surrounding lung parenchyma. In addition, the anatomic dead space depends on the size and posture of the subject. Also see explanation to Question 103 (Stoelting: Pharmacology and Physiology in Anesthetic Practice, ed 4, p 778; Barash: Clinical Anesthesia, ed 7, p 276).
135. The most important buffering system in the body is A. Hemoglobin B. Plasma proteins C. Phosphate D. [HCO3 −]
135. (D) There are three main mechanisms that the body has to prevent changes in pH. The buffer systems (immediate), the ventilatory response (takes minutes), and the renal response (takes hours to days). The buffer systems represent the first line of defense against adverse changes in pH. The [HCO3 −] buffer system is the most important system and represents greater than 50% of the total buffering capacity of the body. Other important buffer systems include hemoglobin, which is responsible for approximately 35% of the buffering capacity of blood, phosphates, plasma proteins, and bone (Stoelting: Pharmacology and Physiology in Anesthetic Practice, ed 4, pp 794–799; Miller: Basics of Anesthesia, ed 6. pp 335–336).
136. A decrease in pH of 0.1 unit will result in A. A decrease in serum potassium concentration [K+] of 0.6 mEq/L B. A decrease in [K+] of 1.2 mEq/L C. An increase in [K+] of 0.6 mEq/L D. An increase in [K+] of 1.2 mEq/L
136. (C) Cardiac dysrhythmias are a common complication associated with acid-base abnormalities. The etiology of these dysrhythmias is related partly to the effects of pH on myocardial potassium homeostasis. As a general rule, plasma K+ increases approximately 0.6 for each 0.1 decrease in pH (Butterworth: Morgan & Mikhail’s Clinical Anesthesia, ed 5, p 1149).
137. An increase in [HCO3 −] of 10 mEq/L will result in an increase in pH of A. 0.10 pH unit B. 0.15 pH unit C. 0.20 pH unit D. 0.25 pH unit
137. (B) Several guidelines can be used in the initial interpretation of ABGs that will permit rapid recognition of the type of acid-base disturbance. These guidelines are as follows: (1) a 1 mm Hg change in Paco2 above or below 40 mm Hg results in a 0.008 unit change in the pH in the opposite direction; (2) the Paco2 will decrease by about 1 mm Hg for every 1 mEq/L reduction in [HCO3 −] below 24 mEq/L; (3) a change in [HCO3 −] of 10 mEq/L from 24 mEq/L will result in a change in pH of approximately 0.15 pH unit in the same direction (Butterworth: Morgan & Mikhail’s Clinical Anesthesia, ed 5, pp 1146, 1156–1157; Miller: Basics of Anesthesia, ed 6, pp 342–343). Respiratory Physiology and Critical Care Medicine 45
138. A 28-year-old, 70-kg woman with ulcerative colitis is receiving a general anesthetic for a colon resection and ileostomy. The patient’s lungs are mechanically ventilated with the following parameters: ˙V E 5000 mL and respiratory rate 10 breaths/min. Assuming no change in ˙V E, how would ˙V A change if the respiratory rate were increased from 10 to 20 breaths/min? A. Increase by 500 mL B. Increase by 1000 mL C. Decrease by 750 mL D. Decrease by 1500 mL
138. (D) A patient with a Vd of 150 mL and a Va of 350 mL (assuming a normal Vt of 500 mL) will have a Vd minute ventilation ( ˙V D) of 1500 mL and a Va minute ventilation (˙V A) of 3500 mL ( ˙V E of 5000 mL) at a respiratory rate of 10 breaths/min. If the respiratory rate is doubled but ˙V E remains unchanged, then the ˙V D would double to 3000 mL and there would be an increase in ˙V D of 1500 mL and a decrease in ˙V A of 1500 mL. Also see explanation to Questions 103 and 134 (Barash: Clinical Anesthesia, ed 7, pp 275–277; West: Respiratory Physiology, ed 9, pp 16–17; Miller: Miller’s Anesthesia, ed 8, pp 446–447).
139. Each of the following will shift the oxyhemoglobin dissociation curve to the right EXCEPT A. Volatile anesthetics B. Decreased Pao2 C. Decreased pH D. Increased temperature
139. (B) In addition to the items listed in this question, other factors that shift the oxyhemoglobin dissociation curve to the right include pregnancy and all abnormal hemoglobins such as hemoglobin S (sickle cell hemoglobin). For reasons unknown, volatile anesthetics increase the P50 of adult hemoglobin by 2 to 3.5 mm Hg. A rightward shift of the oxyhemoglobin dissociation curve will decrease the transfer of O2 from alveoli to hemoglobin and improve release of O2 from hemoglobin to peripheral tissues. Also see explanation to Question 109 (Miller: Basics of Anesthesia, ed 6, p 56; West: Respiratory Physiology, ed 9, pp 79–82).
140. The half-life of carboxyhemoglobin in a patient breathing 100% O2 is A. 5 minutes B. 1 hour C. 2 hours D. 4 hours
140. (B) The most frequent immediate cause of death from fires is carbon monoxide toxicity. Carbon monoxide is a colorless, odorless gas that exerts its adverse effects by decreasing O2 delivery to peripheral tissues. This is accomplished by two mechanisms. First, because the affinity of carbon monoxide for the O2 binding sites on hemoglobin is more than 200 times that of O2, O2 is readily displaced from hemoglobin. Thus, O2 content is reduced. Second, carbon monoxide causes a leftward shift of the oxyhemoglobin dissociation curve, which increases the affinity of hemoglobin for O2 at peripheral tissues. Treatment of carbon monoxide toxicity is administration of 100% O2. Supplemental oxygen decreases the half-time of carboxyhemoglobin from 4 to 6 hours with room air to about 1 hour with 100% oxygen. Breathing 100% oxygen at 3 atm in a hyperbaric chamber reduces the half-time even more to 15 to 30 minutes. See also explanations for Questions 113 and 129 (Barash: Clinical Anesthesia, ed 7, pp 1515–1516; Hines: Stoelting’s Anesthesia and Co-Existing Disease, ed 6, pp 554–555; Miller: Miller’s Anesthesia, ed 8, pp 2679–2680).
141. A disadvantage of using propofol for prolonged sedation (days) of intubated patients in the ICU is potential A. Acidosis B. Tachyphylaxis C. Hyperglycemia D. Bradycardia
141. (A) Propofol infusion syndrome is a rare condition associated with prolonged (greater than 48 hour) administration of propofol at a dose of 5 mg/kg/hr (83 μg/kg/min) or higher. This syndrome was first described in children but later observed in critically ill adults as well. It is manifested by cardiomyopathy with acute cardiac failure, metabolic acidosis, skeletal muscle myopathy, hepatomegaly, hyperkalemia, and lipidemia. It is thought to be related to a failure of free fatty acid transport into the mitochondria and failure of the mitochondrial respiratory chain. Bradycardia can be a late sign with this syndrome and heralds a poor prognosis (Miller: Miller’s Anesthesia, ed 8, p 831).
142. A 17-year-old type 1 diabetic with history of renal failure is in the preoperative holding area awaiting an operation for acute appendicitis. Arterial blood gases are obtained with the following results: Pao2 88 mm Hg, Paco2 32 mm Hg, pH 7.2, [HCO3 −] 12, [Cl−] 115 mEq/L, [Na+] 138 mEq/L, and glucose 251 mg/dL. The most likely cause of this patient’s acidosis is A. Renal tubular acidosis B. Lactic acidosis C. Diabetic ketoacidosis D. Aspirin overdose
142. (A) Calculating the anion gap (i.e., the unmeasured anions in the plasma) is helpful in determining the cause of a metabolic acidosis. Anion gap = [Na+] − ([Cl−] + [HCO3 −]) and is normally 10 to 12 nmol/L. In this case the anion gap = 138 − (115 + 12) = 11, a normal anion gap. Causes of a high anion gap metabolic acidosis include lactic acidosis, ketoacidosis, acute and chronic renal failure, and toxins (e.g., salicylates, ethylene glycol, methanol). Nonanion gap metabolic acidosis include renal tubular acidosis, expansion acidosis (e.g., rapid saline infusion), gastrointestinal (GI) bicarbonate loss (e.g., diarrhea, small bowel drainage), drug-induced hyperkalemia, and acid loads (e.g., ammonium chloride, hyperalimentation). Vomiting and nasogastric drainage are some of the many causes of metabolic alkalosis (Longo: Harrison’s Principles of Internal Medicine, ed 18, pp 365–369; Miller: Basics of Anesthesia, ed 6, pp 340–342).
143. Methods to decrease the incidence of central venous catheter infections include all of the following EXCEPT A. Changing the central catheter every 3 to 4 days over a guidewire B. Using minocycline/rifampin impregnated catheters over chlorhexidine/silver sulfadiazine impregnated catheters for suspected long-term use C. Using the subclavian over the internal jugular route for access D. Using a single lumen over a multilumen catheter
143. (A) Bloodstream infectious complications with central venous catheters are the most common late complication seen with central catheters (>5%). Current Centers for Disease Control and Prevention (CDC) guidelines do not recommend replacing central venous catheters. All the other statements are true. In addition, evidence is suggesting that the use of ultrasound may decrease the time needed to place catheters and the number of skin punctures needed for central vein access and may also decrease infections (Miller: Miller’s Anesthesia, ed 8, p 1367; O’Grady et al: Guidelines for the prevention of intravascular catheter-related infections. Clin Infect Dis 52(9): e164–e166, 2011). 46 Part 1 Basic Sciences
144. Signs of Sarin nerve gas poisoning include all of the following EXCEPT A. Diarrhea B. Urination C. Mydriasis D. Lacrimation
144. (C) Sarin (also called GB), like GA (Tabun), GD (Soman), GF, VR, and VX, is a clear liquid organophosphate that vaporizes at room temperatures. These chemical nerve gases mainly bind with acetylcholinesterase and produce clinical signs of excessive parasympathetic activity. The term DUMBELS—Diarrhea, Urination, Miosis, Bronchorrhea and bronchoconstriction, Emesis, Lacrimation, and Salivation—can help you remember several of the signs. Note the eye signs are pupillary constriction (miosis) and not pupillary dilation (mydriasis). Other signs relate to the cardiovascular system and include bradycardia, prolonged QT interval, and ventricular dysrhythmias. These chemicals also affect the GABA and NMDA receptors and may also cause central nervous system (CNS) excitation (i.e., convulsions) (Barash: Clinical Anesthesia, ed 7, pp 1540–1541; Miller: Miller’s Anesthesia, ed 8, p 2496).
145. Which of the following conditions would be associated with the LEAST risk of venous air embolism during removal of a central line? A. Spontaneous breathing, head up B. Spontaneous breathing, flat C. Spontaneous breathing, Trendelenburg D. Mechanical ventilation, Trendelenburg
145. (D) Venous air embolism occurs when air enters the venous system through an incised or cannulated vein. When cannulating or decannulating central veins, it is important to keep a positive venous-to-atmospheric pressure gradient. This is usually accomplished by placing the site below the level of the heart (i.e., Trendelenburg position). In addition, under mechanical ventilation or when the spontaneously breathing patient exhales or performs a Valsalva maneuver, the venous-to-atmospheric pressure is greater than if a spontaneously breathing patient inhales, a time when the venous pressure may be less than atmospheric pressure (Lobato: Complications in Anesthesiology, pp 198–200; Butterworth: Morgan & Mikhail’s Clinical Anesthesia, ed 5, p 101; Marino’s The ICU Book, ed 4, pp 32–33).
146. Which of the following adverse effects is NOT attributable to respiratory or metabolic acidosis? A. Increased intracranial pressure B. Vasoconstriction C. Increased pulmonary vascular resistance D. Increased serum potassium concentration 34 Part 1 Basic Sciences
146. (B) Adverse physiologic effects of respiratory or metabolic acidosis include CNS depression and increased intracranial pressure (ICP), cardiovascular system depression (partially offset by increased secretion of catecholamines and elevated [Ca++]), cardiac dysrhythmias, vasodilation, hypovolemia (which is a result of decreased precapillary and increased postcapillary sphincter tone), pulmonary hypertension, and hyperkalemia (Butterworth: Morgan & Mikhail’s Clinical Anesthesia, ed 5, pp 1148–1149; Miller: Basics of Anesthesia, ed 6, p 339).
147. Which of the following maneuvers is LEAST likely to raise arterial saturation in a patient in whom the endotracheal tube (ETT) is seated in the right mainstem bronchus? The patient has normal lung function. A. Inflating the pulmonary artery catheter balloon (in the left pulmonary artery) B. Raising hemoglobin from 8 to 12 mg/dL C. Raising Fio2 from 0.8 to 1.0 D. Increasing cardiac output from 2 to 5 L/min
147. (C) Withdrawing the tube into the trachea obviously would improve arterial saturation and is the treatment of choice for inadvertent mainstem intubation. Short of pulling the ETT back, all other successful options address ways of improving arterial oxygenation during one-lung ventilation. In essence, any maneuver that improves the saturation of the venous blood will also improve the saturation of arterial blood (in this question). Normal pulmonary circulation is in series with the systemic circulation. Blood exiting the lungs is nearly 100% oxygenated regardless of the saturation of the venous blood when it exits the right ventricle and enters the lungs via the pulmonary artery. In one-lung ventilation, deliberate or accidental, blood exiting the ventilated side of the lungs (the right side in this question) is also essentially fully saturated, but it mixes with nonoxygenated blood. The nonoxygenated blood has effectively bypassed the lungs by passing through an area that is perfused but not ventilated, that is, a shunt. When the blood from the ventilated lung (nearly 100% oxygenated) mixes with the shunted blood, a mixture will be formed that has saturation less than 100%, but higher than the mixed venous O2 saturation. SvO2 = SaO2 − ˙V O2/ ˙Q × Hgb where Svo2 = mixed venous hemoglobin saturation and Sao2 = arterial oxygen saturation O2 content = 1.39 × [Hgb] × SaO2 + (0.003 × PaO2) The exact saturation of the arterial blood in this question depends on the ratio of blood exiting the right lung versus that exiting the left lung. Fortunately, during one-lung ventilation, the nonventilated lung collapses and in so doing raises its resistance to blood flow. This results in preferentially directing blood to the right ventilated lung. A second factor to consider is how well-saturated the shunted blood is. “Red” blood from the right lung mixes with “blue” blood from the left lung to give a mixture of partially saturated blood. The saturation of the shunted “blue” blood depends on the hemoglobin concentration and cardiac output. From the first equation above you can see that raising either of these would improve the mixed venous oxygen saturation and ultimately the arterial saturation during one-lung ventilation. Inflating the pulmonary artery catheter balloon located in the nonventilated (left) lung would also improve arterial saturation by limiting blood flow to the left lung. Raising the Fio2 from 80% to 100% will do little if anything to improve arterial saturation because the blood exiting the “working” lung is already fully saturated. The small rise in Pao2 that would result from an increase in Fio2, once multiplied by 0.003 (see the second equation above), would be a very small and insignificant number. In other words, raising Fio2 does not improve arterial saturation in the presence of a shunt (Miller: Miller’s Anesthesia, ed 8, p 1386; Miller: Basics of Anesthesia, ed 6, pp 444–445, 636). Respiratory Physiology and Critical Care Medicine 47
148. A 100-kg man is 24 hours status post four-vessel coronary artery bypass graft. Which of the following pulmonary parameters would be compatible with successful extubation in this patient? A. Vital capacity 2.5 L B. Paco2 44 mm Hg C. Maximum inspiratory pressure –38 cm H2O D. All of the above
148. (D) The decision to stop mechanical support of the lungs is based on a variety of factors that can be measured. Guidelines suggesting that cessation of mechanical inflation of the lungs is likely to be successful include a vital capacity greater than 15 mL/kg, arterial Pao2 greater than 60 mm Hg (Fio2 < 0.5), alveolar-arterial (A–a) gradient less than 350 mm Hg (Fio2 = 1.0), arterial pH greater than 7.3, Paco2 less than 50 mm Hg, dead space/tidal volume ratio less than 0.6, and maximum inspiratory pressure of at least −20 cm H2O. In addition to these guidelines, the patient should be hemodynamically stable, conscious, oriented, and in good nutritional status (Butterworth: Morgan & Mikhail’s Clinical Anesthesia, ed 5, pp 1288, 1297; Miller: Basics of Anesthesia, ed 6, p 667).
149. Which of the following can cause a rightward shift of the oxyhemoglobin dissociation curve? A. Methemoglobinemia B. Carboxyhemoglobinemia C. Hypothermia D. Pregnancy
149. (D) A shift to the left in the oxyhemoglobin dissociation curve occurs with fetal hemoglobin, alkalosis, hypothermia, carboxyhemoglobin, methemoglobin, and decreased levels of 2,3-DPG. Storage of blood lowers 2,3-DPG levels in acid-citrate-dextrose stored blood, but minimal changes are seen in 2,3-DPG with citrate-dextrose-stored blood. A shift to the right occurs with acidosis, hyperthermia, increased levels of 2,3-DPG, inhaled anesthetics, and pregnancy (Butterworth: Morgan & Mikhail’s Clinical Anesthesia, ed 5, pp 516–517; Hines: Stoelting’s Anesthesia and Co-Existing Disease, ed 6, p 415).
150. A 24-year-old man is brought to the operating room 1 hour after a motor vehicle accident. He has C7 spinal cord transection and ruptured spleen. Regarding his neurologic injury, anesthetic concerns include A. Risk of hyperkalemia with succinylcholine administration B. Risk of autonomic hyper-reflexia with urinary catheter insertion C. Increased risk of hypothermia D. All of the above
150. (C) With acute spinal cord injuries the major anesthetic concerns are airway management and management of hemodynamic perturbations associated with interruption of the sympathetic nervous system below the level of the transection. Hyperkalemia in response to succinylcholine does not occur until at least 24 hours after the injury. Autonomic hyper-reflexia is not a concern in the acute management of patients with spinal cord injuries. There is no evidence that awake intubation (fiberoptic) is superior to direct laryngoscopy as long as in-line traction is held in both cases. These patients are more susceptible to hypothermia compared with patients without spinal cord injuries because they lack thermoregulation below the level of the cord injury (Hines: Stoelting’s Anesthesia and Co-Existing Disease, ed 6, pp 255–258).
151. After sustaining traumatic brain injury, a 37-year-old patient in the ICU develops polyuria and a plasma sodium concentration of 159 mEq/L. What pathologic condition is associated with these clinical findings? A. Syndrome of inappropriate antidiuretic hormone (SIADH) B. Diabetes mellitus C. Diabetes insipidus D. Cerebral salt wasting syndrome
151. (C) Polyuria of neurogenic (rather than nephrogenic) diabetes insipidus is caused by diminished or absent antidiuretic hormone (ADH) synthesis or release following injury to the hypothalamus, pituitary stalk, or posterior pituitary gland. Hemoconcentration resulting in hypernatremia often results. In contrast, SIADH is associated with excessive amounts of ADH, which in turn causes hyponatremia. Cerebral salt wasting syndrome results from release of brain natriuretic peptide in subarachnoid hemorrhage patients. The resulting natriuresis-mediated electrolyte perturbation is hyponatremia. Diabetes mellitus and spinal shock do not cause hypernatremia (Longo: Harrison’s Principles of Internal Medicine, ed 18, pp 349–351; Butterworth: Morgan & Mikhail’s Clinical Anesthesia, ed 5, p 1115).
152. Which of the following drugs is the best choice for treating hypotension in the setting of severe acidemia? A. Norepinephrine B. Epinephrine C. Phenylephrine D. Vasopressin
152. (D) Vasopressin, also known as antidiuretic hormone, is a naturally occurring peptide synthesized in the hypothalamus and stored in the posterior pituitary. It is used clinically to treat diabetes insipidus, and in the ICU it is used to treat hypotension. Patients with severe sepsis and septic shock have a relative deficiency of vasopressin, and these patients may be sensitive to vasopressin. Vasopressin interacts with a different receptor and, unlike the catecholamines, it is effective even in the presence of acidemia (Miller: Basics of Anesthesia, ed 6, p 676).
153. The end-tidal CO2 measured by an infrared spectrometer is 35 mm Hg. An arterial blood gas sample drawn at exactly the same moment is 45 mm Hg. Which of the following is the LEAST plausible explanation for this? A. Morbid obesity B. Pulmonary embolism C. Intrapulmonary shunt D. Chronic obstructive pulmonary disease (COPD)
153. (C) Confusion may exist between the concepts of shunt versus dead space. Both of these are forms of ˙ V/ ˙Q mismatch. With shunts, there is a gradient between the alveolar and the arterial oxygen partial pressures. Alveolar partial pressure (PA) is calculated from the alveolar gas equation. The Paco2 with shunt is compensated and is usually normal even in the presence of a significant ˙ V/ ˙Q mismatch. Dead space refers to the portion of a breath that does not reach perfused alveoli. In pathologic conditions, such as COPD, morbid obesity, and pulmonary embolism, dead space is increased because air passes into alveoli that are ventilated but not perfused. This air does not participate in gas exchange and simply exits these unperfused alveoli and “dilutes” the carbon dioxide exiting the lungs from the perfused alveoli. Under these circumstances the mixed expired CO2 measured with capnometry will be less than the actual arterial CO2 (Miller: Miller’s Anesthesia, ed 8, pp 444–445; Miller: Basics of Anesthesia, ed 6, pp 58–61).
154. A transfusion-related acute lung injury (TRALI) reaction is suspected in a 48-year-old man in the ICU after a 10-hour operation for scoliosis during which multiple units of blood and factors were administered. Which of the following items is inconsistent with the diagnosis of a TRALI reaction? A. Fever B. Alveolar-to-arterial (A–a) oxygen gradient of 25 mm Hg C. Acute rise in neutrophil count after onset of symptoms D. Bilateral pulmonary infiltrates
154. (C) TRALI reactions are a serious complication of transfusing any product containing plasma, that is, fresh frozen plasma, whole blood, packed red blood cells, platelets, or factor concretes derived from human blood. The clinical diagnosis is made 1 to 2 hours after transfusion (but may occur up to 6 hours later in the ICU). The key features include wide A–a gradient, noncardiogenic pulmonary edema, and leukopenia (not leukocytosis) secondary to sequestration in the lungs. TRALI reactions are one of the leading causes of transfusion-related mortality (Miller: Basics of Anesthesia, ed 6, p 637). 48 Part 1 Basic Sciences
155. If a central line located in the superior vena cava (SVC) is withdrawn such that the tip of the catheter is just proximal to the SVC, it would be located in which vessel? A. Subclavian vein B. Brachiocephalic vein C. Cephalic vein D. Internal jugular vein
155. (B) The right internal jugular vein and the right subclavian vein form the right brachiocephalic vein; similarly, the left internal jugular vein and the left subclavian vein form the left brachiocephalic vein. These two brachiocephalic veins form the SVC (Netter: Atlas of Human Anatomy, ed 5, plates 70, 192, 200, 205).
156. The time course of anticoagulation therapy is variable after different percutaneous coronary interventions (PCIs). Arrange the interventions in order starting with the one requiring the shortest course of aspirin and clopidogrel (Plavix) therapy to the one requiring the longest course. A. Bare-metal stent, percutaneous transluminal coronary angioplasty (PTCA), drug-eluting stent B. Drug-eluting stent, bare-metal stent, PTCA C. PTCA, drug-eluting stent, bare-metal stent D. PTCA, bare-metal stent, drug-eluting stent
156. (D) Patients who have undergone a PCI are placed on a course of a thienopyridine (ticlopidine or clopidogrel) and aspirin. The thienopyridine is used for at least 2 weeks after PTCA, 1 month after a bare-metal stent is placed, and 1 year after a drug-eluting stent is placed. Aspirin is continued for a longer period of time. This is to decrease the chance of thrombosis of the treated coronary artery (ACC/AHA 2007 Guidelines on Perioperative Cardiovascular Evaluation and Care for Noncardiac Surgery: Executive Summary. Anesth Analg 106:698–701, 2008).
157. Basic Life Support Working Group’s single rescuer cardiac compression-ventilation ratio for infant, child, and adult victims (excluding newborns) is A. 10:1 B. 15:2 C. 30:2 D. 60:2
157. (C) The universal compression-ventilation ratio for infant, child, and adult victims (excluding newborns) is 30 chest compressions to two breath cycles (5 cycles in 2 minutes). Once an advanced airway is in place, two rescuers no longer deliver “cycles,” but rather compressions at a rate of 100/min and ventilation is 8 to 10/min. For newborns the ratio is 3:1 (90 compressions and 30 breaths/min) (2010 AHA Guidelines for CPR and Emergency Cardiovascular Care: Circulation 122 (Suppl 3) S688, S692–S693, S913).
158. Which of the features below is suggestive of weaponized anthrax exposure as opposed to a common flulike viral illness? A. Widened mediastinum B. Fever, chills, myalgia C. Severe cough D. Pharyngitis Respiratory Physiology and Critical Care Medicine 35
158. (A) After an incubation period (commonly within 2 weeks), inhalational anthrax symptoms initially look like viral flu (fever, chills, myalgia, and a nonproductive cough). Although leukocytosis is common with anthrax and rare with viral flu, white blood cell (WBC) counts initially may be normal at the time the patient presents. After a short while, the patient suddenly appears critically ill, and without treatment, death can occur within a few days. Substernal chest pain, hypoxemia, cyanosis, dyspnea, abdominal pain, and sepsis syndrome are common with inhaled anthrax but rare with viral flu. After the anthrax spores are inhaled, macrophages phagocytize the spores and transport them to mediastinal lymph nodes where the spores germinate, producing enlarged nodes and a widened mediastinum on the chest x-ray film. A widened mediastinum is not seen with viral flu. Pharyngitis is common with viral flu and occasionally is seen with anthrax (Miller: Basics of Anesthesia, ed 6, pp 691–693; Longo: Harrison’s Principles of Internal Medicine, ed 18, pp 1769–1771).
159. Which of the following factors could not explain a Pao2 of 48 mm Hg in a patient breathing a mixture of nitrous oxide and oxygen? A. Hypoxic gas mixture B. Eisenmenger syndrome C. Profound anemia D. Hypercarbia
159. (C) To answer this question it is helpful to review the alveolar gas equation: PAO2 = FIO2 (Pb − PH2O) − PaCO2/R Pao2 = partial pressure of oxygen in the alveolar gas; Fio2 = fraction of inhaled oxygen; Pb = barometric pressure; Ph2o = vapor pressure at 100% saturation (47 mm Hg at 37° C); Paco2 = partial pressure of CO2 in the alveolar gas; R = respiratory quotient. Any factor that lowers Pao2 (below 100 mm Hg or so) will also lower Pao2. Hypoxic gas mixture lowers Fio2, hence Pao2. Hypercarbia makes the term Paco2/R larger and, therefore, reduces Pao2. Eisenmenger syndrome results in a larger shunt fraction and lower Pao2 on that basis (see explanation to Question 147). In normally functioning lungs, anemia has a minimal impact on Pao2 because physiologic shunt is normally only 2% to 5% of cardiac output (Barash: Clinical Anesthesia, ed 6, pp 277–278).
160. During a left hepatectomy under general isoflurane anesthesia, arterial blood gases are: O2 138, CO2 39, pH 7.38, saturation 99%. At the same time, CO2 on infrared spectrometer is 26 mm Hg. The most plausible explanation for the difference between CO2 measured with infrared spectrometer versus arterial blood gas gradient is A. Mainstem intubation B. Atelectasis C. Shunting through thebesian veins D. Hypovolemia
160. (D) The difference between the Paco2 and the CO2 value measured by the infrared spectrometer is a function of the patient’s physiologic dead space. Physiologic dead space is equal to anatomic dead space plus alveolar dead space. Anatomic dead space is roughly 1 mL/lb of body weight. Because anatomic dead space is relatively “fixed,” changes in physiologic dead space are mainly attributable to changes in alveolar dead space. Alveoli that are ventilated, but not perfused, add to alveolar dead space. In essence, air goes into these alveoli but does not participate in gas exchanges and merely exits the alveoli upon exhalation. Ventilation of dead space serves no useful purpose but does result in “dilution” of the exhaled CO2, thus explaining why the CO2 seen on the infrared spectrometer can be substantially lower than that obtained from arterial blood gas analysis. Several factors increase dead space, including lung diseases such as COPD, cystic fibrosis, and pulmonary emboli. In addition, decreased alveolar perfusion from low cardiac output or hypovolemia may also contribute to increased dead space. Mainstem intubation, atelectasis, shunting through thebesian veins, and ablation of hypoxic pulmonary vasoconstriction by isoflurane are various causes of shunting. Shunting is also a mismatch between ventilation and perfusion, but, in contrast to ˙ V/ ˙Q mismatch from dead space ventilation, shunting results in a normal or nearly normal Paco2 but a larger-than-expected A–a O2 gradient. The only choice in this question that would explain an increase in dead space ventilation is hypovolemia (Barash: Clinical Anesthesia, ed 7, pp 276–277; Miller: Basics of Anesthesia, ed 6, pp 328–329). Respiratory Physiology and Critical Care Medicine 49
161. Under which set of circumstances would energy expenditure per day be the greatest? A. Sepsis with fever B. 60% burn C. Multiple fractures D. 1 hour status post liver transplantation
161. (B) The normal human’s resting energy expenditure as well as the postoperative state is about 1800 kcal/24 hr. With starvation (20 days), energy expenditure decreases to about 1080 kcal/day (60% of normal). Patients who have sustained multiple fractures (2160 kcal/day or 120% of normal), major sepsis (2520 kcal/day or 140% of normal), and burns have increased energy expenditures. The energy expenditure in a patient with a major burn also depends on the temperature of the room. The highest energy expenditure is at a room temperature of 25° C (3819 kcal/day or 212% of normal) and is lower at 33° C (3342 kcal/day or 185% of normal) and at 21° C (3600 kcal/day or 200% of normal) (Miller: Miller’s Anesthesia, ed 8, pp 3136–3138).
162. Select the FALSE statement regarding amiodarone (Cordarone). A. It is shown to decrease mortality after myocardial infarction B. It is indicated for ventricular tachycardia and fibrillation refractory to electrical defibrillation C. Adverse effects include pulmonary fibrosis and thyroid dysfunction D. It is useful in treatment of torsades de pointes
162. (D) Amiodarone is useful in the treatment of a variety of supraventricular and ventricular cardiac arrhythmias. For the treatment of ventricular tachycardia or fibrillation that is refractory to electrical defibrillation, the recommended dose is 300 mg IV. Similar to β-blockers, amiodarone decreases mortality after myocardial infarctions. About 5% to 15% of treated patients develop pulmonary toxicity (especially when doses are >400 mg/day, or underlying lung disease is present) and 2% to 4% develop thyroid dysfunction (amiodarone is a structural analog of thyroid hormone). It has a prolonged elimination halftime of 29 hours and a large volume of distribution. Because it prolongs the QTc interval, it may lead to the production of ventricular tachydysrhythmias and thus is not useful in treating torsades de pointes (Brunton: Goodman & Gilman’s The Pharmacological Basis of Therapeutics, ed 12, pp 834, 837).
163. A 58-year-old woman is awaiting orthotopic liver transplantation for primary biliary cirrhosis in the ICU. An oximetric pulmonary artery catheter is placed and an Svo2 of 90% is measured. Which of the following blood pressure interventions is the LEAST appropriate for treatment of hypotension in this patient? A. Milrinone B. Norepinephrine C. Vasopressin D. Phenylephrine
163. (A) Patients with cirrhosis have hyperdynamic circulations as noted here with the elevated Svo2 of 90%. The cardiac output is usually increased, peripheral vascular resistance is low, intravascular volume is increased, and arteriovenous shunts are present. Hypotension is common. Milrinone is a positive inotrope with vasodilating properties, something this patient does not need. If a treatment for hypotension is needed, drugs with α-agonist properties may be helpful. In addition, vasopressin is also a good choice because it increases systemic vascular resistance (SVR) but does not increase the already high cardiac output (Butterworth: Morgan & Mikhail’s Clinical Anesthesia, ed 5, p 714; Miller: Basics of Anesthesia, ed 6, p 457).
164. Each of the following measures is part of the Surgical Care Improvement Project (SCIP) with the goal of preventing perioperative infection EXCEPT A. Normothermia B. Oxygen saturation above 95% in the OR C. Appropriate hair removal preoperatively D. Removal of urinary catheter by postoperative day 2
164. (B) For many years hand hygiene, wearing surgical masks, and sterile techniques have been used to decrease surgical site infections (SSIs). The CDC has also recommended that patients undergo preoperative showering using antiseptic skin wash products to reduce skin bacteria despite no clear studies showing a direct independent relationship decreasing SSIs. In 2004, the National Surgical Infection Prevention Project gave guidelines for antibiotic prophylaxis, whenever there is more than minimal risk of infection. Prophylactic antibiotics should be administered within 1 hour before surgical incision in appropriately selected patients and discontinued within 24 hours after the surgical end time or 48 hours for cardiac patients. More recently, using evidence-based research, the SCIP has suggested several additional measures to decrease the incidence of surgical site infections, including appropriate hair removal at the surgical site (e.g., using depilatory cream or hair clippers rather than razors), glycemic control in cardiac surgical patients (e.g., serum glucose <200 mg/dL the morning after surgery), removal of urinary catheters (e.g., removal on postoperative day 1 or 2 and reassessment of the need every day thereafter), and maintenance of perioperative normothermia (e.g., core temperature should be 36° C on arrival in the PACU). Interestingly, surgical time was not mentioned (Barash: Clinical Anesthesia, ed 7, pp 304–314; Miller: Basics of Anesthesia, ed 6, pp 746–752; Miller: Miller’s Anesthesia, ed 8, pp 100–101, 1104 ).
165. A 55-year-old man with polycystic liver disease undergoes an 8-hour right hepatectomy. The patient receives 5 units of packed red cells, 1000 mL albumin, and 6 L normal saline. The patient is extubated and taken to a postanesthesia care unit (PACU) where ABGs are: Pao2 135, Paco2 44, pH 7.17, base deficit −11, [HCO3 −], 12, 97% saturation, [Cl−] 119, [Na+] 145, and [K+] 5.6. The most likely cause for this acidosis is A. Lactic acid B. Use of normal saline C. Diabetic ketoacidosis D. Polyethylene glycol from bowel prep
165. (B) This patient has a metabolic acidosis. Recall that anion gap = [Na+] − ([Cl−] + [HCO3 −]) and is normally 10 to 12 nmol/L. In this case the anion gap = 145 − (119 + 12) = 14, which is slightly above the normal anion gap range. In looking at this case, the acidosis is quite profound and would most likely be related to the rapid infusion of normal saline. Lactic acid, ketoacidosis, and ethylene glycol produce a high anion gap metabolic acidosis. Narcotics may produce respiratory but not metabolic acidosis. See also Question 142 (Longo: Harrison’s Principles of Internal Medicine, ed 18, pp 365–369; Butterworth: Morgan & Mikhail’s Clinical Anesthesia, ed 5, p 1165).
166. Which of the following is the LEAST appropriate use of noninvasive positive-pressure ventilation (NIPPV)? A. Acute respiratory distress syndrome (ARDS) B. COPD exacerbation C. Obstructive sleep apnea D. Multiple sclerosis exacerbation
166. (A) Noninvasive positive-pressure ventilation (NIPPV) refers to delivering positive-pressure ventilation to patients by way of a nasal mask, or full face mask, without the placement of an endotracheal or tracheostomy tube. This mode of therapy requires conscious and cooperative patients and does not protect the airway. NIPPV has been very useful in COPD patients and in immunosuppressed patients in acute respiratory failure. It most likely will fail (i.e., intubation would be needed) in patients with pneumonia and ARDS (Miller: Miller’s Anesthesia, ed 8, p 3068). 50 Part 1 Basic Sciences
167. A 68-year-old asthmatic drunk driver comes into the ER after being in a motor vehicle accident. After a difficult intubation, you fail to observe end-tidal CO2 on the monitor. Reasons for this include all of the following EXCEPT A. You intubated the esophagus by mistake B. You forgot to ventilate the patient C. The connection between the circuit and monitor has become disconnected D. The patient also has a pneumothorax, and high airway pressures are needed to adequately ventilate the patient
167. (D) Capnography has been a valuable monitor for the cardiac and pulmonary systems as well as checking the anesthetic equipment. Forgetting to ventilate the patient, intubating the esophagus, and having the sensing tube become disconnected from the monitor quickly will show no CO2 detected. Any significant reduction in lung perfusion (i.e., air embolism, decreased cardiac output, or decreased blood pressure) increases alveolar dead space and leads to a lowering of the detected CO2. A cardiac arrest where there is no blood flow to the lungs and hence no carbon dioxide going to the lungs would also result in no detectable CO2. As CPR is started, detectable CO2 would be a sign of lung perfusion and ventilation. A patient with a pneumothorax and high airway pressures would still give you CO2 readings (Butterworth: Morgan & Mikhail’s Clinical Anesthesia, ed 5, pp 125–127).
168. A 30-year-old woman has undergone a 2-hour abdominal surgical procedure and is sent to the ICU intubated for postoperative monitoring due to suspected sepsis. Three hours later, the ventilator malfunctions and the resident disconnects the patient from the ventilator and hand ventilates the patient with 100% oxygen. The patient has good bilateral breath sounds, the chest rises nicely, and moisture is seen in the ETT. Shortly thereafter, the patient’s heart rate slows to 30 beats/min and the blood pressure is 50 mm Hg systolic. The next intervention that should be done, in addition to chest compressions, is A. Administer atropine B. Start epinephrine C. Confirm ETT position D. Apply external pacemaker 36 Respiratory Physiology and Critical Care Medicine Answers, References, and Explanations
168. (C) Always confirm an adequate Airway and Breathing before treating a Cardiac rhythm (A, B before C). Having the ETT in proper position for several hours does not ensure that it remains in proper position. In this case, the ETT slipped out of the trachea and went into the esophagus. The only way you know the ETT is in the trachea is to see the tube passing between the vocal cords directly with a conventional laryngoscope or by putting a fiberoptic bronchoscope through the tube and seeing carina. Other forms of confirmation such as bilateral breath sounds, adequate chest rise, and moisture in the tube are helpful but could also be seen with an esophageal intubation. Getting a consistent and adequate end tidal CO2 on a monitor confirms some gas exchange, but in cases where blood does not get to the lungs, as in a cardiac arrest, CO2 cannot be removed from the lungs. The first part in the treatment of bradycardia is adequate ventilation with oxygen. After that the other choices may be indicated (Miller: Miller’s Anesthesia, ed 8, p 1654). 51
169. Which of the following muscle relaxants is eliminated the most by renal excretion? A. Pancuronium B. Vecuronium C. Atracurium D. Rocuronium
169. (A) The duration of action of neuromuscular blocking drugs is related to the dose administered, as well as how the drug is metabolized or handled in the body. Succinylcholine normally is rapidly metabolized by plasma cholinesterase and has an ultrashort duration of action. The intermediate-duration neuromuscular blockers atracurium and cisatracurium undergo chemical breakdown in the plasma (Hofmann elimination), as well as ester hydrolysis. Vecuronium and rocuronium also have intermediate duration of actions and undergo primarily hepatic metabolism and biliary excretion with limited renal excretion (10%-25%). Only the long-duration neuromuscular blocker pancuronium is primarily excreted in the urine (80%). In patients with renal failure, the duration of action of neuromuscular blockers is not prolonged with atracurium or cisatracurium; is slightly prolonged with vecuronium and rocuronium; and is markedly prolonged with d-tubocurarine, pancuronium, doxacurium, and pipecuronium. Of the long-duration drugs, 80% of pancuronium, 70% of doxacurium, and 70% of pipecuronium are renally excreted unchanged in the urine. d-tubocurarine has a little more liver excretion and a little less renal elimination compared with pancuronium (Miller: Miller’s Anesthesia, ed 8, pp 975–977). COMPARATIVE PHARMACOLOGY OF NONDEPOLARIZING NEUROMUSCULAR BLOCKING DRUGS Drug ED95 (mg/ kg) Onset to Maximum Twitch Depression (min) Duration to Return to ≥25%* Intubating Dose (mg/kg) Continuous Infusion (mg/kg/ min) Renal Excretion (% Unchanged) Hepatic Degradation (%) Biliary Excretion (% Unchanged) Hydrolysis in Plasma Pancuronium 0.07 3-5 60-90 0.1 80 10 5-10 No Vecuronium 0.05 3-5 20-35 0.08-0.1 1 15-25 20-30 40-75 No Rocuronium 0.3 1-2 20-35 0.6-1.2 10-25 10-20 50-70 No Atracurium 0.2 3-5 20-35 0.4-0.5 6-8 NS NS NA Enzymatic, spontaneous Cisatracurium 0.05 3-5 20-35 0.1 1-1.5 NS NS NS Spontaneous Mivacurium 0.08 2-3 12-20 0.25 5-6 NS NS NS Enzymatic NA, not applicable; NS, not significant. *Control twitch height (minutes). From Miller RD: Basics of Anesthesia, ed 6, Philadelphia, Saunders, 2011, p 151, Table 12-6.
170. All of the following conditions may develop when using propofol for prolonged sedation in the intensive care unit (ICU) EXCEPT A. Pancreatitis B. Hyperlipidemia C. Metabolic acidosis D. Adrenal suppression
170. (D) Pancreatitis has been reported in patients on long-term propofol infusions. Because of the high fat content of propofol solutions (propofol is insoluble in aqueous solutions and is marketed as an emulsion containing 10% soybean oil, 2.25% glycerol, and 1.2% purified egg phosphatide), patients on long-term infusion should be checked for hyperlipidemia and patients receiving TPN should have the Intralipid portion of the TPN reduced. Propofol infusion syndrome is commonly defined as an acute onset of metabolic acidosis associated with cardiac dysfunction (e.g., bradycardia or right bundle branch block), and one of the following: rhabdomyolysis, hypertriglyceridemia, enlarged liver, or renal failure. Propofol decreases myocardial contractility and reduces systemic vascular resistance but does not cause adrenal suppression. The latter is a feature of etomidate administration (Brunton: Goodman & Gilman’s The Pharmacological Basis of Therapeutics, ed 12, pp 536–537; Miller: Basics of Anesthesia, ed 6, p 671).
171. Which of the following β-adrenergic antagonists is a nonselective β1 and β2 blocker? A. Atenolol B. Nadolol C. Esmolol D. Metoprolol
171. (B) β-Adrenergic receptor antagonists are of three generations. First-generation antagonists are nonselective β1 and β2 receptor blockers and include nadolol (Corgard), propranolol (Inderal), sotalol (Betapace), and timolol (Blocadren, Timoptic). Second-generation antagonists are cardioselective β1 receptor blockPharmacology and Pharmacokinetics of Intravenous Drugs 63 ers and include acebutolol (Sectral), atenolol (Tenormin), bisoprolol (Zebeta), esmolol (Brevibloc), and metoprolol (Lopressor). Third-generation β-adrenergic antagonists (mixed antagonists) have nonselective β1 and β2 receptor blocking actions and have additional cardiovascular effects (α1-adrenergic antagonist) and include labetalol (Normodyne, Trandate) and carvedilol (Coreg). Carvedilol also has some antioxidant and anti-inflammatory effects as well. Many of these drugs have additional trade names (Brunton: Goodman & Gilman’s The Pharmacological Basis of Therapeutics, ed 12, pp 320–330; Hemmings; Pharmacology and Physiology for Anesthesia, pp 228–229; Miller: Miller’s Anesthesia, ed 8, pp 370–371).
172. A 78-year-old patient with Parkinson disease undergoes a cataract operation under general anesthesia. In the recovery room, the patient has two episodes of emesis and complains of severe nausea. Which of the following antiemetics would be the best choice for treatment of nausea in this patient? A. Droperidol B. Promethazine C. Ondansetron D. Metoclopramide
172. (C) Parkinson disease (paralysis agitans or shaking palsy) is a degenerative CNS disease. It is caused by greater than 80% destruction of dopaminergic neurons in the substantia nigra of the basal ganglia. Dopamine acts as a neurotransmitter to inhibit the rate of firing of neurons that control the extrapyramidal motor system. The imbalance of neurotransmitters that results leads to the extrapyramidal symptoms of this disease. Symptoms include bradykinesia (slowness of movement), muscular rigidity, resting tremor (that lessens with voluntary movement), and impaired balance. Drugs that can produce extrapyramidal effects, such as the dopamine antagonists droperidol, promethazine, and thiethylperazine, as well as the dopamine and serotonin antagonist metoclopramide, are contraindicated. Ondansetron, a 5-hydroxytryptamine type 3 (5-HT3) receptor antagonist, is the preferred drug to treat nausea and vomiting for this patient (Barash: Clinical Anesthesia, ed 7 , p 621; Hines: Stoelting’s Anesthesia and Co-Existing Disease, ed 6, pp 646–647).
173. Which of the following diseases is associated with increased resistance to neuromuscular blockade with succinylcholine? A. Myasthenia gravis B. Myasthenic syndrome C. Huntington chorea D. Polymyositis
173. (A) In order for depolarizing muscle relaxants such as succinylcholine to work, the drug must interact with the receptor at the myoneural junction. Patients with myasthenia gravis have fewer acetylcholine receptors on the muscle and are more resistant to succinylcholine but are much more sensitive to nondepolarizing muscle relaxants. Patients with myasthenic syndrome (Eaton-Lambert syndrome) have a decreased release of acetylcholine at the myoneural junction; however, the number of receptors is normal. Patients with myasthenic syndrome are more sensitive to both depolarizing and nondepolarizing muscle relaxants. Huntington chorea is a degenerative CNS disease that is associated with decreased plasma cholinesterase activity, and prolonged responses to succinylcholine use have been seen. The response to depolarizing and nondepolarizing muscle relaxants appears to be unchanged in patients with polymyositis. Succinylcholine is contraindicated in patients with Duchenne muscular dystrophy because of the risks of rhabdomyolysis, hyperkalemia, and cardiac arrest. Nondepolarizing muscle relaxants have a normal response in patients with Duchenne muscular dystrophy, although some patients have prominent coexisting skeletal muscle weakness (Fleisher: Anesthesia and Uncommon Diseases, ed 6, pp 264–265, 313–316, 574; Hines: Stoelting’s Anesthesia and Co-Existing Disease, ed 6, pp 247, 444, 448–452).
174. Sedation with which of the following drugs is most likely to resemble normal sleep? A. Propofol B. Midazolam C. Dexmedetomidine D. Ketamine
174. (C) Sedation is commonly used in the ICU to prevent patient injury, decrease anxiety, reduce pain, reduce sympathetic stimulation, and help with ventilator dyssynchrony. Many different drugs have been used, including barbiturates, narcotics (e.g., fentanyl, morphine), benzodiazepines (e.g., midazolam, lorazepam), etomidate, ketamine, antipsychotics (e.g., haloperidol), propofol, and α2-adrenergic agonists (e.g., dexmedetomidine). Although deep sedation was commonly used, more recent evidence has suggested that patients tend to have fewer complications with light sedation and daily awakening (e.g., shorter duration of mechanical ventilation, less cardiovascular depression, and shorter ICU stays). The choice of drugs depends on the particular indications. Dexmedetomidine has several desirable effects, especially in the neurosurgical ICU, including sedation, analgesia, and little effect on respiratory drive. Its sedative properties resemble normal sleep in that the sedated patient can be easily aroused with stimulation and then rapidly fall back to sleep after stimulation ends. Dexmedetomidine does have some disadvantages, such as cost and U.S. Food and Drug Administration (FDA)-approved use for only 24 hours (Barash: Clinical Anesthesia, ed 7, pp 1584, 1599–1600; Miller: Basics of Anesthesia, ed 6, p 672).
175. Which of the following intravenous anesthetics is converted from a water-soluble to a lipid-soluble drug after exposure to the bloodstream? A. Propofol B. Midazolam C. Ketamine D. None of the above
175. (B) Diazepam (Valium) and lorazepam are water-insoluble benzodiazepines and are usually mixed with propylene glycol to become soluble solutions. These propylene glycol solutions are painful when injected. Midazolam has an imidazole ring that allows the drug to be water soluble in an acid pH (pH 3.5). When injected into the bloodstream, midazolam is exposed to the higher physiologic pH and the ring changes shape and the drug becomes lipid soluble. The lipid-soluble form readily crosses the blood-brain barrier to exert its pharmacologic effects. None of the other drugs change form with different pH (Hemmings: Pharmacology and Physiology for Anesthesia, pp 144–145). 64 Part 1 Basic Sciences
176. A 33-year-old, 70-kg patient is brought to the operating room for resection of an anterior pituitary prolactin-secreting tumor. Anesthesia is induced with sevoflurane, nitrous oxide, and oxygen. The patient is intubated and nitrous oxide is discontinued. Anesthesia is maintained with 1.2 minimum alveolar concentration (MAC) sevoflurane in oxygen. The surgeon plans to inject epinephrine into the nasal mucosa to minimize bleeding. What is the maximum volume of a 1:100,000 epinephrine solution that can be administered safely to this patient without producing ventricular arrhythmias? A. 55 mL B. 45 mL C. 35 mL D. 25 mL
176. (C) The amount of submucosally injected epinephrine required to produce ventricular cardiac dysrhythmias (i.e., three or more premature ventricular contractions during or after injection) varies with the volatile anesthetic administered. Patients under halothane anesthesia are particularly sensitive to ventricular arrhythmias, whereas patients with isoflurane, desflurane, and sevoflurane are less sensitive to epinephrine. Fifty percent of patients have ventricular arrhythmias when a dose of 2.1 μg/kg of epinephrine is administered submucosally into patients under halothane anesthesia. Ventricular arrhythmias do not seem to occur when a dose of up to 5 μg/kg of epinephrine is injected submucosally into patients under 1.2 MAC of sevoflurane or isoflurane in oxygen anesthesia. However, when the dose of epinephrine is increased to between 5 and 15 μg/kg, then about one third of patients will exhibit ventricular ectopy under sevoflurane or isoflurane anesthesia. Thus, using the 5 μg/kg maximum dose, a 70-kg patient could receive up to 350 μg of epinephrine (70 kg × 5 μg/kg) or 35 mL of this 1:100,000 solution (10 μg/mL) without ventricular arrhythmias (Johnston: A comparative interaction of epinephrine with enflurane, isoflurane and halothane in man. Anesth Analg 55:709–712, 1976; Navarro: Humans anesthetized with sevoflurane or isoflurane have similar arrhythmic response to epinephrine. Anesthesiology 80:545–549, 1994; Stoelting: Pharmacology and Physiology in Anesthetic Practice, ed 4, pp 54–56; Miller: Miller’s Anesthesia, ed 8, p 713).
177. Patients receiving antihypertensive therapy with propranolol are at increased risk for each of the following EXCEPT A. Blunted response to hypoglycemia B. Bronchoconstriction C. Rebound tachycardia after discontinuation D. Orthostatic hypotension
177. (D) β-Adrenergic receptor antagonists are effective in the treatment of essential hypertension and angina pectoris. They can be used to decrease mortality in patients suffering myocardial infarctions; to treat hyperthyroidism or hypertrophic obstructive cardiomyopathy; and to prevent migraine headaches. Although they are useful drugs, their use is limited by many side effects, which include bronchoconstriction, suppression of insulin secretion, blunting of the catecholamine response to hypoglycemia, excessive myocardial depression, atrioventricular heart block, accentuated increases in plasma concentrations of potassium with intravenous infusion of potassium chloride, fatigue, and rebound tachycardia associated with abrupt drug discontinuation. An important advantage of β-adrenergic receptor antagonists used in treating hypertension is the lack of orthostatic hypotension (Brunton: Goodman & Gilman’s The Pharmacological Basis of Therapeutics, ed 12, pp 320–324; Miller: Basics of Anesthesia, ed 6, pp 745–775; Miller: Miller’s Anesthesia, ed 8, pp 1217–1218).
178. Atropine causes each of the following EXCEPT A. Decreased gastric acid secretion B. Inhibition of salivary secretion C. Increased lower esophageal sphincter tone D. Mydriasis
178. (C) Anticholinergics are rarely given with premedication today unless a specific effect is needed (e.g., drying of the mouth before fiberoptic intubation, prevention of bradycardia, and, rarely, as a mild sedative). Side effects are many and include relaxation or a decrease of the lower esophageal sphincter tone that may make patients more likely to regurgitate gastric contents. Although these drugs can decrease gastric acid secretion and increase gastric pH, the pH effects are small and the dose needed to accomplish this is much higher than clinically used. The following table compares the effects of various anticholinergics (Hemmings, Pharmacology and Physiology for Anesthesia, pp 229–232; Miller: Basics of Anesthesia, ed 6, pp 75–76; Miller: Miller’s Anesthesia, ed 8, pp 377-378). COMPARATIVE EFFECTS OF ANTICHOLINERGICS ADMINISTERED INTRAMUSCULARLY AS PHARMACOLOGIC PREMEDICATION Effect Atropine Scopolamine Glycopyrrolate Antisialagogue effect + +++ ++ Sedative and amnesic effects + +++ 0 Increased gastric fluid pH 0 0 0/+ Central nervous system toxicity + ++ 0 Relaxation of lower esophageal sphincter ++ ++ ++ Mydriasis and cycloplegia + +++ 0 Heart rate ++ 0/+ + From Miller RD: Basics of Anesthesia, ed 6, Philadelphia, Saunders, 2011, p 76, Table 7-3.
179. Which of the following drugs is capable of crossing the blood-brain barrier? A. Neostigmine B. Pyridostigmine C. Edrophonium D. Physostigmine
179. (D) Neostigmine, pyridostigmine, edrophonium, and physostigmine are anticholinesterase drugs. Neostigmine, pyridostigmine, and edrophonium are quaternary ammonium compounds and do not pass the blood-brain barrier. However, physostigmine is a tertiary amine and does cross the blood-brain barrier. This property makes physostigmine useful in the treatment for central anticholinergic syndrome (also called postoperative delirium or atropine toxicity) (Barash: Clinical Anesthesia, ed 7, pp 382–383). Pharmacology and Pharmacokinetics of Intravenous Drugs 65
180. Which drug exerts its main central nervous system (CNS) action by inhibiting the N-methyl-d-aspartate (NMDA) receptors? A. Propofol B. Midazolam C. Etomidate D. Ketamine
180. (D) Whereas propofol, barbiturates, etomidate, and benzodiazepines exert much, if not all, of their pharmacologic effects via the GABA receptors, ketamine has only weak activity on the GABA receptors. Ketamine’s mechanism of action is complex, with most of the effects due to interaction with NMDA receptors. Ketamine also interacts with monoaminergic, muscarinic, and opioid receptors, as well as voltage-sensitive calcium ion channels (Miller: Basics of Anesthesia, ed 6, pp 109–110).
181. Which of the following opioid-receptor agonists has anticholinergic properties? A. Morphine B. Hydromorphone C. Sufentanil D. Meperidine
181. (D) All of the drugs listed are opioids. Meperidine is structurally similar to atropine and possesses mild anticholinergic properties. In contrast to other opioid-receptor agonists, meperidine rarely causes bradycardia but can increase heart rate. Normeperidine, a metabolite of meperidine with some CNS-stimulating properties, may cause delirium and seizures if the level is high enough. This is more likely in patients who have renal impairment and are receiving meperidine over several days (Stoelting: Pharmacology and Physiology in Anesthetic Practice, ed 4, pp 102–104).
182. Which of the following statements about ketamine is FALSE? A. In the United States, it is a racemic mixture of two isomers B. It is a potent cerebral vasodilator and can increase intracranial pressure (ICP) C. Respiratory depression rarely occurs with induction doses D. Its metabolite norketamine is more potent than the parent compound
182. (D) In the United States, ketamine is prepared as a mixture of the two isomers S(+) and R(−). In some countries, the S(+) isomer, which is more potent and has fewer side effects, is available. All of the statements are true except for answer D. Norketamine (ketamine’s primary active metabolite) is one fifth to one third as potent as ketamine and can contribute to prolonged effects (Barash: Clinical Anesthesia, ed 7, pp 743–747; Miller: Basics of Anesthesia, ed 6, pp 109–111).
183. Which of the following vasopressor agents increases systemic blood pressure (BP) indirectly by stimulating the release of norepinephrine from sympathetic nerve fibers and directly by binding to adrenergic receptors? A. Vasopressin B. Ephedrine C. Epinephrine D. Phenylephrine
183. (B) Direct-acting sympathomimetic drugs work directly on the receptors. Indirect-acting sympathomimetic drugs have their effects primarily by entering the neurons and then displacing norepinephrine and causing the release of norepinephrine from the postganglionic sympathetic nerve fibers. Ephedrine, mephentermine, and metaraminol are primarily indirect-acting sympathomimetic agents that also may have some direct-acting properties. The following table summarizes the sympathomimetic agents and their effects on the adrenergic receptors (Miller: Basics of Anesthesia, ed 6, pp 72–73). CLASSIFICATION AND COMPARATIVE PHARMACOLOGY OF SYMPATHOMIMETICS Sympathomimetic α β1 β2 Mechanism of Action Amphetamine ++ + + Indirect Dobutamine 0 +++ 0 Direct Dopamine ++ ++ + Direct Ephedrine ++ + + Indirect and some direct Epinephrine + ++ ++ Direct Isoproterenol 0 +++ +++ Direct Mephentermine ++ + + Indirect Metaraminol ++ + + Indirect and some direct Methoxamine +++ 0 0 Direct Norepinephrine +++ ++ 0 Direct Phenylephrine +++ 0 0 Direct From Stoelting RK: Pharmacology and Physiology in Anesthetic Practice, ed 4, Philadelphia, Lippincott Williams & Wilkins, 2006, p 293.
184. Methadone-induced constipation could be reversed without loss of analgesic effect with which of the following opioid antagonists? A. Naloxone B. Nalmefene C. Naltrexone D. Methylnaltrexone
184. (D) Naloxone, naltrexone, and nalmefene are opioid receptor antagonists that can reverse the central and peripheral effects of opioids (e.g., methadone). Methylnaltrexone is a quaternary ammonium opioid receptor antagonist that does not penetrate the CNS (i.e., does not reverse analgesia) but does antagonize peripheral opioid receptors (i.e., blocks the GI tract’s opioid receptors and can treat opioid-induced constipation). Because of its structure it is not absorbed after oral administration, so it is administered by injection (Hemmings: Pharmacology and Physiology for Anesthesia, pp 265–267; Miller: Miller’s Anesthesia, ed 8, pp 904–906).
185. The treatment of patients with human immunodeficiency virus (HIV) may include indinavir, nelfinavir, or ritonavir. What anesthetic consideration is significant with these drugs? A. Decreased platelet function B. Increased sensitivity to midazolam C. Hypoglycemia D. Hyperkalemia
185. (B) Patients with HIV take at least three drugs simultaneously during their treatment. A variety of antiretroviral drugs such as nucleoside reverse transcriptase inhibitors (NRTIs), non-nucleoside reverse 66 Part 1 Basic Sciences transcriptase inhibitors (NNRTIs), entry inhibitors, integrase inhibitors, and/or protease inhibitors are used. Indinavir, nelfinavir, and ritonavir are three of many protease inhibitors currently available. All protease enzyme inhibitors have metabolic drug interactions. Most (especially ritonavir in clinical doses) irreversibly inhibits CYP3A4 and this inhibition could last for 2 to 3 days after the drug is stopped. CYP3A4 is involved in the metabolism of benzodiazepines (e.g., midazolam) and many opioids (e.g., fentanyl), and these drugs will have higher concentrations and prolonged elimination times when protease inhibitors are used. Protease inhibitors can also induce the production of the CYP enzymes allowing some drugs (e.g., estrogens) to be metabolized more quickly. In addition, protease inhibitors may cause glucose intolerance, disorders in lipid metabolism, premature atherosclerosis, and diastolic dysfunction leading to heart failure, as well as acute tubular necrosis and nephrolithiasis (Brunton: Goodman & Gilman’s The Pharmacological Basis of Therapeutics, ed 12, pp 1623–1660; Hines: Stoelting’s Anesthesia and Co-Existing Disease, ed 6, pp 484–491).
186. Neurokinin-1 (NK1) antagonists such as aprepitant have all the following properties EXCEPT A. Anxiolytic B. Antidepressant C. Analgesic D. Antiemetic
186. (C) Aprepitant is an NK1 antagonist (substance P antagonist) with a long half-life of 9 to 13 hours. It is orally administered for the prevention and treatment of PONV, although it seems better in preventing vomiting. NK1 antagonists may act synergistically with 5-HT3 antagonists and/or dexamethasone. Aprepitant is not associated with QTc prolongation. Although marketed for its antiemetic effects, it has some anxiolytic and mild antidepressant effects as well (Hemmings: Pharmacology and Physiology for Anesthesia, p 512; Miller: Miller’s Anesthesia, ed 8, pp 2637, 2967–2968).
187. Which of the following drugs should be administered with caution to patients receiving echothiophate for the treatment of glaucoma? A. Atropine B. Succinylcholine C. Ketamine D. Remifentanil
187. (B) Echothiophate is an organophosphate that inhibits acetylcholinesterase as well as pseudocholinesterase, which is responsible for the metabolism of succinylcholine and ester-type local anesthetics. It does this by forming a phosphorylated complex with acetylcholinesterase. The topical solution is instilled in the eye for treatment of refractory open-angle glaucoma. The amount of drug absorbed may be sufficient to inhibit acetylcholinesterase and cause prolongation in the duration of action of succinylcholine or mivacurium. Because of this, it is “recommended” to wait at least 3 weeks after the stoppage of echothiophate before the administration of these two muscle relaxants. One must wonder about these “recommendations” because clinical cases have shown that when cholinesterase activity is decreased (from echothiophate) to no activity, the increase in duration of neuromuscular block from succinylcholine was less than 25 minutes (Miller: Miller’s Anesthesia, ed 8, p 379).
188. When one of four thumb twitches in the train-of-four (TOF) stimulation of the ulnar nerve can be elicited, how much suppression would there be if you were measuring a single twitch? A. 20 to 25 B. 45 to 55 C. 75 to 80 D. 90 to 95
188. (D) Monitoring neuromuscular blockade for nondepolarizing muscle relaxants can be done in a variety of ways. The simplest way is to measure the reduction or suppression of a single twitch height. This is commonly performed by observing the twitch response of the thumb’s adductor pollicis muscle, after ulnar nerve stimulation. At 90% to 95% reduction of twitch height (i.e., ED90 to ED95) there is good muscle relaxation for intubation and intra-abdominal surgery. However, measuring the reduction of twitch height is not practical. Because there is good correlation between reduction of twitch height and the number of thumb twitches that can be elicited by TOF stimulation, TOF stimulation is more commonly used where four twitches are administered over 2 seconds. If only one twitch of a TOF is demonstrated, single twitch height is depressed at least 85%; with two to four thumb twitches, 70% to 85% depression is seen. Note that the presence of four twitches does not mean that neuromuscular function has completely recovered; in fact, a significant number of receptors may still be occupied by the muscle relaxant (Barash: Clinical Anesthesia, ed 7, p 544).
189. Which of the following muscle relaxants causes slight histamine release at two to three times the ED95 (effective dose in 95% of subjects) dose? A. Rocuronium B. Pancuronium C. Atracurium D. Cisatracurium
189. (C) There are two major chemical classes of nondepolarizing muscle relaxants: the aminosteroids (-onium drugs) and the benzylisoquinolinium (-urium) drugs. In general, the aminosteroids cause no significant histamine release (at the clinical doses of 2 to 3 × ED95), whereas some of the benzylisoquinolinium drugs can. The histamine release primarily occurs with rapid administration of atracurium but does not occur with cisatracurium or doxacurium. The amount of histamine released is rarely of clinical significance. The cardiovascular effects of neuromuscular blocking drugs occur by three main mechanisms: (1) drug-induced histamine release; (2) effects at cardiac muscarinic receptors; or (3) effects on nicotinic receptors at autonomic ganglia. The following table summarizes the mechanisms for the cardiovascular effects of muscle relaxants (Miller: Miller’s Anesthesia, ed 7, p 882). Pharmacology and Pharmacokinetics of Intravenous Drugs 67 CLINICAL AUTONOMIC EFFECTS OF NEUROMUSCULAR BLOCKING DRUGS Drug Autonomic Ganglia Cardiac Muscarinic Receptors Histamine Release Depolarizing Substance Succinylcholine Stimulates Stimulates Slight Benzylisoquinolinium Compounds Mivacurium None None Slight Atracurium None None Slight Cisatracurium None None None d-tubocurarine Blocks None Moderate Steroidal Compounds Vecuronium None None None Rocuronium None Blocks weakly None Pancuronium None Blocks moderately None From Miller RD: Miller’s Anesthesia, ed 7, Philadelphia, Saunders, 2011, p 882, Table 29-11.
190. Termination of action of the neurotransmitter norepinephrine is achieved predominately by which mechanism? A. Reuptake into postganglionic sympathetic nerve endings (uptake 1) B. Dilution by diffusion away from receptors C. Metabolism by catechol-O-methyltransferase (COMT) D. Metabolism by monoamine oxidase (MAO)
190. (A) Postganglionic sympathetic nerve fibers release norepinephrine from the synaptic vesicles in the nerve terminals. Eighty percent of the released norepinephrine rapidly undergoes reuptake into the sympathetic nerve terminals (uptake 1) and reenters storage vesicles for future release. Only a small amount of the norepinephrine that is reabsorbed is metabolized in the cytoplasm by MAO. Twenty percent of the norepinephrine is diluted by diffusion away from the receptors and can gain access to the circulation. COMT, which is located primarily in the liver, metabolizes this norepinephrine (Miller: Miller’s Anesthesia, ed 8, pp 357–358; Stoelting: Pharmacology and Physiology in Anesthetic Practice, ed 4, pp 700–701).
191. The incidence of unpleasant dreams associated with emergence from ketamine anesthesia can be reduced by the administration of A. Caffeine B. Droperidol C. Physostigmine D. Midazolam Pharmacology and Pharmacokinetics of Intravenous Drugs 53
191. (D) Administration of ketamine may be associated with visual, auditory, and proprioceptive hallucinations. These unpleasant side effects of ketamine occur on emergence and may progress to delirium. The incidence of emergence delirium from ketamine is dose dependent and occurs in approximately 5% to 30% of patients. Emergence delirium is less frequent after repeated administrations of ketamine. The most effective prevention for emergence delirium is administration of a benzodiazepine (midazolam being more effective than diazepam) about 5 minutes before induction of anesthesia with ketamine. Atropine and droperidol given perioperatively may increase the incidence of emergence delirium (Miller: Basics of Anesthesia, ed 6, pp 110–111; Miller: Miller’s Anesthesia, ed 8, pp 827–828).
192. Which of the following premedications is associated with extrapyramidal side effects? A. Metoclopramide B. Diazepam C. Scopolamine D. Glycopyrrolate
192. (A) Extrapyramidal side effects are seen most often with antipsychotic drugs (e.g., phenothiazines, thioxanthenes, and butyrophenones), but they also can be seen with administration of metoclopramide. Metoclopramide, a dopamine antagonist, increases lower esophageal sphincter tone and stimulates gastric and upper intestinal tract motility. Side effects associated with metoclopramide use include mild sedation, dysphoria, agitation, dry mouth, and, in rare instances, dystonic extrapyramidal reactions (oculogyric crises, trismus, torticollis). Akathisia, or the feeling of unease and motor restlessness, has occurred following IV metoclopramide, which may result in cancellation of elective surgery (Miller: Miller’s Anesthesia, ed 8, p 2963; Stoelting: Pharmacology and Physiology in Anesthetic Practice, ed 4, pp 499–502).
193. Succinylcholine, when administered to patients with renal failure, will increase serum [K+] by approximately A. No increase in [K+] B. 0.5 mEq/L C. 1.5 mEq/L D. 2.5 mEq/L
193. (B) Succinylcholine is a depolarizing muscle relaxant that chemically resembles acetylcholine and attaches to the postjunctional membrane ion channel receptors. Sustained opening of ion channels produced by succinylcholine (as opposed to a transient opening with acetylcholine) is associated with leakage of potassium from the interior of cells sufficient to increase plasma concentrations of potassium by about 0.5 mEq/L in normal patients. This slight increase of potassium levels in patients with renal failure is similar to patients with normal renal function (Miller: Miller’s Anesthesia, ed 8, p 963; Miller: Basics of Anesthesia, ed 6, p 148; Stoelting: Pharmacology and Physiology in Anesthetic Practice, ed 4, p 220).
194. Each of the following drugs can enhance the neuromuscular blockade produced by nondepolarizing muscle relaxants EXCEPT A. Calcium B. Aminoglycoside antibiotics C. Magnesium D. Intravenous lidocaine
194. (A) Many drugs can enhance the neuromuscular block produced by nondepolarizing muscle relaxants. These include volatile anesthetics, aminoglycoside antibiotics, magnesium, intravenous local anesthetics, furosemide, dantrolene, calcium channel blockers, and lithium. Calcium does not enhance 68 Part 1 Basic Sciences neuromuscular blockade and, in fact, actually antagonizes the effects of magnesium. In patients with hyperparathyroidism and hypercalcemia there is a decreased sensitivity to nondepolarizing muscle relaxants and shorter durations of action (Miller: Miller’s Anesthesia, ed 8, pp 980–983).
195. Discontinuation of which of the following medications is strongly recommended before elective surgery? A. Clonidine B. Metoprolol C. Monoamine oxidase inhibitors (MAOIs) D. None of the above
195. (D) None of these drugs should be abruptly stopped. Clonidine is a centrally active α-adrenergic agonist that is used in the treatment of hypertension. Severe rebound hypertension can be seen between 8 and 36 hours after the last dose, especially in patients receiving more than 1.2 mg/day. Rebound hypertension, as well as cardiac ischemia, can be seen after discontinuation of β-blocker therapy (e.g., atenolol or metoprolol). In the past, it was recommended to stop MAOIs 2 to 3 weeks before elective surgery because of the possibility of developing hypertensive crisis during surgery. More recently, it has become acceptable to use these drugs up to the time of surgery, because their discontinuance could place the patient at risk for suicide. Certain drug interactions may occur with MAOI use, including skeletal muscle rigidity or hyperpyrexia with meperidine, as well as an exaggerated hypertensive response with the indirect-acting vasopressor ephedrine. Abrupt withdrawal of chronic high-dose tricyclic antidepressant therapy can be associated with withdrawal symptoms (i.e., malaise, chills, coryza, skeletal muscle aching) and is not recommended (Miller: Basics of Anesthesia, ed 6, pp 179–182; Stoelting: Pharmacology and Physiology in Anesthetic Practice, ed 4, pp 401–407).
196. Circulating BNP (B-type natriuretic peptide) is a powerful biomarker predicting outcomes of which of the following? A. Heart B. CNS C. Kidneys D. Organ rejection
196. (A) About 40 years ago it was noted that kidney response varies with the type of shock. In canines, hypovolemic shock reduced renal blood flow to 10% of controls, whereas cardiogenic shock reduced renal blood flow to only 75% of controls. The main difference seemed to be related to the atrial pressures (decreased in hypovolemic shock but increased in cardiogenic shock). About 10 years later, a peptide was isolated from the atrium of rats named atrial or A-type natriuretic peptide (ANP). Later a natriuretic peptide was isolated from porcine brains and was named brain or B-type natriuretic peptide (BNP). In humans, BNP is mainly produced in the cardiac ventricles. Natriuretic peptides are primarily released from the atria (ANP) and ventricles (BNP) when the chambers are overdistended. Thus, in the failing heart, BNP is released. Natriuretic peptides have a main effect on the kidneys to excrete sodium and water. They have vasodilating properties and inhibit the release of renin. Blood levels of BNP are used as a marker for the severity of cardiovascular disease and may have a role in preoperative cardiac risk assessment. Nesiritide is a recombinant BNP and is being studied for the treatment of acute heart failure (Barash: Clinical Anesthesia, ed 7, p 141; Hines: Stoelting’s Anesthesia and Co-Existing Disease, ed 6, pp 123–125, 131; Miller: Miller’s Anesthesia, ed 8, p 3).
197. Hyperkalemia is NOT a risk for patients receiving succinylcholine with which of the following? A. Multiple sclerosis B. Myasthenia gravis C. Guillain-Barré syndrome D. Becker muscular dystrophy
197. (B) Multiple sclerosis (MS) is an acquired inflammatory autoimmune disease in which there is demyelination of nerve fibers within the CNS. In patients with MS and profound neurologic deficits, succinylcholine may cause hyperkalemia and should be avoided, and nondepolarizing muscle relaxants appear safe. Guillain-Barré syndrome is an inflammatory polyneuritis affecting the peripheral nervous system and associated with muscle weakness. In patients with Guillain-Barré, succinylcholine may cause hyperkalemia and should be avoided, whereas nondepolarizing muscle relaxants are not contraindicated but are avoided because of increased sensitivity and possible prolonged muscle weakness in the postoperative period. Duchenne muscular dystrophy and the less common Becker muscular dystrophy are both X-linked recessive diseases. They are characterized by progressive muscle weakness. In 1992 the U.S. Food and Drug Administration issued a warning with regard to the use of succinylcholine in children and adolescents because succinylcholine has been associated with several deaths when administered to patients with unsuspected muscular dystrophy (many developed hyperkalemia and were later diagnosed as having muscular dystrophy). Nondepolarizing muscle relaxants appear safe, but a slower onset may exist. Myasthenia gravis patients have fewer postsynaptic receptors at the myoneural junction, and, if succinylcholine is administered, they appear to be resistant. Larger doses appear needed (1.5-2 mg/ kg) for intubation, and there is no associated hyperkalemic response. The duration of action of succinylcholine, on the other hand, will be prolonged because these patients receive anticholinesterase therapy (pyridostigmine). They are, however, very sensitive to nondepolarizing muscle relaxants, and a greatly reduced dose of a nondepolarizer should be administered, if at all (Fleisher: Anesthesia and Uncommon Diseases, ed 6, pp 267–273, 297-302, 314–315; Miller: Miller’s Anesthesia, ed 8, pp 1266–1284). Pharmacology and Pharmacokinetics of Intravenous Drugs 69
198. Which of the antibiotics below does NOT augment neuromuscular blockade? A. Clindamycin B. Neomycin C. Streptomycin D. Erythromycin
198. (D) Several antibiotics potentiate neuromuscular blockade. The aminoglycosides (neomycin, streptomycin, gentamicin, and tobramycin) and the lincosamides (clindamycin and lincomycin) can augment neuromuscular blockade. The only drug in question that does not affect neuromuscular blockade is erythromycin (of the macrolide antibiotic group). In addition, tetracyclines, penicillins, and cephalosporins do not affect neuromuscular blockade (Barash: Clinical Anesthesia, ed 7, p 541; Miller: Miller’s Anesthesia, ed 8, pp 981–982).
199. A 43-year-old woman with ascites, hepatopulmonary syndrome, and bleeding esophageal varices is admitted to the ICU. Which of the therapies below is LEAST likely to improve symptoms associated with hepatic encephalopathy (HE)? A. Amino acid–rich total parenteral nutrition (TPN) B. Neomycin C. Lactulose D. Flumazenil
199. (A) With liver failure, the liver cannot adequately detoxify noxious chemicals. Among patients with endstage liver disease, 50% to 70% develop hepatic HE. Symptoms vary from mild confusion, drowsiness, and stupor to coma. The etiology of HE is complex. Because an elevation in blood ammonia levels (easily measured) is strongly associated with HE, treatment is aimed at lowering the ammonia level. Other toxins also contribute to HE. To lower the ammonia level, lactulose (which decreases the absorption of ammonia) and neomycin (which reduces the production of ammonia by reducing the ammonia-producing intestinal flora) are commonly administered. Protein restriction is commonly done to decrease ammonia production, so amino acid–rich TPN is not helpful. Flumazenil (a GABA receptor antagonist) has been shown to produce short-duration reversal of the symptoms of HE in some patients and thus suggests that GABA receptors are somehow activated during HE. GABA receptors are responsible for inhibitory neurotransmission in the CNS (Hines: Stoelting’s Anesthesia and Co-Existing Disease, ed 6, p 280; Miller: Basics of Anesthesia, ed 6, p 457; Miller: Miller’s Anesthesia, ed 8, p 541).
200. 100 mg succinylcholine is administered to a 70-kg anesthetized man before intubation. The patient remains paralyzed for 20 minutes. Which of the parameters below is NOT consistent with this finding? A. Dibucaine number 70 B. Heterozygous for atypical cholinesterase C. Incidence of 1/480 D. Presence of fasciculations with this dose
200. (A) In most patients, an intravenous intubating dose of succinylcholine (1 mg/kg = 2 × the ED95) will show neuromuscular blockade that lasts 5 to 10 minutes. The reason for the short duration of action relates to succinylcholine’s very rapid metabolism by typical plasma cholinesterase (also called pseudocholinesterase or butyrylcholinesterase). Some patients, however, have a prolonged effect, which could be due to either a decrease in the quantity or a genetic qualitative change in the enzyme. Quantitative decreases can be seen in patients with malnutrition, liver disease, pregnancy, burns, or advanced age. Cholinesterase activity can also be decreased by the coadministration of various medications including anticholinesterase drugs (e.g., neostigmine), metoclopramide, and esmolol. A marked quantitative reduction (e.g., severe liver disease) can prolong succinylcholine activity about three times the normal duration of block. A marked prolongation of effect is due to the genetic production of atypical pseudocholinesterase (an inactive form). To investigate the genetic or qualitative change, a dibucaine inhibition test is done. The local anesthetic dibucaine can inhibit a normal enzyme more so than an abnormal enzyme. People with a normal dibucaine number of 70 to 80 are homozygous for the normal typical plasma cholinesterase and have the normal 5- to 10-minute neuromuscular blockade. People who are heterozygous (incidence of 1/480) for the atypical plasma cholinesterase have a dibucaine number of 50 to 60 and a block duration of 20 minutes. Patients who are homozygous for the atypical plasma cholinesterase (incidence 1/3200) have a dibucaine number of 20 to 30 and a block duration from 1 to 3 hours. This genetic variation of plasma cholinesterase is the most common abnormality; however, there are also other, less frequent genetic changes in the plasma cholinesterase. See also Question 260 (Miller: Basics of Anesthesia, ed 6, pp 148–149. Miller: Miller’s Anesthesia, ed 8, pp 960–962, 1135–1136).
201. In which of the following situations is succinylcholine most likely to cause severe hyperkalemia? A. 24 hours after a right hemisphere stroke B. 14 days after a severe burn injury C. 24 hours after a midthoracic spinal cord transection D. 2 days with a severe abdominal infection
201. (B) In normal patients, potassium levels increase about 0.5 mEq/L after the administration of succinylcholine. However, in some acquired conditions the potassium level may increase 5 to 7 mEq/L above the baseline potassium level after administration of succinylcholine. This marked elevation of potassium may lead to cardiac arrest. These acquired conditions include the following: (1) denervation injury as caused by spinal cord injury leading to skeletal muscle atrophy; (2) skeletal muscle injury resulting from third-degree burns (until scarring occurs); (3) acute upper motor neuron injury such as stroke; (4) severe skeletal muscle trauma; and (5) severe abdominal infections. In these acquired conditions the potential to increase potassium levels after succinylcholine usually takes a few days to develop, peaks 10 to 50 days after the initial injury, and may persist for 6 months or more. All factors considered, it might be prudent to avoid administration of succinylcholine to any patient more than 24 hours after the conditions listed here. This vulnerability to hyperkalemia may reflect a proliferation of extrajunctional cholinergic receptors, which provide more sites for potassium to leak outward across the cell membrane during depolarization. Some have suggested that the number of receptors is unchanged but that the receptors themselves have altered affinity to acetylcholine or drugs. Similar marked elevations of potassium may develop in cases of undiagnosed myopathy (Miller: Basics of Anesthesia, ed 6, p 149–150). 70 Part 1 Basic Sciences
202. The most common minor side effect reported after flumazenil administration in anesthesia is A. Nausea and/or vomiting B. Dizziness C. Tremors D. Hypertension
202. (A) Although flumazenil (a specific benzodiazepine antagonist) inhibits the activity at the GABA receptor, it works only at the benzodiazepine recognition site and has no effect in reversing other drugs that work on the GABA site (e.g., barbiturates, etomidate, propofol). It has a fast onset (within minutes), with peak brain levels occurring within 6 to 10 minutes, and a relatively short duration of action. Flumazenil can reverse all benzodiazepine CNS effects, including sedative, amnestic, muscle relaxant, and anticonvulsant effects. Side effects are rare, the most common being nausea, vomiting, or both (about 10%). Nausea occurs more commonly when flumazenil is given to patients after general anesthesia than after conscious sedation. Due to its short clinical duration of action, patients receiving flumazenil should be monitored for possible resedation and respiratory depression (Miller: Miller’s Anesthesia, ed 8, pp 843–844; Physicians Desk Reference, ed 63, 2009, pp 2646–2649; Butterworth: Morgan & Mikhail’s Clinical Anesthesiology, ed 5, p 290).
203. Ketorolac A. Is a selective cyclooxygenase-2 (COX-2) inhibitor B. Does not inhibit thromboxane A2 (TXA2) C. Does not inhibit prostaglandin I2 D. Exhibits a dose ceiling effect with regard to analgesia
203. (D) Nonsteroidal anti-inflammatory drugs (NSAIDs) (e.g., aspirin, acetaminophen, indomethacin, ibuprofen, diclofenac, and ketorolac) inhibit COX enzymes that are involved in the conversion of arachidonic acid to prostaglandin, thromboxane, and prostacyclin. COX-1 is involved with platelet aggregation and gastric mucosal protection; COX-2 is involved with pain, inflammation, and fever. TXA2 has prothrombotic and vasoconstricting properties. Prostacyclin I2 has antithrombotic and vasodilating properties. Ketorolac is a nonselective inhibitor of both COX-1 and COX-2 enzymes. Selective COX-2 drugs (e.g., only celecoxib, currently available in the United States) can be used, but in general these have been shown to cause a small increase in thrombotic issues (but fewer effects on gastric mucosa and platelet activity). Because of a ceiling effect with regard to analgesia, ketorolac has only mild-to-moderate analgesic effects (Hemmings: Pharmacology and Physiology for Anesthesia, pp 272–278; Miller: Basics of Anesthesia, ed 6, pp 703–704; Miller: Miller’s Anesthesia, ed 8, pp 2978–2982).
204. A 37-year-old patient with a history of acute intermittent porphyria is scheduled for knee arthroscopy under general anesthesia. Which of the following drugs is contraindicated in this patient? A. Fentanyl B. Isoflurane C. Propofol D. Etomidate
204. (D) Acute intermittent porphyria is the most serious form of porphyria. This disease affects both the central and peripheral nervous systems. An acute intermittent porphyria attack can be triggered by a variety of conditions, including starvation, dehydration, stress, sepsis, and some drugs, such as etomidate and barbiturates. Drugs that are safe or probably safe include local anesthetics, inhaled anesthetics, neuromuscular blocking drugs, some intravenous anesthetics (propofol and ketamine), some analgesics (acetaminophen, aspirin, morphine, fentanyl, sufentanil), antiemetics (droperidol, H2 blockers, metoclopramide, ondansetron), and neostigmine and naloxone. Drugs that are contraindicated include some intravenous anesthetics (barbiturates), some analgesics (ketorolac, pentazocine), and hydantoin anticonvulsants (Barash: Clinical Anesthesia, ed 7, pp 624–625; Hines: Stoelting’s Anesthesia and Co-Existing Disease, ed 6, p 308).
205. A 57-year-old male is discharged after tooth extraction of two molars. His only medication is paroxetine (Paxil), which he takes for depression. Codeine is a poor analgesic choice for this patient because A. It is likely to be ineffective B. It is likely to cause extreme sedation C. He is at increased risk for nausea D. He is at increased risk for serotonin syndrome 54 Part 1 Basic Sciences
205. (A) Cytochrome P450 (CYP) enzymes are involved in the metabolism of many medications. There are many such isoforms and these are further characterized into families with an Arabic number and further characterized into subfamilies (capital letter). The clinical activity of these enzymes can be increased (induced) or decreased (inhibited) by age, genetics, medications, and some foods. CYP2D6 is needed to convert the inactive codeine to the active morphine. Similarly, CYP2D6 also metabolizes oxycodone into active oxymorphone, and inactive hydrocodone into active hydromorphone. CYP2D6 is inhibited by selective serotonin reuptake inhibitors (SSRIs) as well as with quinidine. SSRIs include fluoxetine (Prozac), sertraline (Zoloft), paroxetine (Paxil), fluvoxamine (Luvox), citalopram (Celexa), and escitalopram (Lexapro). Thus, patients taking SSRIs or quinidine will get a poor analgesic effect with codeine, oxycodone, and hydrocodone (Hemmings; Pharmacology and Physiology for Anesthesia, pp 64–65, 183–186; Miller: Basics of Anesthesia, ed 6, p 37).
206. If etomidate were accidentally injected into a left-sided radial arterial line, the most appropriate step to take would be A. Left stellate ganglion block B. Administer intra-arterial clonidine C. Slowly inject dilute (0.1 mEq/L) [HCO3 –] D. Observe
206. (D) Although etomidate causes pain on intravenous injection in up to 80% of patients, the unintentional administration of etomidate into an artery does not result in detrimental effects to the artery (Miller: Miller’s Anesthesia, ed 8, p 852).
207. The most important reason for the more rapid onset and shorter duration of action of fentanyl with single dose compared with morphine is the difference in A. Volume of distribution B. Hepatic clearance C. Protein binding D. Lipid solubility
207. (D) Fentanyl is more lipid soluble than morphine, so it passes through the blood-brain barrier more easily and has a faster onset of action. Fentanyl also has a larger volume of distribution, slower plasma clearance, and longer elimination half-life than morphine. However, the duration of action of fentanyl (when given in small doses) is much shorter than that of morphine because fentanyl is rapidly redistributed from the brain to inactive tissue sites (e.g., lipid sites). In larger doses, these tissue sites can Pharmacology and Pharmacokinetics of Intravenous Drugs 71 become saturated, and the pharmacologic action of fentanyl becomes considerably prolonged (Miller: Basics of Anesthesia, ed 7, p 115–119; Stoelting: Pharmacology and Physiology in Anesthetic Practice, ed 4, pp 104–105).
208. A narcotic infusion is initiated in a patient without a bolus (loading dose). Of the following drugs, which would reach steady state after 2 hours or less of continuous infusion (fentanyl, remifentanil, alfentanil, and morphine)? A. All of these B. Remifentanil and alfentanil C. Alfentanil only D. Remifentanil only
208. (D) This question illustrates the concept of infusion front-end kinetics. This concept is useful for comparing the kinetics of various intravenous agents used in anesthesia. Remifentanil reaches the steady state in less than 1 hour of continuous infusion. Approximately 8 hours are required to reach the steady state with alfentanil and sufentanil, whereas fentanyl and morphine have not achieved the steady state concentration even after 10 hours of continuous infusion. Another important concept is the time after bolus to reach peak effect: bolus front-end kinetics. This concept is more intuitive to most anesthesia providers. Comparing the same narcotics used in this question, alfentanil and remifentanil reach peak concentration at nearly the same time and fentanyl only slightly later (Miller: Basics of Anesthesia, ed 6, p 119). 100 80 60 40 20 0 0 100 200 Infusion begins at time zero 300 400 500 600 Proportion of steady-state Ce (%) Fentanyl Alfentanil Morphine Sufentanil Infusion duration (min) Remifentanil Infusion: Front-end
209. The period of vulnerability after three courses of bleomycin for testicular cancer is A. 1 month B. 1 year C. Lifelong D. No vulnerability with just three courses
209. (C) Bleomycin is used primarily in the treatment of Hodgkin lymphoma and testicular tumors. Bleomycin causes oxidative damage to nucleotides, which leads to breaks in DNA. Although the more common side effects of bleomycin use are mucocutaneous, dose-related pulmonary toxicity is the most serious side effect. Early signs and symptoms of pulmonary toxicity include dry cough, fine rales, and diffuse infiltrates on x-ray. Approximately 5% to 10% of patients will develop pulmonary toxicity, and about 1% will die from this complication. Most believe that the risk of pulmonary toxicity increases with dose (especially total dose >250 mg), in patients older than 40 years of age, in patients with a creatinine clearance (CrCl) of less than 80 mL/min, and in patients with prior chest radiation or preexisting pulmonary disease. Although a relationship appears to exist between the use of bleomycin and the use of high concentrations of oxygen, the details are unclear. Currently, it has been suggested to use the lowest concentration of oxygen consistent with patient safety with a careful evaluation of oxygen saturation with pulse oximetry in any patient who has received bleomycin (Brunton: Goodman & Gilman’s The Pharmacological Basis of Therapeutics, ed 12, pp 1716–1718; Miller: Miller’s Anesthesia, ed 8, p 1951; Stoelting: Pharmacology and Physiology in Anesthetic Practice, ed 4, pp 555–565).
210. The unique advantage of rocuronium over other muscle relaxants is its A. Short duration of action B. Metabolism by pseudocholinesterase C. Onset of action D. Lack of need for reversal
210. (C) The first two letters of the word “rocuronium” stand for “rapid onset.” Of the nondepolarizing muscle relaxants currently available, rocuronium has the most rapid onset of action at clinically useful dosages. Rocuronium is a nondepolarizing neuromuscular relaxant with an intermediate duration of action similar to vecuronium, atracurium, and cisatracurium. At an ED95 dose (0.3 mg/kg), the onset time is 1.5 to 3 minutes, whereas with the other intermediate nondepolarizing muscle relaxants, the onset time is 3 to 7 minutes. At larger doses (i.e., 2 × ED95 or 0.6 mg/kg), onset time can be reduced to 1 to 1.5 minutes (Barash: Clinical Anesthesia, ed 7, p 538).
211. Which of the following statements regarding the efficacy of neuromuscular blockade in the setting of acute hypokalemia is correct? A. There is no effect with depolarizing or nondepolarizing muscle relaxants B. There is resistance to effects of both depolarizing and nondepolarizing muscle relaxants C. There is increased sensitivity to effects of both depolarizing and nondepolarizing muscle relaxants D. There is resistance to depolarizing muscle relaxants and increased sensitivity to nondepolarizing muscle relaxants
211. (D) An acute decrease in serum potassium causes hyperpolarization of cell membranes. This causes resistance to depolarizing neuromuscular blockers and an increased sensitivity to nondepolarizing neuromuscular blockers (Stoelting: Pharmacology and Physiology in Anesthetic Practice, ed 4, pp 226–227). 72 Part 1 Basic Sciences
212. A patient undergoing which of the following operations would be at highest risk for operative recall? A. Laparoscopic cholecystectomy with total intravenous anesthesia (no volatile) B. Cervical spine fusion with MEP (motor evoked potentials) monitoring C. Pneumonectomy with one-lung ventilation D. Emergency splenectomy after falling from a ladder
212. (D) Awareness during general anesthesia is the postoperative recall of events that happened during the anesthetic. Overall incidence has decreased from about 1% 50 years ago to about 0.1% today (with some variations from study to study). Patients at increased risk include patients undergoing cardiac surgery, endoscopic airway surgery, cesarean sections, and trauma surgery (Miller: Miller’s Anesthesia, ed 8, p 1528).
213. A 58-year-old patient is brought to the emergency room with the following symptoms: miosis, abdominal cramping, salivation, loss of bowel and bladder control, bradycardia, ataxia, and skeletal muscle weakness. The most likely diagnosis is A. Central anticholinergic syndrome B. Malignant neuroleptic syndrome C. Anticholinesterase poisoning D. Serotonin syndrome
213. (C) The symptoms described in this patient are consistent with cholinergic stimulation or increased levels of acetylcholine that occur with anticholinesterase poisoning. Stimulation of the parasympathetic nervous system produces miosis, abdominal cramping, excess salivation, loss of bowel and bladder control, bradycardia, and bronchoconstriction. These symptoms are treated with atropine. The acetylcholinesterase reactivator pralidoxime sometimes is added to treat the nicotinic effects of elevation of acetylcholine at the neuromuscular junction of skeletal muscle (i.e., skeletal muscle weakness, apnea). CNS effects of elevated acetylcholine levels can include confusion, ataxia, and coma. In addition, supportive therapy (the ABCs of resuscitation [Airway, Breathing, Circulation, etc.]) is provided as needed (Miller: Miller’s Anesthesia, ed 8, p 2495).
214. Flumazenil A. Is contraindicated in narcotic addicts B. Can be given orally as well as intravenously C. Can produce seizures in chronic benzodiazepine users D. Has a longer elimination half-life compared to midazolam
214. (C) Flumazenil is a benzodiazepine antagonist used to antagonize the benzodiazepine effects on the CNS. It does not reverse the effects of barbiturates, opiates, or alcohol. Seizures can be precipitated in patients who have been on benzodiazepines for long-term sedation or patients showing signs of serious cyclic antidepressant overdosage (e.g., twitching, rigidity, widened QRS complex, hypotension). Flumazenil has a shorter elimination half-life (0.7-1.3 hours) compared with midazolam (2-2.5 hours). Flumazenil is poorly absorbed orally (Miller: Miller’s Anesthesia, 8, p 843; Physicians’ Desk Reference, ed 63, 2009, pp 2646–2649).
215. What percentage of neuromuscular receptors could be blocked and still allow patients to carry out a 5-second head lift? A. 5% B. 15% C. 25% D. 50%
215. (D) Adequate recovery from neuromuscular blockade is believed to occur when 50% or less of receptors are occupied with muscle relaxants. This can be measured with sustained tetanus at 100 Hz, but this test is very painful. Another method requires patient cooperation and consists of a sustained head lift for 5 seconds in the supine position. The “head lift” test is the standard test to determine adequate muscular function (Miller: Basics of Anesthesia, ed 6, p 158).
216. A 25-year-old woman undergoes thyroidectomy under general anesthesia. Ondansetron 4 mg IV is administered as nausea prophylaxis. She complains of nausea in the recovery room. Which of the follow agents is LEAST likely to be of benefit to her for treatment (rescue) of postoperative nausea and vomiting (PONV)? A. Aprepitant B. Granisetron C. Promethazine D. Droperidol
216. (B) PONV is the second-most common complaint reported in the perioperative period (pain is the number one complaint). Many drugs have been used to both prevent (prophylaxis) and to treat (rescue) PONV. Antiemetics were often administered alone, but now combination therapy of two or more drugs such as dopamine antagonists (e.g., droperidol, metoclopramide), histamine antagonists (e.g., diphenhydramine, promazine), anticholinergics (e.g., scopolamine), steroids (e.g., dexamethasone), neurokinin antagonists (e.g., aprepitant), and serotonin antagonist (e.g., ondansetron, dolasetron, granisetron, and palonosetron) are commonly used. Once a serotonin antagonist is given for prophylaxis, adding more of a serotonin antagonist in the PACU does not seem to help. It is better to use an antiemetic from another class of drugs (Hemmings: Pharmacology and Physiology for Anesthesia, pp 503–551; Miller: Miller’s Anesthesia, ed 8, pp 2947, 2969–2970).
217. Which of the following drugs can prevent tachyarrhythmias in patients with Wolff-Parkinson-White (WPW) syndrome? A. Droperidol B. Pancuronium C. Ketamine D. Verapamil
217. (A) Patients with WPW syndrome are predisposed to develop supraventricular arrhythmias. Sympathetic stimulation (e.g., anxiety, hypovolemia), as well as many drugs (e.g., pancuronium, meperidine, ketamine, ephedrine, digoxin, verapamil), can induce tachyarrhythmias, often by enhancing conduction through accessory atrial pathways. Although verapamil is used to treat supraventricular tachyarrhythmias because of its depressant effects on alveolar nodal conduction, it actually may increase the heart rate in patients with WPW syndrome because it can increase conduction of the accessory pathways. Droperidol, in addition to its antidopaminergic properties, has antidysrhythmic properties that protect against epinephrine-induced dysrhythmias. Proposed mechanisms include α-adrenergic receptor blockade and mild local anesthetic effects. Large doses of droperidol (0.2-0.6 mg/kg) can reduce impulse transmission via the accessory pathways responsible for the tachyarrhythmias that occur in patients with WPW syndrome (Stoelting: Pharmacology and Physiology in Anesthetic Practice, ed 4, pp 413–415, 766).
218. The half-life of pseudocholinesterase is A. 1 hour B. 12 hours C. 1 week D. 2 weeks Pharmacology and Pharmacokinetics of Intravenous Drugs 55
218. (B) Pseudocholinesterase (also called plasma cholinesterase) is an enzyme found in plasma and most other tissues (except erythrocytes). Pseudocholinesterase metabolizes the acetylcholine released at the neuromuscular junction, as well as certain drugs such as succinylcholine, mivacurium, and ester-type local anesthetics. It is produced in the liver and has a half-life of approximately 8 to 16 hours. Pseudocholinesterase levels may be reduced in patients with advanced liver disease. The decrease must be greater than 75% before significant prolongation of neuromuscular blockade occurs with succinylcholine (Stoelting: Pharmacology and Physiology in Anesthetic Practice, ed 4, p 218). Pharmacology and Pharmacokinetics of Intravenous Drugs 73
219. Some COX-2 inhibitors (e.g., rofecoxib [Vioxx]) have been withdrawn from the U.S. pharmaceutical market because of serious complications involving A. Platelet inhibition and gastrointestinal (GI) hemorrhage B. Renal failure C. Hypertension D. Promotion of thrombotic state
219. (D) COX inhibitors are useful analgesics for mild-to-moderate pain. There are three types of COX inhibitors: cyclooxygenase-1 (COX-1), cyclooxygenase-2 (COX-2), and cyclooxygenase-3 (COX-3). COX-3 is a variant of COX-1, and there is some controversy as to its existence in humans. COX inhibitors block prostaglandin synthesis in the periphery and in the CNS. COX-1 has GI mucosal protecting properties and stimulates platelet aggregation. Drugs with COX-1 inhibiting properties can cause gastric and duodenal ulcers and can interfere with platelet aggregation. COX-2 is involved in inflammation. NSAIDs are nonspecific COX-1 and COX-2 inhibitors. Selective COX-2 inhibitors such as celecoxib, valdecoxib, and rofecoxib are effective analgesics with anti-inflammatory effects. They have a lower risk of GI complications and antiplatelet properties than the nonspecific COX-1 and COX-2 inhibitors. Because of an increase in serious thromboembolic events (i.e., strokes and myocardial infarctions), both valdecoxib and rofecoxib have been withdrawn from the market. Currently, celecoxib is the only selective COX-2 inhibitor available in the United States. In addition, both the NSAIDs and selective COX-2 inhibitors can transiently decrease renal function, especially in patients with preexisting renal disease and in patients who are hypovolemic. These renal effects can lead to hypertension, edema, and acute renal failure (Hemmings: Pharmacology and Physiology for Anesthesia, pp 272–277; Miller: Basics of Anesthesia, ed 6, pp 703–704; Barash: Clinical Anesthesia, ed 7, p 437).
220. Which of the following equals the anti-inflammatory activity of 50 mg of prednisone (Deltasone)? A. 100 mg cortisol (Solu-Cortef) B. 80 mg methylprednisolone (Solu-Medrol) C. 7.5 mg dexamethasone (Decadron) D. 4 mg betamethasone (Celestone)
220. (C) The adrenal cortex secretes two classes of steroids, the corticosteroids (glucocorticoids and mineralocorticoids) and the androgens. The main glucocorticoid is hydrocortisone, also called cortisol. The glucocorticoids are used primarily for their anti-inflammatory and immunosuppressive effects, but they also have mineralocorticoid activity (i.e., sodium-retaining effects). These drugs differ in potency, amount of mineralocorticoid effect, and duration of action. The normal amount of cortisol produced daily is about 10 mg, but under stress, the level can increase tenfold. The main mineralocorticoid is aldosterone. The normal amount of aldosterone produced daily is about 0.125 mg. Because fludrocortisone has such significant mineralocorticoid activity, it is used only for this. The following table compares several corticosteroids. In this case, 50 mg of prednisone is equivalent in glucocorticoid activity to 7.5 mg of dexamethasone and 200 mg of hydrocortisone (Hardman: Goodman & Gilman’s The Pharmacological Basis of Therapeutics, ed 10, pp 1655–1666; Stoelting: Pharmacology and Physiology in Anesthetic Practice, ed 4, pp 461–464). COMPARATIVE PHARMACOLOGY OF CORTICOSTEROIDS Agent Anti-inflammatory Potency Equivalent Glucocorticoid Dose (mg) Sodium-Retaining Potency Duration of Action (hr) Hydrocortisone or cortisol (Cortef) 1 20 1 8-12 Cortisone (Cortone) 0.8 25 0.8 8-36 Prednisolone (Hydeltrasol) 4 5 0.8 12-36 Prednisone (Deltasone) 4 5 0.8 18-36 Methylprednisolone (Solu-Medrol) 5 4 0.5 12-36 Triamcinolone (Kenalog) 5 4 0 12-36 Betamethasone (Celestone) 25 0.75 0 36-54 Dexamethasone (Decadron) 25 0.75 0 36-54 Fludrocortisone (Florinef) 10 2 250 24 Aldosterone 0 NA 3000 NA, not applicable. From Stoelting RK: Pharmacology and Physiology in Anesthetic Practice, ed 4, Philadelphia, Lippincott Williams & Wilkins, 2006, p 462. 74 Part 1 Basic Sciences
221. The recovery index (RI) of which of the following nondepolarizing muscle relaxants is NOT altered by aging? A. Atracurium B. Vecuronium C. Rocuronium D. Pancuronium
221. (A) The RI of neuromuscular blocking drugs is the time needed for spontaneous recovery of a twitch height from 25% to 75% of the baseline height. The elderly, who tend to have reduced renal and hepatic function, have a prolonged RI for nondepolarizing muscle relaxants that are dependent upon renal or hepatic elimination (e.g., vecuronium, d-tubocurarine, pancuronium, rocuronium). The RI for atracurium and cisatracurium, which are broken down in the plasma, are not prolonged in the elderly (Miller: Miller’s Anesthesia, ed 8, pp 975–976).
222. Side effects associated with cyclosporine therapy include each of the following EXCEPT A. Nephrotoxicity B. Pulmonary toxicity C. Seizures D. Limb paresthesias
222. (B) Cyclosporine is a drug that selectively inhibits helper T-lymphocyte-mediated but not B-lymphocyte– mediated immune responses. It is mainly used alone or in combination with corticosteroids to prevent or treat organ rejection. Other uses include the treatment of Crohn disease, uveitis, psoriasis, and rheumatoid arthritis. Side effects that may accompany the administration of cyclosporine include nephrotoxicity (25%-38%), hypertension, limb paresthesias (50%), headaches, confusion, somnolence, seizures, elevation of liver enzymes, allergic reactions, gum hyperplasia, hirsutism, and hyperglycemia. There appears to be no pulmonary toxicity associated with cyclosporine therapy (Miller: Miller’s Anesthesia, ed 8, p 580).
223. What is the predominant mechanism for succinylcholine- induced tachycardia in adults? A. Direct sympathomimetic effect at postjunctional muscarinic receptors B. Stimulation of nicotinic receptors at autonomic ganglia C. Blockade of nicotinic receptors at autonomic ganglia D. Direct vagolytic effect at postjunctional muscarinic receptors
223. (B) Succinylcholine is basically two acetylcholine molecules hooked together. Succinylcholine may exert cardiovascular effects by: (1) inducing histamine release from mast cells; (2) stimulating autonomic ganglia, which increases neurotransmission at both the sympathetic and parasympathetic nervous systems; and (3) directly stimulating postjunctional cardiac muscarinic receptors. The effect of succinylcholine on heart rate is variable, with both bradycardia and tachycardia possible. The final heart rate depends upon many factors, including the amount of nicotinic stimulation of the sympathetic and parasympathetic ganglia, which is greater for the nondominant autonomic nervous system. For example, when sympathetic nervous system tone is high (as in children), bradycardia is more likely to develop when succinylcholine is administered. When parasympathetic nervous system tone is high (as in many adults), tachycardia, although not common, is more likely to occur when succinylcholine is administered. Bradycardia is more likely to occur when a second intravenous dose of succinylcholine is administered 4 to 5 minutes after the first dose, especially when difficult laryngoscopy (e.g., intense vagal stimulation) is being performed (Miller: Basics of Anesthesia, ed 6, p 150).
224. Bradycardia observed after administration of succinylcholine to children is attributable to which mechanism? A. Nicotinic stimulation at the autonomic ganglia B. Stimulation of the vagus nerve centrally C. Muscarinic stimulation at the sinus node D. Muscarinic blockade at the sinus node
224. (C) Chemically, succinylcholine is two acetylcholine molecules hooked together and has many effects similar to acetylcholine. In addition to causing neuromuscular blockade, succinylcholine stimulates all cholinergic autonomic receptors, including the nicotinic receptors of the sympathetic and parasympathetic ganglia, as well as the muscarinic receptors in the sinus node of the heart. It is this muscarinic effect that causes the bradycardia that can be seen after the administration of succinylcholine in children. Also see explanation to Question 223 (Miller: Miller’s Anesthesia, ed 8, p 962).
225. Which of commonly used drugs below is NOT metabolized by nonspecific esterases? A. Propofol B. Esmolol C. Atracurium D. Remifentanil
225. (A) Propofol’s chemical structure is 2,6-diisopropylphenol (i.e., is not an ester) and thus is not metabolized by esterases. Propofol is rapidly metabolized by the liver to more water-soluble compounds that are then renally excreted. Esmolol is an ester compound and is rapidly metabolized by RBC esterases (short halflife of 9-10 minutes). Atracurium and cisatracurium primarily undergo Hofmann elimination, which is a chemical reaction. Atracurium has a second metabolic route: metabolism by nonspecific plasma esterases. Interestingly, cisatracurium, which is an isolated form of atracurium (1 of the 10 stereoisomers), does not undergo metabolism by nonspecific plasma esterases. The short duration of action of remifentanil is due to its ester structure, which is metabolized by blood and tissue nonspecific esterases. Because of the nonspecific metabolism, its duration of action is not prolonged in patients with pseudocholinesterase deficiency (Miller: Basics of Anesthesia, ed 6, pp 75, 100–101, 125, 154; Miller: Miller’s Anesthesia. ed 8, pp 371, 824, 888–889, 977).
226. Succinylcholine is contraindicated for routine tracheal intubation in children because of an increased incidence of which of the following side effects? A. Hyperkalemia B. Malignant hyperthermia C. Masseter spasm D. Sinus bradycardia
226. (A) Hyperkalemia, malignant hyperthermia, masseter spasm, sinus bradycardia, nodal rhythms, and myalgias are side effects that can be seen after the administration of succinylcholine. In recent years, there have been several case reports of intractable cardiac arrest in apparently healthy children after the administration of succinylcholine. In these cases, hyperkalemia, rhabdomyolysis, and acidosis were documented. Later, muscle biopsy samples demonstrated that many of these cases were subclinical cases of Duchenne muscular dystrophy. For this reason of occasional severe hyperkalemia, succinylcholine is contraindicated for routine tracheal intubation in children (Barash: Clinical Anesthesia, ed 7, p 1227; Miller: Miller’s Anesthesia, ed 8, p 983). Pharmacology and Pharmacokinetics of Intravenous Drugs 75
227. From MOST to LEAST rapid, select the correct temporal sequence of neuromuscular blockade in the adductor of the thumb, the orbicularis oculi, and the diaphragm after administration of an intubating dose of vecuronium to an otherwise healthy patient. A. Diaphragm, orbicularis oculi, thumb B. Orbicularis oculi, diaphragm, thumb C. Orbicularis oculi, thumb, diaphragm D. Orbicularis oculi same as diaphragm, thumb
227. (D) To make intubation easier, it is important to know when the muscles of the airway are maximally relaxed after administration of a neuromuscular relaxant. This often is done with neuromuscular monitoring. However, which muscles one monitors is important because neuromuscular blockade develops faster, lasts a shorter time, and recovers more quickly in the central muscles of the airway (i.e., the larynx, jaw, and diaphragm) than in the more peripheral abductor muscles of the thumb (e.g., ulnar nerve monitoring). Also important is the observation that the pattern of blockade in the orbicularis oculi (e.g., facial nerve monitoring) is similar to that of the laryngeal muscles and the diaphragm. Therefore, when the orbicular oculi muscles are maximally relaxed, intubation would be optimal. When adductor function of the thumb returns to normal, the diaphragm and laryngeal muscles will have recovered (Barash: Clinical Anesthesia, ed 7, p 545).
228. Select the TRUE statement regarding interaction of nondepolarizing neuromuscular blocking drugs when durations of action are dissimilar. A. If a long-acting drug is administered after an intermediate-acting drug, the duration of the long-acting drug will be longer than normal B. If a long-acting drug is administered after an intermediate- acting drug, the duration of the longacting drug will be about the same as expected C. If an intermediate-acting drug is administered after a long-acting drug, the duration of the intermediate-acting drug will be about the same as expected D. If an intermediate-acting drug is administered after a long-acting drug, the duration of action of the intermediate-acting drug will be longer than expected
228. (D) Rarely, it is necessary to change from one nondepolarizing drug to another. A general rule to determine the duration of action of a drug given after another drug of different duration is a matter of simple kinetics. Three half-lives will be required for a clinical changeover so that 95% of the first drug will have cleared for the block duration to begin to take on the characteristics of the second drug. For example, if an intermediate-acting muscle relaxant such as vecuronium is given after a long-acting agent such as pancuronium, the duration of action of vecuronium is prolonged after the first two maintenance doses of vecuronium. After the third maintenance dose the duration of vecuronium is not prolonged (Miller: Miller’s Anesthesia, ed 8, pp 980–981).
229. Select the correct statement regarding the effects of volatile anesthetics on nondepolarizing neuromuscular blocking drugs and the reversal agents. A. Volatile anesthetics potentiate neuromuscular blockade but retard reversal agents B. Volatile anesthetics potentiate both neuromuscular blocking drugs and reversal agents C. Volatile anesthetics retard both neuromuscular blocking drugs and reversal agents D. Volatile anesthetics retard neuromuscular blocking drugs but potentiate reversal agents
229. (A) Volatile anesthetics enhance neuromuscular blockade in a dose-dependent fashion. Recent studies have suggested that antagonism of neuromuscular block is slowed by volatile anesthetics; thus, volatile anesthetic vapor concentrations should be reduced as much as possible at the end of the case to help ensure that reversal will take place as promptly as possible (Miller: Miller’s Anesthesia, ed 8, p 981).
230. Meperidine is contraindicated in patients taking which of the following drugs for Parkinson disease? A. Bromocriptine B. Trihexyphenidyl (Artane) C. Selegiline (Eldepryl) D. Amantadine (Symmetrel) 56 Part 1 Basic Sciences
230. (C) Selegiline is an MAOI that is sometimes used in the treatment of Parkinson disease. Meperidine is the original phenylpiperidine from which a number of other congeners are derived (e.g., fentanyl, sufentanil, alfentanil, remifentanil). Meperidine is rarely used as an analgesic but rather as an anti-shivering drug. Meperidine (as well as methadone and tramadol) is contraindicated in patients taking MAOIs because of the possibility of serotonin syndrome (e.g., agitation, skeletal muscle rigidity, hyperpyrexia) or depression (e.g., hypotension, depressed ventilation, coma) that may result (Miller: Miller’s Anesthesia, ed 8, pp 894–896, 909–910).
231. Emergence delirium occurs most often with A. Sevoflurane B. Desflurane C. Ketamine D. Propofol
231. (A) Some children awaken from general anesthesia and appear restless and inconsolable during the early recovery period from general anesthesia. This is called emergence “excitement” delirium (ED), and more intensive nursing will be needed to prevent such children from hurting themselves as well as prevent them from pulling out intravenous lines or surgical drains. This usually resolves quickly when the child awakens more fully. Although untreated pain is often considered an instigating factor, many children can be pain free and still develop ED. Risk factors include age younger than 5 years (peak incidence, 2-4 years of age), the use of volatile anesthetics (sevoflurane has the highest frequency of ED), otolaryngologic and ophthalmologic surgeries, and anxious parents. Prophylactic treatment with a single IV dose of fentanyl (2.5 μg/kg), clonidine (2 μg/kg), ketamine (0.25 mg/kg), nalbuphine (0.1 mg/kg), or dexmedetomidine (0.15 μg/kg) can decrease the incidence. Some have used IV propofol (1 mg/kg) after turning off sevoflurane at the conclusion of surgery to decrease the incidence of ED. Intranasal fentanyl (1 μg/kg) may be useful when the IV route is unavailable (Davis: Smith’s Anesthesia for Infants and Children, ed 8, p 391; Miller: Basics of Anesthesia, ed 6, p 558; Miller: Miller’s Anesthesia, ed 8, pp 2941–2942).
232. The most common reason for patients to rate anesthesia with etomidate as unsatisfactory is A. PONV B. Pain on injection C. Recall of intubation D. Postoperative hiccups
232. (A) Etomidate, an imidazole derivative, is used most often for induction of general anesthesia, but it also can be used for maintenance of general anesthesia. Etomidate has a relatively short duration of action and provides very stable hemodynamics, even in patients with limited cardiovascular reserve. However, it is associated with several adverse effects. These adverse effects include a high incidence of nausea and vomiting (greater than after thiopental), pain on injection, thrombophlebitis, myoclonic movements, and, sometimes, hiccups. Nausea and vomiting constitute the most common reason patients rate anesthesia with etomidate as unsatisfactory. The addition of fentanyl to etomidate to decrease the pain of injection also increases the incidence of nausea and vomiting (Miller: Miller’s Anesthesia, ed 8, p 852).
233. Which of the following muscle relaxants inhibits the reuptake of norepinephrine by the adrenergic nerves? A. Pancuronium B. Vecuronium C. Rocuronium D. Atracurium
233. (A) Pancuronium tends to increase the heart rate, mean arterial BP, and cardiac output. This may be related to several mechanisms, including a moderate vagolytic effect, norepinephrine release, and decreased reuptake of norepinephrine by adrenergic nerves. The other listed drugs rarely cause direct adrener76 Part 1 Basic Sciences gic stimulation and do not inhibit the uptake of norepinephrine by adrenergic nerves (Miller: Miller’s Anesthesia, ed 8, p 978).
234. The most common side effect of oral dantrolene used to prevent malignant hyperthermia is A. Nausea and vomiting B. Muscle weakness C. Blurred vision D. Tachycardia
234. (B) Dantrolene is a muscle relaxant used orally to help control skeletal muscle spasticity in patients with upper motor neuron lesions, and it can be used acutely in the prevention of malignant hyperthermia in patients undergoing anesthesia. It is given intravenously in the treatment of malignant hyperthermia. Dantrolene has little or no effect on smooth or cardiac muscle at clinical doses. Dantrolene works directly on skeletal muscle by decreasing the amount of calcium released from the sarcoplasmic reticulum. This decreases the excitation–contraction coupling needed for the muscle to contract. The most common side effect of dantrolene administration is skeletal muscle weakness. Other acute side effects include nausea, diarrhea, and blurred vision. When the drug is given intravenously, a brisk diuresis occurs and is related to the mannitol added to make the intravenous solution isotonic. With chronic oral use, patients may rarely develop hepatitis and pleural effusions (Stoelting: Pharmacology and Physiology in Anesthetic Practice, ed 4, pp 596–597).
235. A 65-year-old patient is admitted for right upper quadrant pain. Acute cholecystitis is diagnosed and laparoscopic cholecystectomy planned. The patient has no major medical problems other than type 2 diabetes, for which she takes metformin, and depression, for which she takes paroxetine (SSRI inhibitor). Which of the following best describes the rationale for discontinuation of metformin 48 hours before surgery? A. Risk of metabolic acidosis B. Risk of hypoglycemia C. Risk of serotonin syndrome D. None of the above
235. (D) (Please also see explanation to Question 435.) Diabetes mellitus is a disease characterized by altered metabolism of carbohydrates (usually manifested by hyperglycemia), lipids, and proteins. Ninety percent of diabetic patients in the United States have non–insulin-dependent diabetes mellitus (NIDDM) or type 2 diabetes and a relative deficiency in circulating insulin. Diabetic patients also can have a decreased tissue response to circulating insulin (insulin resistance). Oral hypoglycemic agents, most commonly of the sulfonylurea chemical class, can be used in patients with NIDDM. These sulfonylurea drugs have many metabolic effects, including the initial stimulation of the pancreas to release insulin (chronically, insulin secretion is not increased but the hypoglycemic effects are maintained). Tolbutamide (Orinase) and chlorpropamide (Diabinese) are first-generation analogs. The biguanides metformin (Glucophage) and phenformin work by increasing the action of circulating insulin on peripheral tissues and are called antihyperglycemic, not hypoglycemic, agents. There is no risk of hypoglycemia with metformin even with overnight fasting. Phenformin was withdrawn from the market because of an association with lactic acidosis. Metformin, long thought to cause metabolic acidosis, is now understood to do so only in patients who have abnormal kidney or liver function. SSRIs are drugs commonly used for depression. SSRIs have serious side effects, including hyperpyrexia. There have been reports of serotonin syndrome with SSRI and methylene blue, but not with metformin (Miller: Miller’s Anesthesia, ed 8, pp 1219–1220; Miller: Basics of Anesthesia, ed 6, pp 182–183).
236. A 37-year-old man is brought to the operating room for repair of a broken mandible sustained in a motor vehicle accident. No other injuries are significant. The patient has been in treatment for alcohol abuse and takes disulfiram and naltrexone. Which of the following would be the best technique for management of this patient’s postoperative pain? A. Continue naltrexone with round-the-clock low-dose methadone B. Continue naltrexone with small doses of morphine every 4 hours as needed C. Continue naltrexone with small doses of nalbuphine every 4 hours as needed D. Discontinue naltrexone and treat pain with morphine as needed
236. (D) Disulfiram and naltrexone occasionally are administered orally in alcoholic rehabilitation programs. Disulfiram alters the metabolism of alcohol by irreversibly inactivating the enzyme aldehyde dehydrogenase. If the patient drinks alcohol, there is a buildup of acetaldehyde in the blood. This produces the unpleasant effects of flushing, headache, nausea, vomiting, chest pain, tachycardia, hypotension, and confusion. The alcohol sensitivity with disulfiram use may last up to 2 weeks after the drug is stopped. Naltrexone is used with disulfiram in the treatment of alcohol addiction. It appears to block some of the reinforcing properties of alcohol. Patients taking naltrexone with disulfiram have a lower rate of relapse for alcohol. Naltrexone is a pure opioid antagonist. Patients taking naltrexone at the time of surgery will have markedly elevated opioid requirements if opioids are chosen for pain relief. The duration of action of naltrexone is 24 hours, and the drug should be stopped during the hospitalization to allow better pain control with narcotics, as would be desirable in this major surgical procedure (Hardman: Goodman & Gilman’s The Pharmacological Basis of Therapeutics, ed 10, pp 602–604; Hines: Stoelting’s Anesthesia and Co-Existing Disease, ed 5, p 542; Miller: Miller’s Anesthesia, ed 8, pp 866–868).
237. Which of the following muscle relaxants is most suitable for rapid intubation in a patient in whom succinylcholine is contraindicated? A. Atracurium B. Rocuronium C. Vecuronium D. Cisatracurium
237. (B) Rapid-sequence inductions are performed in cases where rapid control of the airway is needed. Usually this is performed to secure the airway in a patient who should be easily intubated and has a “full stomach.” In these cases, after adequate preoxygenation and suctioning of the airway can be readily performed, an intravenous induction of general anesthesia is performed with cricoid pressure, and a muscle relaxant with a short-onset time is administered. Succinylcholine has the fastest onset time of all neuromuscular relaxants and is the drug of choice. However, in some cases, succinylcholine is contraindicated and another neuromuscular blocker is chosen. Of the drugs listed, rocuronium is the best Pharmacology and Pharmacokinetics of Intravenous Drugs 77 choice because of its rapid onset. Although the onset time of other nondepolarizing neuromuscular relaxants can be sped up with priming (a technique in which 10% of the intubating dose is followed 2 to 4 minutes later with an intravenous induction of general anesthesia and the remaining 90% of the relaxant), rocuronium is fast enough without priming and much simpler to use. In patients who may be difficult to intubate, even with adequate muscle relaxation, an awake intubation should be strongly considered. d-Tubocurare should never have an intubating dose bolused because it causes significant histamine release, and it should be given incrementally over several minutes if used to intubate (Miller: Miller’s Anesthesia, ed 8, p 875).
238. The neuromuscular effects of an intubation dose of vecuronium are terminated by A. Diffusion from the neuromuscular junction back into the plasma B. Nonspecific plasma cholinesterases C. The kidneys D. The liver
238. (A) The effects of nondepolarizing neuromuscular drugs are based on the drug being at the receptor. After intravenous injection of a muscle relaxant, plasma drug concentration immediately starts to decrease. To produce paralysis, the drug must diffuse from the plasma to the neuromuscular junction after injection and bind to the receptors. The drug effect is later terminated by diffusion of drug back into the plasma. Recovery of neuromuscular function occurs when the muscle relaxant diffuses from the neuromuscular junction back into the plasma to be metabolized and/or eliminated from the body (Miller: Miller’s Anesthesia, ed 8, p 871).
239. Respiratory depression produced by which of the following analgesics is not readily reversed by administration of naloxone? A. Meperidine B. Methadone C. Hydromorphone D. Buprenorphine
239. (D) Buprenorphine (Buprenex) is a mixed agonist-antagonist opioid with a very strong affinity for μ receptors. Because of its strong affinity (33 times greater than morphine) and slow dissociation from the receptors, it has a prolonged duration of effect (>8 hours) and shows resistance to reversal from naloxone. In rare cases of respiratory depression, reversal may not be achieved with high doses of naloxone (Miller: Miller’s Anesthesia, ed 8, p 904; Stoelting: Pharmacology and Physiology in Anesthetic Practice, ed 4, p 119).
240. Which of the following intravenous anesthetic agents is associated with the highest incidence of nausea and vomiting? A. Midazolam B. Etomidate C. Ketamine D. Propofol
240. (B) Nausea and vomiting may be associated with any of the drugs listed. Propofol, and perhaps midazolam, may actually be protective in some patients. Of the listed drugs in this question, etomidate has the highest incidence of nausea and vomiting with some reporting an incidence as high as 40% (Barash: Clinical Anesthesia, ed 7, p 489; Miller: Basics of Anesthesia, ed 6, pp 108–112).
241. If naloxone were administered to a patient who is receiving ketorolac for postoperative pain, the most likely result would be A. Bradycardia B. Hypotension C. Pain D. None of the above
241. (D) Naloxone is a pure opioid antagonist (affinity but no intrinsic activity) at all opioid receptors. It mainly is used to reverse narcotic-induced toxicity. In large doses, naloxone may reverse the effects of endogenous opioids that are elevated in conditions of stress (e.g., shock or stroke). Naloxone has no effect on NSAIDs (e.g., ketorolac) (Miller: Miller’s Anesthesia ed 8, pp 905–906).
242. Which drug produces strong pulmonary arterial dilation with the least amount of systemic artery dilation? A. Nitroprusside B. Prostaglandin E1 C. Phentolamine D. Nitric oxide
242. (D) Nitric oxide, nitroglycerin, nitroprusside, phentolamine, amrinone, milrinone, and prostaglandin E all have a vasodilatory effect on the pulmonary arterial tree. However, only nitric oxide has basically no effect on the systemic circulation. The following table compares the relative efficacy of various intravenous vasodilators (Miller: Miller’s Anesthesia, ed 8, pp 3084–3088). RELATIVE EFFICACY OF INTRAVENOUS VASODILATORS ON HEMODYNAMIC VARIABLES Dilation Venous Pulmonary Arterial Systemic Arterial Cardiac Output Nitric oxide 0 +++ 0 ± Nitroglycerin IV +++ + + I, D* Nitroprusside +++ +++ +++ I, D* Phentolamine + + +++ I Hydralazine 0 ? +++ I Nicardipine 0 ? +++ I Amrinone† + + + I Milrinone† + + + I Prostaglandin E1‡ + +++ +++ I, D* 0, none; ±, small and variable; +, mild; +++, strongest effect of that particular drug; D, decrease; I, increase. *Effect on cardiac output depends on net balance of effects on preload, afterload, and myocardial oxygenation. †Amrinone and milrinone are inodilators (they have inotropic plus vasodilating effects). ‡Prostaglandin E1 almost always requires left atrial infusion of norepinephrine to sustain adequate systemic blood pressure. From Stoelting RK, Miller RD: Basics of Anesthesia, ed 5, Philadelphia, Churchill Livingstone, 2006, p 1794. 78 Part 1 Basic Sciences
243. The action of succinylcholine at the neuromuscular junction is terminated by which mechanism? A. Hydrolysis by pseudocholinesterase B. Diffusion into extracellular fluid C. Reuptake into nerve tissue D. Reuptake into muscle tissue Pharmacology and Pharmacokinetics of Intravenous Drugs 57
243. (B) Succinylcholine is rapidly metabolized in the blood by pseudocholinesterase (plasma cholinesterase). This accounts for the large dose required to facilitate intubation. Because pseudocholinesterase is not present at the neuromuscular junction, succinylcholine’s action is terminated after it diffuses into the extracellular fluid (Miller: Miller’s Anesthesia, ed 8, p 961).
244. The LEAST likely side effect of dexmedetomidine in a healthy patient is A. Respiratory arrest B. Bradycardia C. Sinus arrest D. Hypotension
244. (A) Dexmedetomidine is a highly selective α2-adrenergic agonist that is mainly used for sedation. It has a rapid onset of action (<5 minutes) and a peak effect in about 15 minutes. In normovolemic healthy patients, the cardiovascular effects include a decrease in heart rate and cardiac output. The heart-rate changes can be profound, and occasionally sinus arrest may develop. After an IV injection, the BP initially increases (due to peripheral α stimulation), then within 15 minutes returns to normal and is followed by an approximately 15% decrease in BP within an hour. This is related to its CNS α-adrenergic stimulation overriding the peripheral effects. Respiratory changes are minimal, provided that excessive sedation does not produce obstructive apnea. At clinical doses of 1 to 2 μg/kg/min only a mild decrease in tidal volume (Vt) is seen, with no change in respiratory rate. With high doses, the Paco2 may increase about 20% due to a decrease in Vt as the respiratory rate increases (Miller: Miller’s Anesthesia, ed 8, pp 834–838).
245. The advantage of fospropofol (Lusedra) over propofol is the absence of A. Pain on injection B. Risk of hypertriglyceridemia C. Risk of infection, sepsis, or both D. All of the above
245. (D) Fospropofol (Lusedra), approved in December 2008 for monitored anesthesia care, is a prodrug of propofol that, after IV infusion, is rapidly converted into propofol. Because it is water soluble, the problems associated with a lipid vehicle (pain on injection, risk of hypertriglyceridemia, risk of pulmonary embolism, risk of sepsis) are absent (Eisai Corporation product information; Miller: Miller’s Anesthesia, ed 8, pp 822–823).
246. Which of the following features of chronic morphine therapy is NOT subject to tolerance? A. Analgesia B. Respiratory depression C. Constipation D. All are subject to tolerance
246. (C) In addition to analgesia, respiratory depression, nausea, and euphoria, tolerance to sedation with chronic analgesic therapy with morphine will develop after 2 to 3 weeks of treatment. Miosis and constipation occur with narcotic administration regardless of length of therapy. The concept of tolerance is not applicable to these two side effects (Butterworth: Morgan & Mikhail’s Clinical Anesthesiology, ed 5, p 195).
247. A 78-year-old woman with a history of reactive airways disease takes cimetidine (Tagamet) 400 mg at night. An additional dose is given IV 30 minutes before induction of anesthesia for an exploratory laparotomy. Possible side effects associated with this drug include all of the following EXCEPT A. Bradycardia B. Delayed awakening C. Confusion D. Increased metabolism of diazepam
247. (D) H2-receptor antagonists (e.g., cimetidine, ranitidine, famotidine, nizatidine) can be used preoperatively to increase gastric fluid pH before induction of anesthesia. Elevation of gastric fluid pH (above 2.5) is desirable to decrease the incidence and severity of lung damage if aspiration of gastric contents occurs. H2-receptor antagonists are not uncommonly used as a premedication for parturients, patients with symptomatic gastroesophageal reflux, and obese patients (who tend to have very acidic gastric fluid compared to nonobese patients). H2-receptor antagonists, in contrast to metoclopramide, have no effect on lower esophageal sphincter tone, intestinal motility, or gastric emptying. Although the incidence of side effects is low, side effects occasionally may develop in patients, especially when the drug is administered intravenously and when the drugs are administered to the elderly or to patients with hepatic or renal dysfunction. Bradycardia may develop and may be related to the effects on cardiac H2 receptors. Reversible elevation of plasma aminotransaminase enzymes may occur. H2-receptor antagonists cross the blood-brain barrier and may lead to mental confusion or delayed awakening. Cimetidine impairs the metabolism of drugs such as lidocaine, propranolol, and diazepam. This impairment may be related to the binding of cimetidine to the cytochrome P-450 enzymes (Barash: Clinical Anesthesia, ed 7, p 602).
248. Intraoperative allergic reactions are LEAST common after patient exposure to A. Ketamine B. Latex C. Muscle relaxants D. Hydroxyethyl starch
248. (A) Drug sensitivity has been reported in about 3% to 4% of anesthetic-related deaths. Allergic drug reactions have been reported to occur with most drugs administered during anesthesia, with the exception of ketamine and the benzodiazepines. Although most drug-induced allergic reactions occur within 5 to 10 minutes of exposure, reactions to latex products may take longer than 30 minutes to develop (Hines: Stoelting’s Anesthesia and Co-Existing Disease, ed 6, pp 525–529).
249. Which of the following medications would be useful in the definitive treatment of sarin nerve gas poisoning? A. Sodium nitroprusside B. Methylene blue C. Atropine D. All the above are useful
249. (C) Atropine is administered in doses of 2 to 6 mg and is repeated every 5 to 10 minutes until secretions begin to decrease. In most cases, 2 mg every 8 hours is needed. However, doses of 15 to 20 mg are not uncommon and occasionally doses over 1000 mg have been needed. Pralidoxime 600 mg removes the organophosphate compounds from acetylcholinesterase and is often used in conjunction with atropine. Benzodiazepines are often administered to counter the effects of the nerve gases on the GABA system (Barash: Clinical Anesthesiology, ed 7, p 1541). Pharmacology and Pharmacokinetics of Intravenous Drugs 79
250. Alfentanil A. Has a more rapid onset of action compared to fentanyl B. Has a longer duration of action compared with fentanyl C. Is 250 times more potent than fentanyl D. Is excreted unchanged in the urine
250. (A) Alfentanil (a fentanyl analog) is less potent (1/5 to 1/10), has a more rapid onset (within 1.5 minutes), and has a shorter duration of action than fentanyl. The brief duration of action of alfentanil is a result of redistribution to inactive tissue sites and its rapid hepatic metabolism (96% cleared within 1 hour). Renal failure does not alter the clearance of alfentanil (Miller: Basics of Anesthesia, ed 6, p 119, Figure 10-3; Miller: Miller’s Anesthesia, ed 8, p 887; Butterworth: Morgan & Mikhail’s Clinical Anesthesiology, ed 5, p 196).
251. Which of the following medications is NOT useful in the immediate management of status asthmaticus? A. Terbutaline B. Subcutaneous (SQ) epinephrine C. Magnesium sulfate D. Cromolyn
251. (D) Asthma is an inflammatory illness associated with bronchial hyper-reactivity and bronchospasm. Medications effective in the management of acute exacerbations of bronchial asthma include the rapid-onset inhaled β2-adrenergic receptor agonists (e.g., albuterol, pirbuterol, terbutaline), anticholinergic drugs (e.g., inhaled ipratropium), and IV corticosteroids. In an acute attack, ipratropium (slower in onset than β2-adrenergic receptor agonists) can be effective when used in combination with the rapid-onset β2 agonists. When unresolving bronchospasm occurs and is considered life threatening, the diagnosis of status asthmaticus is made. Although treatment often starts with β2 agonists (two to four puffs every 15-20 minutes), when alveolar ventilation is reduced, inhaled agents may not be successful. In this case, SQ epinephrine (adult dose of 0.2 to 1 mg or 0.2 to 1 mL of 1:1000 solution) can be given. Corticosteroids enhance and prolong the response to β2 agonists, and, in status asthmaticus, IV corticosteroids such as cortisol (Solu-Cortef) 2 mg/kg IV bolus followed by 0.5 mg/ kg/hr, or methylprednisolone (Solu-Medrol) 60 to 125 mg every 6 hours, are administered early in the treatment (but may take several hours to work). Supplemental oxygen is given to keep the oxygen saturation greater than 90%. Because Heliox (70% helium and 30% oxygen) is one third the density of oxygen, it can be tried. IV terbutaline starting at a rate of 0.1 μg/kg/min and increased until improvement is seen or significant tachycardia develops may be useful. Magnesium sulfate at a dose of 25 to 40 mg/kg (maximum of 2 g) administered over 20 minutes has been used. Broad-spectrum antibiotics are also started. In severe cases where fatigue sets in and the Paco2 is rising (e.g., >70-80 mm Hg), general anesthesia with mechanical ventilation may be needed. The volatile anesthetics such as isoflurane, halothane, or sevoflurane can be used not only to sedate but also to relax the smooth muscle in the constricted airways. Cromolyn, however, does not relieve bronchospasm. Cromolyn is used prophylactically because it inhibits antigen-induced release of histamine and other autacoids, such as leukotrienes, from mast cells. Aminophylline once was widely used to treat acute asthma but is rarely used today because it adds little to β2-agonist activity and has significant side effects (Hardman: Goodman & Gilman’s The Pharmacological Basis of Therapeutics, ed 10, pp 733–749; Hines: Stoelting’s Anesthesia and Co-Existing Disease, ed 6, pp 185–186).
252. Clonidine A. Is an α2 blocker B. Increases CNS sympathetic response to painful stimuli C. Can be given orally as well as intravenously, but not epidurally or intrathecally D. Decreases postanesthetic shivering
252. (D) Clonidine is an α2-adrenergic agonist. Unlike many peripherally acting antihypertensive drugs (e.g., guanethidine, propranolol, captopril), clonidine primarily stimulates central adrenergic receptors and decreases the sympathetic response. As with other drugs that affect the central release of catecholamines, clonidine not only reduces anesthetic requirements (as represented by a decrease in MAC) but also decreases extremes in arterial BP during anesthesia. Clonidine has analgesic properties and reduces the requirements for opioids. Clonidine has been given orally, intravenously, epidurally, intrathecally, and in peripheral nerve blocks and potentiates the analgesic effect of local anesthetics. α2-Adrenergic agonists can reduce the muscle rigidity seen with the administration of narcotics and can be used to decrease postanesthetic shivering. Patients chronically taking clonidine should not have it discontinued before surgery and should keep taking clonidine to prevent clonidine withdrawal and hypertensive crisis (Miller: Miller’s Anesthesia, ed 8, pp 368, 1218, 1632).
253. The plasma half-time of which of the following drugs is prolonged in patients with end-stage cirrhotic liver disease? A. Diazepam B. Pancuronium C. Alfentanil D. All are prolonged
253. (D) Chronic liver disease may interfere with the metabolism of drugs because of the decreased number of enzyme- containing hepatocytes, decreased hepatic blood flow, or both. Prolonged elimination half-times for morphine, alfentanil, diazepam, lidocaine, pancuronium, and, to a lesser extent, vecuronium have been demonstrated in patients with cirrhosis of the liver. In addition, severe liver disease may decrease the production of cholinesterase (pseudocholinesterase) enzyme, which is necessary for the hydrolysis of ester linkages in drugs such as succinylcholine, and the ester local anesthetics such as procaine (Miller: Basics of Anesthesia, ed 6, p 456).
254. A 24-year-old, 100-kg patient is brought to the emergency room by the fire department after suffering smoke inhalation and third-degree burns on the abdomen, chest, and thighs 30 minutes earlier. The best muscle relaxant choice for the most rapid intubation would be A. 2 mg vecuronium followed by succinylcholine B. 1 mg of vecuronium, then 2 to 4 minutes later, 9 mg vecuronium C. Rocuronium D. Succinylcholine
254. (D) Succinylcholine is the drug of choice (unless contraindicated) when rapid-sequence tracheal intubation is needed. Although hyperkalemic cardiac arrest is a complication of succinylcholine administrations to patients who have sustained burns (as well as crush injuries, spinal cord trauma, or other denervation 80 Part 1 Basic Sciences injuries, chronic illness polyneuropathy, and chronic illness myopathy), the susceptibility for hyperkalemia after a burn injury peaks at 7 to 10 days but may begin as early as 2 days after sustaining a thermal injury. The first 24 hours after the injury are considered safe. Adding a defasciculating dose of a nondepolarizing neuromuscular blocking drug before succinylcholine use to the regimen would slow down achievement of paralysis. Although the “priming” technique of giving 10% of the intubating dose followed 2 to 4 minutes later by the rest of the intubating dose has been used to speed conditions for intubation, it is still slower than succinylcholine, and this technique is rarely used because rocuronium (which provides the most rapid intubating conditions among the nondepolarizing neuromuscular blocking drugs and is a close second behind succinylcholine) is available. An intubating dose of d-tubocurarine should never be given as a bolus because of its moderate histamine release (Miller: Basics of Anesthesia, ed 6, pp 148–149).
255. Clonidine is useful in each of the following applications EXCEPT A. Reducing BP with pheochromocytoma B. Treatment of postoperative shivering C. Protection against perioperative myocardial ischemia D. As an agent for prolonging a bupivacaine spinal
255. (A) Clonidine, a centrally acting α-agonist, decreases sympathetic nervous system outflow and decreases plasma catecholamine concentrations in normal patients, but it has no effect in patients with pheochromocytomas. It is used as an antihypertensive agent for treating essential hypertension, an analgesic when injected epidurally or into the subarachnoid space alone, a drug that prolongs the effect of regional local anesthetics, a drug that can be used to stop shivering (75 μg IV), a drug that can help protect against perioperative myocardial ischemia (when given preoperatively and typically for 4 days after surgery), and a drug that can help decrease the symptoms of narcotic and alcohol withdrawal (Barash: Clinical Anesthesia, ed 7, p 392; Miller: Miller’s Anesthesia, ed 8, p 473; Hines: Stoelting’s Anesthesia and Co-Existing Disease, ed 6, p 394).
256. A 79-year-old man is brought to the operating room for elective repair of bilateral inguinal hernias. The patient has a history of awareness during general anesthesia and refuses regional anesthesia. The patient is preoxygenated before induction of general anesthesia; 5 mg of midazolam and 250 mg of fentanyl are administered. One minute later the patient loses consciousness and chest wall stiffness develops to the extent that positive-pressure ventilation is very difficult. The most appropriate therapy for reversal of chest wall stiffness at this point could include A. Flumazenil B. Naloxone C. Succinylcholine D. Albuterol 58 Part 1 Basic Sciences
256. (C) Skeletal muscle spasm, particularly of the thoracoabdominal muscles (“stiff chest” syndrome), may occur when large doses of opioids are given rapidly. This may be significant enough to prevent adequate ventilation. Although the administration of a muscle relaxant or an opioid antagonist such as naloxone will terminate the skeletal muscle rigidity, reversing the narcotic effect may not be desirable if surgery is needed (Miller: Basics of Anesthesia, ed 6, p 121).
257. Respiratory depression is LEAST after the induction dose of which of the following drugs? A. Etomidate B. Ketamine C. Fentanyl D. Propofol
257. (B) One of the advantages of ketamine is the minimal effect on respirations. After the intravenous induction dose of 2 mg/kg, general anesthesia is induced within 30 to 60 seconds with, at most, a transient decrease in respirations (Paco2 rarely increases more than 3 mm Hg). With unusually high doses, or if opioids are also administered, apnea can occur (Miller: Basics of Anesthesia, ed 6, p 108).
258. A 64-year-old man with colon cancer is anesthetized for hepatic resection of liver metastases. Medical history is significant for ileal conduit surgery for bladder cancer, diabetes treated with glyburide, 50 packs per year smoking history, and family history of malignant hyperthermia. Anesthesia is provided with morphine, midazolam, oxygen, and a propofol infusion. After a 3-unit packed red blood cell (RBC) transfusion and 8 hours of surgery, the following blood gas values are recorded: pH 7.2, CO2 34, [HCO3 –] 14, base deficit −13, [Na+] 135, [K+] 5, [Cl–] 95, glucose 240 mg/dL. The most likely cause of this patient’s acidosis is A. Excessive infusion of normal saline B. Renal tubular acidosis C. Propofol infusion syndrome D. Diabetic ketoacidosis
258. (C) This patient has a partially compensated metabolic acidosis. Metabolic acidosis is commonly divided into those with a normal ion gap, also called hyperchloremic metabolic acidosis (bicarbonate loss is counterbalanced by an increase in chloride levels), and those with a high anion gap. The anion gap can be calculated by determining the difference between the sodium concentration and the sum of the chloride and bicarbonate concentrations (i.e., [Na+] − [Cl–] + [HCO3 –]) and is normally 8 to 14 mEq/L. In this case, the anion gap is 135 − [95 + 14] = 26. This patient, therefore, has a high anion gap acidosis. This question has two forms of acidosis that have a high anion gap: diabetic ketoacidosis (DKA) and propofol infusion syndrome, which causes a lactic acidosis. Because this patient is a type 2 (non–insulin-dependent) diabetic, DKA does not occur and the cause must be propofol infusion syndrome (Hines: Stoelting’s Anesthesia and Co-Existing Disease, ed 6, pp 372–373; Miller: Miller’s Anesthesia, ed 8, p 832).
259. Treatment of neuroleptic malignant syndrome may be carried out with administration of the following drugs EXCEPT A. Amantadine B. Dantrolene C. Bromocriptine D. Physostigmine
259. (D) Neuroleptic malignant syndrome (NMS) can be seen in up to 1% of patients treated with antipsychotic drugs. The syndrome has many features that resemble the condition malignant hyperthermia, including increased metabolism, tachycardia, muscle rigidity, rhabdomyolysis, fever, and acidosis. The mortality rate may be 20% to 30%. There are many differences between NMS and malignant hyperthermia. NMS is not inherited and usually takes 24 to 72 hours to develop after the use of neuroleptic drugs (e.g., phenothiazines, haloperidol), whereas malignant hyperthermia presents more acutely. Stopping the antipsychotic medication is obviously necessary. Because dopamine depletion appears to play a role in causing NMS, the dopamine agonists bromocriptine and amantadine appear useful in the treatment. Abrupt withdrawal of levodopa may also cause this syndrome. Succinylcholine and volatile anesthetics, which are known triggers for malignant hyperthermia, are not triggers for NMS. Dantrolene has been used to treat this condition (Stoelting: Pharmacology and Physiology in Anesthetic Practice, ed 4, pp 412–413). Pharmacology and Pharmacokinetics of Intravenous Drugs 81
260. A patient with a normal quantity of pseudocholinesterase (plasma cholinesterase) has a dibucaine number of 57. A 1 mg/kg dose of intravenous succinylcholine would likely result in A. Hyperkalemic cardiac arrest B. Paralysis lasting 5 to 10 minutes C. Paralysis lasting 20 to 30 minutes D. Paralysis lasting more than 1 to 3 hours
260. (C) Normal pseudocholinesterase is inhibited 80% by dibucaine (dibucaine number of 80), whereas patients with atypical cholinesterase show only 20% inhibition (dibucaine number of 20). Patients who are heterozygous for atypical pseudocholinesterase (as in this case) have intermediate dibucaine numbers ranging from 50% to 60%. Succinylcholine paralysis after an intubating dose of 1 mg/kg lasts up to 10 minutes with normal pseudocholinesterase, up to 30 minutes in patients with the atypical heterozygous pseudocholinesterase, and may persist for 3 hours or longer in patients who have atypical cholinesterase paralysis. See also Question 200 (Miller: Basics of Anesthesia, ed 6, pp 148–149).
261. Cyanide toxicity may be treated with all of the following drugs EXCEPT A. Sodium nitrite B. Hydroxocobalamin C. Sodium thiosulfate D. Methylene blue
261. (D) Cyanide (hydrocyanic acid [HCN], prussic acid) is a rapidly acting poison. Cyanide is commercially used as a pesticide, but it can be released as a gas from burning nitrogen-containing plastics. Sodium nitroprusside (SNP) is metabolized to cyanide and nitric oxide. The cyanide produced from SNP usually is rapidly metabolized to relatively nontoxic thiocyanate (SCN−), which is excreted into the urine. Although rare, cyanide and/or thiocyanate toxicity can develop in patients receiving prolonged high-dose infusions of nitroprusside. Cyanide binds to iron in the ferric state and inhibits cellular respiration. This produces severe lactic acidosis and cytotoxic hypoxia. Because oxygen is not used well, the venous blood is well oxygenated (elevated central venous oxygen levels and patients are not cyanotic). Treatment (adult doses in parenthesis) can include sodium nitrite (NaNO2—300 mg IV over 10 minutes), amyl nitrite (inhalation), sodium thiosulfate (12.5 g IV over 10 minutes), and hydroxocobalamin (5-10 g IV over 20 minutes). Nitrite converts hemoglobin to methemoglobin, which competes with cytochrome oxidase for the cyanide ion forming cyanmethemoglobin. Nitrite can be administered IV as sodium nitrite or by inhalation with amyl nitrite. Sodium thiosulfate (Na2S2O3), the preferred drug, is a sulfur donor that converts cyanide to thiocyanate. Hydroxocobalamin combines with cyanide to form cyanocobalamin or vitamin B12. Methylene blue is not an antidote for cyanide toxicity and can complicate therapy by converting methemoglobin back to hemoglobin and releasing free cyanide. Although oxygen alone (even under hyperbaric conditions) has little benefit, it should be used because it dramatically potentiates the activity of thiosulfate and nitrites (Barash: Clinical Anesthesia, ed 7, pp 403–404; Brunton: Goodman & Gilman’s The Pharmacological Basis of Therapeutics, ed 12, pp 782–783, 793–796; Miller: Miller’s Anesthesia, ed 8, pp 2501–2503).
262. A prolonged neuromuscular block with succinylcholine can be seen in all of the following patients EXCEPT those A. Chronically exposed to malathion B. Treated with echothiophate for glaucoma C. Treated with cyclophosphamide for metastatic cancer D. Having a C5 isoenzyme variant
262. (D) The duration of neuromuscular block by succinylcholine can be markedly prolonged when the total amount of plasma cholinesterase is very low, the amount is normal but of an abnormal type (i.e., atypical plasma cholinesterase), or an anticholinesterase drug (e.g., neostigmine, echothiophate, or the organophosphate insecticide malathion) is administered. To evaluate a prolonged response to succinylcholine, one needs to evaluate both the total amount of cholinesterase (i.e., quantitative test) and the type of cholinesterase (i.e., qualitative test). Atypical plasma cholinesterase is an inherited disorder that occurs in approximately 1 of every 480 patients with heterozygous genome and in approximately 1 of 3200 patients with homozygous genome. The local anesthetic dibucaine can inhibit normal plasma cholinesterase enzyme better than an abnormal enzyme. In patients with normal plasma cholinesterase, the dibucaine inhibition test reports a number around 80 or produces 80% inhibition. Heterozygotes have a dibucaine number of around 50, and patients who are homozygous for the atypical plasma cholinesterase have a number around 20. Total plasma cholinesterase levels can be reduced with decreased production, as occurs with severe chronic liver disease or with the use of some chemotherapeutic drugs (e.g., cyclophosphamide). The dibucaine number is normal when the total plasma cholinesterase levels are reduced, as well as after the use of anticholinesterase drugs. Patients with a C5 isoenzyme variant have increased plasma cholinesterase activity, a more rapid breakdown of succinylcholine, and a shorter duration of action (Brunton: Goodman & Gilman’s The Pharmacological Basis of Therapeutics, ed 12, p 243; Butterworth: Morgan & Mikhail’s Clinical Anesthesiology, ed 5, pp 205–207; Miller: Basics of Anesthesia, ed 6, pp 76, 148–149; Stoelting: Pharmacology and Physiology in Anesthetic Practice, ed 4, pp 216–220).
263. Which of the following statements concerning midazolam is FALSE? A. Midazolam has greater amnestic than sedative properties B. Its breakdown is inhibited by cimetidine C. It produces retrograde amnesia D. It facilitates the actions of the inhibitory neurotransmitter γ-aminobutyric acid (GABA) in the CNS
263. (C) Benzodiazepines are drugs that have the chemical structure of a benzene ring attached to a seven-member diazepine ring. Midazolam, lorazepam, oxazepam, and diazepam are benzodiazepine agonists and flumazenil is an antagonist. Benzodiazepine agonists are all sedatives and possess a number of favorable pharmacologic characteristics, including production of sedation, anxiolysis, anterograde amnesia (acquisition of new information), and anticonvulsant activity. The amnestic properties are greater than the sedative properties, which 82 Part 1 Basic Sciences is why patients sometimes forget what you tell them after the benzodiazepine is given, despite their having what appears to be a lucid discussion with you. They do not produce retrograde amnesia (stored information). They rarely cause significant respiratory or cardiovascular depression and rarely are associated with the development of significant tolerance or physical dependence. The agonist actions of benzodiazepines most likely reflect the ability of these drugs to facilitate the inhibitory neurotransmitter GABA actions in the CNS. Midazolam and diazepam undergo oxidative metabolism, and their metabolites are conjugated with glucuronide before renal excretion. Cimetidine inhibits oxidative metabolism and may prolong the duration of these drugs. Lorazepam and oxazepam primarily undergo conjugation with glucuronic acid, which is not influenced by cimetidine usage or alterations in hepatic function (Brunton: Goodman & Gilman’s The Pharmacological Basis of Therapeutics, ed 12, pp 458–467; Butterworth: Morgan & Mikhail’s Clinical Anesthesiology, ed 5, pp 179–181; Miller: Basics of Anesthesia, ed 6, pp 106–109; Stoelting: Pharmacology and Physiology in Anesthetic Practice, ed 4, pp 140–153).
264. After a 2-hour vertical gastric banding procedure under desflurane, oxygen, and remifentanil anesthesia, the trocar is removed and the wound is closed. Upon emergence, the most likely scenario is A. Adequate analgesia for 2 hours B. Delayed emergence from narcotic C. Pain D. Respiratory depression in postanesthesia care unit (PACU)
264. (C) Remifentanil is an ultrashort-acting opioid most commonly administered by an IV infusion. Its short duration of action is due to its ester linkage, which allows for rapid breakdown by nonspecific plasma and tissue esterases (primarily within erythrocytes). Its metabolism is not significantly influenced by renal failure, hepatic failure, or pseudocholinesterase levels (because it is not metabolized to any significant extent by plasma pseudocholinesterase). The clinical elimination half-time is less than 6 minutes. For monitored anesthesia care sedation after 2 mg of midazolam, an infusion rate of 0.05 to 0.1 μg/kg/min is used in healthy adults. For analgesia during general anesthesia with controlled respirations, a rate of 0.1 to 1.0 μg/ kg/min is commonly used. A loading dose of 1 μg/kg of remifentanil (or 0.5 μg/kg, if a benzodiazepine was also given) can be given IV over 60 to 90 seconds before starting the infusion. Although it effectively suppresses autonomic and hemodynamic responses to painful stimuli and decreases respirations as well, its rapid dissipation of opioid effect produces rapid onset of postoperative pain (in painful surgical operations), unless other analgesics are administered for postoperative pain before stopping the infusion (Barash: Clinical Anesthesia, ed 7, pp 514–515, 832–834; Miller: Miller’s Anesthesia, ed 8, pp 888–897).
265. An oral surgeon is about to perform a full mouth extraction on a 70-kg, 63-year-old man under conscious sedation. What is the maximum dose of lidocaine with epinephrine that he can safely infiltrate? A. 200 mg B. 300 mg C. 400 mg D. 500 mg
265. (D) The maximum recommended single dose of lidocaine given by infiltration is 300 mg of lidocaine without epinephrine and 500 mg of lidocaine with epinephrine. Careful injection in the mouth is recommended due to the vascular nature of that area (Barash: Clinical Anesthesia, ed 7, p 572; Miller: Miller’s Anesthesia, ed 8, p 1041).
266. Postanesthetic shivering can be treated with all of the following EXCEPT A. Naloxone B. Physostigmine C. Magnesium sulfate D. Dexmedetomidine
266. (A) Postoperative shivering can be caused by many factors, including hypothermia, transfusion reactions, and pain, as well as anesthetics. It is uncomfortable for patients and can make monitoring more difficult, but it also can lead to significant increases in oxygen consumption (up to 200%). The exact etiology in many cases is unclear, but, after routine skin surface warming, pharmacologic treatment may be needed. Clonidine, dexmedetomidine, propofol, ketanserin, tramadol, physostigmine, magnesium sulfate, and narcotics (especially meperidine) have been used. Naloxone use may increase pain and does not help decrease shivering (Barash: Clinical Anesthesia, ed 7, p 1574; Miller: Miller’s Anesthesia, ed 8, pp 1636–1638).
267. The main disadvantage of Sugammadex (ORG 25969) compared with neostigmine is A. Recurarization B. Contraindicated with renal failure C. Not effective with benzylisoquinolinium relaxants D. High incidence of allergic reactions
267. (C) Sugammadex is a cyclodextrin (cyclic oligosaccharide) compound that encapsulates nondepolarizing steroidal muscle relaxants (rocuronium > vecuronium >> pancuronium) and produces rapid reversal of profound block (e.g., reversal of 0.6 mg/kg rocuronium in 3 minutes). Because it has no effect on acetylcholinesterase, there is no need to combine it with the anticholinergics atropine or glycopyrrolate. It works only with steroidal muscle relaxants and has no effect on reversing the benzylisoquinolinium relaxants (e.g., atracurium, cisatracurium, doxacurium, d-tubocurarine). There appear to be no cardiovascular effects with sugammadex. It is available only outside the United States at present (Miller: Basics of Anesthesia, ed 6, p 159; Miller: Miller’s Anesthesia, ed 8, p 965).
268. Which of the biologic substances listed below is by itself the greatest determinant of serum osmolality? A. AVP (arginine vasopressin) B. Angiotensin I C. Aldosterone D. Renal prostaglandins (PGE2)
268. (A) Arginine vasopressin (AVP), also called antidiuretic hormone (ADH), has many actions, but its primary role involves controlling serum osmolality by regulating diuresis. AVP is released by the hypothalamus and directly causes the collecting tubules in the kidney to increase water permeability and reabsorption. This increases blood volume and lowers serum osmolality. Below a serum osmolality of 280 mOsm/kg, AVP is barely detectable; however, when the osmolality is greater than 290 mOsm/kg, AVP is maximally secreted. AVP is also secreted when the intravascular volume is detected to be low (e.g., hemorrhage, heart failure, hepatic cirrhosis, and adrenal insufficiency). Angiotensin I is converted to angiotensin II, Pharmacology and Pharmacokinetics of Intravenous Drugs 83 which is a potent vasoconstrictor and increases aldosterone secretion from the adrenal cortex. Aldosterone is a mineralocorticoid and is involved in sodium reabsorption and potassium excretion in the renal tubules. Aldosterone secretion is stimulated by hypovolemic states. Renal prostaglandins are released from the kidney by sympathetic stimulation or by angiotensin II and help modulate the effects of AVP (Butterworth: Morgan & Mikhail’s Clinical Anesthesiology, ed 5, p 738; Brunton: Goodman & Gilman’s The Pharmacological Basis of Therapeutics, ed 12, pp 671–704, 721–730; Miller: Basics of Anesthesia, ed 6, pp 449–450).
269. Above which infusion rate does cyanide toxicity become a concern in a healthy adult receiving sodium nitroprusside? A. 0.5 μg/kg/min B. 2 μg/kg/min C. 10 μg/kg/min D. 20 μg/kg/min Pharmacology and Pharmacokinetics of Intravenous Drugs 59
269. (B) Sodium nitroprusside (SNP) is a rapid-acting, direct-acting peripheral vasodilator that is composed of five cyanide moieties for every NO (nitric oxide) moiety. Sodium nitroprusside undergoes rapid metabolism to release NO as the active ingredient. Healthy adults can easily eliminate the cyanide produced during SNP rates of less than 2 μg/kg/min. Above 2 μg/kg/min and especially if the infusion rate is greater than 10 μg/kg/min for 10 minutes, one should be concerned about cyanide toxicity. An early sign of cyanide toxicity is resistance to the hypotensive effects of SNP infusion, especially when the rate is less than 2 μg/kg/min. Other signs include metabolic acidosis and an elevation of mixed venous Po2 values (Butterworth: Morgan & Mikhail’s Clinical Anesthesiology, ed 5, p 258).
270. Important interactions involving chlorpromazine include all of the following EXCEPT A. Potentiation of the depressant effects of narcotics B. Lowering of the seizure threshold C. Prolongation of the QT interval D. Potentiation of neuromuscular blockade
270. (D) Phenothiazines, such as chlorpromazine (Thorazine), are effective antipsychotic (neuroleptic) drugs that block D2 dopaminergic receptors in the brain. Extrapyramidal effects are not uncommon with these drugs. They also possess antiemetic effects. Phenothiazines with low potency, such as chlorpromazine, have prominent sedative effects, which gradually decrease with treatment. The effects of CNS depressants (e.g., narcotics and barbiturates) are enhanced by concomitant administration of phenothiazines. Lowering the seizure threshold is more common with aliphatic phenothiazines with low potency (e.g., chlorpromazine) compared with piperazine phenothiazines. These drugs are associated with cholestatic jaundice, impotence, dystonia, and photosensitivity. Electrocardiographic abnormalities, such as prolongation of the QT or PR intervals, blunting of T waves, depression of the ST segment, and, on rare occasions, premature ventricular contractions and torsades de pointes, are seen. The antihypertensive effects of guanethidine and guanadrel are blocked by phenothiazines. These drugs have no effect on neuromuscular blockade (Miller: Miller’s Anesthesia, ed 8, p 1219; Hemmings: Pharmacology and Physiology for Anesthesia, pp 189–192).
271. Amrinone A. Is a positive inotropic drug B. Is antagonized by esmolol C. Is a vasoconstrictor D. All the above
271. (A) Amrinone is a noncatecholamine, nonglycoside cardiac inotropic drug that works as a selective phosphodiesterase III (PDE III) inhibitor. Amrinone increases cyclic adenosine monophosphate (cAMP) levels by decreasing cAMP breakdown in the myocardium and vascular smooth muscle. Because the actions of PDE III inhibitors work by a different mechanism than catecholamines (cAMP levels are increased by β-adrenergic receptor stimulation), amrinone can work in the presence of β-blockade and in cases where patients become refractory to catecholamine use. The catecholamine actions can be enhanced with PDE III inhibitors. Amrinone produces both positive inotropic and vasodilatory effects but has no antidysrhythmic effects (Hensley: A Practical Approach to Cardiac Anesthesia, ed 5, p 277).
272. Which statement concerning tricyclic antidepressants in patients receiving general anesthesia is TRUE? A. They should be discontinued 2 weeks before elective operations B. They may decrease the requirement for volatile anesthetics (decrease MAC) C. Meperidine may produce hyperpyrexia in patients taking tricyclic antidepressants D. They may exaggerate the response to ephedrine
272. (D) Tricyclic antidepressants often are administered as the initial treatment of mental depression; however, the more recently developed SSRIs are more frequently used because of fewer side effects. Tricyclic antidepressants work by inhibiting the reuptake of released norepinephrine (and serotonin) into the nerve endings. Although at one time it was recommended to stop tricyclic antidepressants before elective surgery, this has not been shown to be necessary. However, alterations in patient responses to some drugs should be anticipated. The increased availability of neurotransmitters in the CNS can result in increased anesthetic requirements (i.e., increased MAC). In addition, the increased availability of norepinephrine at postsynaptic receptors in the peripheral sympathetic nervous system can be responsible for an exaggerated BP response after administration of an indirect-acting vasopressor such as ephedrine. If a vasopressor is required, a direct-acting drug such as phenylephrine may be preferred. If hypertension occurs and requires treatment, deepening the anesthetic or adding a peripheral vasodilator such as nitroprusside may be needed. The potential for an exaggerated BP response (i.e., hypertensive crisis) is greatest during the acute treatment phase (the first 14-21 days). Chronic treatment is associated with down-regulation receptors and a decreased likelihood of an exaggerated BP response after administration of a sympathomimetic. Tricyclics have significant anticholinergic side effects (e.g., dry mouth, blurred vision, increased heart rate, urinary retention) and caution is especially important in elderly patients who may develop anticholinergic delirium despite the therapeutic doses administered. Caution 84 Part 1 Basic Sciences is advised with the use of meperidine in patients taking MAOIs (not tricyclic antidepressants) because of the possibility of inducing seizure, hyperpyrexia, or coma (Hines: Stoelting’s Anesthesia and Co-Existing Disease, ed 6, pp 535–536).
273. Which of the following types of insulin preparations has the fastest onset of action if administered subcutaneously? A. Glargine (Lantus) B. Lispro (Humalog) C. Regular (Humulin-R) D. NPH (Humulin-N)
273. (B) In normal nondiabetic patients, about 40 units of insulin are secreted every day. There are many SQ insulin preparations available. After SQ administration the onset of action is very rapid with Lispro and Aspart (15 minutes); rapid with Regular (30 minutes); intermediate with NPH or Lente (1-2 hours); and slow with Glargine (1.5 hours) and Ultralente (4-6 hours) (Hines: Stoelting’s Anesthesia and Co- Existing Disease, ed 6, pp 380–381). INSULIN PREPARATIONS Insulin Preparation Hours after Subcutaneous Administration Onset Peak Duration Very rapid acting Lispro (Humalog) 0.25 1-2 3-6 Aspart (NovoLog) 0.25 1-2 3-6 Rapid acting Regular (Humulin-R, Novolin-R) 0.5 2-4 5-8 Intermediate acting NPH (Humulin-N) 1-2 6-10 10-20 Lente 1-2 6-10 10-20 Long acting Glargine (Lantus) 1-2 Peakless About 24 Ultralente 4-6 8-20 24-48 From Hines RL: Stoelting’s Anesthesia and Co-Existing Disease, ed 5, Philadelphia, Saunders, 2008, p 371.
274. Which of the following mechanisms best explains the anticoagulative properties of tirofiban? A. Cyclooxygenase (COX) inhibition B. Interaction with von Willebrand factor (vWF) C. Interaction with antithrombin III D. Enhanced anti-Xa activity
274. (B) The GPIIb/IIIa receptor is specific for platelets. Platelet activation changes the shape of the receptor and increases its affinity for fibrinogen and vWF. GPIIb/IIIa receptor antagonists (e.g., tirofiban, abciximab, and eptifibatide) reversibly bind to the platelet GPIIb/IIIa receptor and block the binding of fibrinogen to platelets. They do not prolong the prothrombin time or the activated partial thromboplastin time. These drugs are administered intravenously as a bolus and then as a continuous infusion. The plasma half-life after a bolus intravenous injection is 2 hours for tirofiban, 2.5 hours for eptifibatide, and only 30 minutes for abciximab. The biologic half-life of these drugs is 4 to 8 hours for tirofiban, 4 to 6 hours for eptifibatide, and 12 to 24 hours for abciximab. The longer duration of action for abciximab is primarily due to clearance by the reticuloendothelial system (tirofiban and eptifibatide are cleared by the kidney) and its stronger affinity to the receptor (Hemmings: Pharmacology and Physiology for Anesthesia, pp 662–664; Miller: Basics of Anesthesia, ed 6, p 359; Miller: Miller’s Anesthesia, ed 8, p 1873).
275. The duration of action of remifentanil is attributable to which mode of metabolism? A. Spontaneous degradation in blood (Hofmann elimination) B. Hydrolysis by nonspecific plasma esterases C. Hydrolysis by pseudocholinesterase D. Rapid metabolism in the large intestine
275. (B) Remifentanil is rapidly hydrolyzed by nonspecific plasma and tissue esterases, making it ideal for an infusion where precise control is sought. The onset and offset of remifentanil is rapid (clinical half-time of <6 minutes). Because the activity of these nonspecific esterases is not usually affected by liver and renal failure, remifentanil is well suited for such patients (Miller: Basics of Anesthesia, ed 6, p 125).
276. Pain at the intravenous site is LEAST with which IV drug? A. Diazepam B. Etomidate C. Ketamine D. Propofol
276. (C) Pain with the intravenous injection is common with diazepam, etomidate, methohexital, and propofol. It is very rare after thiopental and ketamine (Miller: Basics of Anesthesia, ed 6, pp 102,109,112).
277. A 35-year-old patient with a history of grand mal seizures is anesthetized for thyroid biopsy under general anesthesia consisting of 4 mg midazolam with infusion of propofol (150 μg/kg/min) and remifentanil (1 μg/ kg/min). The patient takes phenytoin for control of seizures. After 30 minutes, the infusion is stopped and the patient is transported intubated to the recovery room where he is arousable, but not breathing. The most reasonable course of action would be A. Administer naloxone B. Administer flumazenil C. Administer naloxone and flumazenil D. Ventilate by hand
277. (D) Patients anesthetized with total intravenous anesthesia (TIVA), in this case consisting of midazolam, remifentanil, and propofol, sometimes require a few minutes to resume breathing after the infusions are stopped. Although it may seem appropriate to reverse this patient and avoid the need for hand ventilation, reversing benzodiazepines (midazolam) with flumazenil may precipitate seizures in epileptic patients, and, because remifentanil has such a short elimination half-life (<6 minutes), reversal with naloxone is not necessary. The patient needs a brief period to allow the propofol to wear off, during which hand or mechanical ventilation will be necessary (until the patient breathes spontaneously). Also, muscle weakness must be ruled out if a muscle relaxant has been used, and normocapnia should be assured given that hyperventilation may reduce the arterial CO2 below the apneic threshold (Miller: Miller’s Anesthesia, ed 8, p 897). Pharmacology and Pharmacokinetics of Intravenous Drugs 85
278. Which of the following α-antagonists produces an irreversible blockade? A. Phentolamine B. Prazosin C. Phenoxybenzamine D. Labetalol
278. (C) Phentolamine, prazosin, yohimbine, tolazoline, and terazosin are competitive and reversible α-adrenergic antagonists. Phenoxybenzamine produces an irreversible α-adrenergic blockade. Once phenoxybenzamine’s α-blockade develops, even massive doses of sympathomimetics are ineffective until phenoxybenzamine’s action is terminated by metabolism. Phentolamine and phenoxybenzamine are nonselective α1 and α2 antagonists, prazosin is a selective α1 antagonist, and yohimbine is a selective α2 antagonist (Hemmings: Pharmacology and Physiology for Anesthesia, pp 227–229).
279. Metoprolol is relatively contraindicated for treatment of tachycardia in the setting of A. Hypertrophic obstructive cardiomyopathy (HOCM) B. WPW syndrome (with narrow QRS) C. Prolonged QT syndrome D. Cardiac tamponade
279. (C) Symptomatic bradycardia as a result of excessive β-adrenergic receptor blockade can be treated with a variety of drugs, as well as with a pacemaker. Treatment depends upon severity of symptoms. Atropine can block any parasympathetic nervous system contribution to the bradycardia. If atropine is not effective, then a pure β-adrenergic receptor agonist can be tried. For excessive cardioselective β1 blockade, dobutamine can be used; for a noncardiac selective β1 and β2 blockade, isoproterenol can be chosen. Dopamine is not recommended because the high doses needed to overcome β-adrenergic receptor blockade will cause significant α-adrenergic receptor–induced vasoconstriction. Glucagon at an initial dose of 1 to 10 mg intravenously followed by an infusion of 5 mg/hr often is believed to be the drug of choice for β-adrenergic blockade overdosage. Glucagon increases myocardial contractility and heart rate, primarily by increasing cAMP formation (not via β-adrenergic receptor stimulation) and, to a lesser extent, by stimulating the release of catecholamines. Other drugs that have been used include aminophylline and calcium chloride. Aminophylline inhibits phosphodiesterase, resulting in an increase in cAMP. Thus, like glucagon, aminophylline increases cardiac output and heart rate via a non–β-adrenergic receptor–mediated mechanism. Calcium chloride may prove useful to counteract any decrease in myocardial contractility induced by the β-blockade; however, this effect may be transient (Stoelting: Pharmacology and Physiology in Anesthetic Practice, ed 4, pp 331–332).
280. A dose of 150 mg of IV dantrolene is administered to a 24-year-old, 75-kg man in whom incipient malignant hyperthermia is suspected. An expected consequence of this therapy would be A. Muscle spasticity in the postoperative period B. Hypothermia C. Cardiac dysrhythmias D. Diuresis
280. (D) Dantrolene is a skeletal muscle relaxant that is effective in the treatment of malignant hyperthermia. Dantrolene is formulated with mannitol (300 mg mannitol/20 mg dantrolene) so that diuresis is promoted during dantrolene therapy. Myoglobinuria from malignant hyperthermia–associated muscle breakdown accumulates in the renal tubules and can cause kidney failure if urine output is not maintained. Dantrolene works within the muscle cell to reduce intracellular levels of calcium. In the usual clinical doses, dantrolene has little effect on cardiac muscle contractility. In fulminant malignant hyperthermia, cardiac dysrhythmias may occur, but this is related to perturbations in pH and electrolytes. (Verapamil should not be used, because it interacts with dantrolene and may produce hyperkalemia and myocardial depression. Lidocaine appears safe.) Some side effects of short-term administration include muscle weakness (which may persist for 24 hours after dantrolene therapy is discontinued), nausea and vomiting, diarrhea, blurred vision, and phlebitis. Hypothermia may also occur with malignant hyperthermia treatment but is related to ice packing, not to dantrolene administration per se. When decreasing the fever, cooling should be stopped when core temperature reaches 38° C to avoid hypothermia. Hepatotoxicity has been demonstrated only with long-term use of oral dantrolene (Hines: Stoelting’s Anesthesia and Co-Existing Disease, ed 6, p 638).
281. Atracurium differs from cisatracurium in which way? A. Molecular weight B. Formation of laudanosine C. Histamine release D. No renal metabolism
281. (C) Cisatracurium is a stereoisomer of atracurium and as such has the same molecular weight. Both drugs undergo Hofmann elimination and form laudanosine. Atracurium is also estimated to undergo two thirds of its metabolism via ester hydrolysis catalyzed by nonspecific plasma esterases (not pseudocholinesterase). Neither drug requires renal or hepatic input for its degradation; hence, both can be used with renal or hepatic failure. Atracurium causes histamine release, whereas cisatracurium does not (Miller: Basics of Anesthesia, ed 6, p 154).
282. Signs and symptoms of opioid withdrawal include all of the following EXCEPT A. Increased BP and heart rate B. Seizures C. Abdominal cramps D. Jerking of the legs 60 Part 1 Basic Sciences Group 283-287
282. (B) Withdrawal from opioids is rarely life-threatening but may complicate postoperative care. Opioid withdrawal may spontaneously start within 6 to 12 hours after the last dose of a short-acting opioid and as long as 72 to 84 hours after a long-acting opioid in addicted patients. The duration of withdrawal symptoms also depends on the opioid; for heroin, withdrawal symptoms last 5 to 10 days, and for methadone, even longer. Opioid withdrawal can be precipitated within seconds if naloxone is administered intravenously to an addict. (Naloxone is contraindicated in opioid addicts for this reason.) Signs and symptoms of withdrawal include craving for opioids, restlessness, anxiety, irritability, nausea, vomiting, abdominal cramps, muscle aches, insomnia, sympathetic stimulation (increased heart rate, increased BP, mydriasis, diaphoresis) as well as tremors, jerking of the legs (origin of the term “kicking the habit”), and hyperthermia. Seizures, however, are very rare and if seizures occur, one should consider that withdrawal 86 Part 1 Basic Sciences from other drugs may also be occurring (e.g., from barbiturates) or that an underlying seizure disorder may also exist (Hines: Stoelting’s Anesthesia and Co-Existing Disease, ed 6, p 546). For Questions 283-287: Side effects of each of the intravenous induction agents (thiopental, diazepam, etomidate, propofol, and ketamine) occur. Some are unique for each drug.
283. Adrenal suppression A. Ketamine B. Diazepam C. Etomidate D. Propofo
283. (C) Etomidate is unique among the intravenous induction agents because it can cause adrenocortical suppression by inhibiting the conversion of cholesterol to cortisol. This can occur after a single induction dose and may persist for 4 to 8 hours. The clinical significance of this temporary adrenocortical suppression is unclear. However, in the ICU with prolonged sedation, clinical adrenal insufficiency may develop (i.e., hypotension, hyponatremia, and hyperkalemia). Here corticosteroids should be administered in stress doses (e.g., cortisol 100 mg/day) (Miller: Basics of Anesthesia, ed 6, pp 111–112).
284. Thrombosis, phlebitis, specific antagonist available A. Ketamine B. Diazepam C. Etomidate D. Propofo
284. (B) Diazepam is a benzodiazepine drug and was widely used intravenously for anesthesia until midazolam was developed. Although it is an effective sedative and amnestic drug, diazepam causes significant pain on injection and at times venous irritation and thrombophlebitis. This does not seem to occur with midazolam. Benzodiazepines do not suppress the adrenal gland. The most significant problem with benzodiazepines is respiratory depression. Benzodiazepines are unique among the intravenous sedatives because a specific benzodiazepine receptor antagonist is available (flumazenil). One problem with flumazenil is its relatively short duration of action (half-life about 1 hour), which is shorter than that of diazepam (21-37 hours) and midazolam (1-4 hours) (Miller: Basics of Anesthesia, ed 6, p 109).
285. Pain on injection, severe hypotension in elderly A. Ketamine B. Diazepam C. Etomidate D. Propofo
285. (D) Pain on injection is common with diazepam, etomidate, and propofol but rare with thiopental and ketamine. However, hemodynamic stability is common with etomidate and diazepam, whereas hypotension is common after propofol and thiopental, especially in patients who are volume-depleted or elderly. Hypertension may develop with ketamine use due to its sympathetic nervous system stimulation (Miller: Basics of Anesthesia, ed 6, pp 99–102).
286. Increases ICP A. Ketamine B. Diazepam C. Etomidate D. Propofo
286. (A) ICP tends to fall after the administration of thiopental, etomidate, and propofol and can either fall or remain unchanged with benzodiazepines. Ketamine, however, can increase ICP and should be avoided in patients with intracranial mass lesions and elevated ICP because it can further increase the ICP (Miller: Basics of Anesthesia, ed 6, pp 109–111).
287. Lactic acidosis may develop with prolonged use A. Ketamine B. Diazepam C. Etomidate D. Propofol
287. (D) Propofol infusion syndrome (lactic acidosis) may develop when high-dose infusions (i.e., >75 μg/kg/ min) are infused for longer than 24 hours. Early signs include tachycardia; later on, severe metabolic acidosis, bradyarrhythmias, and myocardial failure may develop. The cause appears to be related to impaired fatty acid oxidation in the mitochondria (Miller: Basics of Anesthesia, ed 6, pp 99–102). For Questions 288-292: Antihypertensive agents are used primarily in the treatment of essential hypertension to reduce BP toward normal. These agents include direct-acting smooth muscle relaxants or vasodilators (e.g., hydralazine), centrally acting α2-sympathetic receptor agonists (e.g., clonidine), peripheral adrenergic receptor antagonists (e.g., labetalol), calcium channel blockers, diuretics, angiotensin-converting enzyme (ACE) inhibitors (e.g., captopril, lisinopril), and angiotensin receptor blockers (ARBs) (Barash: Clinical Anesthesiology, ed 7, pp 392, 399, 403–404).
288. Reduces MAC A. Clonidine B. Hydralazine C. Losartan D. Labetalol
288. (A) Central-acting sympathomimetic agents such as clonidine produce some sedative effects and can reduce the anesthetic requirement or MAC.
289. Blockade of angiotensin receptor A. Clonidine B. Hydralazine C. Losartan D. Labetalol
289. (C) Losartan (Cozaar) blocks the hormone angiotensin at the receptor. It is pharmacologically similar to ACE inhibitors, but with fewer side effects. It is useful for treatment of diabetic patients and those with cardiovascular disease. Hyperkalemia is a potential side effect of therapy with this drug.
290. With high doses may cause a systemic lupus erythematosus–like syndrome A. Clonidine B. Hydralazine C. Losartan D. Labetalol
290. (B) Approximately 10% to 20% of patients who are chronically taking hydralazine (i.e., >6 months) develop a systemic lupus erythematosus–like syndrome, especially if the daily dose is high (e.g., >200 mg). The systemic lupus erythematosus–like syndrome will resolve once hydralazine therapy is discontinued.
291. Produces α-adrenergic receptor and β-adrenergic receptor blockade A. Clonidine B. Hydralazine C. Losartan D. Labetalol
291. (D) Labetalol is an α1-adrenergic receptor and nonselective β-adrenergic receptor antagonist. Pharmacology and Pharmacokinetics of Intravenous Drugs 87
292. May result in severe rebound hypertension when abruptly discontinued A. Clonidine B. Hydralazine C. Losartan D. Labetalol
292. (A) Abrupt discontinuation of chronically administered clonidine (especially if the dose is >1.2 mg/day) may result in severe rebound hypertension within 8 to 36 hours after the last dose. For Question 293: Some drugs inhibit coagulation and do so through a myriad of different pathways. An understanding of these drugs and their mechanisms is helpful to the anesthesia provider.
293. Alternative to heparin for cardiopulmonary bypass A. Argatroban B. Clopidogrel C. Abciximab D. Fondaparinux
293. (A) Patients susceptible to HIT-2 (heparin-induced thrombocytopenia) should wait 3 months for a clinically significant decrease in the antibody titer before receiving heparin. If waiting is not possible and surgery involving cardiopulmonary bypass cannot be delayed, direct thrombin inhibitors like hirudin, bivalirudin, or argatroban can be used as anticoagulants for bypass surgery (Miller: Basics of Anesthesia, ed 6, pp 358–359).
294. Glycoprotein (GP)IIb/IIIa inhibition A. Argatroban B. Clopidogrel C. Abciximab D. Fondaparinux
294. (C) Abciximab (ReoPro, plasma half-life 30 minutes), tirofiban (Aggrastat, plasma half-life 2 hours), and eptifibatide (Integrilin, plasma half-life 2.5 hours) are potent inhibitors of platelet activity. They block the binding of vWF and fibrinogen to the GPIIb/IIIa receptors on platelets. These drugs are used in the treatment of acute coronary syndrome. If surgery is required, therapy with eptifibatide and tirofiban should be stopped for 24 hours. Abciximab should be stopped for 72 hours before an operation. All three of these drugs produce thrombocytopenia and are metabolized by the kidney, but dialysis as reversal is only effective with tirofiban (Barash: Clinical Anesthesia, ed 7, pp 437–438, Miller: Miller’s Anesthesia, ed 8, p 1873; Miller: Basics of Anesthesia, ed 6, pp 357–359).
295. Direct thrombin inhibition A. Argatroban B. Clopidogrel C. Abciximab D. Fondaparinux
295. (A) Argatroban is a direct thrombin inhibitor. Please see explanation and reference for Question 293.
296. Used after angioplasty often for a year or more to prevent restenosis A. Argatroban B. Clopidogrel C. Abciximab D. Fondaparinux
296. (B) The thienopyridine compounds, ticlopidine and clopidogrel, are P2Y12 adenosine diphosphate (ADP) receptor antagonists. Binding to this ADP receptor suppresses expression of GPIIb/IIIa and prevents fibrinogen from binding to platelets. Although platelet function studies, per se, are not a reliable way to test the effects of clopidogrel, there is a test to measure the inhibition of the GPIIb/ IIIa receptor. Clopidogrel is an inactive prodrug that must be metabolized into the active form by liver oxidases. A genetic polymorphism exists whereby patients are unable to oxidize clopidogrel into the active compound, thus making it therapeutically ineffective (Barash: Clinical Anesthesia, ed 7, p 437; Miller: Basics of Anesthesia, ed 6, pp 357–359).
297. Anti-Xa activity mechanism of action A. Argatroban B. Clopidogrel C. Abciximab D. Fondaparinux
297. (D) Fondaparinux is an antagonist of factor Xa. It also binds with antithrombin III. Its principal use is deep vein thrombosis prophylaxis, and there is no antidote for it other than stopping therapy and letting it wear off. Because it is renally eliminated, dose must be reduced in patients with renal failure. It is not approved for patients with history of heparin-induced thrombocytopenia (Barash: Clinical Anesthesia, ed 7, p 439).
298. Of the list, most likely to be associated with opioid induced hyperalgesia A. Methadone B. Remifentanil C. Tapentadol (Nucynta) D. Butorphanol
298. (B) Both acute tolerance to opioids and opioid-induced hyperalgesia (OIH) require more analgesics to treat pain. With tolerance the pharmacologic response is less over time; thus, more opioids are needed to relieve the same amount of pain (e.g., chronic back pain). With OIH there is an exaggerated response to painful stimuli. This can occur under certain situations such as an exaggerated response to pain when a remifentanil infusion is stopped (rapid offset of analgesia). To prevent this when using remifentanil-based anesthesia, it is wise to add a longer duration opioid (e.g., morphine) and/or to add nonopioid analgesics before stopping a remifentanil infusion (if pain is expected in the postoperative period). Although the etiology of OIH is unknown, it may involve both central and peripheral nervous system adaptations involving the NMDA receptor (Barash: Clinical Anesthesia, ed 7, p 506; Hemmings: Pharmacology and Physiology for Anesthesia, pp 267–268).
299. Demonstrates ceiling effect with regard to respiratory depression A. Methadone B. Remifentanil C. Tapentadol (Nucynta) D. Butorphanol
299. (D) Mixed agonist-antagonist drugs, such as butorphanol, nalbuphine, and pentazocine, are partial agonists at the κ receptor and complete competitive antagonists at the μ receptor. Both the analgesia and respiratory depressant effects of these drugs approach a ceiling effect. They are used as analgesics for mild-to-moderate pain. They are also used to reverse excessive opioid-induced respiratory depression due to their μ antagonism, while maintaining some analgesia at the κ receptor (Miller: Miller’s Anesthesia, ed 8, pp 903–904; Hemmings: Pharmacology and Physiology for Anesthesia, pp 265–266). 88 Part 1 Basic Sciences
300. Antagonism of NMDA receptors A. Methadone B. Remifentanil C. Tapentadol (Nucynta) D. Butorphanol
300. (A) Although opioids are mainly thought to work on opioid receptors, methadone is also a most potent NMDA receptor antagonist (6-18 times that of morphine). This property appears to be useful in reducing the effects of opioid tolerance and withdrawal syndrome (Barash: Clinical Anesthesia, ed 7, p 505; Hemmings: Pharmacology and Physiology for Anesthesia, p 264).
301. Norepinephrine reuptake inhibitor (NRI) A. Methadone B. Remifentanil C. Tapentadol (Nucynta) D. Butorphanol
301. (C) Tapentadol (Nucynta) is a new opioid marketed for fewer GI and CNS side effects. It has a dual mechanism of action: as an agonist for the μ receptor site and as a norepinephrine reuptake inhibitor (NRI). It should not be used in patients taking MAOIs, because an adrenergic crisis may develop. It is also contraindicated with SSRIs, because it may lead to serotonin syndrome. It is only available orally (Barash: Clinical Anesthesia, ed 7, p 505; Brunton: Goodman & Gilman’s The Pharmacological Basis of Therapeutics, ed 12, p 508). For Questions 302-305: Depolarizing neuromuscular blockade usually is described as having two phases. Phase I blockade occurs with depolarization of the postjunctional membrane. Phase II blockade occurs when the postjunctional membrane has become repolarized but does not respond normally to acetylcholine (i.e., often termed desensitized, but other factors are involved). This can occur when the dose of succinylcholine is greater than 2 to 4 mg/kg. The response of a muscle to electrical nerve stimulation for a phase II block is similar to that for a nondepolarizing block. Nondepolarizing neuromuscular blockade is only of one type (Miller: Basics of Anesthesia, ed 6, pp 148–149).
302. Block is antagonized with anticholinesterase drugs A. True of nondepolarizing blockade only B. True of phase I depolarizing blockade only C. True of phase II depolarizing blockade only D. True of nondepolarizing and phase II depolarizing blockade
302. (D) Although the mechanisms of a nondepolarizing and a phase II depolarizing block likely are different, they both can be antagonized with anticholinesterase drugs.
303. Block is enhanced with anticholinesterase drugs A. True of nondepolarizing blockade only B. True of phase I depolarizing blockade only C. True of phase II depolarizing blockade only D. True of nondepolarizing and phase II depolarizing blockade
303. (B) Only a phase I depolarizing block is enhanced with the use of anticholinesterase drugs.
304. Post-tetanic facilitation occurs A. True of nondepolarizing blockade only B. True of phase I depolarizing blockade only C. True of phase II depolarizing blockade only D. True of nondepolarizing and phase II depolarizing blockade
304. (D) Post-tetanic facilitation occurs when a single twitch that is induced a short period of time after tetanic stimulation is larger than the amplitude of the tetanus. This occurs with a phase II depolarizing blockade as well as with a nondepolarizing blockade.
305. Sustained response to tetanic stimulus is seen A. True of nondepolarizing blockade only B. True of phase I depolarizing blockade only C. True of phase II depolarizing blockade only D. True of nondepolarizing and phase II depolarizing blockade
305. (B) The amplitude of the muscle response to sustained tetanic stimulation remains the same with phase I depolarizing blockade, but it shows a marked fade with a phase II depolarizing blockade or a nondepolarizing blockade. SUMMARY OF MUSCULAR RESPONSES TO NERVE STIMULATION WITH DIFFERENT TYPES OF BLOCKADE Stimulation Phase I Depolarizing Phase II Depolarizing Nondepolarizing Single twitch Decreased Decreased Decreased Tetanic stimulation Decreased height but no fade Fade Fade Post-tetanic facilitation None Yes Yes Train of four All twitches same, decrease in height Marked fade Marked fade Train-of-four ratio >0.7 <0.4 <0.7 Anticholinesterase Enhances Antagonizes Antagonizes For Questions 306-315: A simple way to measure the potency of inhaled drugs is to measure their MAC values. MAC is the minimum alveolar concentration of an inhaled drug at 1 atmosphere (atm) (1 atm = 760 mm Hg) where 50% of patients do not move in response to a painful stimulus. It is commonly measured as the end-expired drug concentration. Various physiologic or pharmacologic factors can increase or decrease MAC. In general, factors that increase metabolic function of the brain (e.g., hyperthermia) or elevate brain catecholamines (e.g., MAOIs, tricyclic antidepressants, cocaine, acute amphetamine use) increase MAC, and factors that depress function (e.g., intravenous anesthetics, acute ethanol use, narcotics, hypothermia) decrease MAC. Recently, it has been suggested that there might be a genetic component to MAC, because redheaded females have about a 20% increase in MAC compared with dark-haired females (Barash: Clinical Anesthesia, ed 7, pp 458–459). Pharmacology and Pharmacokinetics of Intravenous Drugs 89
306. Amphetamines A. No change in MAC B. Increases MAC C. Decreases MAC D. Acute administration increases MAC; chronic administration decreases MAC
306. (D) Acute amphetamine use increases MAC, whereas chronic amphetamine use decreases MAC.
307. α2 Agonists (clonidine, dexmedetomidine) A. No change in MAC B. Increases MAC C. Decreases MAC D. Acute administration increases MAC; chronic administration decreases MAC
307. (C) α2 Agonists decrease MAC.
308. Hyperthyroidism A. No change in MAC B. Increases MAC C. Decreases MAC D. Acute administration increases MAC; chronic administration decreases MAC
308. (A) Changes in thyroid function (e.g., hyperthyroidism, hypothyroidism) do not seem to affect MAC. However, the cardiovascular response to volatile drugs is altered with thyroid function.
309. Acute ethanol ingestion A. No change in MAC B. Increases MAC C. Decreases MAC D. Acute administration increases MAC; chronic administration decreases MAC
309. (C) With acute administration, ethanol is a CNS depressant and decreases MAC. Chronic ethanol administration increases MAC.
310. Lidocaine A. No change in MAC B. Increases MAC C. Decreases MAC D. Acute administration increases MAC; chronic administration decreases MAC
310. (C) Lidocaine use decreases MAC.
311. Lithium A. No change in MAC B. Increases MAC C. Decreases MAC D. Acute administration increases MAC; chronic administration decreases MAC
311. (C) Patients on lithium therapy have lower MAC values. This may be related to the lower catecholamine levels in the brain.
312. Opioids A. No change in MAC B. Increases MAC C. Decreases MAC D. Acute administration increases MAC; chronic administration decreases MAC
312. (C) Opioids produce a dose-dependent decrease in MAC (up to about 50%).
313. Duration of anesthesia A. No change in MAC B. Increases MAC C. Decreases MAC D. Acute administration increases MAC; chronic administration decreases MAC
313. (A) The duration of anesthesia, as well as the gender of the patient, does not affect MAC.
314. Pregnancy A. No change in MAC B. Increases MAC C. Decreases MAC D. Acute administration increases MAC; chronic administration decreases MAC
314. (C) Pregnancy lowers MAC. This may be related to the sedative effects of progesterone. Pregnant patients also are very sensitive to local anesthetics.
315. Pao2 35 mm Hg A. No change in MAC B. Increases MAC C. Decreases MAC D. Acute administration increases MAC; chronic administration decreases MAC
315. (C) Severe hypoxia (Pao2 of 38 mm Hg), as well as severe anemia (<4.3 mL/oxygen/dL of blood), decreases MAC. For Questions 316-320: The goals of pharmacologic premedication must be individualized to meet each patient’s requirements. Some of these goals include amnesia, relief of anxiety, sedation, analgesia, reduction of gastric fluid volume, elevation of gastric fluid pH, prophylaxis against allergic reactions, and reduction of oral and respiratory secretions. The drugs most commonly used to achieve these goals include benzodiazepines, barbiturates, opioids, H2-receptor antagonists, nonparticulate antacids, antihistamines, and anticholinergic agents. The anticholinergics atropine, scopolamine, and glycopyrrolate are rarely given with premedication today unless a specific effect is needed (e.g., drying of the mouth before fiberoptic intubation, prevention of bradycardia, and, rarely, as a mild sedative). Atropine and scopolamine are tertiary compounds that can readily cross lipid membranes such as the blood-brain barrier. These tertiary amines can produce sedation, amnesia, CNS toxicity (central anticholinergic syndrome manifested as delirium or prolonged somnolence after anesthesia), mydriasis, and cycloplegia (whereas glycopyrrolate, a quaternary compound, does not cross lipid membranes well). All three anticholinergics can cause drying of airway secretions by inhibiting salivation, can cause tachycardia (although bradycardia can be seen in some patients), can decrease the lower esophageal sphincter tone, and can increase body temperature by inhibiting sweating. The main differences are listed in the table following the explanation to Question 178 (Miller: Basics of Anesthesia, ed 6, p 76).
316. Least effective antisialagogue A. Atropine B. Glycopyrrolate C. Scopolamine D. Atropine and scopolamine
316. (A) All three anticholinergics can cause drying of airway secretions by inhibiting salivation, but atropine is the least effective of these drugs.
317. Produces best sedation A. Atropine B. Glycopyrrolate C. Scopolamine D. Atropine and scopolamine
317. (C) To produce sedation, the drug must pass the blood-brain barrier. This is much more prominent with scopolamine and much less so with atropine. Glycopyrrolate does not cause any sedation.
318. Causes greatest increase in heart rate A. Atropine B. Glycopyrrolate C. Scopolamine D. Atropine and scopolamine
318. (A) Atropine has the best blocking effect on muscarinic receptors of the heart.
319. Does not produce central anticholinergic syndrome A. Atropine B. Glycopyrrolate C. Scopolamine D. Atropine and scopolamine
319. (B) The toxic state known as central anticholinergic syndrome requires passage of the drug across the blood-brain barrier and, therefore, precludes glycopyrrolate from causation.
320. May produce mydriasis and cycloplegia when placed topically in the eye A. Atropine B. Glycopyrrolate C. Scopolamine D. Atropine and scopolamine
320. (D) Both atropine and scopolamine can cause ocular effects (scopolamine more so than atropine), including mydriasis and cycloplegia when applied topically to the eye. Caution is suggested when scopolamine is given intramuscularly to patients with glaucoma. IV administration of atropine to prevent or treat bradycardia appears to have little effect on the eye. If a scopolamine patch is placed to help prevent PONV, one needs to carefully wash one’s hands after application, because rubbing an eye with any scopolamine on the fingers may lead to unilateral mydriasis. 90
321. The minimum alveolar concentration (MAC) is highest in neonates (0-30 days old) versus other age groups with which of the following? A. Isoflurane B. Sevoflurane C. Desflurane D. N2O
321. (B) The MAC for inhalation agents varies with age. For most volatile anesthetics, the highest MAC values are for infants 1 to 6 months old. In infants younger than 1 month or older than 6 months, the MAC is lower for isoflurane, halothane, and desflurane. Sevoflurane is different. For sevoflurane, the MAC for neonates 0 to 30 days old is 3.3%, for infants 1 to 6 months old it is 3.2%, and for infants 6 to 12 months old it is 2.5% (Miller: Miller’s Anesthesia, ed 8, p 2764).
322. The rate of increase in the alveolar concentration of a volatile anesthetic relative to the inspired concentration (Fa/Fi) plotted against time is steep during the first moments of inhalation with all volatile anesthetics. The reason for this observation is that A. Volatile anesthetics reduce alveolar ventilation (Va) B. There is minimal anesthetic uptake from the alveoli into pulmonary venous blood C. Volatile anesthetics increase cardiac output initially D. The volume of the anesthetic breathing circuit is small
322. (B) The alveolar partial pressure of a volatile anesthetic, which ultimately determines the depth of general anesthesia, is determined by the relative rates of input to removal of the anesthetic gases to and from the alveoli. Removal of anesthetic gases from the alveoli is accomplished by uptake into the pulmonary venous blood, which is most dependent on an alveolar partial pressure difference. During the initial moments of inhalation of an anesthetic gas, there is no volatile anesthetic in the alveoli to create this partial pressure gradient. Therefore, the uptake for all volatile anesthetic gases will be minimal until the resultant rapid increase in alveolar partial pressure establishes a sufficient alveolar-to-venous partial pressure gradient to promote uptake of the anesthetic gas into the pulmonary venous blood. This will occur in spite of other factors, which are discussed in the explanation to Question 333 (Miller: Miller’s Anesthesia, ed 8, pp 648–649).
323. During spontaneous breathing, volatile anesthetics A. Increase tidal volume (Vt) and decrease respiratory rate B. Increase Vt and increase respiratory rate C. Decrease Vt and decrease respiratory rate D. Decrease Vt and increase respiratory rate
323. (D) At concentrations of 1 MAC or less, volatile anesthetics, as well as the inhaled anesthetic N2O, will produce dose-dependent increases in the respiratory rate in spontaneously breathing patients. This trend continues at concentrations greater than 1 MAC for all of the inhaled anesthetics except isoflurane. With the exception of N2O, the evidence suggests that this effect is caused by direct activation of the respiratory center in the CNS rather than by stimulation of pulmonary stretch receptors. Additionally, volatile anesthetics decrease Vt and significantly alter the breathing pattern from the normal awake pattern of intermittent deep breaths separated by varying time intervals to one of rapid, shallow, regular, and rhythmic breathing (Miller: Miller’s Anesthesia, ed 8, pp 691–692).
324. Which of the following can NOT be considered an advantage of low-flow anesthesia? A. Conservation of fossil fuel B. Less ozone depletion C. Reduced room pollution D. Conservation of absorbent
324. (D) Barium-containing absorbents that interact with volatile anesthetics and produce carbon monoxide and compound A are no longer used in clinical practice. They have been replaced with calcium-containing products such as Amsorb Plus. Consequently, absorbent granules are “consumed” by CO2 produced by the patient, not by the total flow of anesthetic gases. On the contrary, with low flow techniques, recirculation (rebreathing) of expired gases results in more rapid depletion of the CO2 absorbent. Volatile anesthetics are organic compounds, specifically alkanes (halothane) and substituted methylethyl ethers (desflurane, isoflurane) or substituted isopropyl methyl ether (sevoflurane). They are ultimately derived from petroleum sources and are then halogenated to become substituted organic compounds. They join a myriad of other organic halides such as hairspray, propellants, refrigerants, and solvents that collectively contribute to the depletion of the ozone layer in the earth’s atmosphere. The main greenhouse gases are CO2, methane, and N2O. N2O constitutes roughly 5% of the greenhouse gases. Another rationale for the use of low-flow anesthesia is the introduction of less waste into the OR. The disadvantage of low-flow anesthesia is that the Fio2 will continually drop during the administration of anesthesia (unless 100% oxygen is administered), and vigilance is required because this drop may approach or even reach the level of a hypoxic mixture (Miller: Miller’s Anesthesia, ed 8, pp 664–665).
325. The main reason desflurane is not used for inhalation induction in clinical practice is because of A. Its low blood/gas partition coefficient B. Its propensity to produce hypertension in high concentrations C. Its propensity to produce airway irritability D. Its propensity to produce tachyarrhythmias
325. (C) Although desflurane has a low blood/gas partition coefficient (0.42) and should produce rapid induction of anesthesia, its marked pungency and airway irritation make inhalation inductions very difficult. Not only do patients dislike the scent, but the airway irritation often leads to coughing, increased salivation, breath holding, and sometimes laryngospasm (especially if the concentration is rapidly increased). In addition, with abrupt increases in concentration, patients often experience tachycardia and hypertension, thought to be due to increased sympathetic discharge (Miller: Basics of Anesthesia, ed 6, p 95). Pharmacology and Pharmacokinetics of Volatile Anesthetics 97
326. A medical group planning a trip to South America has a large supply of old enflurane vaporizers (vapor pressure = 170 mm Hg). Which volatile agent could be delivered through an enflurane vaporizer in such a manner that the dialed setting equals the vaporizer’s output? A. Desflurane B. Sevoflurane C. Isoflurane D. None; all other volatile agents will be at least 30% off
326. (B) A vaporizer’s specificity is based on the vapor pressure of the anesthetic agent for which it is made. Filling a vaporizer with an agent whose vapor pressure is higher results in a higher concentration in the vaporizer’s output. Similarly, a volatile agent with a lower vapor pressure produces an output with a lower concentration than that seen on the dial. The vapor pressure of enflurane, 172 mm Hg (20° C), most closely approximates the vapor pressure of sevoflurane, which is 160 mm Hg (Miller: Basics of Anesthesia, ed 6, p 81).
327. Select the TRUE statement regarding blood pressure when 1.5 MAC N2O-isoflurane is substituted for 1.5 MAC isoflurane-oxygen. A. Blood pressure is less than awake value but greater than that seen with isoflurane-O2 B. Blood pressure is equal to awake value C. Blood pressure is greater than awake value D. Blood pressure is less than isoflurane-O2 pressure
327. (A) When N2O is substituted for an equal MAC value of isoflurane, the resulting blood pressure is greater than that seen with the same MAC value achieved with isoflurane as the sole anesthetic agent. When administered alone, N2O does not alter arterial blood pressure, stroke volume, systemic vascular resistance, or baroreceptor reflexes. The administration of N2O increases heart rate slightly, which may result in a mild increase in cardiac output. In vitro, N2O has a dose-dependent direct depressant effect on myocardial contractility, which is probably overcome in vivo by sympathetic activation (Miller: Basics of Anesthesia, ed 6, p 93).
328. Which of the following volatile anesthetics decreases systemic vascular resistance? A. Sevoflurane B. Isoflurane C. Desflurane D. All of the above
328. (D) All of the present-day volatile anesthetics reduce blood pressure in a dose-dependent fashion. Desflurane, sevoflurane, and isoflurane cause this primarily through reductions in systemic vascular resistance. The obsolete agents, halothane and enflurane, produce hypotension via direct myocardial depression (Miller: Basics of Anesthesia, ed 6, pp 90–91).
329. With which of the following inhalational agents is cardiac output moderately increased? A. N2O B. Sevoflurane C. Desflurane D. Isoflurane
329. (A) The older agent halothane tended to decrease the cardiac output, whereas sevoflurane, desflurane, and isoflurane tend to maintain cardiac output. N2O tends to increase cardiac output primarily because of the mild increase in sympathetic tone (Stoelting: Pharmacology and Physiology in Anesthetic Practice, ed 4, p 53).
330. Select the FALSE statement about isoflurane (≤1 MAC). A. May attenuate bronchospasm B. Increases right atrial pressure C. Decreases mean arterial pressure D. Decreases cardiac output
330. (D) At concentrations of 1 MAC, isoflurane may attenuate antigen-induced bronchospasm, presumably by decreasing vagal tone. At similar concentrations, isoflurane will not reduce cardiac output in patients with normal left ventricular function. Additionally, isoflurane will decrease stroke volume, mean arterial pressure, and systemic vascular resistance in a dose-dependent manner. Cardiac output remains unchanged because decreases in systemic vascular resistance result in a reflex increase in heart rate that is sufficient to offset the decrease in stroke volume. However, dose-dependent decreases in both stroke volume and cardiac index can be seen when isoflurane is administered in concentrations greater than 1 MAC (Miller: Basics of Anesthesia, ed 6, pp 90–95).
331. Abrupt and large increases in the delivered concentration of which of the following inhalational anesthetics may produce transient increases in systemic blood pressure and heart rate? A. Desflurane B. Isoflurane C. Sevoflurane D. N2O
331. (A) Desflurane can (but does not always) produce increased blood pressure and heart rate when the concentrations are rapidly increased. This may be related to airway irritation and a sympathetic response. This has also occurred with isoflurane, but to a much less frequent and usually lower extent. The other agents listed do not cause this sympathetic response with a rapid increase in concentration. If desflurane is increased slowly or a prior dose of narcotic is given, this increase in blood pressure and heart rate may not occur (Miller: Basics of Anesthesia, ed 6, pp 90–92).
332. Discontinuation of 1 MAC of which volatile anesthetic followed by immediate introduction of 1 MAC of which second volatile anesthetic would temporarily result in the greatest combined anesthetic potency? A. Isoflurane followed by desflurane B. Sevoflurane followed by desflurane C. Desflurane followed by isoflurane D. Desflurane followed by sevoflurane
332. (A) Of all the options listed, desflurane has the lowest solubility constant, which results in a very rapid rise in Fa/Fi. The rate of rise is very similar to that seen with N2O and results in the most rapid attainment of 1 MAC concentration once the new volatile anesthetic has been initiated. Isoflurane has the highest blood/gas solubility coefficient of all the options, reflecting the largest quantity of gas stored in the blood. This reservoir will result in the slowest decline in the alveolar concentration of this volatile agent upon discontinuation. The combination of these different solubilities will ultimately result in the highest combined MAC when 1 MAC of isoflurane is discontinued and 1 MAC of desflurane is introduced (Miller: Basics of Anesthesia, ed 6, p 88; Morgan & Mikhail: Clinical Anesthesiology, ed 4, pp 156–157, 159).
333. Cardiogenic shock has the greatest impact on the rate of increase in Fa/Fi for which of the following volatile anesthetics? A. Isoflurane B. Desflurane C. Sevoflurane D. N2O
333. (A) The alveolar partial pressure of an anesthetic is determined by the rate of input relative to removal of the anesthetic from the alveoli, as explained in Question 322. During induction, the anesthetic gas is removed from the alveoli by uptake into the pulmonary venous blood. The rate of uptake is influenced by cardiac output, the blood/gas solubility coefficient, and the alveolar-to-venous partial pressure difference of the anesthetic. At a lower cardiac output, a slower rate of uptake of volatile anesthetic from the alveoli into the pulmonary venous blood results in a faster rate of increase in the alveolar concentration. This will result in an increased Fa/Fi. Uptake of poorly soluble anesthetic gases from the alveoli is minimal, and the rate of rise of Fa/Fi is rapid and virtually independent of cardiac output. Uptake of the more soluble anesthetics, such as isoflurane, from the alveoli into the pulmonary venous blood can be considerable and will be reflected by a slower rate of rise of the Fa/Fi ratio. Cardiogenic shock will have the 98 Part 1 Basic Sciences smallest impact on the most insoluble agents, such as desflurane, sevoflurane, and N2O, whereas the impact on the rate of rise of Fa/Fi of the relatively soluble anesthetic gases, such as isoflurane, will be more profound (Miller: Miller’s Anesthesia, ed 8, pp 645–646).
334. The vessel-rich group receives what percent of the cardiac output? A. 45% B. 60% C. 75% D. 90%
334. (C) The vessel-rich group that receives approximately 75% of the cardiac output is composed of the brain, heart, spleen, liver, splenic bed, kidneys, and endocrine glands. This group, however, constitutes only 10% of the total body weight. Because of this large blood flow relative to tissue mass, these organs take up a large volume of volatile anesthetic and equilibrate with the partial pressure of the volatile anesthetic in the blood and alveoli during the earliest moments of induction (Miller: Basics of Anesthesia, ed 6, p 87; Miller: Miller’s Anesthesia, ed 8, pp 647–648).
335. What percent desflurane is present in the vaporizing chamber of a desflurane vaporizer (pressurized to 1500 mm Hg and heated to 23° C)? A. Nearly 100% B. 85% C. 65% D. 45%
335. (D) Desflurane is unique among the current commonly used volatile anesthetics because of its high vapor pressure of 664 mm Hg. Because of this, the vaporizer is pressurized to 1500 mm Hg and is electrically heated to 23° C to give more predicable concentrations: 664/1500 = about 44%. If the desflurane is used at 1 atm the concentration will be about 88% (Barash: Clinical Anesthesia, ed 7, pp 666–668; Miller: Basics of Anesthesia, ed 6, pp 202–203; Butterworth: Morgan & Mikhail’s Clinical Anesthesiology, ed 5, pp 60–64).
336. A 25-year-old man is undergoing lymph node dissection for testicular cancer under general anesthesia. He has received four courses of bleomycin. The sevoflurane vaporizer is set at 1.8%, the oxygen at 100 mL/ min, and air at 900/mL/min. The Fio2 of the fresh gas flow is A. 26% B. 29% C. 34% D. 41%
336. (B) Fresh gas flow = 1 L per minute (1000 mL/min). Fio2 = [(100 mL/min) + (900 × 0.21 mL/min)]/1000 mL/min = (100 + 180)/1000 = 289/1000 = 29% Anesthetic flow meters are designed to deliver gases very accurately (Miller: Miller’s Anesthesia, ed 8, pp 760–761).
337. How would a right mainstem intubation affect the rate of increase in arterial partial pressure of volatile anesthetics? A. It would be reduced to the same degree for all volatile anesthetics B. It would be accelerated to the same degree for all volatile anesthetics C. It would be reduced the most for highly soluble agents D. It would be reduced the most for poorly soluble agents
337. (D) The situation described here is a transpulmonary shunt. In patients with transpulmonary shunting, blood emerging from unventilated alveoli contains no anesthetic gas. This anesthetic-deficient blood mixes with blood from adequately ventilated, anesthetic-containing alveoli, producing an arterial anesthetic partial pressure considerably less than expected. Because uptake of anesthetic gas from the alveoli into pulmonary venous blood will be less than normal, transpulmonary shunting accelerates the rate of rise in the Fa/Fi ratio but reduces the rate of increase in the arterial partial pressure of all volatile anesthetics. The degree to which these changes occur depends on the solubility of the given volatile anesthetic. For poorly soluble anesthetics, such as N2O, transpulmonary shunting only slightly accelerates the rate of rise in the Fa/Fi ratio, but it significantly reduces the rate of increase in arterial anesthetic partial pressure. The opposite occurs with highly soluble volatile anesthetics, such as halothane and isoflurane (Miller: Miller’s Anesthesia, ed 8, pp 646–647).
338. During a breast biopsy with the patient under general anesthesia, the end-tidal carbon dioxide (CO2) is 25 mm Hg on infrared spectrometer. Which of the following could NOT account for these findings? A. Mainstem intubation B. Enormous dead space C. Incipient cardiac arrest D. Overventilation
338. (A) CO2 is a very soluble gas. making the end-tidal CO2 (ETCO2) at the level of the alveoli virtually identical to arterial CO2 (Paco2). Because we measure ETCO2 on the total exhaled gas, the alveolar CO2 is diluted with the gas in the dead space (e.g., alveoli are ventilated but are not perfused as well as the Pharmacology and Pharmacokinetics of Volatile Anesthetics 99 respiratory passageways). A gradient of 2 to 5 mm Hg between Paco2 and ETCO2 is seen in normal healthy patients. Any condition that increases dead space or reduces lung perfusion (i.e., increases V/Q) such as pulmonary embolism, severe hypotension, low cardiac output, and cardiac arrest will decrease ETCO2. ETCO2 can also decrease with an increase in minute ventilation (increased removal of CO2) and can decrease with hypothermia (decreased production of CO2). Of course, ETCO2 can rapidly decrease to zero with any failure to ventilate (e.g., esophageal intubation, circuit disconnection, failure to turn the ventilator on after manual ventilation is stopped) as well as with disruption of the sampling lines. Because CO2 rapidly equilibrates between the bloodstream and the alveolar gas, an endotracheal tube that slips into a mainstem gives the same minute ventilation as an endotracheal tube in the trachea (airway pressure, however, would increase). Increased ETCO2 can have many causes, including hypoventilation, rebreathing of exhaled gas, increased absorption of CO2 from the abdomen distended with CO2 during laparoscopy, malignant hyperthermia, sepsis, and administration of bicarbonate used to treat metabolic acidosis (Barash: Clinical Anesthesia, ed 7, pp 704–705; Miller: Basics of Anesthesia, ed 6, pp 328–329; Butterworth: Morgan and Mikhail’s Clinical Anesthesiology, ed 5, pp 123–127).
339. Isoflurane, when administered to healthy patients in concentrations less than 1.0 MAC, will decrease all of the following EXCEPT A. Cardiac output B. Myocardial contractility C. Stroke volume D. Systemic vascular resistance
339. (A) Isoflurane is unique among the volatile agents in that it does not reduce cardiac output (cardiac index) at concentrations of 1 MAC or less in healthy volunteers (Miller: Basics of Anesthesia, ed 6, pp 90–92).
340. Increased Va will accelerate the rate of rise of the Fa/Fi ratio the MOST for A. Desflurane B. Sevoflurane C. Isoflurane D. N2O
340. (C) The rate of input of volatile anesthetics from the anesthesia machine to the alveoli is influenced by three factors: Va, the inspired anesthetic partial pressure, and the characteristics of the anesthetic breathing system. Increased Va will accelerate the rate of increase in Fa/Fi for all volatile anesthetics. However, the magnitude of this effect is dependent on the solubility of the inhaled anesthetic. The rate of increase in Fa/Fi depends very little on Va for poorly soluble anesthetics because the uptake of these is minimal. In contrast, the rate of increase in Fa/Fi for highly soluble anesthetics depends significantly on Va. Isoflurane is the most soluble inhaled anesthetic listed in this question (blood/gas solubility coefficient 1.46). Therefore, an increase in Va will accelerate the rate of increase in Fa/Fi the most for isoflurane. Blood/ gas solubility coefficients for the other volatile anesthetics are as follows: halothane 2.54, enflurane 1.90, sevoflurane 0.69, desflurane 0.42, and N2O 0.46 (Miller: Miller’s Anesthesia, ed 8, pp 647–650).
341. Select the correct order from greatest to least for anesthetic requirement. A. Adults > infants > neonates B. Adults > neonates > infants C. Infants > neonates > adults D. Neonates > adults > infants
341. (C) Anesthetic requirement increases from birth until approximately age 3 to 6 months. Then, with the exception of a slight increase at puberty, anesthetic requirement progressively declines with aging. For example, the MAC for halothane in neonates is approximately 0.87%, in infants it is approximately 1.2%, and in young adults it is approximately 0.75%. A notable exception to this pattern is seen with sevoflurane, for which MAC is the highest with neonates. If the question pertained only to sevoflurane, the correct response would have been C. Please review the answer to Question 321 (Miller: Miller’s Anesthesia, ed 8, p 2764).
342. Which of the following MOST closely determines anesthetic effect? A. Volume percent administered to patient B. Partial pressure at the level of the central nervous system (CNS) C. Solubility in blood D. End-tidal concentration 92 Part 1 Basic Sciences
342. (B) The exact mechanism in which volatile anesthetics exert their effects is not fully understood and remains a topic of considerable research. The most obvious effect of general anesthesia, unconsciousness (hypnosis), is produced at the level of the brain. The end-tidal concentration of the volatile in question reflects the level of anesthesia “seen” by the brain, but only once equilibrium has been reached. At equilibrium, Palveolar = Parterial = PCNS. After three (95% equilibrium) to four (99% equilibrium) time constants, the end-tidal concentration and the partial pressure of the anesthetic at the brain (and blood for that matter) would be the same, provided delivery has remained constant. A time constant is defined as capacity (of the brain) divided by flow (of anesthetic-laden blood) and is expressed by the following equation: τ = V λ ÷ Q The time constant, τ, is about 3 to 4 minutes for modern volatile anesthetics. Accordingly, 10 to 15 minutes must elapse before assuming that the partial pressure of the anesthetic has reached equilibrium in the brain. For this reason, choice D is an incorrect response for this question, because no mention is made of time (Barash: Clinical Anesthesia, ed 7, pp 447–454; Miller: Basics of Anesthesia, ed 6, p 86; Hemmings: Pharmacology and Physiology for Anesthesia, ed 1, pp 50–51).
343. A 31-year-old moderately obese woman is receiving a general anesthetic for cervical spinal fusion. After induction and intubation, the patient is mechanically ventilated with isoflurane at a vaporizer setting of 2.4%. The N2O flow is set at 500 mL/min, and the oxygen flowmeter is set at 250 mL/min. The infrared spectrometer displays an inspired isoflurane concentration of 1.7% and an expired isoflurane concentration of 0.6%. Approximately how many MAC of anesthesia would be represented by the alveolar concentration of anesthetic gases? A. 0.85 MAC B. 1.1 MAC C. 1.8 MAC D. 2.1 MAC
343. (B) Two principles of MAC must be considered in this situation. First, MAC is additive, so the fraction of MAC of each individual gas must be added to arrive at total MAC. The second is that alveolar concentrations of soluble agents are reflected more accurately by end-expiratory concentrations rather than by either inspiratory concentrations or gradients between inspiratory and expiratory concentrations. 100 Part 1 Basic Sciences Because N2O is very insoluble, it is reasonable to assume that equilibrium will be established early. The inspiratory concentration of N2O, approximately 0.6 MAC, should approximate the alveolar concentration. However, the expiratory concentrations of the more soluble volatile anesthetics should be used to estimate the alveolar concentration. The end-expiratory isoflurane concentration of 0.6 reflects approximately 0.5 MAC, which in addition to the 0.6 MAC of N2O would be closest to answer C: 1.1 MAC (Miller: Basics of Anesthesia, ed 6, pp 83–84).
344. The graph in the figure depicts A. The second gas effect B. The concentration effect C. The concentrating effect D. The effect of solubility on the rate of rise of Fa/Fi
344. (B) The figure shown in this question depicts the concentration effect. Note that the inspired anesthetic concentration influences not only the maximum attainable alveolar concentration but also the rate at which the maximum alveolar concentration can be attained. The greater the inhaled anesthetic concentration, the faster the increase in Fa/Fi (Miller: Basics of Anesthesia, ed 6, pp 84–85).
345. The rate of induction of anesthesia with isoflurane would be slower than expected in patients A. With anemia B. With chronic renal failure C. In shock D. With a right-to-left intracardiac shunt
345. (D) The depth of general anesthesia is directly proportional to the alveolar anesthetic partial pressure. The faster the rate of increase in Fa/Fi, the faster the induction of anesthesia. With the exception of a rightto- left intracardiac shunt (see explanation to Question 337 on effect of shunt on the rate of increase in Fa/Fi and explanation to Question 346 on the effect of shunt on arterial anesthetic partial pressure and rate of induction of anesthesia), all of the conditions listed in this question will accelerate the rate of increase in Fa/Fi and thus the rate of induction of anesthesia (Stoelting: Pharmacology and Physiology in Anesthetic Practice, ed 4, p 30).
346. A right-to-left intracardiac shunt would have the GREATEST impact on the rate of inhalation induction with which of the following inhalation anesthetics? A. Desflurane B. Isoflurane C. It would speed up induction for all agents equally D. It would slow down induction for all agents equally
346. (A) In general, a right-to-left intracardiac shunt or transpulmonary shunt will slow the rate of induction of anesthesia. This occurs because of a dilutional effect of shunted blood, which contains no volatile anesthetic, on the arterial anesthetic partial pressure coming from ventilated alveoli. The impact of a rightto- left shunt on the rate of increase in pulmonary arterial anesthetic partial pressure, and ultimately the rate of induction of anesthesia, is greatest for poorly soluble volatile anesthetics. This occurs because the uptake of poorly soluble volatile anesthetics into pulmonary venous blood is minimal; thus, the dilutional effect of the shunt on pulmonary venous anesthetic partial pressure is essentially unopposed. In contrast, the uptake of highly soluble volatile anesthetics is sufficient to partially offset the dilutional effect. Of the anesthetics listed in the question, desflurane is the least soluble (Miller: Miller’s Anesthesia, ed 8, p 645).
347. A left-to-right tissue shunt, such as arteriovenous fistula, physiologically most resembles which of the following? A. A left-to-right intracardiac shunt B. A right-to-left intracardiac shunt C. Ventilation of unperfused alveoli D. A pulmonary embolism
347. (A) Both a left-to-right intracardiac shunt and a left-to-right tissue shunt, such as an arteriovenous fistula, will result in a higher partial pressure of anesthetic gas in the blood returning to the lungs, ultimately resulting in a more rapid rise in Fa/Fi. However, this effect is minimal and in most cases is clinically insignificant (Stoelting: Pharmacology and Physiology in Anesthetic Practice, ed 4, p 30).
348. A fresh gas flow rate of 2 L/min or greater is recommended for administration of sevoflurane because A. The vaporizer cannot accurately deliver the volatile at lesser flow rates B. It prevents the formation of fluoride ions C. It prevents the formation of compound A D. It diminishes rebreathing
348. (D) Sevoflurane is a highly insoluble volatile anesthetic that combines with CO2 absorbents to form a vinyl ether known as compound A. The blood/gas partition coefficient for sevoflurane is 0.69. The vaporizer manufactured by Ohmeda is capable of delivering concentrations ranging from 0.2% to 8% at fresh-gas flow rates of 0.2 to 15 L/min. Its vapor pressure is 160 mm Hg at 20° C, which is similar to the vapor pressure for the other volatile anesthetics except desflurane (664 mm Hg at 20° C). Gas flows greater than 2 L/min prevent the rebreathing of compound A (not the formation of it), thus reducing the possibility of renal toxicity associated with it (Miller: Miller’s Anesthesia, ed 8, p 662).
349. A left-to-right shunt in a neonate with a patent ductus arteriosus (PDA) has what effect on inhalation induction? A. Speeds it up B. Slows down with insoluble volatile agents C. Slows with soluble volatile agents D. No effect with any volatile agent
349. (D) Left-to-right shunts (e.g., PDA, atrial septal defect, ventricular septal defect) are associated with an increase in blood flow through the lungs. With inhalation induction there is no real effect on induction rate. Remember also that a decrease in systemic vascular resistance seen with inhalation agents (e.g., sevoflurane) and positive-pressure ventilation tend to decrease the magnitude of the left-to-right shunt. However, with right-to-left shunts (e.g., tetralogy of Fallot) there is decreased blood flow through the lungs and a slower inhalation induction. With a right-to-left shunt, the decrease in systemic vascular resistance can increase the shunt and lead to a decrease in oxygenation. Intravenous drugs work more rapidly in right-toleft shunts. Halothane may be preferred (to sevoflurane) in right-to-left shunts because halothane decreases contractility and better maintains systemic vascular resistance (Miller: Basics of Anesthesia, ed 6, p 551).
350. Smokers are MOST likely to show a mild but transient increase in airway resistance after intubation and general anesthesia with which of the following? A. Isoflurane B. Sevoflurane C. Halothane D. Desflurane
350. (D) Volatile anesthetics produce minimal bronchodilation unless airway resistance is increased (bronchospasm). This is explained by the fact that airway smooth muscle tone is ordinarily low, and additional bronchodilation is difficult to demonstrate. The irritating effects of desflurane can be reduced by prior administration of fentanyl or morphine (Miller: Basics of Anesthesia, ed 6, p 95). Pharmacology and Pharmacokinetics of Volatile Anesthetics 101
351. If a patient is anesthetized with 6% desflurane in a hyperbaric chamber at 1 atm and the pressure is increased to 2 atm, the desflurane dial should be set to which setting if the anesthesia provider wishes to maintain the anesthetic at the same level? A. 3% B. 6% C. 12% D. Cannot be determined without knowledge of Fio2
351. (A) Please see also Question 342 and its answer. The determinant of anesthetic effect is partial pressure, ultimately at the CNS. If a patient is in a hyperbaric chamber under 2 atm (1520 torr), the effective partial pressure from a desflurane vaporizer would be doubled for any given dial setting in comparison with sea level. A 6% setting at sea level would be 760 × 0.06, or 45.6 mm Hg desflurane. The desflurane vaporizer is unique in that it is more akin to a dual gas blender. To achieve a partial pressure of 45.6 mm Hg (at 2 atm), the dial should be set at 3% (Miller: Miller’s Anesthesia, ed 8, pp 771–772).
352. 0.5 1 0 60 80 70 50 0 5 10 PaO2 (mm Hg) N2O output (L/min) Minutes from end of N2O anesthesia Mean, SE N2O PaO2 The graph above depicts which of the following? A. Diffusion hypoxia B. Second gas effect C. Context sensitive half-time of desflurane D. Concentration effect
352. (A) This classic graph depicts the effect of switching from 21% oxygen and 79% N2O to 21% oxygen and 79% nitrogen—that is, air. When this occurs, large volumes of N2O are released into the lungs and dilute all gases, including oxygen and CO2. The reduction in O2 results in hypoxia, and the resulting fall in CO2 reduces the drive to breathe. This combination occurs at a time when most patients have narcotics and other respiratory depressants in the body. For this reason, it is wise to administer 100% oxygen to patients for several minutes after they emerge from general anesthesia (Miller: Miller’s Anesthesia, ed 8, pp 656–657).
353. Which of the following organs is NOT considered a member of the vessel-rich group? A. Lungs B. Brain C. Heart D. Kidney
353. (A) The vessel-rich group receives 75% of the cardiac output and represents 10% of the weight of a lean adult. In a sense, the lungs receive virtually 100% of the cardiac output, but this is the right-sided CO (the supply side for oxygen) and therefore does not “count” in the classic definition. Lung parenchyma, ironically, uses a very small quantity of oxygen compared with the brain, liver, kidney, and myocardium (Miller: Miller’s Anesthesia, ed 8, p 648).
354. In isovolumic normal human subjects, 1 MAC of isoflurane anesthesia depresses mean arterial pressure by approximately 25%. The single BEST explanation for this is A. Reduction in heart rate B. Venous pooling C. Myocardial depression D. Decreased systemic vascular resistance
354. (D) At 1 MAC concentrations, isoflurane depresses mean arterial pressures primarily by decreasing systemic vascular resistance. The decrease in mean arterial pressure may be greater than that seen with the administration of halothane. However, heart rate will be increased, and stroke volume will decrease to a lesser extent than is seen with the administration of 1 MAC halothane (Miller: Miller’s Anesthesia, ed 8, p 713).
355. If cardiac output and Va are doubled, the effect on the rate of rise of Fa/Fi for isoflurane compared with that which existed immediately before these interventions will be A. Doubled B. Somewhat increased C. Unchanged D. Somewhat decreased
355. (B) Changes in both cardiac output and Va will affect the rates of rise of Fa/Fi, but in opposite directions. An increase in cardiac output will decrease the rate of Fa/Fi, whereas an increase in Va will increase the rate of Fa/Fi. However, these two opposing options do not completely offset each other because the increased cardiac output also accelerates the equilibrium of the anesthetic between the blood and the tissues. This equilibrium results in a narrowing of the alveolar-to-venous partial pressure difference and attenuates the impact of the increased cardiac output on uptake. The net result will be a slight increase in the rate of rise of Fa/Fi (Miller: Miller’s Anesthesia, ed 8, p 646). Doubled Normal Doubled Normal Doubled Normal Nitrous oxide Halothane Methoxyflurane
356. Which of the following characteristics of inhaled anesthetics most closely correlates with recovery from inhaled anesthesia? A. Blood/gas partition coefficient B. Brain/blood partition coefficient C. Fat/blood partition coefficient D. MAC
356. (A) Blood/gas partition coefficient is the option listed that most closely correlates with recovery from inhaled anesthesia. A higher blood/gas partition coefficient reflects a larger quantity of gas dissolved in the blood for a given alveolar concentration. Other factors that affect emergence from anesthesia include Va, cardiac output, tissue concentrations, and metabolism (Miller: Miller’s Anesthesia, ed 8, p 654).
357. After a 12-hour 60% N2O-desflurane anesthetic, evidence of N2O can be best detected by histologic examination of A. Bone marrow B. Renal tubules C. Hepatocytes D. None of the above
357. (A) N2O interferes with the enzyme methionine synthetase, which catalyzes the conversion of homocysteine to methionine. Chronic exposure to N2O leads to a disease state similar to vitamin B12 deficiency, but with one important difference: it is not alleviated with vitamin B12 supplementation. In healthy patients, megaloblastic changes can be seen in the bone marrow after just 12 hours of exposure to 50% N2O (or higher). In patients who are seriously ill, these changes can be seen even earlier. The other disease caused by vitamin B12 deficiency, subacute combined degeneration of the spinal cord, appears only after months of exposure, as is seen in long-term N2O abusers (Miller: Miller’s Anesthesia, ed 8, p 664).
358. An unconscious, spontaneously breathing patient is brought to the operating room (OR) from the intensive care unit for wound débridement. Which of the following maneuvers would serve to slow induction of inhalational anesthesia through the tracheostomy? A. Using isoflurane instead of sevoflurane (using MAC-equivalent inspired concentrations) B. Increasing fresh gas flow from 2 to 6 L/min C. Esmolol 30 mg intravenously D. None of the above
358. (A) Four main factors affect the total or rate of rise of the alveolar concentration of anesthetic (Fa) and hence the inhalation induction of anesthetics. These factors are the inspired concentration of anesthetic (Fi), the solubility of the anesthetic, the Va, and the cardiac output. The rate of rise in Fa/Fi is faster with the less soluble anesthetics, as noted by the blood/gas partition coefficients. The blood/gas partition coefficient measured at 37° C is the least with desflurane (0.45), followed closely by N2O (0.47), then sevoflurane (0.65), isoflurane (1.4), enflurane (1.8), and halothane (2.5); it is the highest with ether (12). Thus, replacing sevoflurane with isoflurane would slow down induction. Increasing the minute ventilation as well as increasing the fresh gas flow rate allows more of the anesthetic to get into the lungs and offset the uptake of anesthetic by the blood, thus speeding the induction of inhalational anesthesia. Decreasing the cardiac output also accelerates the rise of Fa/Fi, resulting in faster inhalation induction (decreases the amount of blood exposed to the lung and decreases the uptake of anesthesia) (Miller: Miller’s Anesthesia, ed 8, pp 647–650; Miller: Basics of Anesthesia, ed 6, pp 84–87).
359. Which of the settings below would give the highest arterial oxygen concentration during inhalation induction of general anesthesia with sevoflurane? Oxygen Air N2O A. L/min 1 2 0 B. L/min 2 0 2 C. L/min 2 2 2 D. L/min 2 3.5 0
359. (B) The table below contains a fifth column, Fio2. Choices B and D appear to be tied at 50%. The question asks for arterial oxygen concentration (not Fio2). During induction of general anesthesia, N2O is rapidly taken up into the blood, resulting in the so-called second gas effect and a concentrating effect. Concentration of oxygen in this manner is termed “alveolar hyperoxygenation” and results in a transient increase in Pao2 of approximately 10% (Miller: Basics of Anesthesia, ed 6, p 85). Oxygen Air N2O Fio2 A. L/min 1 2 0 0.47 B. L/min 2 0 2 0.50 C. L/min 2 2 2 0.40 D. L/min 2 3.5 0 0.50
360. If a patient were anesthetized 90 minutes with 1.25 MAC isoflurane followed by 30 minutes of 1.25 MAC sevoflurane anesthesia, wake-up would be A. The same as 2 hours of isoflurane anesthesia B. The same as 2 hours of sevoflurane anesthesia C. Less than 2 hours of isoflurane anesthesia, but greater than 2 hours of sevoflurane D. Greater than 2 hours of isoflurane anesthesia
360. (A) The insoluble volatile agent desflurane has the advantage of rapid washout and therefore rapid recovery. The downside is the higher cost of desflurane compared with isoflurane. A study was devised to test wake-up after volunteers were anesthetized with isoflurane for the first 75% of the anesthetic and switched to sevoflurane for the last 25%. The results showed that the “hybrid” lasted as long as an anesthetic that consisted of isoflurane alone and proved the futility of this strategy (Miller: Miller’s Anesthesia, ed 8, pp 656–657).
361. An anesthesia circuit is primed in preparation for an inhalation induction (with open adjustable pressure-limiting valve). The anesthesia hose is occluded with a flow of 6 L/min. The anesthesia circuit (canisters, hoses, mask, anesthesia bag) contains 6 L. A machine malfunction allows administration of 100% N2O. Approximately how much N2O would there be in the circuit when the malfunction is discovered at the 1-minute mark? A. 32% B. 48% C. 63% D. 86%
361. (C) Calculation of the washin of N2O requires use of the concept of time constant. Given a volume of 6 L for the circle system, the time constant is 6 L/(6 L/min) or 1 minute. The numbers to remember for time constants are 63%, 84%, and 95% for 1, 2 and 3 time constants, respectively. A properly functioning anesthesia machine would never allow the administration of 100% N2O, but this nightmare scenario is given purely for illustrative purposes (Barash: Clinical Anesthesia, ed 7, p 451).
362. Which of the following factors lowers MAC for volatile anesthetics? A. Serum sodium 151 mEq/L B. Red hair C. Body temperature 38° C D. Acute ethanol ingestion 94 Part 1 Basic Sciences
362. (D) Acute ethanol ingestion is the only factor listed that will reduce MAC. Acute amphetamine ingestion raises MAC, as do hypernatremia, hyperthermia, and naturally occurring red hair. Gender, thyroid function, and Paco2 between 15 and 95 mm Hg and Pao2 greater than 38 mm Hg have no effect on MAC (Miller: Basics of Anesthesia, ed 6, p 82). Pharmacology and Pharmacokinetics of Volatile Anesthetics 103
363. Each of the following factors can influence the partial pressure gradient necessary for the achievement of anesthesia EXCEPT A. Inspired anesthetic concentration B. Cardiac output C. Va D. Ventilation of nonperfused alveoli (dead space)
363. (D) This table summarizes the factors that influence the partial pressure gradients. A right-to-left intrapulmonary shunt affects the delivery of inhaled anesthetics, but lung dead space does not, because the latter does not produce a dilutional effect on the arterial partial pressure of the anesthetic in question (Miller: Basics of Anesthesia, ed 6, pp 84–87). FACTORS DETERMINING PARTIAL PRESSURE GRADIENTS NECESSARY FOR ESTABLISHMENT OF ANESTHESIA Input from Anesthesia Machine to Alveoli Uptake from Alveoli to Pulmonary Blood Uptake from Arterial Blood to Brain Inspired anesthetic concentration Blood gas partition coefficient Brain/blood partition coefficient Alveolar ventilation Cardiac output Cerebral blood flow Characteristics of the anesthesia breathing system Alveolar-to-venous partial pressure difference Arterial-to-venous partial pressure difference From Stoelting RK, Miller RD: Basics of Anesthesia, ed 4, New York, Churchill Livingstone, 2000, p 26.
364. Which of the following volatile anesthetics is unique in containing preservative? A. Sevoflurane B. Desflurane C. Isoflurane D. None of the above
364. (D) Halothane was the only “modern” volatile anesthetic (methoxyflurane also contained a preservative) that contains a preservative, thymol. Because halothane was at risk for degradation into chloride, hydrochloric acid, bromide, hydrobromic acid, and phosgene, it was stored in amber-colored bottles, and thymol was added to prevent spontaneous oxidation. None of the currently used volatile agents contains a preservative (Stoelting: Pharmacology and Physiology in Anesthetic Practice, ed 4, p 44).
365. If the alveolar-to-venous partial pressure difference of a volatile anesthetic (Pa − Pv) is positive (i.e., Pa > Pv) and the arterial-to-venous partial pressure difference (Pa − Pv) is negative (i.e., Pv > Pa), which of the following scenarios is MOST likely to be true? A. The vaporizer has been shut off at the end of the case B. Induction has just started C. Steady state has been achieved D. The vaporizer was shut off during emergence, then turned back on
365. (D) The delivery of anesthetic gases to a patient is a complex series of events that starts with the anesthesia machine and culminates with achievement of an anesthetic partial pressure in the brain (PBr). The partial pressure measured in the blood for any volatile agent is either rising (at first rapidly, then more slowly) or falling (rapidly at first, then more slowly). The vessel-rich group reaches steady state in about 12 minutes (for any dialed level of volatile agent). The rest of the body, however, approaches, but virtually never reaches, equilibrium (e.g., the equilibrium half-time for the fat group is 30 hours for sevoflurane). Hence, a true zero gradient is never achieved in the steady state. When the anesthetic is discontinued or reduced, there is a fall in the arterial partial pressure such that it is less than the venous partial pressure. In fact, when the venous partial pressure exceeds the arterial partial pressure, it means that the volatile agent has been reduced (or shut off) because the lungs are “cleansing” the blood as the volatile-filled blood passes through them. The newly “cleansed” blood then finds its way to the left ventricle with a very low Pa for the volatile agent in question (Barash: Anesthesiology, ed 7, pp 450–453).
366. Anesthetic loss to the plastic and rubber components of the anesthetic circuit, hindering achievement of an adequate inspired concentration, is a factor with which of the following anesthetics? A. Desflurane B. Isoflurane C. Sevoflurane D. N2O
366. (B) Anesthetic agents are soluble in the rubber and plastic components found in the anesthesia machine. This fact can impede the development of anesthetic concentrations of these drugs. The worst offender is the obsolete volatile agent methoxyflurane. However, both isoflurane and halothane are soluble in rubber and plastic, but to a lesser degree. Sevoflurane, desflurane, and N2O have little or no solubility in rubber or plastic. A different but important issue should be borne in mind regarding the loss of sevoflurane. This agent can be destroyed in appreciable quantities by Baralyme (no longer available) and soda lime, but not calcium hydroxide lime (Amsorb) (Miller: Miller’s Anesthesia, ed 8, pp 660–661).
367. Factors predisposing to formation and/or rebreathing of compound A include each of the following EXCEPT A. Low fresh gas flow B. Use of calcium hydroxide lime rather than soda lime C. High absorbent temperatures D. Fresh absorbent
367. (B) Compound A is an ether that forms when sevoflurane interacts with absorbent granules. In rats, compound A is a nephrotoxin that causes damage to the proximal renal tubule. It is believed that compound A is not nephrotoxic in humans, at least not at the concentrations that are achieved clinically (even with fresh gas flows as low as 1 L/min). The factors that lead to increased concentrations of compound A are use of fresh absorbent, use of Baralyme instead of soda lime, high absorbent temperatures, higher concentrations of sevoflurane in the anesthesia system, and closed-circuit or low-flow anesthesia. Calcium hydroxide lime (Amsorb) does not contain KOH or NaOH and does not interact with sevoflurane to produce compound A or other volatile agents to produce carbon monoxide (Miller: Miller’s Anesthesia, ed 8, p 790).
368. The effects of a left-to-right shunt such as an arteriovenous fistula on inhalation induction of anesthesia is to A. Speed up induction B. Slow down induction C. Slow down inhalation induction only if an intracardiac (right-to-left) shunt also exists D. Speed up inhalation induction only if an intracardiac (right-to-left) shunt also exists
368. (D) A left-to-right peripheral shunt such as an arteriovenous fistula delivers volatile-containing venous blood to the lungs. This action offsets the dilutional effect of a right-to-left intracardiac or pulmonary shunt and speeds up induction. The increase in the anesthetic partial pressure from an arteriovenous fistula is detectable only in the setting of a concomitant right-to-left shunt (Miller: Basics of Anesthesia, ed 6, p 87). 104 Part 1 Basic Sciences
369. The following volatile agents are correctly matched with their degree of metabolism (determined by metabolite recovery): A. Sevoflurane 2% B. Isoflurane 0.2% C. Desflurane 0.02% D. All are correctly matched
369. (D) Each of the volatile agents is correctly paired with its percentage of recovered metabolites. Sevoflurane is metabolized 2% to 5% through oxidative pathways using the cytochrome P-450 enzyme pathway. Likewise, the other volatile agents are all oxidatively metabolized in varying degrees. The obsolete anesthetic methoxyflurane underwent 50% metabolism, resulting in high concentrations of fluoride ions and resultant renal failure in some patients. Halothane is unique among the volatile agents in that it can undergo reductive metabolism in the face of low oxygen availability in the liver (Stoelting: Pharmacology and Physiology in Anesthetic Practice, ed 4, pp 77–80).
370. Which of the components below is NOT considered in the process of “washin” of the anesthesia circuit at the onset of administration? A. Infrared spectrometer tubing and reservoir B. Expiratory limb C. Anesthesia bag D. CO2 absorber
370. (A) By definition, the washin of the anesthesia circuit refers to the filling of the components of the circuit with anesthetic gases. The total washin volumes are around 7 L and break down as follows: anesthesia bag 3 L, anesthesia hoses 2 L, and anesthesia absorbent compartment 2 L. All of the components listed are part of the anesthesia circuit except the infrared spectrometer tubing. The infrared spectrometer and mass spectrometer take away (sample) from incoming gases through aspiration but do not dilute them (Miller: Miller’s Anesthesia, ed 8, pp 660–661).
371. Which of the following maneuvers would NOT increase the rate of an inhalation induction? A. Giving the patient an inotropic infusion B. Substituting sevoflurane for isoflurane C. Overpressurizing D. Carrying out the induction in San Diego instead of Denver
371. (A) Increasing minute ventilation is one of two methods for manipulating ventilation to increase the rate of establishing anesthesia. Another method is increasing inspired concentration, which can be achieved by turning up the dial above the desired steady state concentration (overpressurizing) to reach steady state more quickly, or increasing fresh gas flow to reduce or eliminate rebreathing (dilution). Substituting a less soluble anesthetic, such as sevoflurane for isoflurane, also establishes anesthesia more rapidly. Carrying out the induction in San Diego instead of Denver constitutes administering the anesthetic at higher atmospheric (barometric) pressure, which decreases the uptake and hence increases the rate of rise of Fa/Fi—that is, accelerates the establishment of anesthesia. The administration of an inotropic agent increases cardiac output, which also increases uptake and slows the rate of induction (Barash: Clinical Anesthesia, ed 7, pp 451–454; Miller: Basics of Anesthesia, ed 6, pp 84–88).
372. Which of the following anesthetics would undergo 90% elimination the most rapidly after a 6-hour Whipple procedure under 1 MAC for the duration of the operation? A. Isoflurane B. Sevoflurane C. Desflurane D. Sevoflurane and desflurane are tied
372. (C) In a comparison of the pharmacokinetics of elimination for volatile anesthetics, desflurane is the fastest. The time for a 50% reduction (decrement) in the alveolar partial pressure of the “modern” anesthetics is roughly the same: about 5 minutes, regardless of anesthetic duration. For longer anesthetics, however, the 80% and 90% decrement times become markedly different. In the present example, the 90% decrement time for desflurane after a 6-hour anesthetic is 14 minutes. This is in stark contrast to sevoflurane (65 minutes) and isoflurane (86 minutes). Please see Question 376 and its explanation (Miller: Basics of Anesthesia, ed 6, pp 88–90; Miller: Miller’s Anesthesia, ed 8, pp 654–655).
373. After induction and intubation of a healthy patient and institution of a ventilator, the sevoflurane vaporizer is set at 2%, and fresh gas flow is 1 L/min (50% N2O and 50% O2). The inspired concentration on the infrared spectrometer 1 minute later is 1.4%. The MAIN reason for the difference between the dial setting and the concentration shown on the infrared spectrometer is A. Rapid uptake of sevoflurane B. Insufficient fresh gas flow for correct vaporizer function C. Second gas effect D. Dilution
373. (D) A properly functioning vaporizer will produce the concentration set on the dial (plus or minus a small tolerance) provided the fresh gas flow rate is greater than 250 mL/min and less than 15 L/min. The 1 L/min rate in this question is well within the limits of the vaporizer. The fact that rebreathing occurs with a circular anesthesia system causes a significant dilutional effect. It is true that uptake would enhance dilution, but it (uptake), per se, is not the main reason for this discrepancy. Uptake is considered in the discussion of the Fa/Fi ratio. This question addresses the characteristics of the anesthesia machine and the relationship between dial setting and delivered concentration. To achieve a desired concentration (e.g., 2%), you must either raise the fresh gas flow to convert the system to a nonrebreathing system or set the vaporizer to a higher level than is actually desired: the concept of overpressurization. In this era of cost containment, the latter is more economical (Miller: Basics of Anesthesia, ed 6, p 207).
374. After cessation of general anesthesia that consisted of air, oxygen, and a volatile agent only, the patient is given 100% oxygen. Each of the following serves as a reservoir for volatile anesthesia and may delay emergence EXCEPT A. Rebreathed exhaled gases B. The absorbent C. The patient D. Gases emerging from the common gas outlet Pharmacology and Pharmacokinetics of Volatile Anesthetics 95
374. (D) The anesthesia circuit can delay emergence significantly if the patient is not disconnected (functionally) from it. Anesthetic gases become dissolved in the rubber and plastic components of the breathing circuit. Likewise, the soda lime can serve as a depository for anesthetics as well as the patient’s own exhaled gases. To reduce these effects to nearly zero, the fresh gas flow should be raised to at least 5 L/min. Fresh gases emerge via the common gas outlet and do not contain volatile agents or N2O because they (volatile agents and N2O) are shut off during emergence (Miller: Miller’s Anesthesia, ed 8, pp 660–661).
375. Which of the following characteristics of volatile anesthetics is necessary for calculation of the time constant? A. Blood/gas partition coefficient B. Brain/blood partition coefficient C. Oil/gas partition coefficient D. All of the above
375. (B) The time constant is defined as capacity divided by flow. The time constant for a volatile anesthetic is determined by the capacity of a tissue to hold the anesthetic relative to the tissue blood flow. The capacity of a tissue to hold a volatile anesthetic depends both on the size of the tissue and on the affinity of the tissue for the anesthetic. The brain time constant of a volatile anesthetic can be estimated by doubling the brain/blood partition coefficient for the volatile anesthetic. For example, the time constant of halothane Pharmacology and Pharmacokinetics of Volatile Anesthetics 105 (brain/blood partition coefficient of 2.6) for the brain (mass of approximately 1500 g, blood flow of 750 mL/min) is approximately 5.2 minutes (Eger: Anesthetic Uptake and Action, ed 1, pp 85–87; Miller: Basics of Anesthesia, ed 6, p 86).
376. The concept of “context sensitive half-time” emphasizes the importance of the relationship between half time and A. Va B. Blood solubility C. Concentration D. Duration
376. (D) This concept highlights the fact that the difference in half-time values among the volatile anesthetics is similar for all volatiles if the anesthetic duration is very brief. With the administration of volatile anesthetics for longer times, the differences in recovery time become more profound. For example, after a 1-hour anesthetic with desflurane (blood/gas tissue coefficient 0.45), a 95% reduction in the alveolar concentration can be reached in 5 minutes. With an hour-long sevoflurane anesthetic (blood/gas tissue coefficient 0.65), a 95% reduction requires 18 minutes, and an hour-long isoflurane anesthetic (blood/gas tissue coefficient 1.4) requires more than 30 minutes to reach a 95% reduction in the alveolar concentration (Miller: Basics of Anesthesia, ed 6, pp 89–90; Miller: Miller’s Anesthesia, ed 8, pp 654–655).
377. Select the FALSE statement regarding pharmacokinetics for volatile anesthetics. After three time constants A. 6 to 12 minutes have elapsed for “modern anesthetics” B. The arterial-to-venous partial pressure difference (for the volatile) for the brain is very small C. The expired volatile concentration will rise much less slowly than in the preceding 12 minutes D. The venous blood will contain 95% of volatile content of arterial blood
377. (D) After a period of time equal to three time constants, the venous blood exiting the vessel-rich group will be at the 95% level, but the blood as a whole will have a level of less than 95%. The venous blood contains a mixture of blood from the vessel-rich group, the muscle group, the fat group, and the vessel-poor group, and at the three time constant mark will be less than 95% (Miller: Basics of Anesthesia, ed 6, pp 86–88).
378. Halothane (1 MAC) A No change No change Decreased B Decreased Decreased Decreased C Increased Decreased No change or slight increase D Increased Decreased Decreased
378. (A)The information for these questions is summarized in the graphs below. Halothane is unique among the volatile agents listed in that it does not affect the heart rate or systemic vascular resistance in the MAC ranges studied. Sevoflurane reduces heart rate until about 1 MAC, at which time it produces a dose-dependent increase in heart rate (Miller: Basics of Anesthesia, ed 6, pp 90–92).
379. Isoflurane (1 MAC) A No change No change Decreased B Decreased Decreased Decreased C Increased Decreased No change or slight increase D Increased Decreased Decreased
379. (C)The information for these questions is summarized in the graphs below. Halothane is unique among the volatile agents listed in that it does not affect the heart rate or systemic vascular resistance in the MAC ranges studied. Sevoflurane reduces heart rate until about 1 MAC, at which time it produces a dose-dependent increase in heart rate (Miller: Basics of Anesthesia, ed 6, pp 90–92).
380. Desflurane (1 MAC) A No change No change Decreased B Decreased Decreased Decreased C Increased Decreased No change or slight increase D Increased Decreased Decreased
380. (D) The information for these questions is summarized in the graphs below. Halothane is unique among the volatile agents listed in that it does not affect the heart rate or systemic vascular resistance in the MAC ranges studied. Sevoflurane reduces heart rate until about 1 MAC, at which time it produces a dose-dependent increase in heart rate (Miller: Basics of Anesthesia, ed 6, pp 90–92).
381. Sevoflurane (1 MAC) Heart Rate Systemic Vascular Resistance Cardiac Index A No change No change Decreased B Decreased Decreased Decreased C Increased Decreased No change or slight increase D Increased Decreased Decreased
381. (B) The information for these questions is summarized in the graphs below. Halothane is unique among the volatile agents listed in that it does not affect the heart rate or systemic vascular resistance in the MAC ranges studied. Sevoflurane reduces heart rate until about 1 MAC, at which time it produces a dose-dependent increase in heart rate (Miller: Basics of Anesthesia, ed 6, pp 90–92).
382. Each of the following treatments might be useful in restoring a prolonged prothrombin time (PT) to the normal range EXCEPT A. Recombinant factor VIII B. Vitamin K C. Fresh frozen plasma (FFP) D. Cryoprecipitate
382. (A) PT and aPTT are common tests used to evaluate coagulation factors. The PT primarily tests for factor VII in the extrinsic pathway, as well as factors I, II, V, and X of the common pathway. The aPTT primarily tests for factors VIII and IX of the intrinsic pathway, as well as factors I, II, V, and X of the common pathway. Although the PT is prolonged with deficient function of factors I, II, V, VII, or X, it is more sensitive to deficiencies of factor VII and less so with deficiencies of factor I or II. In fact, the PT is not prolonged until the level of fibrinogen (factor I) is less than 100 mg/dL and may be prolonged for only 2 seconds when the level of factor II (prothrombin) is 10% of normal. Factors II, VII, IX, and X are vitamin K–dependent factors, and their formation is blocked with Coumadin therapy. Administering factor VIII will not help a prolonged PT (Miller: Miller’s Anesthesia, ed 8, pp 1872–1874; Barash: Clinical Anesthesia, ed 7, pp 415–416).
383. Proper processing of platelet concentrates (to avoid future hemolytic transfusion reactions) before administration involves A. Type and crossmatching B. ABO and Rh matching C. Rh matching only D. ABO matching only
383. (C) Platelet concentrates contain a fair amount of plasma and white blood cells (WBCs) but relatively few red blood cells (RBCs). Although ABO-compatible platelet transfusions are preferred (platelets survive better, and crossmatching for subsequent RBCs is easier), in emergencies it has been noted that platelets often give adequate hemostasis without regard to ABO compatibility. Even though there are only small quantities of RBCs in platelets, the RBCs present can cause Rh immunization if Rh-positive platelet concentrates are injected into Rh-negative patients. Thus, until childbirth is no longer possible, Rh-negative women should receive only Rh-negative platelets (Miller: Miller’s Anesthesia, ed 8, p 1860; Hoffman: Hematology, ed 6, p 1655).
384. The most common inherited coagulopathy is A. Hemophilia A B. Hemophilia B C. von Willebrand disease (vWD) D. Factor V deficiency
384. (C) Coagulopathies can be inherited or acquired. Of the inherited coagulopathies, vWD is the most common, affecting 1 in 100 to 500 people. Both hemophilia A (factor VIII) deficiency and hemophilia B (factor IX or Christmas disease) are X-linked recessive disorders. Hemophilia A occurs in 1 to 2 per 10,000 male individuals, and hemophilia B occurs in 1 per 100,000 male individuals. Factor V, factor VII, factor X, and prothrombin (factor II) deficiencies are exceedingly rare autosomal recessive disorders (Miller: Miller’s Anesthesia, ed 8, p 1872; Barash: Clinical Anesthesia, ed 7, p 432).
385. In a 70-kg patient, 1 unit of platelet concentrate should increase the platelet count by A. 2000 to 5000/mm3 B. 5000 to 10,000/mm3 C. 15,000 to 20,000/mm3 D. 20,000 to 25,000/mm3
385. (B) Platelet count is increased about 5000 to 10,000/mm3 per unit of platelet concentrate in the typical 70- kg patient. Each unit contains greater than 5.5 × 1010 platelets (Miller: Miller’s Anesthesia, ed 8, pp 1840, 1860; Barash: Clinical Anesthesia, ed 7, p 421).
386. A 68-year-old patient receives a 1-unit transfusion of packed red blood cells (RBCs) in the recovery room after a laparoscopic prostatectomy. As the blood is slowly dripping into his peripheral intravenous line, the patient complains of itching on his chest and arms, but his vital signs remain stable. The antibody most likely responsible for this is directed against A. Rh B. ABO C. MN, P, and Lewis D. None of the above
386. (D) This is an example of a typical allergic reaction. All of the other choices in this question may be involved in hemolytic reactions. Allergic reactions are a form of nonhemolytic transfusion reactions, which are thought to be caused by foreign proteins in the transfused blood. The reactions occur in about 3% of all transfusions, and they present with urticaria, erythema, pruritus, fever, and sometimes respiratory symptoms. When such a reaction occurs, the transfusion is stopped and supportive therapy, including antihistamines, is administered. If the symptoms resolve and there are no signs of a hemolytic reaction (no free hemoglobin in the plasma or urine) or a severe anaphylactic reaction, the transfusion can be resumed (Miller: Miller’s Anesthesia, ed 8, p 1853; Barash: Clinical Anesthesia, ed 7, p 425).
387. The likelihood of a clinically significant hemolytic transfusion reaction resulting from administration of type-specific blood is less than A. 1 in 250 B. 1 in 500 C. 1 in 1000 D. 1 in 10,000
387. (C) Hemolytic transfusion reactions are often the result of clerical error. Three main blood compatibility tests can be performed to reduce the chance of a hemolytic reaction: ABO Rh typing, antibody screening, and crossmatching. With correct ABO and Rh typing, the possibility of an incompatible transfusion is less than 1 per 1000. If you add a type and screen, the possibility of an incompatible transfusion is less than 1 per 10,000. Optimal safety occurs when crossmatching is performed (Miller: Miller’s Anesthesia, ed 8, p 1840).
388. Frozen erythrocytes can be stored for A. 1 year B. 3 years C. 5 years D. 10 years
388. (D) Blood is most often stored as a liquid at about 4° C but can also be frozen for prolonged storage. Because of the added expense of frozen blood, it is used primarily for rare blood types and for autologous use. Blood that has already been collected has a cryoprotective agent (e.g., glycerol) added and is then frozen and stored at a temperature of −65° C (when 40% glycerol is used) or −120° C (when 20% glycerol is used). Currently, the U.S. Food and Drug Administration (FDA) allows frozen blood to be used up to 10 years from the time of collection (Barash: Clinical Anesthesia, ed 7, p 416). 112 Part 2 Clinical Sciences
389. Which of the following clotting factors has the shortest half-life? A. Factor II B. Factor V C. Factor VII D. Factor IX
389. (C) Factor VII is one of the four vitamin K–dependent clotting factors (factors II, VII, IX, and X). It also has the shortest half-life of all the clotting factors (4-6 hours) and is the first factor to become deficient in patients with severe hepatic failure, warfarin (Coumadin) anticoagulation therapy, and vitamin K deficiency. The PT is most sensitive to decreases in factor VII (Barash: Clinical Anesthesia, ed 7, pp 411–412).
390. Which of the measures below does NOT reduce the incidence of transfusion-related acute lung injury (TRALI)? A. Exclusion of female donors B. Use of autologous blood C. Leukocyte reduction D. Use of blood less than 14 days old
390. (C) TRALI occurs within 6 hours of blood component administration. Patients experience noncardiogenic pulmonary edema with acute bilateral pulmonary infiltrates and hypoxemia (PaO2/Fi O2 ≤300 mm Hg or oxygen saturation ≤90% on room air with no evidence of left atrial hypertension). The pathologic changes associated with TRALI are complex and may involve low-pressure pulmonary edema secondary to neutrophil activation and sequestration in the lungs. Older transfusion products (>14 days), female donors (especially multiparous patients), and pooled platelets compared with apheresis platelets are associated with a higher frequency of this condition. Interestingly, although leukocytes may be part of the activation process, leukocyte reduction does not seem to significantly decrease the incidence of TRALI but does decrease the incidence of febrile reactions and the risk of CMV, and it may decrease leukocyteinduced immunomodulation. Treatment for TRALI reactions is supportive (Barash: Clinical Anesthesia, ed 7, pp 417–428; Miller: Basics of Anesthesia, ed 6, p 376; Miller: Miller’s Anesthesia, ed 8, p 1859).
391. A 42-year-old woman is anesthetized for resection of a large (22-kg), highly vascular sarcoma in the abdomen. During the resection, 20 units of RBCs, 6 units of platelets, 10 units of cryoprecipitate, 5 units of FFP, and 1 L of albumin are administered. At the conclusion of the operation, the patient’s vital signs are stable, and she is transported to the intensive care unit. Three and a half hours later, a diagnosis of sepsis is made, and antibiotic therapy is started. Which of the items below would be the most likely cause of sepsis in this patient? A. Packed RBCs B. Cryoprecipitate C. Platelets D. FFP
391. (C) Of the five blood products listed in this question, platelets are the most likely to cause bacterial sepsis. Platelet-related sepsis is estimated to occur in 1 case per 12,000. The source of bacteria can be donor blood or contamination during the collection, processing, and storage of the blood. If platelets are cooled, then rewarmed, the platelets tend not to function very effectively. Because platelets are stored at room temperature of 20 to 24° C, bacteria tend to survive and multiply. All other listed blood products are cooled. Whole blood and packed RBCs are cooled to 4° C (unless they are frozen, which would be colder). FFP and cryoprecipitate are frozen to below −70° C. Albumin is heat sterilized, making it a sterile preparation that then can be safely stored at room temperatures (Miller: Miller’s Anesthesia, ed 8, pp 1859–1860; Barash: Clinical Anesthesia, ed 7, pp 423–425).
392. Blood is routinely screened (serologically) for A. Hepatitis A B. Severe acute respiratory syndrome (SARS) C. West Nile virus D. Bovine spongiform encephalitis (BSE, or mad cow disease)
392. (C) Hepatitis A transmission is very rare and is screened for by history alone (not serologically) because there is no carrier state for the virus and the disease is relatively mild. A decrease in the transmission for various other infectious agents has been attributed to the recent addition of nucleic acid testing (see table). At present, there are no screening tests available for malaria, Chagas, SARS, variant Creutzfeldt-Jakob disease, or BSE (Miller: Miller’s Anesthesia, ed 8, pp 1856–1858; Barash: Clinical Anesthesia, ed 7, pp 415–416). TESTS USED FOR DETECTING INFECTIOUS AGENTS IN ALL UNITS OF BLOOD, 2008 Virus RNA Minipool Antibody To Human immunodeficiency virus (HIV) Nucleic acid technology HIV-1, HIV-2 Hepatitis C virus (HCV) Nucleic acid technology HCV Hepatitis B virus (HBV) HBV Human T-cell lymphotropic virus (HTLV) HTLV-1, HTLV-2 West Nile virus Nucleic acid technology
393. The blood volume of a 10-kg, 1-year-old infant is A. 600 mL B. 800 mL C. 1000 mL D. 1300 mL
393. (B) Blood volume decreases with age. A preterm newborn has a blood volume of 100 to 120 mL/kg, a term newborn has a blood volume of about 90 mL/kg, an infant (3-12 months) has a blood volume of 80 mL/kg, a child older than 1 year has a blood volume of 70 mL/kg, and an adult has a blood volume of 65 mL/kg. This 10-kg, 1-year-old infant would have an estimated blood volume (EBV) of 800 mL (Barash: Clinical Anesthesia, ed 7, p 1246).
394. Which of the infections below is the most common transfusion-related infection? A. Human T-cell lymphotropic virus (HTLV)-II B. Hepatitis B C. Hepatitis C D. Human immunodeficiency virus (HIV)
394. (B) The risk of transfusion-transmitted infection with a unit of screened blood in the United States varies from study to study, but it is very infrequent with CMV because of leukocyte-reduced blood: 1 in 205,000 for hepatitis B, 1 in 1,935,000 for hepatitis C, 1 in 2,135,000 for HIV, 1 in 2,993,000 for HTLV-II, and 1 in more than 1,100,000 for West Nile virus. Thus, the most common transfusionassociated infection in the United States is now hepatitis B. The infective agent for syphilis does not survive at 4° C, making transmission unlikely for whole blood, packed RBCs, FFP, or cryoprecipitate. It is possible for platelets (stored at room temperature) to transmit syphilis (Miller: Miller’s Anesthesia, ed 8, pp 1856–1858). Blood Products, Transfusion, and Fluid Therapy 113
395. A 40-year-old, 78-kg patient with hemophilia A is scheduled for a right total knee arthroplasty. His laboratory test results show a hematocrit of 40, a factor VIII level of 0%, and no inhibitors to factor VIII. How much factor VIII concentrate do you need to give him to bring his factor VIII level to 100%? A. 3000 units B. 2500 units C. 2000 units D. 1500 units
395. (A) The most common type of hemophilia is hemophilia A, an X-linked recessive disease causing a reduction in factor VIII activity. The disease occurs with a frequency of 1 in 5000 male individuals. This disease can be severe (<1% factor VIII), moderate (1%-4% factor VIII), or mild (5%-30% factor VIII). Patients with mild hemophilia rarely have spontaneous bleeding. Laboratory studies show a normal platelet count and normal PT but a prolonged aPTT. The primary goal of preoperative preparation of patients with hemophilia A is to increase plasma factor VIII activity to a level that will ensure adequate hemostasis (i.e., 50%-100%), then maintain a level (>40% factor VIII levels) for 7 to 10 days. One unit of factor VIII is equal to 1 mL of 100% activity of normal plasma. Thus, to calculate the initial dose, first calculate the patient’s blood and then the plasma volume. Then calculate the amount of activity needed to increase the factor VIII level. In this case, the blood volume is 78 kg × 65 mL/kg, or about 5000 mL. Knowing that the RBC volume is 40% (i.e., hematocrit is 40) makes the plasma volume 60%. Thus, the plasma volume is 5000 mL × 0.6, or about 3000 mL. Because the patient is starting at 0% activity and you wish to raise it to 100% activity, you will need 3000 units. (If you wish to raise the activity by 40%, then 3000 mL of plasma × 0.4 for 40% activity = 1200 units.) In addition, because the half-life of factor VIII is about 12 hours, about 1500 units will remain after 12 hours. An infusion of 1500 units in 12 hours, or 125 units per hour, will be a good starting maintenance infusion rate. Factor VIII can be administered as factor VIII concentrate or cryoprecipitate (about 10 units/mL). Patients with factor VIII inhibitors (10%-20% of patients with hemophilia) require more factor VIII. Hematology consultation should be considered for all patients with hemophilia, and routine checking of factor VIII levels should be performed (Marx: Rosen’s Emergency Medicine, ed 8, p 1614).
396. A 38-year-old man is undergoing a total colectomy under general anesthesia. Urine output has been 20 mL/hr for the last 2 hours. Volume replacement has been adequate. The rationale for administering 5 to 10 mg of furosemide to this patient is to A. Offset the effects of increased antidiuretic hormone (ADH) B. Improve renal blood flow C. Convert oliguric renal failure to nonoliguric renal failure D. Offset the effects of increased renin
396. (A) Serum ADH levels increase during painful stimulation associated with surgery, as well as during positivepressure mechanical ventilation. Small doses of furosemide (i.e., 0.1 mg/kg) will counteract this effect during surgery (Miller: Miller’s Anesthesia, ed 8, p 1773; Barash: Clinical Anesthesia, ed 7, pp 344–345).
397. A 65-year-old man involved in a motor vehicle accident (MVA) is brought to the emergency room with a blood pressure of 60 mm Hg systolic. He is transfused with 4 units of type O, Rh-negative whole blood and 4 L of normal saline solution. After the patient is brought to the operating room, his blood type is determined to be A positive. Which of the following is the most appropriate blood type for further intraoperative transfusions? A. Type A, Rh-positive whole blood B. Type O, Rh-negative RBCs C. Type A, Rh-positive RBCs D. Type O, Rh-negative whole blood
397. (B) Type O, Rh-negative blood is also called universal donor blood because the transfused RBCs lack the antigens needed to be hemolyzed. Because the plasma of O-negative blood contains anti-A and anti-B antibodies, it is preferable to administer packed RBCs (with little plasma) over whole blood (lots of plasma) in an emergency. However, if two or more units of type O-negative, uncrossmatched whole blood are administered to a patient and subsequent blood typing reveals the patient’s blood type to be A, B, or AB, then switching back to the patient’s own blood type could lead to major intravascular hemolysis of the transfused RBCs and, therefore, is not advised. The use of type O-negative universal donor whole blood, or preferably RBCs, is recommended. In the male patient or the older female patient who will not have more children, type O-positive whole blood can be administered if few type O, Rh-negative units are available and massive transfusion is anticipated. Only after it is determined that the patient has low enough levels of transfused anti-A and anti-B antibodies should the correct type of blood be administered (Miller: Miller’s Anesthesia, ed 8, p 1840).
398. The criterion used to determine how long blood can be stored before transfusion is A. 90% of transfused erythrocytes must remain in circulation for 24 hours B. 70% of transfused erythrocytes must remain in circulation for 24 hours C. 70% of transfused erythrocytes must remain in circulation for 72 hours D. 75% of transfused erythrocytes must remain in circulation for 7 days
398. (B) The requirement for blood storage states that at least 70% of the erythrocytes must remain in circulation for 24 hours after a transfusion for the transfusion to be successful. Erythrocytes that survive longer than 24 hours after transfusion appear to have a normal life span (Miller: Miller’s Anesthesia, ed 8, p 1841).
399. The rationale for storage of platelets at room temperature (22° C) is A. There is less splenic sequestration B. It optimizes platelet function C. It reduces the chance for infection D. It decreases the incidence of allergic reactions Blood Products, Transfusion, and Fluid Therapy 109
399. (B) At a pH below 6.0 or in cold temperatures such as 4° C (the temperature used for blood storage), platelets undergo irreversible shape changes. The optimal temperature for platelet storage is 22° C ± 2° C, or room temperature. There are two major problems with platelet storage at this recommended temperature. First, the pH falls because of platelet metabolism. Second, bacterial growth is possible, which could potentially lead to sepsis and death. To minimize these problems, platelet storage is limited to 5 days at 22° C (Miller: Miller’s Anesthesia, ed 8, pp 1859–1861; Barash: Clinical Anesthesia, ed 7, pp 417–418).
400. An 18-year-old woman involved in an MVA is brought to the emergency room in shock. She is transfused with 10 units of type O, Rh-negative whole blood over 30 minutes. After infusion of the first 5 units, bleeding is controlled, and her blood pressure rises to 85/51 mm Hg. During the next 15 minutes, as the remaining 5 units are infused, her blood pressure slowly falls to 60 mm Hg systolic. The patient remains in sinus tachycardia at 120 beats/min, but the QT interval is noted to increase from 310 to 470 msec, and the central venous pressure increases from 9 to 20 mm Hg. Her breathing is rapid and shallow. The most likely cause of this scenario is A. Citrate toxicity B. Hyperkalemia C. Hemolytic transfusion reaction D. Tension pneumothorax
400. (A) Whole blood is rarely used today except in emergency cases when the rapid infusion of blood and volume is needed. Stored blood contains citrate, an anticoagulant that binds ionized calcium. When whole blood is rapidly transfused (i.e., >50 mL/70 kg/min) the citrate binds with calcium, producing transient decreases in ionized calcium. The abrupt decrease in ionized calcium can lead to prolonged QT intervals, an increase in left ventricular end-diastolic pressure, and arterial hypotension. Within 5 minutes of stopping the transfusion, ionized calcium levels return to normal. The volume of an average unit of whole blood is 500 mL. This patient received 10 units of whole blood, or 5000 mL, over 30 minutes, then another 5 units in 15 minutes. This averages to a rate greater than 160 mL/min (Miller: Miller’s Anesthesia, ed 8, pp 1840–1841). 114 Part 2 Clinical Sciences
401. A 20-kg, 5-year-old child with a hematocrit of 40% could lose how much blood and still maintain a hematocrit of 30%? A. 140 mL B. 250 mL C. 350 mL D. 450 mL
401. (C) A 20-kg, 5-year-old child has an EBV of 70 mL/kg = 1400 mL. The acceptable blood loss can be determined by use of the following formula: maximum allowable blood loss (in mL) = EBV × (Hcts − Hct1)/ Hcts where EBV is the estimated blood volume (in mL), Hcts is the starting hematocrit, and Hct1 is the lowest acceptable hematocrit. For this patient, the maximal allowable blood loss = 1400 × (40 − 30/40) = 1400 × (10/40) = 350 mL. This assumes that the patient is getting volume expansion with crystalloid (3 mL per mL of blood loss). Also see explanation to Question 393 (Barash: Clinical Anesthesia, ed 7, p 1246).
402. A 100-kg male patient has a measured serum sodium concentration of 105 mEq/L. How much sodium would be needed to bring the serum sodium to 120 mEq/L? A. 600 mEq B. 900 mEq C. 1200 mEq D. 1500 mEq
402. (B) The normal serum sodium concentration is 135 to 145 mEq/L. Hyponatremia occurs when the serum level is less than 135 mEq/L. Clinical symptoms correspond not only to the level of hyponatremia but also to how rapidly sodium levels are falling. Hyponatremia is most commonly not a deficiency in total body sodium but rather is an excess of total body water (e.g., absorption of irrigating fluids as seen in transurethral resection of the prostate syndrome, and syndrome of inappropriate antidiuretic hormone secretion). It can also be caused by an excessive loss of sodium, as is seen in severe sweating, vomiting, diarrhea, burns, and the use of diuretics. With acute falls in serum sodium, neurologic symptoms (confusion, restlessness, drowsiness, seizures, coma) resulting from cerebral edema can be seen at serum levels below 120 mEq/L. Cardiac symptoms (ventricular tachycardia, ventricular fibrillation) can be seen at levels below 100 mEq/L. Therapy for severe hyponatremia includes water restriction, loop diuretics, and at times the administration of hypertonic saline (3% NaCl). The dose of sodium needed for correction can be calculated by multiplying the total body water (TBW = body weight × 0.6) times the increase in sodium desired; that is, Dose of Na+ = Body weight × 0.6 × (desired Na+ level − current Na+ level in mEq/L) In this patient, the calculated dose of sodium would be 100 (weight in kg) × 0.6 × (120 mEq/L-105 mEq/L) = 900 mEq. Three percent NaCl is infused no faster than 100 mL/hr. Too rapid a correction may lead to central pontine myelinolysis. Once the level reaches 120 mEq/L, further treatment usually consists of water restriction and diuretics (Miller: Miller’s Anesthesia, ed 8, p 1773).
403. Paramedics respond to an MVA site and immediately stabilize the neck, secure the airway, and place an intravenous line into a 19-year-old 70-kg man lying in a pool of blood. Before the infusion is started, 3 milliliters of blood are withdrawn for hemoglobin and drug screening. The first responders estimate that the patient has lost one half of his entire blood volume. Given a starting value of 18 g/dL, the new value would likely be A. 9 g/dL B. 11 g/dL C. 14 g/dL D. 17 g/dL
403. (D) The intravascular half-life of crystalloid solution is 20 to 30 minutes, whereas the intravascular halflife of colloid is 3 to 6 hours. To restore intravascular volume, for each mL of blood lost, 3 to 4 mL of crystalloid or 1 mL of colloid is administered. In this case, though, the blood sample is drawn before the infusion is started, so the hemoglobin drawn should be similar to his hemoglobin concentration immediately before the MVA (Butterworth: Morgan & Mikhail’s Clinical Anesthesiology, ed 5, pp 1161–1164).
404. A 23-year-old woman who has been receiving total parenteral nutrition (TPN) (15% dextrose, 5% amino acids, and intralipids) for 3 weeks is scheduled for surgery for severe Crohn disease. Induction of anesthesia and tracheal intubation are uneventful. After peripheral intravenous access is established, the old central line is removed and a new central line is placed at a different site. At the end of the operation, a large volume of fluid is discovered in the chest cavity on chest x-ray film. Arterial blood pressure is 105/70 mm Hg, heart rate is 150 beats/min, and Sao2 is 96% (pulse oximeter). The most appropriate initial step is to A. Place a chest tube B. Change the single-lumen to a double-lumen endotracheal tube C. Start a dopamine infusion D. Check the blood glucose level
404. (D) Abrupt discontinuation of TPN that contains 10% to 20% dextrose may result in profound rebound hypoglycemia. Tachycardia in this patient may signify hypoglycemia. Prompt diagnosis and treatment of severe hypoglycemia are essential if neurologic damage is to be avoided. Whenever a central line is placed for TPN, it should be properly checked before the hypertonic infusion is started (Miller: Miller’s Anesthesia, ed 8, p 1782).
405. In an emergency when there is a limited supply of type O-negative RBCs, type O-positive RBCs are reasonable for transfusion for each of the following patients EXCEPT A. A 60-year-old woman with diabetes who was involved in an MVA B. A 23-year-old man who sustained a gunshot wound to the upper abdomen C. An 84-year-old man with a ruptured abdominal aortic aneurysm D. A 21-year-old, gravida 2, para 1 woman with placenta previa who is bleeding profusely
405. (D) In an emergency when massive amounts of blood are immediately required and the supply of O-negative RBCs in the blood bank is low, it is acceptable to transfuse O-positive RBCs into male patients or into female patients past the age of childbirth before the patient’s blood type is known. This is because delaying blood transfusion for blood typing may be more hazardous to the patient than the risk of a significant transfusion reaction based on Rh type for these patients. However, for the female patient who has the potential for pregnancy, administration of Rh-positive RBCs is not recommended (unless no Rh-negative RBCs are available). This is because an Rh-negative patient who receives Rh-positive RBCs would experience isoimmunization. For these women, future pregnancies with Rh-positive fetuses could be associated with erythroblastosis fetalis. Note: RBCs are preferred over whole blood because Rh-negative whole blood contains a large quantity of anti-A and anti-B antibodies in the plasma (Turgeon: Clinical Hematology, ed 1, pp 50–51).
406. Hetastarch exerts an anticoagulative effect through interference with the function of A. Antithrombin III B. Factor VIII C. Fibrinogen D. Prostacyclin
406. (B) Hetastarch (hydroxyethyl starch) and dextran 70 (glucose polymers with mean molecular weights of 70,000) are colloid solutions that are used for intravascular fluid volume expansion. Both hetastarch and dextran have been associated with allergic reactions, can interfere with coagulation, and can cause hypervolemia. Hetastarch, unlike dextran, does not interfere with crossmatching of blood at the recommended maximal daily dose of 20 mL/kg. Neither compound needs to be administered through a filter. Hetastarch also reduces levels of vWF significantly as well as availability of glycoprotein IIb/IIIa, and it can become directly incorporated into the fibrin clot (Miller: Miller’s Anesthesia, ed 8, p 1783). Blood Products, Transfusion, and Fluid Therapy 115
407. All of the following characterize packed RBCs that have been stored for 35 days at 4° C in citrate phosphate dextrose adenine-1 (CPDA-1) anticoagulant preservative EXCEPT A. Serum potassium greater than 70 mEq/L B. pH less than 7.0 C. Blood glucose less than 100 mg/dL D. P50 of 28
407. (D) RBCs are cooled to about 4° C to decrease cellular metabolism. CPDA-1 is a preservative anticoagulant solution often added to blood. It contains citrate, phosphate, dextrose, and adenine. The citrate is used to bind calcium and acts as an anticoagulant. Phosphate acts as a buffer. Dextrose is added as an energy source for cellular metabolism on the day of donation to raise the blood sugar to greater than 400 mg/dL. At 35 days, the glucose level drops below 100 mg/dL. Adenine is added as a substrate source so that the cells can produce adenosine triphosphate. Other biochemical changes include a fall in pH to about 6.7 and a rise in plasma potassium from around 4 mEq/L on the day of donation to 76 mEq/L at 35 days. Concentrations of 2,3-diphosphoglycerate fall below 1 μM/mL, which causes a leftward shift in the oxyhemoglobin dissociation curve that allows for an increased oxygen affinity for the hemoglobin. This leftward shift produces a P50 value less (not greater) than the normal 26 mm Hg (Miller: Miller’s Anesthesia, ed 8, pp 1841–1842).
408. What is the storage life of whole blood stored with citrate phosphate dextrose (CPD)? A. 14 days B. 21 days C. 35 days D. 42 days 110 Part 2 Clinical Sciences
408. (B) Many preservation solutions are used for whole blood and RBCs. Acid citrate dextrose, CPD, and citrate phosphate double dextrose (CP2D) each allows blood to have a shelf life of 21 days. In 1978, the FDA approved the additive adenine to CPD. This extended the shelf life of blood by 2 weeks. CPDA-1 has a shelf life of 35 days. These solutions were used mainly for whole blood. However, when component therapy became more widespread, it was noted that packing the RBCs by removing the plasma also removed a significant amount of adenine and glucose. By use of an additive solution (which contains primarily adenine, glucose, and saline) to the CPD or CP2D whole blood that has the plasma removed, the packed RBCs can now be stored for 42 days. The three different additive solutions currently used in the United States are Adsol (AS-1), Nutricel (AS-3), and Optisol (AS-5) (Miller: Miller’s Anesthesia, ed 8, p 1841).
409. In the adult, the liver is the primary organ for A. Hemoglobin synthesis B. Hemoglobin degradation C. Factor VIII synthesis D. Antithrombin III synthesis
409. (D) The liver produces most of the coagulation factors except for factor III (tissue thromboplastin), factor IV (calcium), and factor VIII (von Willebrand factor). The liver also produces the coagulation regulatory protein C, protein S, and antithrombin III. Fetal RBCs are produced exclusively by the liver; in the adult, 80% of RBCs are produced by the bone marrow and only 20% are produced in the liver. The degradation of blood is primarily by the reticuloendothelial system (Hemmings: Pharmacology and Physiology for Anesthesia, ed 1, p 477; Miller: Basics of Anesthesia, ed 6, p 456).
410. Anticoagulation with low-molecular-weight heparin (LMWH) can be best monitored through which of the following laboratory tests? A. Activated partial thromboplastin time (aPTT) B. Anti-Xa assay C. Thrombin time D. Reptilase test
410. (B) LMWH is produced by the fractionation or cleaving of “unfractionated heparin (UFH)” into shorter fragments. The anticoagulant properties of UFH and LMWH are complex and somewhat different. UFH binds to and activates antithrombin (more effectively than LMWH) and can be monitored easily with the aPTT. At the usual clinical doses of LMWH, aPTT is not prolonged. LMWH, on the other hand, is more effective in inactivating factor Xa and can be monitored by anti-Xa levels (although commonly this is not performed because of the more predictable action of prophylactic dosing of LMWH). At high doses of LMWH, antifactor Xa values are more commonly measured. Thrombin time is a measure of the ability of thrombin to convert fibrinogen to fibrin. It is prolonged with low amounts of fibrinogen, heparin, and fibrin degradation products (FDPs). A reptilase test is done by adding reptilase to plasma and waiting for a clot to form and is prolonged in the presence of lupus anticoagulant, FDPs, fibrinogen deficiency, or abnormal fibrinogen. It is not prolonged in the presence of heparin (Miller: Miller’s Anesthesia, ed 8, pp 1872–1874; Barash: Clinical Anesthesia, ed 7, p 439).
411. Heparin resistance is likely in patients with which of the following heritable conditions? A. Factor V Leiden mutation B. Prothrombin G20210A gene mutation C. Protein S deficiency D. Antithrombin or antithrombin III (AT3) deficiency
411. (D) The four selections to this question are four of the five major hereditary conditions associated with hypercoagulation. They cause an increased likelihood of clot formation by either increasing prothrombotic proteins (e.g., factor V Leiden mutation, prothrombin G20210A gene mutation) or decreasing endogenous antithrombotic proteins (e.g., antithrombin deficiency, protein C deficiency, protein S deficiency). Clot may also develop if heparin resistance occurs (usual doses produce less than the expected prolongation of the partial thromboplastin time or the activated clotting time) and is not recognized, as during cardiopulmonary bypass. It may occur as a result of excessive binding of heparin to plasma proteins or an insufficient amount of antithrombin. Because heparin binds to and potentiates antithrombin’s activity, conditions with low amounts of antithrombin show resistance. Treatment of AT3 deficiency is replacement of AT3 with either specific AT III concentrate (Thrombate III) or FFP. Replacement of antithrombin to 100% activity is recommended before cardiac surgery in patients with congenital AT3 deficiency (Miller: Miller’s Anesthesia ed 8, pp 1871–1872, 1876–1877; Young: Clinical Hematology, ed 1, pp 1116–1118). 116 Part 2 Clinical Sciences
412. Von Willebrand disease (vWD) could be treated by any of the following EXCEPT A. Cryoprecipitate B. Desmopressin (DDAVP) C. FFP D. Recombinant factor VIII
412. (D) vWD is the most common inherited abnormality affecting platelet function and is caused by a quantitative or qualitative deficiency of a protein called von Willebrand factor (vWF). vWF is produced by endothelial cells and platelets and appears to have two main functions: it acts as an adhesion protein that diverts platelets to sites of vascular injury, and it helps protect factor VIII from inactivation and clearance. Patients with vWD have prolonged bleeding times and a reduced amount of factor VIII. Patients with hemophilia A also have a decrease in factor VIII but normal bleeding times. Type 1 vWD is the most common type (60%-80%) and is associated with a quantitative decrease in circulating plasma vWF caused by a decrease in release of available vWF. Type 2 vWD (20%-30%) has several subtypes and is associated with qualitative deficiency of vWF. Type 3 vWD is the least frequent (1%-5%) and the most severe form, wherein there is almost no vWF and very low factor VIII levels (3%-10% of normal). Treatment of vWD includes DDAVP, which increases the release of available vWF, or blood products that contain vWF and factor III (e.g., cryoprecipitate, FFP, or factor III concentrates). Recombinant factor VIII is not used because it does not contain vWF (Miller: Miller’s Anesthesia, ed 8, pp 1123, 1872; Barash: Clinical Anesthesia, ed 7, p 433).
413. The significance of immunoglobulin A (IgA) antibodies in transfusion medicine is related to A. Allergic reaction B. Febrile reaction C. Delayed hemolytic reaction (immune extravascular reaction) D. Diagnosis of TRALI reaction
413. (A) Although allergic reactions after blood transfusions are common (up to 3%), true nonhemolytic anaphylactic reactions are rare. When anaphylactic reactions develop (often with only a few milliliters of blood or plasma transfused), the signs and symptoms may include dyspnea, bronchospasm, laryngeal edema, chest pain, hypotension, and shock. These reactions are caused by the transfusion of “foreign” IgA protein to patients who have hereditary IgA deficiency and have formed anti-IgA as a result of previous transfusions or from earlier pregnancies. Treatment includes stopping the transfusion and administering epinephrine and steroids. If further transfusion is needed, washed RBCs or RBCs from IgA-deficient donors should be used (Miller: Miller’s Anesthesia, ed 8, p 1853; Barash: Clinical Anesthesia, ed 7, p 426).
414. The most common cause of mortality associated with administration of blood is A. TRALI B. Non-ABO hemolytic transfusion reaction C. Microbial infection D. Anaphylactic reaction
414. (A) For the years 2005 to 2006, 125 confirmed transfusion-related fatalities were listed by the FDA in the United States. The most common cause was TRALI (51%), followed by non-ABO hemolytic transfusion reaction (20%), microbial infection (12%), ABO hemolytic transfusion reaction (7%), death from transfusion-associated circulatory overload (TACO) (7%), and other (2%). Since March 2004, when voluntary bacterial detection testing was implemented for platelet transfusions, there has been a decrease in fatalities associated with transfusion of bacterially contaminated apheresis platelets. Considering about 29 million components are transfused each year (2004 calendar year) in the United States, the reported incidence of death is quite small (www.fda.gov/cber/blood/fatal/0506.htm; Miller: Miller’s Anesthesia, ed 8, pp 1855–1860; Barash: Clinical Anesthesia, ed 7, pp 425–427). TRANSFUSION-RELATED FATALITIES IN THE UNITED STATES, 2004 TO 2006 Cause of Fatality 2004-2006 Average per Year TRALI 86 29 Other reactions (non-ABO hemolytic therapy; anaphylaxis) 67 22 Bacterial contamination 20 7 ABO hemolytic transfusion therapy 15 5 Transfusion not ruled out 31 10 TRALI, transfusion-related acute lung injury. From Miller RD: Miller’s Anesthesia, ed 7, Philadelphia, Saunders, 2011, Table 55-6.
415. Fluid resuscitation during major abdominal surgery with which of the following agents is associated with the BEST survival data? A. 5% Albumin B. 6% Hydroxyethyl starch C. Dextran 70 D. None of the above
415. (D) There is controversy not only as to which intravenous fluid is the best but also how much to give. Most would suggest that isotonic crystalloids should be the initial resuscitative fluids to any trauma patients, and they are certainly less expensive than 5% albumin, 6% hydroxyethyl starch, and dextran 70. Clear advantages of one fluid over another are hard to find (Miller: Miller’s Anesthesia, ed 8, p 1800; Barash: Clinical Anesthesia, ed 7, pp 338–339). Blood Products, Transfusion, and Fluid Therapy 117
416. Which of the following processes reduces the possibility of transmission of cytomegalovirus (CMV) to a susceptible recipient via transfusion of RBCs?
416. (B) Transmission of CMV to patients who have normal immune mechanisms is benign and self-limiting, but in patients who are immunocompromised (e.g., premature newborns, solid organ and bone marrow transplant patients, acquired immunodeficiency syndrome patients), CMV infection can be serious and life threatening. Leukocyte reduction can reduce CMV transmission, but restriction of blood products from seronegative donors is preferred (Miller: Miller’s Anesthesia, ed 8, pp 1857–1858; Barash: Clinical Anesthesia, ed 7, p 424).
417. What is the process aimed at reducing graft-versushost disease (GVHD) in transfusion recipients? A. Washing erythrocytes B. Reduction of leukocytes C. Irradiation D. Storage in Adsol 111 Blood Products, Transfusion, and Fluid Therapy Answers, References, and Explanations
417. (C) GVHD is an often fatal condition that occurs in patients who are immunocompromised. It occurs when donor lymphocytes (graft) establish an immune response against the recipient (host). Blood products that have a significant amount of lymphocytes include RBCs and platelets. FFP and cryoprecipitate appear to be safe. Although directed donor units from first-degree relatives and leukoreduction may reduce the incidence of GVHD, only irradiated products (which inactivates donor lymphocytes) can prevent GVHD (Miller: Miller’s Anesthesia, ed 8, p 1858; Barash: Clinical Anesthesia, ed 7, p 428). 118
418. A 78-year-old patient with a history of hypertension and adult-onset diabetes for which she takes chlorpropamide (Diabinese) is admitted for elective cholecystectomy. On the day of admission, blood glucose is noted to be 270 mg/dL, and the patient is treated with 15 units of regular insulin subcutaneously (SQ) in addition to her regular dose of chlorpropamide. Twenty-four hours later after overnight fasting, the patient is brought to the operating room (OR) without her daily dose of chlorpropamide and is anesthetized. A serum glucose is measured and found to be 35 mg/dL. The MOST likely explanation for this is A. Insulin B. Chlorpropamide C. Hypovolemia D. Effect of general anesthesia
418. (B) Patients with insulin-dependent diabetes and non–insulin-dependent diabetes require special consideration when presenting for surgery. Geriatric age patients come to the OR in the fasting state and without having taken their morning dose of their oral diabetic agent. Chlorpropamide is the longest-acting sulfonylurea and has a duration of action up to 72 hours. Accordingly, it is prudent to measure serum glucose before inducing anesthesia and periodically during the course of the anesthetic and surgery. Regular insulin has a peak effect 2 to 3 hours after SQ administration and a duration of action approximately 6 to 8 hours and would therefore not cause a serum glucose of 35 mg/dL 24 hours after it was administered (Stoelting: Pharmacology and Physiology in Anesthetic Practice, ed 4, pp 479, 483–484).
419. Select the TRUE statement. A. Dibucaine is an ester-type local anesthetic B. A dibucaine number of 20 is normal C. The dibucaine number represents the quantity of normal pseudocholinesterase D. None of the above
419. (D) Dibucaine is an amide-type local anesthetic that inhibits normal pseudocholinesterase by approximately 80%. In patients who are heterozygous for atypical pseudocholinesterase, enzyme activity is inhibited by 40% to 60%. In patients who are homozygous for atypical pseudocholinesterase, enzyme activity is inhibited by only 20%. The dibucaine number is a qualitative assessment of pseudocholinesterase. Quantitative as well as qualitative determination of enzyme activity should be carried out in any patient who is suspected of having a pseudocholinesterase abnormality (Miller: Basics of Anesthesia, ed 6, p 149).
420. A 56-year-old patient with a history of liver disease and osteomyelitis is anesthetized for tibial débridement. After induction and intubation, the wound is inspected and débrided with a total blood loss of 300 mL. The patient is transported intubated to the recovery room, at which time the systolic blood pressure falls to 50 mm Hg. Heart rate is 120 beats/min, arterial blood gases (ABGs) are Pao2 103, Paco2 45, pH 7.3, with 97% O2 saturation with 100% Fio2. Mixed venous blood gases are Pvo2 60 mm Hg, Pvco2 50, and pH 7.25. Which of the following diagnoses is MOST consistent with this clinical picture? A. Hypovolemia B. Congestive heart failure (CHF) C. Cardiac tamponade D. Sepsis with acute respiratory distress syndrome
420. (D) All hypotension can be broadly broken down into two main categories: decreased cardiac output and decreased systemic vascular resistance. Flow or cardiac output can be further subdivided into problems related to decreased heart rate (i.e., bradycardia versus problems related to decreases in stroke volume). Normal Po2 in mixed venous blood is 40 mm Hg. Increased mixed venous arterial oxygen levels can be due to many conditions including high cardiac output, sepsis, left-to-right cardiac shunts, impaired peripheral uptake (e.g., cyanide), and decreased oxygen consumption (e.g., hypothermia), as well as sampling error. The other choices in this question all represent conditions whereby cardiac output is diminished and consequently would not be consistent with the data given in the question (Butterworth: Morgan & Mikhail’s Clinical Anesthesiology, ed 5, pp 360–361).
421. Normal tracheal capillary pressure is A. 10 to 15 mm Hg B. 15 to 20 mm Hg C. 25 to 30 mm Hg D. 35 to 40 mm Hg
421. (C) Tracheal capillary arteriolar pressure (25-35 mm Hg) is important to keep in mind in patients who are intubated with cuffed endotracheal tubes. If the endotracheal tube cuff exerts a pressure greater than capillary arteriolar pressure, tissue ischemia may result. Persistent ischemia may lead to destruction of tracheal rings and tracheomalacia. Endotracheal tubes with low-pressure cuffs are recommended in patients who are to be intubated for periods longer than 48 hours because this will minimize the chances for development of tissue ischemia (Miller: Miller’s Anesthesia, ed 8, pp 1665–1667).
422. How many hours should elapse before performing a single-shot spinal anesthetic in a patient who is receiving 1 mg/kg enoxaparin (Lovenox) twice a day for the treatment of a deep vein thrombosis? A. 6 hours B. 12 hours C. 24 hours D. 48 hours
422. (C) Enoxaparin, dalteparin, and ardeparin are low-molecular-weight heparins (LMWHs). Because of the possibility of spinal and epidural hematoma in the anticoagulated patient with neuraxial blockade, caution is advised. The plasma half-life of LMWH is two to four times longer than standard heparin. These drugs are commonly used for prophylaxis for deep vein thrombosis. These drugs are also used at high doses for treatment of deep vein thrombosis and (off label) as “bridge therapy” for patients chronically anticoagulated with warfarin (Coumadin). In these patients who are being prepared for surgery, Coumadin is discontinued and LMWH started. With high-dose enoxaparin administration (1 mg/kg twice daily), it is recommended to wait at least 24 hours before administration of a single-shot spinal anesthetic (Miller: Miller’s Anesthesia, ed 8, p 1691; Barash: Clinical Anesthesia, ed 7, p 929; Third Consensus Conference on Neuraxial Anesthesia and Anticoagulation, Jan-Feb 2010; http://www.asra.com/publications-anticoagulation-3rd-edition-2010.php).
423. Which of the following peripheral nerves is MOST likely to become injured in patients who are under general anesthesia? A. Ulnar nerve B. Median nerve C. Radial nerve D. Common peroneal nerve
423. (A) The principal mechanism of peripheral nerve injury is ischemia caused by stretching or compression of the nerves. Anesthetized patients are at increased risk for peripheral nerve injuries because they are unconscious and unable to complain about uncomfortable positions that an awake patient would not tolerate and because of reduced muscle tone that facilitates placement of patients into awkward positions. The ulnar nerve in particular is vulnerable because it passes around the posterior aspect of the medial epicondyle of the humerus. The ulnar nerve may become compressed between the medial epicondyle and the sharp edge of the operating table, leading to ischemia and possible nerve injury, which may be transient or permanent (Miller: Basics of Anesthesia, ed 6, pp 310–312). 132 Part 2 Clinical Sciences
424. Which of the following is the most plausible explanation for the lack of analgesia with codeine administration? A. Lack of CYP2D6 enzyme B. VKORC1 polymorphism C. CYP3A4 polymorphism D. Lack of μ receptors
424. (A) The orally administered prodrug codeine (methylmorphine) must be metabolized to morphine in order to work. About 7% to 10% of white patients have an inactive variant of the enzyme CYP2D6, which is the enzyme needed to metabolize codeine. In these patients, as well as in patients who have the normal enzyme but the enzyme is inhibited (e.g., coadministration of quinidine), codeine does not produce analgesia but morphine will produce the expected analgesia. The CYP2D6 enzyme is also needed to metabolize oxycodone into oxymorphone and hydrocodone into hydromorphone. In addition, some patients have a polymorphism form of CYP2D6 that results in very rapid metabolism of codeine and can result in morphine toxicity (Miller: Miller’s Anesthesia, ed 8, pp 574–575).
425. A 62-year-old patient with a bare-metal stent in the mid portion of the left anterior descending artery is scheduled for rotator cuff repair under general anesthesia. The stent was placed 6 weeks before surgery and the patient is on dual therapy (aspirin and clopidogrel). Which of the paradigms below would be best for managing his anticoagulation before surgery? A. Continue both up to the day of surgery B. Stop both 7 to 10 days before surgery C. Stop aspirin and continue clopidogrel D. Stop clopidogrel and continue aspirin
425. (D) Patients who have undergone percutaneous coronary intervention (PCI) with and without stents require dual antiplatelet therapy (usually aspirin and clopidogrel) to prevent restenosis or acute thrombosis at the site of the stent, often for the patient’s lifetime. Cessation of these drugs should be reviewed with the patient’s cardiologist. In general, if the elective surgical procedure may involve bleeding, the elective procedure is delayed for at least 2 weeks after balloon angioplasty without a stent, 6 weeks after a baremetal stent, and 12 months after a drug-eluting stent has been placed. Then the clopidogrel is stopped and restarted as soon as possible after the surgery (aspirin is usually continued). In an emergency situation and when the patient is taking clopidogrel, platelet transfusion may be needed (effectiveness of platelets depends on the last dose of clopidogrel—platelets are effective after 4 hours but much better 24 hours after the last dose of clopidogrel) (Hines: Stoelting’s Anesthesia and Co-Existing Disease, ed 6, pp 13-–14; Miller: Basics of Anesthesia, ed 6, pp 168–170).
426. A patient with which of the following eye diseases would be at greatest risk for retinal damage from hypotension during surgery? A. Strabismus B. Open eye injury C. Glaucoma D. Severe myopia
426. (C) Blood flow to the retina can be decreased by either a decrease in mean arterial pressure or an increase in intraocular pressure. Decreased blood flow and stasis are more likely in patients with glaucoma because of their elevated intraocular pressure. During periods of prolonged hypotension, the incidence of retinal artery thrombosis increases in these patients (Hines: Stoelting’s Anesthesia and Co-Existing Disease, ed 6, pp 253-–254; Miller: Basics of Anesthesia, ed 6, p 487).
427. Naltrexone is A. A narcotic with local anesthetic properties B. An opioid agonist-antagonist similar to nalbuphine C. A pure opioid antagonist with a shorter duration of action than naloxone D. An opioid antagonist used for treatment of previously detoxified heroin addicts
427. (D) Naloxone (Narcan) is a competitive inhibitor at all opioid receptors but has the greatest affinity for μ receptors. Its duration of action is relatively short (elimination half-life of about 1 hour). For this reason, one must be vigilant for the possibility of renarcotization when reversing long-acting narcotics. Naltrexone (ReVia) is the N-cyclopropylmethyl derivative of oxymorphone with a long elimination half-life of 8 to 12 hours. It is currently available only as an oral preparation and is used to block the euphoric effects of injected heroin in addicts who have been previously detoxified. Nalmefene (Revex) is another opioid antagonist that can be administered orally or parenterally and has an extremely long duration of action (elimination terminal half-life of 8.5 hours) (Miller: Miller’s Anesthesia, ed 8, pp 906–907; Butterworth: Morgan & Mikhail’s Clinical Anesthesiology, ed 5, p 290).
428. Which of the following mechanisms is most frequently responsible for hypoxia in the recovery room? A. Ventilation/perfusion mismatch B. Hypoventilation C. Hypoxic gas mixture D. Intracardiac shunt
428. (A) In the recovery room, the most common cause of postoperative hypoxemia is an uneven ventilation/ perfusion distribution caused by loss of lung volume resulting from small airway collapse and atelectasis. Risk factors for ventilation/perfusion mismatch in the postoperative period include old age, obstructive lung disease, obesity, increased intra-abdominal pressure, and immobility. Supplemental oxygen should be administered to keep the Pao2 in the 80 to 100 mm Hg range, which is associated with a 95% saturation of hemoglobin. Other measures can be taken to restore lung volume, which include recovering obese patients in the sitting position, coughing, and deep breathing (Barash: Clinical Anesthesia, ed 7, pp 1566–1567).
429. Hypoparathyroidism secondary to the inadvertent surgical resection of the parathyroid glands during total thyroidectomy typically results in symptoms of hypocalcemia how many hours postoperatively? A. 1 to 2 hours B. 3 to 12 hours C. 12 to 24 hours D. 24 to 72 hours
429. (D) Airway obstruction after total thyroidectomy may be caused by a postoperative hematoma, compression of the trachea, tracheomalacia, bilateral recurrent laryngeal nerve damage, or hypocalcemia resulting from inadvertent removal of the parathyroid glands. Although the airway symptoms of hypocalcemia can develop as early as 1 to 3 hours after surgery, they typically do not develop until 24 to 72 hours postoperatively. Because the laryngeal muscles are particularly sensitive to hypocalcemia, early symptoms may include inspiratory stridor, labored breathing, and eventual laryngospasm. Therapy consists of IV administration of calcium gluconate or calcium chloride (Miller: Basics of Anesthesia, ed 6, p 634; Barash: Clinical Anesthesiology, ed 7, p 1330).
430. Damage to which nerve may lead to wrist drop? A. Radial B. Axillary C. Median D. Ulnar
430. (A) Damage to the radial nerve is manifested by weakness in abduction of the thumb, inability to extend the metacarpophalangeal joints, wrist drop, and numbness in the webbed space between the thumb and General Anesthesia 133 index fingers. The radial nerve passes around the humerus between the middle and lower portions in the spiral groove posteriorly. As it wraps around the bone, the radial nerve can become compressed between it and the OR table, resulting in nerve injury (Barash: Clinical Anesthesia, ed 7, pp 808, 949).
431. The most common cause of bronchiectasis is A. Cigarette smoking B. Air pollution C. α1-Antitrypsin deficiency D. Recurrent bronchial infections
431. (D) Bronchiectasis is one of several obstructive lung diseases characterized by a diminished FEV1 when pulmonary function is evaluated. It is characterized by permanently dilated bronchi that frequently contain purulent secretions. The affected bronchi are often highly vascularized, giving rise to the possibility of hemoptysis. Collateral circulation through the intercostal and bronchial arteries is also possible in these patients. If these vessels connect with the pulmonary circulation, pulmonary hypertension and eventual cor pulmonale are possible sequelae. Any patient with chronic bronchial infections may develop bronchiectasis (Hines: Stoelting’s Anesthesia and Co-Existing Disease, ed 6, pp 195–196).
432. A 6-year-old child is transported to the recovery room after a tonsillectomy. The patient was anesthetized with isoflurane, fentanyl, and N2O. Twenty minutes before emergence and tracheal extubation, droperidol was administered. The anesthesiologist is called to the recovery room because the patient is “making strange eye movements.” The patient’s eyes are rolled back into his head, and his neck is twisted and rigid. The most appropriate drug for treatment of these symptoms is A. Dantrolene B. Diazepam C. Glycopyrrolate D. Diphenhydramine
432. (D) Drugs that block dopamine receptors may cause acute dystonic reactions in some patients. The incidence with droperidol is about 1%. Treatment is the administration of a drug that crosses the blood-brain barrier with anticholinergic properties such as diphenhydramine or benzatropine. Although glycopyrrolate is an anticholinergic drug, it would not be useful in this setting because it does not cross the blood-brain barrier (Miller: Miller’s Anesthesia, ed 8, p 2963; Stoelting: Pharmacology and Physiology in Anesthetic Practice, ed 4, p 414).
433. A 32-year-old army officer is unable to oppose the left thumb and left little finger after an 8-hour exploratory laparotomy under general anesthesia. He had an IV induction through a peripheral IV and had a second IV placed in the antecubital fossa after he was asleep. Damage to which of the following nerves would MOST likely account for this deficit? A. Radial B. Ulnar C. Median D. Musculocutaneous
433. (C) The median nerve is most frequently injured at the antecubital fossa by extravasation of IV drugs that are toxic to neural tissue, or by direct injury caused by the needle during attempts to cannulate the medial cubital or basilic veins. The median nerve provides sensory innervation to the palmar surface of the lateral three and one-half fingers and adjacent palm, and motor function to the abductor pollicis brevis, flexor pollicis brevis, and opponens pollicis muscles (Miller: Basics of Anesthesia, ed 6, p 313).
434. Pheochromocytoma would be MOST likely to coexist with which of the following? A. Insulinoma B. Pituitary adenoma C. Primary hyperaldosteronism (Conn syndrome) D. Medullary carcinoma of the thyroid
434. (D) Pheochromocytoma is an endocrine tumor (with release of catecholamines) in which 90% of patients are hypertensive, 90% of the tumors originate in one adrenal medulla, and 90% of all pheochromocytomas are benign. This disease is rare (<0.1% of hypertension in adults), but when it occurs, it is often seen with a triad of diaphoresis, tachycardia, and headache in patients with hypertension. Other symptoms include palpitations, tremulousness, weight loss, hyperglycemia, hypovolemia, and in some cases dilated cardiomyopathy and CHF. Death as a result of pheochromocytoma is due to cardiac conditions (e.g., myocardial infarction, CHF) or an intracranial bleed. In about 5% of cases, pheochromocytomas show an autosomal dominant pattern and may coexist with other endocrine diseases such as medullary carcinoma of the thyroid and hyperparathyroidism. This combination is called multiple endocrine neoplasia (MEN) type II or IIA (Sipple syndrome). MEN type IIB consists of pheochromocytoma, medullary carcinoma of the thyroid, and neuromas of the oral mucosa. The von Hippel-Lindau disease consists of hemangiomas of the nervous system (i.e., retina or cerebellum), and 10% to 25% of these patients also have a pheochromocytoma. The average-sized pheochromocytoma contains 100 to 800 mg of norepinephrine (Barash: Clinical Anesthesia, ed 7, pp 1339–1340; Hines: Stoelting’s Anesthesia and Co-Existing Disease, ed 6, pp 392–395).
435. Which of the following oral antidiabetic drugs is unique in that it does NOT produce hypoglycemia when administered to a fasting patient? A. Glyburide (Micronase) B. Glipizide (Glucotrol) C. Tolbutamide (Orinase) D. Metformin (Glucophage)
435. (D) Oral agents that are used to help control hyperglycemia in type 2 diabetic patients (relative β-cell insufficiency and insulin resistance) include four major drug classes: 1. Drugs that stimulate insulin secretion (hypoglycemia is a risk) a. sulfonylureas i. first-generation (chlorpropamide, tolazamide, tolbutamide) ii. second-generation (glimepiride, glipizide, glyburide) b. meglitinides (repaglinide, nateglinide) 2. Drugs that decrease hepatic gluconeogenesis (hypoglycemia not a risk) a. biguanides (metformin) 3. Drugs that improve insulin sensitivity (hypoglycemia not a risk) a. thiazolidinediones (rosiglitazone, pioglitazone) b. glitazones 4. Drugs that delay carbohydrate absorption (hypoglycemia not a risk) a. α-glucosidase inhibitors (acarbose, miglitol) 134 Part 2 Clinical Sciences Only drugs that stimulate insulin secretion are a risk for producing hypoglycemia. Initial therapy is usually with second-generation sulfonylureas (more potent and fewer side effects than first-generation sulfonylureas) or with a biguanide (Brunton: Goodman & Gilman’s The Pharmacological Basis of Therapeutics, ed 12, pp 1255–1270; Hines: Stoelting’s Anesthesia and Co-Existing Disease, ed 6, pp 376–380; Stoelting: Pharmacology and Physiology in Anesthetic Practice, ed 4, pp 481–485).
436. The onset of delirium tremens (DTs) after abstinence from alcohol usually occurs in A. 8 to 24 hours B. 24 to 48 hours C. 2 to 4 days D. 4 to 7 days
436. (C) Although early mild symptoms of alcohol withdrawal can be seen within 6 to 8 hours after a substantial drop in the serum alcohol levels, DTs, which is seen in about 5% of patients, is a life-threatening medical emergency that develops 2 to 4 days after the cessation of alcohol in alcoholics. Symptoms of DTs include hallucinations, combativeness, hyperthermia, tachycardia, hypertension or hypotension, and grand mal seizures. Treatment of severe alcohol withdrawal consists of fluid replacement, electrolyte replacement, and IV vitamin administration with particular attention paid to thiamine. Aggressive administration of benzodiazepines is indicated to prevent seizures (5-10 mg of diazepam every 5 minutes until the patient becomes sedated but not unconscious). β-Blockers are used to suppress overactivity of the sympathetic nervous system, and lidocaine may be effective in the treatment of cardiac dysrhythmias (Hines: Stoelting’s Anesthesia and Co-Existing Disease, ed 6, p 544).
437. A 78-year-old retired coal miner with an intraluminal tracheal tumor is scheduled for tracheal resection. Which of the following is a relative contraindication for tracheal resection? A. Need for postoperative mechanical ventilation for underlying lung disease B. Tumor located at the carina C. Documented liver metastases D. Ischemic heart disease with a history of CHF
437. (A) Operations on the trachea may be indicated in patients who have tracheal tumors or patients who had a previous trauma to the trachea resulting in tracheal stenosis or tracheomalacia. Eighty percent of the operations on the trachea involve segmental resection with primary anastomosis, 10% involve resection with prosthetic reconstruction, and another 10% involve insertion of a T-tube stent. These operations frequently are very complicated and require constant communication between the surgeon and the anesthesiologist. Preoperative pulmonary function tests are indicated in all patients who are to undergo elective tracheal resection. Severe lung disease necessitating postoperative mechanical ventilation is a relative contraindication for tracheal resection because positive airway pressure may cause wound dehiscence (Miller: Miller’s Anesthesia, ed 8, pp 1987–1988).
438. A 78-year-old patient with multiple myeloma is admitted to the intensive care unit (ICU) for treatment of hypercalcemia. The primary risk associated with anesthetizing patients with hypercalcemia (levels of 14-16 mg/dL) is A. Coagulopathy B. Cardiac dysrhythmias C. Hypotension D. Laryngospasm 120 Part 2 Clinical Sciences
438. (B) Hypercalcemia is associated with a number of signs and symptoms, including hypertension, dysrhythmias, shortening of QT interval, kidney stones, seizure, nausea and vomiting, weakness, depression, personality changes, psychosis, and even coma. Generally, patients with total serum calcium levels of 12 mg/dL or less do not require any intervention, with the possible exception of rehydration with saline. Higher calcium levels may be associated with clinical symptoms and should be treated before anesthetizing the patient. Caution should be taken with digitalis administration to any patient who is hypercalcemic because some patients may exhibit extreme digitalis sensitivity (Miller: Miller’s Anesthesia, ed 8, p 1794; Barash: Clinical Anesthesia, ed 7, pp 354–355). NORMAL CALCIUM LEVELS Serum Calcium Serum Ionized Calcium Conventional units (mEq/L) 4.5-5.5 mEq/L 2.1-2.6 mEq/L Conventional units (mg/dL) 9.0-11.0 mg/dL 4.25-5.25 mg/dL SI units (mmol/L) 2.25-2.75 mmol/L 1.05-1.30 mmol/L
439. Just before induction of general anesthesia for an 85-year-old demented man with an ischemic bowel, he mentions to you that he forgot to take his greencapped eye drops. He states that not taking it daily will result in blindness. The green-capped eye drops are A. NaCl drops used to prevent his eye from drying out B. Antibiotic drops C. Steroids D. Used to produce miosis
439. (D) Red-top eye drops cause mydriasis and should be used with caution in patients with closed-angle glaucoma. Green-top eye drops cause miosis, and the pupillary constriction helps keep the drainage route open in patients with glaucoma and helps prevent an acute attack of glaucoma. Clear or white-top eye drops do not change pupillary size.
440. A normal, healthy 3-year-old child was involved in a motor vehicle accident. He is coming emergently to the OR. Drug doses need to be calculated, but his weight is not known. What value should be used to estimate the 3-year-old child’s weight? A. 8 kg B. 10 kg C. 12 kg D. 14 kg
440. (D) When reviewing growth curves, the normal 40-week term newborn weighs about 3.5 kg. Infants then double their birth weight by 5 months and triple their weight by 1 year. Therefore, the average 1-year-old weighs 10 kg (22 lb). From the age of 1 to 6 years, children gain about 2 kg per year. Thus, an average 2-year-old weighs 12 kg, 3-year-old weighs 14 kg, 4-year-old weighs 16 kg, 5-year-old weighs 18 kg, and 6-year-old weighs 20 kg. From age 6 to 10 years, children gain about 3 kg per year (Davis: Smith’s Anesthesia for Infants and Children, ed 8, pp A6-A13). General Anesthesia 135
441. A 62-year-old man undergoes an emergency craniotomy for subdural hematoma. Two years earlier, a VVI pacemaker was placed for third-degree heart block. The patient received vancomycin 1 g IV before arriving in the OR. General anesthesia is induced with propofol 160 mg IV and the lungs are hyperventilated to a Paco2 of 25 mm Hg by mask. Just before tracheal intubation, the patient’s heart rate decreases from 70 to 40 beats/min and the pacemaker spikes that were previously present in lead II of the electrocardiogram disappear. The MOST likely cause of bradycardia in this patient is A. Hypocarbia B. Vancomycin allergy C. A side effect of propofol D. Pacemaker battery failure
441. (A) Causes for acute pacemaker malfunction in the OR are numerous and include threshold changes, inhibition, generator failure, and lead or electrode dislodgement or breakage. A VVI pacemaker may be inhibited by myopotentials. In this regard, administration of succinylcholine could actually inhibit a VVI pacemaker. Similarly, electrocautery can inhibit a VVI pacemaker through electromagnetic interference. Should this occur (in most cases, depending on manufacturer), a magnet should be placed over the pacemaker to convert it into a VOO pacemaker, eliminating the possibility of further inhibition. Pacemakers should be evaluated preoperatively to eliminate the possibility of generator failure. Lead breakage or dislodgement is an unlikely cause of pacemaker failure unless the surgeon is working in the vicinity of the electrodes. Acute threshold changes are almost always associated with changes in the serum potassium concentration. In this particular patient, hyperventilation causes a respiratory alkalosis that results in the intracellular shifting of serum potassium. The net result is that the electrical threshold for the pacemaker is raised, preventing ventricular capture (Miller: Miller’s Anesthesia, ed 8, p 1476; Thomas: Manual of Cardiac Anesthesia, ed 2, pp 382–383).
442. A 28-year-old obese patient has diminished breath sounds bilaterally at the lung bases 18 hours after an emergency appendectomy under general anesthesia. Which of the following maneuvers would be LEAST effective in preventing postoperative pulmonary complications in this patient? A. Coughing B. Voluntary deep breathing C. Performing a forced vital capacity (FVC) D. Use of incentive spirometry
442. (C) Therapies aimed at increasing functional residual capacity (FRC) of the lungs are useful in reducing the incidence of postoperative pulmonary complications. Forced expiratory maneuvers may lead to airway closure, which would be of no benefit for this patient (Miller: Miller’s Anesthesia, ed 8, pp 447, 2932–2934).
443. Below what value of cerebral blood flow (CBF) will signs of cerebral ischemia first begin to appear on the electroencephalogram (EEG)? A. 6 mL/100 g/min B. 15 mL/100 g/min C. 22 mL/100 g/min D. 31 mL/100 g/min
443. (C) The human brain is able to maintain neuronal function in the face of decreasing CBF below the normal level of 50 mL/100 g/min. Because O2 delivery is directly related to CBF, EEG evidence of cerebral ischemia will appear if CBF is diminished sufficiently. The CBF reserve, however, is substantial, and the first signs of cerebral ischemia do not appear on EEG until CBF has fallen to approximately 22 mL/100 g/min. When CBF has fallen to 15 mL/100 g/min, the EEG becomes isoelectric. Irreversible membrane damage and cellular death do not occur, however, until CBF falls to 6 mL/100 g/min. Areas of the brain in which CBF falls in the 6 to 15 mL/100 g/min range are referred to as zones of ischemic penumbra. Several hours may elapse in these areas of the brain before irreversible membrane damage occurs (Miller: Miller’s Anesthesia, ed 8, p 410).
444. A 67-year-old patient is mechanically ventilated in the ICU 2 days after repair of a ruptured abdominal aortic aneurysm. To maintain Pao2 in the 60 to 65 range, 10 cm H2O positive end-expiratory pressure (PEEP) is added to the ventilator cycle. The patient’s blood pressure has averaged 110/65 before addition of PEEP. After addition of PEEP, the blood pressure is noted to slowly fall to an average of approximately 95/50. The best explanation for this decrease in blood pressure is A. Tension pneumothorax B. Decreased venous return to the heart C. Increased afterload on the right side of the heart D. Increased afterload on the left side of the heart
444. (B) Positive end-expiratory pressure (PEEP) is the maintenance of positive airway pressure during the entire ventilator cycle. The addition of PEEP to the ventilator cycle is often recommended when Pao2 is not maintained above 60 mm Hg, when breathing an Fio2 of 0.50 or greater. Although not completely understood, PEEP is thought to increase arterial oxygenation, pulmonary compliance, and FRC by expanding previously collapsed but perfused alveoli, thereby decreasing shunt and improving ventilation/ perfusion matching. An important adverse effect of PEEP is a decrease in arterial blood pressure caused by a decrease in venous return, left ventricular filling and stroke volume, and cardiac output. These effects are exaggerated in patients with decreased intravascular fluid volume. Other potential adverse effects of PEEP include pneumothorax, pneumomediastinum, and subcutaneous emphysema (Miller: Miller’s Anesthesia, ed 8, pp 3077–3078; Miller: Basics of Anesthesia, ed 6, p 667).
445. The mechanism of action of clopidogrel is A. Adenosine diphosphate (ADP) receptor blockade (P2Y12) B. Platelet glycoprotein IIB/IIIa antagonism C. Cyclooxygenase COX-1 and COX-2 inhibition D. Direct thrombin inhibition
445. (A) Platelets contain two purinergic receptors (P2Y1 and P2Y12). Clopidogrel (Plavix) is a prodrug and an irreversible inhibitor of platelet P2Y12 receptors, which blocks the ADP receptors and inhibits platelet activation, aggregation, and degranulation. There is wide interindividual variability for clopidogrel to inhibit ADP-induced platelet aggregation, and some patients are resistant to its effects. Glycoprotein IIb/IIIa inhibitors block fibrinogen binding to platelet glycoprotein IIb/IIIa receptors, which is the final common pathway of platelet aggregation and includes the intravenous drugs abciximab (ReoPro), eptifibatide (Integrilin), and tirofiban (Aggrastat). Aspirin, naproxen, and ibuprofen inhibit platelet COX-1 and inhibit the release of ADP by platelets and platelet aggregation. Selective COX-2 inhibitors such as celecoxib, parecoxib, and valdecoxib have no effect on platelet function because only COX-1 inhibitors affect platelet function. Direct thrombin inhibitors suppress platelet function and include the parenteral drugs hirudin, argatroban, lepirudin (Refludan), desirudin (Iprivask), bivalirudin (Angiomax), and drotrecogin α (Xigris), as well as the oral drug dabigatran (Pradaxa, Pradax) and ximelagatran (Brunton: Goodman & Gilman’s The Pharmacological Basis of Therapeutics, ed 12, pp 859-871; Miller: Basics of Anesthesia, ed 6, pp 358-359; Stoelting: Pharmacology and Physiology in Anesthetic Practice, ed 4, pp 277–281, 516–518). 136 Part 2 Clinical Sciences
446. Which of the following is most closely associated with minimum alveolar concentration (MAC)? A. Blood/gas partition coefficient B. Oil/gas partition coefficient C. Vapor pressure D. Brain/blood partition coefficient
446. (B) As a rough approximation, if one divides 150 by the MAC for any given volatile anesthetic, the quotient will be approximately equal to the oil/gas partition coefficient. For example, if one were to divide the MAC of halothane (0.75) into 150, the quotient would be 200, which is very close to the actual oil/gas partition coefficient for halothane (224). Similarly, if one were to divide the MAC of enflurane (1.68) into 150, the quotient would be 89, which is very similar to the oil/gas partition coefficient for enflurane (98). The fact that anesthetics with a high oil/gas partition coefficient (i.e., lipid-soluble agents) have lower MACs supports the Meyer-Overton theory (critical volume hypothesis) (Stoelting: Pharmacology and Physiology in Anesthetic Practice, ed 4, p 29).
447. A 15-year-old, 65-kg patient with Cushing disease is to undergo a transsphenoidal hypophysectomy to remove a pituitary adenoma. General anesthesia is induced with propofol IV, and tracheal intubation is facilitated with vecuronium 0.20 mg/kg IV. Anesthesia is maintained with isoflurane, N2O, and O2. Mannitol 1 g/kg is administered IV to reduce intracranial pressure. At the end of the operation, the patient is extubated and taken to the ICU. Over the next 6 hours the patient has a total urine output of 8.3 L. Serum sodium concentration is 154 mEq/L, serum potassium concentration is 4.8 mEq/L, and serum glucose concentration is 160 mg/dL. Urine specific gravity is 1.002 and urine osmolality is 125 mOsm/L. The most likely cause of the large urine output is A. Osmotic diuresis from mannitol B. Excess mineralocorticoid activity C. Hyperglycemia D. Central diabetes insipidus
447. (D) Diabetes insipidus is characterized by hypernatremia, serum hyperosmolality, polyuria, and urine hypo-osmolality. Diabetes insipidus may occur after any intracranial procedure, but it is particularly common in surgery involving the pituitary gland. It may develop intraoperatively, but it commonly develops 4 to 12 hours postoperatively. Intravenous half-normal saline and dextrose 5% in water are started as replacement fluids. The pharmacologic treatment for diabetes insipidus is desmopressin acetate (synthetic 1-desamino-8-darginine vasopressin [DDAVP]) commonly started when the urine output is greater than 350 to 400 mL/ hr. In a conscious patient, it is not essential to administer DDAVP because the patient may increase his oral intake to compensate for polyuria. In the unconscious patient, however, administration of DDAVP is necessary. Desmopressin (DDAVP) may be administered SQ, IV, or intranasally. Fortunately, diabetes insipidus related to surgery and head trauma usually is transient (Hines: Stoelting’s Anesthesia and Co-Existing Disease, ed 6, pp 404–405; Barash: Clinical Anesthesia, ed 7, p 1013).
448. Scopolamine should not be given as a premedication in patients with which of the following neurologic diseases? A. Parkinson disease B. Alzheimer disease C. Multiple sclerosis D. Narcolepsy General Anesthesia 121
448. (B) The principal feature of Alzheimer disease is progressive dementia. The onset typically occurs after age 60 years and may affect as many as 20% of patients older than age 80 years. In addition to age, other risk factors include history of serious head trauma (e.g., boxing), Down syndrome, and presence of the disease in a parent or sibling. One biochemical feature of this disease is a decrease in the enzyme choline acetyltransferase in the brain. There is a strong correlation between reduced enzyme activity and decreased cognitive function. Interestingly, administration of the anticholinergic drugs scopolamine or atropine (but not glycopyrrolate, which does not cross the blood-brain barrier) causes confusion similar to that seen in the early stages of Alzheimer disease. Conversely, administration of anticholinesterase drugs capable of penetrating the blood-brain barrier, such as donepezil (Aricept), galantamine, rivastigmine (Exelon), and tacrine are used to treat patients with Alzheimer disease. Physostigmine may have beneficial effects in some patients as well. Scopolamine is therefore a poor choice for premedication in patients with Alzheimer disease (Hines: Stoelting’s Anesthesia and Co-Existing Disease, ed 6, p 245; Butterworth: Morgan & Mikhail’s Clinical Anesthesiology, ed 5, pp 619–620).
449. A 63-year-old man is scheduled to undergo a right hemicolectomy under general anesthesia. Anesthesia is induced with propofol 2 mg/kg IV and fentanyl 100 μg IV. Succinylcholine 1.5 mg/kg IV is administered to facilitate tracheal intubation. Anesthesia is maintained with isoflurane and N2O. After all four twitches of the train-of-four stimulus return to baseline values, vecuronium 10 mg IV is administered. Gentamicin 80 mg and cefazolin 1 g are administered IV as a prophylactic treatment. At the end of surgery, two of four thumb twitches can be elicited to trainof- four stimulation of the ulnar nerve, and neuromuscular blockade is antagonized with neostigmine 0.05 mg/kg IV and atropine 0.015 mg/kg IV. The patient, however, begins to move before the incision is completely closed, and succinylcholine 40 mg IV is given. Fifteen minutes later, all anesthetics are discontinued and the patient is ventilated with 100% O2, but the patient remains apneic. The most likely cause of apnea is A. Fentanyl B. Recurarization C. Succinylcholine D. Gentamicin
449. (C) At the end of any general anesthetic, spontaneous ventilation must be restored before the patient can be extubated. The differential diagnosis for persistent apnea includes muscle relaxants (inadequate reversal or pseudocholinesterase deficiency), volatile anesthetics, narcotics, hypocarbia, damage to the phrenic nerves bilaterally, and the possibility of a central nervous system (CNS) event. Succinylcholine is hydrolyzed by pseudocholinesterase to succinylmonocholine and choline. This is further hydrolyzed by plasma cholinesterase to succinic acid and choline. All of the anticholinesterase agents used to reverse nondepolarizing neuromuscular blockade also inhibit pseudocholinesterase. Administration of succinylcholine to any patient who has already received an anticholinesterase will result in a prolonged block from the succinylcholine because it can no longer be easily hydrolyzed. In this patient, therefore, succinylcholine would be by far the most likely cause of apnea at the end of the operation (Stoelting: Pharmacology and Physiology in Anesthetic Practice, ed 4, p 218).
450. A 53-year-old woman with endometrial cancer is undergoing an abdominal hysterectomy under general anesthesia with desflurane. During the first hour of anesthesia, urine output is 100 mL. Blood loss is minimal. When the patient is placed in the Trendelenburg position, the urine output declines to virtually zero. The most likely explanation for this sudden decrease in urine output in this patient is A. Pooling of urine in the dome of the bladder B. Increased central venous pressure C. Increased antidiuretic hormone (ADH) production from surgical stimulation D. Hypovolemia
450. (A) Complete or almost complete cessation of urine flow suggests a postrenal obstruction. However, at times pooling of the urine in the dome of the bladder should be considered as a possible cause of oliguria in this patient in the absence of significant bleeding (Miller: Miller’s Anesthesia, ed 8, pp 556–557).
451. Which of the following diseases is NOT associated with a decrease in Dlco? A. Emphysema B. Obesity C. Pulmonary emboli D. Anemia
451. (B) DL is defined as the diffusing capacity of the lung. When a nontoxic, low concentration of carbon monoxide is used for the measurement, it is called Dlco. The normal value of Dlco is 20 to 30 mL/min/mm Hg and is influenced by the volume of blood (hemoglobin) within the pulmonary circulation. Thus, diseases associated with a decrease in pulmonary blood volume (i.e., anemia, emphysema, hypovolemia, General Anesthesia 137 pulmonary hypertension) will be reflected by a decrease in the Dlco. Dlco is also decreased with oxygen toxicity as well as pulmonary edema. Conditions associated with an increased Dlco include the supine position, exercise, obesity, and left-to-right cardiac shunts (Barash: Clinical Anesthesia, ed 7, pp 369–370, 373–374; Miller: Miller’s Anesthesia, ed 8, p 365).
452. Each of the following postoperative complications of thyroid surgery can result in upper airway obstruction EXCEPT A. Cervical hematoma B. Tetany C. Bilateral superior laryngeal nerve injury D. Bilateral recurrent laryngeal nerve injury
452. (C) Patients undergoing thyroid surgery are at risk for airway obstruction from a number of causes. Postoperative hemorrhage sufficient to cause a large hematoma could compress the trachea and cause airway obstruction because of the close proximity of the thyroid gland to the trachea. Permanent hypoparathyroidism is a rare complication that may cause hypocalcemia leading to progressive stridor followed by laryngospasm. The most common nerve injury after thyroid surgery is damage to the abductor fibers of the recurrent laryngeal nerve. Unilaterally, this is manifested as hoarseness. Bilateral recurrent laryngeal nerve damage, however, may lead to airway obstruction during inspiration. Selective injury of the adductor fibers of the recurrent laryngeal nerve is a possible complication of thyroid surgery. This injury would leave the vocal cords open because the abductor fibers would be unopposed, placing the patient at great risk for aspiration. The superior laryngeal nerve has an extrinsic branch that innervates the cricothyroid muscle (which tenses the vocal cords) and an internal branch that provides sensory innervation to the pharynx above the vocal cords. Bilateral damage to this nerve would result in hoarseness and would predispose the patient to aspiration but would not lead to airway obstruction per se (Miller: Basics of Anesthesia, ed 6, p 469).
453. The MOST sensitive early sign of malignant hyperthermia (MH) during general anesthesia is A. Tachycardia B. Hypertension C. Fever D. Increased end-expiratory CO2 tension (Peco2)
453. (D) MH is a clinical syndrome that may develop rapidly or take hours to manifest, sometimes not occurring until the patient is in the recovery room. Clinical signs include hypertension, tachycardia, respiratory acidosis, metabolic acidosis, muscle rigidity, myoglobinuria, and fever. The diagnosis of MH is unlikely, however, if only one of these signs is manifested. Because MH is a metabolic disorder, one of the first sensitive signs is an increase in the production of CO2 and concomitant respiratory acidosis. This is the most reliable early sign of the syndrome (Barash: Clinical Anesthesia, ed 7, pp 612, 622–624).
454. A 78-year-old woman is anesthetized for a right hemicolectomy for 3 hours. At the end of the operation the patient’s blood pressure is 130/85 mm Hg, heart rate is 84 beats/min, core body temperature is 35.4° C, and Peco2 on infrared spectrometer is 38 mm Hg. Which of the following would be the LEAST plausible reason for prolonged apnea in this patient? A. Residual neuromuscular blockade B. Narcotic overdose C. Unrecognized obstructive pulmonary disease and high baseline Paco2 D. Persistent intraoperative hyperventilation
454. (D) Hyperventilation to Paco2 of 20 mm Hg or higher for more than 2 hours will result in active transport of HCO3 – out of the CNS. This results in spontaneous breathing at a lower (not higher) Paco2. The other choices should be included in the differential diagnosis of apnea (Hines: Stoelting’s Anesthesia and Co-Existing Disease, ed 5, pp 359–361; Miller: Basics of Anesthesia, ed 6, pp 62–64, 340; Barash: Clinical Anesthesia, ed 7, pp 271–273).
455. A 68-year-old woman with severe rheumatoid arthritis undergoes pulmonary function evaluation before an elective abdominal surgery. Forced expiratory volume in 1 second (FEV1) and FVC are within normal limits; however, the maximum voluntary ventilation (MVV) is only 40% of predicted. The next step in the pulmonary function evaluation of this patient should be to A. Obtain ABGs on room air B. Obtain a flow-volume loop C. Obtain a measurement of peak flow D. Obtain a ventilation/perfusion scan
455. (B) Maximum voluntary ventilation (MVV) is a nonspecific pulmonary function test that measures the endurance of the ventilatory muscles and indirectly reflects the compliance of the lung and thorax as well as airway resistance. A decreased MVV may be caused by impairment to inspiration or expiration. In this patient, FEV1 is normal, which strongly suggests that the ventilatory impairment is during inspiration. A flow-volume loop would be a very useful confirmatory test (Barash: Clinical Anesthesia, ed 7, pp 1033–1034).
456. Which of the following is NOT a component of the postanesthetic discharge scoring system (PADSS) used to evaluate the suitability of a patient to be discharged from an ambulatory surgical facility? A. Drinking B. Ambulation C. Absence of nausea and vomiting D. Pain control
456. (A) Guidelines for safe discharge of patients from ambulatory surgical centers include stable vital signs, ability to walk without dizziness, controlled pain, absence of nausea and vomiting, and minimal surgical bleeding. The PADSS is a tool for objectively assessing a patient’s readiness for discharge from the surgical center and includes these five criteria. Requirements to drink fluids and to void before home discharge are controversial and are not parameters included in the PADSS (Barash: Clinical Anesthesia, ed 7, pp 1560–1561).
457. During emergency repair of a mandibular jaw fracture in an otherwise healthy 19-year-old man, the patient’s temperature is noted to rise from 37° C on induction to 38° C after 2 hours of surgery. Which of the following informational items would be LEAST useful in ruling out MH in this patient? A. Normal heart rate and blood pressure B. History of negative caffeine-halothane contracture test carried out 6 months earlier C. History of an uncomplicated general anesthetic at age 16 years with halothane and succinylcholine D. Normal ABGs drawn when the patient’s temperature reached 38° C 122 Part 2 Clinical Sciences
457. (C) Malignant hyperthermia (MH) is a difficult diagnosis to make on clinical grounds alone. Signs of MH may be fulminant or very subtle. They may occur immediately after induction or may not be manifested until the patient has reached the recovery room or even later. MH is a disorder of metabolism and is associated with hypertension, tachycardia, dysrhythmias, respiratory acidosis, metabolic acidosis, muscular rigidity, rhabdomyolysis, and fever. Contrary to what one might believe based on the name of this disease, fever is typically a late finding. Other diseases that may mimic MH include alcohol withdrawal, acute cocaine toxicity, bacteremia, pheochromocytoma, hyperthyroidism, and neuroleptic malignant syndrome. An elevation in temperature alone with normal blood gases, heart rate, and blood pressure, and no evidence of muscle breakdown would very likely not be due to MH. If a patient had been previously subjected to muscle biopsy and caffeine-halothane contracture testing with negative results, MH would be exceedingly 138 Part 2 Clinical Sciences rare, although a false-negative result is possible. A history of a previous anesthetic without MH triggering would be of little reassurance in a patient in whom an MH episode is suspected. It is not uncommon for MH-susceptible individuals to not trigger when a triggering anesthetic is administered initially but develop fulminant MH with a subsequent anesthetic (Barash: Clinical Anesthesia, ed 7, pp 622–624).
458. Which of the following drugs is useful in the treatment of asthma by specifically interfering with the leukotriene pathway? A. Fluticasone (Flovent) B. Ipratropium bromide (Atrovent) C. Triamcinolone (Azmacort) D. Montelukast (Singulair)
458. (D) Asthma is an inflammatory illness that has bronchial hyperreactivity and bronchospasm as a result. Treatment is first directed at the inflammatory component as the underlying problem, reserving bronchodilators for symptomatic use. Because leukotrienes may function as inflammatory mediators, the leukotriene pathway inhibitors such as zileuton and the leukotriene receptor antagonist montelukast (Singulair) are being used for treatment of asthma. Zileuton and montelukast are available only as oral preparations, whereas the other drugs listed are given by inhalation. Fluticasone and triamcinolone are anti-inflammatory corticosteroids. Ipratropium is a quaternary ammonium compound formed by the introduction of an isopropyl group to the N atom of atropine and produces effects similar to those of atropine. One unexpected finding is a relative lack of effect on mucociliary clearance, which makes it useful in patients with airway disease, especially if parasympathetic tone of the airways is increased. Salmeterol is a β2-selective adrenergic drug (Hardman: Goodman & Gilman’s The Pharmacological Basis of Therapeutics, ed 11, pp 721–725, 730–731; Stoelting: Pharmacology and Physiology in Anesthetic Practice, ed 4, pp 271, 427).
459. A 68-year-old, 100-kg patient is undergoing a transurethral resection of the prostate gland under general anesthesia. Upon arrival in the recovery room, the patient appears restless and confused. Serum sodium is checked and found to be 110 mEq/L. How many mEq of sodium are needed to raise the serum [Na+] to 120 mEq/L? A. 300 mEq B. 400 mEq C. 500 mEq D. 600 mEq
459. (D) Acute decreases in serum sodium, due to absorption of bladder irrigating fluids, rarely cause symptoms unless the sodium level drops below 120 mEq/L. At this level, tissue edema may develop and clinical neurologic signs (e.g., restlessness, nausea, confusion, seizures, and coma) or ECG changes (e.g., widening of the QRS complex, elevation of the ST segment, ventricular tachycardia, or ventricular fibrillation) may be manifested. Treatment of mild decreases in serum sodium (i.e., 120-135 mEq/L with no neurologic or ECG changes) is by fluid restriction and/or administration of a diuretic such as furosemide. When the sodium level drops below 120 mEq/L and neurologic symptoms or changes in the ECG develop, sodium chloride administration is needed. To calculate the amount needed, one multiplies the patient’s total body water (i.e., 0.6 × body weight = TBW) by the change in sodium desired. In this case, the TBW is 60 L (0.6 × 100 kg) and the change of sodium is 10 mEq (120 mEq/L − 110 mEq/L), thus 60 L × 10 mEq/L = 600 mEq. Caution is advised in administering sodium because too rapid administration may lead to demyelinating CNS lesions. The recommended rate of 3% sodium chloride (513 mEq/L) is 1 to 2 mL/kg/hr. Serum sodium levels should be checked at least every hour until the sodium level increases above 120 mEq/L (Barash: Clinical Anesthesia, ed 7, pp 341–344).
460. Trismus after administration of succinylcholine IV signals the onset of MH in what percentage of patients? A. Less than 50% B. 50% C. 75% D. 85%
460. (A) Trismus (masseter spasm) is characterized by rigidity of the jaw muscles while the limb muscles remain flaccid after administration of succinylcholine. Trismus may herald the onset of MH in some patients but may be due to a number of other causes and may occur in normal patients. It previously had been believed that 50% of patients who experience trismus after administration of succinylcholine would go on to develop MH. Recent evidence suggests, however, that the incidence is less. If masseter spasm occurs in a patient after administration of succinylcholine, the most conservative course would be to cancel the operation. If cancellation of the operation is not feasible, then a nontriggering anesthetic should be used, and the anesthesiologist should pay close attention for any signs of MH (Miller: Miller’s Anesthesia, ed 8, p 1296).
461. A 45-year-old man is brought to the OR emergently for repair of a ruptured abdominal aortic aneurysm. Anesthesia is induced with ketamine 2 mg/kg IV, and tracheal intubation is facilitated with succinylcholine 1.5 mg/kg IV. Immediately after tracheal intubation, the patient’s blood pressure falls from 110/80 to 50/20 mm Hg. What is the MOST likely cause of the sudden severe hypotension in this patient? A. Hypovolemia B. Direct myocardial depression from ketamine C. Vasovagal response to direct laryngoscopy D. Arteriolar vasodilation from succinylcholinemediated histamine release
461. (B) Ketamine is unique among the IV induction agents in that it usually produces cardiac stimulation manifested by increased heart rate, mean arterial pressure, and cardiac output. Ketamine is believed to have a centrally mediated sympathetic nervous system stimulating effect. This effect is, however, not related to dose. In isolated rabbit and canine hearts and in intact dogs, ketamine has been demonstrated to produce myocardial depression. Clinically, however, the myocardial depressant properties of ketamine are overridden by its sympathetic nervous system stimulating properties. When systemic catecholamines have been depleted or when the patient is under deep anesthesia, the myocardial depressant properties of ketamine may predominate (Stoelting: Pharmacology and Physiology in Anesthetic Practice, ed 4, p 172).
462. MH is believed to involve a generalized disorder of membrane permeability to A. Sodium B. Potassium C. Calcium D. Magnesium
462. (C) In the normal muscle cell, depolarization results in release of calcium from the sarcoplasmic reticulum. The increased intracellular calcium concentration results in muscle contraction. The calcium then is rapidly taken up via calcium pumps back into the sarcoplasmic reticulum, resulting in relaxation. Both the release and reuptake of calcium are energy-requiring processes (i.e., result in the hydrolysis General Anesthesia 139 of adenosine triphosphate [ATP]). Dantrolene, the pharmacologic treatment for MH, blocks release of calcium from the sarcoplasmic reticulum without affecting the reuptake process. The defect in MH is thought to be decreased control of intracellular calcium stores preventing muscle relaxation (Barash: Clinical Anesthesia, ed 7, pp 622–624).
463. A 25-year-old man with a history of testicular cancer is scheduled to undergo an exploratory laparotomy under general anesthesia. He has received bleomycin for metastatic disease. Which of the following is an important consideration concerning the pulmonary toxicity of bleomycin? A. N2O should not be used B. Preoperative pulmonary function tests should be obtained C. The patient should be ventilated at a slow rate and inspiratory-to-expiratory (I:E) ratio of 1:3 D. Fio2 should be less than 0.3
463. (D) Approximately 4% of patients treated with bleomycin develop pulmonary toxicity, which manifests as severe pulmonary fibrosis and hypoxemia. Death from severe pulmonary toxicity occurs in approximately 1% to 2% of patients treated with bleomycin. Patients who are at greater risk for bleomycininduced pulmonary toxicity include elderly patients, those receiving more than 200 to 400 mg, those with coexisting lung disease, and those recently exposed to bleomycin. In addition, there is evidence that prior radiotherapy and possibly receipt of enriched concentrations of O2 (i.e., inspired oxygen >30%) during surgery increase risk of pulmonary toxicity. Clinically, patients gradually develop dyspnea, a nonproductive cough, and hypoxemia, and pulmonary function tests typically demonstrate changes in gas flow and lung volumes consistent with restrictive pulmonary disease. If radiographic evidence such as bilateral diffuse interstitial infiltrates appears, pulmonary fibrosis usually is irreversible (Stoelting: Pharmacology and Physiology in Anesthetic Practice, ed 4, pp 564–565).
464. A 39-year-old obese woman undergoes an abdominal hysterectomy under general anesthesia. Induction of anesthesia is uneventful. Sao2 is 98% during the first 15 minutes of the operation with 50% oxygen and 50% N2O. Then, at the request of the surgeon, N2O is discontinued (now 50% oxygen, 50% N2), the head is flexed, and the patient is placed in the Trendelenburg position to improve surgical exposure, and Sao2 falls to 90%. The MOST likely explanation for this desaturation is A. Diffusion hypoxia B. Decreased functional residual capacity (FRC) C. Mainstem intubation D. Decreased cardiac output
464. (C) Head flexion can advance the tube up to 1.9 cm toward the carina and in some cases convert an endotracheal intubation into an endobronchial intubation. Extension of the head has the opposite effect and can withdraw the tube up to 1.9 cm, resulting in extubation of some patients. Turning the head laterally can move the distal tip of the endotracheal tube about 0.7 cm away from the carina. The Trendelenburg position causes a cephalad shift of the mediastinum and can cause the endotracheal tube to migrate distally as well (Miller: Basics of Anesthesia, ed 6, p 242).
465. How long after intravitreal injection of sulfur hexafluoride and air can N2O be used without risk of increasing intraocular pressure? A. 1 hour B. 24 hours C. 10 days D. 1 month
465. (C) Sulfur hexafluoride is sometimes injected in the vitreous in patients with a detached retina to mechanically facilitate reattachment. To prevent changes in the size of the gas bubble, the patients should be given 100% O2 15 minutes before injection of sulfur hexafluoride. If these patients are anesthetized with general anesthesia within 10 days, N2O should not be given because N2O can diffuse into the gas bubble, increasing intraocular pressure, and may result in blindness (Barash: Clinical Anesthesia, ed 7, pp 1391–1392).
466. A 54-year-old woman is undergoing a total thyroidectomy under general anesthesia. The patient is awakened in the OR, the mouth and pharynx are suctioned, and after intact laryngeal reflexes are demonstrated, the endotracheal tube is removed. Two days later, the anesthesiologist is consulted because the patient has severe stridor and upper airway obstruction. The most likely cause of airway obstruction in this patient is A. Damage to the recurrent laryngeal nerve B. Hematoma C. Tracheomalacia D. Hypocalcemia
466. (D) The symptoms of hypocalcemia, which manifest as laryngospasm or laryngeal stridor, usually develop within the first 24 to 96 hours after total thyroidectomy. After the airway is established and secured, the patient should be treated with IV calcium in the form of either calcium gluconate or calcium chloride (Barash: Clinical Anesthesia, ed 7, pp 352–353, 1330).
467. A 27-year-old obese woman is scheduled to undergo foot surgery under general anesthesia. She underwent a subtotal thyroidectomy 3 years ago and takes levothyroxine (Synthroid). Which of the following laboratory tests would be the MOST useful in evaluating whether this patient is euthyroid? A. Total plasma thyroxine (T4) B. Total plasma triiodothyronine (T3) C. Thyroid-stimulating hormone (TSH) D. Resin triiodothyronine uptake General Anesthesia 123
467. (C) Because the circulating levels of T3 and T4 regulate TSH release from the anterior pituitary gland by a negative feedback mechanism, a normal plasma concentration of TSH confirms a euthyroid state. The pharmacologic treatment of choice for patients with hypothyroidism is sodium levothyroxine (T4). Sodium levothyronine (triiodothyronine, T3) and desiccated thyroid are alternate therapeutic agents (Barash: Clinical Anesthesia, ed 7, p 1328; Hines: Stoelting’s Anesthesia and Co-Existing Disease, ed 6, pp 389–390).
468. An 85-year-old man with no previous medical history except for cataracts is undergoing a transurethral resection of the prostate gland under spinal anesthesia. Twenty minutes into the procedure the patient becomes restless. Over the next 20 minutes, his blood pressure increases from 110/70 to 140/90 mm Hg and his heart rate slows from 90 to 50 beats/min. The patient is noted to have some difficulty breathing. The most likely cause of these symptoms in this patient is A. Volume overload B. Hyponatremia C. High spinal D. Bladder perforation
468. (A) Large quantities of irrigating fluid can be absorbed during transurethral resection of the prostate gland because the open venous sinuses in the prostate allow the irrigation fluid to be absorbed. On average, from 10 to 30 mL of fluid per minute are absorbed, and during long cases this can amount to several liters, causing hypertension, reflex bradycardia, and pulmonary congestion. Treatment consists of fluid restriction and a loop diuretic (e.g., furosemide) when the [Na+] level is greater than 120 mEq/L. Rarely does the amount of fluid absorbed cause significant hyponatremia ([Na+] <120 mEq/L). In these cases of significant hyponatremia, 3% sodium chloride may be infused slowly intravenously (in addition to the loop diuretic and fluid restriction) until the sodium level reaches 120 mEq/L (Barash: Clinical Anesthesia, ed 7, pp 1428–1429).
469. A 17-year-old patient with third-degree burns over 30% of his body is scheduled for débridement and skin grafting 12 days after sustaining a thermal injury. Select the TRUE statement regarding the use of depolarizing and nondepolarizing muscle relaxants in this patient, compared with normal patients. A. Sensitivity to both depolarizing and nondepolarizing muscle relaxants is increased B. Sensitivity to both depolarizing and nondepolarizing muscle relaxants is decreased C. Sensitivity to depolarizing muscle relaxants is increased while sensitivity to nondepolarizing muscle relaxants is decreased D. Sensitivity to depolarizing muscle relaxants is decreased while sensitivity to nondepolarizing muscle relaxants is increased
469. (C) Patients who have sustained thermal injuries are at risk for massive potassium release and potential cardiac arrest if succinylcholine is administered 24 hours or more after they sustain the burn, and they remain at risk until the burn has healed. This increased sensitivity to succinylcholine is thought to be related to proliferation of extrajunctional receptors. These same receptors are thought to be related to the increased requirement for nondepolarizing neuromuscular blocking agents in these patients (Barash: Clinical Anesthesia, ed 7, p 1523). 140 Part 2 Clinical Sciences
470. A patient undergoes parotid gland removal under general anesthesia. Each of the following assesses facial nerve function EXCEPT A. Clenching teeth B. Closing eyes C. Pursing lips D. Eyebrow lift
470. (A) The facial nerve (seventh cranial nerve) runs within the substance of the parotid gland and might become damaged during parotid surgery. The facial nerve innervates the lacrimal, submandibular, and sublingual glands, is sensory to the anterior two thirds of the tongue, and innervates all of the muscle of facial expression (including the orbicularis oculi—close the eyelids; orbicularis oris—purse the lips; frontalis—raise the eyebrows). The trigeminal nerve (fifth cranial nerve) innervates the muscles of mastication (masseter, temporalis, medial, and lateral pterygoids), which are used to clench the teeth (Miller: Basics of Anesthesia, ed 6, p 497; Orient: Sapira’s Art and Science of Bedside Diagnosis ed 4, pp 533–537).
471. A 65-year-old patient with a history of chronic obstructive pulmonary disease and coronary artery disease (CAD) undergoes a laparoscopic nephrectomy uneventfully under general desflurane anesthesia. In the recovery room, ABGs are as follows: Pao2 60 mm Hg, Paco2 50 mm Hg, pH 7.35, and hemoglobin 8.1 g/dL. Which of the following steps would produce the greatest increase in O2 delivery to the myocardium? A. Administration of 100% O2 with a close-fitting mask B. Administration of 35% O2 with a Venturi mask C. Administer 1 ampule of HCO3 D. Transfuse with 2 units of packed red blood cells (RBCs)
471. (D) One gram of hemoglobin can combine with 1.34 mL of O2. None of the other choices in this question will do as much to increase the O2-carrying capacity of this patient’s blood as a transfusion (Stoelting: Pharmacology and Physiology in Anesthetic Practice, ed 4, pp 787, 849).
472. Allergic reactions occurring during the immediate perioperative period are MOST commonly attributable to administration of A. Muscle relaxants B. Local anesthetics C. Antibiotics D. Opioids
472. (A) Many of the drugs commonly administered during surgery and anesthesia have the potential to evoke allergic reactions (e.g., morphine, propofol, local anesthetics, antibiotics, and protamine, as well as other materials used during surgery, such as vascular graft material, chymopapain, and latex). Virtually all drugs administered IV have been reported to cause allergic reactions. Possible exceptions include benzodiazepines and ketamine. An allergic reaction should be considered when there is an abrupt fall in blood pressure accompanied by increases in heart rate that exceed 30% of the control values. Greater than 60% of all drug-induced allergic reactions observed during the perioperative period are attributable to muscle relaxants. Latex allergy is thought to be responsible for 15% of allergic reactions under anesthesia, sometimes including reactions originally attributed to other substances. Patients at risk for latex allergy include health care workers and patients with spina bifida. Although most drug-induced allergic reactions develop within 5 to 10 minutes of exposure, latex signs typically take more than 30 minutes to develop (Hines: Stoelting’s Anesthesia and Co-Existing Disease, ed 6, pp 525–529).
473. Caution is advised when using succinylcholine in patients with Huntington chorea because A. They are at increased risk for MH B. Potassium release may be excessive C. They may have a decreased concentration of pseudocholinesterase D. There may be adverse interactions between succinylcholine and phenothiazine
473. (C) Decreased levels of pseudocholinesterase have been reported in patients with Huntington chorea. For this reason, the effects of succinylcholine may be prolonged in some of these patients. It has been suggested that the sensitivity to nondepolarizing muscle relaxants is also increased (Hines: Stoelting’s Anesthesia and Co-Existing Disease, ed 6, p 247).
474. Which of the following would NOT result in an increase in intraocular pressure? A. Increase in Paco2 from 35 to 40 mm Hg B. 100 mg IM succinylcholine C. Acute rise in venous pressure from coughing D. 100 mg IV succinylcholine in a patient in whom eye muscles have been detached from the globe
474. (A) Normal intraocular pressure is 10 to 22 mm Hg. In general, IV anesthetics, with the possible exception of ketamine, decrease intraocular pressure. In addition, nondepolarizing neuromuscular blockers, inhaled anesthetics, narcotics, carbonic anhydrase inhibitors, osmotic diuretics, and hypothermia decrease intraocular pressure. However, elevation of Paco2 out of the physiologic range, as seen with hypoventilation as well as arterial hypoxemia, will increase intraocular pressure. Depolarizing neuromuscular blockers, such as succinylcholine, also increase intraocular pressure. This increase in intraocular pressure occurs when succinylcholine is administered IM or IV. Pretreatment with a nondepolarizing muscle relaxant before administering succinylcholine may attenuate the rise in intraocular pressure. The mechanism for the increase in intraocular pressure after succinylcholine use is related to drug-induced cycloplegia rather than contraction of extraocular muscles, as this increase in intraocular pressure will occur even if the intraocular muscles are cut. The greatest increase in intraocular pressure occurs with coughing or vomiting, where the intraocular pressure may increase as much as 35 to 50 mm Hg. The proposed mechanism for the acute increase in intraocular pressure is an increase in venous pressure. There does not appear to be a change in intraocular pressure with changes within normal physiologic ranges in arterial blood pressure or Paco2 (Barash: Clinical Anesthesia, ed 7, pp 1375–1376; Miller: Basics of Anesthesia, ed 6, pp 487–488).
475. An apnea-hypopnea index of 30 means A. Episodes of hypopnea are 30 times more common than apnea B. Apnea/hypopnea episodes occur at a rate of 30 per sleep cycle C. Episodes of apnea and hypopnea occur at a rate of 30 per hour D. Apnea/hypopnea episodes last 30 seconds
475. (C) The apnea-hypopnea index (AHI) is used to quantify the number of apnea or hypopnea episodes that occur per hour. Apnea is defined as no ventilation for periods of 10 seconds or more. Hypopnea is defined as a 50% decrease in airflow or a decrease sufficient to cause a decrease in oxygen saturation of 4%. An AHI of greater than 30 signifies severe OSA (Miller: Miller’s Anesthesia, ed 8, p 2203; Lobato: Complications in Anesthesiology, p 625).
476. Which of the following preoperative pulmonary function tests is NOT associated with an increased operative risk for pneumonectomy? A. FEV1 less than 50% of the FVC B. FEV1 less than 2 L C. Maximum breathing capacity less than 50% of predicted D. Residual volume/total lung capacity (TLC) less than 50%
476. (D) Any patient who is scheduled for a pneumonectomy should undergo a series of preoperative pulmonary function tests. These tests are generally conducted in three phases. The tests listed in this question pertain to the first battery of pulmonary function tests, which are whole-lung tests. Residual volume to General Anesthesia 141 TLC greater than 50% (not <50%) is associated with an increased operative risk. If the results of any of the initial whole-lung tests are below the acceptable limits, a second phase of testing should be carried out in which the function of each lung is evaluated separately. The predicted postoperative FEV1 after the second phase of pulmonary function testing is carried out should be greater than 0.85 L. If the criteria for the second level of pulmonary function testing cannot be met and pneumonectomy is still desired, then a third level of testing should be carried out. During the third phase of testing, postoperative conditions mimicking pneumonectomy are produced by occluding the pulmonary artery with a balloon on the side that is to be resected. Results of this test that are consistent with poor outcome after pneumonectomy include mean pulmonary artery pressure greater than 40 mm Hg, Paco2 greater than 60 mm Hg, or Pao2 less than 45 mm Hg (Miller: Miller’s Anesthesia, ed 8, pp 1943–1945, 1981–1982).
477. A 26-year-old man is undergoing an emergency exploratory laparotomy under general anesthesia with isoflurane. Sao2 is 89% on the pulse oximeter. Pao2 on ABGs is 77 mm Hg. The patient’s core body temperature is 35° C. What is the corrected Pao2? A. 68 mm Hg B. 72 mm Hg C. 77 mm Hg D. 86 mm Hg 124 Part 2 Clinical Sciences
477. (A) Measured Pao2 should be decreased about 6% for each degree Celsius cooler the patient’s temperature is than the electrode (37° C). Because the patient is 2° C cooler than the electrode, a 12% decrease (9 mm Hg) would be expected in this patient (77 mm Hg − 9 mm Hg = 68 mm Hg) (Miller: Basics of Anesthesia, ed 6, p. 338).
478. A 27-year-old patient with a 10-year history of Crohn disease is scheduled to undergo drainage of a rectal abscess under general anesthesia. His preoperative medications include prednisone, sulfasalazine, and cyanocobalamin. He has no known allergies and is otherwise healthy. Before induction of anesthesia, the patient is noted to have central cyanosis and the pulse oximeter shows an Sao2 of 89%, which does not increase after the administration of 100% O2 for 2 minutes. ABGs are as follows: Pao2 490 mm Hg, Paco2 32 mm Hg, pH 7.43, Sao2 89%. The MOST likely cause of these findings is A. Presence of sulfhemoglobin B. Presence of methemoglobin C. Presence of cyanhemoglobin D. Presence of carboxyhemoglobin
478. (A) The two main causes of central cyanosis are decreased arterial oxygen saturation and hemoglobin abnormalities (e.g., methemoglobinemia and sulfhemoglobinemia). Sulfasalazine (Azulfidine) can cause the formation of sulfhemoglobin. Sulfhemoglobin, like methemoglobin, may cause low O2 saturation in the face of high Pao2. There is no treatment for sulfhemoglobinemia except to wait for the destruction of the erythrocytes (Hines: Stoelting’s Anesthesia and Co-Existing Disease, ed 6, pp 296, 415).
479. Low-molecular-weight heparin (LMWH) A. Is as likely to cause heparin-induced thrombocytopenia (HIT) as unfractionated heparin B. Should be monitored with partial thromboplastin time (PTT) for clinical effect C. Can be fully reversed with protamine D. LMWH has a longer plasma half-life than unfractionated heparin
479. (D) Unfractionated heparin is a mixture of highly sulfated glycosaminoglycans with molecular weights of 5000 to 30,000 daltons. The onset of action of unfractionated heparin is immediate, the plasma halflife is ½ hour to 2 hours, and it can be completely reversed with protamine. Clinically, monitoring of anticoagulation is usually performed with the aPTT test with a target prolongation of 1.5 to 2 times control. When unfractionated heparin is used for cardiopulmonary bypass, the doses are much higher and it is monitored with the activated clotting time or ACT test (>400 seconds is usually considered safe for cardiopulmonary bypass). LMWHs are 4000 to 5000 daltons in size, the onset of action is 20 to 60 minutes, the plasma half-life is 4.5 hours, and it can only be partially reversed (65%) with protamine. Monitoring the LMWH’s effects is not performed, because the PT and the aPTT tests are most often unaffected. LMWHs have a much lower risk for HIT compared to the unfractionated heparin (Miller: Basics of Anesthesia, ed 6, pp 357–358; Miller: Miller’s Anesthesia, ed 8, pp 1872–1873).
480. In a given patient, if a creatinine of 1.0 corresponds to a glomerular filtration rate (GFR) of 120 mL/min, a creatinine of 4.0 would correspond to A. 20 mL/min B. 30 mL/min C. 40 mL/min D. 50 mL/min
480. (B) Serum creatinine is inversely proportional to the GFR. With the increase in creatinine by a factor of 4, the GFR is divided by four; that is, 120/4 = 30 mL/min (Lobato: Complications in Anesthesiology, p 433; Miller: Miller’s Anesthesia, ed 8, pp 558–559).
481. The incidence of each of the following is increased in patients with Down syndrome (trisomy 21) EXCEPT A. Malignant hyperthermia B. Congenital heart disease C. Smaller trachea D. Occipito-atlantoaxial instability
481. (A) Trisomy 21 or Down syndrome is the most common human chromosomal syndrome seen. An increased incidence of congenital hypothyroidism occurs. About one fourth of children with Down syndrome and many adults have smaller tracheas than predicted and require an endotracheal tube that is one or two sizes smaller. One should avoid unnecessary flexion or extension of the neck during intubation because occipito-atlantoaxial instability occurs in about 15% to 20% of patients. Because subluxation is relatively uncommon, routine neck radiographs for all Down syndrome patients are excessive. More than 40% of Down syndrome children have congenital heart disease (e.g., endocardial cushion defects, ventricular septal defects, tetralogy of Fallot, patent ductus arteriosus). Although some children have hypotonia, an increased incidence of MH has not been reported in these patients (Baum: Anesthesia for Genetic, Metabolic, and Dysmorphic Syndromes of Childhood, ed 2, pp 105–107; Hines: Stoelting’s Anesthesia and Co-Existing Disease, ed 6, pp 634–635).
482. A 55-year-old man is to undergo a laparoscopic cholecystectomy under general anesthesia. The patient has a 40-pack-per-year smoking history and a history of CHF. The patient receives metoclopramide and scopolamine preoperatively. General anesthesia is induced with ketamine, and the patient undergoes the procedure uneventfully. However, in the recovery room the patient complains of not being able to see objects “up close.” Which of the following would be the MOST likely cause of this complaint? A. Emergence delirium from ketamine anesthesia B. Effect of scopolamine C. Effect of Trendelenburg position D. Corneal abrasion
482. (B) Scopolamine is an anticholinergic that may produce mydriasis and cycloplegia. This can result in the inability of a patient’s eyes to accommodate (Stoelting: Basics of Anesthesia, ed 6, p 76).
483. MH and neuroleptic malignant syndrome share each of the following characteristics EXCEPT A. Generalized muscular rigidity B. Hyperthermia C. Effectively treated with dantrolene D. Flaccid paralysis after administration of vecuronium
483. (D) Neuroleptic malignant syndrome is a potentially fatal disease that affects 0.5% to 1% of all patients being treated with neuroleptic (antipsychotic) drugs. The syndrome develops gradually over 1 to 3 days in young males and is characterized by the following: (1) hyperthermia, (2) skeletal muscle rigidity, (3) autonomic instability manifested by changes in blood pressure and heart rate, and (4) fluctuating 142 Part 2 Clinical Sciences levels of consciousness. The mortality from neuroleptic malignant syndrome is 20% to 30%. Liver transaminases and creatine phosphokinase levels are often elevated in these patients. Treatment includes supportive care and administration of dantrolene. This disease may mimic MH because of its many similarities. One difference between neuroleptic malignant syndrome and MH is the fact that nondepolarizing muscle relaxants such as vecuronium or cisatracurium will cause flaccid paralysis in patients with neuroleptic malignant syndrome but not in patients with MH (Hines: Stoelting’s Anesthesia and Co-Existing Disease, ed 6, pp 540, 635–640).
484. A 23-year-old man involved in a motor vehicle accident is brought to the OR for open reduction and internal fixation of bilateral leg fractures under general anesthesia. During the surgery the patient is transfused with 7 units of type AB, Rh-negative packed RBCs and 3 units of platelets. At the end of the procedure, the endotracheal tube is removed and the patient is taken to the ICU. Postoperatively, the patient complains of shortness of breath and arterial hypoxemia is noted. His temperature is 38° C, heart rate is 146 beats/min, blood pressure is 105/69 mm Hg, and respiratory rate is 36 breaths/min. In addition, the patient is noted to have a fine petechial rash on his neck, chest, and shoulders. Which of the following is the MOST likely cause of these signs and symptoms? A. Pulmonary embolism B. Transfusion reaction to packed RBCs C. Transfusion-related acute lung injury (TRALI reaction) D. Fat embolism
484. (D) The classic signs of fat embolism include tachycardia, dyspnea, mental confusion, and fever, and frequently there may be a petechial rash on the upper part of the body. Fat embolism is more common after long bone fractures (e.g., femur and tibia) and usually occurs between 12 and 72 hours after long bone fractures (Hines: Stoelting’s Anesthesia and Co-Existing Disease, ed 6, pp 213–214).
485. Remifentanil is metabolized primarily by A. Kidneys B. Liver C. Nonspecific esterases D. Pseudocholinesterase
485. (C) Remifentanil is an ultrashort-acting narcotic. Chemically it is a derivative of piperidine (like fentanyl), but remifentanil has an ester linkage and is rapidly broken down by nonspecific plasma as well as tissue esterases. The elimination half-life is less than 20 minutes and is best administered by a continuous infusion. Pseudocholinesterase deficiency or renal or hepatic failure does not affect remifentanil’s rapid metabolism (Barash: Clinical Anesthesia, ed 7, p 509; Stoelting: Pharmacology and Physiology in Anesthetic Practice, ed 4, p 114).
486. A term infant with good muscle tone and strong cry has an 83% saturation on room air 5 minutes after delivery. The MOST appropriate action at this point would be A. Bag and mask ventilation with 100% oxygen B. Intubate and ventilate with 100% oxygen C. Spontaneous breathing with 100% oxygen D. Observe
486. (D) A term infant with a strong cry and good muscle tone does not require oxygen therapy based on a 5-minute saturation alone. The fetal lungs make a rapid transition from a fluid-filled organ to an airfilled organ. As the zones of atelectasis open, the saturation rises. The table below shows acceptable preductal oxygen saturation as a function of time. Minutes Preductal Oxygen Saturation 1 60%-65% 2 65%-70% 3 70%-75% 4 75%-80% 5 80%-85% 10 85%-95% Data from Kattwinkel J et al: Neonatal resuscitation: 2010 American Heart Association Guidelines for Cardiopulmonary Resuscitation and Emergency Cardiovascular Care, Pediatrics 126:e1400–e1413, 2010.
487. Patients who undergo extracorporeal shock wave lithotripsy are at increased risk for A. Venous air embolism B. Pneumothorax C. Hypotension with regional anesthesia at the end of the procedure D. Postdural puncture headache with spinal anesthesia
487. (C) Anesthesia for extracorporeal shock wave lithotripsy may be accomplished with either general anesthesia or epidural anesthesia. When a patient is submerged in the stainless steel tub, the peripheral vasculature becomes compressed by the hydrostatic pressure, resulting in an increase in preload. Removing the patient from the tank has the opposite effect. In patients who have received epidural anesthesia, there is an increased incidence of hypotension caused by epidural-induced sympathectomy after they emerge from the bath (Miller: Basics of Anesthesia, ed 6, p 627).
488. The most common reason for admitting outpatients to the hospital following general anesthesia is A. Nausea and vomiting B. Inability to void C. Inability to ambulate D. Surgical pain General Anesthesia 125
488. (A) The most common reason for unexpected hospital admission after outpatient general anesthesia, as well as a prolonged recovery-room stay (for both adults and children), is nausea and vomiting. Two other reasons for a prolonged recovery-room stay are pain and drowsiness (Barash: Clinical Anesthesia, ed 7, pp 854, 856).
489. A 37-year-old man with myasthenia gravis arrives in the emergency room confused and agitated after a 2-day history of weakness and increased difficulty breathing. ABGs on room air are Pao2 60 mm Hg, Paco2 51 mm Hg, HCO3 – 25 mEq/L, pH 7.3, Sao2 of 90%. His respiratory rate is 30 breaths/min and tidal volume (Vt) is 4 mL/kg. After administration of edrophonium 2 mg IV, his Vt declines to 2 mL/kg. What should be the most appropriate step in the management of this patient at this time? A. Tracheal intubation and mechanical ventilation B. Repeat the test dose of edrophonium C. Administer neostigmine D. Administer atropine for cholinergic crisis
489. (A) Cholinergic crisis can be differentiated from myasthenic crisis by administering small IV doses of anticholinesterases. With a cholinergic crisis, there are significant muscarinic effects (e.g., salivation, bradycardia, miosis) and an accentuated muscle weakness. Because this patient’s Vt decreased with the administration of edrophonium, the diagnosis of cholinergic crisis is made. Although atropine may be needed to treat the cholinergic symptoms, muscle weakness will be worse and these patients need to be intubated until the muscle strength returns (Hines: Stoelting’s Anesthesia and Co-Existing Disease, ed 6, p 450). General Anesthesia 143
490. Select the FALSE statement regarding tramadol (Ultram). A. Ondansetron may interfere with part of tramadol’s analgesia B. Tramadol is associated with seizures in patients taking selective serotonin reuptake inhibitors (SSRIs) C. It is relatively safe in patients whose pain makes them suicidal D. Its analgesic effects are partially antagonized by naloxone
490. (C) Tramadol, a synthetic codeine analog, is a centrally acting analgesic. It can be used for mild to moderate pain but is not as effective as morphine or meperidine for severe or chronic pain. One drawback for tramadol’s perioperative use is its high incidence of nausea and vomiting. Its mechanism of action for analgesia is complex. It is a weak μ-receptor agonist, it inhibits serotonin and norepinephrine reuptake, and it enhances serotonin release. Tramadol-induced analgesia is not entirely reversed with naloxone; however, the respiratory depression and sedation can be reversed. Ondansetron, a serotonin antagonist, may interfere with part of tramadol’s analgesic action. Because of its low μ-receptor agonist activity, it may be less likely to produce physical dependence than other stronger narcotics. Seizures have been reported in patients receiving tramadol alone. The drug should be used with caution in patients taking drugs that lower the seizure threshold, such as tricyclic antidepressants and SSRIs. It has some monoamine oxidase (MAO) inhibiting activity and should not be used in patients taking MAO inhibitors. Another warning is its use in patients who are depressed or suicidal. Tramadol is not recommended in depressed or suicidal patients because excessive doses, either alone or with other CNS depressants including alcohol, are a major cause of drug-related deaths with fatalities reported within the first hour of overdosage. Patients who are depressed or suicidal are better managed with non-narcotic analgesics (Hardman: Goodman & Gilman’s The Pharmacological Basis of Therapeutics, ed 10, p 590; Physicians’ Desk Reference 2009, ed 63, pp 2428–2431; Stoelting: Pharmacology and Physiology in Anesthetic Practice, ed 4, p 117).
491. In statistical hypothesis testing, if the P value is less than the predetermined α value, which of the following is most likely? A. The observed result is unlikely under the null hypothesis B. The observed result is unlikely under an alternative hypothesis C. The sample size is too small D. The predetermined power is too low
491. (A) The null hypothesis states that there is no difference between two groups of data, while the alternative hypothesis states the opposite or that there is a difference between the groups. The P value is derived from a test statistic and is the probability that we could have observed a difference if in reality the null hypothesis was true and there was not a difference. If the P value is less than a predetermined level of significance (the α value, often set at = 0.05) then the null hypothesis (no difference) is rejected and the differences observed are stated to be statistically significant (P < 0.05). It can then be stated that it is unlikely (calculated to be less than a 1 in 20 probability) that the differences detected in the two groups occurred by random chance or that the null hypothesis was true. When the P value is less than α but there actually is not a difference between the groups, it is called a type 1 error. On the other hand, if no statistically significant differences are detected (P value > α), we accept that the null hypothesis (no difference exists) is true. If we accept the null hypothesis when the alternative hypothesis (there is a difference) is in fact true, a type 2 error has occurred. Type 2 errors are related to the power of the study. Power is the probability of rejecting the null hypothesis (no difference) when a specific alternative hypothesis (difference) is correct. Power is related to the magnitude of the difference to detect, the variability of the data, the α level, and the sample size. Often a power of 0.8 is selected, meaning that we accept an 80% probability that the null hypothesis (no difference) is true or that there is also a 20% chance that a difference does exist but was not observed. Larger sample sizes make it easier to observe that a difference exists and increase the power of an analysis (Miller: Miller’s Anesthesia, ed 8, pp 3250–3251).
492. A 72-year-old man undergoes emergency repair of an abdominal aortic aneurysm. In the first hour after release of the suprarenal cross-clamp, urine output is only 10 mL. After administration of furosemide 20 mg IV, urine output increases to 100 mL/hr. Urine [Na+] is 43 mEq/L, and urine osmolality is 210 mOsm/L. The MOST likely cause of the initial oliguria is A. Increased ADH B. Renal hypoperfusion C. Acute tubular necrosis D. Impossible to differentiate
492. (D) In the absence of diuretics, oliguria associated with urine sodium concentration greater than 40 mEq/L and urine osmolality less than 400 mOsm/L is strongly suggestive of intrinsic renal disease (e.g., acute tubule necrosis), whereas prerenal causes have urine sodium concentration less than 20 mEq/L and urine osmolality greater than 400 mOsm/L. Furosemide, mannitol, and dopamine, however, obscure the accurate diagnosis (Hines: Stoelting’s Anesthesia and Co-Existing Disease, ed 6, pp 335–338; Miller: Basics of Anesthesia, ed 6, pp 450–452).
493. A healthy 25-year-old man is anesthetized for a sagittal split osteotomy. Anesthesia is induced with propofol, hydromorphone, and vecuronium and maintained with 2.1% sevoflurane and 50% N2O. After induction, the nose is prepped with 4% lidocaine and 1% phenylephrine, and the patient is intubated through the right naris. Before emergence, the surgeon performs a bilateral inferior alveolar nerve block. The patient is reversed with neostigmine and glycopyrrolate. When the patient awakens, he is noted to have an 8-mm pupil on the right and a 3-mm pupil on the left. Results of physical examination are otherwise unremarkable. The most likely explanation for the dilated pupil is A. Right stellate ganglion block B. Accidental introduction of lidocaine into right eye C. Accidental introduction of phenylephrine into right eye D. Glycopyrrolate
493. (C) In an unconscious patient, a unilateral dilated pupil would be a matter of grave concern. In an awake patient with a normal neurologic examination, however, it is less worrisome. An inferior alveolar nerve block involves injection of about 2 mL of 2% lidocaine around the inferior alveolar nerve just behind the molars in the lower jaw. Even a grossly misdirected needle probably could not reach the stellate ganglion, but were it possible, the result would be a Horner syndrome (miosis, not mydriasis, ptosis, anhidrosis, and vasodilation over the face). Blockade of the ciliary ganglion could cause mydriasis on the ipsilateral side, but reaching the ciliary ganglion, located between the optic nerve and lateral rectus muscle about 1 cm from the posterior limit of the orbit, would be almost impossible with a needle directed toward the mandible. Glycopyrrolate administered systemically does not cause mydriasis, as it is not capable of crossing the blood-brain barrier. Lidocaine instilled directly into the eye does not 144 Part 2 Clinical Sciences produce mydriasis, but phenylephrine does. Care must be taken not to spray local anesthetic (with or without vasoconstrictor) into the eyes while applying topical anesthesia to the nares (Stoelting: Pharmacology and Physiology in Anesthetic Practice, ed 4, p 304).
494. A 40-year-old man is undergoing a left inguinal hernia repair under general anesthesia in San Diego, California. N2O is administered at 3 L/min, O2 at 1 L/min, and isoflurane at 0.85%. What MAC is this patient receiving? A. 0.8 B. 1.25 C. 1.50 D. 1.75
494. (C) MAC is the minimum alveolar concentration of anesthetic that will prevent movement of 50% of patients when a skin incision is made at sea level (e.g., San Diego). MAC × 1.3 will prevent movement in 95% of patients. In this question, total gas flow is 4 L/min (1 L/min + 3 L/min). Roughly 75% of the total gas is N2O. The MAC of N2O is 104%. The patient is receiving about 0.75 MAC N2O. The MAC for isoflurane is 1.15. A concentration of 0.85% would represent 0.75 MAC. Because MACs are additive, the total MAC would be 1.5 (Barash: Clinical Anesthesia, ed 7, pp 458–459; Miller: Basics of Anesthesia, ed 6, p 82).
495. An otherwise healthy 140-kg, 24-year-old man is scheduled for vocal cord surgery under general anesthesia. Which of the following statements concerning his cardiac output at 140 kg compared with his cardiac output at his ideal body weight (70 kg) is CORRECT? A. Cardiac output is the same B. Cardiac output is increased by 10% C. Cardiac output is increased by 50% D. Cardiac output is doubled
495. (D) Cardiac output increases by about 100 mL/min for each kilogram of weight gained. It is estimated that every kilogram of adipose tissue contains nearly 3000 m of additional blood vessels. The additional cardiac output is due to ventricular dilation and increased stroke volume, as resting heart rates are not increased in obese patients (Hines: Stoelting’s Anesthesia and Co-Existing Disease, ed 6, p 318; Miller: Basics of Anesthesia, ed 6, pp 83–84).
496. Fenoldopam may be used as an alternative to which of the following? A. Epinephrine B. Phenylephrine C. Sodium nitroprusside D. Dopamine
496. (C) Fenoldopam (Corlopam) is a selective dopamine-1 receptor agonist with significant vasodilating properties. It has moderate affinity for α2 receptors but has no affinity for dopamine-2, α1, β, 5-hydroxytryptamine type 1 (5-HT1), or 5-HT2 receptors. It is used for treatment of patients with severe hypertension (especially with reduced renal function) and is administered as an IV infusion. It can be used as an alternative to sodium nitroprusside and has the advantage of no thiocyanate toxicity, rebound effect, or “coronary steal” effect, but it does contain sodium bisulfite and is contraindicated in patients with a known sulfite sensitivity. Dopexamine (Dopacard) is a synthetic analog related to dopamine with intrinsic activity at dopamine as well as β2 receptors and is used as an inotropic agent (Miller: Miller’s Anesthesia, ed 8, pp 367–368).
497. A 58-year-old hemophiliac is scheduled for total knee arthroplasty. His factor VIII levels are 35% of normal. Which of the following would be the most appropriate therapy before surgery? A. Administer sufficient cryoprecipitate to raise factor VIII levels to 50% normal B. Administer factor VIII concentrates to achieve levels of 100% normal C. Transfuse fresh frozen plasma until factor VIII levels are 100% normal D. None of the above 126 Part 2 Clinical Sciences
497. (B) Ideally, factor VIII levels should be raised to 100% predicted before elective surgery to ensure that the levels will not fall below 30% intraoperatively. Thirty percent of the normal factor VIII concentration or greater is thought to be necessary for a patient who is to undergo major surgery. Elimination half-time of factor VIII is 12 hours. This may be accomplished with factor VIII concentrate or cryoprecipitate. Fresh frozen plasma is no longer considered therapy for hemophilia (Hines: Stoelting’s Anesthesia and Co-Existing Disease, ed 6, p 421).
498. A 16-year-old boy whose maternal uncle has hemophilia A is scheduled for wisdom tooth extraction. Which test below would be the best screening test for hemophilia A? A. PTT B. Prothrombin time (PT) C. Thrombin time D. Bleeding time
498. (A) Hemophilia A is associated with decreased levels of factor VIII. PTT tests the intrinsic coagulation cascade and would be abnormally elevated in all but the most mild disease. A normal PTT is 25 to 35 seconds. Platelet count, PT, and bleeding times are normal (see also explanation to Question 395) (Hines: Stoelting’s Anesthesia and Co-Existing Disease, ed 6, p 421; Barash: Clinical Anesthesia, ed 7, pp 433–434).
499. The reason four twitches are used in the train-of-four to determine degree of neuromuscular blockade versus five (or more) is A. Comparison of greater than four twitches is too difficult B. Four twitches inform the user of the degree of blockade in the useful clinical range (i.e., 75%- 100% blockade) C. Post-tetanic facilitation will begin to appear after four twitches D. There would be no additional decrement in twitch height after four twitches
499. (D) Conventional peripheral nerve stimulators deliver four twitches at 2 Hz spaced 0.5 second apart. These devices were designed with the knowledge that successive twitches deplete acetylcholine stores. After the fourth twitch, there is no additional decrement in twitch height (Miller: Basics of Anesthesia, ed 6, p 156).
500. A 57-year-old man is undergoing a right eye enucleation under general anesthesia. The patient has no history of cardiac disease. During the operation, 5-mm ST-segment elevation is noted on lead II and the patient develops complete heart block. The coronary artery most likely affected is A. Circumflex coronary artery B. Right coronary artery C. Left main coronary artery D. Left anterior descending coronary artery
500. (B) Inferior ischemia is associated with blockage or spasm of the right coronary artery. The right coronary artery supplies blood to the atrioventricular node in 90% of patients. Complete heart block therefore is not unexpected in patients with severe CAD involving the right coronary artery (Hines: Stoelting’s Anesthesia and Co-Existing Disease, ed 6, pp 24–25).
501. Each of the following may increase MAC for volatile anesthetics EXCEPT A. Cocaine B. Hyperthyroidism C. Hypernatremia D. Tricyclic antidepressants
501. (B) MAC is influenced by a variety of disease states, conditions, drugs, and other factors. Drugs that increase CNS catecholamines, such as MAO inhibitors, tricyclic antidepressants, acute amphetamine ingestion, and cocaine, increase MAC. Other factors that increase MAC include hyperthermia, hypernatremia, patients with natural red hair, and infancy. It is interesting that MAC values are higher for infants than for neonates or older children and adults. Thyroid gland dysfunction including hyperthyroidism does not affect the MAC. Factors that lower MAC include narcotics, IV anesthetics, local anesthetics (except cocaine) and other sedatives, age (6% per decade), hypothermia, hypoxia, and severe anemia (e.g., Hgb < 5). The following table modified from the references in this question summarizes the impact of various factors on MAC (Barash: Clinical Anesthesia, ed 7, pp 458-459; Butterworth: Morgan & Mikhail’s Clinical Anesthesiology, ed 5, p 164; Miller: Basics of Anesthesia, ed 6, p 82). General Anesthesia 145 IMPACT OF PHYSIOLOGIC AND PHARMACOLOGIC FACTORS ON MINIMUM ALVEOLAR CONCENTRATION (MAC) No Change in MAC Increase in MAC Decrease in MAC Duration of anesthesia Type of surgery Hyperthyroidism Hypothyroidism Gender Hyperkalemia Drugs that increase CNS catecholamines (MAO inhibitors, tricyclic antidepressants, acute amphetamine use, cocaine, ephedrine) Chronic ethanol abuse Hyperthermia Hypothermia Infants Patients with natural red hair CNS depressants (narcotics, IV anesthetics, chronic amphetamine use) Acute ethanol use Hypernatremia Hyponatremia Increasing age Pregnancy Hypoxia CNS, central nervous system; MAO, monoamine oxidase.
502. A 37-year-old patient with a history of manic-depressive illness is scheduled to undergo surgery for removal of an intramedullary rod in the left tibia. Which of the following statements regarding potential untoward effects of lithium therapy is NOT true? A. Long-term administration may be associated with nephrogenic diabetes insipidus B. Administration of succinylcholine to patients treated with lithium may result in hyperkalemia C. Long-term therapy may be associated with hypothyroidism D. Duration of action of vecuronium may be prolonged
502. (B) Long-term lithium therapy in patients with manic-depressive illness may be associated with nephrogenic diabetes insipidus. Hypothyroidism may develop in about 5% of patients because lithium can inhibit the release of thyroid hormones. Lithium is almost 100% renally excreted. Reabsorption occurs at the proximal convoluted tubule and is inversely related to the concentration of sodium in the glomerular filtrate. Consequently, administration of diuretics (mainly thiazide, but to a lesser extent loop diuretics) may lead to the development of toxic lithium levels. Lithium has sedative properties and may reduce the need for IV and inhalational anesthetic agents. It may prolong the duration of action of both pancuronium and succinylcholine, but it is not associated with an exaggerated release of potassium when succinylcholine is administered (Brunton: Goodman & Gilman’s The Pharmacological Basis of Therapeutics, ed 12, pp 448–449; Hines: Stoelting’s Anesthesia and Co-Existing Disease, ed 6, p 539).
503. Treatment of hypotension in a patient anesthetized for resection of metastatic carcinoid would be best accomplished with A. Epinephrine B. Ephedrine C. Vasopressin (DDAVP) D. Octreotide
503. (D) Carcinoid tumors can arise wherever enterochromaffin cells are present. Most (>70%) originate in the intestine, and about 20% originate in the lung. Of those that originate in the gastrointestinal tract, 50% occur in the appendix, 25% in the ileum, and 20% in the rectum. These interesting tumors were called carcinoid because they were originally believed not to metastasize. We now know this is not true. The hormones released by the nonmetastatic tumors reach the liver by the portal vein and are rapidly inactivated. However, once metastases reach the liver, the released hormones reach the systemic circulation and produce signs and symptoms of the “carcinoid syndrome.” Symptoms include cutaneous flushing, abdominal pain, vomiting, diarrhea, hypotension or hypertension, bronchospasm, and hyperglycemia. The natural hormone somatostatin suppresses the release of serotonin and other vasoactive substances from the tumor. Because the half-life is about 3 minutes, somatostatin is given by infusion. Octreotide is a synthetic somatostatin analog with a half-life of 2.5 hours and is given SQ or IV for the prevention and treatment of carcinoid symptoms (e.g., hypotension, hypertension, bronchospasm). However, the treatment of hypotension in patients with carcinoid disease is different because ephedrine, epinephrine, and norepinephrine can release vasoactive hormones from the tumor and make the hypotension worse. Hypotension is best treated with fluids and IV octreotide or somatostatin. Hypertension is treated with deepening the anesthetic and administering octreotide, somatostatin, or labetalol. Bronchospasm is treated with IV octreotide, somatostatin, or nebulized ipratropium. When giving anesthesia to these patients it is probably wise to avoid drugs that release histamine and other vasoactive hormones that may precipitate symptoms. Propofol or etomidate are good induction agents, followed by maintenance anesthesia with a volatile anesthetic (e.g., isoflurane, sevoflurane, or desflurane) and/ or nitrous oxide with oxygen. Vecuronium, cisatracurium, and rocuronium appear to be safe muscle relaxants. Fentanyl, sufentanil, alfentanil, remifentanil, and benzodiazepines are also safe to use. The serotonin antagonist ondansetron is a useful antiemetic (Hines: Stoelting’s Anesthesia and Co-Existing Disease, ed 6, pp 297–298).
504. A 75-year-old man is scheduled to undergo elective orchiectomy for prostate cancer. The patient has selected spinal anesthesia. What is the minimum dermatomal level that must be achieved to carry out this operation? A. T4 B. T10 C. L3 D. S1
504. (B) Testicular innervation can be traced up to the T10 dermatomal level. For this reason, any operation that involves manipulation or traction on the testicles must have adequate anesthesia to prevent pain. This can be achieved with spinal or epidural anesthesia, which is associated with a T10 level of blockade (Barash: Clinical Anesthesia, ed 7, p 916). 146 Part 2 Clinical Sciences
505. A 31-year-old patient has been in the ICU on a ventilator for 24 hours after a motor vehicle accident. The patient does not open his eyes to any stimulus and has no verbal or motor response. The Glasgow Coma Scale corresponding to this patient would be A. 0 B. 1 C. 2 D. 3
505. (D) The Glasgow Coma Scale has three categories: eye opening, for which a maximum of 4 points can be received; best verbal response, for a maximum of 5 points; and best motor response, for a maximum of 6 points. The higher the score, the better the response; the minimal score for each category is 1. Mild head injury scores are 13 to 15, moderate are 9 to 12, and severe are 3 to 8. This severe head-injured patient is totally unresponsive and would receive a score of 3 (Barash: Clinical Anesthesia, ed 7, p 1018).
506. Hypoglycemia is more likely to occur in the diabetic surgical patient with which of the following diseases? A. Renal disease B. Rheumatoid arthritis requiring high-dosage prednisone C. Chronic obstructive lung disease treated with a terbutaline inhaler and aminophylline D. Manic-depressive disorder treated with lithium
506. (A) Insulin metabolism involves both the liver and kidneys. Renal dysfunction, however, has a greater impact on insulin metabolism than does hepatic dysfunction. In fact, unexpected prolonged effects of insulin sometimes are seen in patients with renal disease (Stoelting: Pharmacology and Physiology in Anesthetic Practice, ed 4, p 478).
507. Which of the following is most likely to be associated with a falsely elevated Sao2 as measured by pulse oximetry (dual wave)? A. Hemoglobin F B. Carboxyhemoglobin C. Methylene blue dye D. Fluorescein dye
507. (B) Most pulse oximeters illuminate tissue with two wavelengths of light: 660-nm red light and 940-nm infrared light. Because carboxyhemoglobin has an absorbance at 660 nm, very similar to O2 hemoglobin, it produces a falsely elevated Sao2 when present in the blood. Hemoglobin F, bilirubin, and fluorescein dye have no effect on pulse oximetry. Methylene blue, as well as indigo carmine and indocyanine green, lowers the Sao2 as measured by pulse oximetry. Methemoglobin absorbs red and infrared light equally well and gives saturation readings of 85% (Barash: Clinical Anesthesia, ed 7, pp 702–703; Miller: Basics of Anesthesia, ed 6, p 327).
508. Select the FALSE statement regarding clinical performance and sleep deprivation A. A period of vulnerability has been identified between 2 am and 7 am B. There is an increased incidence of motor vehicle accidents in post-call house staff C. When patient simulation was used to study sleep deprivation in anesthesia residents, no reduction in clinical performance was demonstrable D. After inception of restriction of resident work hours in July 2003, a reduction in patient death rates was shown to be less in hospitals with large numbers of resident physicians versus those with fewer General Anesthesia 127
508. (D) On March 4, 1984, Libby Zion, an 18-year-old college freshman, was admitted with a high fever, dehydration, and chills to a New York Hospital and died within a day. The cause of her death was widely believed to be due to a drug interaction between phenelzine, which she had taken for depression, and meperidine, which was used to calm her down. This led to a serotonin syndrome and more agitation. During the night her temperature rose to 107° F (42° C) and she suffered a cardiac arrest and could not be resuscitated. Cocaine had been detected in her body and may have contributed to her death as well. This case was used to exemplify the fact that the intern and residents taking care of her were overworked, and this eventually led to New York State Department of Health Code, Section 405, known as the Libby Zion Law, which limits the amount of work for residents to 80 hours per week. In 2003, the Accreditation Council for Graduate Medical Education (ACGME) adopted regulations for medical training in the United States. Since then, studies have looked at fatigue and clinical performance. A major peak in vulnerability occurs between 2 am and 7 am, with a smaller peak in the midafternoon. Single-occupant motor vehicle accidents occur more frequently in the morning. Although patient simulation of the effects of sleep deprivation have been studied, psychomotor performance and mood have been affected, but clinical performance was not affected. No difference in mortality rates were seen in the 2 years before compared to the 2 years after the 2003 guidelines were put into effect, and no difference in mortality was noted when large teaching programs (thought to be the most affected) were compared to smaller programs (Lerner: A Life-Changing Case for Doctors in Training, New York Times, August 14, 2011; Miller: Miller’s Anesthesia, ed 8, p 3239; New York State Department of Health Code, Section 405, known as the Libby Zion Law).
509. Gabapentin (Neurontin) as used in the treatment of chronic pain belongs to the same broad class of drugs as A. Carbamazepine B. Imipramine C. Clonidine D. Fluoxetine (Prozac)
509. (A) Gabapentin, an anticonvulsant, was developed to be a centrally active γ-aminobutyric acid (GABA) agonist but does not appear to interact with GABA receptors. Its mechanism for producing analgesia is unclear, but it may involve inhibition of voltage-activated calcium channels as well as potentiating GABA release. Carbamazepine slows the recovery rate of voltage-gated sodium channels, but it also is an anticonvulsant. Carbamazepine is indicated in the treatment of trigeminal neuralgia (Benzon: Essentials of Pain Medicine, ed 3, pp 123–129).
510. A 72-year-old man with a history of smoking, hypertension, and CHF undergoes a colonoscopy under sedation. The night before the procedure, he took his bowel prep but omitted his metoprolol and lisinopril. At the end of the procedure, his oxygen saturation is 83% and blood pressure is 175/85 mm Hg, and the ECG shows sinus rhythm with a heart rate of 120. Rales are easily heard in both lung fields. The patient is intubated. Echocardiogram shows 80% ejection fraction (EF). Which of the items below would be LEAST helpful in management? A. PEEP B. Furosemide C. Increase Fio2 D. Esmolol
510. (B) In evaluating this patient in heart failure (e.g., rales), one observes that the EF is high (e.g., 80%), afterload is high (e.g., elevated systolic blood pressure), and the heart rate is high (e.g., 120 beats/min). Although he has diffuse rales (often a sign of high preload and fluid overload), this patient is actually dehydrated from his bowel prep, and his left ventricle does not fill properly. To compensate for the low filling volume, the heart rate increases. Patients with heart failure and a normal ejection fraction (HFNEF), previously called diastolic heart failure, have signs of left-sided heart failure. To better understand this, think of the heart as a hydraulic pump that you need to not only empty effectively (during systole) but also need to fill effectively (during diastole). So in this case, your main goals are to slow the heart rate to allow the left ventricle adequate time to fill (e.g., with a β-blocker such as esmolol) and to better oxygenate him (e.g., increase the Fio2 and add PEEP). The diuretic furosemide would exacerbate General Anesthesia 147 the situation. Other conditions in which the left ventricle does not fill effectively include less compliant ventricular walls (e.g., thick from long-standing hypertension or aortic valve stenosis, fibrotic walls), less room to fill (e.g., cardiac tamponade), loss of the atrial kick (e.g., atrial fibrillation), and valvular stenosis (e.g., mitral stenosis) (Miller: Basics of Anesthesia, ed 6, p 172; Butterworth: Morgan & Mikhail’s Clinical Anesthesiology, ed 5, pp 419–421).
511. A 47-year-old morbidly obese patient develops bilateral blindness (only able to perceive light) after a 6-hour, three-segment laminectomy and fusion. The patient received 6 units of blood and 5 L of lactated Ringer solution. A mean arterial blood pressure was maintained at 50 to 60 mm Hg. The MOST likely structure involved in this visual loss is A. Central retinal artery B. Optic nerve C. Retina D. Cerebral cortex
511. (B) Perioperative visual loss associated with nonocular surgery is rare and may result from corneal trauma, retinal artery occlusion, retinal vein occlusion, optic nerve ischemia, or cortical disease. Although overall it is a rare problem, it may develop in up to 1% of prone spinal surgical cases and is most commonly due to ischemic optic neuropathy. The cause is unknown and multifactorial. Associated factors include prolonged intraoperative hypotension, anemia (Hgb <8), large intraoperative blood loss, prolonged surgery, and facial edema. It is more common in males and in patients with peripheral vascular disease, diabetes mellitus, and in tobacco users (Miller: Miller’s Anesthesia, ed 8, pp 3011–3012).
512. Each of the following statements regarding postoperative shivering is true EXCEPT A. It may increase metabolism and oxygen consumption significantly B. It may be treated with meperidine C. It may be treated with droperidol D. It does not occur in the absence of hypothermia
512. (D) Postoperative shivering or postanesthetic tremor can occur during recovery from all types of general anesthesia. If profound, shivering can increase metabolic rate and O2 consumption (100% to 200%) with an associated increase in cardiac output and minute ventilation. Although shivering usually occurs in patients with decreased body temperature, it also may occur in patients with normal body temperature after anesthesia. Postanesthesia shivering is best treated by a combination of supplemental oxygen, rewarming the patient, and/or administering IV meperidine. Other less frequently used pharmacologic treatments include clonidine, magnesium sulfate, calcium chloride, chlorpromazine, droperidol, and other opioids (e.g., butorphanol). Application of radiant heat to the face, head, neck, chest, and abdomen has been shown to eliminate shivering within minutes in postoperative patients, despite low core body temperatures (Butterworth: Morgan & Mikhail’s Clinical Anesthesiology, ed 5, pp 1185, 1264; Miller: Basics of Anesthesia, ed 6, p 643).
513. Electrocardiographic (ECG) changes associated with hyperkalemia include A. Increased P wave amplitude B. Shortened PR interval C. Narrowed and peaked T waves D. Increase in U-wave amplitude
513. (C) The ECG signs of hyperkalemia include narrowed and peaked T waves (earliest manifestation of hyperkalemia), decrease in P-wave amplitude, prolonged PR interval, and a widened QRS interval. In extreme cases, the ECG can appear as a sine wave as well as cardiac arrhythmias (e.g., sinus arrest, supraventricular tachycardia, atrial fibrillation, premature ventricular contractions, ventricular tachycardia, and ventricular fibrillation). These changes are potentiated by hypocalcemia, and intravenous calcium can rapidly correct some of these ECG changes. An increase in U-wave amplitude suggests hypokalemia, not hyperkalemia (Miller: Miller’s Anesthesia, ed 8, pp 1205–1206).
514. A 24-year-old is undergoing open reduction of an ankle fracture under general anesthesia with sevoflurane, N2O, and O2 through a laryngeal mask airway (LMA). Just after the vaporizer dial is turned up to 2%, the patient begins spontaneously breathing, but the inspiratory valve is not fully closing. The likely result of this (malfunctioning valve) is an increase in the inspired concentration of A. N2O B. CO2 C. O2 D. All of the above
514. (B) If the inspiratory valve becomes stuck in the open position, it will “malfunction” only during exhalation because, during inhalation, it is supposed to be open. During the exhalation phase of breathing, exhaled gases will exit through the expiratory valve into the expiratory limb of the circuit and beyond (proper path), as well as through the inspiratory valve into the inspiratory limb of the circuit (errant path). Gases traveling into the inspiratory limb (old gas) will be returned to the patient with next breath. The volume of recently exhaled gas is now drawn back into the patient’s lungs along with the “new” gas that would be inspired in a fully functional breathing circuit. The net effect is that oxygen, sevoflurane, and N2O will all be diluted, but the patient rebreathes CO2; thus, it will be the only gas with an increased inspired concentration (normal inspired CO2 is zero) as a result of the stuck inspiratory valve (Miller: Basics of Anesthesia, ed 6, p 208).
515. Each of the following is associated with acromegalic patients undergoing transsphenoidal hypophysectomy EXCEPT A. Enlargement of the tongue and epiglottis B. Narrowing of the glottic opening C. Nasal turbinate enlargement D. Continuous positive airway pressure (CPAP) should be used postoperatively because obstructive sleep apnea (OSA) is common
515. (D) Enlargement of the tongue and epiglottis predisposes the patient to upper airway obstruction and makes visualization of the vocal cords more difficult. The vocal cords are enlarged, making the glottic opening narrower. In addition, subglottic narrowing may be present as well as tracheal compression from an enlarged thyroid (seen in about 25% of acromegalic patients). This often necessitates the use of a narrower endotracheal tube than one might choose based on the facial enlargement. The placement of nasal airways may be more difficult due to the enlarged nasal turbinates. The use of CPAP is contraindicated after transsphenoidal hypophysectomy (Barash: Clinical Anesthesia, ed 7, p 1351; Hines: Stoelting’s Anesthesia and Co-Existing Disease, ed 6, p 404).
516. Evidence of an anaphylactic reaction to atracurium 1 to 2 hours after the episode could be best established by measuring blood levels of A. Tryptase B. Laudanosine C. Histamine D. Bradykinin
516. (A) There are four types of immune-mediated allergic reactions. Anaphylaxis is a type I IgE-mediated reaction that involves mast cells and basophils. Anaphylactoid reactions appear like anaphylaxis but are not immune mediated. Tryptase is a neutral protease normally stored in mast cells but is released into 148 Part 2 Clinical Sciences systemic circulation during anaphylactic but not anaphylactoid reactions. Tryptase levels would need to be measured within 1 to 2 hours of the suspected allergic reaction. Plasma histamine levels return to baseline within 30 to 60 minutes of an anaphylactic reaction. Laudanosine is a normal metabolic product of atracurium metabolism (Hines: Stoelting’s Anesthesia and Co-Existing Disease, ed 6, pp 523–524; Butterworth: Morgan & Mikhail’s Clinical Anesthesiology, ed 5, pp 1217–1221).
517. Which of the following findings is NOT consistent with a diagnosis of malignant hyperthermia? A. Paco2 150 mm Hg B. MVo2 50 mm Hg C. pH 6.9 D. Onset of symptoms an hour after end of operation
517. (B) Signs of MH reflect the hypermetabolic state (up to 10 times normal) that develops. Clinical signs include tachycardia, tachypnea, arterial hypoxemia, hypercarbia (e.g., Paco2 100-200 mm Hg), metabolic, and respiratory acidosis (e.g., pH 6.80-7.15), hyperkalemia, hypotension, muscle rigidity, trismus after succinylcholine administration, and increased body temperature. Mixed venous oxygen tension would be very low. The clinical presentations are quite variable, and some reactions may not develop until the postoperative period (Hines: Stoelting’s Anesthesia and Co-Existing Disease, ed 6, pp 635–640).
518. A 52-year-old business executive undergoes a radical retropubic prostatectomy uneventfully under general isoflurane anesthesia. He takes fluoxetine (Prozac) for depression. Upon discharge, which of the following analgesics would be the best choice for postoperative pain management in this patient? A. Oxycodone plus aspirin (Percodan) B. Hydrocodone with acetaminophen (Vicodin) C. Codeine with acetaminophen (Tylenol No. 3) D. Hydromorphone (Dilaudid) 128 Part 2 Clinical Sciences
518. (D) The SSRI fluoxetine is one of the most potent inhibitors of the cytochrome P-450 enzymes CYP3A4 and CYP2D6. CYP2D6 facilitates the conversion of codeine to morphine, meaning the response from a “normal” dose would be less than expected because of decreased conversion. Oxycodone and hydrocodone are metabolized by CYP2D6 to their active form as well, and a “normal” dose of these would give less response than expected. Thus, codeine, oxycodone, and hydrocodone would be poor analgesic choices for patients taking SSRIs. CYP3A4 is responsible for the metabolism of fentanyl, sufentanil, and alfentanil. Remifentanil is metabolized by nonspecific plasma esterases (Miller: Basics of Anesthesia, ed 6, p 37).
519. Anesthesia is induced in a 50-year-old, 125-kg man for anterior cervical fusion. The patient is placed on a ventilator. Peak airway pressure is noted to be 20 cm H2O with O2 saturation 99% on pulse oximeter. An hour later, the peak airway pressure rises to 40 cm H2O and Paco2 is 38 mm Hg on infrared spectrometer and on O2 saturation falls to 88%. Blood pressure and heart rate are unchanged. The MOST likely cause of these findings is A. Mainstem intubation B. Thrombotic pulmonary embolism C. Tension pneumothorax D. Venous air embolism
519. (A) Symptoms of a mainstem or bronchial intubation include asymmetric chest expansion, unilateral breath sounds, elevation of peak airway pressures, and ABG abnormalities (e.g., hypoxemia). Frequently, bronchial intubation is intentional (e.g., thoracic surgery with double-lumen endotracheal tubes), but, if undetected with a single-lumen tube, atelectasis, hypoxia, and pulmonary edema may result in time. Peak airway pressures can also increase with many conditions such as airway obstruction (e.g., kinked endotracheal tube, secretions, overinflated cuffs), bronchospasm, increasing Vt, increase in chest wall muscle tone (rigid chest with narcotics, coughing), and tension pneumothorax. If a tension pneumothorax develops, associated hypotension usually is present. Pulmonary embolism would not cause the peak airway pressure to rise as in this case (Lobato: Complications in Anesthesiology, pp 101–102).
520. The phase of liver transplantation where the greatest degree of hemodynamic instability is expected is A. Induction B. Dissection phase C. Anhepatic phase D. Reperfusion phase
520. (D) Although hemodynamic instability can occur at any time during liver transplantation, it is during the initial part of the reperfusion phase, when the vascular clamps are removed from the liver graft, when cardiovascular instability is most marked. At this time there can be profound hypotension, reduced cardiac contractility, cardiac arrhythmias, and hyperkalemic cardiac arrest. Epinephrine, atropine, calcium, and sodium bicarbonate should be available, as well as blood products, during this critical part of the surgery (Miller: Miller’s Anesthesia, ed 8, pp 2281–2282; Miller: Basics of Anesthesia, ed 6, p 584).
521. Which of the following drugs is (are) likely to prolong nondepolarizing neuromuscular blockade? A. Prednisone B. Diltiazem C. Clindamycin D. All of the above
521. (D) Metabolic and physiologic conditions as well as certain medications can contribute to a prolonged duration of action of nondepolarizing neuromuscular blockade. Metabolic and physiologic conditions include respiratory acidosis, myasthenia syndromes, hepatic/renal failure, hypocalcemia, hypothermia, and hypermagnesemia. Both inhaled and local anesthetics as well as corticosteroids, many antibiotics (e.g., polymyxins, aminoglycosides, lincosamines [e.g., clindamycin], metronidazole [Flagyl]), calcium channel blockers, dantrolene, and furosemide can prolong nondepolarizing neuromuscular blockade (Miller: Basics of Anesthesia, ed 6, pp 633–634).
522. Which of the factors in adults listed below is the strongest independent predictor of postoperative nausea and vomiting (PONV) in most studies? A. Female gender B. History of PONV C. History of migraines D. History of cigarette smoking
522. (A) PONV is the second most common complaint from patients after surgery (postoperative pain is the number one complaint). Of the many independent predictors of PONV in adult prospective studies, female gender is the strongest predictor for PONV and the need for postoperative antiemetic rescue treatments. It is interesting to note that although patients often experience nausea when smoking their first cigarettes, smokers have a lower incidence of PONV compared to nonsmokers. Other predictors of PONV include nonsmokers, previous history of PONV, history of migraine headaches, use of postoperative narcotics, lengthy surgical procedures, use of nitrous oxide, and the use of volatile anesthetics (Miller: Miller’s Anesthesia, ed 8, pp 2947–2954).
523. Near the end of a 3-hour colectomy, the surgeon complains that the patient is not relaxed. Two twitch monitors placed at different locations show only one twitch of a train-of-four. Blood gases are reported to be pH 6.9, CO2 82, K 4.6. The most appropriate action would be A. Administer more vecuronium B. Administer bicarbonate C. Increase minute ventilation D. Administer dantrolene
523. (D) Rare muscle diseases can have dramatic anesthetic implications. MH is among the most important manifestations of a muscular disorder. MH is thought to be caused by alterations in calcium control in General Anesthesia 149 muscle sarcoplasmic reticulum in response to succinylcholine or potent volatile anesthetics (most likely mediated by mutations of the ryanodine receptor). Because MH is a disorder in muscle metabolism, rigidity during administration of a volatile anesthetic or after succinylcholine use may be the presenting sign. Additionally, administration of any muscle relaxant would not provide muscle relaxation, and succinylcholine would be contraindicated. The patient does have a respiratory and metabolic acidosis and significantly increasing minute ventilation with 100% oxygen, and the use of sodium bicarbonate would be needed; however, stopping the triggering agent and administration of dantrolene is most important (Hines: Stoelting’s Anesthesia and Co-Existing Disease, ed 6, pp 635–640).
524. A 22-year-old parturient is anesthetized for an emergency laparoscopic cholecystectomy. She is in the twenty-fourth week of gestation and receives general sevoflurane anesthesia and has received rocuronium for muscle relaxation. Just before emergence, muscle relaxation is reversed with glycopyrrolate and neostigmine. Three minutes later, the fetal heart rate falls to 88 beats/min. The most likely cause of this is A. Fetal head compression B. Uteroplacental insufficiency C. Fetal hypoxia D. Reversal agents
524. (D) Atropine and scopolamine cross the placenta easily, whereas glycopyrrolate is poorly transferred across the placenta. Although neostigmine crosses the placenta poorly, enough does cross the placenta and can cause fetal bradycardia in utero. That is why it is better to reverse muscle relaxants in pregnant patients for nondelivery surgery with neostigmine and atropine (Butterworth: Morgan & Mikhail’s Clinical Anesthesiology, ed 5, p 229).
525. A 43-year-old woman with end-stage liver disease is admitted to the ICU. Which therapy is LEAST likely to improve symptoms associated with hepatic encephalopathy (HE)? A. Amino acid–rich total parenteral nutrition (TPN) B. Neomycin C. Lactulose D. Flumazenil
525. (A) With liver failure, the liver cannot adequately detoxify noxious chemicals. Fifty to seventy percent of patients with end-stage liver disease develop HE. Symptoms vary from mild confusion, drowsiness, and stupor to coma. The etiology of HE is complex. Because an elevation in blood ammonia levels (easily measured) is strongly associated with HE, treatment is aimed at lowering the ammonia level. Other toxins also contribute to HE. To lower the ammonia level, lactulose (which decreases the absorption of ammonia) and neomycin (which reduces the production of ammonia by reducing the ammonia-producing intestinal flora) are commonly administered. Protein restriction is commonly done to decrease ammonia production, so amino acid–rich TPN is not helpful. Flumazenil (a GABA receptor antagonist) has been shown to produce short-duration reversal of the symptoms of HE in some patients and thus suggests that GABA receptors are somehow activated during HE. GABA receptors are responsible for inhibitory neurotransmission in the CNS (Hines: Stoelting’s Anesthesia and Co-Existing Disease, ed 6, p 280; Miller: Basics of Anesthesia, ed 6, p 457; Miller: Miller’s Anesthesia, ed 8, p 541).
526. Ketorolac is contraindicated in patients undergoing scoliosis surgery because of A. Renal effects B. Risk of postoperative hemorrhage C. Effects on bone healing D. Effects on pulmonary function
526. (C) Ketorolac is one of the few nonsteroidal anti-inflammatory drugs (NSAIDs) approved for parenteral use. Although NSAIDs have analgesic and anti-inflammatory effects without ventilatory depression, they also inhibit platelet aggregation, can produce gastric ulceration, are associated with renal dysfunction, and may impair bone healing. NSAIDs are contraindicated in patients undergoing spinal fusion, where bone healing is essential to a successful surgical procedure (Miller: Miller’s Anesthesia, ed 8, p 2982).
527. Causes of sickling in patients with sickle cell anemia include all of the following EXCEPT A. Inhaled nitric oxide B. Dehydration C. Metabolic acidosis D. Hypothermia
527. (A) Sickle cell anemia is an inherited disease that affects approximately 0.3% to 1% of the black population in the United States. Affected patients are homozygous for hemoglobin S such that 70% to 98% of the hemoglobin found in their RBCs is of the unstable S type, resulting in severe hemolytic anemia. Factors that favor the formation of sickle cells include arterial hypoxemia, acidosis, dehydration, and reductions in body temperature. Inhaled nitric oxide and other new investigational drugs may help reduce the sickling process and may even unsickle cells (Butterworth: Morgan & Mikhail’s Clinical Anesthesiology, ed 5, pp 1177–1180; Hines: Stoeling’s Anesthesia and Co-Existing Disease, ed 6, pp 411–412).
528. Which of the following factors is the greatest predictor of sleep apneas in an adult? A. Neck circumference B. Micrognathia C. Weight D. Body mass index (BMI)
528. (A) Although many books suggest that obesity is the most common cause of OSA, more recent data suggest that a large neck circumference (>44 cm) reflects pharyngeal fat deposition and is more strongly correlated with OSA than obesity (BMI >30). Other risk factors include male gender, middle age, evening alcohol consumption, or sleep-inducing medications (Hines: Stoelting’s Anesthesia and Co-Existing Disease, ed 6, p 320; Miller: Miller’s Anesthesia, ed 8, pp 2203–2204; Miller: Basics of Anesthesia, ed 6, pp 435–436).
529. The greatest number of malpractice claims made against anesthesiologists (according to the American Society of Anesthesiologists [ASA] closed claims task force) is associated with which adverse outcome? A. Eye injury B. Brain damage C. Nerve damage D. Death
529. (D) The ASA closed claims task force lists the leading causes of malpractice claims against anesthesiologists in the 1990s to be death (22%), followed by nerve damage (21%) and brain damage (10%) (Barash: Clinical Anesthesia, ed 7, pp 100–101).
530. Resynchronization therapy A. Is indicated for short QRS complexes B. Is contraindicated in patients with coronary artery disease C. Requires pacemaker implantation D. Is usually accomplished with biphasic defibrillator
530. (C) Cardiac resynchronization therapy (CRT) is used in patients with heart failure (EF <35%) and ventricular conductive delay (prolonged QRS complex usually is 120 to 150 msec). The conduction delay 150 Part 2 Clinical Sciences creates a mechanical dyssynchrony and worsens the heart failure. CRT requires biventricular pacing with one lead in the coronary sinus to activate the left ventricle. CRT has nothing to do with breathing. Although CRT has nothing to do with an implantable cardioverter-defibrillator (ICD), many patients may require both because typically a patient with poor left ventricle function is also at risk for sudden death. Most of these patients also have underlying CAD (Hines: Stoelting’s Anesthesia and Co-Existing Disease, ed 6, p 129; Miller: Miller’s Anesthesia, ed 8, pp 2078–2079).
531. The underlying feature in patients with syndrome X is A. Hypertension B. Morbid obesity C. Hypoglycemia D. Insulin resistance General Anesthesia 129
531. (D) Patients with syndrome X (also called metabolic syndrome X) have insulin resistance that leads to elevated levels of insulin and the metabolic changes that occur with elevated insulin levels, except that hypoglycemia does not develop. Associated with it are low levels of high-density lipoproteins, hypertension, and increased plasminogen activator inhibitor-1 levels, which are associated with CAD. Many of these patients are obese (Miller: Miller’s Anesthesia, ed 8, pp 2201–2203).
532. A 65-year-old hospitalized patient is being treated for pain from pancreatic cancer and is well controlled on 30 mg IV morphine per day. What is the equivalent total oral daily dosage of morphine in this patient for discharge planning? A. 10 mg B. 30 mg C. 90 mg D. 120 mg
532. (C) The parenteral-to-oral conversion for morphine sulfate is 1:3; thus, 30 mg morphine parenterally would be similar to 30 mg × 3 = 90 mg of morphine orally. The parenteral-to-oral conversion for methadone is 1:2 (Brunton: Goodman & Gilman’s The Pharmacological Basis of Therapeutics, ed 12, p 498).
533. A 64-year-old patient is brought to the postanesthesia care unit after a 7-hour cosmetic surgery operation under 1.7% sevoflurane anesthesia for the entire case. Which of the following describes the sevoflurane concentration in the vessel-rich group (VRG), the muscle group (MG), and the fat or vessel-poor group (VPG) immediately after the vaporizer is turned off? A. VRG: falling, MG: falling, VPG: rising B. VRG: falling, MG: rising, VPG: rising C. VRG: rising, MG: falling, VPG: falling D. All three compartments (VRG, MG, and VPG) falling
533. (A) The VRG comprises only 10% of the body but receives 75% of the cardiac output. Equilibrium with alveolar partial pressure is rapid (8 to 10 minutes [4 time constants]). After that point, uptake is accounted for by the MG and this equilibrium would be approached in a time frame on the order of 2 to 4 hours. The last compartment to reach equilibrium is the VPG, which includes fat. This equilibrium requires many hours, even days, to be achieved. VPG VRG MG Vaporizer shut off When the vaporizer is turned off, the alveolar (arterial) partial pressure falls rapidly. The partial pressure in the VRG would also fall, as would the MG. The fat continues to take up volatile anesthetic for hours and actually contributes to recovery. The partial pressure of gas in the VPG at the time the vaporizer is turned off would be lower than the partial pressure in the VRG and MG and thus would initially take up some anesthetic from the higher pressure VRG and MG (Miller: Miller’s Anesthesia, ed 8, pp 639, 654–655).
534. Hazards of O2 administration include A. Retinopathy of prematurity B. Bronchopulmonary dysplasia C. Adsorption atelectasis D. All of the above
534. (D) Retinopathy of prematurity (retrolental fibroplasia) is a hazard associated with O2 administration to neonates up to 44 weeks (gestational age + life age). It is especially a hazard in the extremely premature (birth weight <1000 g and gestational age <28 weeks). Bronchopulmonary dysplasia is a chronic lung disorder that afflicts infants who required mechanical ventilation at birth to treat respiratory distress syndrome. CO2 retention is a hazard in patients with chronic obstructive lung disease. Adsorption atelectasis is a potential hazard of oxygen administration in any patient receiving oxygen concentrations greater than 50%. It results from rapid uptake of oxygen into the circulation greater than the delivery General Anesthesia 151 of oxygen by ventilation. Normally, the presence of nitrogen serves as an internal splint, protecting the alveoli from collapse. Prolonged high concentration of oxygen can damage “normal lungs” if given for prolonged periods of time and may lead from mild irritation to tracheobronchitis to pulmonary interstitial edema to pulmonary fibrosis (Miller: Miller’s Anesthesia, ed 8, pp 457–460, 2670; Butterworth: Morgan & Mikhail’s Clinical Anesthesiology, ed 5, pp 1287–1288).
535. Which of the following nerves is NOT derived from a cranial nerve? A. Great auricular B. Infraorbital C. Supratrochlear D. Supraorbital
535. (A) All of the nerves listed in this question are derived from the fifth cranial nerve (trigeminal nerve) except the great auricular nerve. The ophthalmic nerve (V1 branch of trigeminal nerve) gives rise to the supratrochlear, infratrochlear, and supraorbital nerves. The infraorbital nerve is a branch of V2 (maxillary branch of the trigeminal nerve). The mental nerve is a branch of V3 (mandibular nerve). The great auricular nerve arises from branches of C2 and C3 spinal nerves and innervates the skin of the outer ear, the mastoid process, and the parotid gland (Miller: Miller’s Anesthesia, ed 8, pp 1722–1724).
536. A 45-year-old woman is experiencing progressive mental deterioration over a 6-hour period, 5 days after emergency evacuation of a large subarachnoid hemorrhage and clipping of a middle cerebral artery aneurysm. The MOST likely cause for deterioration is A. Cerebral edema B. Improper placement of the aneurysm clip C. Recurrent cerebral hemorrhage D. Vasospasm
536. (D) Cerebral vasospasm is often associated in patients who have suffered a subarachnoid bleed. Angiographic evidence of vasospasm can be noted in up to 70% of patients; however, clinical vasospasm with detectable ischemia (e.g., mental confusion, lethargy, focal motor, and speech impairments) is detected in about 30% of patients. When clinical vasospasm develops, it usually occurs between 4 and 12 days after the bleed. Although it may resolve spontaneously, it may also progress to coma and death within a few hours or days. Rebleeding tends to occur earlier (i.e., within 24 hours) (Barash: Clinical Anesthesia, ed 6, pp 1585–1586).
537. The period of vulnerability after three courses of bleomycin for testicular cancer is A. 1 month B. 1 year C. Lifelong D. No vulnerability with just three courses
537. (C) Bleomycin is used primarily in the treatment of Hodgkin lymphoma and testicular tumors. Bleomycin causes oxidative damage to nucleotides, which leads to breaks in DNA. Although the more common side effects of bleomycin use are mucocutaneous, it is the dose-related pulmonary toxicity that is the most serious side effect. Early signs and symptoms of pulmonary toxicity include dry cough, fine rales, and diffuse infiltrates on radiograph. Approximately 5% to 10% of patients will develop pulmonary toxicity, and about 1% will die from this complication. Most believe that the risk of pulmonary toxicity increases with dose (especially total dose >250 mg), patients older than 40 years of age, patients with a creatinine clearance (CrCl) of <80 mL/min, and in patients with prior chest radiation or preexisting pulmonary disease. Although a relationship appears to exist between the use of bleomycin and the use of high concentrations of oxygen, the details are unclear. Currently, it has been suggested to use the lowest concentration of oxygen consistent with patient safety, with a careful evaluation of oxygen saturation with pulse oximetry in any patient who has received bleomycin (Brunton: Goodman & Gilman’s The Pharmacological Basis of Therapeutics, ed 12, pp 1716–1718; Miller: Miller’s Anesthesia, ed 8, p 1943; Stoelting: Pharmacology and Physiology in Anesthetic Practice, ed 4, pp 555–565).
538. The most common adverse cardiac event in the pediatric population is A. Hypotension B. Bradycardia C. Tachycardia D. Bigeminy
538. (B) The most common adverse cardiac event in the pediatric population is bradycardia. An outcome study from the Medical College of Virginia examined the incidence of bradycardia in nearly 8000 children younger than 4 years old. The most common causes of bradycardia were cardiac disease or surgery and inhalation anesthesia, followed by hypoxemia. Of those children who had bradycardia, hypotension occurred in 30%, asystole or ventricular fibrillation in 10%, and death in 8%. Tachycardia, which is common, is not an adverse event (Davis: Smith’s Anesthesia for Infants and Children, ed 8, pp 1232–1236; Barash: Clinical Anesthesia, ed 7, p 1245; Butterworth: Morgan & Mikhail’s Clinical Anesthesiology, ed 5, p 879).
539. Each of the following is a predictor of difficulty with mask ventilation EXCEPT A. Presence of beard B. BMI greater than 26 C. Presence of teeth D. Age greater than 55
539. (C) Mask ventilation, one of the most basic anesthesia techniques, can be challenging in some patients. Use of mask ventilation in patients who are prone to airway obstruction can be more difficult because of extra airway tissue (i.e., obese patients with a BMI >26), patients without teeth (i.e., tongue is closer to the roof of the mouth, and face conformity may not fit the mask well), and patients who snore (i.e., already have reason for airway obstruction). Mask ventilation can also be more difficult in patients who have a beard (i.e., harder to get a good mask seal), patients whose age is older than 55 years, patients with facial tumors, and patients with facial trauma. Use of an oral airway may be needed in many of these patients (Miller: Basics of Anesthesia, ed 6, p 227; Miller: Miller’s Anesthesia, ed 8, p 1651).
540. In a patient with compartment syndrome, which of the following signs would be the last to appear? A. Pulselessness B. Pain C. Paresthesia D. Paralysis
540. (A) Whenever perfusion to an extremity is inadequate (e.g., trauma or poor perfusion), hypoxic edema develops, producing swelling. When this occurs in a compartment, tissue pressures rise, decreasing capillary perfusion. Symptoms of compartment syndrome include extreme pain unrelieved by analgesics, paresthesias, paralysis, and pallor. Extensive rhabdomyolysis may develop as well as permanent nerve 152 Part 2 Clinical Sciences and muscle injury in the compartment. Because the problem is at the tissue level, pulses and capillary refill may still be present. Treatment includes fasciotomy to relieve the elevated pressure (Barash: Clinical Anesthesia, ed 7, p 1514; Miller: Miller’s Anesthesia, ed 8, p 2450).
541. Select the TRUE statement regarding the dose per kilogram of body weight and duration, respectively, of local anesthetics for spinals in infants compared with adults. A. Greater dose and longer duration B. Greater dose and shorter duration C. Greater dose and duration is the same D. Smaller dose and longer duration
541. (B) The amount and distribution of cerebrospinal fluid (CSF) is different in neonates compared with adults. The neonate has about 4 mL/kg of CSF compared to the adult’s 2 mL/kg. In addition, almost half of the neonate’s CSF is in the spinal subarachnoid space, compared with about a quarter of the adult’s CSF in the spinal subarachnoid space. These factors help explain why the dose is greater in neonates and infants and of shorter duration compared to adults (Miller: Miller’s Anesthesia, ed 8, pp 2727–2728).
542. A number 6 endotracheal tube indicates which size? A. 6-mm internal diameter (ID) B. 6-mm external diameter C. 6-mm external circumference D. 6-mm internal circumference
542. (A) Endotracheal tube sizes are measured according to the ID. They are available in 0.5-mm ID increments (Miller: Basics of Anesthesia, ed 6, p 230).
543. If a patient were to become trapped in the magnetic resonance imaging (MRI) scanner by a metal object and the engineers decided to quench the magnet, the greatest hazard to the patient would be A. Heat B. Cold C. Fire D. Noise
543. (B) MRI scanners have superconducting electrical currents that produce large magnetic fields (up to 6 m) and are always “on”. The presence of any ferromagnetic objects in the room may cause a missile-type injury when the objects are strongly attracted to the scanner. If a patient is pinned into the scanner by a magnetic object that flew into the scanner, the MRI technicians may have to turn off the superconducting magnet. During magnetic shutdown (quench) the scanner will become extremely cold (Miller: Basics of Anesthesia, ed 6, p 621).
544. A 25-year-old black man is brought to the emergency room unconscious. Supplemental oxygen is administered, and a pulse oximeter is placed on his finger and a reading of 98% is recorded. Arterial gas sampling at the same time shows Pao2 of 190 mm Hg, pH 7.2, and O2 saturation of 90%. Presence of which of the following could explain the discrepancies between these two readings? A. Methemoglobin (Hb Met) B. Sickle cell hemoglobin C. Carboxyhemoglobin (HbCO) D. Hemoglobin shifted to right 130 Part 2 Clinical Sciences
544. (C) Carbon monoxide is a colorless, odorless gas that binds to hemoglobin with an affinity more than 200 times stronger than oxygen. Inhalation of CO is a major cause of morbidity and mortality in the United States. A dual-wave (660 nm and 940 nm) pulse oximeter is incapable of distinguishing CO hemoglobin from oxyhemoglobin, but the distinction is easily made in the clinical laboratory with a co-oximeter. Significant quantities of methemoglobin would result in a saturation of 85% of the pulse oximeter. The slight right shift from a mild acidemia would be insufficient to account for 90% saturation in the face of a Pao2 of 190. Furthermore, the pulse oximeter reading would be nearly the same as the co-oximeter value (Miller: Miller’s Anesthesia, ed 8, pp 2679–2680; Hines: Stoelting’s Anesthesia and Co-Existing Disease, ed 6, pp 554–555).
545. During surgery for correction of scoliosis, somatosensory evoked potential (SSEP) monitoring is employed. An increase in SSEP latency and a decrease in amplitude could be explained by each of the following EXCEPT A. Anterior spinal artery syndrome B. Propofol infusion (200 μg/kg/min) C. Hypotension D. 2 MAC isoflurane anesthesia
545. (A) The pathway for SSEP monitoring of the lower extremity starts with a stimulus of the posterior tibial nerve, which generates an impulse that passes through the dorsal root ganglion into the dorsal (posterior) columns and then to the dorsal column nuclei. Second-order nerves carry the impulse across the midline to the thalamus, and the impulse travels over third-order nerves to the sensory cortex of the brain. Electrodes in the scalp record the electrical activity in the brain. Severe hypotension or ischemia in any portion of the pathway along which the induced signal is conducted can result in a reduced evoked potential amplitude or increased latency. Volatile anesthetic administration in MAC values greater than 0.5 to 0.75 can produce a similar effect. Barbiturates, benzodiazepines, propofol, and other sedative drugs can likewise interfere with SSEP monitoring. Anterior spinal artery syndrome affects the anterior (motor) portion of the spinal cord and does not interfere with SSEP monitoring (Miller: Basics of Anesthesia, ed 6, pp 327–328).
546. In which of the following conditions would the response to atropine be MOST pronounced? A. Diabetic autonomic neuropathy B. Brain death C. Status post heart transplant D. High (C8) spinal anesthesia
546. (D) Diabetic autonomic neuropathy can affect the autonomic nervous system to such an extent that atropine and propranolol would have little effect (because there would be nothing to block). After heart transplantation, the new heart (donor heart) is denervated and will not respond to autonomic nervous system blocking drugs. Brain death by definition is associated with absence of autonomic function. A high spinal would be associated with total sympathectomy, and propranolol would have no effect on heart rate, but the vagus nerve would be unaffected. Atropine would have no effect on a patient with atrial fibrillation and complete heart block (Hines: Stoelting’s Anesthesia and Co-Existing Disease, ed 6, pp 26–28, 383; Miller: Basics of Anesthesia, ed 6, pp 281, 585–586).
547. Skin lesions all appear at the same stage and at the same time A. Smallpox B. Anthrax C. Plague D. Botulism E. Ebola virus
547. (A)There are three categories of biological weapons: A, B, and C. All of the diseases in this question are in the highly contagious Category A agents. Smallpox is caused by a virus (Variola major) and in 1980 was declared extinct by the World Health Organization. The incubation period was 7 to 14 days, and patients with the disease presented with malaise, headache, and fever. Two to 4 days later a characteristic rash develops where all lesions are at the same stage (papules, vesicles, pustules, and scabs). Exposed patients and health care workers who received a vaccination within 4 days of exposure had greatly attenuated symptoms. Unvaccinated patients who were untreated had a mortality rate of greater than 30%. Patients who previously had been vaccinated had a lower mortality rate. Treatment includes the drug cidofovir. Anthrax is caused by an aerobic gram-positive spore-forming bacillus (Bacillus anthracis) and has three primary forms: cutaneous, gastrointestinal, and inhalational. Weaponized anthrax is mainly an inhalational disease. Inhalational anthrax symptoms occur within 1 to 7 days of exposure and initially look like viral flu (fever, chills, myalgia, and a nonproductive cough). Later on, the patient’s mediastinal lymph nodes, where the spores germinate, enlarge, producing a widened mediastinum that can be seen on a chest x-ray film. Treatment is primarily with ciprofloxacin; prophylaxis to exposed personnel includes 60 days of ciprofloxacin. Mortality rate for inhaled anthrax is greater than 80%. Plague is caused by a gram-negative coccobacillus (Yersinia pestis) and has two forms: bubonic and pneumonic. With the more common bubonic plague, there is painful swelling of the lymph nodes (buboes), which can grow to 5 to 10 cm in diameter. The patients develop cyanosis, shock, and gangrene in peripheral tissues (black death). If the lungs become infected then pneumonic plague develops, which, if untreated, has 100% mortality. Treatment is primarily with streptomycin, although gentamicin, tetracycline, and chloramphenicol have been used. Botulism is caused by the toxin from Clostridium botulinum. Because this disease is due to a neurotoxin, it is not contagious. The neurotoxin affects cholinergic neurons and prevents the release of acetylcholine. Symptoms typically develop within 12 to 36 hours of exposure and include acute flaccid paralysis, decreased salivation, ileus, and urinary retention. There are no sensory deficits. With appropriate supportive care and trivalent equine antitoxin, the mortality rate is less than 5%. Without the use of antitoxin, patients may take 2 to 8 weeks to recover. Mortality rate is 5% to 10%. There are more than 18 hemorrhagic fever viruses, including Ebola virus. The incubation period is 2 to 21 days, and patients present with fever, myalgias, headaches, thrombocytopenia, and hemorrhagic complications (petechiae, ecchymosis). Untreated, the mortality rate for Ebola virus is 90%. Treatment includes the drug ribavirin (Barash: Clinical Anesthesia, ed 7, pp 1543–1545; Miller: Miller’s Anesthesia, ed 8, pp 2501–2502; Miller: Basics of Anesthesia, ed 6, pp 691–695).
548. Ciprofloxacin for 60 days is prophylaxis for exposed patients A. Smallpox B. Anthrax C. Plague D. Botulism E. Ebola virus
548. (B)There are three categories of biological weapons: A, B, and C. All of the diseases in this question are in the highly contagious Category A agents. Smallpox is caused by a virus (Variola major) and in 1980 was declared extinct by the World Health Organization. The incubation period was 7 to 14 days, and patients with the disease presented with malaise, headache, and fever. Two to 4 days later a characteristic rash develops where all lesions are at the same stage (papules, vesicles, pustules, and scabs). Exposed patients and health care workers who received a vaccination within 4 days of exposure had greatly attenuated symptoms. Unvaccinated patients who were untreated had a mortality rate of greater than 30%. Patients who previously had been vaccinated had a lower mortality rate. Treatment includes the drug cidofovir. Anthrax is caused by an aerobic gram-positive spore-forming bacillus (Bacillus anthracis) and has three primary forms: cutaneous, gastrointestinal, and inhalational. Weaponized anthrax is mainly an inhalational disease. Inhalational anthrax symptoms occur within 1 to 7 days of exposure and initially look like viral flu (fever, chills, myalgia, and a nonproductive cough). Later on, the patient’s mediastinal lymph nodes, where the spores germinate, enlarge, producing a widened mediastinum that can be seen on a chest x-ray film. Treatment is primarily with ciprofloxacin; prophylaxis to exposed personnel includes 60 days of ciprofloxacin. Mortality rate for inhaled anthrax is greater than 80%. Plague is caused by a gram-negative coccobacillus (Yersinia pestis) and has two forms: bubonic and pneumonic. With the more common bubonic plague, there is painful swelling of the lymph nodes (buboes), which can grow to 5 to 10 cm in diameter. The patients develop cyanosis, shock, and gangrene in peripheral tissues (black death). If the lungs become infected then pneumonic plague develops, which, if untreated, has 100% mortality. Treatment is primarily with streptomycin, although gentamicin, tetracycline, and chloramphenicol have been used. Botulism is caused by the toxin from Clostridium botulinum. Because this disease is due to a neurotoxin, it is not contagious. The neurotoxin affects cholinergic neurons and prevents the release of acetylcholine. Symptoms typically develop within 12 to 36 hours of exposure and include acute flaccid paralysis, decreased salivation, ileus, and urinary retention. There are no sensory deficits. With appropriate supportive care and trivalent equine antitoxin, the mortality rate is less than 5%. Without the use of antitoxin, patients may take 2 to 8 weeks to recover. Mortality rate is 5% to 10%. There are more than 18 hemorrhagic fever viruses, including Ebola virus. The incubation period is 2 to 21 days, and patients present with fever, myalgias, headaches, thrombocytopenia, and hemorrhagic complications (petechiae, ecchymosis). Untreated, the mortality rate for Ebola virus is 90%. Treatment includes the drug ribavirin (Barash: Clinical Anesthesia, ed 7, pp 1543–1545; Miller: Miller’s Anesthesia, ed 8, pp 2501–2502; Miller: Basics of Anesthesia, ed 6, pp 691–695).
549. Not contagious A. Smallpox B. Anthrax C. Plague D. Botulism E. Ebola virus
549. (D)There are three categories of biological weapons: A, B, and C. All of the diseases in this question are in the highly contagious Category A agents. Smallpox is caused by a virus (Variola major) and in 1980 was declared extinct by the World Health Organization. The incubation period was 7 to 14 days, and patients with the disease presented with malaise, headache, and fever. Two to 4 days later a characteristic rash develops where all lesions are at the same stage (papules, vesicles, pustules, and scabs). Exposed patients and health care workers who received a vaccination within 4 days of exposure had greatly attenuated symptoms. Unvaccinated patients who were untreated had a mortality rate of greater than 30%. Patients who previously had been vaccinated had a lower mortality rate. Treatment includes the drug cidofovir. Anthrax is caused by an aerobic gram-positive spore-forming bacillus (Bacillus anthracis) and has three primary forms: cutaneous, gastrointestinal, and inhalational. Weaponized anthrax is mainly an inhalational disease. Inhalational anthrax symptoms occur within 1 to 7 days of exposure and initially look like viral flu (fever, chills, myalgia, and a nonproductive cough). Later on, the patient’s mediastinal lymph nodes, where the spores germinate, enlarge, producing a widened mediastinum that can be seen on a chest x-ray film. Treatment is primarily with ciprofloxacin; prophylaxis to exposed personnel includes 60 days of ciprofloxacin. Mortality rate for inhaled anthrax is greater than 80%. Plague is caused by a gram-negative coccobacillus (Yersinia pestis) and has two forms: bubonic and pneumonic. With the more common bubonic plague, there is painful swelling of the lymph nodes (buboes), which can grow to 5 to 10 cm in diameter. The patients develop cyanosis, shock, and gangrene in peripheral tissues (black death). If the lungs become infected then pneumonic plague develops, which, if untreated, has 100% mortality. Treatment is primarily with streptomycin, although gentamicin, tetracycline, and chloramphenicol have been used. Botulism is caused by the toxin from Clostridium botulinum. Because this disease is due to a neurotoxin, it is not contagious. The neurotoxin affects cholinergic neurons and prevents the release of acetylcholine. Symptoms typically develop within 12 to 36 hours of exposure and include acute flaccid paralysis, decreased salivation, ileus, and urinary retention. There are no sensory deficits. With appropriate supportive care and trivalent equine antitoxin, the mortality rate is less than 5%. Without the use of antitoxin, patients may take 2 to 8 weeks to recover. Mortality rate is 5% to 10%. There are more than 18 hemorrhagic fever viruses, including Ebola virus. The incubation period is 2 to 21 days, and patients present with fever, myalgias, headaches, thrombocytopenia, and hemorrhagic complications (petechiae, ecchymosis). Untreated, the mortality rate for Ebola virus is 90%. Treatment includes the drug ribavirin (Barash: Clinical Anesthesia, ed 7, pp 1543–1545; Miller: Miller’s Anesthesia, ed 8, pp 2501–2502; Miller: Basics of Anesthesia, ed 6, pp 691–695).
550. Treatment may include streptomycin, gentamicin, or tetracycline A. Smallpox B. Anthrax C. Plague D. Botulism E. Ebola virus
550. (C) There are three categories of biological weapons: A, B, and C. All of the diseases in this question are in the highly contagious Category A agents. Smallpox is caused by a virus (Variola major) and in 1980 was declared extinct by the World Health Organization. The incubation period was 7 to 14 days, and patients with the disease presented with malaise, headache, and fever. Two to 4 days later a characteristic rash develops where all lesions are at the same stage (papules, vesicles, pustules, and scabs). Exposed patients and health care workers who received a vaccination within 4 days of exposure had greatly attenuated symptoms. Unvaccinated patients who were untreated had a mortality rate of greater than 30%. Patients who previously had been vaccinated had a lower mortality rate. Treatment includes the drug cidofovir. Anthrax is caused by an aerobic gram-positive spore-forming bacillus (Bacillus anthracis) and has three primary forms: cutaneous, gastrointestinal, and inhalational. Weaponized anthrax is mainly an inhalational disease. Inhalational anthrax symptoms occur within 1 to 7 days of exposure and initially look like viral flu (fever, chills, myalgia, and a nonproductive cough). Later on, the patient’s mediastinal lymph nodes, where the spores germinate, enlarge, producing a widened mediastinum that can be seen on a chest x-ray film. Treatment is primarily with ciprofloxacin; prophylaxis to exposed personnel includes 60 days of ciprofloxacin. Mortality rate for inhaled anthrax is greater than 80%. Plague is caused by a gram-negative coccobacillus (Yersinia pestis) and has two forms: bubonic and pneumonic. With the more common bubonic plague, there is painful swelling of the lymph nodes (buboes), which can grow to 5 to 10 cm in diameter. The patients develop cyanosis, shock, and gangrene in peripheral tissues (black death). If the lungs become infected then pneumonic plague develops, which, if untreated, has 100% mortality. Treatment is primarily with streptomycin, although gentamicin, tetracycline, and chloramphenicol have been used. Botulism is caused by the toxin from Clostridium botulinum. Because this disease is due to a neurotoxin, it is not contagious. The neurotoxin affects cholinergic neurons and prevents the release of acetylcholine. Symptoms typically develop within 12 to 36 hours of exposure and include acute flaccid paralysis, decreased salivation, ileus, and urinary retention. There are no sensory deficits. With appropriate supportive care and trivalent equine antitoxin, the mortality rate is less than 5%. Without the use of antitoxin, patients may take 2 to 8 weeks to recover. Mortality rate is 5% to 10%. There are more than 18 hemorrhagic fever viruses, including Ebola virus. The incubation period is 2 to 21 days, and patients present with fever, myalgias, headaches, thrombocytopenia, and hemorrhagic complications (petechiae, ecchymosis). Untreated, the mortality rate for Ebola virus is 90%. Treatment includes the drug ribavirin (Barash: Clinical Anesthesia, ed 7, pp 1543–1545; Miller: Miller’s Anesthesia, ed 8, pp 2501–2502; Miller: Basics of Anesthesia, ed 6, pp 691–695).
551. Treatment includes trivalent equine antitoxin A. Smallpox B. Anthrax C. Plague D. Botulism E. Ebola virus
551. (D)There are three categories of biological weapons: A, B, and C. All of the diseases in this question are in the highly contagious Category A agents. Smallpox is caused by a virus (Variola major) and in 1980 was declared extinct by the World Health Organization. The incubation period was 7 to 14 days, and patients with the disease presented with malaise, headache, and fever. Two to 4 days later a characteristic rash develops where all lesions are at the same stage (papules, vesicles, pustules, and scabs). Exposed patients and health care workers who received a vaccination within 4 days of exposure had greatly attenuated symptoms. Unvaccinated patients who were untreated had a mortality rate of greater than 30%. Patients who previously had been vaccinated had a lower mortality rate. Treatment includes the drug cidofovir. Anthrax is caused by an aerobic gram-positive spore-forming bacillus (Bacillus anthracis) and has three primary forms: cutaneous, gastrointestinal, and inhalational. Weaponized anthrax is mainly an inhalational disease. Inhalational anthrax symptoms occur within 1 to 7 days of exposure and initially look like viral flu (fever, chills, myalgia, and a nonproductive cough). Later on, the patient’s mediastinal lymph nodes, where the spores germinate, enlarge, producing a widened mediastinum that can be seen on a chest x-ray film. Treatment is primarily with ciprofloxacin; prophylaxis to exposed personnel includes 60 days of ciprofloxacin. Mortality rate for inhaled anthrax is greater than 80%. Plague is caused by a gram-negative coccobacillus (Yersinia pestis) and has two forms: bubonic and pneumonic. With the more common bubonic plague, there is painful swelling of the lymph nodes (buboes), which can grow to 5 to 10 cm in diameter. The patients develop cyanosis, shock, and gangrene in peripheral tissues (black death). If the lungs become infected then pneumonic plague develops, which, if untreated, has 100% mortality. Treatment is primarily with streptomycin, although gentamicin, tetracycline, and chloramphenicol have been used. Botulism is caused by the toxin from Clostridium botulinum. Because this disease is due to a neurotoxin, it is not contagious. The neurotoxin affects cholinergic neurons and prevents the release of acetylcholine. Symptoms typically develop within 12 to 36 hours of exposure and include acute flaccid paralysis, decreased salivation, ileus, and urinary retention. There are no sensory deficits. With appropriate supportive care and trivalent equine antitoxin, the mortality rate is less than 5%. Without the use of antitoxin, patients may take 2 to 8 weeks to recover. Mortality rate is 5% to 10%. There are more than 18 hemorrhagic fever viruses, including Ebola virus. The incubation period is 2 to 21 days, and patients present with fever, myalgias, headaches, thrombocytopenia, and hemorrhagic complications (petechiae, ecchymosis). Untreated, the mortality rate for Ebola virus is 90%. Treatment includes the drug ribavirin (Barash: Clinical Anesthesia, ed 7, pp 1543–1545; Miller: Miller’s Anesthesia, ed 8, pp 2501–2502; Miller: Basics of Anesthesia, ed 6, pp 691–695).
552. Three primary types: cutaneous, gastrointestinal, and inhalation A. Smallpox B. Anthrax C. Plague D. Botulism E. Ebola virus
552. (B)There are three categories of biological weapons: A, B, and C. All of the diseases in this question are in the highly contagious Category A agents. Smallpox is caused by a virus (Variola major) and in 1980 was declared extinct by the World Health Organization. The incubation period was 7 to 14 days, and patients with the disease presented with malaise, headache, and fever. Two to 4 days later a characteristic rash develops where all lesions are at the same stage (papules, vesicles, pustules, and scabs). Exposed patients and health care workers who received a vaccination within 4 days of exposure had greatly attenuated symptoms. Unvaccinated patients who were untreated had a mortality rate of greater than 30%. Patients who previously had been vaccinated had a lower mortality rate. Treatment includes the drug cidofovir. Anthrax is caused by an aerobic gram-positive spore-forming bacillus (Bacillus anthracis) and has three primary forms: cutaneous, gastrointestinal, and inhalational. Weaponized anthrax is mainly an inhalational disease. Inhalational anthrax symptoms occur within 1 to 7 days of exposure and initially look like viral flu (fever, chills, myalgia, and a nonproductive cough). Later on, the patient’s mediastinal lymph nodes, where the spores germinate, enlarge, producing a widened mediastinum that can be seen on a chest x-ray film. Treatment is primarily with ciprofloxacin; prophylaxis to exposed personnel includes 60 days of ciprofloxacin. Mortality rate for inhaled anthrax is greater than 80%. Plague is caused by a gram-negative coccobacillus (Yersinia pestis) and has two forms: bubonic and pneumonic. With the more common bubonic plague, there is painful swelling of the lymph nodes (buboes), which can grow to 5 to 10 cm in diameter. The patients develop cyanosis, shock, and gangrene in peripheral tissues (black death). If the lungs become infected then pneumonic plague develops, which, if untreated, has 100% mortality. Treatment is primarily with streptomycin, although gentamicin, tetracycline, and chloramphenicol have been used. Botulism is caused by the toxin from Clostridium botulinum. Because this disease is due to a neurotoxin, it is not contagious. The neurotoxin affects cholinergic neurons and prevents the release of acetylcholine. Symptoms typically develop within 12 to 36 hours of exposure and include acute flaccid paralysis, decreased salivation, ileus, and urinary retention. There are no sensory deficits. With appropriate supportive care and trivalent equine antitoxin, the mortality rate is less than 5%. Without the use of antitoxin, patients may take 2 to 8 weeks to recover. Mortality rate is 5% to 10%. There are more than 18 hemorrhagic fever viruses, including Ebola virus. The incubation period is 2 to 21 days, and patients present with fever, myalgias, headaches, thrombocytopenia, and hemorrhagic complications (petechiae, ecchymosis). Untreated, the mortality rate for Ebola virus is 90%. Treatment includes the drug ribavirin (Barash: Clinical Anesthesia, ed 7, pp 1543–1545; Miller: Miller’s Anesthesia, ed 8, pp 2501–2502; Miller: Basics of Anesthesia, ed 6, pp 691–695).
553. Vaccine may prevent or greatly attenuate symptoms if given within 4 days of exposure A. Smallpox B. Anthrax C. Plague D. Botulism E. Ebola virus
553. (A)There are three categories of biological weapons: A, B, and C. All of the diseases in this question are in the highly contagious Category A agents. Smallpox is caused by a virus (Variola major) and in 1980 was declared extinct by the World Health Organization. The incubation period was 7 to 14 days, and patients with the disease presented with malaise, headache, and fever. Two to 4 days later a characteristic rash develops where all lesions are at the same stage (papules, vesicles, pustules, and scabs). Exposed patients and health care workers who received a vaccination within 4 days of exposure had greatly attenuated symptoms. Unvaccinated patients who were untreated had a mortality rate of greater than 30%. Patients who previously had been vaccinated had a lower mortality rate. Treatment includes the drug cidofovir. Anthrax is caused by an aerobic gram-positive spore-forming bacillus (Bacillus anthracis) and has three primary forms: cutaneous, gastrointestinal, and inhalational. Weaponized anthrax is mainly an inhalational disease. Inhalational anthrax symptoms occur within 1 to 7 days of exposure and initially look like viral flu (fever, chills, myalgia, and a nonproductive cough). Later on, the patient’s mediastinal lymph nodes, where the spores germinate, enlarge, producing a widened mediastinum that can be seen on a chest x-ray film. Treatment is primarily with ciprofloxacin; prophylaxis to exposed personnel includes 60 days of ciprofloxacin. Mortality rate for inhaled anthrax is greater than 80%. Plague is caused by a gram-negative coccobacillus (Yersinia pestis) and has two forms: bubonic and pneumonic. With the more common bubonic plague, there is painful swelling of the lymph nodes (buboes), which can grow to 5 to 10 cm in diameter. The patients develop cyanosis, shock, and gangrene in peripheral tissues (black death). If the lungs become infected then pneumonic plague develops, which, if untreated, has 100% mortality. Treatment is primarily with streptomycin, although gentamicin, tetracycline, and chloramphenicol have been used. Botulism is caused by the toxin from Clostridium botulinum. Because this disease is due to a neurotoxin, it is not contagious. The neurotoxin affects cholinergic neurons and prevents the release of acetylcholine. Symptoms typically develop within 12 to 36 hours of exposure and include acute flaccid paralysis, decreased salivation, ileus, and urinary retention. There are no sensory deficits. With appropriate supportive care and trivalent equine antitoxin, the mortality rate is less than 5%. Without the use of antitoxin, patients may take 2 to 8 weeks to recover. Mortality rate is 5% to 10%. There are more than 18 hemorrhagic fever viruses, including Ebola virus. The incubation period is 2 to 21 days, and patients present with fever, myalgias, headaches, thrombocytopenia, and hemorrhagic complications (petechiae, ecchymosis). Untreated, the mortality rate for Ebola virus is 90%. Treatment includes the drug ribavirin (Barash: Clinical Anesthesia, ed 7, pp 1543–1545; Miller: Miller’s Anesthesia, ed 8, pp 2501–2502; Miller: Basics of Anesthesia, ed 6, pp 691–695).
554. Hemorrhagic fever A. Smallpox B. Anthrax C. Plague D. Botulism E. Ebola virus
554. (E) There are three categories of biological weapons: A, B, and C. All of the diseases in this question are in the highly contagious Category A agents. Smallpox is caused by a virus (Variola major) and in 1980 was declared extinct by the World Health Organization. The incubation period was 7 to 14 days, and patients with the disease presented with malaise, headache, and fever. Two to 4 days later a characteristic rash develops where all lesions are at the same stage (papules, vesicles, pustules, and scabs). Exposed patients and health care workers who received a vaccination within 4 days of exposure had greatly attenuated symptoms. Unvaccinated patients who were untreated had a mortality rate of greater than 30%. Patients who previously had been vaccinated had a lower mortality rate. Treatment includes the drug cidofovir. Anthrax is caused by an aerobic gram-positive spore-forming bacillus (Bacillus anthracis) and has three primary forms: cutaneous, gastrointestinal, and inhalational. Weaponized anthrax is mainly an inhalational disease. Inhalational anthrax symptoms occur within 1 to 7 days of exposure and initially look like viral flu (fever, chills, myalgia, and a nonproductive cough). Later on, the patient’s mediastinal lymph nodes, where the spores germinate, enlarge, producing a widened mediastinum that can be seen on a chest x-ray film. Treatment is primarily with ciprofloxacin; prophylaxis to exposed personnel includes 60 days of ciprofloxacin. Mortality rate for inhaled anthrax is greater than 80%. Plague is caused by a gram-negative coccobacillus (Yersinia pestis) and has two forms: bubonic and pneumonic. With the more common bubonic plague, there is painful swelling of the lymph nodes (buboes), which can grow to 5 to 10 cm in diameter. The patients develop cyanosis, shock, and gangrene in peripheral tissues (black death). If the lungs become infected then pneumonic plague develops, which, if untreated, has 100% mortality. Treatment is primarily with streptomycin, although gentamicin, tetracycline, and chloramphenicol have been used. Botulism is caused by the toxin from Clostridium botulinum. Because this disease is due to a neurotoxin, it is not contagious. The neurotoxin affects cholinergic neurons and prevents the release of acetylcholine. Symptoms typically develop within 12 to 36 hours of exposure and include acute flaccid paralysis, decreased salivation, ileus, and urinary retention. There are no sensory deficits. With appropriate supportive care and trivalent equine antitoxin, the mortality rate is less than 5%. Without the use of antitoxin, patients may take 2 to 8 weeks to recover. Mortality rate is 5% to 10%. There are more than 18 hemorrhagic fever viruses, including Ebola virus. The incubation period is 2 to 21 days, and patients present with fever, myalgias, headaches, thrombocytopenia, and hemorrhagic complications (petechiae, ecchymosis). Untreated, the mortality rate for Ebola virus is 90%. Treatment includes the drug ribavirin (Barash: Clinical Anesthesia, ed 7, pp 1543–1545; Miller: Miller’s Anesthesia, ed 8, pp 2501–2502; Miller: Basics of Anesthesia, ed 6, pp 691–695).
555. Decreased FEV1/FVC ratio A. Pulmonary emphysema B. Chronic bronchitis C. Restrictive pulmonary disease D. Pulmonary emphysema and chronic bronchitis E. Pulmonary emphysema and restrictive pulmonary disease
555. (D)Pulmonary function tests can be used to classify patients with chronic pulmonary disease into those with obstructive airway diseases (e.g., asthma, pulmonary emphysema, and chronic bronchitis) and those with restrictive pulmonary diseases (e.g., pulmonary fibrosis, scoliosis). The forced expiratory volume in 1 second or FEV1 is the amount of air expired in 1 second and commonly is expressed as a percentage of the forced vital capacity, or FEV1/FVC. The normal FEV1/FVC is 75% to 80%. In the presence of 154 Part 2 Clinical Sciences obstructive airway disease, FEV1 of less than 70% has mild obstruction, less than 60% has moderate obstruction, and less than 50% has severe obstruction. Patients with obstructive lung disease also have a normal (asthma) or increase in (bronchitis, emphysema) TLC and FRC. In the presence of restrictive pulmonary disease, FEV1 is reduced, but because FVC is also reduced, the FEV1/FVC is normal. Patients with restrictive disease have a TLC, FRC, and total pulmonary compliance that are reduced. In patients with pulmonary emphysema, lung compliance is increased because the elastic recoil of the lungs is decreased (Miller: Miller’s Anesthesia, ed 8, p 1149; Miller: Basics of Anesthesia, ed 6, pp 431-–434).
556. Decreased total pulmonary compliance A. Pulmonary emphysema B. Chronic bronchitis C. Restrictive pulmonary disease D. Pulmonary emphysema and chronic bronchitis E. Pulmonary emphysema and restrictive pulmonary disease
556. (C)Pulmonary function tests can be used to classify patients with chronic pulmonary disease into those with obstructive airway diseases (e.g., asthma, pulmonary emphysema, and chronic bronchitis) and those with restrictive pulmonary diseases (e.g., pulmonary fibrosis, scoliosis). The forced expiratory volume in 1 second or FEV1 is the amount of air expired in 1 second and commonly is expressed as a percentage of the forced vital capacity, or FEV1/FVC. The normal FEV1/FVC is 75% to 80%. In the presence of 154 Part 2 Clinical Sciences obstructive airway disease, FEV1 of less than 70% has mild obstruction, less than 60% has moderate obstruction, and less than 50% has severe obstruction. Patients with obstructive lung disease also have a normal (asthma) or increase in (bronchitis, emphysema) TLC and FRC. In the presence of restrictive pulmonary disease, FEV1 is reduced, but because FVC is also reduced, the FEV1/FVC is normal. Patients with restrictive disease have a TLC, FRC, and total pulmonary compliance that are reduced. In patients with pulmonary emphysema, lung compliance is increased because the elastic recoil of the lungs is decreased (Miller: Miller’s Anesthesia, ed 8, p 1149; Miller: Basics of Anesthesia, ed 6, pp 431-–434).
557. Increased TLC A. Pulmonary emphysema B. Chronic bronchitis C. Restrictive pulmonary disease D. Pulmonary emphysema and chronic bronchitis E. Pulmonary emphysema and restrictive pulmonary disease
557. (D)Pulmonary function tests can be used to classify patients with chronic pulmonary disease into those with obstructive airway diseases (e.g., asthma, pulmonary emphysema, and chronic bronchitis) and those with restrictive pulmonary diseases (e.g., pulmonary fibrosis, scoliosis). The forced expiratory volume in 1 second or FEV1 is the amount of air expired in 1 second and commonly is expressed as a percentage of the forced vital capacity, or FEV1/FVC. The normal FEV1/FVC is 75% to 80%. In the presence of 154 Part 2 Clinical Sciences obstructive airway disease, FEV1 of less than 70% has mild obstruction, less than 60% has moderate obstruction, and less than 50% has severe obstruction. Patients with obstructive lung disease also have a normal (asthma) or increase in (bronchitis, emphysema) TLC and FRC. In the presence of restrictive pulmonary disease, FEV1 is reduced, but because FVC is also reduced, the FEV1/FVC is normal. Patients with restrictive disease have a TLC, FRC, and total pulmonary compliance that are reduced. In patients with pulmonary emphysema, lung compliance is increased because the elastic recoil of the lungs is decreased (Miller: Miller’s Anesthesia, ed 8, p 1149; Miller: Basics of Anesthesia, ed 6, pp 431-–434).
558. Decreased FRC A. Pulmonary emphysema B. Chronic bronchitis C. Restrictive pulmonary disease D. Pulmonary emphysema and chronic bronchitis E. Pulmonary emphysema and restrictive pulmonary disease
558. (C)Pulmonary function tests can be used to classify patients with chronic pulmonary disease into those with obstructive airway diseases (e.g., asthma, pulmonary emphysema, and chronic bronchitis) and those with restrictive pulmonary diseases (e.g., pulmonary fibrosis, scoliosis). The forced expiratory volume in 1 second or FEV1 is the amount of air expired in 1 second and commonly is expressed as a percentage of the forced vital capacity, or FEV1/FVC. The normal FEV1/FVC is 75% to 80%. In the presence of 154 Part 2 Clinical Sciences obstructive airway disease, FEV1 of less than 70% has mild obstruction, less than 60% has moderate obstruction, and less than 50% has severe obstruction. Patients with obstructive lung disease also have a normal (asthma) or increase in (bronchitis, emphysema) TLC and FRC. In the presence of restrictive pulmonary disease, FEV1 is reduced, but because FVC is also reduced, the FEV1/FVC is normal. Patients with restrictive disease have a TLC, FRC, and total pulmonary compliance that are reduced. In patients with pulmonary emphysema, lung compliance is increased because the elastic recoil of the lungs is decreased (Miller: Miller’s Anesthesia, ed 8, p 1149; Miller: Basics of Anesthesia, ed 6, pp 431-–434).
559. Decreased FEV1, normal FEV1/FVC ratio A. Pulmonary emphysema B. Chronic bronchitis C. Restrictive pulmonary disease D. Pulmonary emphysema and chronic bronchitis E. Pulmonary emphysema and restrictive pulmonary disease
559. (C)Pulmonary function tests can be used to classify patients with chronic pulmonary disease into those with obstructive airway diseases (e.g., asthma, pulmonary emphysema, and chronic bronchitis) and those with restrictive pulmonary diseases (e.g., pulmonary fibrosis, scoliosis). The forced expiratory volume in 1 second or FEV1 is the amount of air expired in 1 second and commonly is expressed as a percentage of the forced vital capacity, or FEV1/FVC. The normal FEV1/FVC is 75% to 80%. In the presence of 154 Part 2 Clinical Sciences obstructive airway disease, FEV1 of less than 70% has mild obstruction, less than 60% has moderate obstruction, and less than 50% has severe obstruction. Patients with obstructive lung disease also have a normal (asthma) or increase in (bronchitis, emphysema) TLC and FRC. In the presence of restrictive pulmonary disease, FEV1 is reduced, but because FVC is also reduced, the FEV1/FVC is normal. Patients with restrictive disease have a TLC, FRC, and total pulmonary compliance that are reduced. In patients with pulmonary emphysema, lung compliance is increased because the elastic recoil of the lungs is decreased (Miller: Miller’s Anesthesia, ed 8, p 1149; Miller: Basics of Anesthesia, ed 6, pp 431-–434).
560. Increased lung compliance due to loss of elastic recoil of the lung A. Pulmonary emphysema B. Chronic bronchitis C. Restrictive pulmonary disease D. Pulmonary emphysema and chronic bronchitis E. Pulmonary emphysema and restrictive pulmonary disease Questions 561-566:
560. (A) Pulmonary function tests can be used to classify patients with chronic pulmonary disease into those with obstructive airway diseases (e.g., asthma, pulmonary emphysema, and chronic bronchitis) and those with restrictive pulmonary diseases (e.g., pulmonary fibrosis, scoliosis). The forced expiratory volume in 1 second or FEV1 is the amount of air expired in 1 second and commonly is expressed as a percentage of the forced vital capacity, or FEV1/FVC. The normal FEV1/FVC is 75% to 80%. In the presence of 154 Part 2 Clinical Sciences obstructive airway disease, FEV1 of less than 70% has mild obstruction, less than 60% has moderate obstruction, and less than 50% has severe obstruction. Patients with obstructive lung disease also have a normal (asthma) or increase in (bronchitis, emphysema) TLC and FRC. In the presence of restrictive pulmonary disease, FEV1 is reduced, but because FVC is also reduced, the FEV1/FVC is normal. Patients with restrictive disease have a TLC, FRC, and total pulmonary compliance that are reduced. In patients with pulmonary emphysema, lung compliance is increased because the elastic recoil of the lungs is decreased (Miller: Miller’s Anesthesia, ed 8, p 1149; Miller: Basics of Anesthesia, ed 6, pp 431-–434).
561. Weakness of all muscles below the knee A. Sciatic nerve injury B. Common peroneal nerve injury C. Femoral nerve injury D. Obturator nerve injury E. Lateral femoral cutaneous nerve injury
561. (A)In many cases of peripheral nerve injuries, the mechanism of injury is largely unknown; however, stretching or compression of the nerves can lead to nerve ischemia and damage. In the lithotomy position, hyperflexion of the hips and/or extension of the knees can aggravate stretch of the sciatic nerve. Also in the lithotomy position, compression of the common peroneal nerve between the head of the fibula and the metal supporting frame can occur. The common peroneal nerve is the most common nerve injured in the lithotomy position. Proper padding between the metal leg braces and positioning of the legs will limit the occurrence of these injuries. The sciatic nerve provides motor function for all the skeletal muscles below the knees and sensory innervation for the lateral half of the leg and most of the foot. Injury to the common peroneal nerve, a branch of the sciatic nerve, causes a footdrop from the impaired ankle dorsiflexion and the loss of foot eversion and toe extension. Injury to the femoral or obturator nerves can occur with excessive retraction during lower abdominal surgery. The obturator nerve can also be injured during a difficult forceps vaginal delivery or by excessive flexion of the thigh to the groin. Injury to the femoral nerve will manifest as decreased extension of the knee (paresis of the quadriceps femoris muscle) and numbness over the anterior aspect of the thigh and medial/anteromedial side of the leg. The inability to adduct the leg and thigh as well as numbness over the medial side of the thigh are clinical manifestations consistent with damage to the obturator nerve. Excessive flexion of the hip on the abdomen can cause a neuropathy of the lateral femoral cutaneous nerve (sensory only) resulting in numbness of the lateral aspect of the thigh (Miller: Miller’s Anesthesia, ed 8, pp 1256–1258; Miller: Basics of Anesthesia, ed 6, pp 304, 305, 313, 314).
562. Footdrop; loss of dorsal extension of the toes A. Sciatic nerve injury B. Common peroneal nerve injury C. Femoral nerve injury D. Obturator nerve injury E. Lateral femoral cutaneous nerve injury
562. (B)In many cases of peripheral nerve injuries, the mechanism of injury is largely unknown; however, stretching or compression of the nerves can lead to nerve ischemia and damage. In the lithotomy position, hyperflexion of the hips and/or extension of the knees can aggravate stretch of the sciatic nerve. Also in the lithotomy position, compression of the common peroneal nerve between the head of the fibula and the metal supporting frame can occur. The common peroneal nerve is the most common nerve injured in the lithotomy position. Proper padding between the metal leg braces and positioning of the legs will limit the occurrence of these injuries. The sciatic nerve provides motor function for all the skeletal muscles below the knees and sensory innervation for the lateral half of the leg and most of the foot. Injury to the common peroneal nerve, a branch of the sciatic nerve, causes a footdrop from the impaired ankle dorsiflexion and the loss of foot eversion and toe extension. Injury to the femoral or obturator nerves can occur with excessive retraction during lower abdominal surgery. The obturator nerve can also be injured during a difficult forceps vaginal delivery or by excessive flexion of the thigh to the groin. Injury to the femoral nerve will manifest as decreased extension of the knee (paresis of the quadriceps femoris muscle) and numbness over the anterior aspect of the thigh and medial/anteromedial side of the leg. The inability to adduct the leg and thigh as well as numbness over the medial side of the thigh are clinical manifestations consistent with damage to the obturator nerve. Excessive flexion of the hip on the abdomen can cause a neuropathy of the lateral femoral cutaneous nerve (sensory only) resulting in numbness of the lateral aspect of the thigh (Miller: Miller’s Anesthesia, ed 8, pp 1256–1258; Miller: Basics of Anesthesia, ed 6, pp 304, 305, 313, 314). 155
563. Weakness of the muscles that extend the knee A. Sciatic nerve injury B. Common peroneal nerve injury C. Femoral nerve injury D. Obturator nerve injury E. Lateral femoral cutaneous nerve injury
563. (C)In many cases of peripheral nerve injuries, the mechanism of injury is largely unknown; however, stretching or compression of the nerves can lead to nerve ischemia and damage. In the lithotomy position, hyperflexion of the hips and/or extension of the knees can aggravate stretch of the sciatic nerve. Also in the lithotomy position, compression of the common peroneal nerve between the head of the fibula and the metal supporting frame can occur. The common peroneal nerve is the most common nerve injured in the lithotomy position. Proper padding between the metal leg braces and positioning of the legs will limit the occurrence of these injuries. The sciatic nerve provides motor function for all the skeletal muscles below the knees and sensory innervation for the lateral half of the leg and most of the foot. Injury to the common peroneal nerve, a branch of the sciatic nerve, causes a footdrop from the impaired ankle dorsiflexion and the loss of foot eversion and toe extension. Injury to the femoral or obturator nerves can occur with excessive retraction during lower abdominal surgery. The obturator nerve can also be injured during a difficult forceps vaginal delivery or by excessive flexion of the thigh to the groin. Injury to the femoral nerve will manifest as decreased extension of the knee (paresis of the quadriceps femoris muscle) and numbness over the anterior aspect of the thigh and medial/anteromedial side of the leg. The inability to adduct the leg and thigh as well as numbness over the medial side of the thigh are clinical manifestations consistent with damage to the obturator nerve. Excessive flexion of the hip on the abdomen can cause a neuropathy of the lateral femoral cutaneous nerve (sensory only) resulting in numbness of the lateral aspect of the thigh (Miller: Miller’s Anesthesia, ed 8, pp 1256–1258; Miller: Basics of Anesthesia, ed 6, pp 304, 305, 313, 314). 155
564. Inability to adduct the leg; diminished sensation over the medial side of the thigh A. Sciatic nerve injury B. Common peroneal nerve injury C. Femoral nerve injury D. Obturator nerve injury E. Lateral femoral cutaneous nerve injury
564. (D)In many cases of peripheral nerve injuries, the mechanism of injury is largely unknown; however, stretching or compression of the nerves can lead to nerve ischemia and damage. In the lithotomy position, hyperflexion of the hips and/or extension of the knees can aggravate stretch of the sciatic nerve. Also in the lithotomy position, compression of the common peroneal nerve between the head of the fibula and the metal supporting frame can occur. The common peroneal nerve is the most common nerve injured in the lithotomy position. Proper padding between the metal leg braces and positioning of the legs will limit the occurrence of these injuries. The sciatic nerve provides motor function for all the skeletal muscles below the knees and sensory innervation for the lateral half of the leg and most of the foot. Injury to the common peroneal nerve, a branch of the sciatic nerve, causes a footdrop from the impaired ankle dorsiflexion and the loss of foot eversion and toe extension. Injury to the femoral or obturator nerves can occur with excessive retraction during lower abdominal surgery. The obturator nerve can also be injured during a difficult forceps vaginal delivery or by excessive flexion of the thigh to the groin. Injury to the femoral nerve will manifest as decreased extension of the knee (paresis of the quadriceps femoris muscle) and numbness over the anterior aspect of the thigh and medial/anteromedial side of the leg. The inability to adduct the leg and thigh as well as numbness over the medial side of the thigh are clinical manifestations consistent with damage to the obturator nerve. Excessive flexion of the hip on the abdomen can cause a neuropathy of the lateral femoral cutaneous nerve (sensory only) resulting in numbness of the lateral aspect of the thigh (Miller: Miller’s Anesthesia, ed 8, pp 1256–1258; Miller: Basics of Anesthesia, ed 6, pp 304, 305, 313, 314). 155
565. Most commonly caused by placement of patient into the lithotomy position A. Sciatic nerve injury B. Common peroneal nerve injury C. Femoral nerve injury D. Obturator nerve injury E. Lateral femoral cutaneous nerve injury
565. (B)In many cases of peripheral nerve injuries, the mechanism of injury is largely unknown; however, stretching or compression of the nerves can lead to nerve ischemia and damage. In the lithotomy position, hyperflexion of the hips and/or extension of the knees can aggravate stretch of the sciatic nerve. Also in the lithotomy position, compression of the common peroneal nerve between the head of the fibula and the metal supporting frame can occur. The common peroneal nerve is the most common nerve injured in the lithotomy position. Proper padding between the metal leg braces and positioning of the legs will limit the occurrence of these injuries. The sciatic nerve provides motor function for all the skeletal muscles below the knees and sensory innervation for the lateral half of the leg and most of the foot. Injury to the common peroneal nerve, a branch of the sciatic nerve, causes a footdrop from the impaired ankle dorsiflexion and the loss of foot eversion and toe extension. Injury to the femoral or obturator nerves can occur with excessive retraction during lower abdominal surgery. The obturator nerve can also be injured during a difficult forceps vaginal delivery or by excessive flexion of the thigh to the groin. Injury to the femoral nerve will manifest as decreased extension of the knee (paresis of the quadriceps femoris muscle) and numbness over the anterior aspect of the thigh and medial/anteromedial side of the leg. The inability to adduct the leg and thigh as well as numbness over the medial side of the thigh are clinical manifestations consistent with damage to the obturator nerve. Excessive flexion of the hip on the abdomen can cause a neuropathy of the lateral femoral cutaneous nerve (sensory only) resulting in numbness of the lateral aspect of the thigh (Miller: Miller’s Anesthesia, ed 8, pp 1256–1258; Miller: Basics of Anesthesia, ed 6, pp 304, 305, 313, 314). 155
566. Numbness over the lateral aspect of the thigh A. Sciatic nerve injury B. Common peroneal nerve injury C. Femoral nerve injury D. Obturator nerve injury E. Lateral femoral cutaneous nerve injury
566. (E) In many cases of peripheral nerve injuries, the mechanism of injury is largely unknown; however, stretching or compression of the nerves can lead to nerve ischemia and damage. In the lithotomy position, hyperflexion of the hips and/or extension of the knees can aggravate stretch of the sciatic nerve. Also in the lithotomy position, compression of the common peroneal nerve between the head of the fibula and the metal supporting frame can occur. The common peroneal nerve is the most common nerve injured in the lithotomy position. Proper padding between the metal leg braces and positioning of the legs will limit the occurrence of these injuries. The sciatic nerve provides motor function for all the skeletal muscles below the knees and sensory innervation for the lateral half of the leg and most of the foot. Injury to the common peroneal nerve, a branch of the sciatic nerve, causes a footdrop from the impaired ankle dorsiflexion and the loss of foot eversion and toe extension. Injury to the femoral or obturator nerves can occur with excessive retraction during lower abdominal surgery. The obturator nerve can also be injured during a difficult forceps vaginal delivery or by excessive flexion of the thigh to the groin. Injury to the femoral nerve will manifest as decreased extension of the knee (paresis of the quadriceps femoris muscle) and numbness over the anterior aspect of the thigh and medial/anteromedial side of the leg. The inability to adduct the leg and thigh as well as numbness over the medial side of the thigh are clinical manifestations consistent with damage to the obturator nerve. Excessive flexion of the hip on the abdomen can cause a neuropathy of the lateral femoral cutaneous nerve (sensory only) resulting in numbness of the lateral aspect of the thigh (Miller: Miller’s Anesthesia, ed 8, pp 1256–1258; Miller: Basics of Anesthesia, ed 6, pp 304, 305, 313, 314). 155
567. A previously healthy 1-month-old infant with a strong family history of sickle cell anemia is brought to the emergency room with an incarcerated inguinal hernia. Which of the following should be carried out before surgery? A. Hemoglobin electrophoresis B. Peripheral smear C. Hematology consultation D. None of the above
567. (D) At birth, the concentration of hemoglobin F (fetal hemoglobin) is about 80% and reaches its lowest level by 2 to 4 months of age. Sickle cell anemia (hemoglobin SS) is an inherited disorder of the β-chain of the adult hemoglobin molecule caused by a single amino acid substitution. It has an incidence of about 0.2% in the African-American population, in contrast to the relatively benign heterozygous condition, sickle cell trait (hemoglobin AS), which affects 8% to 10% of the same group. Sickling can occur in homozygous patients who become hypoxic, acidotic, hypothermic, or dehydrated. The predominant hemoglobin in this 1-month-old infant is hemoglobin F, which would temporarily protect the infant from the manifestations of sickle cell anemia were he or she homozygous for hemoglobin S. The patient should, however, be worked up for sickle cell anemia at some point in early life (if hemoglobin electrophoresis was not done as part of routine newborn screening in at-risk populations), but such a workup is not a prerequisite for surgery at 1 month of age (Davis: Smith’s Anesthesia for Infants and Children, ed 8, pp 284, 1062, 1130; Hines: Stoelting’s Anesthesia and Co-Existing Disease, ed 6, pp 411–412; Miller: Miller’s Anesthesia, ed 8, pp 1211–1212).
568. In the premature newborn, the glottis is at which level relative to the cervical spine? A. C3 B. C4 C. C5 D. C6
568. (A) The glottis of a premature newborn is at the level of C3, for the term newborn the level is C4, and in the adult the glottis is at the C5 level. The relatively high glottis makes intubation more difficult in the premature newborn (i.e., more tissue and less distance) (Barash: Clinical Anesthesia, ed 7, p 1185; Davis: Smith’s Anesthesia for Infants and Children, ed 8, p 351; Miller: Miller’s Anesthesia, ed 8, pp 2757–2761).
569. A 5-month-old infant is scheduled for an elective operative reduction of a right inguinal hernia. Spinal anesthesia is performed. The first sign of a high spinal block in this patient would be A. Hypotension B. Tachycardia C. Hypoxia D. Asystole
569. (C) Spinal anesthesia can be administered safely to children of all ages. Hypotension secondary to a loss of sympathetic tone, common in the adult, is rare in the child younger than 5 years of age even with levels as high as T3. Because of this hemodynamic stability, some pediatric anesthesiologists start an IV line in the leg after the spinal anesthetic is administered to the infant. Respiratory depression including apnea and hypoxia with associated bradycardia will likely be initial signs associated with a total spinal block in the infant (Barash: Clinical Anesthesia, ed 7, pp 1196–1197; Davis: Smith’s Anesthesia for Infants and Children, ed 8, pp 463–465).
570. What percentage of a term newborn’s total body weight consists of water? A. 45% B. 60% C. 75% D. 90%
570. (C) The body compartment volumes change with age. Muscle contains about 75% water, whereas adipose tissue contains only 10% water. Overall, total body water decreases with age mainly due to a decrease in extracellular fluid, whereas the muscle and fat content increases. The fraction of total body weight that consists of water is 80% in premature newborns, 75% in term newborns, and 60% in 6-monthold infants and in adults. These alterations in body composition have implications for the volume of distribution and redistribution of drugs (Davis: Smith’s Anesthesia for Infants and Children, ed 8, p 123; Miller: Miller’s Anesthesia, ed 8, pp 2763–2764).
571. What is the maximum Fio2 that can be administered to the mother without increasing the risk of retinopathy of prematurity (ROP) in the fetus in utero? A. 0.35 B. 0.50 C. 0.75 D. 1.0
571. (D) The fetal Pao2 does not increase above 60 mm Hg when 100% O2 is administered to the mother because of the high O2 consumption of the placenta and uneven distribution of the maternal and fetal blood flow in the placenta. For these reasons, the Fio2 administered to the mother is not a factor in the etiology of ROP in utero (Suresh: Shnider and Levinson’s Anesthesia for Obstetrics, ed 5, p 811).
572. Which of the following patients is LEAST likely to develop ROP? A. A term infant, 46 weeks’ postconceptual age (PCA), exposed to 100% oxygen for 6 hours B. A premature infant, 29 weeks’ PCA, exposed to a Pao2 of 150 mm Hg for 1 hour C. A premature infant, 28 weeks’ PCA, never exposed to supplemental oxygen D. A cyanotic infant with tetralogy of Fallot, 34 weeks’ PCA, receiving supplemental oxygen
572. (A) Retinopathy of prematurity (ROP), formally called retrolental fibroplasia, typically occurs in newborns who are born at less than 35 weeks’ gestational age. It is the second leading cause of childhood blindness in the United States. The risk of ROP is inversely related to age and birth weight, with a significant risk occurring in infants weighing less than 1500 g. ROP occurs in about 70% of infants who weigh less than 1000 g at birth; fortunately, 80% to 90% of these have spontaneous regression of the retinal changes. The risk is negligible after 44 weeks’ postconceptual age. The mechanism for ROP is complex and is related to the complicated process of retinal development and maturation. Under normal circumstances, retinal vasculature develops from the optic disk toward the periphery of the retina. This process typically is completed by 40 to 44 weeks of gestation. Hyperoxia causes constriction of the retinal arterioles, resulting in swelling and degeneration of the endothelium that disrupts normal retinal development. Vascularization of the retina resumes in an abnormal fashion when normoxic conditions 164 Part 2 Clinical Sciences return, resulting in neovascularization and scarring of the retina. In the worst-case scenario, this process can lead to retinal detachment and blindness. Consequently, hyperoxia should be avoided when anesthetizing preterm infants. Exposure of preterm infants to Pao2 greater than 80 mm Hg for prolonged periods of time may be associated with increased incidence and severity of retinopathy of prematurity. To reduce this risk, it is recommended that the oxygen saturation be maintained between 88% and 93% (about Pao2 of 50-70 mm Hg) during anesthesia. On the other hand, one must never compromise oxygen delivery to a neonate’s brain to protect the eyes. Although oxygen toxicity has been strongly associated with ROP, other factors are also important, such as respiratory distress syndrome, mechanical ventilation, hypoxia, hypocarbia, hypercarbia, blood transfusions, sepsis, congenital infections, and vitamin E deficiency. In fact, newborns with cyanotic congenital heart disease who have never been exposed to supplemental oxygen therapy have also developed ROP (Davis: Smith’s Anesthesia for Infants and Children, ed 8, p 883; Hines: Stoelting’s Anesthesia and Co-Existing Disease, ed 6, pp 591–592; Miller: Basics of Anesthesia, ed 6, p 564).
573. A 5-week-old male infant is brought to the emergency room with projectile vomiting. At the time of admission the patient is lethargic with a respiratory rate of 16 breaths/min and has had no urine output in the preceding 3 hours. A diagnosis of pyloric stenosis is made, and the infant is brought emergently to the operating room (OR) for pyloromyotomy. The MOST appropriate anesthetic management would be A. Awake intubation after placing an oral gastric tube B. Inhalation induction with sevoflurane with cricoid pressure C. Awake saphenous IV catheter or an intraosseous needle placement followed by a rapid-sequence induction with ketamine, atropine, and rocuronium D. Postpone surgery
573. (D) This patient has signs consistent with severe dehydration and needs resuscitation with fluid and electrolytes before surgery. Surgery should be delayed until there is thorough evaluation and treatment of the fluid and electrolyte imbalances. Pyloric stenosis occurs in approximately 1 in every 300 live births, making it one of the most common gastrointestinal abnormalities seen in the first 6 months of life. Pyloric stenosis occurs as frequently in preterm as in term neonates, and there is a predilection for male infants. Persistent vomiting usually manifests itself between the second and sixth weeks of age and can result in dehydration, hypokalemia, hypochloremia, and metabolic alkalosis. Fluid resuscitation should be initiated with isotonic saline. If an IV line catheter cannot be established, an intraosseous needle should be placed. After the patient voids, potassium then can be safely added to the IV fluids. Once there has been adequate hydration and correction of the electrolyte and acid-base abnormalities, the patient can more safely undergo anesthesia and surgery. Although several days may be required to restore normal fluid and electrolyte balance in some children, most respond within 12 to 48 hours (Davis: Smith’s Anesthesia for Infants and Children, ed 8, pp 750–751; Hines: Stoelting’s Anesthesia and Co-Existing Disease, ed 6, pp 600–601).
574. Which figure of esophageal atresia (EA) or tracheoesophageal fistula (TEF) is the MOST common? A B C D
574. (A) EA and TEFs result from failure of the esophagus and the trachea to completely separate during development. Incidence is approximately 1 in 4000 live births. Although each of the listed answers is possible, Figure A represents the most common type (86% of cases) called a Type C TEF (EA with a distal TEF). In the delivery room, one is unable to pass a suction catheter into the stomach and, if an x-ray is taken, the presence of air in the stomach suggests a fistula between the trachea and the stomach. If it is not detected in the delivery room, the newborn tends to have excessive oral secretions and is unable to feed. In addition, because the fetuses cannot swallow, there is a higher incidence of maternal polyhydramnios and premature deliveries. Note: About 20% of patients with EA or TEF have major cardiovascular anomalies (e.g., atrial septal defect [ASD], ventricular septal defect [VSD], tetralogy of Fallot, atrioventricular [AV] canal, coarctation of the aorta). Figure B (8% of cases) is a Type A TEF (EA without a TEF). Figure C (4% of cases) is a Type E TEF (TEF without an EA), and is also called an H-type TEF. Figure D (1% of cases) is a Type D TEF (EA with a proximal and a distal TEF). Type B (1% of cases; not shown) is a Type B TEF (EA with a proximal TEF). See also Question 591 (Davis: Smith’s Anesthesia for Infants and Children, ed 8, pp 574–579; Hines: Stoelting’s Anesthesia and Co- Existing Disease, ed 6, pp 581–582, 596–598; Miller: Basics of Anesthesia, ed 6, pp 561–562).
575. What is the maximum allowable blood loss (MABL) for a 10-kg, 11-month-old infant whose starting hematocrit (Hct) is 36 and the minimal acceptable Hct is 25? A. 110 mL B. 245 mL C. 350 mL D. Cannot be calculated without additional information
575. (B) To calculate the MABL, the following formula is commonly used: MABL = Estimated blood volume × (starting hematocrit − target hematocrit) Starting hematocrit The estimated blood volume (EBV) in mL/kg for a premature infant is 90 to 100 mL/kg, term newborns is 80 to 90 mL/kg, 3-month-olds to 1-year-olds is 75 to 80 mL/kg, 3-year-olds to 6-year-olds is 70 to 75 mL/kg, and older than 6 years of age is 65 to 70 mL/kg. In this case, using 80 mL/kg, the EBV for the 10-kg 11-month-old, we have an EBV of 800 mL. MABL = 800 mL (36 − 25)/36 = about 245 mL Before infusing blood, the circulating blood volume is usually expanded with crystalloids in a ratio of 3 mL of crystalloid for each mL of blood lost (Davis: Smith’s Anesthesia for Infants and Children, ed 8, pp 384–385, 409; Miller: Miller’s Anesthesia, ed 8, pp 2784–2785). Pediatric Physiology and Anesthesia 165
576. What volume of packed red blood cells (PRBCs) with an Hct of 60 is needed to raise the Hct from 20 to 28 in a 10-kg, 11-month-old? A. 55 mL B. 105 mL C. 155 mL D. Cannot be calculated without additional information
576. (B) If blood loss exceeds the MABL replacement, PRBCs are usually needed. The normal Hct of PRBCs is 60% to 80%. To calculate the volume of PRBCs to be transfused, the following formula is used: Volume of PRBCs = Estimated blood volume × (desired Hct − present Hct) Hct of the PRBCs In this case, volume to be infused = 800 × (28 − 20)/60 = 106 mL (Davis: Smith’s Anesthesia for Infants and Children, ed 8, pp 384–385; Miller: Miller’s Anesthesia, ed 8, pp 2784–2785 ).
577. Reasons for selecting a cuffed endotracheal tube over an uncuffed endotracheal tube include all of the following EXCEPT A. Fewer intubations and endotracheal tubes are needed B. Less chance for airway fires C. Spontaneous breathing is easier D. Aspiration of gastric contents is less likely
577. (C) Given that cuffed endotracheal tubes are often chosen to be a size smaller (i.e., 0.5 mm) than uncuffed endotracheal tubes, the lumen is narrower and, therefore, spontaneous breathing is more difficult. Because a smaller endotracheal tube can be used with a cuff, fewer intubations are needed to select the correct tube size. Also because of the cuff, less gas leaks from the trachea into the pharynx, allowing administration of lower gas flows with potential cost savings as well as less environmental pollution. The gases are less likely to leak into the pharynx, and this should decrease the chance of an airway fire when high oxygen or nitrous oxide concentrations are used with cautery in the oral cavity. To further decrease the chance of an airway fire, most anesthesiologists would avoid the use of nitrous oxide and would decrease the Fio2 to around 0.30 if oxygen saturations are acceptable. The chance of aspiration of gastric contents should also be less likely (Davis: Smith’s Anesthesia for Infants and Children, ed 8, pp 356–357; Miller: Basics of Anesthesia, ed 6, p 554).
578. An otherwise healthy 4-year-old male patient is undergoing elective tonsillectomy. Before induction of general anesthesia, the patient is breathing at a rate of 20 breaths/min. An inhalation induction is begun with sevoflurane, nitrous oxide, and oxygen. Ninety seconds later, the patient is noted to breathe at a rate of 40 breaths/min. This rapid respiratory rate most likely represents A. Hypoxia B. Hypercarbia and early development of malignant hyperthermia (MH) C. The excitement stage of anesthesia D. Aspiration of gastric contents
578. (C) Inhalation agents are respiratory depressants. In general, they increase the respiratory rate and decrease the tidal volume (Vt) of respirations and are associated with an increase in Paco2. When inducing a child with an inhalation agent, especially below the minimum alveolar concentration (MAC) level, the respiratory pattern can vary and include breath holding, excessive hyperventilation, and laryngospasm. Although the stages of inhalation anesthesia were classically described with ether, similar stages are seen with the newer inhalation agents, but because the signs are less pronounced they are rarely described anymore. The classic stages of depth of ether anesthesia include the first stage of anesthesia (analgesia). Patients in the first stage can respond to verbal stimulation, have an intact lid reflex, have normal respiratory patterns and intact airway reflexes, and have some analgesia. The second stage of anesthesia (delirium or excitement stage) is associated with unconsciousness, irregular and unpredictable respiratory patterns (including hyperventilation), nonpurposeful muscle movements, and the risk of clinically important reflex activity (e.g., laryngospasm, vomiting, cardiac arrhythmias). The third stage of anesthesia (surgical anesthesia) is associated with a return to more regular periodic respirations and is the level associated with the achievement of MAC. MAC is noted by the absence of movement (in 50% of patients) in response to a surgical incision. As anesthesia is deepened, stage 4 (respiratory paralysis) is associated with respiratory and cardiovascular arrest. In the case cited in this question, the second stage of anesthesia is demonstrated. Note: MH triggered by the sole use of volatile anesthetics produces an elevation of carbon dioxide levels with tachypnea and tachycardia, but this is rare during the first 20 minutes of an anesthetic. Sevoflurane and desflurane seem to be less of a trigger than halothane. Mild hypothermia, propofol, nondepolarizing neuromuscular blockers, and tranquilizers may delay or prevent MH from developing. Succinylcholine (the only depolarizing neuromuscular blocker in use today) often hastens the development of MH in susceptible patients. Aspiration of gastric contents would more likely lead to laryngospasms, wheezing, and hypoxia (Davis: Smith’s Anesthesia for Infants and Children, ed 8, pp 230–231; Miller: Miller’s Anesthesia, ed 8, pp 691–692, 1294–1295; Butterworth: Morgan & Mikhail’s Clinical Anesthesiology, ed 5, pp 890–891).
579. A healthy 3-kg, 1-month-old neonate is anesthetized for an inguinal hernia repair. An inhalation induction with sevoflurane is carried out and the patient is intubated. Before the surgical incision, the systolic blood pressure is noted to be 65 mm Hg and the heart rate is 130 beats/min. The most appropriate intervention for this patient’s blood pressure would be A. Administration of ephedrine B. Administration of phenylephrine C. 50-mL fluid bolus D. None of the above
579. (D) The hemodynamic indices described in this question are normal for healthy 1-month-old neonates (Miller: Basics of Anesthesia, ed 6, pp 548–550). COMPARISON OF CARDIOVASCULAR VARIABLES Neonate (<30 days) 6-12 Months of Age 3-5 Years of Age Adults (>16) Weight (kg) 3 7-10 14-18 70 Oxygen consumption ( mL/kg/min) 6-8 5 4 3 Systolic blood pressure (mm Hg) 60-75 70-90 80-100 100-125 Heart rate (beats/min) 120-160 100-140 80-120 60-100 Data from Miller RD: Basics of Anesthesia, ed 6, Philadelphia, Saunders, 2011, pp 548–550. 166 Part 2 Clinical Sciences
580. A 5-year-old boy is anesthetized for elective repair of an umbilical hernia. General anesthesia is induced and maintained with sevoflurane, nitrous oxide, and oxygen via an anesthesia mask. At the conclusion of the operation, the patient is taken to the recovery room and subsequently discharged to the outpatient ward. Before discharge, the patient’s mother notes that the urine appears dark brown (cola-colored). The most appropriate action at this time would be A. Discharge the patient with instructions to return if urine color does not normalize B. Discharge the patient in 3 hours if no other signs or symptoms are manifested C. Obtain serum creatinine and blood urea nitrogen (BUN) levels and discharge the patient if they are normal D. Evaluate the patient for MH
580. (D) Dark brown or cola-colored urine (i.e., myoglobinemia) may be caused by rhabdomyolysis, a possible sign of MH, and this patient should be evaluated. More typical signs and symptoms of MH include tachycardia, tachypnea, hypercarbia, hyperkalemia with peaked T waves, acidosis, increased sympathetic activity, irregular heartbeat, mottled cyanotic skin, profuse sweating, and a late sign of increased temperature (> 1.5° C over 5 minutes or temperature >38.8° C). Supportive laboratory tests for MH include elevated serum creatine phosphokinase (CPK); myoglobin in the serum and urine; increased serum potassium, calcium, and lactate levels; and a metabolic/respiratory acidosis on an arterial blood gas. If the presumed diagnosis is MH, therapy should be initiated (Davis: Smith’s Anesthesia for Infants and Children, ed 8, pp 1186–1189; Hines: Stoelting’s Anesthesia and Co-Existing Disease, ed 6, pp 635–637 ).
581. At what maximum inspiratory pressure should an endotracheal tube leak in a child? A. 5 to 15 cm H2O B. 15 to 25 cm H2O C. 25 to 35 cm H2O D. None of the above
581. (B) In infants and young children, there should be a small air leak around the endotracheal tube at peak inflation pressures of approximately 15 to 25 cm H2O. The leak test can be performed by slowly increasing the airway pressure and listening with a stethoscope over the larynx to hear when a leak develops. An air leak within this pressure range allows for adequate ventilation and reduces the incidence of postintubation croup. The most common cause of postintubation croup is a tight-fitting endotracheal tube without a leak at 30 to 40 cm H2O (Davis: Smith’s Anesthesia for Infants and Children, ed 8, pp 356–357, 389–390).
582. A premature newborn delivered at 32 weeks of gestation is brought to the OR for repair of a left-sided congenital diaphragmatic hernia (CDH). After an awake tracheal intubation, general anesthesia is maintained with sevoflurane, O2, and fentanyl. Shortly thereafter, the anesthesiologist notes significant difficulty with adequate ventilation. The Sao2 subsequently falls to 65%, and the heart rate decreases to 50 beats/min. What would be the most appropriate step to take at this time? A. Pull the endotracheal tube from the right mainstem bronchus B. Ventilate with positive end-expiratory pressure (PEEP) and administer furosemide C. Place a chest tube on the right side after confirming a tension pneumothorax D. Pull out the endotracheal tube, mask ventilate, and re-intubate the patient
582. (C) A congenital diaphragmatic hernia (CDH) is the herniation of abdominal viscera into the chest cavity through a defect in the diaphragm and occurs in approximately 1 in every 3000 live births. Most CDHs occur through a defect in the left side of the diaphragm and produce the classic triad of dyspnea, cyanosis, and apparent dextrocardia. Symptoms depend upon the degree of herniation and the amount of respiratory compromise. Some newborns show significant respiratory compromise in the delivery room, whereas others deteriorate hours later. If ventilation is needed, intubation is preferred over mask ventilation (mask ventilation may push some gas into the stomach, increasing respiratory compromise). Usually, immediate intubation of the trachea with gentle respiratory support is needed, but occasionally intubation is performed later in the OR. Oral or nasogastric tubes are placed early to prevent gastric distention and worsening respiratory compromise. Because CDH is associated with hypoplastic lungs, current ventilatory support aims at maintaining a preductal oxygen saturation of 90% to 95%, using low airway pressures and allowing for moderate permissive hypercarbia (Paco2 of 60-65 mm Hg). If the patient experiences sudden oxygen desaturation during positive-pressure ventilation, a tension pneumothorax should be suspected (usually on the contralateral side to the CDH) and if confirmed, a chest tube should be placed. Despite intensive treatments, about 40% to 50% of these newborns will die in the newborn period. At one time, these patients were rushed to the OR; now they are usually stabilized (sometimes for 5-15 days) and more electively taken to the OR. Also see explanation for Question 626 (Davis: Smith’s Anesthesia for Infants and Children, ed 8, pp 567–574; Hines: Stoelting’s Anesthesia and Co-Existing Disease, ed 6, pp 594–596; Miller: Miller’s Anesthesia, ed 8, pp 2792–2793).
583. An 8-year-old boy found at the site of a motor vehicle accident (MVA) has arrived in the OR for exploratory laparotomy. He has not received any sedation or pain medication because he appeared “confused and did not seem bothered.” He is tachycardic, with thready distal pulses and cold extremities. In spite of a 500-mL fluid bolus, the patient has produced minimal urine. What is the approximate percentage of blood volume loss in this patient? A. <20% B. 25% C. 40% D. Cannot determine Pediatric Physiology and Anesthesia 157
583. (B) Unlike adults, children maintain stable hemodynamics until reaching a 25% to 35% loss of their circulating blood volume. This is thought to be related to their high sympathetic tone that produces profound peripheral vasoconstriction in an effort to maintain blood pressure. There are, however, clinical signs that herald incipient shock before blood pressure changes. He is most likely approximately 25% depleted and not in the less than 20% range, because he is confused and lethargic and not just anxious with normal mentation (<20%). His renal status is oliguric (25% loss) instead of anuric, which would correspond to 40% blood volume depletion (Davis: Smith’s Anesthesia for Infants and Children, ed 8, pp 980–981).
584. In a 6-year-old child, the length of an oral endotracheal tube (from the alveolar ridge to the midtrachea) most often is A. 10 cm B. 13 cm C. 15 cm D. 18 cm
584. (C) The depth of insertion of an oral endotracheal tube from the alveolar ridge to the midtracheal level is approximately 7 cm for a 1-kg newborn, 8 cm for a 2-kg newborn, 9 cm for a 3-kg newborn, and 10 cm for a typical 3.5-kg term newborn. There are many ways to estimate the appropriate depth of insertion of an oral endotracheal tube (in centimeters) for infants and children. One method is using age (e.g., >3 years): (Age in years)/2 + 12 = tube length inserted In this 6-year-old child: 6/2 + 12 = 15 cm Another way is to multiply the internal diameter (ID) size of the endotracheal tube by 3. For example, when you use a 5.0 ID size endotracheal tube, insert the tube about 15 cm. When using a cuffed endotracheal tube, the cuff should be visualized as just passing the vocal cords. If an uncuffed endotracheal tube is used, the tube is inserted to the first or second line on the tube at the level of the vocal cords (Davis: Smith’s Anesthesia for Infants and Children, ed 8, p 356). Pediatric Physiology and Anesthesia 167
585. Which of the following is the most suitable replacement fluid for a 3-year-old, 14-kg child undergoing repair of clubfeet? A. D5W B. D5 ½NS C. Normal saline D. Lactated Ringer solution
585. (D) In elective cases, intravenous fluids are administered to replace fluid deficits from preoperative fasting, to maintain maintenance fluid requirements, and to replace ongoing fluid losses from the surgical procedure. In emergency cases, fluid may also be needed to restore intravascular volume, if hypovolemia occurs from the emergency condition. Maintenance fluid requirements follow the 4:2:1 rule, where 4 mL/kg is administered for the first 10 kg of weight, 2 mL/kg for the next 10 kg of weight, and 1 mL/kg for any weight over 20 kg. Thus, for this 14- kg child, the deficit is calculated to be [(4 mL × 10 kg) + (2 mL × 4 kg)] per hour × 10 hours = 480 mL. This is probably a slight overestimate given that the fasting patient conserves fluid. In general, half of the fluid deficit + the hourly maintenance fluid is administered in the first hour of anesthesia, one fourth of the deficit + maintenance fluids for the second and third hours, then maintenance fluids thereafter + replacement fluids for ongoing losses. Glucose solutions are commonly administered to pediatric patients when the development of hypoglycemia is greatest, namely neonates and any patient who is critically ill or has hepatic dysfunction. Typically, healthy children older than 1 year of age (or >10-kg weight) do not require supplemental glucose during surgery, because their glycogen stores are adequate for the stress of surgery. The two most common isotonic solutions used are lactated Ringer solution and PlasmaLyte A solution. Most would avoid the use of normal saline because there is a risk of developing hyperchloremic metabolic acidosis. Normal saline contains 154 mEq/L of Na+, which causes the kidney to excrete bicarbonate to preserve electrical neutrality (Davis: Smith’s Anesthesia for Infants and Children, ed 8, p 383; Miller: Basics of Anesthesia, ed 6, pp 552–553; Miller: Miller’s Anesthesia, ed 8, pp 2783–2784).
586. An otherwise healthy 14-day-old neonate is transported to the OR well-hydrated for surgery for a bowel obstruction. A rapid-sequence induction is planned. Compared with the adult dose, the dose of succinylcholine administered to this patient should be A. Diminished because of the immature nervous system B. The same as the adult dose C. Decreased because of decreased acetylcholine receptors D. Increased because of a greater volume of distribution
586. (D) Neonates and infants (<2 years of age) require more succinylcholine per body weight than do older children and adults to produce neuromuscular blockade, because the extracellular fluid volume is much greater in neonates and infants. Because the volume of distribution of succinylcholine is greater, the recommended dose of succinylcholine in neonates and infants to provide optimal conditions for tracheal intubation is 2 mg/kg instead of the 1 mg/kg used for adults (Davis: Smith’s Anesthesia for Infants and Children, ed 8, pp 247, 537; Miller: Miller’s Anesthesia, ed 8, p 2771).
587. The most common cause of neonatal bradycardia (heart rate <100 beats/min) in the delivery room is A. Congenital heart disease B. Maternal drug intoxication (narcotics, alcohol, magnesium, barbiturates, digitoxin) C. Postpartum cold stress D. Hypoxemia
587. (D) Heart rates less than 100 beats/min are poorly tolerated in the neonate because of the reduced cardiac output and poor tissue perfusion that develops. Congenital heart disease, such as congenital heart block or congenital heart failure, is rare and can be diagnosed by neonatal electrocardiogram and echocardiogram. Maternal medications during labor and delivery rarely cause bradycardia; however, fetal distress as a result of hypoxia may cause it. Fever as well as maternal administration of β-mimetics (e.g., terbutaline, ritodrine) tend to cause tachycardia. Cold stress of the neonate may lead to hypoxemia, which will promote persistence of the fetal circulation, which is why a neutral thermal environment to minimize heat loss is important. However, the most common cause of neonatal bradycardia in the delivery room is respiratory failure resulting in hypoxia and acidosis. In the OR, bradycardia results from hypoxia, vagal stimulation, and the depressant effects of anesthetic agents (e.g., halothane), which can lead to cardiac arrest (Davis: Smith’s Anesthesia for Infants and Children, ed 8, pp 513–514).
588. A 10-week-old infant born at 31 weeks’ gestation is anesthetized for repair of an inguinal hernia. General anesthesia is induced by mask with sevoflurane, an endotracheal tube is placed, and anesthesia is maintained with sevoflurane and oxygen. What is the best postoperative pain management for this patient? A. Caudal block with 0.25% bupivacaine, 1 mL/kg, and admitted to a pediatric ward for overnight observation B. Caudal block with 0.25% bupivacaine, 2 mL/kg, and admitted to a pediatric ward for overnight observation C. Oral pain medication (acetaminophen) and discharged home D. Fentanyl, 1 mL IV, and admitted to a pediatric ward for overnight observation
588. (A) Apnea spells are defined as cessation of breathing for at least 15 seconds and are often accompanied by bradycardia and/or cyanosis. Infants (especially former premature newborns) younger than 60 weeks’ PCA are at risk for apnea after general anesthesia, although most cases will occur in infants less than 45 weeks’ PCA. These patients should be admitted to the hospital and have at least 12 apnea-free hours of monitoring before discharge. This child was born at 31 weeks’ estimated gestational age and is now 10 weeks old or is 41 weeks’ PCA and needs to be admitted. Of the postoperative analgesia plans listed with overnight observation, answer A is the most appropriate. Answers B and D include analgesic doses that are too high. Anemia (Hct <30) also appears to increase the chances for postoperative apneic spells (Davis: Smith’s Anesthesia for Infants and Children, ed 8, p 388; Miller: Miller’s Anesthesia, ed 8, pp 2793–2794).
589. A 6-year-old, 20-kg girl develops pulseless ventricular tachycardia after induction of general anesthesia for a tonsillectomy. The anesthesiologist intubates the child, administers 100% oxygen, and starts chest compressions. When the biphasic defibrillator quickly arrives in the OR and is attached to the child, the defibrillator should be charged to what energy level for the initial shock? A. 20 joules (J) B. 40 joules (J) C. 60 joules (J) D. 80 joules (J)
589. (B) The treatment for documented ventricular fibrillation or pulseless ventricular tachycardia is electrical defibrillation as soon as possible. Cardiopulmonary resuscitation is performed until the defibrillator arrives and defibrillation is attempted. With manual defibrillators (monophasic or biphasic) the initial dose should be 2 J/kg, increasing to 4 J/kg up to a maximum of 10 J/kg (or adult dose). In this 20-kg child the initial dose is 20 × 2 J/kg = 40 J. Automated external defibrillators (AEDs) can be safely used in children 1 to 8 years of age. When using an AED, it is best to use one with a pediatric attenuator system, which decreases the delivered energy to doses appropriate for children (Davis: Smith’s Anesthesia 168 Part 2 Clinical Sciences for Infants and Children, ed 8, pp 1229–1230; 2010 American Heart Association Guidelines for Cardiopulmonary Resuscitation and Emergency Cardiovascular Care, Circulation 122:S706–S719, 2010).
590. The spinal cord of newborns extends to the A. L1 vertebra B. L2-L3 vertebrae C. L4-L5 vertebrae D. S1 vertebra
590. (B) L1 Conus medullaris Dural sac Adult Infant Conus medullaris Dural sac L2 L3 L4 L5 S1 S2 S3 S4 S5 L1 L2 L3 L4 L5 S1 S2 S3 S4 S5 Common teaching states that the spinal cord of the newborn or infant ends at L3 and the dural sac ends at S3, so lumbar puncture should be performed in these children no higher than the L4-L5 interspace. Recent data using ultrasound suggest that the spinal cord of newborns ends at L2. For the adult, the spinal cord ends at L1 and the dural sac ends at S1 (Davis: Smith’s Anesthesia for Infants and Children, ed 8, pp 463–464).
591. The most common initial symptom of EA and TEF is A. Respiratory distress at delivery (e.g., retractions, tachypnea) B. Projectile vomiting C. Hypoxia D. Regurgitation during feeding
591. (D) EA and TEF may be suspected prenatally when the mother has polyhydramnios; otherwise it is suspected soon after birth when excessive oral secretions, drooling, or coughing are noted and an oral suction catheter cannot be passed into the stomach. Because the passage of an oral gastric tube is not routine in many centers, the first manifestation of EA occurs when the newborn has trouble breathing (e.g., coughing) and regurgitates with the first feeding. After the diagnosis is made, these patients should be placed in the head-up position and the blind upper pouch of the esophagus should be decompressed with a suction tube immediately to reduce pulmonary aspiration of secretions. Other abnormalities associated with EA and TEF include VACTERL (Vertebral abnormalities, imperforate Anus, Congenital heart disease, TracheoEsophageal fistula, Renal abnormalities, Limb abnormalities). See also Question 574 (Davis: Smith’s Anesthesia for Infants and Children, ed 8, pp 574–576; Hines: Stoelting’s Anesthesia and Co-Existing Disease, ed 6, pp 596–597; Miller: Miller’s Anesthesia, ed 8, p 2792).
592. A 4-kg, 3-hour-old newborn with macrosomia and large fontanelles is scheduled for surgical repair of an omphalocele. Physical examination reveals macroglossia but no other anomalies. Which of the following is likely to occur in this patient? A. Hyperkalemia B. Metabolic acidosis C. Hypoxemia D. Hypoglycemia
592. (D) Omphalocele is the external herniation of abdominal viscera through the base of the umbilical cord. It occurs in about 1 of 5000 births. Thirty percent of these newborns will die in the neonatal period, primarily from cardiac defects or prematurity. Some of these newborns with omphalocele have a syndrome called Beckwith- Wiedemann syndrome. This syndrome is characterized by omphalocele, organomegaly, macrosomia, large fontanelles, macroglossia, polycythemia, and hypoglycemia. These patients may be very difficult to intubate because of their significant macroglossia (Hines: Stoelting’s Anesthesia and Co-Existing Disease, ed 6, p 598).
593. Which of the following is the LEAST appropriate technique for induction of general anesthesia in a newborn for surgical repair of TEF? A. Awake tracheal intubation B. Inhalation induction with spontaneous ventilation and tracheal intubation C. Inhalation induction using positive-pressure bag and mask ventilation and tracheal intubation D. Rapid-sequence IV induction and tracheal intubation
593. (C) Anesthesia for patients with EA and TEF can be safely induced with either an intravenous or volatile anesthetic. However, positive-pressure bag and mask ventilation should be avoided because it will force gas into the stomach, potentially making ventilation of the lungs more difficult. A frequently used technique to facilitate correct placement of the endotracheal tube is to advance the tube into a bronchus. While listening over the stomach, slowly withdraw the tube until breath sounds are heard over the stomach. Advance the tube until these sounds become diminished. Bronchoscopy is used by some anesthesiologists to make sure only one fistula is present and to help position the endotracheal tube (Davis: Smith’s Anesthesia for Infants and Children, ed 8, pp 576–577; Hines: Stoelting’s Anesthesia and Co-Existing Disease, ed 6, pp 597–598).
594. A 3-year-old with cough and sore throat, but no fever, is scheduled for tonsillectomy. Physical examination reveals minimal inspiratory wheezing. Chest x-ray reveals small left lower lobe (LLL) infiltrate. The best course of action would be A. Administer IV steroids and proceed B. Delay for 10 to 14 days and treat with oral antibiotics C. Postpone surgery for at least 1 month D. Proceed 158 Part 2 Clinical Sciences
594. (C) This child most likely has a lower respiratory infection. The planned procedure should be delayed for a period of 4 to 6 weeks. This child may have early manifestations of pneumonia with the LLL infiltrate and should be evaluated by a pediatrician. Without a physical assessment, simply starting oral antibiotic therapy would be ill advised. Pediatric Physiology and Anesthesia 169 The specific time to reschedule surgery for children with upper respiratory infections (URIs) is not absolute. Generally acceptable guidelines for postponement of elective surgeries for these patients suggest 1 to 2 weeks after recovery from the acute illness. Manifestation of URI include (1) mildly sore or scratchy throat; (2) change in feeding or level of activity; (3) cough or sneezing; (4) rhinorrhea (new or change in consistency); (5) nasal congestion; (6) fever higher than 101° F (38.8° C); and (7) inflamed throat or hoarse voice. The presence of these signs and symptoms increases the likelihood of postoperative airway complications and may necessitate an overnight admission. Children with preexisting reactive airway disease, regardless of etiology, who develop URI are at higher risk of postoperative complications, and the threshold for postponing surgery should be even lower than for similar patients without comorbidities (Davis: Smith’s Anesthesia for Infants and Children, ed 8, pp 1112–1114: Miller: Basics of Anesthesia, ed 5, p 555).
595. The predicted blood volume in a 4-kg neonate is A. 240 mL B. 280 mL C. 340 mL D. 400 mL
595. (C) The estimated blood volume (EBV) of healthy full-term neonates is approximately 80 to 90 mL/kg. For this 4-kg neonate, the volume is 320 to 360 mL. Premature newborns have an EBV of 90 to 100 mL/ kg, whereas the 3- to 12-month-old infant has an EBV of 75 to 80 mL/kg (Davis: Smith’s Anesthesia for Infants and Children, ed 8, p 409).
596. The pulmonary vascular resistance in newborns decreases to that of adults by age A. 1 to 2 days B. 1 to 2 weeks C. 1 to 2 months D. 1 year
596. (C) In the fetus, pulmonary vascular resistance is extremely high. In utero, most of the right ventricular output bypasses the lungs and flows into the descending aorta through the ductus arteriosus. With the onset of ventilation at birth the pulmonary vascular resistance suddenly decreases, enabling blood to flow more easily through the lungs. Pulmonary vascular resistance continues to decrease after birth, reaching adult levels by 1 to 2 months of life. This is when pulmonary overcirculation might occur and result in pulmonary edema and eventual failure. The increase in Pao2 not only acts as a pulmonary artery vasodilator (along with the lowering of the Paco2) but also acts as a vasoconstrictor to the ductus arteriosus (thus further assisting the change from the fetal to the adult circulation) (Davis: Smith’s Anesthesia for Infants and Children, ed 8, pp 86–87, 519).
597. A 10-month-old infant is undergoing elective repair of a left testicular hydrocele under general anesthesia with isoflurane, nitrous oxide, oxygen, and fentanyl. All of the following are effective and reasonable means of preventing hypothermia in this patient EXCEPT A. Placement of an infrared heater over the operating table and prewarming the OR B. Covering the OR table with a heating blanket C. Wrapping the extremities with sheet wadding and covering the head with a cloth cap D. Ventilating the patient with a Mapleson D circuit at low gas flows (e.g., 50 mL/kg/min)
597. (D) A comprehensive understanding of thermoregulation and a meticulous attention to detail during the anesthetic care of infants are both necessary to minimize intraoperative heat loss. In anesthetized infants, heat loss occurs through the transfer of heat from the patient to the environment in one of four ways: radiation (transfer between objects not in contact), conduction (transfer between objects in contact), convection (transfer to moving molecules such as air and fluid), and evaporation. Of these, radiation and convection account for about 75% of the infant’s heat loss. For this reason, placement of an infrared heater over the OR table and prewarming the OR atmosphere are the most effective means of preventing hypothermia in these patients. Covering the OR table with a heating blanket; ventilating the patient with warm, humidified anesthetic gases; wrapping the extremities of the patient with sheet wadding; and covering the patient’s head with a cloth or plastic cap can also reduce heat loss and prevent hypothermia. Convective forced-air warmers can help prevent a decrease in body temperature and also have been effective in rewarming hypothermic patients. A Mapleson D breathing circuit is not a circle system and does not preserve heat or moisture. To prevent rebreathing of expired gases, spontaneous breathing flow rates need to be two to three times the minute ventilation, and for controlled ventilation fresh gas flows need to be greater than 90 mL/kg/min. Low flows such as 50 mL/kg/min with Mapleson circuits are inadequate and will result in respiratory acidosis (Davis: Smith’s Anesthesia for Infants and Children, ed 8, pp 162–165, 294–296; Miller: Miller’s Anesthesia, ed 8, pp 1627–1628).
598. Central postoperative depression of ventilation in a full-term neonate is MOST likely to occur after surgery for which of the following? A. Gastroschisis B. Omphalocele C. Tracheoesophageal fistula D. Pyloric stenosis
598. (D) Although all of these conditions can produce ventilatory depression in the postoperative period, only pyloric stenosis produces central nervous system (CNS) depression of respiration. Patients with pyloric stenosis have protracted vomiting that leads to dehydration, hypokalemia, hyponatremia, hypochloremia, and metabolic alkalosis. Postoperative ventilatory depression frequently occurs in infants with pyloric stenosis, thought to be related to cerebrospinal fluid (CSF) alkalosis that is worsened by intraoperative hyperventilation of the lungs. Thus, these patients should be fully awake with a normal rate and pattern of respiration before extubation is considered. This is one reason infants with pyloric stenosis should be stabilized and hydrated before coming to the OR. The other conditions listed can lead to mechanical, not central, causes of respiratory difficulty in the postoperative period (Davis: Smith’s Anesthesia for Infants and Children, ed 8, pp 750–751; Hines: Stoelting’s Anesthesia and Co-Existing Disease, ed 6, pp 600–601). 170 Part 2 Clinical Sciences
599. A premature male neonate born at 34 weeks of gestation is scheduled to undergo emergency repair of a left-sided diaphragmatic hernia. Which of the following vessels could be cannulated for preductal arterial blood sampling? A. Femoral artery B. Umbilical artery C. Right radial artery D. Left radial artery
599. (C) Newborns with diaphragmatic hernia have significant respiratory difficulty. In addition to their hypoplastic lungs, persistent pulmonary hypertension is present, producing right-to-left shunting through the patent ductus arteriosus. To more appropriately administer the anesthetic, a preductal (ductus arteriosus) artery should be cannulated to monitor arterial blood gases and blood pressure. The right radial or temporal arteries arise from vessels that originate from the aorta proximal to the ductus arteriosus. The oxygen saturation monitors should be placed on the right arm as well (Hines: Stoelting’s Anesthesia and Co-Existing Disease, ed 6, pp 595–596 ).
600. In which of the following patients would the minimum alveolar concentration (MAC) for isoflurane be the greatest? A. A premature infant 30 weeks’ PCA B. Full-term neonate C. 3-month-old infant D. 19-year-old man with hyperthyroidism
600. (C) The MAC for isoflurane is greatest at age 3 months. The MAC is lower in preterm neonates compared with term neonates. The low MAC in the newborns may be related to the immaturity of the CNS and/or related to the elevated levels of progesterone and β-endorphins. The increase in MAC in the first few weeks after birth seems to be related to the falling progesterone levels. After age 3 months, the MAC of these volatile anesthetics steadily declines with aging except for a slight increase at puberty. For reasons that are unclear, the MAC for sevoflurane is similar in neonates and infants younger than 1 year (3.2%). The MAC of sevoflurane then decreases with age (1 to 12 years, 2.5%; 40-year-old, 2%) (Davis: Smith’s Anesthesia for Infants and Children, ed 8, pp 190, 556; Hines: Stoelting’s Anesthesia and Co-Existing Disease, ed 6, p 587 ).
601. A 40-kg, 10-year-old child sustains a thermal injury to his legs, buttocks, and back. The estimated area involved is 50%. Using only crystalloid fluids, how much fluid should be administered during the first 24 hours after the burn? A. 2.5 L B. 5.5 L C. 8.0 L D. 10.0 L
601. (C) Several formulas (none ideal) have been used as a guide for initial fluid resuscitation in burn injuries. Intravascular fluid-volume deficits in patients with burn injuries are roughly proportional to the extent and depth of the burn. The Parkland formula, more recently renamed the Consensus formula, is perhaps the most commonly used formula. This formula estimates fluid needs to be 4 mL/kg of crystalloid for each percent of body surface area burned. Thus, in this case: 4 × 40 (kg) × 50 (%) = 8000 mL. Approximately two thirds of this fluid should be replaced with isotonic crystalloid solutions during the first 8 hours after the injury, the rest over the next 16 hours. This estimate is modified clinically by the patient’s clinical response as noted by the vital signs and urine output (target urine output of 0.5 mL/kg/ hr) (Davis: Smith’s Anesthesia for Infants and Children, ed 8, pp 1017–1018; Miller: Basics of Anesthesia, ed 6, p 685 ).
602. An otherwise healthy 3-month-old black female infant with a hemoglobin of 19 mg/dL at birth presents for elective repair of an inguinal hernia. Her preoperative hemoglobin is 10 mg/dL. Her father has a history of polycystic kidney disease. The most likely explanation for this patient’s anemia is A. Sickle cell anemia B. Iron deficiency C. Undiagnosed polycystic kidney disease D. It is a normal finding
602. (D) The most likely explanation for the “falling” hemoglobin level in this patient is that this is a normal physiologic finding. At birth, a full-term infant has a hemoglobin level of approximately 15 to 20 g/dL. A physiologic anemia occurs by age 2 to 3 months, resulting in hemoglobin concentrations of approximately 10 to 11 g/dL. After 3 months, there is a progressive increase in hemoglobin concentration, which reaches levels similar to that of adults by age 6 to 9 months. For premature infants, the anemia is more pronounced (often to as low as 8.0 g/dL), occurs earlier, and persists longer (Davis: Smith’s Anesthesia for Infants and Children, ed 8, pp 398–399).
603. The anesthesiologist is called to the emergency room by the pediatrician to help manage a 3-year-old boy with a high fever and upper airway obstruction. His mother stated that earlier that afternoon, he complained of a sore throat and hoarseness. The patient is sitting erect and leaning forward; has inspiratory stridor, tachypnea, and sternal retractions; and is drooling. Which of the following is the MOST appropriate management of airway obstruction in this patient? A. Aerosolized racemic epinephrine B. Awake tracheal intubation in the emergency room or the OR if time permits C. Transfer to the OR, inhalation induction, and tracheal intubation D. Transfer to the OR, IV induction, paralysis with succinylcholine, and tracheal intubation
603. (C) This history is consistent with an acute life-threatening cause of upper airway obstruction called epiglottitis (or more appropriately supraglottitis because other supraglottic structures are involved as well). In the past, more than 75% of cases were caused by Haemophilus influenza type b (Hib). With widespread immunization against H. influenzae, this condition has become much less frequent and the causes now include Haemophilus parainfluenzae, group A streptococci, pneumococci, and staphylococci. This condition is a medical emergency that usually starts out as a severe sore throat and rapidly progresses to the “four Ds” (dysphagia, dysphonia, dyspnea, and drooling). It can progress rapidly and cause death within 6 to 12 hours after the onset of symptoms. The child typically is seen sitting up, appears dyspneic with the mouth open, is drooling, and has a high fever and tachycardia. Inspiratory stridor is a late finding and suggests impending complete upper airway obstruction. When suspected, the anesthesiologist and otolaryngologist should be notified and the child immediately transferred to the OR (with the parent if appropriate) before complete upper airway obstruction ensues. In the OR, anesthesia should be induced with sevoflurane and oxygen with the child in a sitting position. Sevoflurane is less likely to induce laryngospasm than isoflurane or desflurane. IV access should be established as soon as the child is deeply anesthetized. Atropine (0.02 mg/kg) should be administered to block vagally mediated bradycardia induced by direct laryngoscopy. Muscle relaxants are contraindicated because they can cause complete obstruction of the upper airway in these patients. The trachea should be intubated under direct laryngoscopy when the depth of anesthesia is sufficient to blunt laryngeal reflexes. Also see explanation for Question 619 (Davis: Smith’s Anesthesia for Infants and Children, ed 8, pp 811–813; Hines: Stoelting’s Anesthesia and Co-Existing Disease, ed 6, pp 614–615). Pediatric Physiology and Anesthesia 171
604. A 2-year-old child with cerebral palsy and known severe gastroesophageal reflux (with frequent nightly aspiration) and a seizure disorder is scheduled to undergo iliopsoas release under general anesthesia. Which of the following would be the preferred technique for inducing general anesthesia in this patient? A. Inhalation induction with sevoflurane followed by tracheal intubation B. IV induction with propofol followed by laryngeal mask airway C. IV induction with etomidate and vecuronium followed by tracheal intubation D. Rapid-sequence induction with propofol and succinylcholine followed by tracheal intubation Pediatric Physiology and Anesthesia 159
604. (D) Cerebral palsy is a CNS symptom complex. The most common clinical manifestation is skeletal muscle spasticity. It is usually classified according to the extremity affected (e.g., monoplegia, hemiplegia, diplegia, or quadriplegia) and the characteristics of the neurologic dysfunction (spastic, hypotonic, dystonic, athetotic). Other manifestations include cerebellar ataxia, seizure disorders, varying degrees of mental retardation, and speech deficits. Gastroesophageal reflux is also common. For this reason, the preferred induction of general anesthesia in these patients should include a rapid-sequence IV induction with propofol followed by immediate tracheal intubation. Etomidate, ketamine, and methohexital are proconvulsants in patients with underlying seizure disorders and should probably be avoided. Even though these patients have skeletal muscle spasticity, there have been no reports of succinylcholine-induced hyperkalemia. The response to nondepolarizing muscle relaxants is normal in most reports; however, some have reported resistance to nondepolarizing muscle relaxants. For a rapid-sequence induction, succinylcholine is faster than vecuronium. Rocuronium could also be used due to its rapid onset (Davis: Smith’s Anesthesia for Infants and Children, ed 8, pp 863–865; Hines: Stoelting’s Anesthesia and Co-Existing Disease, ed 6, p 605).
605. A 7-week-old male infant is admitted to the pediatric intensive care unit (ICU) with a bowel obstruction. His laboratory values are sodium 120 mEq/L, chloride 85 mEq/L, glucose 85 mg/dL, and potassium 2.0 mEq/L. Respiratory rate is 20 breaths/min, and according to the patient’s mother, urine output has been 0 for the last 4 hours. The most appropriate fluid for resuscitation of this patient would be A. D2.5W with 0.45 sodium chloride and 20 mEq/L potassium chloride B. 0.45% sodium chloride C. 0.9% sodium chloride with 30 mEq/L potassium chloride D. 0.9% sodium chloride
605. (D) The symptoms described in this patient are consistent with severe dehydration. Thus, the vascular volume should be expanded initially with an isotonic saline solution or a colloid solution until the patient voids. When the urine output increases, potassium can be added to the IV fluids. Although glucose administration for long procedures may prevent hypoglycemia, D5W alone or with a crystalloid solution should not be used to replace fluid deficits (Davis: Smith’s Anesthesia for Infants and Children, ed 8, pp 126–129).
606. A 12-hour-old, 1800-g neonate, 30 weeks’ postgestational age, is noted in the ICU to begin making twitching movements. Blood pressure is 45 mm Hg systolic, blood glucose is 50 mg/dL, and urine output is 5 mL/hr. The O2 saturation on pulse oximeter is 88%. The MOST appropriate course of action to take at this point would be A. Administer calcium gluconate (2 mL of 10% solution) B. Glucose 10 mg IV over 5 minutes (2 mL of D5W) C. Hyperventilate with 100% O2 D. Administer a 20-mL bolus of 5% albumin
606. (A) Preterm infants have very limited calcium reserves and are very susceptible to hypocalcemia. Hypocalcemia (serum ionized calcium level <1.5 mEq/L) manifests itself in a number of nonspecific ways, including irritability, twitching, hypotension, and seizure. A dose of 10 to 20 mg/kg of elemental calcium administered over 5 to 10 minutes will be an appropriate starting dose and may need to be repeated every 6 to 8 hours until the calcium levels stabilize. Bradycardia and occasionally asystole can be seen if it is injected too rapidly. In this infant of 1800 g, the starting dose of 1.8 kg × 10 mg/kg = 18 mg of elemental calcium can be used. Calcium gluconate 10% solution contains about 9 mg/mL of elemental calcium so the dose is 2 mL. Some of the signs of hypoglycemia are similar to those of hypocalcemia and include seizure, irritability, hypotension, and sometimes bradycardia and apnea. In the patient described in this question, the glucose has already been measured at 50 mg/dL, which is acceptable for a preterm infant. An O2 saturation of 88% is also acceptable because the patient is at risk for ROP (i.e., <44 weeks’ PCA). Hyperventilation would cause alkalosis, which would decrease the unbound fraction of calcium and make the patient more susceptible to seizures. Furthermore, calcium binds to albumin, which would further reduce the free calcium. Because the urine output is more than adequate, it is unlikely that the patient needs a fluid bolus to correct hypotension (Hines: Stoelting’s Anesthesia and Co-Existing Disease, ed 6, pp 593–594).
607. A Eutectic Mixture of Local Anesthetics (EMLA) cream is a mixture of which local anesthetics? A. Lidocaine 2.5% and prilocaine 2.5% B. Lidocaine 2.5% and benzocaine 2.5% C. Prilocaine 2% and benzocaine 2% D. Lidocaine 4%
607. (A) EMLA cream contains lidocaine (2.5%) and prilocaine (2.5%). When the 5% EMLA cream is applied to dry intact skin and covered with an occlusive dressing for at least 1 hour, topical anesthesia to a depth of 5 mm is obtained. Four percent lidocaine (ELA-Max) can also be used and requires only 30 minutes to become effective (Davis: Smith’s Anesthesia for Infants and Children, ed 8, p 441).
608. Advantages of catheterization of the umbilical artery versus the umbilical vein in a newborn include all of the following EXCEPT A. It allows assessment of oxygenation B. Hepatic damage from hypertonic infusion is avoided C. It permits assessment of systemic blood pressure D. It is easier to cannulate
608. (D) Although the umbilical vein is larger and easier to cannulate than the umbilical artery, the umbilical vein will not allow for adequate assessment of arterial blood gases or systemic blood pressure. Additionally, administration of drugs or hypertonic solutions into the umbilical vein may be hazardous, because the catheter can become wedged in a portal radicle, possibly leading to hepatic necrosis or portal vein thrombosis. To prevent this, the umbilical vein catheter tip is advanced only 2 to 3 cm into the umbilical vein (to a point where blood can first be aspirated). Careful placement of an umbilical artery catheter is equally important. The tip of the umbilical artery catheter should be placed just above the bifurcation of the aorta and below the level of the renal arteries (L2). All intra-arterial catheters are associated with thrombosis or embolism in these vessels, but fortunately, serious injuries are rare. Because there are two arteries and only one vein, difficulty with one artery nonetheless offers another artery to use (Miller: Miller’s Anesthesia, ed 8, p 2879).
609. The TRUE statement concerning thermoregulation in neonates is which of the following? A. A significant proportion of their heat loss can be accounted for by their small surface area–to-weight ratio B. They compensate for hypothermia by shivering C. The principal method of heat production is metabolism of brown fat D. Heat loss through conduction can be reduced by humidification of inspired gases
609. (C) Because of the large surface area–to-weight ratio, the thin layer of insulating subcutaneous fat, and the limited ability to compensate for cold stress, neonates and infants are at greater risk for intraoperative hypothermia than adults. Infants younger than 3 months do not produce heat by shivering; their principal method of thermogenesis is metabolism of brown fat. Heat loss can occur by radiation, conduction, 172 Part 2 Clinical Sciences convection, and evaporation. Heat loss through evaporation (not conduction) can be reduced by humidification of inspired gases. Heat loss by conduction (not convection) is reduced with the use of a warming blanket (Miller: Miller’s Anesthesia, ed 8, p 2763).

610. Normal values for a healthy 6-month-old, 7-kg infant include A. Hemoglobin 17 g/dL B. Heart rate 90 beats/min C. Respiratory rate 30 breaths/min D. Systolic blood pressure of 60

610. (C) In a 6-month-old infant, a normal hemoglobin value is approximately 11 to 12 g/dL. The normal heart rate is about 100 to 140 beats/min, systolic blood pressure is 70 to 90, and the respiratory rate is about 25 to 35 breaths/min (Miller: Basics of Anesthesia, ed 6, pp 547–550).
611. A 5-year-old child undergoing strabismus surgery under general anesthesia suddenly develops sinus bradycardia and intermittent ventricular escape beats but is hemodynamically stable. Which therapy is appropriate for treating this arrhythmia? A. Tell the surgeon to stop pulling on the eye muscle B. Tell the surgeon to do a retrobulbar block C. Decrease the depth of the volatile anesthetic D. Administer atropine
611. (A) The oculocardiac reflex (OCR) is commonly defined as a 10% to 20% decrease in heart rate that is sustained for more than 5 seconds. It can be induced by traction on extraocular muscles, pressure on the eye, orbital hematoma, ocular trauma, or eye pain. It is commonly seen with strabismus operations and may produce a wide variety of cardiac arrhythmias, including sinus bradycardia, nodal bradycardia, ectopic beats, ventricular fibrillation, and, rarely, asystole (1 in 2200 strabismus operations). The initial treatment of this is to stop the stimulus (i.e., tell the surgeon to stop what he or she is doing). This reflex quickly responds, and future similar stimulation typically elicits less of a response. In many cases no further treatment is necessary. Increasing the depth of general anesthesia may help to block the reflex, as may reassessing the adequacy of ventilation (because hypercarbia and hypoxemia decrease the threshold to elicit the OCR). A retrobulbar block will prevent the reflex. Infiltrating lidocaine locally into the recti muscles may be effective in preventing and treating the OCR. Atropine (0.01-0.02 mg/kg) or glycopyrrolate can be administered intravenously if the arrhythmia persists. Some advocate the prophylactic use of atropine or glycopyrrolate during strabismus surgery, especially in children (Davis: Smith’s Anesthesia for Infants and Children, ed 8, pp 880–888; Miller: Basics of Anesthesia, ed 6, pp 487–488).
612. Which of the following respiratory indices is increased in neonates compared with adults? A. Tidal volume (Vt) (mL/kg) B. Alveolar ventilation (mL/kg/min) C. Functional residual capacity (mL/kg) D. Paco2
612. (B) There is no difference in Vt (mL/kg) between neonates and adults. Neonates have a high O2 consumption (about twice that of adults). To compensate for the increased oxygen demand, alveolar ventilation is increased (also about twice the adult). The increase in alveolar ventilation explains the slightly lower Paco2. Of note, the pH also is slightly lower. The reduced functional residual capacity with the increased O2 consumption places the neonate at an increased risk for hypoxia during general anesthesia if there is any difficulty with ventilation (Hines: Stoelting’s Anesthesia and Co-Existing Disease, ed 6, pp 584–586; Miller: Basics of Anesthesia, ed 6, pp 547–548). PHYSIOLOGIC VARIABLES Neonate Infant 5-Year-Old Adult Weight (kg) 3 4-10 18 70 Respiratory rate (breaths/min) 35 25-30 20-25 15 Alveolar ventilation (mL/kg/min) 130 60 Tidal volume (mL/kg) 6 6 Vital capacity (mL/kg) 35 70 Functional residual capacity (mL/kg) 30 35 Oxygen consumption (mL/kg/min) 6.5 5 4 3-3.5 Carbon dioxide production (mL/kg/min) 6 3 Pao2 (room air, mm Hg) 60-90 80-100 Paco2 (room air, mm Hg) 30-35 35-45 Arterial pH 7.34-7.40 7.35-7.45
613. A 14-year-old girl with neurofibromatosis is anesthetized for resection of an acoustic neuroma. Each of the following may potentially complicate the anesthetic management of this patient EXCEPT A. Presence of a pheochromocytoma B. Upper airway obstruction from a laryngeal neurofibroma C. Intracranial hypertension D. Increased risk for MH
613. (D) Neurofibromatosis (von Recklinghausen disease) is an autosomal dominant genetic disorder characterized by multiple neurofibromas involving the skin, peripheral nervous system, and central nervous system. The clinical features of this disease are diverse and always progress with time. The anesthetic management of patients with neurofibromatosis can be complicated by the associated clinical manifestations of this disease. For example, a pheochromocytoma may be present in approximately 1% of patients. If this goes unrecognized, severe hypertension can occur during anesthesia. Intracranial tumors occur in 5% to 10% of patients, and signs and symptoms of intracranial hypertension may develop. If intracranial pressure is elevated, efforts to reduce intracranial pressure should be initiated. Finally, airway patency may Pediatric Physiology and Anesthesia 173 become compromised by an enlarging laryngeal neurofibroma. Abnormal responses to both depolarizing neuromuscular blocking agents (sensitive or resistant) and nondepolarizing neuromuscular blocking agents (sensitive) have been described. There is no evidence that these patients are at increased risk for MH (Davis: Smith’s Anesthesia for Infants and Children, ed 8, p 843; Hines: Stoelting’s: Anesthesia and Co-Existing Disease, ed 6, pp 244–245).
614. With which of the following congenital anomalies is persistent right-to-left intracardiac shunting of blood MOST likely? A. TEF B. Gastroschisis C. Omphalocele D. CDH
614. (D) The initial decline (from fetal levels) in pulmonary vascular resistance and rise in pulmonary blood flow is dependent on adequate function of the endothelial cells in the pulmonary vasculature. With a CDH, this process does not occur normally. This is caused by pulmonary hypoplasia and pulmonary hypertension. Patients with each of the other anomalies also are at risk for right-to-left intracardiac shunting, but each one can be readily treated, thus avoiding significant shunting and concomitant hypoxemia. In patients with CDH, shunting is exceedingly difficult to manage because of pulmonary hypertension, pulmonary hypoplasia, and endothelial changes (Davis: Smith’s Anesthesia for Infants and Children, ed 8, pp 57 –58).
615. The most reliable method of determining mild dehydration in a child is by the observation of A. Dryness of mucous membrane B. Skin turgor and fontanelles C. Urine output D. Blood pressure
615. (C) The amount of dehydration that children have can be assessed by a variety of observations. For mild dehydration (5% weight loss—fluid deficit of 50 mL/kg) the only abnormal finding of the listed answers is a decrease in urine output less than 2 mL/kg/hr. With moderate dehydration (10% weight loss—fluid deficit of 100 mL/kg) mucous membranes would be dry, skin turgor would be decreased, urine output would be less than 1 mL/kg/hr, the anterior fontanelle would be depressed, and blood pressure would be normal to low. With severe dehydration (15% weight loss—fluid deficit of 150 mL/kg) mucous membranes would be very dry, skin turgor would be greatly decreased, urine output would be less than 0.5 mL/kg/hr, the anterior fontanelle would be markedly depressed, and blood pressure would be reduced and orthostatic (Davis: Smith’s Anesthesia for Infants and Children, ed 8, pp 126–127).
616. Postoperative bleeding following tonsillectomy occurs most commonly A. By the first 6 hours B. 6 to 24 hours after surgery C. On the third postoperative day D. On the seventh postoperative day 160 Part 2 Clinical Sciences
616. (A) Postoperative bleeding after a tonsillectomy occurs in 0.1% to 8% of cases. The bleeding is defined as primary if it occurs within 24 hours and secondary if more than 24 hours after surgery. Primary bleeding tends to be more profuse than secondary bleeding. Secondary bleeding (1 to 10 days postoperatively) occurs when the eschar covering of the tonsillar bed sloughs. Because bleeding most often occurs within the first 6 hours after the surgery (75% of bleeding cases), most outpatient units keep patients for at least 6 to 8 hours after the surgery is completed. The amount of bleeding tends to be underestimated because often a large amount of blood is swallowed (Barash: Clinical Anesthesia, ed 7, pp 1357–1360; Davis: Smith’s Anesthesia for Infants and Children, ed 8, pp 799–800).
617. A 9-year-old undergoing sinus surgery is treated with an unmeasured amount of 0.5% phenylephrine by the surgeon, and the patient develops a blood pressure of 250/150. The most appropriate treatment for this would be A. Administer verapamil B. Administer esmolol C. Administer labetalol D. Administer phentolamine
617. (D) Ear, nose, and throat surgeons often use vasoconstrictors (e.g., phenylephrine, cocaine, or oxymetazoline) to control bleeding in pharyngeal and nasal surgery. For adults, the initial dose of phenylephrine is up to 0.5 mg (4 drops of a 0.25% solution). For children less than 25 kg, the initial dose is up to 20 μg/kg. When excessive doses are used, severe hypertension and cardiovascular decompensation may develop due to the marked increase in peripheral vascular resistance. This also shifts blood from the peripheral site into the pulmonary vasculature (which is less sensitive to vasoconstrictors) and increases left ventricular filling pressure. In this case, the use of labetalol and deepening the anesthesia has been associated with severe pulmonary edema, cardiac arrest, and death. If labetalol or a β-blocker (e.g., esmolol) is used and congestive heart failure develops, consider using high-dose glucagon (5-10 mg) to counteract the loss of cardiac contractility. This may also occur with the use of calcium channel blockers. Baroreceptor-induced bradycardia may not occur in the pediatric patient who has been pretreated with atropine or glycopyrrolate during the anesthetic. The hypertension may be short lived, and deepening the inhalation anesthetic may help; however, treatment of severe hypertension is most effective with direct vasodilators or α-adrenergic receptor antagonists (e.g., phentolamine). The maximum recommended dose for cocaine (usually a 4% solution) is 1.5 to 3 mg/kg with a maximum dose of 200 mg (Groudine et al: New York State Guidelines on the Topical Use of Phenylephrine in the Operating Room, Anesthesiology 92:859–864, 2000; Miller: Miller’s Anesthesia, ed 8, p 2535).
618. A 6-kg, 3-month-old male infant undergoes a left inguinal herniorrhaphy with a spinal anesthetic. Typically, how long would 0.5 mL of a 0.5% bupivacaine solution last? A. Less than 30 minutes B. 30 to 60 minutes C. 60 to 90 minutes D. 90 minutes to 2 hours
618. (C) The total volume of CSF in the newborn is 4 mL/kg compared with the adult’s 2 mL/kg. Infants require higher volumes based on weight compared to adults; the duration of block appears to be shorter. Tetracaine and bupivacaine are the most commonly used drugs for spinal anesthesia in infants. For infants who weigh 5 to 15 kg, a dose of 0.4 mg/kg of 1% tetracaine will last about 80 minutes; 0.4 mg/ kg (or 0.08 mL/kg) of 0.5% bupivacaine would last about 70 to 80 minutes. This child weighs 6 kg, so the dose would be 2.4 mg or 0.48 mL (about 0.5 mL) of 0.5% bupivacaine. For infants less than 5 kg, the dose is larger (i.e., 0.5 mg/kg for tetracaine and 0.5 mg/kg for bupivacaine), and the duration 174 Part 2 Clinical Sciences of the anesthetic is about 5 minutes shorter. If epinephrine is added, the duration of a tetracaine spinal anesthetic is about 30% to 50% longer. Epinephrine added to bupivacaine has little effect (Davis: Smith’s Anesthesia for Infants and Children, ed 8, p 464).
619. In addition to inspiratory stridor, which sign or symptom is consistent with epiglottitis? A. Rapid onset in less than 24 hours B. Mild temperature elevation (<39° C) C. Age younger than 2 years D. Rhinorrhea
619. (A) About 80% of children with inspiratory stridor have croup (laryngotracheobronchitis) and about 5% have epiglottitis (also called acute supraglottitis). All of the answers listed except A, as well as a “barking cough,” refer to signs and symptoms of croup, a viral illness of the subglottic area that usually presents in children younger than 2 years of age. Croup usually presents with a relatively slow onset (over 24-72 hours) and is associated with a mild fever (rarely over 39° C) and lymphocytosis. Patients with acute epiglottitis are usually older, that is, 2 to 6 years of age. The onset of signs and symptoms of acute epiglottitis is typically rapid, that is, less than 24 hours. Patients present with difficulty swallowing, a high fever (often >39° C), and inspiratory stridor. Other signs and symptoms include drooling, lethargy, cyanosis, tachypnea, neutrophilia, and a propensity to sit up and lean forward (in an attempt to maintain their airway). Total upper airway obstruction can occur in these children at any time because of the rapid progression of the disease. For this reason, attempts to visualize the epiglottis should not be undertaken until the patient is in the OR and appropriate preparations are completed for direct laryngoscopy and tracheal intubation, and possible emergency tracheostomy. The definitive treatment of acute epiglottitis includes appropriate antibiotic therapy and a secured airway. Also see explanation to Question 603 (Davis: Smith’s Anesthesia for Infants and Children, ed 8, pp 811–813; Hines: Stoelting’s Anesthesia and Co-Existing Disease, ed 6, pp 614–617 ).
620. Which of the following statements regarding resuscitation of the infant by health care providers is NOT correct? A. Mouth-to-mouth or mouth-to nose ventilation at a rate of 12 to 20 breaths/min is performed when breathing is inadequate but an adequate pulse is present B. Start chest compressions when the pulse is less than 60 beats/min and there are signs of poor tissue perfusion C. Chest compression depth is 1/5 the anteroposterior diameter of the chest (about 1 cm) D. Compression-to-ventilation ratio is 30:2 for oneperson and 15:2 for two-person cardiopulmonary resuscitation (CPR)
620. (C) The technique for CPR of infants (<1 year) and children (ages 1 year to puberty) is different from that of adults. Puberty is defined here as breast development in females and axillary hair in males. More emphasis has been placed on “push hard and push fast,” given that chest compression depth and speed are often inadequate. If only ventilation is needed, the rate for adults is 10 to 12 breaths/min, whereas for children and infants, the rate is 12 to 20 breaths/min. For adults, sternal compressions should be performed with the heel of one hand placed on top of the other hand and compressing the lower half of the sternum at least 5 cm (2 inches). For children, sternal compressions should be performed with the heel of one or two hands compressing the lower half of the sternum at least one third the depth of the chest or about 5 cm (2 inches). For infants, lone rescuers compress the sternum with two fingers placed just below the intermammary line. Sternal compressions are performed at least one third the depth of the chest or about 4 cm (1.5 inches). When two rescuers are present, compressions are performed by encircling the infant’s chest with both hands and placing the thumbs together over the lower third of the sternum. The compression rate is the same for adults, children, and infants, that is, approximately 100 compressions/min. Lay rescuers should not check for pulses in infants or children because they often feel a pulse that is not present. When health care providers palpate for pulses, the brachial artery is preferred in the infant and the carotid or femoral is preferred in the child. A universal compression-to- ventilation ratio of 30:2 is used for infants, children, and adults by single rescuers; a rate of 15:2 is used for two-person infant CPR. Earlier specifications called for a 5:1 ratio, but more recent evidence shows 30:2 to be more effective. For newborns, however, a ratio of 3:1 (90 compressions and 30 ventilations/min) is used (2010 American Heart Association Guidelines for Cardiopulmonary Resuscitation and Emergency Cardiovascular Care, Circulation 122:S685–S705, S862–S875, 2010).
621. All of the following are true statements concerning physiology of newborns compared with that of adults EXCEPT A. Newborns have a greater percentage of total body water compared with adults B. Newborns have a higher glomerular filtration rate (GFR) than adults C. Newborns’ hearts are relatively noncompliant compared with adults D. Newborns’ diaphragms have a lower proportion of type I muscle fibers (i.e., fatigue resistant, highly oxidative fibers)
621. (B) Body composition changes dramatically during the first year of life. Total body water is about 80% for a term newborn compared with 55% for an adult woman and 60% for an adult man. Drugs that are water soluble (such as many antibiotics) will need to have higher mg/kg dose to achieve the desired blood concentrations. With the corresponding lower fat content of the preterm newborn (<5%) and term newborn (10%) compared with the adult (15+%), fat-soluble drugs that depend on redistribution will have a longer clinical effect. The GFR of newborns is low at birth and doubles or triples over the first 3 months of life, with a slower rise until adult values are reached by 1 to 2 years of age. This decrease in renal function can delay excretion of drugs that are dependent on renal clearance for elimination. The relatively noncompliant heart of a newborn gives it a limited capacity to deal with a volume load, compared with the adult. The preterm newborn has 10%, the term newborn has 25%, and the adult has 55% of type I muscle fibers (i.e., fatigue resistant, highly oxidative fibers). The lower proportion of type I fibers predisposes the newborn’s primary respiratory muscle fibers to fatigue (Miller: Miller’s Anesthesia, ed 8, pp 2763–2765; Miller: Basics of Anesthesia, ed 6, pp 547–551). Pediatric Physiology and Anesthesia 175
622. Which of the following statements concerning the anatomy of the infant and the adult airway is NOT true? A. An infant’s tongue is relatively large in relation to the oropharynx compared with an adult’s B. The larynx is in a more cephalic position in infants than in adults C. The vocal cords are in a more horizontal position within the larynx in infants than in adults D. The narrowest part of the infant and adult larynx is at the level of the cricoid cartilage
622. (C) Thyroid cartilage Cricoid cartilage Adult A Infant B P A P A The anatomy of the infant’s airway is different in some respects from that of the adult. The narrowest part of the adult larynx and the infant airway has been reevaluated to be similar (i.e., the narrowest part is at the level of the cricoid ring). The infant head and tongue are relatively larger than the adult. The larynx is more cephalic in the infant than in the adult (infant’s C3-C4, adult’s C4-C5) making straight blades more useful than curved blades for laryngoscopy in infants. The infant’s epiglottis is short, stubby, and “omega” shaped. The vocal cords in the infant are in a diagonal (not horizontal) position within the larynx. The diagonal position makes it more likely to have the endotracheal tube lodge in the anterior commissure rather than slide down the trachea (Davis: Smith’s Anesthesia for Infants and Children, ed 8, p 351; Miller: Miller’s Anesthesia, ed 8, p 2761).
623. Which of the following operations would be associated with the LEAST incidence of postoperative nausea and vomiting (PONV) in a 5-year-old boy? A. Tonsillectomy B. Strabismus surgery C. Myringotomy tube placement D. Orchiopexy
623. (C) Children at the highest risk for PONV include those whose surgery lasts more than 30 minutes, those older than 3 years of age, those with a family or patient history of PONV, those undergoing strabismus surgery, and, finally, cases where narcotics are routinely needed, such as tonsillectomy, orchiopexy and herniorrhaphy. Brief procedures with minimal pain such as myringotomy tube placements have a low incidence of PONV. In cases where PONV is likely, prophylaxis is recommended (Davis: Smith’s Anesthesia for Infants and Children, ed 8, pp 1074–1075, 1285).
624. Anomalies and features associated with Down syndrome include A. Smaller tracheas B. Atlanto-occipital instability C. Thyroid hypofunction D. All of the above
624. (D) Down syndrome (trisomy 21) occurs in 1 in 600 to 800 live births. More than half of trisomy conceptions spontaneously abort. Although variable degrees of mental retardation are common, many other significant conditions are present. Congenital cardiac lesions are seen in about half of these patients (complete AV canal, VSDs, patent ductus arteriosus, ASDs, tetralogy of Fallot) and commonly necessitate prophylactic antibiotics. Other findings include hearing loss, short neck, small mouth, narrow nasopharynx, large tongue, thyroid hypofunction (50%), atlanto-occipital instability (15%-20%, which is most often asymptomatic), and smaller airways. Despite these abnormalities, tracheal intubation usually is not difficult in the hands of an experienced anesthesiologist. The size of the endotracheal tube used to create an air “leak” with increasing airway pressure should be one to two sizes smaller because of the smaller trachea. For example, in children age 18 months to 8 years the endotracheal tube size is 1 mm smaller (Davis: Smith’s Anesthesia for Infants and Children, ed 8, pp 1172–1174; Hines: Stoelting’s Anesthesia and Co-Existing Disease, ed 6, pp 634–635; Miller: Miller’s Anesthesia, ed 8, p 1201; Shott: Down syndrome: analysis of airway size and a guide for appropriate intubation, Laryngoscope 110:585–592, 2000).
625. Congenital syndromes frequently associated with cardiac abnormalities include all of the following EXCEPT A. TEF B. Meningomyelocele C. Omphalocele D. Gastroschisis
625. (D) Congenital cardiac abnormalities frequently occur in association with CDHs, TEFs, meningomyeloceles, and omphaloceles. Gastroschisis is rarely associated with other congenital anomalies (Hines: Stoelting’s Anesthesia and Co-Existing Disease, ed 6, pp 594–599, 608–609).
626. Appropriate management of a neonate born with CDH should include A. Insertion of an orogastric tube B. Expansion of the hypoplastic lung with positivepressure ventilation C. Hyperventilation to keep the Paco2 below 40 and pH greater than 7.40 D. Rapid transport to the OR for surgical correction
626. (A) Newborns with CDH often present with respiratory distress immediately after birth. They often have a flat (scaphoid) abdomen because some of the intestines herniate into the chest and are, therefore, not in 176 Part 2 Clinical Sciences the abdomen. Immediate care includes endotracheal intubation for ventilatory support in infants with respiratory distress and the placement of an orogastric or nasogastric tube to evacuate the stomach. Ventilation of the lungs with a bag and mask may cause more respiratory compromise by producing gastric and intestinal distention and is relatively contraindicated. When ventilating the newborn with an endotracheal tube, you must remember not to try to expand the lungs to normal size because the lungs are hypoplastic and prone to rupturing and producing a pneumothorax. Although at one time hyperventilation was recommended, more recently better outcomes have been found when moderate permissive hypercarbia has been used (Paco2 60-65 mm Hg range). Associated congenital anomalies include CNS anomalies (e.g., spina bifida, hydrocephalus, anencephaly), cardiovascular (e.g., hypoplastic left heart syndrome, ASDs and VSDs, coarctation, tetralogy of Fallot), gastrointestinal (e.g., malrotation, atresia), and genitourinary (e.g., hypospadias). Rushing the child to the OR does not increase survival. It appears better to stabilize the child and look for associated congenital anomalies (seen in up to 50% of these children) before proceeding with surgery. See explanation for Question 582 (Davis: Smith’s Anesthesia for Infants and Children, ed 8, pp 567–574; Hines: Stoelting’s Anesthesia and Co-Existing Disease, ed 6, pp 594–596).
627. Factors associated with an increased incidence of laryngospasm include all of the following EXCEPT A. Age older than 5 years B. Presence of an airway anomaly C. Presence of an active upper respiratory infection (URI) D. Use of a laryngeal mask airway Pediatric Physiology and Anesthesia 161
627. (A) Respiratory adverse events are common in children undergoing noncardiac surgeries. They are more often associated with younger children (<5 years of age compared with >5 years of age), in children with a recent URI (although a few studies question this), in children with airway anomalies (e.g., cleft palate, subglottic stenosis, Pierre Robin syndrome), and when a laryngeal mask airway is used, compared with the use of mask anesthesia (Davis: Smith’s Anesthesia for Infants and Children, ed 7, pp 1161–1162; Flick et al: Risk factors for laryngospasm in children during general anesthesia, Pediatric Anesthesia 18:289–296, 2008).
628. Which of the following statements regarding perioperative cardiac arrest in children is NOT correct? A. Cardiac arrest is more common in neonates than infants or older children B. “Equipment related” causes occur in more than 25% of cardiac arrests C. Resuscitation is more often successful if the cause is anesthesia-related rather than nonanesthesia related D. Emergency surgery is associated with greater than five times the chance of a cardiac arrest
628. (B) Perioperative cardiac arrest is often defined as the need for CPR during anesthesia care (OR and PACU). It is more than four times more frequent in neonates (0-30 days) than infants or children. Causes of cardiac arrest vary from study to study and often are medication-related (e.g., inhalation or intravenous overdosage, succinylcholine-induced dysrhythmia, medication “swaps,” high spinal anesthesia, local anesthetic toxicity, opioid-induced respiratory depression, inadequate reversal of muscle relaxants), cardiovascular- related (e.g., hemorrhage, hyperkalemia, vagal reflexes, embolism, sepsis), respiratory-related (e.g., inadequate ventilation, loss of the airway, aspiration, pneumothorax), and equipment-related (e.g., disconnects, stuck valves). Of these, equipment-related causes are relatively rare (about 4% of cases). About 90% of cardiac arrests due to anesthesia-related episodes are reversed (adequate native heartbeat and blood pressure for at least 20 minutes after the arrest). If the cardiac arrest is not anesthesia-related, outcome is worse (of these, only about 50%-60% are reversed). Regardless of surgical procedure, children with congenital heart disease have a greater chance of a cardiac arrest. Emergency surgery is associated with a six times greater incidence of cardiac arrest than elective surgery (Davis: Smith’s Anesthesia for Infants and Children, ed 8, pp 1201–1205; Flick et al: Perioperative cardiac arrests in children between 1988 and 2005 at a tertiary referral center, Anesthesiology 106:226–237, 2007).
629. Which of the following represents the greatest risk for postoperative apnea in an infant? A. PCA of 60 weeks B. Hemoglobin 10 g/dL C. Recovery in the postanesthesia care unit (PACU) after pyloric stenosis repair D. 20th weight percentile on growth chart
629. (C) After surgical repair of pyloric stenosis, prolonged emergence from anesthesia is not uncommon even with minimal narcotic administration. It is thought that these patients require very little opioid analgesia, because of perturbations in CSF pH, a consequence of prolonged and persistent emesis. Loss of hydrochloric acid from the stomach produces a metabolic alkalosis with concomitant CSF alkalosis. Even after correction of the serum alkalosis, the pH in the CSF could still be high because equilibration with the blood may not have been achieved. Term infants younger than 44 weeks are high risk for postoperative apnea and, therefore, cannot be anesthetized as an outpatient. By contrast, premature infants older than 60 weeks are at a much lower risk for postoperative apnea and, therefore, can be anesthetized as outpatients if discharge criteria are otherwise met. At 8 to 12 weeks of age, the hemoglobin reaches the physiologic nadir of 10 to 11 g/dL. As the transition from Hgb F to Hgb A occurs, the infants experience the so-called physiologic anemia of infancy. The relationship between hemoglobin concentration and apnea of prematurity is controversial. It is not clear what hemoglobin level would place an otherwise healthy infant at risk for apnea (Davis: Smith’s Anesthesia for Infants and Children, ed 8, pp 36, 398, 750–751).
630. Which of the following statements regarding the Mapleson D breathing circuit is FALSE? A. It has a proximal fresh gas inflow and a distal overflow valve B. With an inspiratory-to-expiratory (I:E) breathing ratio of 1:2, rebreathing is eliminated with spontaneous ventilation when the fresh gas flow is three times the minute ventilation C. To eliminate rebreathing, higher fresh gas flows are needed with controlled ventilation than with spontaneous ventilation D. The Mapleson D circuit is the most widely used of the Mapleson circuits for pediatric anesthesia
630. (C) All of the statements are correct except for C. The Mapleson D system is a semiopen anesthetic system with a proximal fresh gas inflow and a distal overflow valve. To eliminate rebreathing, higher fresh gas flows are needed with spontaneous ventilation than with controlled ventilation. Note the Mapleson A system is a semiopen anesthetic system with a distal fresh gas flow and a proximal overflow valve (Davis: Smith’s Anesthesia for Infants and Children, ed 8, pp 294–297 ). Pediatric Physiology and Anesthesia 177
631. Which of the following is LEAST likely to reduce the incidence of postoperative apnea in preterm infants undergoing surgery for inguinal hernia repair? A. Delaying operation until 60 weeks’ postconceptual age B. Preoperative correction of anemia C. Caffeine administration D. Spinal anesthetic with ketamine sedation
631. (D) The respiratory center of newborns is not fully developed at birth. Newborns not uncommonly show two types of pauses in respiration. Periodic breathing exists if the pauses are short (i.e., 5-10 seconds) and not associated with a decrease in heart rate or oxygen saturation. Periodic breathing can be seen in up to 78% of full-term newborns and more commonly in premature newborns. Episodes of central apnea of infancy, also called apnea and bradycardia (A&B) spells, are longer, more significant, and less common than periodic breathing. With A&B spells, the respiratory pauses are usually longer than 15 to 20 seconds and are associated with a decrease in heart rate (<100), a decrease in oxygen saturation, cyanosis, and/or pallor. Treatment is usually tactile stimulation for A&B spells, whereas periodic breathing patterns do not need treatment. Untreated A&B spells can be lethal. Postoperative apnea is inversely correlated with both gestational age at birth (GA) and PCA (PCA = GA + chronologic age) up to 60 weeks’ PCA. Postoperative apnea is highest in the first 4 to 6 hours but can present up to 12 hours after surgery. Postoperative apnea is also associated with infants who have had a history of A&B spells as well as anemia (Hct <30). Caffeine has been used as a respiratory stimulant to decrease the incidence and severity of postoperative apnea. Although spinal anesthesia with no sedation has a lower incidence of apnea, compared with general anesthesia, the addition of any sedation such as ketamine increases the incidence of apnea more than that observed with general anesthesia. Some controversy exists as to when young infants can be treated as outpatients due to this risk of postoperative apnea. In our practice, healthy full-term infants (>38 weeks’ GA) who have not reached 44 weeks’ PCA and healthy preterm infants (<38 weeks’ GA) who have not reached 50 weeks are admitted for overnight monitoring (Davis: Smith’s Anesthesia for Infants and Children, ed 8, pp 35–36, 1283–1284 ).
632. Air should not be used to identify the epidural space in children because of the risk of A. Venous air embolism B. Infection C. Subcutaneous emphysema D. Epidural hematoma
632. (A) The loss-of-resistance technique used when placing an epidural needle into the epidural space of a child should be done with saline rather than air to decrease the risk of an air embolism. Note: the loss of resistance is much more subtle in the child compared with the adult when the epidural space is located (Davis: Smith’s Anesthesia for Infants and Children, ed 8, pp 474–475).
633. Induction of general anesthesia for an elective operation should be delayed how many hours after breastfeeding? A. 2 hours B. 4 hours C. 6 hours D. No fasting needed because breast milk is OK
633. (B) The fasting recommendation to reduce the risk of pulmonary aspiration of gastric contents is commonly called “the 2-4-6-8 rule.” MINIMUM FASTING PERIODS Ingested Material Minimum Fasting Period Clear fluids (water, Jello, apple or grape juice) 2 hr Breast milk 4 hr Infant formula, nonhuman milk, orange juice 6 hr Solid food (toast or cereal) or high-fat meals 8 hr From Davis PJ et al: Smith’s Anesthesia for Infants and Children, ed 8, Philadelphia, Saunders, 2011, pp 288–289.
634. In the infant, hypothermia would LEAST likely manifest as A. Metabolic acidosis B. Prolonged duration of action of nondepolarizing muscle relaxants C. Hyperglycemia D. Bradycardia
634. (C) In neonates or infants, hypothermia can increase total body oxygen consumption, produce metabolic acidosis and hypoglycemia (not hyperglycemia), depress ventilation, decrease metabolism of drugs, prolong the duration of action of nondepolarizing muscle relaxants, produce coagulopathies and platelet dysfunction, and increase wound infections. Therefore, monitoring the body temperature and maneuvers to minimize or eliminate significant loss of body heat during anesthesia for neonates and small infants are essential during the perioperative period (Davis: Smith’s Anesthesia for Infants and Children, ed 8, pp 174–175; Miller: Miller’s Anesthesia, ed 8, pp 1631–1632, 2763; Butterworth: Morgan & Mikhail’s Clinical Anesthesiology, ed 5, pp 879–880).
635. Necrotizing enterocolitis (NEC) has all of the following characteristics EXCEPT A. Most have thrombocytopenia (<70,000/mm3) and a prolonged prothrombin time (PT) and activated partial thromboplastin time (aPTT) B. Commonly associated with decreased cardiac output in the presence of fetal asphyxia or postnatal respiratory complications C. Umbilical artery catheters are useful to assess acidbase status D. Occurs in 10% to 20% of newborns weighing less than 1500 g
635. (C) NEC follows intestinal mucosal injury from ischemia and classically occurs in premature infants and in infants with low birth weight (typically <2500 g). In very-low-birth-weight (VLBW) newborns less than 1500 g, the incidence of NEC is 10% to 20%. NEC carries a high mortality rate (10%-30% if medically treated and a higher mortality rate if surgery is needed). These children may be acidotic, hypoxic, and in shock. Most have thrombocytopenia (50,000-70,000/mm3), prolonged PT, and prolonged aPTT. NEC is most commonly associated with decreased cardiac output in the presence of fetal asphyxia or postnatal respiratory complications in the early postnatal period. Other factors associated with the pathogenesis of NEC include a history of umbilical artery catheterization, enteral feeding of small preterm infants, bacterial infection, polycythemia, and gram-negative endotoxemia. Although umbilical artery catheters are often used in the newborn period, these should be removed if NEC develops, because they 178 Part 2 Clinical Sciences may compromise mesenteric blood flow. Unless there is evidence of intestinal necrosis or perforation, nonoperative therapy should be instituted. This includes cessation of enteral feeding, decompression of the stomach, administration of broad-spectrum antibiotics, fluid and electrolyte therapy, parenteral nutrition, and correction of hematologic abnormalities. Inotropic drugs may be needed in the presence of shock. Postoperatively these infants require ventilator support, and inotropes often are needed for cardiovascular support (Hines: Stoelting’s Anesthesia and Co-Existing Disease, ed 6, pp 601–602; Davis: Smith’s Anesthesia for Infants and Children, ed 8, pp 579–584 ).
636. Which of the following narcotics has a shorter half-life in the newborn compared with older children? A. Alfentanil B. Fentanyl C. Remifentanil D. Sufentanil
636. (C) Remifentanil is a unique opioid in newborns because its half-life is shorter in newborns than in older children. Remifentanil is rapidly metabolized by nonspecific plasma and tissue cholinesterases in the blood and has a higher volume of distribution in children younger than 2 months of age compared with children older than 2 months of age. Remifentanil does not accumulate even at high doses. The other narcotics have a relatively short half-life due to redistribution when given in low doses. Elimination is more important for the other listed narcotics at high doses (Davis: Smith’s Anesthesia for Infants and Children, ed 7, pp 428; Miller: Miller’s Anesthesia, ed 8, pp 2757, 2769–2771).
637. In a newborn, access to the vena cava can be gained by passage of a catheter through the A. Ductus arteriosus B. Ductus venosus C. Umbilical arteries D. Foramen ovale
637. (B) In the neonatal period, the newborn’s circulation can be accessed via the umbilical artery or vein. The umbilical vein is larger and easier to cannulate. A size 5 Fr. catheter (3.5 Fr. catheter in premature newborns) is commonly used. The umbilical vein catheter can be inserted through the ductus venosus directly into the vena cava (and usually positioned just above the diaphragm). Cannulation of the umbilical artery will lead the catheter tip into the aorta (Davis: Smith’s Anesthesia for Infants and Children, ed 8, pp 559–560; Miller: Basics of Anesthesia, ed 6, p 549).
638. A 5-year-old girl with hemolytic-uremic syndrome (HUS) is brought to the OR for placement of a dialysis catheter. Medical issues typical for this disease include A. Thrombocytopenia B. Increased intracranial pressure C. Pancreatitis D. All of the above
638. (D) HUS is one of the most common acquired causes of acute renal failure in children. Patients present with abdominal cramping, bloody diarrhea, and vomiting; it is often caused by the toxin from Escherichia coli O157. About 10% of children with bloody diarrhea caused by E. coli O157 progress to HUS. HUS is characterized by a triad of microangiopathic hemolytic anemia (Hgb levels around 4-5 g/dL), thrombocytopenia (platelet destruction as well as sequestration of platelets in the liver and spleen), and acute nephropathy. Although the age of children most frequently affected by this disease is between 6 months and 4 years, HUS can occur from the neonatal period through adulthood. Occasionally, CNS abnormalities develop (e.g., decreased levels of consciousness, seizures, and at times cerebral edema and increased intracranial pressure). Pancreatitis is common and congestive heart failure may develop as a result of fluid overload, hypertension, and myocardial depression from the toxins. Treatment is supportive and many of these children will require temporary peritoneal dialysis. The mortality rate is less than 5% (Hines: Stoelting’s Anesthesia and Co-Existing Disease, ed 6, p 425; Miller: Miller’s Anesthesia, ed 8, pp 2904–2905).
639. A 3-year-old child status post resection of Wilms tumor at age 2 years is receiving doxorubicin (Adriamycin) and cyclophosphamide for metastatic disease. The patient is scheduled for placement of a Hickman catheter for continued chemotherapy. Anesthetic concerns related to this patient’s chemotherapeutic treatment include each of the following EXCEPT A. Thrombocytopenia B. Inhibition of plasma cholinesterase C. Cardiac depression D. Pulmonary fibrosis 162 Part 2 Clinical Sciences
639. (D) Wilms tumor, also called nephroblastoma, is a common abdominal malignancy of children. Children commonly present with increasing abdominal girth and have a palpable mass. Peak age of diagnosis is 1 to 3 years. Renal function is usually preserved but hypertension, often mild, is common (60%). Fever, hematuria, and anemia are often present. Treatment consists of surgery, radiation, and chemotherapy. Chemotherapeutic drugs used in this tumor include dactinomycin, doxorubicin (Adriamycin), vincristine, and cyclophosphamide (Cytoxan). Bone marrow suppression (e.g., anemia, thrombocytopenia) can occur with all cytotoxic drugs. Because cardiomyopathy can occur with cyclophosphamide (>100 mg/ m2) and with doxorubicin (>220 mg/m2), preoperative echocardiography should be considered even in asymptomatic patients. Late cardiac dysfunction may develop 7 to 14 years after treatment. Alkylating agents, such as cyclophosphamide, inhibit plasma cholinesterases, which may affect the metabolism of succinylcholine. Pulmonary fibrosis and/or pneumonitis can occur in patients who have received bleomycin (the patient in this case did not receive bleomycin). This pulmonary toxicity may be related to high-inspired oxygen concentrations and excessive fluid administration. Vincristine has several CNS side effects, including peripheral neuropathy, impaired sensorium, and encephalopathy and renal toxicity (Davis: Smith’s Anesthesia for Infants and Children, ed 8, pp 751–754, 1138–1141; Hines: Stoelting’s Anesthesia and Co-Existing Disease, ed 6, pp 628–629; Miller: Miller’s Anesthesia, ed 8, pp 1216–1217).
640. Preoperatively, hypotension (i.e., decompensated shock) is characterized by a systolic blood pressure A. Less than 60 mm Hg for the term neonate (0-28 days old) B. Less than 70 mm Hg for infants 1 to 12 months old C. Less than 70 mm Hg + (2 × age in years) for children 1 to 10 years old D. All of the above
640. (D) All of the answers are correct. Shock occurs when perfusion to vital organs is inadequate to meet the organ’s metabolic demands. When shock is developing, cardiac output is initially well-maintained by increasing the heart rate and myocardial contractility. When cardiac output falls, blood pressure can only be maintained by a compensatory vasoconstriction. Shock is classified as compensated shock Pediatric Physiology and Anesthesia 179 ( systolic blood pressure in the normal range) or decompensated shock (systolic blood pressure less than the 5th percentile for age). If hypotension is present, one must be vigorous in treatment. Treatment is often begun with volume expansion; however, other causes of hypotension must be considered and treated as necessary (e.g., tension pneumothorax, pericardial tamponade, neurologic injury). Hypotension (i.e., decompensated shock) is based on systolic blood pressures and is correctly described in each of the choices in the question. In addition, for children 10 years of age or older, hypotension is a systolic blood pressure less than 90 (2010 American Heart Association Guidelines for Cardiopulmonary Resuscitation and Emergency Cardiovascular Care, Circulation 122:S878, 2010).
641. What percent of the adult’s GFR (indexed to body surface area) does a 2-year-old possess? A. 30% B. 50% C. 75% D. 100%
641. (D) At birth, the GFR is 15% to 30% of adult values and increases to about 50% by 10 days of life and to 75% by 6 months. Renal function is complete by 2 years of age (Miller: Basics of Anesthesia, ed 6, p 550; Miller: Miller’s Anesthesia, ed 8, p 2762).
642. Each of the following results in a reduction of the incidence of postoperative vomiting (POV) in children undergoing strabismus surgery EXCEPT A. IV hydration of 30 mL/kg/hr B. Dexamethasone 0.15 to 1 mg/kg IV C. Ondansetron 50 to 200 μg/kg IV D. Anticholinergics (atropine 10-20 μg/kg or glycopyrrolate 10 μg/kg) 163 Pediatric Physiology and Anesthesia Answers, References, and Explanations
642. (D) Prophylaxis for POV is recommended for patients undergoing strabismus surgery, because untreated, the incidence is 40% to 90% of patients. No benefit was demonstrated with the use of anticholinergic medications or with gastric content evacuation before emergence from anesthesia. IV hydration is very important. Recent studies have recommended that “superhydration” with 30 mL/kg/hr of lactated Ringer solution decreases the PONV rate by about half when compared to 10 mL/kg/hr fluid use. Decreasing or avoiding narcotic analgesics has also been effective. Avoiding the maintenance use of nitrous oxide remains controversial (Davis: Smith’s Anesthesia for Infants and Children, ed 8, pp 881–882). 180
643. Which of the following drugs does NOT pass the placenta easily? A. Etomidate B. Ephedrine C. Atropine D. Glycopyrrolate
643. (D) The fetal/maternal (F/M) drug ratio is a way to quantitatively describe drug transfer across the placenta. Time is also important when considering how much drug crosses into the fetus. Many anesthetic drugs cross the placenta such as local anesthetics, intravenous induction agents (e.g., propofol [F/M ratio of 0.7- 1.1], etomidate [F/M ratio of 0.5], ketamine [F/M ratio of 0.5]), inhalation agents (e.g., volatile anesthetics and nitrous oxide [F/M ratio of 0.7]), and narcotics (e.g., fentanyl [F/M ratio of 0.4], remifentanil [F/M ratio of 0.9], morphine [F/M ratio of 0.6]) and with time may affect the fetus/newborn. For vasopressors, ephedrine has an F/M ratio of 0.7, whereas phenylephrine has an F/M ratio of 0.2. The ionized neuromuscular blocking agents do not readily cross the placenta (F/M ratios of nondepolarizing drugs are around 0.1-0.2); succinylcholine, a depolarizing muscle relaxant, crosses very poorly as well. The anticholinergic drugs atropine and scopolamine have F/M drug ratios of 1.0 and readily cross the placenta, whereas glycopyrrolate has an F/M drug ratio of 0.1 and poorly crosses the placenta. Because the anticholinesterase agents (neostigmine, pyridostigmine, and edrophonium) cross the placenta to a limited extent but more so than glycopyrrolate, a pregnant patient undergoing nonobstetric surgery in which neuromuscular blocking drugs are being reversed with anticholinesterase agents should have atropine rather than glycopyrrolate used with the anticholinesterase mixture to prevent possible fetal bradycardia (Chestnut: Chestnut’s Obstetric Anesthesia, ed 5, pp 63–69; Suresh: Shnider and Levinson’s Anesthesia for Obstetrics, ed 5, pp 47–51).
644. A 38-year-old obese patient is receiving subcutaneous low-molecular-weight heparin (LMWH) for thromboprophylaxis. She received her epidural 14 hours after the heparin was stopped and developed Horner syndrome on the left side 30 minutes after placement of an epidural for an elective cesarean section. On physical examination, a T4 anesthetic level is noted, but aside from the Horner syndrome no other findings are revealed. The most appropriate course of action at this time would be to A. Remove the epidural B. Consult a neurosurgeon C. Obtain a computed tomographic scan D. None of the above
644. (D) After low-dose prophylaxis with low-molecular-weight heparin (LMWH) (e.g., enoxaparin 0.5 mg/ kg daily), a time of at least 10 to 12 hours should elapse prior to performing neuraxial techniques to decrease the likelihood of an epidural hematoma forming (at least 24 hours after high-dose LMWH [e.g., enoxaparin 1 mg/kg twice daily or 1.5 mg/kg daily] used for therapeutic anticoagulation). If the patient has back pain and unexpected neurologic paralysis, a workup for a hematoma should be performed. This case demonstrates a benign condition in which the sympathetic nerve supply to the eye is blocked (Horner syndrome [triad of miosis, ptosis, and anhidrosis]). This occasionally develops after a lumbar epidural anesthetic even when the highest dermatome level blocked is below T5. It may be related to the superficial anatomic location of the descending spinal sympathetic fibers that lie just below the spinal pia of the dorsolateral funiculus (which is within diffusion range of subanesthetic concentrations of local anesthetics in the cerebrospinal fluid) as well as increased sensitivity of local anesthetics during pregnancy (Chestnut: Chestnut’s Obstetric Anesthesia, ed 5, pp 923–925, 1046–1048; Suresh: Shnider and Levinson’s Anesthesia for Obstetrics, ed 5, pp 133, 355–359; ASRA Practice Advisory: Regional Anesthesia in the Patient Receiving Antithrombotic or Thrombolytic Therapy—Third Consensus Conference on Neuraxial Anesthesia and Anticoagulation, Jan-Feb 2010, www.asra.com/consensus-statements).
645. What percentage of all pregnancies in the United States is affected by hypertension? A. 2% B. 7% C. 12% D. 17%
645. (B) In the United States, hypertension (sustained systolic blood pressure [SBP] >140 mm Hg or a sustained diastolic blood pressure [DBP] >90 mm Hg) occurs with an overall incidence of approximately 5% to 10% of all pregnancies. Hypertension is a leading cause of maternal death worldwide. Hypertension during pregnancy is divided into four groups: preeclampsia-eclampsia, chronic hypertension (of any cause), chronic hypertension with superimposed preeclampsia, and gestational hypertension. Preeclampsiaeclampsia is a hypertensive disorder of pregnancy usually associated with proteinuria (≥300 mg protein per 24-hour urine collection). Recently (November 2013), the presence of proteinuria is no longer needed for the designation of preeclampsia-eclampsia. The reason for the change is that some patients develop proteinuria late and have their diagnosis and needed treatment delayed. Current definition of preeclampsia-eclampsia is the new onset of hypertension associated with thrombocytopenia (platelet count <100,000/mL), impaired liver function, renal insufficiency (serum creatinine >1.1 mg/dL or doubling of serum creatinine in the absence of any other renal disease), pulmonary edema, or newonset cerebral or visual disturbances. Gestational hypertension is just new onset of hypertension. With both preeclampsia-eclampsia and gestational hypertension, the hypertension resolves several days after delivery. Preeclampsia-eclampsia rarely occurs before the 20th week of gestation (unless a hydatidiform mole is present). The incidence of preeclampsia is significantly higher in parturients with a hydatidiform mole, multiple gestations, obesity, polyhydramnios, or diabetes and occurs more commonly with the first pregnancy. Mothers with preeclampsia during their first pregnancy have a 33% chance of having preeclampsia in subsequent pregnancies. Preeclampsia can progress to eclampsia (preeclampsia Obstetric Physiology and Anesthesia 189 accompanied by a seizure not related to other conditions). Eighty percent of the seizures occur before or during delivery; 85% of the remaining 20% will have the seizure within the first 24 hours after delivery. Approximately 5% of untreated parturients with preeclampsia will develop eclampsia (American College of Obstetricians and Gynecologists Task Force on Hypertension in Pregnancy, November 2013 Website; Chestnut: Chestnut’s Obstetric Anesthesia, ed 5, pp 825–829; Suresh: Shnider and Levinson’s Anesthesia for Obstetrics, ed 5, pp 437–438).
646. A 16-year-old, anxious, preeclamptic patient in active labor develops back pain after the placement of an epidural for labor analgesia. The pain is severe, and the patient has more weakness of the legs than expected. The most appropriate course of action at this time would be to A. Inject a higher concentration of a local anesthetic or add intravenous (IV) narcotics B. Replace the epidural and use epidural narcotics to decrease the motor weakness C. Reassure her that she will get better with delivery D. Consult a neurosurgeon
646. (D) Epidural hematomas and epidural abscesses are quite rare. Severe back pain and/or leg weakness that is greater than expected (or the recurrence of weakness after partial recovery of a neuraxial block) are presenting symptoms of spinal cord compression. Epidural hematomas can develop within 12 hours of a neuraxial procedure, whereas epidural abscesses usually take days to develop and also present with fever and leukocytosis. These conditions need imaging (e.g., magnetic resonance imaging [MRI]) and neurosurgical consultation. Studies have shown that when spinal cord decompression occurs within 8 hours of the onset of paralysis, neurologic recovery is significantly better than after 8 hours. Although epidural hematoma formation is rare, clotting disorders and perhaps marked difficulty in placing a block could lead to epidural bleeding and hematoma formation. Because the preeclamptic patient may develop a coagulopathy, one should carefully evaluate her coagulation status prior to initiating a regional block. Most anesthesiologists would evaluate a platelet count in the preeclamptic patient and look for any clinical signs of unexplained bleeding prior to initiating a regional block. Because an epidural blood patch often is performed with 20 mL of blood, the epidural hematoma that causes spinal cord compression must be significantly greater (Chestnut: Chestnut’s Obstetric Anesthesia, ed 5, pp 749–750; Suresh: Shnider and Levinson’s Anesthesia for Obstetrics, ed 5, p 415).
647. Magnesium sulfate (MgSO4) is used as an anticonvulsant in patients with preeclampsia as well as a tocolytic to prevent preterm delivery. MgSO4 may produce any of the following effects EXCEPT A. Sedation B. Respiratory paralysis C. Inhibition of acetylcholine release at the myoneural junction D. Hypertension when used with nifedipine
647. (D) The normal serum magnesium level is 1.5 to 2 mEq/L, with a therapeutic range of 4 to 8 mEq/L. Note: many laboratories report values in mg/dL (1 mEq/L = 1.2 mg/dL). As magnesium sulfate is administered IV, patients often note a warm feeling in the vein as well as some sedation. With increasing serum levels, loss of deep tendon reflexes occurs at 10 mEq/L (12 mg/dL), respiratory paralysis occurs at 15 mEq/L (18 mg/dL), and cardiac arrest at greater than 25 mEq/L (>30 mg/dL) can occur. Magnesium decreases the release of acetylcholine (ACh) at the myoneural junction and decreases the sensitivity of the motor endplate to ACh. This can produce marked potentiation of nondepolarizing muscle relaxants. The effect on depolarizing muscle relaxants is less clear, and most clinicians use standard intubating doses of succinylcholine (i.e., 1 to 1.5 mg/kg) followed by a markedly reduced dose of a nondepolarizing relaxant if needed. Because magnesium antagonizes the effects of α-adrenergic agonists, ephedrine is usually preferred over phenylephrine if a vasopressor is needed to restore blood pressure, along with fluids, after a neuraxial blockade. When a calcium channel blocker, such as nifedipine, is administered along with magnesium, greater hypotension has resulted. The antidote for magnesium toxicity is calcium (which, if needed, should be administered slowly) (Chestnut: Chestnut’s Obstetric Anesthesia, ed 5, pp 803–804, 838–839, 848; Suresh: Shnider and Levinson’s Anesthesia for Obstetrics, ed 5, pp 282, 448).
648. Normal fetal heart rate (FHR) is A. 60 to 100 beats/min B. 90 to 130 beats/min C. 110 to 160 beats/min D. 150 to 200 beats/min
648. (C) Fetal monitors consist of a two-channel recorder for simultaneous recording of FHR and uterine activity. In looking at the FHR, one assesses the baseline rate, the FHR variability, and the periodic changes (accelerations or decelerations) that occur with uterine contractions. The normal FHR varies between 110 and 160 beats/min. See also Answer 703 (Chestnut: Chestnut’s Obstetric Anesthesia, ed 5, pp 150– 151; Suresh: Anesthesia for Obstetrics, ed 5, pp 70–75).
649. Which of the following is the LEAST likely cause of pregnancy-related deaths in the United States (1998-2005)? A. General anesthesia (failed intubation or aspiration) B. Hemorrhage C. Thrombotic pulmonary embolism D. Hypertensive disorders of pregnancy
649. (A) Worldwide, hemorrhage (H), infection (I), and hypertensive disorders of pregnancy (preeclampsia [P]), or HIP, account for more than half of all maternal deaths. In the developed world, hypertensive disorders, embolic disorders, and hemorrhage account for slightly less than half of maternal deaths. In the United States between 1998 and 2005, seven conditions each account for 10% to 13% of all pregnancy-related deaths (hemorrhage, hypertensive disorders of pregnancy, sepsis, thrombotic pulmonary embolism, cardiomyopathy, other cardiovascular disorders, and noncardiovascular disorders). Anesthetic complications are a rare cause of maternal death (1%) (Chestnut: Chestnut’s Obstetric Anesthesia, ed 5, pp 932–941; Suresh: Shnider and Levinson’s Anesthesia for Obstetrics, ed 5, p 740).
650. Drugs useful in the treatment of uterine atony in an asthmatic patient with severe preeclampsia include A. Oxytocin (Pitocin) only B. Ergonovine (Ergotrate) or methylergonovine (Methergine) only C. 15-Methyl prostaglandin F2α (PGF2α) (Carboprost, Hemabate) only D. All of the above are safe and can be used alone or in combination with the others
650. (A) Uterine atony is a common cause of postpartum hemorrhage (2%-5% of all vaginal deliveries). Treatment consists of uterine massage, drugs, and, in rare cases, hysterectomy. Drugs commonly used include oxytocin, ergot alkaloids (ergonovine, methylergonovine), and prostaglandins (PGE2, PGF2α, 190 Part 2 Clinical Sciences 15-methyl PGF2α). Oxytocin is the first-line drug used for the treatment of uterine atony and may be used in patients with asthma or hypertensive disorders of pregnancy. If oxytocin is given as an IV bolus, vasodilation and hypotension often result. The ergot alkaloids are associated with a high incidence of nausea and vomiting. They not infrequently cause vasoconstriction, producing elevations in blood pressure, and are contraindicated in patients with hypertension (and in this case preeclampsia). Ergot alkaloids have also been associated with bronchospasm (rarely) and may not be appropriate in asthmatic patients. Thus, the ergot alkaloids are relatively contraindicated in patients with hypertension (such as preeclampsia), coronary artery disease, and asthma. The prostaglandin 15-methyl PGF2α (Carboprost, Hemabate) is the only prostaglandin currently approved for uterine atony in the United States and may cause significant bronchospasm in susceptible patients (and is contraindicated in asthmatic patients). Other smooth muscle contraction-associated side effects of prostaglandin 15-methyl PGF2α include venoconstriction as well as gastrointestinal muscle spasm (nausea, vomiting, and diarrhea) (Chestnut: Chestnut’s Obstetric Anesthesia, ed 5, pp 589–590, 888–891; Suresh: Shnider and Levinson’s Anesthesia for Obstetrics, ed 5, p 321).
651. What is the P50 of fetal hemoglobin at term? A. 12 B. 18 C. 24 D. 30
651. (B) Newborns have high hemoglobin levels around 15 to 20 g/100 mL. The term P50 denotes the blood oxygen tension (Pao2) that produces 50% saturation of erythrocyte hemoglobin. The P50 value of fetal hemoglobin is 18 mm Hg versus the adult value of 27 mm Hg. Thus, fetal hemoglobin has a higher affinity for oxygen than maternal hemoglobin (Chestnut: Chestnut’s Obstetric Anesthesia, ed 5, pp 83–84; Suresh: Shnider and Levinson’s Anesthesia for Obstetrics, ed 5, pp 26–27).
652. Side effects of terbutaline include all of the following EXCEPT A. Hypertension B. Hyperglycemia C. Pulmonary edema D. Hypokalemia
652. (A) Terbutaline is a β-adrenergic agonist with tocolytic properties and can be administered intravenously and subcutaneously as well as orally. Side effects are similar to those of other β-adrenergic drugs and include tachycardia, hypotension, myocardial ischemia, pulmonary edema (0.3% incidence), hypoxemia (inhibition of hypoxic pulmonary vasoconstriction), hyperglycemia (30% incidence), metabolic (lactic) acidosis, hypokalemia (39% incidence and due to a shift of potassium from extracellular to intracellular space), anxiety, and nervousness. Electrocardiogram (ECG) changes in ST segment depression and T wave flattening or inversion may occur and typically resolve after stopping the β-adrenergic therapy. Whether these ECG changes reflect myocardial ischemia or hypokalemia is unclear (Chestnut: Chestnut’s Obstetric Anesthesia, ed 5, pp 802–803; Suresh: Shnider and Levinson’s Anesthesia for Obstetrics, ed 5, pp 280–286).
653. Cardiac output increases dramatically during pregnancy and delivery. The cardiac output returns to nonpregnant values by how long postpartum? A. 12 hours B. 1 day C. 2 weeks D. 6 months
653. (C) The numerous changes that take place in the cardiovascular system during pregnancy provide for the needs of the fetus and prepare the mother for labor and delivery. During the first trimester of pregnancy, cardiac output increases by approximately 30% to 40%. At term, the cardiac output is increased 50% over nonpregnant values. This increase in cardiac output is due to an increase in stroke volume and an increase in heart rate. During labor, the cardiac output increases another 10% to 15% during the latent phase, 25% to 30% during the active phase, and 40% to 45% during the expulsive stage. Each uterine contraction increases the cardiac output by about 10% to 25%. The greatest increase in cardiac output occurs immediately after delivery of the newborn, when the cardiac output can increase to 75% above prelabor values. This final increase in cardiac output is attributed primarily to autotransfusion and increased venous return associated with uterine involution. Cardiac output falls to prelabor values within 2 days after delivery; however, it takes about 2 weeks for the cardiac output to decrease to nonpregnant values (Chestnut: Chestnut’s Obstetric Anesthesia, ed 5, pp 16–18; Suresh: Shnider and Levinson’s Anesthesia for Obstetrics, ed 5, pp 1–2).
654. A 32-year-old parturient with a history of spinal fusion, severe asthma, and hypertension (blood pressure 180/110) is brought to the operating room wheezing. She needs an emergency cesarean section under general anesthesia for a prolapsed umbilical cord. Which of the following induction agents would be MOST appropriate for her induction? A. Sevoflurane B. Midazolam C. Ketamine D. Propofol
654. (D) Asthma occurs in about 4% to 8% of all pregnancies. Although sevoflurane is a good induction agent for asthmatic patients, a rapid-sequence IV induction with endotracheal intubation to secure the airway is preferred. Because midazolam has a slow onset of action, it is not recommended for a rapid-sequence induction. When inducing general anesthesia in an asthmatic patient, it is imperative to establish an adequate depth of anesthesia before placing an endotracheal tube. If the patient is “light,” then severe bronchospasm may occur. In patients with asthma, intravenous induction will work with ketamine or propofol. Ketamine is considered by many as the induction agent of choice due to its mild bronchodilator properties, but because propofol (also a good induction agent in asthmatic patients) does not stimulate the cardiovascular system as ketamine does, in this case propofol would be preferred in this patient with hypertensive disorders of pregnancy. In patients with mild asthma who do not need the accessory muscles of respiration regional anesthesia should be strongly considered if time permits, because it Obstetric Physiology and Anesthesia 191 would eliminate the need for endotracheal intubation. In addition, inhaled β2-adrenergic agonist (e.g., albuterol) and intravenous steroids may be beneficial as well (Chestnut: Chestnut’s Obstetric Anesthesia, ed 5, pp 1179–1186; Shnider and Levinson’s Anesthesia for Obstetrics, ed 5, pp 524–535).
655. Uterine blood flow at term pregnancy typically increases to about A. 100 mL/min B. 250 mL/min C. 500 mL/min D. 750 mL/min
655. (D) Uterine blood flow increases dramatically from 50 to 100 mL/min before pregnancy to about 700 to 900 mL/min at term (i.e., >1 unit of blood per minute). From 70% to 90% of the uterine blood flow at term goes to the intervillous spaces. Uterine blood flow is related to the perfusion pressure (uterine arterial pressure minus uterine venous pressure) divided by the uterine vascular resistance. Thus, factors that decrease uterine blood flow include systemic hypotension, aortocaval compression, uterine contraction, and vasoconstriction (Chestnut: Chestnut’s Obstetric Anesthesia, ed 5, pp 40–42; Suresh: Shnider and Levinson’s Anesthesia for Obstetrics, ed 5, pp 23–24).
656. Which one of the following statements is TRUE regarding human immunodeficiency virus (HIV) infected parturients? A. Central neurologic blockade as well as epidural blood patches increase the chance of neurologic complications B. Ninety percent of newborns of untreated HIVseropositive mothers become infected in utero, during vaginal delivery, or with breastfeeding C. The pharmacologic effects of benzodiazepines and narcotics are prolonged in patients taking protease inhibitors D. The risk of seroconversion after percutaneous exposure to HIV-infected blood is about 5%
656. (C) Central neurologic blockade (i.e., epidural, spinal, or combined spinal-epidural) as well as epidural blood patches, appear to be safe for HIV-infected parturients. Vertical transmission from the mother to the newborn can occur in 15% to 40% when the mother is untreated. With antiretroviral therapy and elective cesarean delivery, the rate of transmission is reduced to about 1% to 2%. The risk of developing HIV after a needlestick injury with HIV-infected blood is 0.3%. (Risk of developing hepatitis B from a needlestick injury with hepatitis B–infected blood is 30% and hepatitis C from a needlestick injury with hepatitic C–infected blood is 2% to 4%.) Patients taking protease inhibitors as part of their drug therapy have inhibition of cytochrome P-450, and both benzodiazepines as well as narcotics have prolonged effects (Chestnut: Chestnut’s Obstetric Anesthesia, ed 5, pp 1058–1064; Suresh: Shnider and Levinson’s Anesthesia for Obstetrics, ed 5, pp 595–604).
657. Which of the following cardiovascular parameters is decreased at term? A. Central venous pressure B. Pulmonary capillary wedge pressure C. Systemic vascular resistance D. Left ventricular end-systolic volume
657. (C) There is no change in central venous pressure, pulmonary capillary wedge pressure, pulmonary artery diastolic pressure, or left ventricular end-systolic volume. Left ventricular end-diastolic volume is increased, as is stroke volume, ejection fraction, heart rate, and cardiac output. Systemic vascular resistance is decreased about 20% (Chestnut: Chestnut’s Obstetric Anesthesia, ed 5, pp 16–19; Suresh: Shnider and Levinson’s Anesthesia for Obstetrics, ed 5, pp 1–3).
658. Which of the following signs and symptoms is NOT associated with amniotic fluid embolism? A. Chest pain B. Bleeding (disseminated intravascular coagulation [DIC]) C. Pulmonary vasospasm with severe pulmonary hypertension and right heart failure D. Left ventricular failure and pulmonary edema
658. (A) Amniotic fluid embolism (AFE) is a very rare but serious complication of labor and delivery that results from the entrance of amniotic fluid and constituents of amniotic fluid into the maternal systemic circulation. About 10% of maternal deaths are caused by AFE, and two thirds of these deaths occur within 5 hours. Of those patients who survive the AFE, about 50% have significant neurologic dysfunction. For AFE to occur, the placental membranes must be ruptured, and abnormal open sinusoids at the uteroplacental site or lacerations of endocervical veins must exist. The classic triad is acute hypoxemia, hemodynamic collapse (i.e., severe hypotension), and coagulopathy without an obvious cause. More than 80% of these women develop cardiopulmonary arrest. Hemodynamic monitoring often shows a biphasic response; initially pulmonary vasospasm with severe pulmonary hypertension and right heart dysfunction is seen, followed by left ventricular failure and pulmonary edema. DIC occurs in about 66% of cases, and seizures occur about 50% of the time. Recently, AFE is believed to be a bit different from a pure embolic event, because findings of anaphylaxis and septic shock also are involved. Bronchospasm, however, is rare (<15%) during an AFE, and chest pain is very rare (2% of patients) (Chestnut: Chestnut’s Obstetric Anesthesia, ed 5, pp 915–920; Hines: Stoelting’s Anesthesia and Co-Existing Disease, ed 6, pp 571–572; Suresh: Shnider and Levinson’s Anesthesia for Obstetrics, ed 5, pp 333–348).
659. When is the fetus most susceptible to the effects of teratogenic agents? A. 1 to 2 weeks of gestation B. 3 to 8 weeks of gestation C. 9 to 14 weeks of gestation D. 15 to 20 weeks of gestation
659. (B) Organogenesis mainly occurs between the 15th and 56th days (3-8 weeks) of gestation in humans and is the time during which the fetus is most susceptible to teratogenic agents. Although all commonly used anesthetic drugs are teratogenic in some animal species, there is no conclusive evidence to implicate any currently used local anesthetics, IV induction agents, or volatile anesthetic agents in the causation of human congenital anomalies (Chestnut: Chestnut’s Obstetric Anesthesia, ed 5, pp 360–366; Suresh: Shnider and Levinson’s Anesthesia for Obstetrics, ed 5, pp 806–809).
660. A 28-week estimated gestational age (EGA), 1000-g male infant is born to a 24-year-old mother who is addicted to heroin. The mother admits taking an extra “hit” of heroin before coming to the hospital because she was nervous. The infant’s respiratory depression would be best managed by A. 0.1 mg/kg naloxone intramuscularly (IM) in the newborn’s thigh muscle B. 0.1 mg/kg naloxone down the endotracheal tube C. 0.4 mg naloxone IM to the mother during the second stage of labor D. None of the above
660. (D) Opioid abuse during pregnancy is estimated to occur in about 5% of patients in the United States, most often with the nonprescription use of pain-relieving drugs such as oxycodone. Other opioids include morphine, heroin, methadone, meperidine, and fentanyl. The problems associated with abuse are many and include the drug effect itself and substances mixed with the narcotics (e.g., talc, cornstarch), as well as infection and malnutrition. Newborn respiratory depression as manifested by a low respiratory rate is 192 Part 2 Clinical Sciences treated with controlled ventilation but not with naloxone. Naloxone can precipitate an acute withdrawal reaction and should not be administered to patients with chronic narcotic use (mother or newborn). The dose of naloxone to treat narcotic-induced respiratory depression in the nonaddicted newborn was 0.1 mg/kg, but more recent data suggest that it may worsen the neurologic damage caused by asphyxia. Current recommendations are to assist ventilation until the narcotic effects wear off and not to use naloxone (Chestnut: Chestnut’s Obstetric Anesthesia, ed 5, pp 177, 1209–1213; Suresh: Shnider and Levinson’s Anesthesia for Obstetrics, ed 5, pp 253, 693–696).
661. Cardiac output is GREATEST A. During the first trimester of pregnancy B. During the third trimester of pregnancy C. During labor D. Immediately after delivery of the newborn
661. (D) Immediately after delivery, the cardiac output can increase 75% above prelabor values. This is thought to result from autotransfusion and increased venous return to the heart associated with involution of the uterus, as well as increased blood return as the result of the lithotomy position. See also Answer 653 (Chestnut: Chestnut’s Obstetric Anesthesia, ed 5, pp 16–18; Suresh: Shnider and Levinson’s Anesthesia for Obstetrics, ed 5, pp 1–2).
662. A 1000-g, 27-week EGA boy is born with a heart rate of 80 beats/min. He has slow irregular respiratory efforts, grimaces when a suction catheter is in inserted into the mouth and nose for suctioning, and flexes his limbs some but is totally cyanotic. The umbilical cord has only two vessels. The 1-minute Apgar score would be A. 3 B. 4 C. 6 D. 7
662. (B) The Apgar score is a subjective scoring system used to evaluate the newborn and is commonly performed 1 minute and 5 minutes after delivery. If the score is less than 7, the scoring is also performed at 10, 15, and 20 minutes after delivery. A value of 0, 1, or 2 is given to each of five signs (heart rate, respiratory effort, reflex irritability, muscle tone, and color) and totaled. In this case, the child gets 1 point for heart rate, 1 point for respiratory effort, 1 point for reflex irritability, 1 point for muscle tone, and 0 points for color. THE APGAR SCORE Sign 0 1 2 Total This Case 1. Heart rate Absent <100 >100 ___1___ 2. Respiratory effort Absent Slow, irregular Good, crying ___1___ 3. Reflex irritability No response Grimace Cough or sneeze ___1___ 4. Muscle tone Flaccid Some flexion Active motion ___1___ 5. Color Blue or pale Pink body with blue extremities Completely pink ___0___ Sum = ___4___ An Apgar score of 7 to 10 is normal, 4 to 6 is moderate, and 0 to 3 indicates severe depression. Weight, gestational age, and sex are not factors included in the scoring system (Chestnut: Chestnut’s Obstetric Anesthesia, ed 5, pp 168–170; Suresh: Shnider and Levinson’s Anesthesia for Obstetrics, ed 5, 244–246).
663. Which of the following respiratory parameters is NOT increased in the parturient? A. Minute ventilation B. Tidal volume (Vt) C. Arterial Pao2 D. Serum bicarbonate 182 Part 2 Clinical Sciences
663. (D) The respiratory system undergoes many important changes during pregnancy. Oxygen consumption increases about 20% to 60%. To help supply the needed oxygen for the metabolically active mother and fetus, minute ventilation (MV) increases about 45% to 50%. The increase in MV is primarily due to an increase in tidal volume (Vt) of 40% to 45%, with a slight increase in respiratory rate. The increase in MV produces a fall in the Paco2 to approximately 30 to 32 mm Hg, and a respiratory alkalosis develops. To help get the pH back to normal, the serum bicarbonate level falls an average of 4 mEq/L. The arterial Pao2 increases slightly due to the fall in Paco2 (Chestnut: Chestnut’s Obstetric Anesthesia, ed 5, pp 19–22; Suresh: Shnider and Levinson’s Anesthesia for Obstetrics, ed 5, pp 6–8).
664. Which of the following drugs should NOT be used during transvaginal oocyte retrieval (TVOR) for assisted reproductive technology (ART)? A. Propofol B. Ketamine C. Midazolam D. All are safe and can be used
664. (D) In 2011, there were more than 163,000 ART cycles in the United States, with live births occurring in almost 40% of these procedures. The oocytes can be retrieved by laparoscopy or, more commonly now, transvaginally (transvaginal oocyte retrieval [TVOR]). Most anesthetic drugs have been studied and found not to be a problem, including propofol, midazolam, ketamine, alfentanil, fentanyl, remifentanil, and meperidine. When general anesthesia was used (laparoscopic retrieval), isoflurane with and without nitrous oxide was usually used and appeared safe. However, with increased time during general anesthesia, the oocytes retrieved earlier had better fertilization rates than the oocytes obtained near the end of the laparoscopy. It is unclear whether this was due to our anesthetics or to the lowered pH as a result of the carbon dioxide pneumoperitoneum. Etomidate has not been widely used, and patient numbers are too small to recommend its use. When morphine is used in high doses in animal studies, chromosomal abnormalities are very common (25%-33%), and morphine is not recommended for ART procedures. It Obstetric Physiology and Anesthesia 193 is recommended to avoid using the dopamine antagonists (e.g., droperidol and metoclopramide) during ART cycles because these drugs induce hyperprolactinemia, which impairs ovarian follicular maturation. A single dose immediately prior to oocyte retrieval probably is safe. The 5-hydroxytrptamine type 3 (5-HT3) receptor antagonists are commonly used, but there is insufficient evidence to recommend their use. The phenothiazines and the antihistamine H1-receptor antagonists are thought to be preferred because they have been studied without adverse effects (Chestnut: Chestnut’s Obstetric Anesthesia, ed 5, pp 326–337; Suresh: Shnider and Levinson’s Anesthesia for Obstetrics, ed 5, pp 765–773).
665. Which of the following conditions is associated with increased bleeding during pregnancy? A. Lupus anticoagulant B. Factor V Leiden mutation C. Protein C deficiency D. None of the above
665. (D) All of the conditions listed in this question, as well as deficiencies of antithrombin III and protein S (a cofactor for protein C), lead to hypercoagulable states. Unless treated with anticoagulation therapy, these conditions will have an increased frequency of thrombosis. These conditions may also cause placental thrombosis and insufficiency, and can increase the incidence of obstetric conditions such as intrauterine growth restriction, preeclampsia, placental abruption, and intrauterine death. Lupus anticoagulant, also called lupus antibody, is a prothrombotic agent. It gets its name because the presence of these antibodies causes an increase in the activated partial thromboplastin (aPTT) test, as these antibodies interfere with phospholipids used to induce in vitro coagulation. However, in vivo, these antibodies interact with platelet membrane phospholipids, increasing adhesions and the aggregation of platelets. Factor V Leiden mutation allows factor V to persist longer in the circulation (not metabolized as rapidly by activated protein C), leading to a hypercoagulable state. Protein C inhibits activated clotting factors V and VIII; thus, during a deficiency state, factors V and VIII persist longer in the circulation, leading again to a hypercoagulable state. During pregnancy, the incidence of thrombosis with protein C deficiency is about 25% unless anticoagulation therapy is administered (Chestnut: Chestnut’s Obstetric Anesthesia, ed 5, pp 951–952, 1048–1049).
666. What is the BEST way to prevent autonomic hyperreflexia in a quadriplegic woman who is to undergo induction of labor? The complete spinal cord lesion occurred 2 years ago. A. Only IV drugs should be used; spinal and epidural anesthesia are contraindicated B. Spinal or epidural lumbar local anesthetics such as bupivacaine alone are effective C. Spinal or epidural narcotics such as fentanyl alone are effective D. Autonomic hyperreflexia appears only when the complete spinal cord lesion is below T6, so there is no need to worry
666. (B) Patients with complete spinal cord lesions above T10 do not have pain with labor. However, about 85% of women with complete spinal cord lesions at the T6 and higher level will develop autonomic hyperreflexia (severe headache, hypertension, bradycardia, sweating above the lesion, and facial flushing) during labor and delivery. Autonomic hyperreflexia typically occurs with the contractions and disappears between contractions. An epidural or a spinal with local anesthetics works well to prevent and/or treat autonomic hyperreflexia. Epidural narcotics such as fentanyl alone are not effective (unless the narcotic is meperidine, which has local anesthetic properties in addition to narcotic effects). To check whether the epidural or spinal that is loaded with a local anesthetic is working in a quadriplegic patient, check the reflexes below the expected level of anesthesia (e.g., patellar) before and after the block. The local anesthetic concentration for labor epidurals (alone without narcotics) typically is 0.25% or higher. If a cesarean section is needed, 2% lidocaine with epinephrine (1:200,000) has been reported to be safe. If a cesarean section is needed with general anesthesia, typical IV anesthetics and inhalation drugs are used except for muscle relaxation, where succinylcholine is contraindicated (hyperkalemic response) and a nondepolarizing muscle relaxant such as rocuronium is preferred (Chestnut: Chestnut’s Obstetric Anesthesia, ed 5, pp 1117–1120; Suresh: Shnider and Levinson’s Anesthesia for Obstetrics, ed 5, p 564).
667. A 24-year-old gravida 2, para 1 parturient is anesthetized for emergency cesarean section. On emergence from general anesthesia, the endotracheal tube is removed and the patient becomes cyanotic. Oxygen is administered by positive-pressure bag and mask ventilation. High airway pressures are necessary to ventilate the patient, and wheezing is noted over both lung fields. The patient’s blood pressure falls from 120/80 to 60/30 mm Hg, and heart rate increases from 105 to 180 beats/min. The MOST likely cause of these manifestations is A. Amniotic fluid embolism B. Mucous plug in trachea C. Pneumothorax D. Aspiration
667. (D) Many of the signs are consistent with the choices described in this question. From the temporal perspective, gastric acid aspiration is the most likely cause, because aspiration can develop not only on induction but also on extubation, as in this case. That is why it is so important to always empty the patient’s stomach with an orogastric tube after an endotracheal tube is placed in any pregnant patient over 20 weeks’ undergoing general anesthesia, and extubate the patient when she is fully awake and responsive. Morbidity and mortality occurring after gastric acid aspiration is determined by both the amount and the pH of the aspirated material. Based on an animal study in which 0.4 mL/kg with a pH less than 2.5 injected into the right mainstem of one rhesus monkey caused death, many have used that definition (0.4 mL/kg with a pH <2.5) to categorize patients who are “at risk” for significant aspiration morbidity and mortality. Using these values, up to 70% of women who fasted before elective cesarean section are “at risk for aspiration.” Recently, it has been noted that the volume needed to cause aspiration in primates should be greater (e.g., 0.8 mL/kg) and the pH less than 2.5. Regardless of the definition of the “patient at risk,” when aspiration occurs it can be lethal. Bronchospasm (often associated with higher airway pressures) and wheezing are suggestive of gastric acid aspiration and not amniotic fluid embolism. Other signs and symptoms of aspiration include sudden coughing or laryngospasm, dyspnea, tachypnea, the presence of foreign material in the mouth or posterior pharynx, chest wall retraction, cyanosis not relieved by oxygen supplementation, tachycardia, hypotension, and the development of pinky frothy exudates. The 194 Part 2 Clinical Sciences onset of these signs and symptoms is usually rapid. Early treatment consists of supplemental oxygen with positive-pressure ventilation, PEEP, or continuous positive airway pressure, and suctioning of the airway can decrease the incidence of mortality from acid aspiration. Mortality seems to be reduced when protective ventilatory strategies are used (i.e., tidal volumes of 6 mL/kg with plateau pressures of <30 cm H2O are better than if 12 mL/kg and plateau pressures of 50 cm H2O are used). Conservative as compared to liberal fluid management (guided by central venous pressures and/or pulmonary artery wedge pressures) also appears to improve lung function. The use of prophylactic antibiotics and/or steroids has not been helpful (Chestnut: Chestnut’s Obstetric Anesthesia, ed 5, pp 669–675; Suresh: Shnider and Levinson’s Anesthesia for Obstetrics, ed 5, pp 403–411).
668. A 29-year-old gravida 1, para 0 parturient at 8 weeks of gestation is to undergo an emergency appendectomy under general anesthesia with isoflurane, N2O, and oxygen. Which of the following is a proven untoward consequence of general anesthesia in the unborn fetus? A. Congenital heart disease B. Cleft palate C. Behavioral defects D. None of the above
668. (D) The primary objectives in the anesthetic management of parturients undergoing general anesthesia for nonobstetric surgery are as follows: to (1) ensure maternal safety; (2) avoid teratogenic drugs; (3) avoid intrauterine fetal asphyxia; and (4) prevent the induction of preterm labor. Premature onset of labor is the most common complication associated with surgery during the second trimester of pregnancy. Performance of intra-abdominal procedures in which the uterus is manipulated is the most significant factor in causing premature labor in these patients. Neurosurgical, orthopedic, thoracic, or other surgical procedures that do not involve manipulation of the uterus do not cause preterm labor. No anesthetic agent or technique has been found to be significantly associated with a higher or lower incidence of preterm labor. Furthermore, there is no evidence that the risk of developing any of the conditions listed in this question is increased for the offspring of patients who receive general anesthesia during pregnancy (Suresh: Shnider and Levinson’s Anesthesia for Obstetrics, ed 5, pp 804–816).
669. A lumbar epidural is placed in a 24-year-old gravida 1, para 0 parturient with myasthenia gravis (MG) for labor. Select the TRUE statement regarding neonatal MG. A. The newborn is almost always affected with myasthenia B. The newborn is affected by maternal immunoglobulin M (IgM) antibodies C. The newborn may require anticholinesterase therapy for up to 4 weeks D. The newborn will need lifelong treatment
669. (C) Myasthenia gravis (MG) is an autoimmune neuromuscular disease in which immunoglobulin G (IgG) antibodies are directed against the ACh receptors in skeletal muscle, causing patients to present with general muscle weakness and easy fatigability. Smooth muscle and cardiac muscle are not affected. About 10% to 20% of newborns born to mothers with MG are transiently affected because the IgG antibody is transferred through the placenta. Neonatal MG is characterized by muscle weakness (e.g., hypotonia, respiratory difficulty) and may appear within the first 4 days of life (80% appear within the first 24 hours). Anticholinesterase therapy may be required for several weeks until the maternal IgG antibodies are metabolized (Chestnut: Chestnut’s Obstetric Anesthesia, ed 5, pp 1120–1122; Suresh: Shnider and Levinson’s Anesthesia for Obstetrics, ed 5, pp 537–539).
670. A patient having which of the following conditions is LEAST likely to develop DIC? A. Severe preeclampsia B. Placenta abruption C. Placenta previa (bleeding) D. Dead fetus syndrome
670. (C) Disseminated intravascular coagulation (DIC) is an acquired coagulopathy characterized by excessive fibrin deposition, depression of the normal coagulation inhibition mechanism, and impaired fibrin degradation. The formation of clots causes a depletion of platelets and factors. Laboratory diagnosis of DIC is based on the demonstration of abnormalities in platelet count (i.e., <100,000/mm3), prolonged prothrombin time (i.e., >3 seconds above normal), presence of fibrin degradation products, and fibrinogen level (i.e., ≤1 g/L). DIC is associated with the following obstetric conditions: placental abruption, dead fetus syndrome, amniotic fluid embolism, gram-negative sepsis, and severe preeclampsia. Placental abruption is the most common cause of DIC in pregnant patients. If one looks at severe placenta abruptions (in which the abruption is large enough to cause fetal death), about 30% of patients will develop DIC within 8 hours of the abruption. Nonobstetric causes of DIC include sepsis and malignancy. Patients with placenta previa who are bleeding do not develop DIC because the blood loss does not induce a coagulopathy (Barash: Clinical Anesthesia, ed 7, pp 435–437; Chestnut: Chestnut’s Obstetric Anesthesia, ed 5, pp 1045–1046; Suresh: Shnider and Levinson’s Anesthesia for Obstetrics, ed 5, pp 311–321, 444–445, 574–575).
671. A 28-year-old gravida 1, para 0 parturient with Eisenmenger syndrome (pulmonary hypertension with an intracardiac right-to-left or bidirectional shunt) is to undergo placement of a lumbar epidural for analgesia during labor. It may be wise to avoid a local anesthetic with epinephrine in this patient because it A. Lowers pulmonary vascular resistance B. Lowers systemic vascular resistance C. Increases heart rate D. Causes excessive increases in systolic blood pressure
671. (B) Eisenmenger syndrome may develop in patients with uncorrected left-to-right intracardiac shunting such as for ventricular septal defect, atrial septal defect, or patent ductus arteriosus. In this syndrome, the pulmonary and vascular tone and right ventricular muscle undergo changes in response to the shunt, producing pulmonary hypertension and a change in the direction of the shunt to a right-to-left or bidirectional type with peripheral cyanosis. The maternal mortality rate is 30% to 50%. Approximately 3% of all patients with congenital heart defects will develop this condition over time. When the Eisenmenger syndrome develops, the pulmonary vascular resistance becomes fixed, making this condition not amenable to surgical correction. Survival beyond age 40 years is uncommon. Any event or drug that increases pulmonary vascular resistance (e.g., hypercarbia, acidosis, hypoxia) or decreases systemic vascular resistance will worsen the right-to-left shunt, will exacerbate peripheral cyanosis, and may precipitate right ventricular heart failure in these patients. Controversy exists regarding pain management for these Obstetric Physiology and Anesthesia 195 patients because pain can elevate pulmonary artery pressures and cause more shunting. Many practitioners prefer a narcotic-based analgesic (spinal or epidural). Because these patients are very dependent upon preload and afterload, placing invasive monitors (central venous pressure and arterial catheter), and using the pulse oximeter to evaluate the amount of shunting, aggressive treatment of any fall in preload or peripheral vascular resistance can be performed. It should be recalled that centrally administered local anesthetics reduce preload and afterload. Low-dose epinephrine, which can be used to decrease the absorption of local anesthetics, should be used cautiously, if at all, because a further decrease in systemic vascular resistance may result from the β effect of absorbed epinephrine, and intravascular injection may elevate pulmonary pressures more, exacerbating the right-to-left shunt (Chestnut: Chestnut’s Obstetric Anesthesia, ed 5, p 975; Fleisher: Anesthesia and Uncommon Diseases, ed 5, pp 118–119; Hines: Stoelting’s Anesthesia and Co-Existing Disease, ed 6, pp 59–60; Suresh: Shnider and Levinson’s Anesthesia for Obstetrics, ed 5, pp 491–492).
672. Which of the following patients is MOST likely to need an emergency hysterectomy for uncontrolled bleeding at the time of delivery? A. Patient undergoing cesarean section after an unsuccessful trial of labor after cesarean (TOLAC) B. Patient with quadruplets C. Patient with a placenta previa (not bleeding) for an elective repeat cesarean section D. Patient with an abdominal pregnancy
672. (C) The need for a hysterectomy for a planned repeat cesarean delivery is 0.3%, for a successful vaginal birth after cesarean is 0.1%, and for an unsuccessful TOLAC is 0.5%. With multiple gestations, uterine atony is common, and the need for a hysterectomy is sixfold a normal delivery. However, the patient with placenta previa and a previous scar in the uterus has the highest chance of needing an emergency hysterectomy for uncontrolled bleeding at the time of delivery because of the associated placenta accreta (abnormally adherent placenta). The incidence of placenta accreta in a patient with placenta previa and no previous cesarean section is 3% to 4%, with one previous cesarean section is about 10% to 25%, and with two or more previous cesarean sections is 40% to almost 70%. About two thirds of patients with placenta accreta will require a cesarean hysterectomy. The average blood loss during an emergency obstetric hysterectomy is 5 to 7 units of blood (Chestnut: Chestnut’s Obstetric Anesthesia, ed 5, pp 893–895; Suresh: Shnider and Levinson’s Anesthesia for Obstetrics, ed 5, pp 147–149, 274–275, 311–321).
673. The MOST common injury recorded in the American Society of Anesthesiologists’ (ASA’s) Closed Claim Project regarding obstetric anesthetic claims is A. Pain during anesthesia B. Maternal nerve damage C. Headache D. Aspiration pneumonitis Obstetric Physiology and Anesthesia 183
673. (B) According to the ASA’s Closed Claim Project for Obstetric Anesthesia Claims (640 claims as of December 2010 report), maternal nerve damage (19%), neonatal brain damage (16%), and maternal death (15%) were the three most frequent claims. Other causes include headache (11%), back pain (10%), neonatal death (9%), emotional distress (8%), maternal brain damage (7%), pain during anesthesia (6%), and aspiration pneumonitis (1%) (Chestnut: Chestnut’s Obstetric Anesthesia, ed 5, pp 776–779).
674. Which of the following statements about chorioamnionitis is FALSE? A. Chorioamnionitis occurs in about 1% of all pregnancies B. Clinical signs include temperature higher than 38° C, maternal and fetal tachycardia, and uterine tenderness C. Antibiotics are administered only after delivery, because intrapartum antibiotics may “obscure the results of neonatal blood cultures” D. Epidural anesthesia can be safely administered
674. (C) Chorioamnionitis occurs in about 1% of all pregnancies. It includes the clinical signs and symptoms of infection, temperature higher than 38° C, maternal and fetal tachycardia, uterine tenderness (about 10% of patients) and/or foul-smelling amniotic fluid. Prompt delivery is the cornerstone of therapy. At one time it was thought that antibiotics should be administered only after delivery because antepartum or intrapartum antibiotics may “obscure the results of neonatal blood cultures.” However, early antepartum treatment with antibiotics leads to a decrease in maternal and neonatal morbidity, compared to delaying the antibiotics until after delivery, and is currently recommended. Epidural anesthesia has been shown to be commonly used and safe in these patients, preferably after antibiotics have been started. It seems prudent, however, to always individualize care and to weigh the risks versus the benefits of epidural anesthesia in a patient with suspected bacteremia (Chestnut: Chestnut’s Obstetric Anesthesia, ed 5, pp 862–873).
675. Which of the following statements regarding newborns with thick meconium-stained amniotic fluid is TRUE? A. Routine intrapartum oropharyngeal and nasopharyngeal suction is not recommended B. Intubation is required for all such newborns C. Antibiotics and steroids are often needed to treat the infection D. Respiratory distress syndrome (RDS) is common
675. (A) Meconium-stained amniotic fluid occurs in about 5% to 15% of all deliveries. Although intrapartum oropharyngeal and nasopharyngeal suction for all newborns born to mothers with meconium staining has been routine care for many years, current evidence shows no real benefit, and it is no longer recommended. Intubation and tracheal suction should be performed only in newborns who are not vigorous and does not depend upon the consistency of the meconium-stained fluid as was once recommended. In newborns who are vigorous (i.e., strong respiratory efforts, good muscle tone, and heart rate >100 beats/ min), no further treatment is needed. Because meconium is sterile, antibiotics are not needed. Steroids have not been necessary in the treatment of meconium-stained newborns. RDS is a condition that occurs as a result of low levels of pulmonary surfactant in the alveoli. RDS occurs in premature newborns, whereas meconium staining occurs typically in older, often post-term, newborns (Chestnut: Chestnut’s Obstetric Anesthesia, ed 5, pp 156–157, 179–180; Suresh: Shnider and Levinson’s Anesthesia for Obstetrics, ed 5, pp 251–252). 196 Part 2 Clinical Sciences
676. A 38-year-old primiparous patient with placenta previa and active vaginal bleeding arrives in the operating room with a systolic blood pressure of 85 mm Hg. A cesarean section is planned. The patient is lightheaded and scared. Which of the following anesthetic induction plans would be most appropriate for this patient? A. Spinal anesthetic with 12 to 15 mg bupivacaine B. General anesthetic induction with 2 to 2.8 mg/kg propofol and paralysis with 1 to 1.5 mg/kg succinylcholine C. General anesthesia induction with 0.75 to 1 mg/kg ketamine and paralysis with 1 to 1.5 mg/kg succinylcholine D. Replace lost blood volume first, then use any anesthetic the patient wishes
676. (C) Total Partial Marginal Three Variations of Placenta Previa Placenta previa occurs when the placenta implants on the lower uterine segment so that all (total) or part of the placenta (partial) covers the internal cervical os. A marginal placenta previa occurs when the placenta lies close to but does not cover the internal cervical os. Placenta previa occurs in about 0.5% of all pregnancies and has a maternal mortality of less than 1% but a fetal mortality approaching 20% (primarily because of prematurity and intrauterine asphyxia). Patients typically present with painless vaginal bleeding that stops spontaneously (first bleed). Delivery is cesarean and is often made a few weeks after the “first” bleed, when the baby’s lungs are more mature (e.g., after 37 weeks EGA). A later bleed can be uncontrolled and may be accompanied by significant hypovolemia and hypotension. Regional anesthesia is contraindicated in severely hypovolemic patients. Replacing blood loss may not be practical because bleeding may be quicker than replacement is possible (i.e., may be >1 unit/min). A rapid-sequence general anesthetic (assuming acceptable airway) is preferred. Ketamine (0.75-1 mg/kg) as well as etomidate (0.3 mg/kg) supports the cardiovascular system better than propofol. In rare but severe cases of hypovolemic shock, all IV anesthetics may cause the blood pressure to fall further, and succinylcholine alone may be all that is required. In these severe cases, maternal recall should be considered secondary to maternal safety. In cases in which a difficult intubation is likely and the patient is hypovolemic, an infiltration local anesthetic may be best (Chestnut: Chestnut’s Obstetric Anesthesia, ed 5, pp 571, 882–885; Suresh: Shnider and Levinson’s Anesthesia for Obstetrics, ed 5, pp 314–316).
677. Which of the following lung volumes or capacities change the LEAST during pregnancy? A. Tidal volume (Vt) B. Functional residual capacity (FRC) C. Expiratory reserve volume (ERV) D. Vital capacity (VC)
677. (D) At term pregnancy, Vt increases about 40% to 45%, and the inspiratory reserve volume (IRV) increases about 5%. A decrease occurs in both the expiratory reserve volume (ERV; 20%-25%) and the residual volume (RV; 15%-20%). A capacity is defined as two or more lung volumes. Functional residual capacity (FRC = ERV + RV) is decreased about 15% to 20% and is partly responsible for the rapid fall in maternal oxygenation that occurs with apnea during the induction of general anesthesia. Total lung capacity (TLC = Vt + IRV + ERV + RV) decreases about 5%, whereas vital capacity (VC = Vt + IRV + ERV) remains unchanged (Chestnut: Chestnut’s Obstetric Anesthesia, ed 5, pp 19–21; Suresh: Shnider and Levinson’s Anesthesia for Obstetrics, ed 5, pp 6–7).
678. General anesthesia is induced in a 35-year-old patient for elective cesarean section. No part of the glottic apparatus is visible after two unsuccessful attempts to intubate, but mask ventilation is adequate. The most appropriate step at this point would be to A. Wake up the patient B. Attempt a blind nasal intubation C. Continue mask ventilation and cricoid pressure D. Use a laryngeal mask airway
678. (A) Evaluation of the airway should be performed before the induction of any general anesthetic. In cases in which an unrecognized difficult airway exists (unable to perform endotracheal intubation in a reasonable period of time), the patient should be awakened if the procedure is purely elective and if the fetus has minimal or no fetal distress (as in this elective case). A regional anesthetic or awake intubation then can be safely performed. In cases of fetal or maternal distress, other options for securing the airway may be necessary (Chestnut: Chestnut’s Obstetric Anesthesia, ed 5, pp 700–701; Suresh: Shnider and Levinson’s Anesthesia for Obstetrics, ed 5, pp 382–388).
679. Which patients describe their labor pain as being the MOST intense? A. Primipara patients attending prepared childbirth classes B. Primipara patients not attending prepared childbirth classes C. Multipara patients attending prepared childbirth classes D. Multipara patients not attending prepared childbirth classes
679. (B) Labor pain is some of the most intense pain that people can experience. In general, primparous patients have more pain than multiparous patients. Primparous women who have attended prepared childbirth Obstetric Physiology and Anesthesia 197 classes have somewhat less pain than women who have not attended prepared childbirth classes. For women who have experienced labor and delivery, attending prepared childbirth classes does not seem to affect the amount of pain that they experience. Labor pain appears to exceed chronic low back pain, nonterminal cancer pain, postherpetic neuralgia, or the pain from a fracture. Patients with causalgia or patients experiencing an amputation of a digit have more pain than the parturient (Miller: Miller’s Anesthesia, ed 8, p 2339).
680. Which of the following properties of epidurally administered local anesthetics determines the extent to which epinephrine will prolong the duration of blockade? A. Molecular weight B. Lipid solubility C. pKa D. Concentration
680. (B) Epinephrine is primarily added to local anesthetics to check for the IV placement of an epidural catheter, to decrease the vascular uptake of local anesthetics, or to increase the intensity and duration of the block. By producing vasoconstriction of the epidural blood vessels, vascular uptake of the local anesthetic is reduced, allowing more of the drug to enter the nervous tissue. The more lipid soluble the local anesthetic, the less effect epinephrine has (e.g., lidocaine is prolonged much more than bupivacaine when epinephrine is added to the local anesthetic) (Chestnut: Chestnut’s Obstetric Anesthesia, ed 5, pp 288–289).
681. Which intrathecal narcotic can be used as a sole agent for cesarean section (i.e., without an ester or amide local anesthetic)? A. Morphine B. Fentanyl C. Meperidine D. None of the above; a local anesthetic is needed
681. (C) Meperidine is unique among narcotics in that it demonstrates local anesthetic actions in addition to its narcotic effects and can be used alone for anesthesia for cesarean section. The dose is about 1 mg/kg. When added to a local anesthetic for postoperative analgesia, a 10-mg dose of meperidine can produce 4 to 6 hours of analgesia (Chestnut: Chestnut’s Obstetric Anesthesia, ed 5, pp 472, 562).
682. A 23-year-old parturient in the first trimester is brought to the operating room for emergency appendectomy. General anesthesia is planned. Which drug has a U.S. Food and Drug Administration (FDA) Use-In-Pregnancy rating of D (studies in humans and in investigational or postmarketing data demonstrate fetal risk; nevertheless, potential benefits may outweigh potential risk)? A. Nitrous oxide B. Isoflurane C. Midazolam D. None of the above
682. (C) An increased risk of congenital malformations has been suggested by several older studies with the use of minor tranquilizers such as diazepam, meprobamate, and chlordiazepoxide during the first trimester of pregnancy. The cause-and-effect relationship has not been proved; in fact, several newer studies failed to show an association between minor tranquilizers and congenital malformations. The FDA developed a rating schedule for medication use in pregnancy (see table below). FDA USE-IN-PREGNANCY RATINGS Schedule Interpretation A Controlled studies show no risk B No evidence of risk in humans C Risk cannot be ruled out D Positive evidence of risk X Contraindicated in pregnancy The FDA classification for diazepam and midazolam is D (Chestnut: Chestnut’s Obstetric Anesthesia, ed 5, p 364; Suresh: Shnider and Levinson’s Anesthesia for Obstetrics, ed 5, pp 806–809; Physicians’ Desk Reference 2014, ed 68, p 211).
683. True statements regarding inclusion of intrathecal morphine, fentanyl, or sufentanil in obstetric anesthesia practice include each of the following EXCEPT A. The chief site of action is the substantia gelatinosa of the dorsal horn of the spinal column B. There is no motor and no sympathetic blockade C. Pain relief is adequate for the second stage of labor D. Lipophilic narcotics are associated with less respiratory depression than nonlipophilic narcotics
683. (C) Intrathecal opiates (e.g., morphine, fentanyl, sufentanil) are very effective in relieving the visceral pain during the first stage of labor. Intrathecal opiates administered alone (except for meperidine, which has local anesthetic properties) do not provide adequate pain relief for second-stage somatic pain (Chestnut: Chestnut’s Obstetric Anesthesia, ed 5, pp 277–282, 465–468; Suresh: Shnider and Levinson’s Anesthesia for Obstetrics, ed 5, pp 184–187).
684. The MOST common side effect of intraspinal narcotics in the obstetric population is A. Pruritus B. Nausea and vomiting C. Respiratory depression D. Urinary retention 184 Part 2 Clinical Sciences
684. (A) The most common side effect of intraspinal narcotics is pruritus. The next most common side effects are nausea and vomiting, followed by urinary retention and drowsiness. Respiratory depression and headache may occur but are relatively infrequent (Chestnut: Chestnut’s Obstetric Anesthesia, ed 5, pp 283–287; Suresh: Shnider and Levinson’s Anesthesia for Obstetrics, ed 5, pp 185–186).
685. A 110-kg (242-lb), gravida 1, para 0 woman has a blood pressure of 180/95 during an office visit at the 18th week of gestation and 170/95 one week later. She has some ankle but no facial edema, and no protein detected in her urine. These findings would be classified as A. Gestational hypertension B. Preeclampsia C. Chronic hypertension D. Chronic hypertension with superimposed preeclampsia
685. (C) Hypertension (systolic blood pressure [SBP] >140 or an increase >30 mm Hg over baseline; diastolic blood pressure [DBP] >90 or an increase of 15 mm Hg over baseline) occurs in about 7% of all pregnancies. It is classified into four types (preeclampsia-eclampsia, chronic hypertension [of any cause], chronic hypertension with superimposed preeclampsia, and gestational hypertension). Preeclampsia-eclampsia is the new onset of hypertension associated with thrombocytopenia (platelet count <100,000/mm3), impaired liver function, renal insufficiency (serum creatinine >1.1 mg/dL, or doubling of serum creatinine in the absence of any other renal disease), pulmonary edema, or new-onset 198 Part 2 Clinical Sciences cerebral or visual disturbances. Gestational hypertension, which is isolated new-onset hypertension (usually after 37 weeks) that resolves by 12 weeks’ postpartum, is a retrospective diagnosis. Preeclampsia rarely occurs before 20 weeks’ EGA except in patients with gestational trophoblastic neoplasms (e.g., molar pregnancy); if seizures occur in a patient with preeclampsia, the condition is called eclampsia. HELLP syndrome (Hemolysis, Elevated Liver enzymes, and Low Platelet count) is a variant of preeclampsia. Chronic hypertension is persistent hypertension before, during, and after pregnancy (e.g., >6 weeks’ postpartum). Chronic hypertension with superimposed preeclampsia occurs when a patient with chronic hypertension develops preeclampsia. See also Question 645 (American College of Obstetricians and Gynecologists Task Force on Hypertension in Pregnancy, November 2013 Website; Chestnut: Chestnut’s Obstetric Anesthesia, ed 5, pp 825–826; Suresh: Shnider and Levinson’s Anesthesia for Obstetrics, ed 5, pp 437–438).
686. An epidural is placed into a 32-year-old parturient in active labor receiving magnesium therapy for preeclampsia. Five minutes after administration of the test dose, the loading dose of bupivacaine and fentanyl is administered. The patient becomes panic-stricken, wrestles briefly with the reassuring nurses, gasps for air, seizes, and develops cardiovascular collapse. During resuscitation, blood is oozing from the IV sites and a pink froth is noted in the endotracheal tube. The MOST likely diagnosis is A. Amniotic fluid embolism B. High spinal C. Intravascular bupivacaine injection D. Eclampsia
686. (A) Amniotic fluid embolism is a rare condition (5 per 100,000 live births). It presents in a variety of ways but often in a dramatic way, with acute hypoxemia, cardiovascular collapse, DIC, and, in about 50% of cases, a seizure. Patients with a high spinal or epidural may complain of dyspnea, but they also have marked weakness and would certainly not be able to wrestle or struggle with their health care providers. Patients experiencing an intravascular injection of local anesthetic present with central nervous system (CNS) signs of toxicity (light-headedness, visual or auditory disturbances, muscular twitching, convulsion, coma) or, at higher levels, cardiovascular collapse. Magnesium overdosage is also associated with muscle weakness. The typical eclamptic seizure is tonic–clonic. Patients with eclampsia do not complain of dyspnea, although an associated aspiration may produce similar symptoms. See Question 658 (Chestnut: Chestnut’s Obstetric Anesthesia, ed 5, pp 915–920; Suresh: Shnider and Levinson’s Anesthesia for Obstetrics, ed 5, pp 333–348).
687. Which of the following narcotics has the LONGEST duration of action when added during a cesarean section under epidural anesthesia? A. 50 to 100 μg fentanyl B. 10 to 20 μg sufentanil C. 3 to 4 mg morphine D. 50 to 75 mg meperidine
687. (C) Epidural fentanyl (50-100 μg) and epidural sufentanil (10-20 μg) each has a duration of action for about 2 to 4 hours. Epidural meperidine (50-75 mg) has an intermediate duration of action of 4 to 12 hours, whereas epidural morphine (3-4 mg) has the longest duration of action, of 12 to 24 hours (Chestnut: Chestnut’s Obstetric Anesthesia, ed 5, pp 566–567).
688. Which of the following is NOT increased during pregnancy? A. Renal plasma flow B. Creatinine clearance C. Blood urea nitrogen (BUN) D. Glucose excretion
688. (C) The renal system undergoes dramatic anatomic (increase in kidney size as well as dilation of the ureters) and functional changes in pregnancy. Renal plasma flow increases about 75% to 85%, and glomerular filtration rate (GFR) increases about 50% and is reflected by an increase in clearance of urea, creatinine, and uric acid. Because of the increased clearance, we see a decrease in BUN to 8 to 9 mg/dL, serum creatinine to 0.5 to 0.6 mg/dL, and serum urate to 2.0 to 3.0 mg/dL. Glucosuria is common and is attributed to both the increase in GFR and a reduced renal tubular resorption of glucose (Chestnut: Chestnut’s Obstetric Anesthesia, ed 5, p 27; Miller: Miller’s Anesthesia, ed 8, p 2348).
689. Which inhalation anesthetic does NOT produce uterine relaxation? A. Isoflurane B. Sevoflurane C. Nitrous oxide D. All produce uterine relaxation
689. (C) All volatile halogenated anesthetic agents (e.g., halothane, enflurane, isoflurane, desflurane, sevoflurane) cause a dose-related relaxation of uterine smooth muscle. With anesthetic concentrations of 0.2 MAC, the decrease in uterine activity is slight, and these agents have been used for inhalation analgesia during labor. At 0.5 MAC, uterine relaxation is more significant, but the uterine response to oxytocin remains intact. Nitrous oxide does not affect uterine activity (Chestnut: Chestnut’s Obstetric Anesthesia, ed 5, pp 452–454; 575–576; Suresh: Shnider and Levinson’s Anesthesia for Obstetrics, ed 5, pp 156–157, 176–177).
690. Passive diffusion of substances across the placenta is enhanced by all of the following EXCEPT A. Low molecular weight of the substance B. High water solubility of the substance C. Low degree of ionization of the substance D. Large concentration gradient of the drug
690. (B) Passive diffusion is the primary means for the placental transfer of drugs. Factors that promote diffusion of drugs across placental membranes include decreased maternal protein binding (although some believe that this is not very important because of rapid diffusion of drugs from protein), low molecular weight (<500 Da), high lipid solubility (low water solubility), a low degree of ionization, and a large concentration gradient across the membranes. Highly ionized drugs, such as neuromuscular drugs, do not pass the placenta in significant amounts (Chestnut: Chestnut’s Obstetric Anesthesia, ed 5, pp 63–65; Suresh: Shnider and Levinson’s Anesthesia for Obstetrics, ed 5, pp 19–23).
691. Cesarean delivery is associated with a blood loss of about A. 250 mL B. 500 mL C. 750 mL D. 1000 mL
691. (D) The average blood loss associated with a vaginal delivery is about 600 mL and after a cesarean delivery is about 1000 mL (Chestnut: Chestnut’s Obstetric Anesthesia, ed 5, pp 24–25).
692. Which of the following statements is CORRECT in describing differences between fetal and maternal blood during labor? A. Fetal blood has a lower hemoglobin concentration than does maternal blood B. Fetal placental blood flow is twice maternal placental blood flow C. Fetal hemoglobin has a greater affinity for O2 than does maternal hemoglobin D. The fetal oxyhemoglobin dissociation curve is shifted to the right of the maternal oxyhemoglobin dissociation curve
692. (C) The fetus has several compensatory mechanisms for dealing with low O2 pressures (umbilical vein Po2 approximately equal to 30 mm Hg when the mother is breathing room air) to which it is exposed. Obstetric Physiology and Anesthesia 199 These include a higher hemoglobin concentration (15-20 g/dL) and the presence of fetal hemoglobin, which has a greater affinity for oxygen (the fetal oxyhemoglobin dissociation curve is shifted to the left of the maternal oxyhemoglobin dissociation curve). At term, maternal blood flow through the placenta (700 mL/min) is about double the fetal blood flow through the placenta (300-360 mL/min). Fetal blood has a lower pH than maternal blood, which may be related to the higher Paco2 levels seen in fetal blood (Suresh: Shnider and Levinson’s Anesthesia for Obstetrics, ed 5, pp 22–27).
693. In general, morbidly obese patients have a higher incidence of all of the following EXCEPT A. Cesarean deliveries B. Postdural puncture headaches (PDPHs) C. Preeclampsia D. Thromboembolic diseases
693. (B) An obese patient has a body mass index (BMI) greater than or equal to 30 kg/m2, and a morbidly obese patient has a BMI greater than or equal to 40 kg/m2. The obese and morbidly obese patient (28.9% and 8%, respectively, of nonpregnant women of childbirth age in the United States) is at increased risk for several comorbid diseases, including obstructive sleep apnea, diabetes, hypertension, and cardiovascular disease. Obstetric-related increased incidences include gestational diabetes, preeclampsia, thromboembolic diseases, wound infections, postpartum hemorrhage, and cesarean deliveries. The increased incidence of cesarean deliveries may relate to an increase in abnormal presentations, fetal macrosomia, meconium staining, late decelerations in the FHR, and prolonged labor. Anesthetic challenges include increased risk of aspiration, difficulty finding adequate venous access, difficulty with mask ventilation, difficulty with endotracheal intubation, difficulty in performing regional anesthesia, operative positioning, and prolonged surgery. Interestingly, obese and morbidly obese patients appear to have a lower incidence of PDPHs. Etiology for the lower incidence is unclear (Chestnut: Chestnut’s Obstetric Anesthesia, ed 5, pp 1141–1153; Miller: Miller’s Anesthesia, ed 8, p 2349; Suresh: Shnider and Levinson’s Anesthesia for Obstetrics, ed 5, pp 428, 580–592).
694. A term infant with good muscle tone and a strong cry has an oxygen saturation of 83%, breathing room air 5 minutes after delivery. The MOST appropriate action at this point would be A. Supplemental increased oxygen concentration with a blender up to 50% by a face mask B. Spontaneous breathing with 100% oxygen by face mask C. Positive-pressure ventilation with 100% oxygen D. Observation
694. (D) Normal healthy term newborns breathing room air take a while for the oxygen saturations to rise to normal 90% to 95% levels. In caring for the newborn who is not breathing, bag and mask ventilation with room air is now recommended, with targeted preductal oxygen saturations (right hand or wrist) increases of about 5% for each minute of the first 5 minutes of life starting at 1 minute oxygen saturation of 60% to 65% (at 2 minutes 65%-70%, at 3 minutes 70%-75%, at 4 minutes 75%-80% and at 5 minutes 80%-85%). After 5 minutes, oxygen saturation more slowly increases to 85% to 95% by 10 minutes of life. If higher concentrations of oxygen are needed to reach the targeted oxygen saturations (especially in preterm newborns <32 weeks), a blender for oxygen and air can be used. For this newborn, an oxygen saturation of 83% at 5 minutes is appropriate, and observation only is needed (American Heart Association: Part 11 – Neonatal Resuscitation, Circulation 122:S516–S521, 2010; Neonatal Resuscitation Textbook, ed 6, America Heart Association and the American Academy of Pediatrics, pp 37–58).
695. Which condition BEST describes the third-trimester maternal condition with the following signs and symptoms: new-onset vaginal bleeding that stops, no pain, no fetal distress? A. Placenta abruption B. Placenta previa C. Uterine rupture D. Vasa previa
695. (B) Second- and third-trimester obstetric hemorrhage is not uncommon in obstetrics. Placenta previa (where the placenta is near the margin or covering the cervical os) is classically described as painless vaginal bleeding during the second or third trimester that is not associated with maternal shock or fetal distress with the first episode of bleeding. However, with a second or third episode, bleeding may continue. Placental abruption (separation of the placenta from the uterine wall after 20 weeks’ EGA and prior to delivery) more typically is associated with abdominal pain and can be associated with fetal distress. Bleeding for placenta abruption may be revealed or concealed behind the placenta. Uterine rupture usually presents with severe abdominal pain and fetal distress. Vasa previa refers to the velamentous insertion of the fetal vessels over the cervical os, which means that the fetal blood vessels are not protected by the placenta or the umbilical cord and are ahead of the presenting part of the fetus. When the fetal membranes rupture, a tear in a fetal blood vessel may develop, leading to fetal exsanguination (Chestnut: Chestnut’s Obstetric Anesthesia, ed 5, pp 882–888; Hines: Stoelting’s Anesthesia and Co-Existing Disease, ed 6, pp 566–570; Suresh: Shnider and Levinson’s Anesthesia for Obstetrics, ed 5, pp 312–317).
696. During the second stage of labor, complete pain relief can be obtained with A. Paracervical block B. Neuraxial block with fentanyl and morphine C. Pudendal nerve block D. Lumbar epidural block with bupivacaine and no narcotic Obstetric Physiology and Anesthesia 185
696. (D) The first stage of labor starts with the onset of labor and ends with complete cervical dilation (10 cm). It is visceral pain, associated with uterine contractions and dilation of the cervix, and is transmitted via the autonomic nervous system through the sympathetic fibers that pass through the paracervical region and enter the CNS at T10-L1 segments. The second stage of labor includes these pathways and adds the somatic fibers of the birth canal that are transmitted via the pudendal nerve entering the CNS at S2-S4. Neuraxial block (spinal and/or epidural) with only narcotics can be useful for first-stage pain; however, the somatic pain is not well treated with narcotics alone. A local anesthetic–induced lumbar epidural block with or without narcotics can produce complete anesthesia during both first and second stage of labor pain. If a low spinal or saddle block is performed with local anesthetics (covering only 200 Part 2 Clinical Sciences sacral areas), the uterine contraction pain still will be felt. Paracervical blocks block only the first-stage pain. Pudendal blocks block the somatic component during the second stage but not the visceral pain of uterine contractions (Chestnut: Chestnut’s Obstetric Anesthesia, ed 5, pp 412–415, 459–480, 518–527; Suresh: Shnider and Levinson’s Anesthesia for Obstetrics, ed 5, pp 119–133).
697. Anesthetic considerations for open fetal surgery include all of the following EXCEPT A. Uterine relaxation is essential B. Maternal hypotension (mean blood pressure <65 mm Hg) can be treated with phenylephrine or ephedrine C. Vecuronium at the ED95 dose of 0.04 mg/kg should be administered IM or IV by the obstetrician or surgeon if fetal muscle relaxation is needed D. Normal fetal oxygen saturation is 50% to 70%
697. (C) Anesthetic considerations for open fetal surgery include administering anesthesia for the mother and the child, giving excellent uterine relaxation, maintaining an adequate maternal blood pressure, providing muscle relaxation to the fetus if needed, and preventing postoperative premature labor. Uterine relaxation is needed to prevent uterine contractions with possible separation of the placenta from the uterine wall. High-dose volatile anesthetics (e.g., 2 or 3 MAC) can provide excellent maternal anesthesia as well as uterine relaxation and anesthesia for the fetus. If additional anesthesia is needed, IV narcotics can be used (e.g., remifentanil infusions are often used). If one chooses to use a lower dose of volatile anesthetics, nitroglycerin infusion can be used to keep the uterus from contracting. Maternal hypotension (mean blood pressure <65) is not uncommon and is treated with more left uterine tilt, fluids, and, if needed, phenylephrine or ephedrine. Monitoring the fetal oxygen saturation reveals normal values of 50% to 70%; values less than 50% signal impaired placental perfusion (e.g., maternal hypotension, cord compression). If the obstetrician needs the fetus to be paralyzed, then a neuromuscular blocking drug must be given directly into the fetus because placental transfer is poor. The dose, however, must be larger than if the fetus were delivered because the blood volume of the fetus includes the placental blood as well as the blood in the fetus. Typically the dose is about four times the effective dose in 95% of subjects (ED95) or for vecuronium is 0.2 mg/kg. Magnesium sulfate should be started to decrease the chance of premature labor at the end of the surgery as the volatile anesthetic concentration is decreased or the nitroglycerin infusion is discontinued. One should recall that the magnesium sulfate potentiates neuromuscular blocking drugs significantly (Chestnut: Chestnut’s Obstetric Anesthesia, ed 5, pp 135–141; Suresh: Shnider and Levinson’s Anesthesia for Obstetrics, ed 5, pp 792–799).
698. 15-Methyl PGF2α is administered directly into the myometrium to treat uterine atony in a 28-year-old mother. Possible effects from treatment with this drug include A. Nausea and vomiting B. Bronchospasm C. Hypoxia D. All of the above
698. (D) 15-Methyl PGF2α (carboprost, Hemabate) is the preferred prostaglandin for use in the treatment of refractory uterine atony (after oxytocin). The dose is 0.25 mg injected intramuscularly or directly into the uterine wall, repeated as needed every 15 to 30 minutes with a maximum total dose of 2 mg. It has several important side effects, such as bronchospasm, ventilation-to-perfusion mismatch with an increase in intrapulmonary shunting, and hypoxemia. Other side effects include gastrointestinal spasms (e.g., nausea, vomiting, and diarrhea) (Chestnut: Chestnut’s Obstetric Anesthesia, ed 5, p 891; Suresh: Shnider and Levinson’s Anesthesia for Obstetrics, ed 5, p 321).
699. Which of the following statements regarding MgSO4 therapy for preeclampsia is TRUE? A. The therapeutic range for serum magnesium is 10 to 15 mEq/L B. High serum magnesium levels can be estimated by changes in deep tendon patellar reflexes in a patient with an epidural anesthetic loaded for a cesarean section C. Excessive serum magnesium levels cause widening of the QRS complex D. As soon as delivery occurs, the chance for eclampsia no longer exists and the magnesium should be reversed so that postpartum bleeding is less likely to occur
699. (C) International consensus states that magnesium sulfate (MgSO4) is the anticonvulsant of choice in the preeclamptic patient. In addition to its anticonvulsant effect, MgSO4 has many other actions on skeletal and cardiac muscles. MgSO4 is usually started as an intravenous bolus of 6 g over 20 minutes followed by an infusion of 2 g/hr (provided that kidney function is normal). Clinical monitoring for toxicity is performed looking at deep tendon reflexes, and blood levels are often performed and reported in either mEq/L or mg/dL (1 mEq/L = 1.22 mg/dL). The therapeutic range for serum MgSO4 is 4 to 8 mEq/L (4.8-9.6 mg/dL). In an unanesthetized patient, a loss of deep tendon reflexes occurs at 10 mEq/L (12 mg/dL), respiratory arrest occurs at 15 mEq/L (18 mg/dL), and cardiac arrest occurs at 25 mEq/L (30 mg/dL). As long as deep tendon reflexes are present, significant toxicity is unlikely. In a patient with an epidural or spinal anesthetic loaded for a cesarean section, the patellar reflex is often depressed by the local anesthetic; estimation of deep tendon reflexes should be done with the biceps tendon (unless a total spinal develops). Electrocardiographic (ECG) changes, including PR interval prolongation and QRS complex widening, occur at serum levels of 5 to 10 mEq/L (6-12 mg/dL), sinoatrial and atrioventricular block at 15 mEq/L (18 mg/dL), and cardiac arrest at levels greater than 25 mEq/L (30 mg/dL). The treatment for magnesium toxicity is calcium. The dose of 1 g of calcium gluconate (10 mL of a 10% solution) can be administered slowly over at least 2 minutes to treat high magnesium levels. Rapid administration may take away the anticonvulsant effects, so careful slow titration is recommended. About 60% of eclamptic seizures occur before delivery. Most postpartum seizures develop in the first 24 hours after delivery, but eclamptic seizures may occur as late as 22 days after delivery (Miller: Miller’s Anesthesia, ed 8, p 2348; Suresh: Shnider and Levinson’s Anesthesia for Obstetrics, ed 5, 448–449).
700. While moving a parturient from the birthing room to the operating room for an emergency cesarean section for a prolapsed umbilical cord, the patient develops cough, wheezing, and stridor and becomes cyanotic. The trachea is intubated, and food is noted in the pharynx. Appropriate treatment in this patient should consist of A. Intravenous lidocaine to suppress the cough B. Glucocorticoids C. 100% oxygen and positive end-expiratory pressure (PEEP) D. Saline lavage
700. (C) Three different aspiration syndromes have been described in the general population: aspiration of particulate matter causing airway obstruction, aspiration of acid fluid causing aspiration pneumonitis Obstetric Physiology and Anesthesia 201 (Mendelson syndrome), and aspiration of gram-positive, gram-negative, and anaerobic bacteria causing aspiration pneumonia. Aspiration pneumonia has the highest mortality rate but fortunately occurs only with an associated bowel obstruction, which is rarely a problem in obstetrics. Symptoms of aspiration pneumonitis include coughing, tachypnea, tachycardia, bronchospasm, and hypoxemia. Treatment is supportive and includes the Heimlich maneuver if a large foreign body is lodged in the trachea (which is unlikely in the fasting laboring patient), endotracheal intubation, suctioning the airway to remove particulate material, administration of increased concentrations of oxygen, and application of PEEP to achieve oxygenation goals as needed (prophylactic PEEP does not provide any benefit). Coughing is due to the airway irritation and is most effectively decreased with muscle paralysis. Intravenous lidocaine would not be effective. Use of saline or bicarbonate lavage does not decrease lung damage and can worsen hypoxemia. Glucocorticoids or other anti-inflammatory drugs have not been effective in limiting the inflammation and may increase the risk of secondary bacterial infection (Chestnut: Chestnut’s Obstetric Anesthesia, ed 5, pp 671–675; Suresh: Shnider and Levinson’s Anesthesia for Obstetrics, ed 5, pp 403–405).
701. Aortocaval compression starts to become significant in a normal pregnancy at how many weeks EGA? A. 10 weeks B. 15 weeks C. 20 weeks D. 25 weeks
701. (C) Aortocaval compression typically is not a problem until about 18 to 20 weeks’ gestation, when the uterus is large enough to compress the aorta and vena cava when the patient assumes the supine position. If the uterus is larger than normal (e.g., multiple gestations or polyhydramnios), then aortocaval compression may appear earlier (Chestnut: Chestnut’s Obstetric Anesthesia, ed 5, p 340; Suresh: Shnider and Levinson’s Anesthesia for Obstetrics, ed 5, p 5).
702. Which agent is the MOST useful for raising the gastric pH just before induction of general anesthesia for emergency cesarean section? A. Ranitidine B. Sodium citrate C. Metoclopramide D. Magnesium hydroxide and aluminum hydroxide
702. (B) Cimetidine and ranitidine are H2-receptor antagonists that will increase gastric pH but take at least 30 minutes to work. Metoclopramide is not an antacid but may be useful by increasing the lower esophageal sphincter tone. Only liquid antacids raise gastric pH quickly. Sodium citrate, a clear nonparticulate antacid (0.3 M sodium citrate) is preferred over particulate antacids (aluminum hydroxide, magnesium trisilicate, magnesium hydroxide) because clear nonparticulate antacids cause less pulmonary damage if aspirated. Sodium citrate 30 mL neutralizes 255 mL of HCl with a pH of 1.0. Neutralization of gastric acid occurs rapidly (i.e., <5 minutes) and will last up to an hour (Chestnut: Chestnut’s Obstetric Anesthesia, ed 5, pp 675–677; Suresh: Shnider and Levinson’s Anesthesia for Obstetrics, ed 5, pp 407–408).
703. Causes of fetal bradycardia include all of the following EXCEPT A. Maternal smoking of cigarettes B. Neostigmine and glycopyrrolate reversal of neuromuscular blockers C. Acidosis D. Umbilical cord compression
703. (A) Causes of fetal bradycardia (FHR <110 beats/min) include hypotension, excessive uterine activity, hypoxemia, acidosis, complete heart block, and some drugs. Atropine readily crosses the placenta but at low doses does not seem to cause fetal tachycardia; at high doses, it may produce tachycardia. The combination of neostigmine, which crosses the placenta slightly, and glycopyrrolate, which does not cross the placenta well, has been associated with fetal bradycardia, which is why neostigmine with atropine is preferred when reversing neuromuscular blockers if a fetus is present. Bradycardias are associated with early decelerations (head compression with vagal stimulation), late decelerations (fetal hypoxemia with vagal stimulation or myocardial failure), and variable decelerations (umbilical cord compressions with vagal stimulation). Causes of fetal tachycardia (FHR >160 beats/min) include infection, fever, maternal cigarette smoking, fetal paroxysmal supraventricular tachycardia, and some drugs (ritodrine, terbutaline, atropine) (Chestnut: Chestnut’s Obstetric Anesthesia, ed 5, pp 68, 150–159; Suresh: Shnider and Levinson’s Anesthesia for Obstetrics, ed 5, pp 69–73, 843).
704. Most cases of cerebral palsy (CP) are due to conditions during A. Antepartum B. Labor C. Delivery D. The first 30 days of life
704. (A) CP is a nonprogressive disorder of the CNS arising from lesions in the brain that occurred during development (in utero 75%, at birth 10%, soon after birth 15%). CP is associated with impairment of motor function. Mental retardation may or may not be present and is not an essential diagnostic criterion. The cause is unknown and most likely multifactorial. Associated conditions include maternal mental retardation (now called intellectual disability), birth weight of less than 2000 g and fetal malformations, breech presentation (but not breech vaginal delivery), severe proteinuria during the second half of pregnancy, third-trimester bleeding, and gestational age less than 32 weeks, but many other factors may play a role. It occurs in about 2 per 1000 live births. At one time, FHR monitoring was thought to be able to prevent CP, but this has not happened. In fact, among patients with new-onset late deceleration patterns, the false-positive rate is 99% if used to predict the development of CP. This is not to say that intrapartum asphyxial insults do not cause damage; they might, and they probably account for some cases of CP. There is also a very weak association of low Apgar scores and CP; in fact, most children who develop CP had a 5-minute Apgar score that was normal (Chestnut: Chestnut’s Obstetric Anesthesia, 202 Part 2 Clinical Sciences ed 5, pp 193–197; Miller: Miller’s Anesthesia, ed 8, p 2337; Suresh: Shnider and Levinson’s Anesthesia for Obstetrics, ed 5, pp 68–69).
705. All of the following statements regarding pregnant diabetic patients are true EXCEPT A. Gestational diabetes mellitus (DM) occurs in about 7% of all pregnancies in the United States B. Insulin readily crosses the placenta and causes larger babies C. Cesarean section is more common in diabetic pregnancies D. Diabetic ketoacidosis (DKA) occurs in 1% to 2% of type 1 DM pregnancies
705. (B) DM is the most common endocrine problem associated with pregnancy. Type 1 DM (due to a decrease in insulin secretion) occurs in 1 of every 700 to 1000 gestations. Gestational diabetes, which occurs only during pregnancy, is currently seen in about 7% of all pregnancies in the United States. Although substantial advances in the obstetric and anesthetic management of diabetic parturients have been made, maternal and fetal mortality are still higher in these patients than in parturients without diabetes. DKA has decreased from 9% to currently around 1% to 2% of type 1 DM pregnancies. One important goal of insulin therapy in these patients is to avoid both hyperglycemia and hypoglycemia. In general, insulin requirements in type 1 diabetic patients initially decrease during early pregnancy to their lowest requirement by around 16 to 18 weeks (10%-20% reduction in dose), then increase above prepregnant values around 26 weeks to reach values that are highest at term (50% above prepregnant dose). The dose requirements then rapidly decrease at the time of delivery. Insulin does not readily cross the placenta and therefore does not have any direct effects on glucose metabolism in the fetus. Glucose, however, readily crosses the placenta. Preeclampsia and large-for-gestational-age fetuses occur more frequently in parturient women with diabetes. Because of fetal macrosomia, cesarean section is more common in diabetic than nondiabetic patients (Chestnut: Chestnut’s Obstetric Anesthesia, ed 5, pp 1003–1012; Suresh: Shnider and Levinson’s Anesthesia for Obstetrics, ed 5, pp 462–472).
706. In addition to the postural component of a postdural puncture headache (PDPH), signs and symptoms may include any of the following EXCEPT A. Double vision B. Hearing changes C. Neck stiffness D. Fever
706. (D) PDPHs are positional headaches (exacerbated by sitting or standing and relieved with recumbency) that usually present within 48 hours of a dural puncture (but could take up to a week to present) and typically resolve in 2 to 14 days. They are bilateral and typically located in the frontal or occipital regions. In one prospective series of nonobstetric patients with PDPH, symptoms included nausea (60%), vomiting (24%), neck stiffness (43%), ocular changes (photophobia, diplopia, difficulty in accommodation) (13%), and auditory changes (hearing loss, hyperacusis, tinnitus) (12%). Although postpartum seizures have been associated with PDPH, other etiologies are more likely. Seizures, lethargy, fever, nuchal rigidity, focal neurologic deficits (other than listed above), and a unilateral location suggest other headache etiologies (Chestnut: Chestnut’s Obstetric Anesthesia, ed 5, pp 713–721; Suresh: Shnider and Levinson’s Anesthesia for Obstetrics, ed 5, pp 425–430).
707. Early decelerations may occur in response to A. Fetal head compression B. Uteroplacental insufficiency C. Maternal hypotension D. Umbilical cord compression
707. (A) There are several periodic FHR patterns. Accelerations in FHR in response to fetal movement signify fetal well-being. Decelerations are a decrease in FHR of at least 15 beats/min that last at least 15 seconds. Early decelerations are decreases in FHR that are usually less than 20 beats/min and occur concomitantly with uterine contractions. Typically they are smooth and are mirror images of the uterine contractions. They are not associated with fetal compromise and are caused by head compression, which produces a vagal slowing of the FHR. Late decelerations are decreases in FHR that occur 10 to 30 seconds after the onset of a contraction and end 10 to 30 seconds after the end of a contraction. They are due to uteroplacental insufficiency and can result whenever uterine blood flow decreases. The delayed onset is due to the time required to sense a low oxygen tension. The decrease in FHR may be a vagal reflex (mild cases) or may be due to direct myocardial depression from hypoxia (severe cases). Typically, in severe cases, beat-to-beat variability is decreased or absent as well. Variable decelerations are abrupt decreases in FHR that vary in shape, depth, and duration from contraction to contraction. They are thought to be due to transient umbilical cord compression. A sinusoidal pattern is a regular smooth wavelike pattern with no short-term variability. It may be caused by severe fetal anemia or may result from the maternal administration of narcotics (Chestnut: Chestnut’s Obstetric Anesthesia, ed 5, p 101; Suresh: Shnider and Levinson’s Anesthesia for Obstetrics, ed 5, pp 71–73, 245).
708. Agents that are useful for decreasing the incidence of shivering during cesarean section under regional anesthesia or for treating shivering include all of the following EXCEPT A. Administration of intrathecal local anesthetic with fentanyl and/or morphine B. Intravenous magnesium sulfate C. Administration of epidural local anesthetic solutions with epinephrine D. Intravenous meperidine 186 Part 2 Clinical Sciences
708. (C) Shivering occurs in 15% to 20% of all normal vaginal deliveries. The frequency increases from 20% to 85% of patients receiving epidural or spinal anesthesia for cesarean deliveries. The postulated reason is that neuraxial anesthesia impairs centrally mediated peripheral vasoconstriction and shivering thresholds and allows greater environmental heat loss (core to peripheral heat redistribution). Intrathecal narcotics (e.g., especially fentanyl with morphine) and epidural narcotics (e.g., fentanyl, sufentanil, meperidine, butorphanol), when added to local anesthetics, decrease the incidence of maternal shivering. Intravenous meperidine (25 mg), clonidine (75 μg), ketanserin (10 mg), magnesium sulfate (30 mg/kg), or dexmedetomidine decrease the incidence of shivering. Warming the epidural anesthesia solution to body temperature has no effect on the incidence of shivering; however, adding epinephrine to the local Obstetric Physiology and Anesthesia 203 anesthetic appears to increase the frequency of shivering (Chestnut: Chestnut’s Obstetric Anesthesia, ed 5, pp 483, 588–589, 646).
709. An umbilical arterial blood gas sample at the time of an emergency cesarean delivery shows a Po2 of 20 mm Hg, a Pco2 of 50 mm Hg, a bicarbonate value of 22 mEq/L, and a pH of 7.25. This shows A. Severe hypoxemia B. Respiratory acidosis C. Metabolic acidosis D. Normal values
709. (D) The values listed in the question are normal umbilical cord values. The chart is modified from values listed in the Chestnut and Suresh references (Chestnut: Chestnut’s Obstetric Anesthesia, ed 5, pp 170–171; Suresh: Shnider and Levinson’s Anesthesia for Obstetrics, ed 5, p 246). NORMAL VALUES FOR UMBILICAL CORD BLOOD Cord Blood pH Pco2 (mm Hg) Po2 (mm Hg) Bicarbonate (mEq/L) Arterial 7.25 50 20 22 Venous 7.35 40 30 20 From Chestnut DH et al: Chestnut’s Obstetric Anesthesia: Principles and Practice, ed 4, Philadelphia, Mosby, 2009, pp 161–162.
710. Which condition MOST frequently requires blood transfusions during or after a cesarean delivery? A. Multiple gestations B. Placenta abruption C. Placenta previa D. Postpartum hemorrhage
710. (D) For all obstetric-related admissions, the incidence of transfusion of blood is less than 1%. The most common reason for transfusion was postpartum hemorrhage. Estimates suggest that about one third of transfusions were not appropriate with current guidelines (Chestnut: Chestnut’s Obstetric Anesthesia, ed 5, pp 888, 899–902).
711. All of the following are appropriate techniques or drug doses to be used in resuscitating a depressed term newborn EXCEPT A. Begin ventilation with air rather than 100% oxygen B. If the heart rate is less than 60 beats/min, start chest compressions (ratio of chest compressions to ventilations is 3:1) C. After adequate ventilation and chest compressions, administer 0.1 mg/kg of epinephrine IV D. After 10 minutes of no detectable heart rate, it may be reasonable to discontinue resuscitation efforts
711. (C) Resuscitation guidelines continue to evolve. The American Heart Association 2010 Consensus Guidelines for Neonatal Resuscitation recommends beginning resuscitation of a newborn by first opening the airway, then warming, drying, and stimulating the newborn for the first 30 seconds of life. If in 30 seconds the heart rate is less than 100 beats/min or the newborn is gasping or apneic, then positivepressure ventilation is started and oximetry monitoring is suggested. In term newborns, resuscitation is started with air rather than with 100% oxygen. However, preterm newborns (<32 weeks’ gestation) may need air blended with oxygen to reach adequate oxygen saturations. The oximetry probe should be placed preductally (i.e., on the right wrist) to assess oxygenation. If, after the first minute of life,the heart rate is less than 100 beats/min, one should ensure adequate positive-pressure ventilation and consider endotracheal intubation. If the heart rate is now less than 60 beats/min, chest compressions should begin, with a chest compression-to-ventilation ratio of 3:1. At this point the newborn receives 30 breaths and 90 compressions/min (e.g., one and two and three and breath). If the newborn is known to have a cardiac etiology, then a higher compression-to-ventilation ratio should be considered (e.g., 15:2). If the heart rate is less than 60 beats/min after chest compressions and positive-pressure ventilation have been started for at least 30 seconds, consider administering epinephrine. The correct dose is 0.01 mg/kg IV. If the newborn is intubated and IV access has not yet been achieved, consider administering a higher dose of epinephrine such as 0.05 to 0.1 mg/kg down the endotracheal tube (the higher dose is used because blood levels are unpredictable after endotracheal instillation). In a newborn with blood loss, volume expansion is needed and can be achieved with normal saline, Ringer’s lactate, or type O Rh-negative blood. There is little evidence of any benefit with volume expansion in the absence of blood loss. Rarely will a narcotic antagonist (e.g., naloxone), sodium bicarbonate, or a vasopressor be needed for resuscitation. If after 10 minutes there is no detectable heart rate, it may be appropriate to discontinue resuscitation (although many factors can contribute to continue resuscitation beyond 10 minutes) (American Heart Association; Part 11 – Neonatal Resuscitation, Circulation 122, pp S516–S523).
712. After a vaginal delivery under epidural anesthesia, a healthy 8-lb baby is born. The 23-year-old now gravida 1, para 1 woman is noted to have a temperature of 38.2° C. A leukocyte count is obtained and is 15,000/ mm3. The most appropriate course of action would be to A. Get a blood culture B. Start antibiotics C. Administer a sedative D. Observe
712. (D) Although epidural anesthesia often causes a fall in body temperature (due to the vasodilation and redistribution of body heat and loss to the environment), some women develop a rise in body temperature even though there is no evidence of infection. This rise in body temperature of greater than 38° C (100.4° F) usually occurs only when the epidural was used for at least 4 to 5 hours (frequency of 1%-36% of patients). The etiology of this rise in temperature in some women is unclear but includes three main factors (thermoregulatory, effect of systemic opioids, and inflammation). Epidural anesthesia may decrease sweating and the hyperventilation associated with labor, as well as shivering, which may increase body temperature. The use of intravenous systemic opioids may decrease the incidence of fever. Inflammation may play an important role because maternal temperatures are similar in women with or without epidural anesthesia when histologic examination of the placentas reveals the absence of placental inflammation. It may be that the temperature rise was merely an association with obstetric 204 Part 2 Clinical Sciences factors such as nulliparity with prolonged labor, more frequent cervical examinations, prolonged rupture of membranes, or early chorioamnionitis. The prepregnant blood leukocyte count of 6000/mm3 rises during pregnancy to 9000 to 11,000/mm3. During labor the leukocyte count increases to 13,000/mm3, and during the first postpartum day is on average 15,000/mm3 (Chestnut: Chestnut’s Obstetric Anesthesia, ed 5, pp 25, 867–871).
713. Compared with a healthy 25-year-old primigravida, which of the following conditions is NOT associated with a significantly higher incidence of hypertensive disorders of pregnancy? A. Multiple gestations B. Cigarette smoking (>1 pack/day) C. Obesity D. Placental abruption
713. (B) Although the cause of hypertensive disorders of pregnancy (HDP) is not known, many factors are associated with a higher frequency of HDP. Factors include nulliparous woman, age (especially <20 years and >40 years), family history of HDP or a previous history of HDP, some chronic medical conditions (e.g., hypertension, diabetes, obesity, thrombotic vascular disease), some obstetric conditions (e.g., placental abruption, intrauterine growth restriction, fetal death) and conditions in which the uterus is rapidly enlarging (e.g., multiple gestations, polyhydramnios, hydatidiform mole). Although smoking is associated with many adverse pregnancy outcomes, there appears to be a lower incidence of HDP in women who smoke (Chestnut: Chestnut’s Obstetric Anesthesia, ed 5, pp 827–829; Suresh: Shnider and Levinson’s Anesthesia for Obstetrics, ed 5, pp 683, 689).
714. Adverse effects (on the mother) associated with aortocaval compression by the gravid uterus include A. Nausea and vomiting B. Changes in cerebration C. Fetal distress D. All of the above
714. (D) Aortocaval compression, as its name suggests, produces both compression of the aorta (increase in afterload) as well as compression of the vena cava (decrease in venous return). The patient’s response is variable. Although some women have no symptoms, up to 15% of pregnant patients at term will, over several minutes in the supine position, develop hypotension and bradycardia (also called the supine hypotension syndrome). Some women will actually show an increase in brachial artery blood pressure due to the increase in afterload. These women may have a condition referred to as concealed hypotension (blood pressure above the compression that is adequate but blood pressure below the compression that is reduced). Because the blood supply to the uterus is distal to the aortic compression and uterine blood flow is decreased, the fetus may develop fetal distress. Other signs and symptoms of aortocaval compression include nausea, vomiting, pallor, and changes in cerebration (Chestnut: Chestnut’s Obstetric Anesthesia, ed 5, p 18; Suresh: Shnider and Levinson’s Anesthesia for Obstetrics, ed 5, p 5).
715. Which of the following statements regarding a pregnant patient abusing cocaine is FALSE? A. Hypertension, arrhythmias, myocardial ischemia, and tachycardia may occur with the rapid-sequence induction of general anesthesia in the acutely intoxicated patient B. The MAC for general anesthetics is increased in chronic cocaine addicts C. Some states consider in utero drug exposure to be a form of child abuse and require physicians to report these patients D. If a vasopressor is needed to treat hypotension, phenylephrine is preferred over ephedrine
715. (B) Cocaine can produce life-threatening complications that are usually related to the accumulation of catecholamines, and patients may present with the classic signs of toxemia (i.e., hypertension and proteinuria) as well as chest pain. The typical half-life of cocaine is 30 to 90 minutes, but the acute effects can last as long as 6 hours. Because some states consider in utero cocaine exposure a form of child abuse that requires physicians to report positive drug tests in pregnant women, many patients who use cocaine have no prenatal care. Urine tests may be positive for 24 to 72 hours after cocaine use (depending on the amount used). Life-threatening events are more common with general than regional anesthesia. The most frequent problem with induction of general anesthesia is severe hypertension. Arrhythmias, myocardial ischemia, and tachycardia may also occur with the induction of general anesthesia. Labetalol and nitroglycerin have been used to treat these conditions. The MAC level is increased in patients who are acutely intoxicated, whereas patients chronically abusing cocaine have a lower MAC (due to the depletion of catecholamines). These patients are at risk for hypotension, which is commonly seen after the induction of regional anesthesia for cesarean section. Ephedrine may not be an effective vasopressor in these catecholamine-depleted patients. Phenylephrine, a direct-acting drug, is a better vasopressor (Chestnut: Chestnut’s Obstetric Anesthesia, ed 5, pp 1204–1207; Suresh: Shnider and Levinson’s Anesthesia for Obstetrics, ed 5, pp 690–692).
716. Each of the following is correct when advising the surgeon to perform infiltration anesthesia for an emergency cesarean delivery when general and neuraxial anesthesia is contraindicated EXCEPT A. A midline incision is most desirable B. The rectus muscle should be injected to provide good skin analgesia C. Bupivacaine with bicarbonate is the local anesthetic of choice D. Mild sedation with ketamine and midazolam is permissible
716. (C) In cases of emergency cesarean section when general anesthesia is contraindicated (e.g., poor airway when one questions one’s ability to intubate and/or ventilate the patient), and neuraxial anesthesia is contraindicated (e.g., severe hypovolemia or coagulopathy), emergency infiltration anesthesia is acceptable. All of the choices are correct except the choice of a local anesthetic. As the surgeon will be injecting a fair volume of local anesthetic (often 100 mL), and as bupivacaine has a slow onset and potentially dangerous cardiac toxicity with large doses, bupivacaine is a poor choice. A dose of 0.5% lidocaine (plasma half-life of 90 minutes) is often used because it is readily available and relatively safe. Chloroprocaine may be safer because it also has a fast onset and its plasma half-life is extremely short (23 seconds). Both midazolam and ketamine may lead to some amnesia for the patient, which may be advantageous in this emergency situation; however, too much of the IV drugs could obtund the patient and may lead to aspiration of gastric contents. A good coach at the head of the bed may be invaluable for reassuring the patient as to the care (Chestnut: Chestnut’s Obstetric Anesthesia, ed 5, pp 577–578). Obstetric Physiology and Anesthesia 205
717. A 24-year-old primiparous woman is undergoing an elective cesarean section (breech position). After prehydration with 1500 mL of saline, a spinal anesthetic is performed; 5 minutes later, the blood pressure is noted to be 80/40 mm Hg and the heart rate is 110 beats/min. The BEST treatment (best fetal pH) after ensuring that adequate left uterine displacement is performed would be A. Phenylephrine B. Ephedrine C. Epinephrine D. 1000 mL 5% dextrose in lactated Ringer’s solution
717. (A) The most common complication after a spinal or epidural anesthetic is placed is systemic hypotension. Because the cardiac output is influenced by four main factors (preload, afterload, contractility, and heart rate and rhythm), treatment is directed at these factors. First, consideration of more left uterine displacement is made (which can increase preload). Next, administering more intravenous fluids to increase preload is done if the preload of fluid administration is inadequate. Intravenous fluids with dextrose are used only for maintenance fluids and should not be used to prevent or treat hypotension from regional anesthesia because the fluid load causes significant maternal and fetal hyperglycemia and hyperinsulinemia. After delivery, the sugar supply for the newborn stops but the insulin response continues, often causing fetal hypoglycemia after delivery. It should be noted that 5% albumin solutions are expensive and are not recommended for routine use to treat hypotension. Consideration of the use of vasopressors and/or drugs that increase contractility are commonly needed to increase afterload and increase cardiac contractility. Initial laboratory studies with pregnant ewes suggested that ephedrine was a better choice compared with phenylephrine and other α-adrenergic agonists, when looking at changes in uterine blood flow. In these initial studies the blood pressure was raised from normal to higher levels, and ephedrine was the drug of choice because phenylephrine decreased uterine blood flow, whereas ephedrine did not. However, raising a normal pressure to higher levels is not the same thing as raising a low blood pressure to normal. In more recent human studies looking at ephedrine and phenylephrine use, studies have noted no difference in the prophylactic or treatment use of these drugs for maternal hypotension. It was also noted that maternal bradycardia was more common with maternal phenylephrine administration, whereas maternal tachycardia was more common with maternal ephedrine administration; also, neonatal arterial pH was slightly higher when phenylephrine was used as compared with ephedrine. Why this occurs is unclear but may be related to ephedrine’s ability to cross the placenta causing β-adrenergic stimulation in the newborn (F/M blood ratio is 0.7 for ephedrine and 0.2 for phenylephrine). In this patient who has left uterine displacement, adequate IV hydration, and a heart rate of 110 beats/min, phenylephrine would be the preferred vasopressor. If the mother has hypotension with bradycardia, ephedrine might be a better choice. Epinephrine is rarely needed but should be available and used when there is severe hypotension that is not responsive to phenylephrine or ephedrine, especially when there is associated fetal bradycardia (Chestnut: Chestnut’s Obstetric Anesthesia, ed 5, pp 480–481, 580–583; Suresh: Shnider and Levinson’s Anesthesia for Obstetrics, ed 5, pp 50, 135–136, 174).
718. A woman has been admitted for a dilation and evacuation (D&E) at 10 weeks’ EGA. She has some persistent bleeding and cramping after the expulsion of some tissue. Her obstetric condition is called A. A threatened abortion B. An inevitable abortion C. A complete abortion D. An incomplete abortion
718. (D) A threatened abortion is defined as uterine bleeding without cervical dilation before 20 weeks’ gestation. Bleeding may be accompanied by uterine cramps or backache. Half of these cases will go on to spontaneously abort. An inevitable abortion has cervical dilation and/or rupture of membranes and will spontaneously abort. A complete abortion occurs when there is complete expulsion of the fetus and the placenta, and in these cases there is no need for a dilation and curettage (D&C). If there is only partial expulsion of tissue, as in this case, an incomplete abortion has occurred, and this requires a D&E to remove the remaining fetal or placental tissue. In these cases the cervix has usually dilated some and the patient usually can be managed with some mild sedation, because the most painful part of a D&E is cervical dilation. A paracervical block can be most useful for pain control during the procedure if the cervix needs to be dilated. A fetal death that is unrecognized for several weeks is called a missed abortion, and if this occurs at an advanced gestational age, DIC may result. A habitual or recurrent abortion refers to the occurrence of three or more consecutive spontaneous abortions (Chestnut: Chestnut’s Obstetric Anesthesia, ed 5, pp 345–348).
719. The action of epidural narcotics is antagonized by the prior or concomitant administration of which of the following epidurally administered local anesthetics? A. Lidocaine B. Bupivacaine C. Ropivacaine D. Chloroprocaine Obstetric Physiology and Anesthesia 187
719. (D) 2-Chloroprocaine administered epidurally appears to decrease the quality and duration of subsequently administered fentanyl or morphine and also to reduce the effectiveness of bupivacaine. The exact mechanism is unclear but does not seem to be related to the acid pH of chloroprocaine (because neutralization with bicarbonate has similar antagonistic properties). Butorphanol (a κ-receptor agonist) does not appear to be antagonized (Chestnut: Chestnut’s Obstetric Anesthesia, ed 5, p 272; Suresh: Shnider and Levinson’s Anesthesia for Obstetrics, ed 5, p 172).
720. Factors associated with advanced molar pregnancy (i.e., >14- to 16-week size uterus) include all of the following EXCEPT A. Hypertensive disorders of pregnancy B. Hypothyroidism C. Acute cardiopulmonary distress D. Hyperemesis gravidarum
720. (B) Earlier diagnosis of complete molar pregnancies has decreased the incidence of medical complications. However, excessive uterine size occurs in up to one half of patients with a complete molar pregnancy and is associated with a high incidence of medical complications. Medical complications when the uterine size is greater than 14 to 16 weeks’ gestational size include ovarian theca-lutein cysts (4%-50%), hyperemesis gravidarum (15%-30%), hypertensive disorders of pregnancy (11%-27%), anemia (hemoglobin 206 Part 2 Clinical Sciences <10 g/dL) (10%-54%), acute cardiopulmonary distress (6%-27%), malignant sequelae (metastasis) (4%-36%), and hyperthyroidism (1%-7%) (Chestnut: Chestnut’s Obstetric Anesthesia, ed 5, pp 351–354).
721. Refractory cardiac arrest is MOST likely after the rapid unintentional IV injection of which of the following local anesthetics? A. Lidocaine B. Bupivacaine C. Ropivacaine D. Chloroprocaine
721. (B) Several cases of maternal cardiac arrest have occurred in pregnant women who were administered bupivacaine (Marcaine, Sensorcaine). Typically, the patients received an unintentional IV bolus of 0.75% bupivacaine intended for the epidural space. They had a brief grand mal seizure followed by cardiovascular collapse. Successful treatment was often prolonged and involved basic resuscitation (intubation, ventilation with 100% oxygen, cardiac compression with left uterine tilt, defibrillation, epinephrine, vasopressin, atropine), as well as rapid delivery of the fetus (if possible within 4-5 minutes). Delivery of the fetus makes successful resuscitation of the mother more likely. Incremental small injections of local anesthetic looking for toxicity should decrease the chance for cardiovascular collapse. Bupivacaine 0.75% now is considered contraindicated for use in the epidural space of parturients. Recent literature (since 2006) has shown that the IV injection of 20% Intralipid may be helpful (see Question 722). Both levobupivacaine (Chirocaine) and ropivacaine (Naropin) were developed to have a long duration of action, like bupivacaine but with less cardiac toxicity. Although these compounds have less cardiac toxicity than bupivacaine, they are more cardiac toxic than lidocaine (intermediate duration of action) and chloroprocaine (short duration of action) (ASRA.com – Downloadable checklist for Treatment of Local Anesthetic Systemic Toxicity 9/19/11; Chestnut: Chestnut’s Obstetric Anesthesia, ed 5, p 266; Miller: Miller’s Anesthesia, ed 7, pp 932–934; Suresh: Shnider and Levinson’s Anesthesia for Obstetrics, ed 5, pp 108–109).
722. American Society of Regional Anesthesia (ASRA) guidelines for the treatment of local anesthetic systemic toxicity (LAST) for cardiac arrhythmias include the use of Intralipid and the AVOIDANCE of all of the following drugs EXCEPT A. Vasopressin B. β-Blockers C. Calcium channel blockers D. Low-dose epinephrine (<1 μg/kg)
722. (D) Current (2012) ASRA guidelines for LAST for a 70-kg patient with cardiovascular collapse include the following: 1. Get help 2. Initial focus—airway management, seizure suppression, and consideration for cardiopulmonary bypass 3. Management of cardiac arrhythmias—basic and advanced cardiac life support, the AVOIDANCE of vasopressin, calcium channel blockers, β-blocker, or local anesthetics. Epinephrine doses should be reduced to <1 μg/kg. 4. Lipid emulsion therapy should be started. For a 70-kg patient, the initial bolus Intralipid (20%) dose is 1.5 mL/kg (lean body mass) or about 100 mL over 1 minute, followed by a continuous infusion of 0.25 mL/kg/min (about 18 mL/min). Repeat the bolus one or two times for persistent cardiovascular collapse and double in continuous infusion rate if the blood pressure remains low. Continue the infusion for at least 10 minutes after cardiovascular stability is attained. The upper limit of Intralipid (20%) is 10 mL/kg over 30 minutes. 5. Post LAST events at www.lipidrescue.org and report use of lipid to www.lipidregistry.org (ASRA.com – Downloadable checklist for Treatment of Local Anesthetic Systemic Toxicity 9/19/11).
723. Transient neurologic syndrome (TNS) is MOST commonly seen after the spinal anesthetic injection of which local anesthetic? A. Lidocaine B. Bupivacaine C. Prilocaine D. Tetracaine
723. (A) Transient neurologic syndrome (TNS), formerly called transient radicular irritation (TRI), occurs most commonly after spinal anesthesia with lidocaine (Xylocaine). Symptoms include back pain that develops after the block resolves and radiates to the buttocks and legs. The pain is not associated with motor or sensory loss or electromyographic changes. It can be severe, requiring hospital admission of outpatients, and typically resolves within 1 to 4 days. It appears to occur more commonly when outpatients are operated on in the lithotomy position and appears to be less likely when patients are pregnant (Chestnut: Chestnut’s Obstetric Anesthesia, ed 5, p 756; Suresh: Shnider and Levinson’s Anesthesia for Obstetrics, ed 5, pp 112– 113).
724. You have a well-working T10 labor epidural in a woman with a questionable difficult airway and have just been informed that an urgent cesarean section is needed for a nonreassuring FHR tracing. Which of the following local anesthetics would give you the SLOWEST onset of surgical anesthesia? A. 3% chloroprocaine with freshly added epinephrine (1:200,000) B. 2% lidocaine with freshly added epinephrine (1:200,000) C. 2% lidocaine and epinephrine with added bicarbonate D. 0.5% levobupivacaine with fentanyl
724. (D) When dealing with an urgent cesarean section in a patient with a well-functioning labor epidural, raising the level of anesthesia is often chosen. Of the commonly used local anesthetics, 2-chloroprocaine and lidocaine have much faster onsets of action than bupivacaine or levobupivacaine, which have relatively slow onsets of action. Alkalinization of the local anesthetic with bicarbonate shifts more of the local anesthetic molecules to the nonionized and more lipid-soluble form for a faster onset (and a more solid block); however, it does take a little time to mix the solution. Typically, 1 mL of 8.4% sodium bicarbonate (1 mEq/mL) is added to each 10 mL of 2-chloroprocaine or lidocaine. If one adds sodium bicarbonate to levobupivacaine (or ropivacaine or bupivacaine), only 0.1 mL (0.1 mEq) is added to each 10 mL, or else the bupivacaine will precipitate out. Fentanyl is often added to the local anesthetic solution for a more solid block and for some postoperative analgesia. Onset times reported differ among studies, but the following times are reported: onset for 3% 2-chloroprocaine as well as 2% lidocaine with Obstetric Physiology and Anesthesia 207 freshly added epinephrine is 8 minutes, using the premixed 2% lidocaine with epinephrine and adding bicarbonate will give an onset time of 5 minutes, 3% 2-chloroprocaine with bicarbonate has an onset of 3 minutes, and levobupivacaine with fentanyl has an onset of 10 to 11 minutes. Another option when dealing with a patient with a difficult airway is to use a higher concentration of local anesthetic from the beginning of the labor (e.g., 0.5% bupivacaine) and a higher level (to T6) so that if an emergency cesarean section is needed, the patient has a level that is almost high enough and dense enough for the operation. If this is done, the obstetrician, nurses, and patient need to know that the block will be quite dense and an operative forceps delivery may be needed (Chestnut: Chestnut’s Obstetric Anesthesia, ed 5, pp 568–569; Suresh: Shnider and Levinson’s Anesthesia for Obstetrics, ed 5, pp 172–173).
725. Which local anesthetic has the MOST rapid metabolism in maternal and fetal blood? A. Lidocaine B. Bupivacaine C. Ropivacaine D. Chloroprocaine 188 Obstetric Physiology and Anesthesia Answers, References, and Explanations
725. (D) Chloroprocaine (Nesacaine) is an ester-type local anesthetic that is rapidly metabolized by plasma pseudocholinesterase. The in vitro half-life is 11 to 21 seconds for maternal blood and 43 seconds for fetal blood. After an epidural injection, the maternal in vivo half-life is less than 7 minutes; the longer duration in vivo is due to the continual absorption of chloroprocaine from the epidural space. All the other local anesthetics are amides and require liver metabolism (Chestnut: Chestnut’s Obstetric Anesthesia, ed 5, pp 263-265; Suresh: Shnider and Levinson’s Anesthesia for Obstetrics, ed 5, pp 107–108). 208
726. Following clipping of an anterior communicating artery aneurysm, a 59-year-old man is admitted to the intensive care unit (ICU). Serum sodium is 115 mEq/L, 24-hour urine sodium collection is 350 mmol (normal range 40-117 mmol/24 hr), and central venous pressure (CVP) is 1 cm H2O. The MOST likely cause of these findings is A. Tubular necrosis B. Diabetes insipidus C. Cerebral salt-wasting syndrome D. Syndrome of inappropriate antidiuretic hormone (SIADH)
726. (C) The triad associated with cerebral salt-wasting syndrome consists of hyponatremia, volume contraction, and urine sodium concentrations inappropriately high for the given level of serum sodium. It is mainly seen in patients with subarachnoid hemorrhage (SAH). A possible etiology may be release of brain natriuretic peptide, leading to excess urinary sodium excretion. It is treated with volume replacement, using normal to hypertonic intravenous sodium chloride solution, but avoiding overly rapid serum sodium correction, as this may result in central pontine myelinolysis. Cerebral salt-wasting syndrome (usually hypovolemic) is difficult to differentiate from SIADH (usually normovolemic or mildly hypervolemic) because patients with SAH can have high antidiuretic hormone (ADH) levels secondary to trauma, pain, etc. A definitive diagnosis requires demonstration of a negative sodium balance over several days in the setting of ongoing hypovolemia or obtaining a 24-hour urine sodium sample. Clinically, the former is often not feasible because of competing interests for prophylaxis or treatment of cerebral vasospasm with moderate hypervolemia. In the setting of cerebral salt-wasting syndrome, the 24-hour sodium value is elevated. In contrast, hyponatremia associated with SIADH is due to renal retention of free water (rather than renal loss of sodium). Accordingly, the quantity of sodium collected over the 24-hour period and CVP should be relatively normal in SIADH patients. The patient presented in this question is hyponatremic and has a low CVP and a 24-hour urine sodium that is clearly elevated. Collectively, this supports the diagnosis of cerebral salt-wasting syndrome. Diabetes insipidus and primary hyperaldosteronism are incorrect responses, as both would be associated with increased plasma sodium concentrations. Tubular necrosis has nothing to do with this pathophysiologic process (Miller: Miller’s Anesthesia, ed 8, p 2177).
727. Intracranial hypertension is defined as a sustained increase in intracranial pressure (ICP) above A. 5 mm Hg B. 15 mm Hg C. 25 mm Hg D. 40 mm Hg
727. (B) Elevated ICP frequently is the final stage of a pathologic cerebral insult (e.g., head injury, intracranial tumor, subarachnoid hemorrhage, metabolic encephalopathy, or hydrocephalus). The intracranial contents consist of three compartments: brain parenchyma (80%-85%), blood (5%-10%), and CSF (5%- 10%). None of these components is compressible; accordingly, an increase in the volume of any of these requires a compensatory decrease in the volume of one or both of the other components to avoid the development of intracranial hypertension. Normal ICP is less than 15 mm Hg. As measured in the supine position, intracranial hypertension is defined as a sustained increase in ICP above 15 to 20 mm Hg (Miller: Basics of Anesthesia, ed 6, p 478).
728. Calculate cerebral perfusion pressure from the following data: blood pressure (BP) 100/70, heart rate (HR) 65 beats/min, and ICP 15 mm Hg. A. 60 mm Hg B. 65 mm Hg C. 70 mm Hg D. 75 mm Hg
728. (B) Cerebral perfusion pressure is equal to mean arterial pressure (MAP) minus ICP. In the present case, MAP equals 80 mm Hg (diastolic pressure, 70, plus one third the pulse pressure, 10). Thus, 80 − 15 = 65 (Miller: Basics of Anesthesia, ed 6, p 478).
729. The afferent input for somatosensory evoked potentials (SSEPs) is carried through which spinal cord tract? A. Spinocerebellar B. Spinothalamic C. Dorsal columns D. Corticospinal
729. (C) Somatosensory evoked potentials (SSEPs) are voltage signals that appear in response to electrical stimulation of peripheral nerves. The impulse elicited by electrical stimulation of a peripheral nerve ascends the ipsilateral dorsal column of the spinal cord, decussates in the medulla oblongata, and is ultimately recorded on the contralateral somatosensory cortex of the brain. The signals are composed of negative and positive voltage deflections with specific latencies and amplitudes. In general, the earlier deflections represent impulses and synapses within the spinal cord or brain stem, whereas the later impulses represent thalamic and/or cortical synapses. Intraoperative monitoring of SSEPs provides the ability to assess the integrity of the sensory structures along this ascending neural pathway (e.g., a peripheral nerve such as the posterior tibial nerve, dorsal columns, brain stem, medial lemniscus, internal capsule, and somatosensory cortex) (Miller: Basics of Anesthesia, ed 6, p 328; Miller: Miller’s Anesthesia, ed 8, p 1488).
730. By what percentage does cerebral blood flow (CBF) change for each mm Hg increase in Paco2? A. 1% B. 2% C. 7% D. 10%
730. (B) Hyperventilation of the lungs causes constriction of cerebral blood vessels, which reduces global cerebral blood flow (CBF) and cerebral blood volume (CBV). This effect is mediated by changes in the pH induced in the extracellular fluid. In contrast to autoregulation, CO2 reactivity is preserved in most patients with severe brain injury; thus, hyperventilation can rapidly lower ICP through the reduction in CBV. Although the effects of hyperventilation on CBV and ICP are almost immediate, the duration of effect wanes after 6 to 10 hours of hyperventilation and may last up to 24 to 36 hours, because the pH of the extracellular fluid equilibrates to the lower Paco2 level. Generally speaking, CBF increases Neurologic Physiology and Anesthesia 215 (or decreases) by approximately 2% for each mm Hg increase (or decrease) in Paco2. CBF increases (or decreases) 1 mL/100 g/min per 1 mm Hg increase (or decrease) in Paco2. Because normal global CBF is 50 mL/100 g/min, a 1-mL/100 g/min alteration in CBF represents a 2% change (Hines: Stoelting’s Anesthesia and Co-Existing Disease, ed 5, p 200; Miller: Miller’s Anesthesia, ed 8, pp 388–390, Box 17-1).
731. Which of the following intravenous anesthetics is contraindicated in patients with intracranial hypertension? A. Propofol B. Fentanyl C. Ketamine D. All are acceptable
731. (C) Of the choices listed in this question, ketamine is the only intravenous anesthetic that is not recommended for patients with intracranial hypertension because it increases cerebral metabolic rate (CMR), CBF, CBV, and ICP. Barbiturates, etomidate, and propofol decrease CMR, CBF, CBV, and ICP. All three of these agents indirectly decrease CBF by their inhibitory effect on CMR. However, unlike thiopental, etomidate also has a direct vasoconstrictor effect on the cerebral vasculature. One potential advantage of etomidate over thiopental is that it does not produce significant cardiovascular depression. Although not as pronounced as the barbiturates, benzodiazepines such as midazolam also reduce CMR and CBF. Flumazenil, a benzodiazepine antagonist, has been reported to reverse the effect of midazolam on CMR, CBF, CBV, and ICP. Consequently, flumazenil should be avoided in midazolam-anesthetized patients known to have intracranial hypertension. Generally speaking, the opioid anesthetics, such as morphine and fentanyl, cause either a minor reduction or have no effect on CBF and CMR. Although many of these IV drugs reduce ICP, none has been shown to provide neuroprotection to humans with either focal (like stroke) or global (like cardiac arrest) ischemia (Miller: Basics of Anesthesia, ed 6, p 478).
732. The term luxury perfusion refers to a situation that occurs in the brain when A. Blood flow has resumed after a period of ischemia B. Blood flow is directed from a normal region of the brain to an ischemic region C. Vasoparalysis exists D. The Robin Hood phenomenon exists
732. (C) During acute focal cerebral ischemia, regional vasoparalysis results in impaired coupling between CBF and CMR. Consequently, CBF exceeds CMR and is passively associated with systemic arterial blood pressure. Under these circumstances, autoregulation and the reactivity of the cerebrovasculature to carbon dioxide is also impaired. Thus, tight control of systemic arterial blood pressure is important in managing patients with focal ischemia, because cerebral perfusion is highly dependent on mean arterial blood pressure. Blood flow directed from a normal region of the brain to an ischemic region is known as the “Robin Hood phenomenon” (i.e., robbing from the rich and giving to the poor). Thus, choices B and D are synonymous incorrect responses (Miller: Miller’s Anesthesia, ed 8, p 401).
733. A 62-year-old patient is scheduled to undergo resection of a frontal lobe intracranial tumor under general anesthesia. Preoperatively, the patient is alert and oriented, and has no focal neurologic deficits. Within what range should Paco2 be maintained during surgery? A. 15 and 20 mm Hg B. 20 and 25 mm Hg C. 25 and 30 mm Hg D. 40 and 45 mm Hg
733. (C) Cerebral ischemia has been reported in both humans and laboratory animals when the Paco2 is reduced below 20 mm Hg. It is likely that cerebral ischemia is caused by a leftward shift of the oxyhemoglobin dissociation curve (produced by the severe alkalosis) and possibly by intense cerebral vasoconstriction. A leftward shift of the oxyhemoglobin dissociation curve increases the affinity of hemoglobin for O2, which reduces off-loading of O2 from hemoglobin at the capillary bed. This effect combined with decreased CBF can result in cerebral ischemia. Combined with the fact that there is very little additional benefit in terms of reducing CBV and ICP, it is recommended to limit acute hyperventilation of the lungs to a Paco2 of 25 to 30 mm Hg. Within this range, reduction in ICP is maximal, and risk of cerebral ischemia is minimal. As an aside, hyperventilation-induced respiratory alkalosis can precipitate hypokalemia. Specifically, serum potassium decreases 0.6 mEq/L for each 0.1-unit increase in pH. Thus, overly aggressive hyperventilation should be guarded against to avoid possible hypokalemia-induced cardiac arrhythmias (Miller: Miller’s Anesthesia, ed 8, pp 2163–2164).
734. A 2-year-old child is anesthetized for resection of a posterior fossa tumor. Preoperatively, the patient is lethargic and disoriented. Which of the following is MOST likely to adversely alter ICP? A. 5% Dextrose in water B. Normal saline C. Lactated Ringer solution D. 5% Albumin
734. (A) Five percent dextrose in water (D5W) is contraindicated in neurosurgical patients with intracranial hypertension for two reasons. First, D5W easily passes through the blood-brain barrier. Once in the brain tissue, glucose is rapidly metabolized, leaving only free water, which causes cerebral edema. Second, hyperglycemia is associated with increased severity of neurologic damage in patients with cerebral ischemia. The etiology of hyperglycemia-induced worsening of neurologic injury is associated with simple biochemical processes. Aerobic: glucose + oxygen → 6CO2 + 6H2O + 36 ATP (energy efficient) Anaerobic: glucose → 2 lactate + 2H + + 2 ATP (energy inefficient) Both lactate and H+ are harmful to compromised neurons and glia. Furthermore, in the setting of hyperglycemia the anaerobic reaction is forced to the right, resulting in additional accumulation of these toxic metabolites, thereby worsening neurologic outcome (Miller: Miller’s Anesthesia, ed 8, pp 2172–2173).
735. A 22-year-old patient is anesthetized for resection of a temporal lobe tumor. Preoperatively, he is lethargic and confused. After induction of general anesthesia, which of the following would be the MOST appropriate drug to control systemic arterial blood pressure during direct laryngoscopy and tracheal intubation? A. Esmolol B. Nitroprusside C. Hydralazine D. Isoflurane
735. (A) Except esmolol, all of the drugs listed are potent cerebral vasodilators capable of further increasing ICP, which would be highly undesirable in this patient. By contrast, esmolol is a cardioselective β1-adrenergic receptor antagonist with rapid onset and short duration of action due to hydrolysis by red blood cell 216 Part 2 Clinical Sciences esterases. Plasma cholinesterases and red cell membrane acetylcholinesterase do not play a role in its degradation. Esmolol effectively blunts the sympathetic response to direct laryngoscopy and tracheal intubation, yet is devoid of deleterious effects on CBV or ICP (Miller: Miller’s Anesthesia, ed 8, pp 394- 395; Cottrell: Cottrell and Young’s Neuroanesthesia, ed 5, pp 82–83).
736. Normal global CBF is A. 25 mL/100 g/min B. 50 mL/100 g/min C. 75 mL/100 g/min D. 100 mL/100 g/min
736. (B) Normal global CBF is approximately 45 to 55 mL/100 g/min. Cortical CBF (gray matter) is approximately 75 to 80 mL/100 g/min, and subcortical CBF (mostly white matter) is approximately 20 mL/100 g/min. Factors that regulate CBF include Paco2, Pao2, CMR, cerebral perfusion pressure, autoregulation, and the autonomic nervous system (Miller: Miller’s Anesthesia, ed 8, p 388, Box 17-1).
737. The lower and upper mean arterial blood pressure limits of CBF autoregulation are, respectively, A. 25 and 125 mm Hg B. 25 and 200 mm Hg C. 40 and 250 mm Hg D. 60 and 160 mm Hg
737. (D) Cerebral blood flow (CBF) autoregulation is the intrinsic capability of the cerebral vasculature to adjust its resistance to maintain CBF constant over a wide range of mean arterial blood pressures. In normotensive adults, lower and upper limits of autoregulation are cerebral perfusion pressures of 50 to 60 mm Hg and 150 to 160 mm Hg, respectively. Above or below the limits of CBF autoregulation, CBF is pressure dependent. Although the precise mechanism of CBF autoregulation is not known, it is thought to result from an intrinsic characteristic of cerebral vascular smooth muscle that has not yet been identified (Morgan: Clinical Anesthesiology, ed 4, p 616; Miller: Miller’s Anesthesia, ed 8, p 391).
738. How much will CBF increase in a patient whose Paco2 is increased from 35 to 45 mm Hg? A. There is no relationship between Paco2 and CBF B. 10 mL/100 g/min C. 20 mL/100 g/min D. 40 mL/100 g/min
738. (B) Cerebral blood flow (CBF) will increase by approximately 1 mL/100 g/min for every 1-mm Hg increase in Paco2 (i.e., approximately 2%). This effect is caused by a CO2-mediated decrease in the pH of the extracellular fluid surrounding the cerebral vessels, which causes cerebral vasodilatation. The pH changes rapidly because CO2 diffuses freely across the cerebral vascular endothelium into the extracellular fluid. However, the change in pH wanes after 6 to 10 hours because extracellular fluid pH is gradually normalized by reabsorption of HCO3 – and excretion of H by the kidneys. An increase in Paco2 of 10 mm Hg (from 35 to 45 mm Hg) will result in an increase in CBF of approximately 10 mL/100 g/ min (Miller: Miller’s Anesthesia, ed 8, p 390).
739. Select the FALSE statement concerning autonomic hyperreflexia. A. Distention of a hollow viscus below the level of the spinal cord transection can elicit autonomic hyperreflexia B. Up to 85% of patients with a spinal cord transection above the T6 dermatome will exhibit autonomic hyperreflexia under general anesthesia C. Propranolol is effective in treating hypertension associated with autonomic hyperreflexia D. Spinal anesthesia is effective in preventing autonomic hyperreflexia
739. (C) Autonomic hyperreflexia is a neurologic disorder that occurs in association with resolution of spinal shock and a return of spinal cord reflexes. Cutaneous or visceral stimulation (such as distention of the urinary bladder or rectum) below the level of the spinal cord transection initiates afferent impulses that are transmitted to the spinal cord at this level, which subsequently elicits reflex sympathetic activity over the splanchnic nerves. Because modulation of this reflex sympathetic activity from higher centers in the central nervous system is lost (as a result of the spinal cord transection), the reflex sympathetic activity below the level of the injury results in intense generalized vasoconstriction and hypertension. Bradycardia occurs secondary to activation of baroreceptor reflexes arising from the carotid or aortic sinus. The incidence of autonomic hyperreflexia during general anesthesia depends on the level of the spinal cord transection. Approximately 85% of patients with a spinal cord transection above the T6 dermatome will exhibit this reflex during general anesthesia. In contrast, it is difficult to elicit this reflex in patients with a spinal cord transection below the T10 dermatome. Treatment of autonomic hyperreflexia is with α-adrenergic receptor antagonists (e.g., phentolamine), direct-acting vasodilators (e.g., nitroprusside, fenoldopam, or nitroglycerin), and deep general or regional anesthesia. Patients with autonomic hyperreflexia should not be treated initially with propranolol or other β-adrenergic receptor antagonists for three reasons. First, bradycardia can be potentiated by β1-adrenergic receptor blockade; second, β2-adrenergic receptor blockade in skeletal muscle will leave the α-adrenergic properties of circulating catecholamines unopposed, thereby causing a paradoxical hypertensive response; and third, a combination of unopposed α-mediated vasoconstriction coupled with β1-adrenergic negative inotropy could result in congestive heart failure (Miller: Miller’s Anesthesia, ed 8, pp 382–383).
740. What is the normal cerebral metabolic rate for oxygen (CMRO2) per minute? A. 0.5 mL/100 g brain tissue B. 2.0 mL/100 g brain tissue C. 3.5 mL/100 g brain tissue D. 7.5 mL/100 g brain tissue
740. (C) The brain is an obligate aerobe, as it cannot store oxygen. Under normal circumstances, there is a substantial safety margin in that the delivery of oxygen is considerably greater than demand. Oxygen consumption is in the range of 3 to 5 mL/100 g of brain tissue/min, whereas the delivery of oxygen is approximately 50 mL blood/100 g brain tissue/min. Whole-brain oxygen consumption represents about 20% of total-body oxygen utilization (Miller: Miller’s Anesthesia, ed 8, p 388).
741. A 14-year-old girl with severe scoliosis is to undergo spine surgery. Anesthesia is maintained with fentanyl, N2O 50% in O2, vecuronium, and isoflurane. Neurologic function of the spinal cord is monitored by SSEPs. In reference to the SSEP waveform, spinal cord ischemia would be manifested as A. Increased amplitude and increased latency B. Decreased amplitude and increased latency C. Decreased amplitude and decreased latency D. Increased amplitude and decreased latency
741. (B) Somatosensory evoked potentials (SSEPs) are composed of negative and positive voltage deflections with specific latencies and amplitudes. Baseline values for latency and amplitude must be established for each patient prior to surgery because the characteristics of SSEP waveforms change with recording circumstances Neurologic Physiology and Anesthesia 217 (e.g., the latency becomes greater and the amplitude becomes smaller as the distance between the neural generator and the recording electrode is increased). Ischemia, neurologic injury, or transection of a neural pathway (i.e., surgical causes) will result in a decrease in signal amplitude and/or increase in signal latency. Alternatively, such changes may result from medications administered by the anesthesia care team (e.g., isoflurane, sevoflurane, desflurane, propofol, barbiturates, benzodiazepines). Taken together, should signal decay occur during surgery, it is imperative for the anesthesiologist to have a very systematic approach to elucidate the etiology of signal change. More specifically, nonsurgical causes should be promptly ruled out (e.g., was a medication recently administered or dose changed, hypothermia, hypotension, anemia, hypoxemia). Also see explanation to Question 746 (Miller: Miller’s Anesthesia, ed 8, pp 1520–1521).
742. For each 1° C decrease in body temperature, how much will CMRO2 be diminished? A. 3% B. 5% C. 6% D. 10%
742. (C) The cerebral metabolic rate for oxygen (CMRO2) decreases approximately 6% per 1° C of temperature reduction. Hypothermia has been reported to improve neurologic outcome after focal or global brain ischemia. Historically, the extent of brain protection was thought to be proportional to the magnitude of hypothermiamediated reduction in CMRO2. However, more recent studies have demonstrated that temperature reductions of a mere 1° C to 2° C significantly improve postischemic neurologic outcome. Proposed mechanisms for temperature modulation of postischemic neurologic outcome include alterations in CMRO2, bloodbrain barrier stability, membrane depolarization, ion homeostasis (e.g., calcium fluxes), neurotransmitter release (e.g., glutamate or aspartate), enzyme function (e.g., phospholipase, xanthine oxidase, or nitric oxide synthase), and free radical production or scavenging (Miller: Miller’s Anesthesia, ed 8, p 390).
743. A 24-year-old carpenter is treated for a closed head injury sustained 3 days earlier after falling from a roof. He has been hemodynamically stable. Despite aggressive efforts to pharmacologically reduce ICP, he is now unconscious and unresponsive to painful stimuli. All of the following are clinical criteria consistent with a diagnosis of brain death in this patient EXCEPT A. Persistent apnea for 10 minutes B. Absence of pupillary light reflex C. Persistent spinal reflexes D. Decorticate posturing
743. (D) Brain death is defined as irreversible cessation of brain function. It is extremely important to identify and reverse any factors that can mimic the clinical or laboratory criteria for brain death, such as hypothermia, drug intoxication (hypnotic sedatives and major tranquilizers), or metabolic encephalopathy. Clinical criteria for brain death can be divided into those that are related to cortical function and those that are related to brain stem function. Absence of cortical function is manifested by lack of spontaneous motor activity, consciousness, and purposeful movement in response to painful stimuli. Absence of brain stem function is manifested by the inability to elicit reflexes, such as the pupillary response to light and the corneal, oculocephalic, oculovestibular, oropharyngeal, and respiratory reflexes. For example, in patients without brain stem function, there is no increase in heart rate when atropine is administered intravenously (due to absence of native vagal tone from the brain stem), and there is no respiratory effort during apnea even when the Paco2 is greater than 60 mm Hg. Decerebrate and decorticate posturing are not consistent with the diagnosis of brain death (Miller: Miller’s Anesthesia, ed 8, pp 2317–2326).
744. Which of the following is the MOST sensitive means of detecting venous air embolism (VAE)? A. Electroencephalography (EEG) B. Pulmonary artery catheter C. Transesophageal echocardiography D. Right atrial catheterization
744. (C) The most common complications associated with the surgical sitting position include venous air embolism (VAE), paradoxical VAE, cardiovascular instability, pneumocephalus, subdural hematoma, peripheral neuropathy, and quadriplegia (quadriplegia is possibly caused by compression ischemia of the cervical spinal cord in patients with aberrant spinal cord blood supply). VAE occurs when air is entrained into open veins in the presence of negative intraluminal pressures (i.e., negative with respect to atmospheric pressure). Significant VAE can result in reduced cardiac output and profound hypoxia. Current devices used to detect VAE include the transesophageal echocardiograph, Doppler ultrasound, pulmonary artery catheter, infrared spectrometer (to monitor changes in Peco2 and Pen2), right atrial catheter, and esophageal stethoscope (to listen for a “mill wheel” cardiac murmur). The most sensitive means of diagnosing VAE include transesophageal echocardiography or precordial Doppler monitoring (see explanation to Question 748) (Miller: Miller’s Anesthesia, ed 8, p 2170; Faust: Anesthesiology Review, ed 3, pp 389–391, Figure 158-1). VAE volume No physiologic changes T-echo Doppler PAP ET-CO2 CO CVP BP ECG STETHO Modest physiologic changes Clinically apparent changes Cardiovascular collapse Decreasing sensitivity 218 Part 2 Clinical Sciences
745. When intracranial hypertension exists, the main compensatory mechanism from the body is A. Increased absorption of cerebrospinal fluid (CSF) at the intracranial arachnoid villi B. Increased absorption of CSF in the spinal arachnoid villi C. Shifting of CSF from intracranial to spinal subarachnoid space D. Reduction of cerebral blood volume due to compression of intracranial arteries
745. (C) Intracranial pressure is determined by the pressure contribution of three volume compartments: brain parenchyma 80% to 90%, CSF 5% to 10%, and blood 5% to 10%. Under normal circumstances, ICP is maintained within the normal range (i.e., ≤15 mm Hg) over a wide range of intracranial volumes (ICVs) due to the following three compensatory mechanisms: (1) translocation of CSF from the intracranial to spinal subarachnoid space; (2) translocation of intracranial blood (primarily venous) to systemic circulation; and (3) reabsorption of CSF across arachnoid villi into the dural venous sinus and, ultimately, into systemic circulation. Once these compensatory mechanisms are exhausted, small increases in ICV result in large increases in ICP (i.e., a situation of increased intracranial elastance), which leaves the brain vulnerable to ischemia and herniation. CSF production is fairly constant (0.35 to 0.40 mL/min) regardless of ICP (Miller: Miller’s Anesthesia, ed 8, p 2159, Figure 63-3; Faust: Anesthesiology Review, ed 3, p 376).
746. Administration of vecuronium during spinal surgery may interfere with monitoring of A. Dorsal columns B. Corticospinal tract C. Electrocorticography D. Bispectral index
746. (B) Postoperative neurologic dysfunction is a rare but serious complication of spinal reconstructive surgery. In cases where spinal cord dysfunction is more likely to occur (e.g., surgical correction of severe scoliosis), spinal cord monitoring is used to identify ischemia and ideally to allow the surgeon time to modify the procedure to reverse any spinal cord dysfunction seen during general anesthesia. Somatosensory evoked potentials (SSEPs) involve repetitive stimulation of the extremity and monitoring the signals at the level of the scalp. SSEPs are used to monitor the dorsal columns of the spinal cord. As this area is sensory, neuromuscular blockers such as vecuronium do not affect SSEP monitoring. Motor evoked potentials (MEPs) monitoring is used to monitor corticospinal tracts (motor pathways) that are not assessed with SSEPs. Neuromuscular blockers interfere with MEP monitoring and should not be used. Electrocorticography (ECoG) monitoring is used to identify epileptogenic foci during seizure surgery or to assess cerebral cortical integrity during carotid endarterectomy. The ECoG is altered by drugs that affect the seizure threshold (e.g., benzodiazepines as well as volatile anesthetics). The bispectral index monitor (BIS) uses processed electroencephalographic (EEG) signals to measure level of consciousness in an attempt to decrease intraoperative awareness (Miller: Basics of Anesthesia, ed 6, p 480).
747. Patients can be safely imaged in the magnetic resonance imaging (MRI) scanner with conventional versions of which of the following monitors? A. Pulmonary artery catheter with cardiac output probe B. Foley catheter with temperature probe C. Electrocardiography (ECG) electrodes D. Arterial line
747. (D) The MRI scanner is potentially dangerous for several reasons. The most obvious is the risk of projectiles traveling toward the patient. Objects made of iron, nickel, and cobalt are strongly pulled by the constant magnetic force (up to 3 T). A more insidious but equally dangerous hazard is represented by indwelling devices, such as pacemakers, pumps, aneurysm clips, and orthopedic prostheses. The interactions between these and the magnetic field can be harmful or even lethal for the patient under some circumstances. Finally, the antenna effect of the MRI scanner can induce heat in wires that are in close proximity to the patient. For this reason, pulmonary artery (PA) catheters and urinary catheters with temperature wires embedded in them cannot be used in patients undergoing MRI scanning. Standard pulse oximeters and ECG wires are also unacceptable, but special MRI-compatible probes with fiberoptic “cables” can be safely used as can “wireless” ECG patches. Arterial lines do not pose a problem in the scanner because no wires come into contact with the patient. The fluid-filled tubing is not ferromagnetic and the transducer is fixed, away from the patient (Faust: Anesthesiology Review, ed 3, pp 533–534; Miller: Miller’s Anesthesia, ed 8, p 2660). Neurologic Physiology and Anesthesia 219
748. What is the minimum quantity of intracardiac air that can be detected by a precordial Doppler? A. 0.25 mL B. 5.0 mL C. 10 mL D. 25 mL 210 Part 2 Clinical Sciences
748. (A) Except for the transesophageal echocardiograph (TEE), the Doppler ultrasound is the most sensitive device for detection of intracardiac air. Under ideal circumstances, as little as 0.25 mL of intracardiac air can be detected by this device. In contrast, TEE can detect even smaller volumes of intracardiac air (Cottrell: Cottrell and Young’s Neuroanesthesia, ed 5, p 210; Miller: Basics of Anesthesia, ed 6, p 482).
749. With regard to regulation of blood flow, the correct order of vascular responsiveness to Paco2 from most to least sensitive is A. Cerebrum > spinal cord > cerebellum B. Cerebrum > cerebellum > spinal cord C. Cerebellum > cerebrum > spinal cord D. Cerebellum > spinal cord > cerebrum
749. (B) Arterial carbon dioxide tension (Paco2) is one of the most important extracerebral biochemical factors regulating CBF. The cerebral vasculature is most sensitive to changes in Paco2 within the physiologic range (i.e., approximately 20-80 mm Hg). In general, the regional sensitivity of the cerebral vasculature to changes in Paco2 (i.e., CO2 responsiveness) is directly proportional to the resting CMR for each region of the brain. Thus, regional CO2 responsiveness is greatest in the cerebrum, less in the cerebellum, and least in the spinal cord (Cottrell: Cottrell and Young’s Neuroanesthesia, ed 5, pp 25–26).
750. Select the TRUE statement concerning administration of glucose-containing solutions to the patient with a closed head injury versus a patient with a spinal cord injury. A. Glucose-containing solutions are contraindicated in both patient groups B. Glucose-containing solutions are contraindicated in patients with closed head injury but acceptable in patients with spinal cord injuries C. Glucose-containing solutions are acceptable in patients with closed head injuries but contraindicated in patients with spinal cord injuries D. Glucose-containing solutions may be given to either patient group if blood glucose concentrations do not exceed 200 mg/dL
750. (A) Both laboratory and clinical studies have reported that hyperglycemia at the time of either focal (e.g., stroke) or global (e.g., systemic shock or cardiac arrest) ischemia results in a worsening of neurologic outcome (i.e., both histologic and functional). Unfortunately, it is not widely appreciated that the administration of glucose does not need to produce high blood glucose levels to augment postischemic cerebral injury. Thus, glucose-containing solutions should not be administered to patients who are at risk for either cerebral or spinal cord injury (Gupta: Essentials of Neuroanesthesia and Neurointensive Care, ed 1, pp 238-240; Cottrell: Cottrell and Young’s Neuroanesthesia, ed 5, p 409).
751. A 67-year-old patient is scheduled to undergo posterior cervical fusion in the sitting position under general anesthesia. A central venous catheter is inserted from the right basilic vein and advanced toward the heart. Intravascular electrocardiography (ECG; with the exploring electrode attached to the V lead) is used to aid in placement of the catheter. After the catheter is advanced 45 cm, the tracing shown in the figure is noted on the electrocardiogram. At this time the anesthesiologist should A. Advance the catheter 5 cm B. Advance the catheter slightly C. Leave the catheter in the present position D. Withdraw the catheter 1 cm
751. (D) The tip of multiorificed right atrial catheters must be accurately placed at the junction of the superior vena cava and right atrium, because air has a tendency to localize at this junction. Several methods can be used to ensure that the catheter tip is accurately positioned at this junction. For example, a chest x-ray film can be obtained. However, there may be difficulty in interpreting the position of the tip of the catheter, and the catheter could migrate after the x-ray film is obtained. Cardiovascular pressures could be monitored, but this technique requires that the tip of the catheter first be introduced into the right ventricle and then pulled back into the right atrium. Introduction of the tip of the catheter into the right ventricle could cause dysrhythmias, heart block, or bleeding, or rupture of cardiac structures. A technique frequently used to accurately place multiorificed catheters at the junction of the superior vena cava and right atrium is intravascular ECG. The appropriate position of the catheter is confirmed when a large negative P complex is obtained on the electrocardiogram. The P complex shown in the figure of this question is biphasic, which indicates that the tip of the catheter is in the midatrial position and should be withdrawn slightly until there is a large negative downward configuration of the P complex. Finally, transesophageal echocardiography can be used to confirm catheter tip placement (Atlee: Complications in Anesthesia, ed 2, p 582, Table 144-1).
752. Critical CBF in patients anesthetized with isoflurane is A. 5 mL/100 g/min B. 10 mL/100 g/min C. 18 mL/100 g/min D. 25 mL/100 g/min
752. (B) Critical CBF is the CBF below which EEG evidence of cerebral ischemia begins to appear. Critical CBF in patients anesthetized with isoflurane, desflurane, or sevoflurane is approximately 10 mL/100 g/min. In contrast, critical CBF in patients anesthetized with halothane is 18 to 20 mL/100 g/min, and critical CBF in patients anesthetized with enflurane is about 15 mL/100 g/min. Based on studies that compared the requirement for shunt placement after carotid artery cross-clamping in patients under isoflurane, enflurane, and halothane anesthesia, it appears that isoflurane might provide some degree of cerebral protection against incomplete regional cerebral ischemia in humans; however, anesthesia-modulated brain protection remains speculative, as outcome-based evidence is lacking in humans (Cottrell: Cottrell and Young’s Neuroanesthesia, ed 5, pp 30–31).
753. What effect does cerebral ischemia have on CBF autoregulation? A. CBF autoregulation is ablated B. CBF autoregulation is ablated at low cerebral perfusion pressures but remains intact at high cerebral perfusion pressures C. CBF autoregulation is ablated at high cerebral perfusion pressures but remains intact at low cerebral perfusion pressures D. The CBF autoregulatory curve is shifted to the right
753. (A) It is important to note that CBF autoregulation is easily impaired and modified by numerous factors, such as cerebral vasodilators (including volatile anesthetics), chronic hypertension, and cerebral ischemia. Cerebral ischemia abolishes CBF autoregulation such that CBF becomes passively dependent on the cerebral perfusion pressure (Cottrell: Cottrell and Young’s Neuroanesthesia, ed 5, pp. 30–31).
754. The MOST rapid maneuver available for lowering ICP in a patient with a large intracranial mass is A. Mannitol, 1 g/kg IV B. Methylprednisolone, 30 mg/kg IV C. Hyperventilation to 25 mm Hg Paco2 D. Furosemide, 1 mg/kg IV
754. (C) Changes in plasma Paco2 will affect cerebral vascular tone. Hypocarbia (associated with hyperventilation) will rapidly cause vasoconstriction, thereby reducing CBF, CBV, and ICP. Thus, hyperventilation is the technique that will be most rapidly available to decrease ICP in patients with an intracranial mass (Cottrell: Cottrell and Young’s Neuroanesthesia, ed 5, pp 187–188). 220 Part 2 Clinical Sciences
755. What effect does propofol have on the CO2 responsiveness of the cerebral vasculature? A. Propofol attenuates the effect of hypocarbia on CBF B. Propofol attenuates the effect of hypercarbia on CBF C. Propofol augments the effect of hypocarbia on CBF D. Propofol does not affect CO2 reactivity at a dose used clinically
755. (D) In general, the cerebrovascular response to changes in Paco2 is preserved after the administration of intravenous anesthetics. Specifically, in humans, CO2 reactivity is maintained with propofol or barbiturate concentrations sufficient to produce burst suppression on the EEG (Miller: Miller’s Anesthesia, ed 8, pp 396–397).
756. Cerebral autoregulation is MOST likely to remain intact A. Immediately after cerebral aneurysm rupture B. In a patient with traumatic brain injury and a Glasgow coma scale of 3 C. With total intravenous anesthesia (TIVA) anesthetic using propofol D. With 2.5% end-tidal sevoflurane anesthesia
756. (C) Cerebral autoregulation can be impaired under certain conditions including brain tumors, arteriovenous malformations, subarachnoid hemorrhage, intracranial surgery, and traumatic brain injury (TBI). Volatile anesthetics above 1 MAC impair cerebral autoregulation; however, in low doses (i.e., below 1 MAC), autoregulation is maintained. Total intravenous anesthesia (TIVA) does not impair cerebral autoregulation (Miller: Basics of Anesthesia, ed 6, p 478; Miller: Miller’s Anesthesia, ed 8, pp 2176–2187).
757. A 72-year-old patient undergoing resection of an astrocytoma in the sitting position suddenly develops hypotension. Air is heard on the precordial Doppler ultrasound. Each of the following therapeutic maneuvers to treat VAE is appropriate EXCEPT A. Discontinue N2O B. Apply jugular venous pressure C. Implement positive end-expiratory pressure (PEEP) D. Administer epinephrine to treat hypotension Neurologic Physiology and Anesthesia 211
757. (C) The general approach to treating patients following VAE is to: (1) stop further air entrainment; (2) aspirate entrained air; (3) prevent expansion of existing air; and (4) support cardiovascular function. Cessation of subsequent air entrainment is achieved by flooding the surgical field with irrigation fluid. Additionally, noncollapsible veins can be sealed using electrocautery, vessel ligation, or bone wax. Neck veins can be compressed as a means of increasing jugular venous pressure, which mitigates or prevents further air entry and helps localize the source of air. A multiorificed right atrial catheter, placed before the event, is the most effective means of aspirating VAE. To prevent expansion of the VAE, nitrous oxide is immediately discontinued. Cardiovascular function is supported using inotropes, vasopressors, and intravenous fluids as indicated. Of the response options provided, PEEP is the least correct answer. Approximately 20% to 30% of humans have a probe patent foramen ovale. Initiation of PEEP may increase the risk of paradoxical embolism or decrease venous effluent from the calvarium, resulting in increased CBV and ICP (Barash: Clinical Anesthesia, ed 7, p 1446; Miller: Miller’s Anesthesia, ed 8, p 2172).
758. Which of the following is the LEAST likely sequela of VAE during posterior fossa surgery in the upright position? A. Increase in pulmonary dead space B. Bronchoconstriction C. Stroke D. Pulmonary and systemic hypertension
758. (D) The risk of venous air entrainment exists whenever the operative field is above the level of the right atrium. Air enters the venous circulation and travels to the right atrium, where it continues into the right ventricle and passes into the lungs or passes right to left through a patent foramen ovale. Passage through the patent foramen may lead to stroke or, if air finds its way into coronary arteries, myocardial infarction and cardiac arrest. Air in the pulmonary artery can increase pulmonary vascular resistance (PVR) and cause right heart strain and dysrhythmias. Reflex bronchoconstriction may be caused by microvascular bubbles and by the release of inflammatory mediators from endothelial cells resulting in hypoxemia. VAE causes hypotension, not hypertension, and death is usually from cardiovascular collapse (Barash: Clinical Anesthesia, ed 7, p 1446; Cottrell: Cottrell and Young’s Neuroanesthesia, ed 5, pp 207–208; Faust: Anesthesiology Review, ed 3, pp 389–391, Figure 158-1).
759. A 55-year-old business executive is scheduled for colonoscopy and polypectomy under general anesthesia. A bruit is auscultated over the right carotid artery on physical examination. The patient is otherwise healthy. Which of the following would be the MOST appropriate course of action? A. Cancel surgery and obtain coronary angiogram B. Cancel surgery and obtain Doppler ultrasound carotid blood flow studies C. Cancel surgery and obtain dobutamine stress echocardiogram D. Proceed with surgery
759. (B) Surgical treatment of carotid artery stenosis greatly decreases the risk of stroke, especially in men with a stenosis diameter greater than 70%. Studies show a high rate of stroke in patients with asymptomatic carotid stenosis greater than 75%, and 80% of carotid atherothrombotic strokes occur without warning. The Asymptomatic Carotid Atherosclerosis Study, the largest completed clinical trial, demonstrated that patients with asymptomatic carotid stenosis (≥60%) who were treated with carotid endarterectomy and aspirin have a reduced 5-year risk of ipsilateral stroke compared with patients treated with aspirin alone (5.1% versus 11.0%). Doppler studies also show that 70% to 75% stenosis represents the point at which a pressure drop across the stenosis is likely to occur. Thus, if collateral circulation is not adequate, low-flow transient ischemic attacks and infarcts occur. It would be considered most appropriate to further study the patient’s carotid artery disease before proceeding with an elective case (Cottrell: Cottrell and Young’s Neuroanesthesia, ed 5, pp 279–285; Hines: Stoelting’s Anesthesia and Co-Existing Disease, ed 6, pp 165–167.)
760. How long after a stroke can anesthesia for surgery be carried out with about the same risk of a perioperative occlusive vascular accident as existed immediately before the previous stroke? A. 1 week B. 6 weeks C. 6 months D. 1 year
760. (B) In patients who have had a cerebral vascular accident as a result of occlusive vascular disease, there is a loss of normal vasomotor responses to changes in Paco2 and arterial blood pressure in the areas of ischemia (i.e., vasomotor paralysis), as well as disruption of the blood-brain barrier. Approximately 4 to 6 weeks is required for these changes to stabilize. Therefore, it is recommended that anesthesia for elective non-neurologic surgical procedures be postponed for at least 4 weeks and preferably 6 weeks after an occlusive vascular accident, to minimize the risk of a subsequent perioperative occlusive vascular accident (Miller: Miller’s Anesthesia, ed 8, p 1127). Neurologic Physiology and Anesthesia 221
761. A 13-year-old boy is anesthetized with 0.5% isoflurane, 50% N2O, and fentanyl for scoliosis repair. Somatosensory evoked potentials (SSEP) monitoring is conducted during the procedure. Which of the following structures is NOT involved in conveyance of the stimulus from the posterior tibial nerve to the cerebral cortex? A. Corticospinal tract B. Medial lemniscus C. Brain stem D. Internal capsule
761. (A) Somatosensory evoked potentials (SSEPs) recorded on the contralateral cerebral cortex are the physiologic response of the nervous system to peripheral nerve stimulation. Extraction of SSEPs from the background EEG is accomplished by computerized signal averaging for summation. SSEPs assess the integrity of the peripheral nerve (usually posterior tibial or median), dorsal column, brain stem, medial lemniscus, internal capsule, and contralateral somatosensory cortex. However, they do not evaluate the integrity of the ventral or lateral spinothalamic tracts or the corticospinal tract. The corticospinal tract is readily eliminated from the answer set because it is a motor (rather than sensory) pathway (Miller: Miller’s Anesthesia, ed 8, p 1497).
762. A 19-year-old woman is undergoing surgery for a Harrington rod placement. General anesthesia is administered with desflurane, nitrous oxide, and fentanyl. After completion of spinal instrumentation, a wake-up test is undertaken. Four thumb twitches are present when the nerve stimulator attached to the ulnar nerve is activated. The volatile anesthetic and nitrous oxide have been discontinued for 10 minutes when the patient is asked to move her hands and feet. After repeated commands, the patient still does not move her hands or feet. The most appropriate intervention at this time would be A. 3 mg neostigmine plus 0.6 mg glycopyrrolate IV B. 20 μg naloxone IV C. 0.1 mg flumazenil IV D. Reduce the distraction on the rods
762. (B) The differential diagnosis for a nonmoving patient during a wake-up test includes presence of neuromuscular blockade, inadequate volatile or nitrous oxide washout, or presence of opiates or sedative hypnotic medication. There are also a few other extremely rare central causes, such as stroke. Because gross neuromuscular blockade has worn off in this patient and the volatile anesthetic and nitrous oxide have largely been washed out, a trial of low-dose naloxone would not be unreasonable. An initial small dose (e.g., 20 μg) may be all that is needed to reverse the effects of the morphine. If this dose is not effective, it should be repeated. Reducing distraction on the Harrington rods would be considered only if the patient squeezed her hands, yet failed to move her feet (Yao: Yao and Artusio’s Anesthesiology, ed 7, pp 1261–1262).
763. A 75-year-old patient is undergoing craniotomy for resection of a large astrocytoma. During administration of isoflurane anesthesia, arterial blood gas sampling reveals a Paco2 of 30 mm Hg. At this time, this patient’s global cerebral blood flow would be approximately A. 20 mL × 100 g/brain weight/min B. 30 mL × 100 g/brain weight/min C. 40 mL × 100 g/brain weight/min D. 50 mL × 100 g/brain weight/min
763. (C) Arterial CO2 tension (Paco2) is the single most potent physiologic determinant of CBF and CBV. Between Paco2 values of 20 and 80 mm Hg, CBF decreases 1 to 1.5 mL × 100 g/brain weight/min and CBV decreases approximately 0.05 mL × 100 g/brain weight for each 1 mm Hg decrease in Paco2. Decreasing the Paco2 to 25 to 30 mm Hg should provide near-maximal reductions in CBF, CBV, and ICP, lasting up to 24 hours, without adversely affecting acid–base/electrolyte (e.g., decreases in potassium or ionized calcium) status or decreasing cerebral oxygen delivery (i.e., as a result of intense cerebral vasoconstriction and a leftward shift of the oxyhemoglobin dissociation curve). Because this patient’s Paco2 is 10 mm Hg below normal, CBF also would be reduced to approximately 35 to 40 mL × 100 g/brain weight/min (Miller: Miller’s Anesthesia, ed 8, p 391; Cottrell: Cottrell and Young’s Neuroanesthesia, ed 5, p 25).
764. A 24-year-old patient is brought to the intensive care unit after sustaining a closed head injury in a motor vehicle accident. Each of the following would be useful in managing intracranial hypertension in this patient EXCEPT A. Corticosteroids B. Propofol C. Hyperventilation to a Paco2 of 35 mm Hg D. Osmotic diuretics
764. (A) Intracranial pressure (ICP) is determined by the relationship between the intracranial vault (formed by the skull), volume of brain parenchyma, volume of CSF, and CBV. Studies evaluating the effectiveness of corticosteroids in the setting of head injury, or global or focal brain ischemia, have demonstrated either no improvement or a worsening of neurologic outcome. All intravenous anesthetics, except ketamine, cause some degree of reduction in CMR, CBF, CBV, and ICP (provided ventilation is not depressed). As an aside, regarding intravenous anesthetics, barbiturates are thought to be the “gold standard” for anesthetic-mediated brain protective therapy in animal models of focal or incomplete global brain ischemia. However, this has yet to be proved in humans. In the setting of traumatic brain injury, hyperventilation is an acceptable intervention. However, the Brain Trauma Foundation advises against aggressive hyperventilation, because the data suggest worsening of outcomes associated with Paco2 values below 25 to 30 mm Hg. Both osmotic and loop diuretics are effective in reducing ICP. Provided the patient is hemodynamically stable, elevation of the head above the level of the heart facilitates effluent of blood from the calvarium, which results in decreases in CBV and ICP (Cottrell: Cottrell and Young’s Neuroanesthesia, ed 5, pp 320–322, Box 18-4; Barash: Clinical Anesthesia, ed 7, pp 1504-1505; BTF Guidelines, J Neurotrauma 2007:S87–S90).
765. Preoperative treatment of subarachnoid hemorrhage (SAH) patients, without concomitant cerebral vasospasm, might include any of the following EXCEPT A. Induced hypertension (to 20% above baseline) B. Administration of nimodipine C. Sedation D. Administration of antiepileptic drugs
765. (A) After SAH, patients may experience rebleeding, cerebral vasospasm, intracranial hypertension, and seizures. Provided that the patient is not experiencing cerebral vasospasm, hypertension should be avoided in order to minimize aneurysmal wall tension, thereby mitigating the risk of re-rupture. In contrast, had this patient been in vasospasm, induced hypertension would have been an appropriate therapeutic intervention (also see explanation to Question 776). Hypertension is avoided, in part, by the administration of sedative and analgesic medications. Antiepileptic drugs and calcium channel blockers (e.g., nimodipine) often are administered in an attempt to prevent or mitigate seizures and adverse sequelae of cerebral vasospasm, respectively (Cottrell: Cottrell and Young’s Neuroanesthesia, ed 5, pp 222–224; Hines: Stoelting’s Anesthesia and Co-Existing Disease, ed 6, pp 235–238). 222 Part 2 Clinical Sciences
766. Which of the following pharmacologic agents would have the LEAST effect on somatosensory evoked potentials? A. Isoflurane B. Nitrous oxide C. Vecuronium D. Etomidate
766. (C) Somatosensory evoked potentials (SSEPs) are used to monitor the integrity of sensory pathways in the nervous system during neurosurgical or orthopedic surgery (also see explanation to Question 761). Volatile anesthetics (e.g., isoflurane), barbiturates (e.g., sodium thiopental), and propofol decrease the amplitude and increase the latency of SSEP waveforms. Nitrous oxide decreases the amplitude but has no effect on latency. Etomidate increases both the amplitude and latency. In contrast, nondepolarizing muscle relaxants (e.g., vecuronium) have no effect on sensory pathways of the nervous system and thus can be used during SSEP monitoring (Cottrell: Cottrell and Young’s Neuroanesthesia, ed 5, pp 125–126).
767. A 75-year-old patient with signs and symptoms of a leaking cerebral aneurysm is brought to the emergency room for evaluation. T-wave inversion, a prolongation of the QT interval, and U waves are noted on the preoperative electrocardiogram. Appropriate action at this point would be A. Begin infusion of nitroglycerin B. Check serum calcium and potassium C. Administer esmolol D. Place a pulmonary artery catheter
767. (B) In addition to ECG changes (e.g., T-wave inversion, depression of the ST segment, appearance of U waves, prolonged QT interval, and, rarely, Q waves), abnormal thallium scintigraphy, regional wallmotion abnormalities, and elevated creatine kinase–MB isoenzymes have been reported in patients with SAH. Although they have historically been considered functionally insignificant neurogenic phenomena, there is increasing evidence that these changes may be a sign of underlying myocardial ischemia. However, even if myocardial ischemia is present, it seems to have a minimal impact on patient outcome (i.e., morbidity and mortality). Because electrolyte abnormalities (e.g., hypokalemia or hypocalcemia) may contribute to the etiology of the ECG changes, it would probably be most appropriate to quantify these electrolytes before initiating other therapies or canceling emergency surgery. Nitroglycerin is a potent cerebral vasodilator that could have a deleterious effect on ICP in patients with increased intracranial elastance (see explanation to Question 745) (Cottrell: Cottrell and Young’s Neuroanesthesia, ed 5, pp 220–221).
768. Which of the following pharmacologic agents would have the LEAST effect on transcranial motor evoked potentials (MEPs)? A. Isoflurane B. Nitrous oxide C. Etomidate D. Fentanyl 212 Part 2 Clinical Sciences
768. (D) The principal neuromonitoring modalities currently used to assess spinal cord integrity are: somatosensory evoked potentials (SSEPs), motor evoked potentials (MEPs), and electromyography (EMG). Each of these evaluate a functionally separate area of the nervous system. The neuromonitoring technique(s) depends on the location of the surgery and the patient’s preexisting neurologic deficits. MEPs are used to monitor the integrity of motor pathways in the nervous system during neurosurgical, orthopedic, or major vascular (e.g., procedures that involve cross-clamping of the thoracic aorta) surgery. Electrical or magnetic stimulation of the motor cortex produces an evoked potential that is propagated via descending motor pathways and can be recorded from the spinal epidural space, spinal cord, peripheral nerve, or the muscle itself. In general, inhalational and intravenous anesthetics decrease the amplitude and increase the latency of the MEP response. Opioids (e.g., fentanyl, sufentanil) are the exception to this rule and have little, if any, effect on MEP monitoring. The correct order from greatest to least for the sensitivity of neurophysiologic monitoring techniques to volatile anesthetics is MEP > SSEP >>> EMG (Cottrell: Cottrell and Young’s Neuroanesthesia, ed 5, pp 125–126; Deiner S: Highlights of anesthetic considerations for intraoperative neuromonitoring, Semin Cardiothorac Vasc Anesth 14:51–53, 2010).
769. Ketamine A. Decreases cerebral blood flow (CBF) B. Augments the CO2 responsiveness of the cerebral vasculature C. Reduces cerebral metabolic rate (CMR) D. Increases cerebral blood volume (CBV)
769. (D) Ketamine is thought to increase CBF and, consequently, CBV and ICP, by two mechanisms: (1) there may be a direct effect on cerebral vascular smooth muscle to cause vasodilation, and (2) there may be a “coupled” effect caused by an increase in CMR. There is some controversy regarding the effect of ketamine on CBF/CMR coupling. Animal studies in vivo indicate that CMR and CBF are increased proportionally in structures of the limbic system. In contrast, there is evidence from one human study that although ketamine increased CBF (up to 62%), CMR remained unchanged. Cerebral CO2 responsiveness and autoregulation are not altered by ketamine (Miller: Miller’s Anesthesia, ed 8, pp 833–834).
770. CMR is decreased by A. Isoflurane B. Seizure C. Hyperthermia D. Ketamine
770. (A) In contrast to ketamine and increased neural activity (e.g., seizures or hyperthermia), which increase CBF and CMR, volatile anesthetics cause a simultaneous, dose-dependent increase in CBF and decrease in CMR (i.e., volatile anesthetics “uncouple” global CBF and CMR) (Miller: Miller’s Anesthesia, ed 8, p 390).
771. Which of the following is LEAST likely to impair CBF autoregulation? A. Sevoflurane 2 minimum alveolar concentration (MAC) B. Intracranial tumors C. Nitrous oxide 50% D. Cerebral ischemia
771. (C) Maintenance of a relatively constant CBF despite changes in systemic mean arterial blood pressure is termed autoregulation. The upper and lower limits of autoregulation, in normotensive adult humans, are cerebral perfusion pressures of 150 to 160 and 50 to 60 mm Hg, respectively. Autoregulation appears to be impaired by intracranial tumors, head trauma, and volatile anesthetics. By contrast, nitrous oxide, barbiturates, and fentanyl do not appear to disturb autoregulation (Faust: Anesthesiology Review, ed 3, pp 57–59; Hines: Stoelting’s Anesthesia and Co-Existing Disease, ed 6, pp 219–221). Neurologic Physiology and Anesthesia 223
772. An 18-year-old patient is brought to the intensive care unit after sustaining a cervical spine injury and quadriplegia during a motor vehicle accident. In the first 24 hours after the injury, the patient is at risk for A. Hypothermia, hypotension, pulmonary edema B. Fever, hypertension C. Fever, hypotension, hypoglycemia D. Autonomic hyperreflexia
772. (A) Acute spinal cord injury above T4 to T6 produces a sympathectomy below the level of injury, which decreases systemic arteriolar and venous vasomotor tone, and abolishes vasopressor reflexes (i.e., spinal shock). This pathophysiologic process may continue for up to 6 weeks after injury. As spinal shock resolves, patients with spinal cord injuries cephalad to T4 to T6 may develop autonomic hyperreflexia (i.e., acute generalized sympathetic hyperactivity as a result of stimulation below the level of injury). Neurogenic pulmonary edema may develop during either spinal shock or autonomic hyperreflexia. Thermoregulation is lost, resulting in poikilothermia, because the hypothalamic thermoregulatory center is unable to communicate with the peripheral sympathetic pathways. In the cool environment of the intensive care unit, spinal cord injury patients are unable to vasoconstrict below the level of injury and thus may experience hypothermia. Loss of sympathetic-mediated vasomotor tone also results in hypotension (Hines: Stoelting’s Anesthesia and Co-Existing Disease, ed 6, pp 255–260).
773. Signs and symptoms of intracranial hypertension include A. Papilledema B. Headache C. Nausea and vomiting D. All of the above
773. (D) Signs and symptoms of intracranial hypertension include nausea and vomiting, altered level of consciousness, papilledema, seizure activity, personality changes, and coma. Additionally, patients may manifest a constellation of clinical signs referred to as Cushing triad (i.e., systemic hypertension, bradycardia, and irregular breathing pattern) (Hines: Stoelting’s Anesthesia and Co-Existing Disease, ed 6, p 222).
774. An 89-year-old man with a history of transient ischemic attacks is scheduled to undergo a carotid endarterectomy under general anesthesia. Which of the following would be appropriate in the anesthetic management of this patient? A. Hyperventilation of the lungs to a Paco2 of 30 mm Hg to reduce ICP B. Injection of local anesthetic around the carotid body to prevent bradycardia C. Initiation of deliberate hypotension (after induction of anesthesia) to reduce bleeding D. Induction of anesthesia with propofol
774. (D) General anesthesia can be induced safely in patients with carotid artery disease using intravenous anesthetics, such as thiopental, midazolam, propofol, or etomidate. Isoflurane, in conjunction with N2O or opioids, is a good choice for maintenance of anesthesia in these patients, because critical CBF is reduced during isoflurane, sevoflurane, or desflurane anesthesia, which may provide some cerebral protection (also see explanation to Question 752). Arterial blood pressure and Paco2 should be maintained in the normal ranges for each patient because the vasculature within ischemic regions of the brain have lost the ability to autoregulate CBF and respond to changes in Paco2. Marked reductions in arterial blood pressure may reduce CBF (especially via collateral channels) to ischemic brain tissue. Theoretically, if Paco2 is increased from normal, cerebral blood vessels surrounding the region of ischemia that retain normal CO2 responsiveness will dilate, diverting regional cerebral blood flow away from the ischemic brain tissue (i.e., steal phenomenon). Conversely, if the Paco2 is reduced from normal, the cerebral blood vessels surrounding the ischemic brain tissue will constrict, diverting regional CBF (rCBF) to ischemic areas of the brain (inverse steal phenomenon or Robin Hood effect). Hyperventilating the lungs in an attempt to produce the inverse steal phenomenon is not recommended because the actual effect may be unpredictable and supportive evidence in humans that this is beneficial is lacking. The carotid sinus (not carotid body) baroreceptor reflex can be blunted by intravenous injection of atropine or by local infiltration of the area of the carotid sinus with a local anesthetic (Cottrell: Cottrell and Young’s Neuroanesthesia, ed 5, pp 278–279, 285–288).
775. Anesthetics that decrease ICP include A. Fentanyl B. Nitrous oxide C. Propofol D. All of the above
775. (C) In general, all volatile anesthetics (e.g., isoflurane, sevoflurane, and desflurane) are potent direct cerebral vasodilators that produce dose-dependent increases in CBF, CBV, and ultimately ICP when concentrations exceed 0.6 MAC. The order of vasodilator potency is approximately halothane ≫ enflurane > isoflurane = sevoflurane = desflurane. As discussed in the response to Question 731, opioids have little, if any, effect on CMR, CBF, or ICP (provided minute ventilation is maintained). The effect of N2O on CBF, CBV, and ICP is controversial. In a number of animal and human studies, N2O increased CBF by 35% to 103%. Conversely, in other animal studies, N2O was consistently found to have only minimal effects on CBF. Differences between species may be one factor contributing to these conflicting results. Because N2O appears to increase CBF and CBV in humans, it seems prudent to discontinue N2O in patients in whom intracranial hypertension is not responsive to other therapeutic maneuvers. Propofol and barbiturates are potent cerebral vasoconstrictors and can decrease ICP (Hines: Stoelting’s Anesthesia and Co-Existing Disease, ed 6, pp 223–224).
776. Therapy that is useful in the treatment of cerebral vasospasm includes all of the following EXCEPT A. Blood pressure elevation B. Hemodilution C. Diuretics D. Calcium channel blockers
776. (C) After SAH, the incidence and severity of cerebral vasospasm have been reported to correlate with the amount and location of blood in the calvarium. Angiographic evidence of vasospasm has been noted in up to 70% of SAH patients. However, clinically significant vasospasm occurs in only 20% to 30% of SAH patients. The incidence peaks approximately 7 days after SAH. Calcium channel blockers (e.g., nimodipine) decrease the morbidity and mortality associated with vasospasm, but investigators have been unable to demonstrate any significant change in the incidence or severity of vasospasm. This suggests that the beneficial effects of nimodipine may be related to inhibition of primary and secondary 224 Part 2 Clinical Sciences ischemic cascades, rather than direct cerebral vasodilation. Treatment of vasospasm also includes “triple H therapy” (Hypervolemia, induced Hypertension, and Hemodilution) and cerebral angioplasty. The rationale for induced hypervolemia and hypertension is that ischemic regions of brain have impaired autoregulation, and thus CBF is perfusion pressure dependent. Hemodilution is thought to increase blood flow through the cerebral microcirculation (because of improved rheology and reactive hyperemia). One argument against hemodilution is that increases in CBF are offset by concomitant decreases in the oxygen-carrying capacity. Taken together, blood pressure reductions and diuretic use are incorrect responses to this condition (Cottrell: Cottrell and Young’s Neuroanesthesia, ed 5, pp 223–224).
777. All of the following are associated with acromegalic patients undergoing transsphenoidal hypophysectomy EXCEPT A. Enlargement of the tongue and epiglottis B. Narrowing of the glottic opening C. A 20% to 30% incidence of difficult intubation D. Increased need for postoperative continuous positive airway pressure (CPAP) because obstructive sleep apnea (OSA) is more common
777. (D) Enlargement of the tongue and epiglottis predisposes the patient to upper airway obstruction and makes visualization of the vocal cords more difficult. The vocal cords are enlarged, making the glottic opening narrower. In addition, subglottic narrowing may be present as well as tracheal compression from an enlarged thyroid (seen in about 25% of acromegalic patients). This often necessitates the use of a narrower endotracheal tube than one might choose based on the facial enlargement. The placement of nasal airways may be more difficult due to the enlarged nasal turbinates. The use of CPAP is contraindicated after transsphenoidal hypophysectomy (Miller: Miller’s Anesthesia, ed 8, p 2188; Gupta: Essentials of Neuroanesthesia and Neurointensive Care, ed 1, pp 144–145; Fleisher: Anesthesia and Uncommon Diseases, ed 6, pp 417).
778. The CBF autoregulatory curve is shifted to the right by A. Hypoxia B. Volatile anesthetics C. Hypercarbia D. Chronic hypertension
778. (D) Chronic hypertension shifts the CBF autoregulatory curve to the right. The clinical significance of this observation is that CBF could decrease and cerebral ischemia could occur at a higher mean systemic arterial blood pressure in patients with chronic hypertension compared with normotensive patients. Chronic antihypertensive therapy to control systemic blood pressures within the normal range may restore normal CBF autoregulation (Cottrell: Cottrell and Young’s Neuroanesthesia, ed 5, p 29).
779. Autoregulation is abolished by A. Hyperbaric oxygen B. Cardiopulmonary bypass with a core temperature of 27° C C. Chronic hypertension D. 3% Isoflurane
779. (D) Cerebral autoregulation is disturbed in a number of diseases (e.g., acute cerebral ischemia, mass lesions, trauma, inflammation, prematurity, neonatal asphyxia, and diabetes mellitus). The final common pathway of dysfunction, in its most extreme form, is termed “vasomotor paralysis.” Hyperoxia has little or no effect on autoregulation. During normothermic and moderate hypothermic (i.e., approximately 27° C) cardiopulmonary bypass, autoregulation is well preserved. Chronic hypertension causes a rightward shift of the autoregulation curve toward higher upper and lower cerebral perfusion pressure limits (also see explanation to Question 778). Autoregulation is impaired by volatile anesthetics (e.g., isoflurane). At greater than 2 MAC, autoregulation is abolished (Faust: Anesthesiology Review, ed 3, pp 58-59; Cottrell: Cottrell and Young’s Neuroanesthesia, ed 5, p 88).
780. Etomidate does all of the following EXCEPT A. Abolishes CO2 reactivity B. Reduces CMRO2 C. Increases both SSEP amplitude and latency D. Reduces CBF
780. (A) The cerebral pharmacologic profile of etomidate is similar to that of thiopental and propofol in that it produces a dose-related decrease in the CMR and CBF (via direct cerebral vasoconstriction and coupling to decreased CMR). As noted, after barbiturate administration, intravenous etomidate does not disturb cerebral autoregulation or CO2 reactivity, as discussed in the explanation to Question 755. Etomidate increases both amplitude and latency during SSEP monitoring (Cottrell: Cottrell and Young’s Neuroanesthesia, ed 5, p 84).
781. Following a motor vehicle accident, a 25-year-old man is brought to the operating room for repair of facial lacerations and fractures, and abdominal exploration. The patient is extremely micrognathic and weighs 150 kg (330 lb). Acceptable techniques for securing the airway include A. Laryngeal mask airway B. Awake fiberoptic intubation C. Direct laryngoscopy after rapid-sequence induction D. Blind nasal intubation
781. (B) Nasal intubation should be avoided in patients with suspected anterior basal skull fractures (e.g., disruption of the cribriform plate of the ethmoid bone) or sinus injuries. Because approximately 10% of head injury patients have associated cervical spine injuries, it is prudent to assume that all head injury patients have coexisting cervical spine injury until proved otherwise. Additionally, the patient described in this question may have abnormal airway anatomy because of extreme micrognathia, facial injuries, and obesity. Taken together, direct laryngoscopy with rapid-sequence induction is probably not an acceptable technique for securing this patient’s airway. In contrast, awake intubation by direct, video, or fiberoptic laryngoscopy or performance of tracheostomy are considered appropriate techniques for tracheal intubation of this patient. Mask and laryngeal mask airway (LMA) techniques may provide a patent airway but do not ensure protection of the airway against aspiration of gastric contents (Cottrell: Cottrell and Young’s Neuroanesthesia, ed 5, pp 375–378).
782. After resection of a grade II astrocytoma in a 60-yearold patient, the serum sodium is 127 mEq/L. Urine sodium is 25 mEq/L. Therapy could include which of the following? A. Intranasal or IV vasopressin (DDAVP) B. 500 mL 3% saline over 30 minutes C. Chlorpropamide D. Demeclocycline Neurologic Physiology and Anesthesia 213
782. (D) This patient has mild hyponatremia and is unable to excrete a dilute urine as noted by the urine sodium greater than 20 mEq/L. These are consistent with the syndrome of inappropriate secretion of ADH (SIADH). Antidiuretic hormone (ADH) is also known as vasopressin. SIADH may result from a variety Neurologic Physiology and Anesthesia 225 of causes including central nervous system lesions, pulmonary infections, hypothyroidism, and drugs (e.g., chlorpropamide, narcotics). After identifying the cause, treatment is started and usually consists mainly of water restriction. With severe hyponatremia (i.e., Na less than 120 mEq/L and signs of mental confusion), aggressive treatment with hypertonic sodium chloride may be needed; however, too much and too rapid infusion, as in choice B, may induce central pontine myelinolysis and may cause permanent brain damage. With severe hyponatremia, the dose of 200 to 300 mL of a 3% solution of sodium chloride is usually administered over several hours. The antibiotic demeclocycline interferes with ADH at the level of the renal tubules to produce dilute urine and is sometimes used for the treatment of SIADH. In the future, the experimental drug tolvaptan (OPC-41061) may replace demeclocycline. Tolvaptan is a vasopressin antagonist. Desmopressin acetate (DDAVP) is used to treat patients with complete diabetes insipidus (DI), whereas chlorpropamide is used to treat incomplete DI. In contrast to SIADH, patients with DI have a lack of ADH and have high output of poorly concentrated urine and hypernatremia. Leaving the patient intubated and hyperventilating him or her will not help (Barash: Clinical Anesthesia, ed 7, p 1352; Miller: Miller’s Anesthesia, ed 8, pp 1787–1789).
783. A 48-year-old, 110-kg man with history of meningioma is scheduled for craniotomy for tumor debulking. His wife states he has been somnolent and confused. On examination he is noted to be hyperventilating and sleepy, but arousable and hypertensive. Useful measures for his anesthetic include A. Rapid-sequence induction using succinylcholine B. Hyperventilation to 20 mm Hg C. 10 cm H2O PEEP to reduce atelectasis D. Esmolol to reduce response to intubation
783. (D) This patient has several signs consistent with elevated intracranial pressure: hypertension, hyperventilation, and somnolence. Use of morphine premedication is ill-advised because it would sedate him further, blunt his hyperventilation, and thus raise ICP. Furthermore, narcotics in this setting can lower blood pressure sufficiently to alter cerebral perfusion pressure. Use of PEEP can promote impairment of venous drainage as well as raise ICP in patients with intracranial hypertension. Hyperventilation is an effective maneuver for lowering ICP in the short term. As discussed in the explanations to Questions 733 and 764, Paco2 levels in the range of 25 to 30 mm Hg suffice for this, and there is no evidence that additional hyperventilation has any added therapeutic benefit. Use of esmolol prior to intubation may blunt the hyperdynamic response to laryngoscopy and prevent ICP elevation (Hines: Stoelting’s Anesthesia and Co-Existing Disease, ed 6, pp 222–223, 226).
784. If, during an MRI scan, a patient were to become pinned by a large (50 kg) metallic object, the appropriate course of action would be to A. Stop the scan immediately to release the magnet B. Summon enough people to pull the object away C. Interrupt electrical power for 60 seconds to release the magnetic force D. None of the above
784. (D) MRI scanners contain powerful magnets that range from 0.5 to 3 T (5000-30,000 G). By contrast, the Earth’s magnetic field is 0.5 G. Metal objects brought into the scanner room can become dangerous projectiles that fly toward the middle of the magnet, where the patient is located. Small items can be pulled away, but larger items may not be removable even with a winch and thus require a magnet shutdown, the process known as a quench. MRI magnets are always on. Stopping the scan or cutting the power to magnet for 60 seconds does not release the magnetic force. Quenching is an expensive process that causes the cooling medium (liquid helium) to boil off and vent to the outside. During this process, the coils become resistive and cease superconducting, thereby diminishing magnetic field strength. Attempting to pull the object described in this question away from the magnet would be nearly impossible; but, even if it could be successfully carried out, there would be great risk. For example, if the grip were lost and the object released, it could fly toward the patient inside the scanner. Cutting up metallic objects attached to the scanner (if a nonferromagnetic saw could be found) would be equally if not more dangerous than attempting a pull away (Stoelting: Basics of Anesthesia, ed 6, pp 620–621).
785. A 45-year-old man is undergoing a posterior cervical fusion in the sitting position. Induction of anesthesia and tracheal intubation are uneventful. Anesthesia is maintained with N2O 50% in O2, and sevoflurane. Suddenly, air is heard on the precordial Doppler ultrasound. Other observations consistent with VAE include A. Decreased Paco2 B. Decreased central venous pressure C. Decreased pulmonary and arterial blood pressure D. Decreased end-tidal CO2
785. (D) Progressive entrainment of air into the pulmonary microcirculation reduces lung perfusion and increases pulmonary vascular resistance and alveolar dead-space ventilation. The increase in pulmonary vascular resistance is reflected by increases in pulmonary arterial and central venous pressures. A large air embolus can result in right ventricular outflow obstruction, which will dramatically reduce cardiac output, resulting in systemic hypotension. Increased alveolar dead space results in a decrease in end-tidal CO2. In severe VAE, CO2 cannot be eliminated and Paco2 increases. End-tidal N2 increases because air diffuses into the pulmonary alveoli. The sensitivity of continuous end-tidal CO2 monitoring is similar to that for continuous end-tidal N2 monitoring (also see explanation to Question 744) (Faust: Anesthesiology Review, ed 3, pp 389–391, Figure 158-1).
786. In patients with increased ICP, hyperventilation is typically limited to a Paco2 of 25 to 30 mm Hg because additional hyperventilation A. Is virtually impossible B. Causes brain ischemia due to a rightward shifting of the oxyhemoglobin dissociation curve C. May be associated with a worsening of neurologic outcome D. Could result in paradoxical cerebral vasodilation
786. (C) The cerebrovascular response to hyperventilation was reviewed in the explanations to Questions 733, 763, 764, and 783. Hyperventilation, and the resulting respiratory alkalosis, causes a leftward (not rightward) shifting of the oxyhemoglobin dissociation curve. In doing so, hemoglobin undergoes a conformation change, making it more reluctant to release oxygen at the tissue level. As discussed in the explanation to Question 733, hyperventilation-induced respiratory alkalosis can precipitate hypokalemia. Specifically, serum potassium decreases 0.6 mEq/L for each 0.1-unit increase in pH. Thus, overly aggressive hyperventilation should be guarded against to avoid electrolyte perturbations that may result in cardiac arrhythmias (Miller: Miller’s Anesthesia, ed 8, pp 2163–2164). 226 Part 2 Clinical Sciences
787. Of the measures below, which is the LEAST useful in response to suspected VAE during a neurosurgical procedure in the upright position? A. Application of 10 cm H2O PEEP B. Discontinuation of N2O C. Placement of wax on cut bone edges D. Trendelenburg position 214 Neurologic Physiology and Anesthesia Answers, References, and Explanations
787. (A) VAE is a hazard of any operation in which the operative field is located above the heart. As discussed in the explanation for Question 757, measures to successfully manage VAE include prevention of further air entrainment (Trendelenburg position, flooding surgical field with saline, placement of wax on cut bone edges), removal of air from the right atrium if a catheter is indwelling, supporting hemodynamics (e.g., with calcium, vasopressors, and inotropes), and discontinuation of N2O to prevent bubble expansion. Some neuroanesthesiologists avoid use of N2O in any instance where there is a chance of VAE (Barash: Clinical Anesthesia, ed 7, p 1446; Miller: Miller’s Anesthesia, ed 8, pp 2172–2173). 227
788. Tachyphylaxis to local anesthetics is MOST closely related to which of the following? A. Speed of injection B. Dosing interval C. Volume of local anesthetic D. pH of solution
788. (B) Tachyphylaxis is a well-known phenomenon associated with repeated injections of local anesthetics leading to decreased effectiveness. Interestingly, the dosing interval seems most important in the development of tachyphylaxis. If the dosing interval is short (and there is no pain between injections), tachyphylaxis does not develop. However, with longer dosing intervals (and pain between injections), tachyphylaxis develops (Miller: Miller’s Anesthesia, ed 8, pp 1051–1052).
789. Which of the following techniques is LEAST effective in the treatment of pruritus from administration of neuraxial opiates? A. Nalbuphine 5 mg IV B. Dexmedetomidine 30 μg IV C. Diphenhydramine 50 mg IV D. Propofol 10 mg IV
789. (B) The treatment of pruritus, the most common side effect of neuraxial opiates, is primarily with opioid antagonists, mixed opioid agonist–antagonists, and antihistamine drugs (by their sedating effects). Nalbuphine is a mixed opioid agonist–antagonist; diphenhydramine has antihistamine properties. Propofol at very low doses (e.g., 10 mg) has been useful to treat pruritus not only induced by neuraxial opiates but also the pruritus associated with cholestatic liver disease. Propofol does not affect analgesia, whereas opioid antagonists and mixed agonist–antagonists may reverse some or all of the analgesia, depending upon dose. Dexmedetomidine is a highly selective α2-receptor agonist that has a faster onset and shorter duration of action compared with clonidine. Dexmedetomidine has analgesic properties, can potentiate neuraxial analgesia when injected spinally, and can perhaps decrease the incidence of pruritus by reducing the amount of narcotic dose used. It does not treat pruritus (Barash: Clinical Anesthesia, ed 7, p 519; Miller: Miller’s Anesthesia, ed 8, pp 2986–2987).
790. The MAXIMUM dose of lidocaine containing 1:200,000 epinephrine that can be administered to a 70-kg patient for most major regional anesthetic techniques (and excluding spinal and IV regional) is A. 100 mg B. 200 mg C. 500 mg D. 1000 mg
790. (C) The maximum dose of local anesthetics containing 1:200,000 epinephrine that can be used for major nerve blocks in a healthy 70-kg adult is lidocaine, 500 mg; mepivacaine, 500 mg; prilocaine, 600 mg; bupivacaine, 225 mg; levobupivacaine, 225 mg; ropivacaine 250 mg (Miller: Miller’s Anesthesia, ed 8, p 1043, Table 36-6).
791. Which of the following concentrations of epinephrine corresponds to a 1:200,000 mixture? A. 0.5 μg/mL B. 5 μg/mL C. 50 μg/mL D. 0.5 mg/mL
791. (B) 1:200,000 means 1 g/200,000 mL = 1000 mg/200,000 mL = 1 mg/200 mL 1 mg/200 mL = 1000 μg/200 mL = 10 μg/2 mL = 5 μg/mL
792. An anesthesia pain service consult is sought for a 78-year-old patient with a complaint of pain in the distribution of the trigeminal nerve. The patient has no other medical problems except a history of congestive heart failure, for which he takes digoxin and thiazide. In addition to his chief complaint, the patient over the last 72 hours has complained of dysesthesia in the feet, difficulty with vision, and emesis times three. The MOST appropriate step at this time would be A. Trigeminal nerve block with bupivacaine B. Neurologic workup for multiple sclerosis C. Administration of fentanyl and ondansetron D. Obtaining a digoxin level
792. (D) The early signs of digitalis toxicity include loss of appetite and nausea and vomiting. In some patients, there may be pain that is similar to trigeminal neuralgia. Pain or discomfort in the feet and pain and discomfort in the extremities may be a feature of digitalis toxicity. Transient visual disturbances (e.g., amblyopia, scotomata) have been reported in patients with digitalis toxicity. In this patient, it would be prudent to obtain a digoxin level as an early part of the workup for these complaints. He may also have true trigeminal neuralgia, and workup for this condition can be undertaken after digitalis toxicity has been ruled out (Stoelting: Pharmacology and Physiology in Anesthetic Practice, ed 4, pp 314–315).
793. Which of the following is the EARLIEST sign of lidocaine toxicity from a high blood level? A. Shivering B. Nystagmus C. Light-headedness and dizziness D. Tonic-clonic seizures
793. (C) Toxic reactions to local anesthetics are usually due to intravascular or intrathecal injection or to an excessive dosage. The initial symptoms of local anesthetic toxicity from high blood levels (inadvertent IV injection or excessive dosages) are light-headedness and dizziness, and numbness of the tongue. Patients also may note perioral numbness and tinnitus. Progressive CNS excitatory effects include visual disturbances (difficulty focusing), auditory disturbances (tinnitus), shivering, muscular twitching, and, ultimately, generalized tonic-clonic seizures. CNS depression can ensue, leading to respiratory depression or arrest. Higher levels can lead to cardiovascular collapse. To help prevent excessively high levels of local anesthetic, common practice is to aspirate for blood and inject the local anesthetic slowly and incrementally, looking for signs of toxicity (and, if appropriate, adding epinephrine to use as an intravascular marker as noted by an increase in heart rate and blood pressure) (Barash: Clinical Anesthesia, ed 7, pp 572–575; Miller: Miller’s Anesthesia, ed 8, pp 1048–1052).
794. An analgesic effect similar to the epidural administration of 5 mg of morphine could be achieved by which dose of intrathecal morphine? A. 0.05 mg B. 0.3 mg C. 1 mg D. Morphine should not be injected into the intrathecal space
794. (B) The site of action of spinally administered opiates is the substantia gelatinosa of the spinal cord. Epidural administration is complicated by factors related to dural penetration, absorption in fat, and systemic uptake; therefore, the quantity of intrathecally administered opioid required to achieve effective 238 Part 2 Clinical Sciences analgesia is typically much smaller. Lipid-soluble opioids (e.g., fentanyl) have a faster onset of action but a shorter duration of action compared to the more water soluble opioids (e.g., morphine). A dose of 1 to 5 mg of epidural morphine is approximately equal to an intrathecal dose of 0.1 to 0.3 mg of morphine. Onset time for epidural administration is 30 to 60 minutes with a peak effect in 90 to 120 minutes. Onset time for intrathecal administration is shorter than for epidural administration. Duration of 12 to 24 hours of analgesic effect can be expected by either route with morphine (Barash: Clinical Anesthesia, ed 7, pp 1627–1630; Miller: Miller’s Anesthesia, ed 8, pp 2983–2984, Table 98-4).
795. Which local anesthetic undergoes the LEAST hepatic clearance? A. Chloroprocaine B. Bupivacaine C. Ropivacaine D. Lidocaine
795. (A) Commonly injected local anesthetics are divided chemically into two groups: the amino esters (esters) and the amino amides (amides). The esters include procaine, chloroprocaine, and tetracaine (all have one letter i in the name). The amides are lidocaine, mepivacaine, prilocaine, bupivacaine, levobupivacaine, etidocaine, and ropivacaine (all have two i’s in the name). The esters undergo plasma clearance by cholinesterases and have relatively short half-lives, whereas the amides undergo hepatic clearance and have longer half-lives (Butterworth: Morgan & Mikhail’s Clinical Anesthesiology, ed 5, pp 266–271; Miller: Miller’s Anesthesia, ed 8, p 1046).
796. Which of the following is the MOST important disadvantage of interscalene brachial plexus block compared with other approaches? A. Large volumes of local anesthetics required B. Frequent sparing of the ulnar nerve C. Frequent sparing of the musculocutaneous nerve D. High incidence of pneumothorax
796. (B) The major disadvantage of the interscalene block for hand and forearm surgery is that blockade of the inferior trunk (C8-T1) is often incomplete. Supplementation of the ulnar nerve often is required. The risk of pneumothorax is quite low, but blockade of the ipsilateral phrenic nerve occurs in up to 100% of blocks. This can cause respiratory compromise in patients with significant lung disease. Horner syndrome from blockade of the stellate ganglion can occur in 70% to 90% of patients if large volumes of local anesthetic are injected (Hebl: Mayo Clinic Atlas of Regional Anesthesia and Ultrasound-Guided Nerve Blockade, ed 1, pp 191–205; Miller: Miller’s Anesthesia, ed 8, pp 1724–1727).
797. A 68-year-old woman is to undergo lower extremity surgery under spinal anesthesia. Which of the following statements concerning the immediate physiologic response to the surgical incision is TRUE? A. The cardiovascular (CV) response to stress will be blocked, but the adrenergic response will not B. The adrenergic response to stress will be blocked, but the CV response will not C. Both the adrenergic and CV responses will be blocked D. Neither the adrenergic nor the CV response will be blocked
797. (C) Surgical trauma includes a wide variety of physiologic responses. General anesthesia has no or only a slight inhibitory effect on endocrine and metabolic responses to surgery. Regional anesthesia inhibits the nociceptive signal from reaching the CNS and, therefore, has a significant inhibitory effect on the stress response, including adrenergic, cardiovascular, metabolic, immunologic, and pituitary. This effect is most pronounced with procedures on the lower part of the body and less with major abdominal and thoracic procedures. The variable effect is probably due to unblocked afferents (i.e., vagal, phrenic, or sympathetic) (Barash: Clinical Anesthesia, ed 7, p 1353; Miller: Miller’s Anesthesia ed 8, pp 3139–3141).
798. The “snap” felt just before entering the epidural space represents passage through which ligament? A. Posterior longitudinal ligament B. Ligamentum flavum C. Supraspinous ligament D. Interspinous ligament
798. (B) The structures that are traversed by a needle placed in the midline prior to the epidural space are as follows: skin, subcutaneous tissue, supraspinous ligament, interspinous ligament, and ligamentum flavum. The ligamentum flavum is tough and dense, and a change in the resistance to advancing the needle is often perceived and to many feels like a “snap.” The anterior and posterior longitudinal ligaments bind the vertebral bodies together. See also explanation and diagram in Question 870 (Barash: Clinical Anesthesia, ed 7, pp 913–914; Miller: Miller’s Anesthesia, ed 8, pp 1685–1688).
799. The common element thought to be present in cases of cauda equina syndrome after continuous spinal anesthesia is A. Use of microcatheter B. Maldistribution of local anesthetic C. Administration of lidocaine D. Addition of epinephrine
799. (B) The symptoms of cauda equina syndrome include low back pain, bilateral lower extremity weakness, saddle anesthesia, and loss of bowel and bladder control. Pooling of local anesthetics in dependent areas of the spine within the subarachnoid space has been identified as the causative factor in cases of cauda equina syndrome. Microlumen catheters (27-gauge and smaller) may enhance the nonuniform distribution of solutions within the intrathecal space, but cauda equina syndrome has been associated with the use of larger catheters, 5% lidocaine with dextrose, and 2% lidocaine, as well as 0.5% tetracaine (Barash: Clinical Anesthesia, ed 7, pp 576, 928; Miller: Basics of Anesthesia, ed 6, p 269).
800. When performing a single-shot spinal anesthetic, the level of block for motor, sensory, and sympathetic blocks differs often by at least two dermatomes. Which of the following sequences is correct from the highest to the lowest level of block? A. Sensory, sympathetic, motor B. Sympathetic, sensory, motor C. Sympathetic, motor, sensory D. Sensory, motor, sympathetic
800. (B) Differential nerve blockade is a complex process with both peripheral nerve blocks and central nerve blocks. With spinal anesthesia, the sympathetic nerve block may be anywhere between two and six dermatomes higher than the sensory block, as noted by pin prick. Sensory block is two to three dermatomes higher than the motor block. However, with epidural anesthesia, the sympathetic and sensory blocks tend to be at the same dermatome level and are higher than the motor block (Barash: Clinical Anesthesia, ed 7, p 923).
801. A 95-year-old woman has persistent and prolonged thoracic pain after a herpes zoster infection. Which of the treatments below would be the LEAST efficacious in the treatment of her pain? A. Oral amitriptyline B. Oral clonidine C. Topical capsaicin ointment D. Topical lidocaine patch
801. (B) Acute herpes zoster is due to the reactivation of the varicella-zoster virus. Acute treatment includes symptomatic pain treatment and antiviral drugs (e.g., acyclovir, famciclovir, or valacyclovir). It is Anatomy, Regional Anesthesia, and Pain Management 239 typically a benign and self-limiting disease in patients younger than 50 years of age. As one gets older, the incidence of postherpetic neuralgia (PHN), defined as pain persisting for more than 3 months after resolution of the rash, increases. The incidence of PHN is about 30% to 50% in patients older than 50 years. Treatment of established PHN has been shown to be resistant to interventions and, thus, can be difficult. However, proven therapies include tricyclic antidepressants, anticonvulsants, opioids, topical local anesthetics (e.g., 5% lidocaine patch), topical capsaicin, and TENS. Sympathetic blocks can provide excellent analgesia but are most useful during the more acute stages of the disease rather than during the late chronic stages. Sympathetic blocks in the acute stages may decrease the incidence of PHN. Oral clonidine, which is used to treat hypertension and opioid withdrawal, has not been shown to be an effective treatment for PHN (Barash: Clinical Anesthesia. ed 7, p 1657; Butterworth: Morgan & Mikhail’s Clinical Anesthesiology, ed 5, pp 1049–1050; Raj: Practical Management of Pain, ed 3, pp 187–189).
802. The deep peroneal nerve innervates the A. Lateral aspect of the dorsum of the foot B. Entire dorsum of the foot C. Web space between the great toe and the second toe D. Medial aspect of the dorsum of the foot
802. (C) The deep peroneal nerve innervates the short extensors of the toes and the skin of the web space between the great and second toe. The deep peroneal nerve is blocked at the ankle by infiltration between the tendons of the anterior tibial and extensor hallucis longus muscles (Hebl: Mayo Clinic Atlas of Regional Anesthesia and Ultrasound-Guided Nerve Blockade, ed 1, pp 424–427, 446–450).
803. The correct arrangement of local anesthetics in order of their ability to produce cardiotoxicity from most to least is A. Bupivacaine, lidocaine, ropivacaine B. Bupivacaine, ropivacaine, lidocaine C. Ropivacaine, bupivacaine, lidocaine D. Lidocaine, ropivacaine, bupivacaine
803. (B) CNS toxicity from local anesthetics generally parallels anesthetic potency (e.g., bupivacaine is four times as potent as lidocaine, and ropivacaine is three times as potent as lidocaine). Cardiovascular (CV) toxicity occurs at a higher blood level than CNS toxicity. For bupivacaine and ropivacaine, CV toxicity occurs at two times the CNS dose, whereas for lidocaine the CV toxicity occurs at seven times the CNS toxicity levels, making lidocaine the least cardiotoxic and bupivacaine the most cardiotoxic of the listed local anesthetics (Barash: Clinical Anesthesia, ed 7, pp 573–575; Miller: Miller’s Anesthesia, ed 8, pp 1049–1050).
804. Allodynia is defined as A. Spontaneous pain in an area or region that is anesthetic B. Pain initiated or caused by a primary lesion or dysfunction in the nervous system C. An increased response to a stimulus that is normally painful D. Pain caused by a stimulus that does not normally provoke pain
804. (D) The International Association for the Study of Pain (IASP) has defined several pain terms. Anesthesia dolorosa refers to spontaneous pain in an area or region that is anesthetic. Neuropathic pain is pain initiated or caused by a primary lesion or dysfunction in the nervous system. Dysesthesia is an unpleasant abnormal sensation, whether spontaneous or evoked. Hyperalgesia is an increased response to a stimulus that is normally painful. Allodynia is pain caused by a stimulus that does not normally provoke pain (Barash: Clinical Anesthesia, ed 7, pp 1649–1650; Butterworth: Morgan & Mikhail’s Clinical Anesthesiology, ed 5, pp 1025–1026).
805. The primary mechanism by which the action of tetracaine is terminated when used for spinal anesthesia is A. Systemic absorption B. Uptake into neurons C. Hydrolysis by pseudocholinesterase D. Hydrolysis by nonspecific esterases
805. (A) Ester local anesthetics are hydrolyzed by cholinesterase enzymes that are present mainly in plasma and, in a smaller amount, in the liver. Because there are no cholinesterase enzymes present in cerebrospinal fluid (CSF), the anesthetic effect of tetracaine will persist until it is absorbed into systemic circulation. The rate of hydrolysis varies, with chloroprocaine being fastest, procaine intermediate, and tetracaine the slowest. Toxicity is inversely related to the rate of hydrolysis; tetracaine is, therefore, the most toxic of the three esters listed in this question (Butterworth: Morgan & Mikhail’s Clinical Anesthesiology, ed 5, pp 270–271).
806. Complex regional pain syndrome type I (reflex sympathetic dystrophy [RSD]) is differentiated from complex regional pain syndrome type II (causalgia) by knowledge of its A. Etiology B. Chronicity C. Type of symptoms D. Rapidity of onset
806. (A) Complex regional pain syndrome type I (CRPS type I), also called RSD, is a clinical syndrome of continuous burning pain, usually occurring after minor trauma. Patients present with various sensory, motor, autonomic, and trophic changes. Complex regional pain syndrome type II (CRPS type II; causalgia) exhibits the same features of RSD, but there is a preceding nerve injury (e.g., median nerve of the upper extremity or tibial division of the sciatic nerve in the lower extremity) (Barash: Clinical Anesthesia, ed 7, pp 1657–1658; Butterworth: Morgan & Mikhail’s Clinical Anesthesiology, ed 5, pp 1048–1049).
807. The primary determinant of local anesthetic potency is A. pKa B. Molecular weight C. Lipid solubility D. Protein binding
807. (C) The potency of local anesthetics is directly related to their lipid solubility. In general, the speed or onset of action of local anesthetics is related to the pKa of the drug. Drugs with lower pKa values have a higher amount of non-ionized molecules at physiologic pH and penetrate the lipid portion of nerves faster (an exception is chloroprocaine, which has a fast onset of action that may be related to the higher concentration of drug used) (Barash: Clinical Anesthesia, ed 7, pp 566–567; Butterworth: Morgan & Mikhail’s Clinical Anesthesiology, ed 5, pp 268–269). 240 Part 2 Clinical Sciences
808. Which of the following would have the GREATEST effect on the level of sensory blockade after a subarachnoid injection of hyperbaric 0.75% bupivacaine? A. Patient age B. Addition of epinephrine to the local anesthetic solution C. Patient weight D. Patient position
808. (D) Many factors have an effect on the sensory level after a subarachnoid injection. The baricity of the solution and the patient position (e.g., lateral, sitting, prone) are the most important determinants of sensory level. The other listed options have little to no effect on sensory level. Patient height also has little effect on sensory level (Barash: Clinical Anesthesia, ed 7, pp 916–919; Miller: Miller’s Anesthesia, ed 8, pp 1693–1694).
809. Which of the following local anesthetics would produce the LOWEST concentration in the fetus relative to the maternal serum concentration during a continuous lumbar epidural? A. Ropivacaine B. Bupivacaine C. Lidocaine D. Chloroprocaine
809. (D) Chloroprocaine is an ester local anesthetic that is rapidly metabolized by pseudocholinesterase. With the epidural injection of chloroprocaine, very little drug is available to cross the placenta, because the halflife is about 45 seconds in the mother (and that which crosses is also rapidly metabolized, making fetal effects essentially nonsignificant). The amide local anesthetics (e.g., ropivacaine, bupivacaine, lidocaine) undergo liver metabolism and have relatively long half-lives, but with prolonged epidural administration may accumulate in the fetus (Barash: Clinical Anesthesia, ed 7, p 1148; Miller: Miller’s Anesthesia, ed 8, p 2344).
810. Severe hypotension associated with high spinal anesthesia is caused primarily by A. Decreased cardiac output secondary to decreased preload B. Decreased systemic vascular resistance C. Decreased cardiac output secondary to bradycardia D. Decreased cardiac output secondary to decreased myocardial contractility Anatomy, Regional Anesthesia, and Pain Management 229
810. (A) Hypotension with a high spinal anesthesia is related to sympathetic blockade, venodilation (decreases preload), arterial dilation (decreases afterload), and a decrease in heart rate (cardioaccelerator fibers T1-T4 blockade and a fall in right atrial filling that affects the intrinsic chronotropic stretch receptors). With a high spinal, the decrease in venous dilation is the predominant cause of hypotension (Barash: Clinical Anesthesia, ed 7, pp 923–925; Miller: Miller’s Anesthesia, ed 8, pp 1688–1690; Miller: Basics of Anesthesia, ed 6, p 270).
811. Select the one TRUE statement concerning phantom limb pain. A. The incidence of phantom limb pain increases with more distal amputations B. Most amputees do not experience phantom limb pain C. Nerve blocks may be used to decrease the incidence of phantom limb pain D. Traumatic amputees have a much higher incidence of phantom limb pain than nontraumatic amputees
811. (C) The incidence of phantom limb pain is estimated to be up to 80% after an amputation. This pain may be immediate but, in many cases, will develop within a few days of the amputation. The pain also may not be present all the time but only a few days a month. The incidence of phantom limb pain does not differ between traumatic and nontraumatic amputees. The incidence of phantom pain increases with more proximal amputation. About 50% of patients will have a decrease in pain over time; the rest have no change or an increase in pain with time. Although very difficult to treat, nerve blocks are commonly used in the perioperative setting to decrease the incidence of phantom limb pain. Oral agents such as opioids, antidepressants, and gabapentin are commonly used as well as TENS units, spinal cord stimulators, and biofeedback methods (Barash: Clinical Anesthesia, ed 7, p 1658).
812. Which of the following is TRUE regarding intravenous regional anesthesia (Bier block)? A. Useful for postoperative pain in extremity surgery B. Can be used for extremity surgeries lasting 2 to 3 hours C. Bupivacaine is the drug of choice for prolonged blocks D. Lidocaine is most commonly used
812. (D) Intravenous regional anesthesia (IVRA, or Bier blocks after August Bier, who first described the technique) is simple to perform and is usually done only on an upper extremity. A small 20- or 22-gauge IV catheter is placed in the extremity to be blocked, then the limb is raised and an Esmarch bandage is wrapped around the extremity to remove as much blood from the limb as possible, followed by the inflation of a tourniquet to 250 to 300 mm Hg, or 2.5 times the patient’s systolic pressure, and injection of a local anesthetic into the limb. An intravenous line is always placed in another site (not below the tourniquet) in case sedation is needed for tourniquet pain or if local anesthetic toxicity develops when the tourniquet is eventually released. Typically, a minimum of 40 to 45 minutes of tourniquet time is needed to have enough local anesthetic to diffuse into the tissues to prevent serious systemic local anesthetic toxicity from developing when the tourniquet is deflated. For safety, the tourniquet is deflated for about 5 seconds and then reinflated for 45 seconds while one looks for signs of toxicity. This should be repeated four to five more times. Postoperative analgesia is lost once the tourniquet is deflated and the local anesthetic diffuses from the nerves. Tourniquet times less than 60 to 90 minutes are used to prevent pain and nerve damage from the tourniquet. Lidocaine 0.5% at a dose of 1.5 to 3 mg/kg is the most commonly administered local anesthetic because of its relative safety and effectiveness. About a 10-minute period is needed for surgical anesthesia to develop. Bupivacaine is not recommended for Bier blocks because of reports of cardiovascular toxicity and death that have occurred after the tourniquet was released (Barash: Clinical Anesthesia, ed 7, p 970; Hebl: Mayo Clinic Atlas of Regional Anesthesia and Ultrasound-Guided Nerve Blockade, pp 317–320; Miller: Basics of Anesthesia, ed 6, pp 194, 297).
813. Select the FALSE statement regarding spinal anatomy and spinal anesthesia. A. The addition of phenylephrine to lidocaine will prolong spinal anesthesia B. A high thoracic sensory block will result in total sympathetic blockade C. The largest vertebral interspace is L5-S1 D. The dural sac extends to the S4-S5 interspace
813. (D) Both phenylephrine and epinephrine will prolong a spinal anesthetic when administering lidocaine. The Taylor approach for spinal anesthesia uses a paramedian approach to the L5-S1 interspace—the largest interspace of the vertebral column. The sympathetic nervous system originates in the thoracic and lumbar spinal cord T1-L3; therefore, a high thoracic sensory level can cause a complete sympathetic block. The dural sac extends to S2, not S4-S5. The spinal cord extends to L3 in the infant and L1-L2 in adults (Barash: Clinical Anesthesia, ed 7, pp 906–920; Miller: Miller’s Anesthesia, ed 8, pp 1684–1693). Anatomy, Regional Anesthesia, and Pain Management 241
814. Four days after a left total hip arthroplasty, an obese 62-year-old woman complains of severe back pain in the region where the epidural was placed. Over the ensuing 72 hours, the back pain gradually worsens and a severe aching pain that radiates down the left leg to the knee develops. The MOST likely diagnosis is A. Epidural abscess B. Epidural hematoma C. Anterior spinal artery syndrome D. Meralgia paresthetica
814. (A) Development of an epidural abscess is fortunately an exceedingly rare complication of spinal and epidural anesthesia. Most anesthetic-related epidural abscesses are associated with epidural catheters. When an epidural abscess is developing, prompt recognition and treatment are essential if permanent sequelae are to be avoided. Symptoms from an epidural abscess may not become apparent until several days (mean, 5 days) after placement of the block. There are four clinical stages of epidural abscess symptom progression. Initially, localized back pain develops. The second stage includes nerve root or radicular pain. The third stage involves motor and sensory deficits or sphincter dysfunction, followed by the last stage of paraplegia. Unlike an epidural hematoma, in which severe back pain is the key feature, patients with epidural abscesses will complain of radicular pain approximately 3 days after development of the back pain. Fever may develop with an abscess and is rare with a hematoma. A magnetic resonance imaging (MRI) scan is helpful in the diagnosis. Anterior spinal artery syndrome is characterized predominantly by motor weakness or paralysis of the lower extremities. Meralgia paresthetica is related to entrapment of the lateral femoral cutaneous nerve as it courses below the inguinal ligament and is associated with burning pain over the lateral aspect of the thigh. It is not a complication of epidural anesthesia (Butterworth: Morgan & Mikhail’s Clinical Anesthesiology, ed 5, pp 970–972).
815. Which of the following choices is NOT consistent with a limb affected by complex regional pain syndrome? A. Allodynia B. Dermatomal distribution of pain C. Atrophy of the involved extremity D. Hyperesthesia
815. (B) Complex regional pain syndromes are associated with trauma. The main feature is burning and continuous pain that is exacerbated by normal movement, cutaneous stimulation, or stress, usually weeks after the injury. The pain is not anatomically distributed. Other associated features include cool, red, clammy skin and hair loss in the involved extremity. Chronic cases may be associated with atrophy and osteoporosis (Barash: Clinical Anesthesia, ed 7, pp 1657–1658; Butterworth: Morgan & Mikhail’s Clinical Anesthesiology, ed 5, pp 1048–1049).
816. The MAIN advantage of neurolytic nerve blockade with phenol versus alcohol is A. Denser blockade B. Blockade is permanent C. The effects of the block can be evaluated immediately D. The block is less painful
816. (D) Neurolytic blockade with phenol (6%-10% in glycerine) is painless because phenol has a dual action as both a local anesthetic and a neurolytic agent. The initial block wears off over a 24-hour period, during which time neurolysis occurs. For this reason you must wait a day to determine the effectiveness of the neurolytic block. Alcohol (50%-100% ethanol) is painful on injection and should be preceded by local anesthetic injection. Unfortunately, there is no neurolytic agent that affects only sympathetic fibers (Barash: Clinical Anesthesia, ed 7, pp 1658–1659; Miller: Miller’s Anesthesia, ed 8, pp 1910–1911).
817. How much local anesthetic should be administered per spinal segment to patients between 20 and 40 years of age receiving a lumbar epidural anesthetic? A. 0.25 to 0.5 mL B. 0.5 to 1.0 mL C. 1 to 2 mL D. 2 to 3 mL
817. (C) In general, each 1 to 2 mL of local anesthetic will anesthetize about one spinal segment in the 20- to 40-year-old patient. Because of the negative intrathoracic pressure transmitted to the epidural space with breathing, about two thirds of the segments are blocked above the level of the lumbar placement and one third of segments are blocked below the injection. For example, to achieve a T4 block when an epidural is placed at the L2-L3 space, about 10 segments above and five segments below the epidural would be needed (15 segments) or about 15 to 30 mL. As one gets older, the dose of local anesthetic mL/segment decreases (e.g., an 80-year-old may need 0.75-1.5 mL/segment). Also, pregnant patients are more sensitive to local anesthetics, and reduced doses are needed (Barash: Clinical Anesthesia, ed 7, pp 920–922; Butterworth: Morgan & Mikhail’s Clinical Anesthesiology, ed 5, p 962; Miller: Basics of Anesthesia, ed 6, p 277).
818. The artery of Adamkiewicz MOST frequently arises from the aorta at which spinal level? A. T1-T4 B. T5-T8 C. T9-T12 D. L1-L4
818. (C) Blood supply to the spinal cord comes from several sources. The anterior spinal artery is derived from the vertebral arteries and runs the entire length of the spinal cord and supplies the anterior two thirds of the cord. There are segmental arteries from the aorta that join the anterior spinal artery to help supply the spinal cord. One of the larger arteries is called the artery of Adamkiewicz, which arises from the lower thoracic area (T9-T12). Damage to this artery can lead to ischemia for the lower two thirds of the spinal cord and paraplegia. The posterior one third of the cord is supplied by two posterior spinal arteries that also arise from the vertebral arteries and receive some blood supply from the segmental arteries (Barash: Clinical Anesthesia, ed 7, pp 997–998; Miller: Basics of Anesthesia, ed 6, pp 260–261). 242 Part 2 Clinical Sciences
819. Which local anesthetic has the longest elimination half-time (T½)? A. Bupivacaine B. Lidocaine C. Mepivacaine D. Ropivacaine
819. (A) Amino ester local anesthetics undergo hydrolysis in the bloodstream and tend to have short elimination half-times. Amino amides undergo biotransformation by the liver and have longer elimination half-times. The elimination half-time for bupivacaine is 3.5 hours, for levobupivacaine is 3.5 hours, for lidocaine is 1.6 hours, for mepivacaine is 1.0 hour, for procaine is 0.1 hour, and for ropivacaine is 1.9 hours (Hemmings: Pharmacology and Physiology for Anesthesia, ed 1, p 298).
820. Important landmarks for performing a sciatic nerve block (classic approach of Labat) include A. Iliac crest, sacral hiatus, and greater trochanter B. Iliac crest, coccyx, and greater trochanter C. Posterior superior iliac spine, coccyx, and greater trochanter D. Posterior superior iliac spine, greater trochanter, and sacral hiatus
820. (D) To perform a sciatic nerve block, first draw a line from the posterior superior iliac spine to the greater trochanter of the femur, then draw a 5-cm line perpendicular from the midpoint of this line caudally and a second line from the sacral hiatus to the greater trochanter. The intersection of the second line with the perpendicular line marks the point of entry (Hebl: Mayo Clinic Atlas of Regional Anesthesia and Ultrasound-Guided Nerve Blockade, ed 1, pp 405–412; Miller: Miller’s Anesthesia, ed 8, pp 1742–1743).
821. A 76-year-old female patient is undergoing a carotid endarterectomy under a deep cervical plexus nerve block. Which of the following complications would be LEAST likely with this unilateral block? A. Unilateral phrenic nerve paralysis B. Subarachnoid injection C. Blockade of the spinal accessory nerve D. Vertebral artery injection
821. (C) Deep cervical plexus blocks (C2, C3, and C4) can be used for unilateral neck anesthesia for carotid endarterectomy and cervical node dissections. Complications of deep cervical plexus block include injection of the local anesthetic into the vertebral artery, subarachnoid space, or epidural space. Other nerves that may be anesthetized include the phrenic nerve (which is why bilateral deep cervical plexus blocks should be performed with caution, if at all), and the recurrent laryngeal nerve. Some local anesthetic may spread outside the deep cervical fascia and may produce blockade of the sympathetic chain, producing Horner syndrome. Inadvertent blockade of the recurrent laryngeal nerve has also been reported. The spinal accessory nerve is cranial nerve XI and innervates the sternocleidomastoid muscle as well as the trapezius muscle. The accessory nerve comes out cephalad to the injections (Barash: Clinical Anesthesia, ed 7, pp 946–947; Hebl: Mayo Clinic Atlas of Regional Anesthesia and Ultrasound-Guided Nerve Blockade, ed 1, pp 179–185).
822. A retrobulbar block anesthetizes each of the following nerves EXCEPT A. Ciliary nerves B. Cranial nerve III (oculomotor nerve) C. Cranial nerve V (facial nerve) D. Cranial nerve VI (abducens nerve)
822. (C) A retrobulbar block anesthetizes the three cranial nerves responsible for movement of the eye ( cranial nerve III—oculomotor nerve, cranial nerve IV—trochlear nerve, and cranial nerve VI—abducens nerve). The ciliary ganglion (deep within the orbit and lateral to the optic nerve) and ciliary nerves are also blocked, providing anesthesia to the conjunctiva, cornea, and uvea. Branches of the facial nerve (cranial nerve V) are not blocked by the retrobulbar block but are often separately blocked to produce akinesia of the eyelids (Barash: Clinical Anesthesia, ed 7, pp 1383–1386; Brown: Atlas of Regional Anesthesia, ed 3, pp 185–188). Anatomy, Regional Anesthesia, and Pain Management 243
823. Which of the following muscles of the larynx is innervated by the external branch of the superior laryngeal nerve? A. Vocalis muscle B. Thyroarytenoid muscles C. Posterior cricoarytenoid muscle D. Cricothyroid muscle 230 Part 2 Clinical Sciences
823. (D) The vagus nerve innervates the airway by two branches: the superior laryngeal nerves and the recurrent laryngeal nerves. All of the muscles of the larynx are innervated by the recurrent laryngeal nerve except for the cricothyroid muscle. The superior laryngeal nerve divides into the internal and external laryngeal branches. The external laryngeal branch innervates the cricothyroid muscle. The internal laryngeal branch provides sensory fibers to the cords, epiglottis, and arytenoids (Barash: Clinical Anesthesia, ed 7, pp 763–764; Butterworth: Morgan & Mikhail’s Clinical Anesthesiology, ed 5, pp 310–312).
824. All the following agents are acceptable for use in a Bier block EXCEPT A. 0.5% Lidocaine B. 0.5% Mepivacaine C. 0.25% Bupivacaine D. 0.5% Prilocaine
824. (C) Because of the potential for cardiotoxicity and because bupivacaine has no advantages over other local anesthetics in this setting, it is contraindicated for use in intravenous regional anesthesia (Miller: Miller’s Anesthesia, ed 8, p 1736).
825. The stellate ganglion lies in closest proximity to which of the following vascular structures? A. Common carotid artery B. Internal carotid artery C. Vertebral artery D. Aorta
825. (C) The stellate ganglion usually lies in front of the neck of the first rib. The vertebral artery lies anterior to the ganglion, as it has just originated from the subclavian artery. After passing over the ganglion, it enters the vertebral foramen and lies posterior to the anterior tubercle of C6 (Brown: Atlas of Regional Anesthesia, ed 3, pp 199–203; Miller: Miller’s Anesthesia, ed 8, p 1732).
826. Which of the following structures in the antecubital fossa is the MOST medial? A. Brachial artery B. Radial nerve C. Tendon of the biceps D. Median nerve
826. (D) The median nerve is the most medial structure in the antecubital fossa. To block this nerve, first the brachial artery is palpated at the level of the intercondylar line between the medial and lateral epicondyles, and then a needle is inserted just medial to the artery and directed perpendicularly to the skin (Butterworth: Morgan & Mikhail’s Clinical Anesthesiology, ed 5, pp 994–995; Hebl: Mayo Clinic Atlas of Regional Anesthesia and Ultrasound-Guided Nerve Blockade, ed 1, pp 286–288).
827. During placement of an epidural in a 78-year-old patient scheduled for a total knee arthroplasty, the patient complains of a sharp sustained pain radiating down his left leg as the catheter is inserted to 2 cm. The MOST appropriate action at this time would be to A. Leave the catheter at 2 cm, and give a test dose B. Give a small dose to relieve pain, then advance 1 cm C. Withdraw the catheter 1 cm, then give a test dose D. Withdraw the needle and catheter, then reinsert in a new position
827. (D) When an epidural catheter is placed without fluoroscopic guidance, the exact location of the needle tip relative to the anatomic structures of the back can only be surmised. If malposition of either the needle or the catheter is suspected, it is prudent to withdraw the entire apparatus and reinsert a second time. In this case, it is possible that the catheter tip has found its way into a nerve root. Under these circumstances, injection of a local anesthetic or narcotic could produce pressure that could possibly lead to ischemia and neurologic damage. During placement or injection of a needle or epidural catheter, a paresthesia that is sustained is always a warning sign that should be heeded (Barash: Clinical Anesthesia, ed 7, p 910; Butterworth: Morgan & Mikhail’s Clinical Anesthesiology, ed 5, p 949; Raj: Practical Management of Pain, ed 3, p 650). 244 Part 2 Clinical Sciences
828. Cutaneous innervation of the plantar surface of the foot is provided by the A. Sural nerve B. Posterior tibial nerve C. Saphenous nerve D. Deep peroneal nerve
828. (B) There are five nerves that innervate the ankle and foot: the posterior tibial, sural, superficial peroneal, deep peroneal, and saphenous nerves. These nerves are superficial at the level of the ankle and are easy to block. The posterior branch of the tibial nerve gives rise to the medial and lateral plantar nerves, which supply the plantar surface of the foot (Barash: Clinical Anesthesia, ed 7, pp 990– 991; Hebl: Mayo Clinic Atlas of Regional Anesthesia and Ultrasound-Guided Nerve Blockade, ed 1, pp 444–448).
829. Which of the following local anesthetics has the LOWEST ratio of dosage required for cardiovascular collapse to dosage required for central nervous system (CNS) toxicity? A. Lidocaine B. Etidocaine C. Bupivacaine D. Prilocaine
829. (C) In general, in both in vivo and in vitro studies there is an overall direct correlation between anesthetic’s potency and its direct depressant effect on myocardial contractility. The ratio of dosage required for cardiovascular system (CVS) toxicity in animal models compared with CNS toxicity is lowest for bupivacaine, levobupivacaine, and ropivacaine (2.0). Ratios for other local anesthetics are as follows: prilocaine, 3.1; procaine and chloroprocaine, 3.7; etidocaine, 4.4; lidocaine and mepivacaine, 7.1. Remember that this question refers only to the ratio of CVS to CNS toxicity; it does not refer to which drug is more cardiotoxic (Barash: Clinical Anesthesia, ed 7, pp 572–575).
830. A 57-year-old patient is scheduled for hemorrhoidectomy. The patient has a history of mild chronic obstructive pulmonary disease, hypertension, and traumatic foot amputation from a tractor accident. His only hospitalizations were for two suicide attempts related to phantom limb sensations 10 years ago. He takes phenelzine (Nardil), thiazide, and potassium. Which of the following anesthetic techniques would be MOST appropriate for this patient? A. Spinal anesthetic with 0.5% hyperbaric bupivacaine B. Epidural anesthetic with 0.5% bupivacaine C. Local infiltration with lidocaine and epinephrine, sedation with propofol and meperidine D. General anesthesia with propofol, succinylcholine, nitrous oxide, and fentanyl
830. (D) Reactivation of phantom limb sensations has been reported in patients who have received both spinal and epidural anesthetics (90% in some series). In the majority of these cases (80%), phantom limb sensation persisted until the block receded. With a history of phantom limb sensations that drove this patient to attempt suicide, it is probably wise to avoid spinal and epidural anesthetics. Phenelzine (Nardil) is a monoamine oxidase (MAO) inhibitor that is occasionally used for the treatment of depression. Any anesthetic or combination of techniques that involves meperidine is contraindicated in patients receiving MAO inhibitors. The combination of meperidine and MAO inhibitors has been associated with hyperthermia, hypotension, hypertension, ventilatory depression, skeletal muscle rigidity, seizures, and coma. Because of this unfavorable drug interaction, meperidine should be avoided in patients receiving MAO inhibitors. Accordingly, the only acceptable choice in this question would be general anesthesia with propofol, succinylcholine, nitrous oxide, and fentanyl. As an interesting side point, the drug phenelzine prolongs the duration of action of succinylcholine by decreasing plasma cholinesterase activity (Miller: Miller’s Anesthesia, ed 8, p 909; Raj: Practical Management of Pain, ed 3, p 212; Waldman: Pain Management, ed 2, Chapter 32).
831. If the recurrent laryngeal nerve were transected bilaterally, the vocal cords would A. Be in the open position B. Be in the closed position C. Be in the intermediate position (i.e., 2-3 mm apart) D. Not be affected unless the superior laryngeal nerve were also injured
831. (C) The recurrent laryngeal nerve innervates all the muscles of the larynx (e.g., abductors and adductors) except the cricothyroid muscle (which tenses the vocal cords and is innervated by the external branch of the superior laryngeal nerve). With complete bilateral transections of the recurrent laryngeal nerve, both the abductor and adductor muscles are affected, and the vocal cords will adopt an intermediate position (i.e., lie within 2-3 mm of the midline). Acute complete injury to the recurrent laryngeal nerves can result in stridor and respiratory distress requiring treatment (e.g., intubation and possible tracheostomy). If a patient sustained a partial bilateral paralysis of the recurrent laryngeal nerve that affected only the abductor muscles, then the unopposed adductor muscles would bring the cords together (i.e., closed) and complete airway obstruction would ensue (Butterworth: Morgan & Mikhail’s Clinical Anesthesiology, ed 5, pp 310–312; Miller: Miller’s Anesthesia, ed 8, p 2526).
832. A 63-year-old woman undergoes total knee arthroplasty under spinal anesthesia. Two days later she complains of a severe headache. Pain intensity is not related to posture. The LEAST likely cause of this headache is A. Caffeine withdrawal B. Viral illness C. Migraine D. Postdural puncture headache (PDPH)
832. (D) PDPH is due to a loss of CSF through a dural puncture and characteristically has a postural component. When supine, the headache is usually gone but may be mild in some cases. When the head is elevated, the headache may be severe, is bilateral, and may be associated with diplopia, nausea, and vomiting. The headache pain is typically frontal and/or occipital in location. Typically the onset of the headache is 12 to 24 hours after a dural puncture and lasts several days if untreated (rarely it can last for months). The other headaches listed rarely have a significant postural component (Barash: Clinical Anesthesia, ed 7, pp 926–927; Miller: Basics of Anesthesia, ed 6, pp 271–272).
833. What is the CORRECT order of structures (from cephalad to caudad) in the intercostal space? A. Nerve, artery, vein B. Vein, nerve, artery C. Vein, artery, nerve D. Artery, nerve, vein
833. (C) VAN (Vein, Artery, Nerve) describes the anatomic relationship of the intercostal structures deep to the lower border of the ribs from the cephalad to caudal direction. The block is performed by walking off the inferior edge of the rib with the needle, typically about 5 to 7 cm from midline. The two principal risks are pneumothorax and intravascular injection of local anesthetics. Because of the close proximity of the vein and artery to the nerve, intercostal blocks have relatively high blood levels as compared to other blocks (e.g., epidural, brachial plexus, brachial plexus block, infiltration), and caution with dose is needed if many levels are blocked (Butterworth: Morgan & Mikhail’s Clinical Anesthesiology, ed 5, pp 1018–1019). Anatomy, Regional Anesthesia, and Pain Management 245
834. Which of the following types of regional anesthesia is associated with the GREATEST serum concentration of lidocaine? A. Intercostal B. Epidural C. Brachial plexus D. Femoral nerve block
834. (A) The site of injection of the local anesthetic is one of the most important factors influencing systemic local anesthetic absorption and toxicity. The degree of absorption from the site of injection depends on the blood supply to that site. Areas that have the greatest blood supply have the greatest systemic absorption. For this reason, the greatest plasma concentration of local anesthetic occurs after an intercostal block, followed by caudal epidural, lumbar epidural, brachial plexus, sciatic/femoral nerve block, and subcutaneous (Barash: Clinical Anesthesia, ed 7, pp 569–570; Miller: Miller’s Anesthesia, ed 8, p 1046).
835. Differences in which of the following local anesthetic properties account for the fact that the onset of an epidural block with 3% 2-chloroprocaine is more rapid than 2% lidocaine? A. Protein binding B. pKa C. Lipid solubility D. Concentration Anatomy, Regional Anesthesia, and Pain Management 231
835. (D) Local anesthetics are weak bases. The neutral (non-ionized) form of the molecule is able to pass through the lipid nerve cell membrane, whereas the ionized (protonated) form actually produces anesthesia. Chloroprocaine has the highest pKa of local anesthetics, meaning that a greater percentage of it will exist in the ionized form at any given pH than any of the other local anesthetics. Despite this fact, 3% chloroprocaine has a more rapid onset than 2% lidocaine, presumably because of the greater number of molecules (concentration). However, if one compares onset time for 1.5% lidocaine against 1.5% chloroprocaine, the former will have a more rapid onset (Miller: Miller’s Anesthesia, ed 8, p 1039).
836. A 69-year-old man with a history of diabetes mellitus and chronic renal failure is to undergo placement of a dialysis fistula under regional anesthesia. During needle manipulation for a supraclavicular brachial plexus block, the patient begins to cough and complain of chest pain and shortness of breath. The MOST likely diagnosis is A. Angina B. Pneumothorax C. Phrenic nerve irritation D. Intravascular injection of local anesthetic
836. (B) The risk of pneumothorax is a significant limitation for supraclavicular brachial plexus blocks (traditionally the incidence is 0.5% to 6% depending upon experience; with the ultrasound technique, the incidence may be lower). Furthermore, the technique is difficult to teach and describe. For these reasons, this block should not be performed in patients in whom a pneumothorax or phrenic nerve block (30%-60% of patients) would result in significant dyspnea or respiratory distress. A pneumothorax should be considered if the patient begins to complain of chest pain or shortness of breath or begins to cough during placement of supraclavicular brachial plexus block. In some cases, symptoms of a pneumothorax may be delayed up to 24 hours (Barash: Clinical Anesthesia, ed 7, pp 961–962; Hebl: Mayo Clinic Atlas of Regional Anesthesia and Ultrasound-Guided Nerve Blockade, ed 1, pp 225–231; Miller: Miller’s Anesthesia, ed 8, pp 1727–1728).
837. Each of the following statements is true concerning a femoral nerve block EXCEPT A. The femoral nerve primarily arises from the second to the fourth lumbar nerve roots B. The femoral nerve provides sensation to the anterior and medial aspect of the thigh C. The femoral nerve lies lateral to the femoral artery and femoral vein D. Proper needle placement produces sartorius muscle contraction without patellar movement when electrically stimulated
837. (D) The femoral nerve is the largest branch of the lumbar plexus (it primarily arises from the second to fourth lumbar nerve roots). The femoral nerve divides into an anterior and a posterior division. The anterior division provides motor innervation to the sartorius muscle and cutaneous sensation to the anterior and medial aspects of the thigh. The posterior division innervates the quadriceps muscle and cutaneous sensation to the anterior, medial, and lateral aspects of the knee as well as the articular aspects of the knee joint. The nerve passes under the inguinal ligament and lies just lateral to the femoral artery and vein. If the stimulating needle produces sartorius muscle contraction without patellar movement, then you are too anterior for proper femoral nerve blockade, and the needle needs to be advanced in a more posterior (i.e., deeper) direction. Proper needle placement will elicit quadriceps muscle contraction with patellar elevation that disappears with local anesthetic injection (Hebl: Mayo Clinic Atlas of Regional Anesthesia and Ultrasound-Guided Nerve Blockade, ed 1, pp 347–362).
838. If a needle is introduced 1.5 cm inferior and 1.5 cm lateral to the pubic tubercle, to which nerve will it lie in close proximity? A. Obturator nerve B. Femoral nerve C. Lateral femoral cutaneous nerve D. Ilioinguinal nerve
838. (A) The obturator nerve provides variable cutaneous innervation of the thigh and can be used to supplement femoral and sciatic nerve blockade for patients having lower extremity surgery. An obturator nerve block is achieved by placement of the needle 1 to 2 cm lateral to and 1 to 2 cm below the pubic tubercle. After contact with the pubic bone, the needle is withdrawn and walked cephalad to identify the obturator canal. Between 10 and 15 mL of local anesthetic should be placed in the canal. If a nerve stimulator is used, contraction of the adductor muscles with nerve stimulation indicates proximity to the nerve (Barash: Clinical Anesthesia, ed 7, pp 982–983; Hebl: Mayo Clinic Atlas of Regional Anesthesia and Ultrasound-Guided Nerve Blockade, ed 1, pp 386–394; Miller: Miller’s Anesthesia, ed 8, pp 1741–1742).
839. The MOST common complication associated with a supraclavicular brachial plexus block is A. Blockade of the phrenic nerve B. Intravascular injection into the vertebral artery C. Blockade of the recurrent laryngeal nerve D. Pneumothorax
839. (A) The most serious complication associated with a supraclavicular brachial plexus block is pneumothorax, which fortunately is rare (0.5%-5%). The most common complication is a phrenic nerve block, which is usually mild and relatively common (30%-60% of blocks). Bilateral supraclavicular blocks, however, are not recommended due to the possibility of bilateral phrenic nerve paralysis or pneumothoraces. Other potential complications include Horner syndrome (ipsilateral eye ptosis, miosis, and anhidrosis), nerve damage or neuritis, infection, or intravascular injection (Barash: Clinical Anesthesia, ed 7, pp 962; Hebl: Mayo Clinic Atlas of Regional Anesthesia and Ultrasound-Guided Nerve Blockade, ed 1, p 231; Miller: Miller’s Anesthesia, ed 8, pp 1727–1728). 246 Part 2 Clinical Sciences
840. Which portion of the upper extremity is NOT innervated by the brachial plexus? A. Posterior medial portion of the arm B. Elbow C. Lateral portion of the forearm D. Medial portion of the forearm
840. (A) The arm receives sensory innervation from the brachial plexus except for the shoulder, which is innervated by the supraclavicular nerves from the cervical plexus, and the posterior medial aspect of the arm, which is supplied by the intercostobrachial nerve (Hebl: Mayo Clinic Atlas of Regional Anesthesia and Ultrasound-Guided Nerve Blockade, ed 1, pp 58–70; Miller: Basics of Anesthesia, ed 6, pp 287–292).
841. Which section of the brachial plexus is blocked with a supraclavicular block? A. Roots/trunks B. Trunks/divisions C. Cords D. Branches
841. (B) The brachial plexus starts out at the root level from the ventral rami of C5-T1 with a small amount from C4 and T2. These roots at the level of the scalene muscle become the three trunks: superior, middle, and inferior. The trunks then divide into the dorsal and ventral divisions at the lateral edge of the first rib. When the divisions enter the axilla, they become the cords: posterior, lateral, and medial. At the lateral border of the pectoralis muscle, they become the five peripheral nerves: radial, musculocutaneous, median, ulnar, and axillary. The interscalene block is at the level of the roots/trunks (but spares the inferior trunk); the supraclavicular block is at the level of the trunks/divisions; the infraclavicular block is at the level of the cords; and the axillary block is at the level of the branches (Barash: Clinical Anesthesia, ed 7, pp 959–966; Hebl: Mayo Clinic Atlas of Regional Anesthesia and Ultrasound-Guided Nerve Blockade, ed 1, pp 225–226; Miller: Basics of Anesthesia, ed 6, pp 287–292).
842. A celiac plexus block would NOT effectively treat pain resulting from a malignancy involving which of the following organs? A. Uterus B. Stomach C. Pancreas D. Gallbladder
842. (A) The celiac plexus innervates most of the abdominal viscera, including the lower esophagus, stomach, all of the small intestine, and the large intestine up to the splenic flexure as well as the pancreas, liver, biliary tract, spleen, kidneys, adrenal glands, and omentum. The pelvic organs (e.g., uterus, ovaries, prostate, distal colon) are supplied by the hypogastric plexus (Barash: Clinical Anesthesia, ed 7, pp 1658–1659; Butterworth: Morgan & Mikhail’s Clinical Anesthesiology, ed 5, pp 1073–1075).
843. A healthy 27-year-old woman stepped on a nail and is to undergo débridement of a wound on her right great toe. She is anxious about general anesthesia but agrees to an ankle block with mild sedation. Which nerves must be adequately blocked in order to perform the surgery? A. Deep peroneal, posterior tibial, saphenous, sural B. Deep peroneal, saphenous, superficial peroneal, sural C. Deep peroneal, posterior tibial, superficial peroneal, sural D. Deep peroneal, superficial peroneal, posterior tibial, saphenous
843. (D) The great toe is innervated mainly by the deep peroneal, posterior tibial, superficial peroneal, and occasionally by the saphenous nerve. All four of these nerves should be blocked for surgery on the great toe. The sural nerve is the fifth nerve for ankle blocks but covers only the lateral side of the foot, and not the medial side or great toe area (Butterworth: Morgan & Mikhail’s Clinical Anesthesiology, ed 5, pp 1015–1017; Hebl: Mayo Clinic Atlas of Regional Anesthesia and Ultrasound-Guided Nerve Blockade, ed 1, pp 443–452).
844. A 54-year-old man is administered morphine via patient-controlled analgesia (PCA) pump after a left total hip arthroplasty. The pump is programmed to deliver a maximum dose of 2 mg every 15 minutes (lockout time) as needed for patient comfort. The total maximum dose that can be delivered in 4 hours is 30 mg. On the first day the patient receives 15 doses every 4 hours by pressing the delivery button every 15 to 18 minutes. How should his pain control be further managed? A. Discontinue the PCA pump and administer intramuscular morphine B. Increase the lockout time from 15 to 25 minutes C. Change the analgesic from morphine to meperidine D. Increase the dose to 3 mg every 15 minutes as needed up to a total maximum dose of 40 mg every 4 hours
844. (D) Frequent dosing by a patient receiving postoperative analgesia through a PCA pump suggests the need to increase the magnitude of the dose. It is important to keep in mind that a patient should be given a sufficient loading dose of narcotic before initiative therapy with a PCA pump. Otherwise, the patient will be playing the frustrating game of “catch up.” The most commonly used narcotics in the United States for PCA pump use are morphine, fentanyl, and hydromorphone. Meperidine should not be used as the narcotic for PCA pumps, since the toxic metabolite normeperidine may accumulate (Barash: Clinical Anesthesia, ed 7, pp 1626–1627).
845. The mechanism of low-frequency transcutaneous electrical nerve stimulation (TENS) units in relieving pain is A. Direct electrical inhibition of type A-δ and C fibers B. Depletion of neurotransmitter in nociceptors C. Hyperpolarization of spinothalamic tract neurons D. Activation of inhibitory neurons
845. (D) TENS produces a tingling or vibratory sensation in the area in which pads are placed. Although the exact mechanism is unclear, it is thought that TENS units produce analgesia by releasing endogenous endorphins, since its effects are partially blocked by naloxone. These endorphins have an inhibitory effect at the spinal cord level and augment descending inhibitory pathways (Butterworth: Morgan & Mikhail’s Clinical Anesthesiology, ed 5, p 1081; Miller: Miller’s Anesthesia, ed 8, pp 2339, 2991).
846. Epidural use of which of the following opioids would result in the GREATEST incidence of delayed respiratory depression? A. Sufentanil B. Fentanyl C. Morphine sulfate D. Hydromorphone 232 Part 2 Clinical Sciences
846. (C) Although the more hydrophilic drugs such as morphine have a longer duration of action of analgesia, they also have a higher potential for inducing delayed respiratory depression through cephalad migration in the CNS, as compared with the more lipid-soluble drugs listed in this question (Barash: Clinical Anesthesia, ed 7, pp 1627–1629; Miller: Miller’s Anesthesia, ed 8, p 2983).
847. A 21-year-old patient reports tingling in her thumb during her cesarean section under epidural anesthesia. To which dermatomal level would this correspond? A. C5 B. C6 C. C7 D. C8
847. (B) The thumb corresponds to dermatome C6, the second and middle fingers correspond to dermatome C7, and the fourth and little fingers correspond to dermatome C8 (Hebl: Mayo Clinic Atlas of Regional Anesthesia and Ultrasound-Guided Nerve Blockade, ed 1, p 86; Miller: Basics of Anesthesia, ed 6, pp 258–260).
848. Which of the following would hasten the onset and increase the clinical duration of action of a local anesthetic, and provide the GREATEST depth of motor and sensory blockade when used for epidural anesthesia? A. Increasing the volume of local anesthetic B. Increasing the concentration of local anesthetic C. Increasing the dose D. Placing the patient in the head-down position
848. (C) Increasing the total dose (mass) of local anesthetic is more efficacious in hastening the onset and increasing the duration of an epidural anesthetic than increasing the volume or increasing the concentration (while holding the total dose constant) (Barash: Clinical Anesthesia, ed 7, p 921). Anatomy, Regional Anesthesia, and Pain Management 247
849. Select the FALSE statement concerning neurolytic nerve blocks. A. Destruction of peripheral nerves can be followed by a denervation hypersensitivity that is worse than the original pain B. Neurolytic blocks should be reserved for patients with short life expectancies C. Neurolytic blockade with phenol is permanent D. Intrathecal neurolysis may be an effective management for certain pain conditions
849. (C) Alcohol and phenol are similar in their ability to cause nonselective damage to neural tissues. Alcohol causes pain when injected and sometimes is mixed with bupivacaine, whereas phenol is relatively painless. Alcohol has a slightly longer duration of analgesia (3-6 months) compared to phenol (2-3 months). Neural tissue will regenerate; therefore, neurolytic blocks are never “permanent,” and neurolysis can lead to denervation hypersensitivity, which can be extremely painful (Butterworth: Morgan & Mikhail’s Clinical Anesthesiology, ed 5, pp 1079–1080; Miller: Miller’s Anesthesia, ed 8, p 1911).
850. Transient neurologic symptoms (TNS) after spinal anesthesia is associated with each of the following EXCEPT A. Lidocaine B. Lithotomy position C. Ambulatory anesthesia D. Concentration of local anesthetic injected
850. (D) TNS, previously called transient radicular irritation (TRI), can occur in 4% to 40% of patients after spinal anesthesia with lidocaine in ambulatory patients undergoing surgery in the lithotomy position or knee arthroscopy. The baricity, concentration injected (lidocaine 0.5%-5%), addition of epinephrine, presence of dextrose, or hypotension does not seem to be related to the development of TNS. The symptoms of TNS include pain or sensory abnormalities in the lower back, buttocks, or lower extremities. Although TNS has been reported with all local anesthetics, the incidence is significantly greater with lidocaine (Barash: Clinical Anesthesia, ed 7, pp 576, 928; Miller: Miller’s Anesthesia, ed 8, p 1692).
851. After you select the appropriate ultrasound transducer, you can adjust several factors to optimize the image for regional anesthesia. Which of the following descriptions is FALSE? A. Frequency—higher frequency ultrasound use is better for viewing deep structures B. Depth—adjusted to limit the centimeters of viewing area on the monitor C. Gain—increased gain produces increased brightness D. Frequency—higher frequency ultrasound use produces better image resolution
851. (A) After the proper transducer is selected, you can adjust the frequency, depth, and gain to optimize an image. In general, higher-frequency ultrasound waves provide better image quality (i.e., better resolution) due to the higher number of cycles per second of transmitted and reflected energy used to produce the image. However, higher frequency waves have more signal attenuation at increasing depths and cannot penetrate to deeper tissue levels. Therefore, higher-frequency ultrasound is typically used for shallower structures, and lower frequencies are used for deeper structures. Usually the depth is adjusted so the structure in question is in the center, top-to-bottom, of the image. Increasing the gain increases, or amplifies, the reflected signal energy and increases the brightness of the image (Hebl: Mayo Clinic Atlas of Regional Anesthesia and Ultrasound-Guided Nerve Blockade, ed 1, pp 99–112).
852. Each of the following is associated with an increased incidence of PDPHs EXCEPT A. Younger adults B. Early ambulation C. Pregnancy D. Large needle size
852. (B) Younger adults have a higher incidence of PDPH than older adults or children. Women have a slightly higher incidence than men. Pregnant women have a higher incidence than nonpregnant women. Since the incidence and severity of PDPH relates to the amount of CSF leakage through the dural hole, it makes sense that the larger the needle and the more holes in the dura, the greater incidence of PDPH. In addition, the shape of the tip of the needle is important; a cutting needle (e.g., Quincke) has a greater incidence of PDPH than noncutting needles (e.g., Whitacre, Sprotte). The incidence of headache has been shown to be less when the dural fibers are split longitudinally rather than when they are cut while the needle is held in a transverse direction. The timing of ambulation relative to dural puncture has not been shown to affect the incidence of postspinal headache. The block should wear off before ambulation is attempted (Barash: Clinical Anesthesia, ed 7, pp 926–927; Miller: Miller’s Anesthesia, ed 8, pp 1694–1695).
853. Each of the following items describes pain in the abdominal viscera EXCEPT A. Pain is transmitted via the vagus nerve B. The nerve fibers are type C C. Pain is characterized by a dull aching or burning sensation D. Distention of the transverse colon causes more pain than surgical transection
853. (A) Virtually all pain arising in the thoracic or abdominal viscera is transmitted via the sympathetic nervous system in unmyelinated type C fibers. Visceral pain is dull, aching, burning, and nonspecific. Visceral pain is caused by any stimulus that excites nociceptive nerve endings in diffuse areas. In this regard, distention of a hollow viscus causes a greater sensation of pain than does the highly localized damage produced by transecting the gut. Although the vagus nerve has a large amount of afferent fibers, they do not include pain fibers (Brunton: Goodman & Gilman’s The Pharmacological Basis of Therapeutics, ed 12, pp 174–175, 567–570; Raj: Practical Management of Pain, ed 3, pp 223–225).
854. Which of the following blocks has the LONGEST duration of action when bupivacaine with epinephrine is administered? A. Axillary B. Epidural C. Infiltration D. Spinal
854. (A) The duration of regional blocks is different between local anesthetics as well as with different location of blocks. When bupivacaine with epinephrine (1:200,000) is used, epidural anesthesia may last 180 to 350 minutes; infiltration anesthesia may last 180 to 240 minutes; and major nerve blocks such as axillary block may last 360 to 720 minutes. Spinal bupivacaine without epinephrine may last 90 to 200 minutes; if epinephrine (0.2-0.3 mg) is added to the spinal block, it will last about 50% longer (Miller: Miller’s Anesthesia, ed 8, pp 1041–1044).
855. All of the following statements concerning a psoas compartment block are true EXCEPT A. Compartmental block is used to provide unilateral anesthesia to the proximal aspect of the thigh and hip B. Stimulation of the quadriceps muscle demonstrates good needle placement C. Complete leg anesthesia can be obtained when combined with a sciatic nerve block D. Continuous catheters are not used because the amount of drug infused would lead to toxicity
855. (D) Psoas compartment block is also called the posterior lumbar plexus block and can be used for any procedure in which a lumbar plexus block is required, but most often it is used for analgesia for the proximal aspect of the thigh and hip. When combined with a sciatic block, complete leg anesthesia will result. Remembering that the femoral nerve (which innervates the quadriceps muscles) is a distal branch helps one to understand why quadriceps muscle contraction is useful in locating the plexus with a stimulating needle (1-1.5 mA). If the hamstring muscles are stimulated, the needle is too caudally located, and the 248 Part 2 Clinical Sciences needle should be aimed in a more cephalad direction. Continuous psoas catheters are commonly used for postoperative analgesia (Barash: Clinical Anesthesia, ed 7, pp 978–980; Hebl: Mayo Clinic Atlas of Regional Anesthesia and Ultrasound-Guided Nerve Blockade, ed 1, pp 333–345).
856. A 35-year-old woman receives a popliteal block for ankle and foot surgery. Which other nerve must be blocked in order to have complete anesthesia of the foot? A. Superficial peroneal nerve B. Sural nerve C. Saphenous nerve D. Posterior tibial nerve
856. (C) All of the nerves of the foot (with the exception of the saphenous) are derived from the sciatic nerve. The sciatic nerve distally becomes the tibial and peroneal nerves, which can be blocked at the popliteal fossa for surgery below the knee. The saphenous nerve is a branch of the femoral nerve and provides sensory innervation along the medial aspect of the lower leg between the knee and the medial malleolus, and must also be blocked for surgery below the knee (Hebl: Mayo Clinic Atlas of Regional Anesthesia and Ultrasound-Guided Nerve Blockade, ed 1, pp 423–426; Butterworth: Morgan & Mikhail’s Clinical Anesthesiology, ed 5, pp 1013–1015).
857. The most common complication of a celiac plexus block is A. Hypotension B. Seizure C. Retroperitoneal hematoma D. Constipation
857. (A) The sympathectomy produced by a celiac plexus block causes hypotension by decreasing preload to the heart. This complication can be avoided by volume loading the patient with lactated Ringer solution. By blocking the sympathetic chain, unopposed parasympathetic activity may also result in increased gastrointestinal activity and transient diarrhea. Back pain is also common. Paraplegia may result from spasm of the lumbar segmental arteries that perfuse the spinal cord, direct vascular or neurologic injury, or retrograde spread of drug to the nerve roots and spinal cord. Seizure is possible with an intravascular injection. Retroperitoneal hematoma is also possible, but rare (Barash: Clinical Anesthesia, ed 7, pp 1658–1659; Butterworth: Morgan & Mikhail’s Clinical Anesthesiology, ed 5, pp 1073–1074).
858. The occipital portion of the skull receives sensory innervation from A. Spinal accessory nerve (nerve XI) B. Facial nerve (nerve VII) C. Ophthalmic branch of trigeminal nerve (nerve V) D. Cervical plexus
858. (D) The occiput receives sensory innervation from the greater and lesser occipital nerves (C2 and C3 spinal roots), which are terminal branches of the cervical plexus. Blockade of these nerves is usually carried out as a diagnostic step in the evaluation of head and neck pain (Barash: Clinical Anesthesia, ed 7, pp 946–947, 958–959; Butterworth: Morgan & Mikhail’s Clinical Anesthesiology, ed 5, p 1065).
859. Each of the following is a potential complication of thoracic paravertebral blocks EXCEPT A. Pneumothorax B. Epidural spread of local anesthetic C. Hypertension D. Total spinal Anatomy, Regional Anesthesia, and Pain Management 233
859. (C) Thoracic paravertebral blocks are used for surgical anesthesia and postoperative analgesia for breast, axillary, or chest wall surgery. The major complication is a pneumothorax. Since the paravertebral space is continuous with the epidural space medially, epidural spread may result if large volumes of local anesthetic are injected into the paravertebral. Typically 5 mL are injected at each of three sites for unilateral paravertebral blocks, and 3 mL per each of six sites (three on each side) if bilateral paravertebral blocks are performed. If the needle is directed too medially, then the intrathecal space may be entered (dural sleeves extend to the level of the intervertebral foramina) with the possibility of a total spinal if 5 to 10 mL is injected. The sympathetic chain is in the anterior part of the paravertebral space, and sympathetic blockade may develop; however, hypotension would be more likely than hypertension to develop from blocking the sympathetic chain (Barash: Clinical Anesthesia, ed 7, pp 972–975; Butterworth: Morgan & Mikhail’s Clinical Anesthesiology, ed 5, pp 1019, 1067–1068; Hebl: Mayo Clinic Atlas of Regional Anesthesia and Ultrasound Guided Peripheral Nerve Blockade, ed 1, pp 323–329).
860. After placement of an epidural catheter in a 55-yearold patient for total hip arthroplasty, an entire epidural dose is administered into the subarachnoid space. Physiologic effects consistent with subarachnoid injection of large volumes of local anesthetic include all of the following EXCEPT A. Hypotension and bradycardia B. Respiratory depression C. Constricted pupils D. Possible cauda equina syndrome
860. (C) With the unintentional injection of an epidural dose of local anesthetic into the subarachnoid space, spinal anesthesia develops rapidly. Blockade of the sympathetic fibers (T1-L2) produces hypotension, particularly if the patient is hypovolemic. Bradycardia is produced by blocking the cardiac accelerator fibers (T1-T4). Respiratory arrest is due to hypoperfusion of the respiratory centers as well as paralysis of the phrenic nerve (C3-C5). The pupils become dilated (mydriasis) after intrathecal injection of large quantities of local anesthetics; they will return to normal size after the block recedes. Cauda equina syndrome has occasionally developed when the epidural dose was unintentionally administered into the subarachnoid space (most commonly with chloroprocaine). If one suspects an unintentional placement of the epidural dose subarachnoid, supportive methods are initially used (the basic ABC’s of resuscitation). One can also aspirate CSF from the epidural catheter (if it was inserted) to help remove some of the drug as well as reducing the pressure in the subarachnoid space, which might help better perfuse the spinal cord and decrease the chance of cauda equina syndrome developing (Barash: Clinical Anesthesia, ed 7, pp 927–928; Miller: Miller’s Anesthesia, ed 8, pp 1690, 1702; Southorn: Reducing the potential morbidity of an unintentional spinal anaesthetic by aspirating cerebrospinal fluid, Br J Anaesth 76:467–469, 1996).
861. A 49-year-old type 1 diabetic patient with a long history of burning pain in the right lower extremity receives a spinal anesthetic with 100 mg of procaine with 5% dextrose. The patient reports no relief in symptoms but has complete bilateral motor blockade. What diagnosis is consistent with this differential blockade examination? A. Diabetic neuropathy B. Central pain C. Myofascial pain D. Complex regional pain syndrome I (RSD)
861. (B) Somatic pain in the extremities is relieved with spinal anesthesia. If a patient fails to obtain pain relief despite complete sympathetic, sensory, and motor blockade, a “central” mechanism for the pain is likely or the lesion causing the pain is higher in the CNS than the level of blockade achieved by the spinal. Anatomy, Regional Anesthesia, and Pain Management 249 Central pain states may include encephalization, psychogenic pain, or malingering. Persistence of pain in the lower extremities after successful spinal blockade suggests a central source or psychological source of pain (Miller: Miller’s Anesthesia, ed 8, pp 1898–1910; McMahon: Wall and Melzack’s Textbook of Pain, ed 6, Chapter 69).
862. An 18-year-old man has a seizure during placement of an interscalene brachial plexus block with 0.5% bupivacaine. The anesthesiologist begins to hyperventilate the patient’s lungs with 100% O2 using an anesthesia bag and mask. The rationale for this therapy includes all of the following EXCEPT A. The therapy helps to prevent and treat hypoxia B. Hyperventilation decreases blood flow and delivery of local anesthetic to the brain C. Hyperventilation elevates the seizure threshold D. Hyperventilation induces alkalosis and converts local anesthetics to the protonated (ionized) form, which is less likely to cross the cell membranes
862. (D) During a seizure, both arterial hypoxemia and acidosis (metabolic and respiratory) develop due to the increased oxygen consumption from contracting muscles and hypoventilation that occurs. Administration of 100% O2 helps to prevent and treat hypoxemia. Elevated CO2 not only enhances cerebral blood flow and delivery of local anesthetic to the brain but also diffuses into neural tissue, causing intracellular pH to fall. Because local anesthetics are either amino esters or amino amides, lowering the pH allows more binding of hydrogen ions to the amino group, making it more ionic or protonated, which traps the local anesthetic inside the cells. Hyperventilation can reverse many of the changes that occur with acidosis (i.e., causes cerebral vasoconstriction and can decrease delivery of local anesthetic to the brain). Hyperventilation induces hypokalemia and respiratory alkalosis, both of which result in hyperpolarization of nerve membranes and elevation of the seizure threshold. Hyperventilation also raises the patient’s pH (respiratory alkalosis) and converts local anesthetics into the non-ionized (nonprotonated) form, which crosses the membrane more easily than the ionized form, which is detrimental. Benzodiazepines and/or propofol are used to suppress the seizure activity (Barash: Clinical Anesthesia, ed 7, p 575; Miller: Miller’s Anesthesia, ed 8, pp 1048–1050).
863. Para-aminobenzoic acid is a metabolite of A. Mepivacaine B. Ropivacaine C. Bupivacaine D. Procaine
863. (D) Para-aminobenzoic acid is a metabolite of the ester-type local anesthetics. Local anesthetics may be placed into two distinct categories based on their chemical structure: amino esters or amino amides. The amides (two i’s in the name), which are ropivacaine, lidocaine, etidocaine, prilocaine, mepivacaine, and bupivacaine, are metabolized in the liver. The ester local anesthetics (one i in the name) are cocaine, procaine, chloroprocaine, tetracaine, and benzocaine. These drugs are metabolized by the enzyme pseudocholinesterase found in the blood. Para-aminobenzoic acid is a metabolic breakdown product of ester anesthetic and is responsible for allergic reactions in some individuals (Butterworth: Morgan & Mikhail’s Clinical Anesthesiology, ed 5, pp 270–271; Hemmings: Pharmacology and Physiology for Anesthesia, ed 1, pp 298–303).
864. Which statement concerning peripheral nerve structure and function is FALSE? A. Both nonmyelinated and myelinated nerves are surrounded by Schwann cells B. The speed of propagation of an action potential along a nerve axon is greatly enhanced by myelin C. Generation of an action potential is an “all-ornothing” phenomenon D. Myelination renders nerves less sensitive to local anesthetic blockade
864. (D) Peripheral nerve axons are always enveloped by a Schwann cell. The myelinated nerves may be enveloped many times by the same Schwann cell. Transmission of nerve impulses (i.e., action potentials) along nonmyelinated nerves occurs in a continuous fashion, whereas transmission along myelinated nerves occurs by saltatory conduction from one node of Ranvier to the next. Myelination speeds transmission of neurologic impulses; it also renders nerves more susceptible to local anesthetic blockade. An action potential is associated with an inward flux of sodium that occurs after a certain membrane threshold has been exceeded (Miller: Miller’s Anesthesia, ed 8, pp 1031–1035).
865. A 42-year-old woman with a morbid fear of general anesthesia receives an interscalene block for shoulder arthroscopy consisting of 20 mL 0.5% ropivacaine. Much of her arm, shoulder, and hand are numb, but the patient complains of pain as the incision is made at the upper portion of the shoulder. The most appropriate next step is to A. Repeat block B. Perform intercostobrachial block C. Perform superficial cervical plexus block D. Perform a deep cervical plexus block
865. (C) The needle insertion site for an interscalene block is C6 (i.e., lateral to the cricoid cartilage). Local anesthetics usually spread to C5, C6, and C7, which supply much, but not all, of the cutaneous innervation to the shoulder. With low-to-moderate volume blocks, there will be sparing of the C3-C4 nerve roots, which supply some of the innervation to the anterior shoulder. Of note, C8 and T1 may also be spared, often resulting in the need for ulnar nerve supplementation if this block were used for a hand operation. Complete anesthesia for shoulder arthroscopy may require a supplemental superficial cervical plexus with use of low-to-moderate volumes of a local anesthetic (Hebl: Mayo Clinic Atlas of Regional Anesthesia and Ultrasound Guided Peripheral Nerve Blockade, ed 1, pp 185–193).
866. According to the 2004 American Society of Regional Anesthesia and Pain Medicine (ASRA) practice advisory on infectious complications of regional anesthesia and pain medicine, the MOST important action to maintain aseptic technique and prevent cross-contamination during regional anesthesia techniques is A. Wearing a surgical gown B. Hand washing C. Using soap and water instead of alcohol-based antiseptics D. Using povidone-iodine (e.g., Betadine) instead of alcohol-based chlorhexidine to scrub
866. (B) Hand washing is one of the most important techniques to prevent infections, especially when alcoholbased antiseptic solutions are used with sterile gloves. Although soap and water remove bacteria, they do not effectively kill organisms. Antiseptic solutions with alcohol appear to be better than nonalcoholic antiseptics (e.g., povidone-iodine). Nail length does not appear to be a risk factor for infections, because the majority of bacterial growth occurs along the proximal 1 mm of nail adjacent to the subungual skin. Universal use of gowns and gloves does not appear to be better than gloves alone in preventing infections in intensive care units (ICUs) and presumably is less important than adequate hand washing and use of sterile gloves (Hebl: Infectious complications: a new practice advisory, Reg Anesth Pain Med 31:289–290, 2006; Hebl: The importance and implications of aseptic techniques during regional anesthesia, Reg Anesth Pain Med 31:311–323, 2006). 250 Part 2 Clinical Sciences
867. A 75-year-old woman with a history of pulmonary embolism is scheduled for a right lower lobectomy for lung cancer. She is receiving dalteparin (Fragmin) for deep vein thrombosis (DVT) prophylaxis. How long after her last dose should one wait prior to placement of a thoracic epidural? A. 12 hours B. 24 hours C. 72 hours D. No waiting is necessary since the dose for prophylaxis is low
867. (A) In patients taking low-molecular-weight heparin (LMWH) (e.g., enoxaparin, dalteparin, tinzaparin), caution should be exercised before proceeding with an epidural or spinal anesthetic because of the risk of producing an epidural or spinal hematoma. The amount of time between the last dose of the LMWH and the relative safety of starting a central neuraxial block depends on the dose of the LMWH. At the lower doses, used for thromboprophylaxis, the LMWH should be held at least 10 to 12 hours prior to the block. At the higher doses, used to treat an established DVT, one should wait at least 24 hours after the last dose of LMWH prior to the block (Barash: Clinical Anesthesia, ed 7, p 929; Miller: Miller’s Anesthesia, ed 8, pp 1702, 2344–2345; Horlocker: Regional anesthesia in the patient receiving antithrombotic or thrombolytic therapy: American Society of Regional Anesthesia and Pain Medicine Evidence-Based Guidelines (Third Edition), Reg Anesth Pain Med 35:64–101, 2010).
868. How long should a patient be off clopidogrel (Plavix) before a central neuraxial block is performed? A. 24 hours B. 7 days C. 14 days D. No waiting necessary
868. (B) Taking nonsteroidal anti-inflammatory drugs (NSAIDs), ticlopidine, and clopidogrel exert effects on platelet function. NSAIDs are not a problem if given alone before epidural or spinal anesthesia; however, patients taking ticlopidine should wait 14 days and patients taking clopidogrel should wait 7 days before having a neuraxial block placed, because of the increased risk of spinal hematoma formation. Keep in mind that caution is always needed and that the ASRA statement “Careful preoperative assessment of the patient to identify alterations of health that might contribute to bleeding is crucial” is important (Barash: Clinical Anesthesia, ed 7, p 929; Horlocker: Regional anesthesia in the patient receiving antithrombotic or thrombolytic therapy: American Society of Regional Anesthesia and Pain Medicine Evidence-Based Guidelines (Third Edition), Reg Anesth Pain Med 35:64–101, 2010).
869. Addition of bicarbonate to local anesthetics results in A. Delayed onset of action B. Reduced toxicity C. Increased duration of action D. Reduced pain with skin infiltration
869. (D) Adding sodium bicarbonate to local anesthetic solutions hastens the onset of action of the local anesthetics, especially when the local anesthetic solution contains epinephrine (which is produced at a lower pH). By raising the pH, more of the local anesthetic is in the non-ionized, more lipid-soluble state. Raising the pH too much (i.e., >6.05-8) would cause precipitation of the local anesthetic. Some studies have shown that alkalization of the local anesthetic may decrease the duration of a peripheral block, especially if epinephrine was not added. It also seems to decrease pain with skin infiltration. Pain on injection can also be decreased by a slow injection of the local anesthetic (Barash: Clinical Anesthesia, ed 7, pp 567–568; Miller: Miller’s Anesthesia, ed 8, p 1040).
870. Through which of the following would a spinal needle NOT pass during a midline placement of a subarachnoid block in the L3-L4 lumbar space? A. Supraspinous ligament B. Interspinous ligament C. Posterior longitudinal ligament D. Dura mater 234 Part 2 Clinical Sciences
870. (C) This figure shows the anatomic structures that must be traversed by the spinal needle during the performance of a subarachnoid block. The structures include the skin, subcutaneous tissue, supraspinous ligament, interspinous ligament, ligamentum flavum, the epidural space, and finally the dura (posteriorly). If you were to continue to advance the spinal needle, you would encounter the dura (anteriorly) while exiting the subarachnoid space, the posterior longitudinal ligament, the periosteum of the vertebral body, and finally bone (Cousins: Neural Blockade in Clinical Anesthesia and Management of Pain, ed 3, p 205). Anatomy, Regional Anesthesia, and Pain Management 251
871. What epidural dose of bupivacaine will give sensory analgesia similar to 10 mL of 2% lidocaine? A. 5 mL of 0.25% B. 10 mL of 0.25% C. 5 mL of 0.5% D. 10 mL of 0.5%
871. (D) In the epidural space, bupivacaine (as well as levobupivacaine) is four times more potent than lidocaine, so 0.5% bupivacaine is similar to 2% lidocaine for analgesia. The duration of the bupivacaine block will be longer because bupivacaine has a long duration of action and lidocaine has an intermediate duration of action. In addition, motor block would be less for bupivacaine compared with lidocaine, since there is more of a greater difference between sensory and motor block for bupivacaine as compared with lidocaine (Barash: Clinical Anesthesia, ed 7, pp 920–922; Miller: Basics of Anesthesia, ed 6, pp 134, 277).
872. Each of the following additives to a spinal anesthetic possesses analgesic properties EXCEPT A. Clonidine B. Hydromorphone C. Epinephrine D. All of the above have analgesic properties
872. (D) Drugs with α-adrenergic agonist activity (phenylephrine, 2-5 mg; epinephrine, 0.2-0.5 mg; clonidine, 75-150 mg) possess some analgesic activity but less than narcotics and local anesthetics. In addition, these intrathecal α-adrenergic agonists may reduce systemic/vascular uptake of local anesthetics, thereby enhancing their effects, including hypotension. Clonidine alone, when administered neuraxially, is an effective analgesic. Neostigmine has some mild analgesia properties, but experience is limited. Opioids (e.g., fentanyl, sufentanil, hydromorphone, and morphine) added to the spinal solution enhance surgical anesthesia and provide postoperative pain relief. Fentanyl or sufentanil is commonly added for short surgical procedures (outpatient), whereas hydromorphone or morphine can be used when longer postoperative analgesia is desired for inpatients (Barash: Clinical Anesthesia, ed 7, pp 919–920; Miller: Miller’s Anesthesia, ed 8, pp 1693, 2983).
873. Which of the following local anesthetics is inappropriately paired with a clinical application because of its properties or toxicity? A. Tetracaine, topical anesthesia B. Bupivacaine, intravenous anesthesia C. Prilocaine, infiltrative anesthesia D. Chloroprocaine, epidural anesthesia
873. (B) For topical anesthesia, lidocaine, tetracaine, cocaine, dibucaine, and benzocaine are effective, as well as the combination of lidocaine and prilocaine, or EMLA cream. For intravenous regional anesthesia or Bier blocks, many drugs have been used. Ester local anesthetics are not used for IV regional blocks because they can be broken down in the bloodstream (by plasma ester hydrolysis), which can shorten the drug’s duration of action and can also cause thrombophlebitis of the vein (reported with chloroprocaine). Because cardiovascular collapse has been reported with bupivacaine, it should not be used for intravenous regional anesthesia. Lidocaine and prilocaine are used for Bier blocks because of their relative safety. For infiltrative and epidural anesthesia, almost all local anesthetics can be used (with the exception of cocaine and benzocaine, which are used only topically) (Butterworth: Morgan & Mikhail’s Clinical Anesthesiology, ed 5, p 272; Miller: Miller’s Anesthesia, ed 8, pp 1041–1044, 1736).
874. Discharge criteria from the postanesthesia care unit would be reached FASTEST after a 20- to 30-mL volume of which of the following epidurally administered local anesthetics? A. 3% 2-Chloroprocaine B. 2% Lidocaine C. 0.75% Ropivacaine D. 0.5% Levobupivacaine
874. (A) Procaine and 2-chloroprocaine have a short duration of action; lidocaine, mepivacaine, and prilocaine have an intermediate duration of action; and etidocaine, bupivacaine, levobupivacaine, tetracaine, and ropivacaine have a long duration of action. For similar sensory anesthesia, a higher concentration of local anesthetic is needed for the short duration of local anesthetics compared with both the intermediate and long-duration agents, because they are less potent (Barash: Clinical Anesthesia, ed 7, pp 920–922; Miller: Miller’s Anesthesia, ed 8, pp 1710–1711).
875. A caudal block (performed under sevoflurane general anesthesia) with 0.25% bupivacaine and 1:200,000 epinephrine is planned for postoperative analgesia after bilateral inguinal hernia repair in a 5-month-old patient. Each of the following would be consistent with an intravascular injection EXCEPT A. Systolic blood pressure increase by greater than 15 mm Hg B. Heart rate decrease by greater than 10 beats/min C. Ventricular extrasystoles D. Increase in T-wave amplitude >25% over baseline
875. (B) Under sevoflurane general anesthesia, an increase in the T-wave amplitude of 25% (usually in lead II), an increase in heart rate of 10 beats/min, or a systolic blood pressure increase greater than 15 mm Hg is considered a positive dose response to an epinephrine-containing local anesthetic solution. Under total intravenous anesthesia, an increase in blood pressure is more sensitive than an increase in T-wave amplitude or an increase in heart rate. As always, slow incremental dosing is safer than a large bolus dose (Barash: Clinical Anesthesia, ed 7, pp 1247–1248; Davis: Smith’s Anesthesia for Infants and Children, ed 8, pp 456–457).
876. Which is NOT a potential complication of a stellate ganglion block? A. Recurrent laryngeal nerve paralysis B. Subarachnoid block C. Brachial plexus block D. Increased heart rate
876. (D) All of the choices listed are potential complications of stellate ganglion blockade except an increase in heart rate. The stellate ganglion supplies sympathetic fibers to the upper extremity and head and some to the heart. Loss of the cardiac acceleratory fibers may slow the heart rate, not speed it up. Other potential complications of stellate ganglion blockade include accidental injection of the local anesthetic into a vertebral artery, resulting in seizure, phrenic nerve paralysis, and inadvertent cervical epidural (Miller: Basics of Anesthesia, ed 6, pp 707–710; Miller: Miller’s Anesthesia, ed 8, p 1732).
877. An axillary block using the transarterial approach with 0.5% bupivacaine and epinephrine (1:200,000) is performed in a 70-kg patient. A 30-mL quantity is injected posterior to the axillary artery and 30 mL anterior to it. How many milligrams have been injected, and was the maximum recommended dose exceeded? A. 150 mg bupivacaine, 150 μg epinephrine did not exceed maximum dose B. 150 mg bupivacaine, 150 μg epinephrine exceeded maximum dose C. 300 mg bupivacaine, 300 μg epinephrine did not exceed maximum dose D. 300 mg bupivacaine, 300 μg epinephrine exceeded maximum dose
877. (D) A total of 60 mL of 0.5% bupivacaine with epinephrine (1:200,000) was used. A 0.5% solution = 0.5 g in 100 mL of fluid = 500 mg/100 mL = 5 mg/mL. A 1:200,000 solution means 1 g in 200,000 mL = 1000 mg/ 200,000 mL = 1 mg/200 mL = 1000 μg/200 mL = 5 μg/mL. Therefore 60 mL of 0.5% bupivacaine contains 60 mL × 5 mg/mL = 300 mg bupivacaine and 1:200,000 epinephrine 60 mL × 5 μg/mL = 300 μg of epinephrine. For a major nerve block, the maximum recommended dose with epinephrine (1:200,000) is 500 mg for lidocaine and mepivacaine, 600 mg with prilocaine, and 225 mg with bupivacaine. 252 Part 2 Clinical Sciences Epinephrine is used in the local anesthetic to check for intravascular injection of the incremental doses and is not contraindicated but should be included for this block. Typically 40 to 45 mL is used for the transarterial approach to the axillary block (Barash: Clinical Anesthesia, ed 7, p 572; Miller: Miller’s Anesthesia, ed 8, pp 1043, 1728–1729).
878. Three days after knee arthroscopy under spinal anesthesia, a 55-year-old patient complains of double vision and difficulty hearing. The other likely finding would be A. Headache B. Fever C. Weakness in legs D. Mental status changes
878. (A) Postdural puncture headaches (spinal headaches) usually develop within 12 to 72 hours after a dural puncture but may develop immediately or take months to develop. The most characteristic symptom is a postural component in which the headache occurs in the upright position and is usually completely gone when the patient is in the supine position. The headache is typically frontal and/or occipital in location. Other symptoms include nausea, vomiting, anorexia, visual disturbances (blurred vision, double vision, photophobia), and occasionally hearing loss (routinely found with auditory testing) (Butterworth: Morgan & Mikhail’s Clinical Anesthesiology, ed 5, pp 969–970; Miller: Basics of Anesthesia, ed 6, pp 271–272).
879. Which of the following statements is TRUE concerning transversus abdominis plane (TAP) block? A. Ultrasound is useful in finding the intercostal nerves B. The local anesthetic is injected directly into the transversus abdominis muscle C. The subcostal, ilioinguinal, and iliohypogastric nerves are blocked D. 10 mL of local anesthetic is all that is needed for good spread
879. (C) TAP block is used to provide abdominal wall analgesia. The subcostal (T12), ilioinguinal (L1), and iliohypogastric (L1) nerves are the nerves primarily blocked. Ultrasound is often used to locate the proper plane where the local anesthetic is injected, since the nerves are too small to visualize. After visualization of the three abdominal wall muscles, the external oblique, the internal oblique, and the transversus abdominis muscles, the needle is inserted. The local anesthetic is injected into the muscle plane between the internal oblique and the transversus abdominis muscles (which is where these nerves travel) and not the muscle for effective analgesia. Typically 20 to 30 mL of local anesthetic (e.g., 2 mg/kg of bupivacaine) is needed for adequate spread of local anesthetic (Barash: Clinical Anesthesia, ed 7, pp 975–976; Butterworth: Morgan & Mikhail’s Clinical Anesthesiology, ed 5, pp 1021–1022).
880. Which of the following nerves can be electrically stimulated at the ankle to produce flexion of the toes? A. Posterior tibial nerve B. Saphenous nerve C. Deep peroneal nerve D. Superficial peroneal nerve
880. (A) Five nerves are blocked when performing an ankle block. The saphenous, superficial peroneal, and sural nerves are all sensory below the ankle, and electrical stimulation would have no effect. Stimulation of the posterior tibial nerve causes flexion of the toes by stimulating the flexor digitorum brevis muscles and abduction of the first toe by stimulating the abductor hallucis muscles. The posterior tibial nerve also is sensory to most of the plantar part of the foot. Stimulation of the deep peroneal nerve causes extension of the toes by stimulating the extensor digitorum brevis muscles. The deep peroneal nerve has a small sensory branch for the first interdigital cleft. From a practical standpoint, many anesthesiologists perform a purely infiltration block of these nerves. If a nerve stimulator is used, it is mainly used to find the posterior tibial nerve, which can be difficult to anesthetize if small volumes of local anesthetic are administered. The posterior tibial nerve can be difficult to stimulate in diabetic patients with diabetic neuropathy (Hebl: Mayo Clinic Atlas of Regional Anesthesia and Ultrasound-Guided Nerve Blockade, ed 1, pp 443–446; Barash: Clinical Anesthesia, ed 7, pp 990–992).
881. Which motor response from peripheral nerve stimulation is INCORRECTLY paired with the appropriate nerve? A. Musculocutaneous nerve—flexion of the forearm at the elbow B. Radial nerve—extension of all digits as well as the wrist and forearm C. Ulnar nerve—abduction of the thumb D. Median nerve—flexion of the wrist, pronation of the forearm Anatomy, Regional Anesthesia, and Pain Management 235
881. (C) Peripheral nerve stimulation is a common technique when performing axillary nerve blocks. The desired motor response from the nerve can be seen with 0.5 mA or less. The musculocutaneous nerve elicits elbow flexion. The radial nerve elicits extension of all the digits, the wrist, and the elbow, as well as supination of the forearm. The ulnar nerve elicits flexion at the wrist, fourth and fifth digits, and adduction (not abduction) of the thumb. The median nerve elicits flexion at the wrist and second and third digits as well as opposition of the thumb and pronation of the forearm (Butterworth: Morgan & Mikhail’s Clinical Anesthesiology, ed 5, p 992; Hebl: Mayo Clinic Atlas of Regional Anesthesia and Ultrasound-Guided Nerve Blockade, ed 1, pp 256–260).
882. During an airway exam, a 53-year-old patient mentions that his right thumb tingles and then becomes numb if he extends his head for more than a few seconds. This symptom MOST likely represents a(n) A. Unstable C-spine B. Lhermitte’s phenomenon C. C6 nerve root irritation D. C8 radiculopathy
882. (C) Unilateral numbness or paresthesia in the upper extremity during extension of the neck usually represents nerve root impingement at the vertebral foramina. C6 nerve distribution is the thumb. Specifically, unilateral degenerative changes restrict the foramen to such a degree that it compresses and irritates the nerve root traversing the vertebral foramen when the head is extended. Treatment ranges from NSAIDs to steroids and may require surgical intervention if there is muscle weakness. Lhermitte sign, named after Jean Lhermitte, occurs when head flexion causes shooting sensations down the back and into the lower limbs. It is a sign of posterior column disease (Hebl: Mayo Clinic Atlas of Regional Anesthesia and Ultrasound-Guided Nerve Blockade, ed 1, p 86; Miller: Miller’s Anesthesia, ed 8, p 1725).
883. When performing an interscalene block with a peripheral nerve stimulator, you note diaphragmatic movement. You should now A. Inject the local anesthetic, as the needle is in an appropriate location B. Redirect the needle in an anterior direction C. Redirect the needle in a posterior direction D. Advance the needle about 0.5 cm more and inject
883. (C) Although a successful interscalene block causes ipsilateral phrenic nerve paralysis in almost 100% of patients, identifying the phrenic nerve means that you are anterior to the brachial plexus and that you Anatomy, Regional Anesthesia, and Pain Management 253 should reposition your needle. You should redirect the needle in a posterior direction (Barash: Clinical Anesthesia, ed 7, pp 959–961; Hebl: Mayo Clinic Atlas of Regional Anesthesia and Ultrasound-Guided Nerve Blockade, ed 1, pp 195–199; Miller: Miller’s Anesthesia, ed 8, pp 1725–1727).
884. During placement of an interscalene block, the patient becomes hypotensive, bradycardic, apneic, and cyanotic. The MOST likely cause is A. Vertebral artery injection B. Phrenic nerve blockade C. Total spinal D. Stellate ganglion block
884. (C) With an intravascular injection, the main symptoms would most likely be CNS toxicity (e.g., seizures), as blood flow is directly to the brain. The Bezold-Jarisch reflex (hypotension and bradycardia) has been reported in awake, sitting patients undergoing shoulder surgery with an interscalene block. This may be related to intracardiac mechanoreceptors being stimulated by the decreased venous return in the sitting position. This leads to decreased sympathetic tone and increased parasympathetic tone. Breathing is still present with this reflex. Block of the stellate ganglion would produce Horner syndrome, which is not associated with breathing abnormalities. Injection into the intrathecal space is uncommon, but possible (especially if the needle is not pointed in the caudal direction), and would lead to a total spinal block with little local anesthetic injected (e.g., hypotension, bradycardia respiratory paralysis that would lead to cyanosis) (Barash: Clinical Anesthesia, ed 7, pp 959–961; Hebl: Mayo Clinic Atlas of Regional Anesthesia and Ultrasound-Guided Nerve Blockade, ed 1, pp 203–205; Miller: Miller’s Anesthesia, ed 8, pp 1725–1727).
885. The reason that ropivacaine is marketed as pure S enantiomers is because the S form is associated with A. Increased potency B. Longer duration C. Reduced cardiac toxicity D. Reduced incidence of anaphylaxis
885. (C) The pipecoloxylidide local anesthetics (mepivacaine, bupivacaine, ropivacaine, and levobupivacaine) are chiral drugs, which means that they have an asymmetric carbon atom (i.e., have a left or S and a right or R hand configuration). Mepivacaine and bupivacaine are produced as racemic mixtures (50% S:50% R). The pure S forms show reduced neurotoxicity and reduced cardiotoxicity (e.g., ropivacaine and levobupivacaine). Clinical studies suggest that the pure S forms have a slight decrease in potency and a shorter duration of action compared with racemic mixtures. Lidocaine is an achiral compound (i.e., has no chiral carbon atom) (Barash: Clinical Anesthesia, ed 7, pp 566–567; Brunton: Goodman & Gilman’s The Pharmacological Basis of Therapeutics ed 12, pp 565–574).
886. Nerves that originate from the sacral plexus include each of the following EXCEPT A. Femoral nerve B. Tibial nerve C. Sciatic nerve D. Common peroneal nerve
886. (A) Nerves to the lower extremity emerge from the L1-S4 nerve roots. The upper roots (mainly L1-L4) form the lumbar plexus, which gives rise to the genitofemoral (L1-L2), lateral femoral cutaneous (L2-L3), obturator (L2-L4), and the femoral (L2-L4) nerves. A branch from the lumbar plexus (L4) along with the sacral plexus (L4-S3) gives rise to the sciatic nerve. Branches of the sciatic nerve include the common peroneal (branches to make the superficial and deep) and the tibial, and the sural nerves (Barash: Clinical Anesthesia, ed 7, pp 952–955; Miller: Miller’s Anesthesia, ed 8, p 1736).
887. The only technique shown to prevent anesthetic-related nerve injury during placement of peripheral nerve blocks is A. Ultrasound-guided regional technique B. Transarterial technique C. Nerve stimulator D. None of the above
887. (D) Anesthetic-related nerve injuries to the brachial plexus are rare and poorly understood. The only way to minimize nerve injury is to minimize trauma to neural fibers. Although ultrasound-guided technique is promising, currently there is no clinical evidence for this (Neal: Upper extremity regional anesthesia: Essentials of our current understanding, 2008, Reg Anesth Pain Med 34:134–170, 2009).
888. An axillary block is performed on a healthy 19- year-old athlete. A 30-mL quantity of 0.75% bupivacaine is injected incrementally. Five minutes after the bupivacaine injection, the patient has a seizure and experiences cardiovascular collapse. Which of the measures below is NOT indicated? A. Begin chest compressions at 100 per minute B. Ventilate with 100% oxygen C. Bolus propofol to bind local anesthetic D. Infuse 20% lipid emulsion
888. (C) Local anesthetic systemic toxicity (LAST) is a multisystem phenomenon, but the most crucial manifestation involves the heart (atrioventricular conduction block, arrhythmias, myocardial depression, and cardiac arrest). In this case of cardiovascular (CV) collapse, treatment consists of getting help with the initial focus of airway management and CV support (i.e., basic and advanced cardiac life support). BUT AVOID the use of vasopressin, calcium channel blockers, β-blockers, or local anesthetics. Epinephrine doses should be reduced to less than 1 μg/kg. Lipid emulsion therapy should be started; the initial bolus of 20% Intralipid is 1.5 mL/kg (lean body mass) over 1 minute, followed by a continuous infusion of 0.25 mL/kg/min. Repeat the bolus one or two times for persistent CV collapse and double the continuous infusion rate if the blood pressure remains low. Continue the infusion for at least 10 minutes after CV stability is attained. The upper limit of 20% Intralipid is 10 mL/kg over 30 minutes. Failure to respond with the above treatment should prompt consideration for cardiopulmonary bypass. Although propofol is formulated as a lipid emulsion and as such would bind bupivacaine to some degree, the cardiac depressant effects of propofol would far overshadow any therapeutic benefit of binding bupivacaine. Also see explanation for Question 722 (ASRA.com: Downloadable Checklist for Treatment of Local Anesthetic Systemic Toxicity 9/19/11; Barash: Clinical Anesthesia, ed 7, p 1155; Miller: Basics of Anesthesia, ed 6, p 138).
889. The structure MOST likely to be blocked during placement of an interscalene block in addition to the brachial plexus is the A. Phrenic nerve B. Vertebral artery C. Recurrent laryngeal nerve D. Vagus nerve
889. (A) When performing an interscalene block, the needle is usually inserted where the line extending lateral to the cricoid cartilage (C6 level) intersects the interscalene groove. The needle is inserted perpendicular to 254 Part 2 Clinical Sciences the skin and is slowly advanced in a medial, caudal, and slightly posterior direction. The caudal direction is used to decrease the chance of injecting the local anesthetic into the vertebral artery, or obtaining a spinal or epidural block. Injecting into the vertebral artery may lead to an immediate convulsion since the local anesthetic would go directly to the brain. The phrenic nerve is routinely blocked (100% of the time) and, in healthy patients, rarely leads to symptoms. However, in patients with borderline respiratory insufficiency, respiratory compromise can result. Occasionally the recurrent laryngeal nerve is blocked. Unilateral paralysis rarely is clinically significant, but if contralateral recurrent paralysis existed preoperatively then complete airway obstruction may develop. The vagus nerve can also be blocked but is rarely clinically significant (Miller: Basics of Anesthesia, ed 6, pp 288–289; Miller: Miller’s Anesthesia, ed 8, pp 1725–1728).
890. All of the following are symptoms of a developing epidural hematoma EXCEPT A. Radicular back pain B. Bowel and bladder dysfunction C. Motor deficits D. Fever
890. (D) Epidural hematomas are rare complications of spinal anesthesia (1:220,000) and epidural anesthesia (1:150,000). However, in the presence of LMWH, the incidence is much higher: 1:40,000 with spinal anesthesia and 1:3000 with continuous epidural catheter. Clinical symptoms include radicular back pain, bowel and bladder dysfunction, and sensory or motor deficits. An MRI is the diagnostic test of choice, and prompt (<8 hours) decompressive laminectomy is the treatment of choice. Epidural abscesses typically progress slowly compared to epidural hematomas and are also associated with fever. See also explanation for Question 814. (Barash: Clinical Anesthesia, ed 7, p 929; Butterworth: Morgan & Mikhail’s Clinical Anesthesiology, ed 5, pp 970–972; Fleisher: Anesthesia and Uncommon Diseases, ed 6, pp 562–563; Miller: Miller’s Anesthesia, ed 8, pp 2344–2345).
891. In addition to C nerve fibers, which nerve fibers carry pain impulses? A. A-alpha (Aα) B. A-beta (Aβ) C. A-delta (Aδ) D. B

891. (C) Peripheral nerves are classified according to the fiber size and physiologic properties such as the presence or absence of myelin, conduction velocity, location, and function. All type A fibers are myelinated. These fibers are subclassified into four groups based on their diameter, location, and function. A-alpha (Aα) and A-beta (Aβ) fibers are 6 to 22 μm in diameter and have conduction velocities of 30 to 120 m/ sec. Aα fibers are efferent to the skeletal muscles. Aβ fibers are afferent from the skin and joints to provide touch and proprioception sensations. A-gamma (Aγ) fibers are 3 to 6 μm in diameter, have conduction velocities of 15 to 35 m/sec, and are efferent to the muscle spindles to provide muscle tone. A-delta (Aδ) fibers are 1 to 4 μm in diameter and have conduction velocities of 5 to 25 m/sec and are afferent fibers, which provide sharp localized pain and temperature and touch sensations. B fibers are myelinated, preganglionic sympathetic nerve fibers that are less than 3 μm in diameter, have medium conduction velocities 3 to 15 m/sec, and are involved with various autonomic nervous system control. C fibers are nonmyelinated, postganglionic sympathetic nerves that are 0.3 to 1.3 μm in diameter and have slow conduction velocities of 0.1 to 2 m/sec. C fibers are afferent sensory nerves involved with nonlocalized pain, temperature, and touch sensations (Barash: Clinical Anesthesia, ed 7, pp 1646–1648; Miller: Miller’s Anesthesia, ed 8, pp 1013–1014).

892. An intradural mass lesion at the tip of a drug infusion catheter is LEAST likely to present as A. Increasing pain B. Development of numbness in T8 dermatomal pattern C. Hypopnea D. Perianal numbness
892. (C) Overdose of intrathecal opiates would not be a sign of an intradural mass lesion. Granulomas at the tip of intrathecal catheters used with intrathecal drug delivery systems are gaining increased attention. Granulomas are more frequently associated with high concentrations and doses of either morphine (>10 mg/day) or hydromorphone (>10 mg/day). Most patients who will develop granulomas receive the intrathecal medications for more than 6 months. Presenting symptoms may include loss of drug effect, new pain or paresthesias, or neurologic deficits. Patients should be routinely screened for signs and symptoms of granuloma formation at scheduled intrathecal pump refill appointments. In suspicious cases, patients should undergo prompt diagnostic imaging and consideration of neurosurgical consultation (Barash: Clinical Anesthesia, ed 7, pp 1665–1668; Butterworth: Morgan & Mikhail’s Clinical Anesthesiology, ed 5, pp 1059–1060; Miller: Miller’s Anesthesia, ed 8, pp 1911–1912).
893. Benzocaine has all of the following properties EXCEPT A. It is a weak alkali B. It is used only topically C. It is metabolized by an esterase in the blood D. It can promote formation of methemoglobin
893. (A) In addition to benzocaine, tetracaine, cocaine, and lidocaine can also be used as topical anesthetics. Pseudocholinesterase, the enzyme responsible for the metabolism of succinylcholine, metabolizes the ester local anesthetics, benzocaine, procaine, chloroprocaine, and tetracaine. Benzocaine does promote the formation of methemoglobin but is not alone in that regard, as prilocaine also causes formation of methemoglobin. The pKa of benzocaine is 3.5, which qualifies it as a weak acid and as such exists in uncharged form at physiologic pH. All other local anesthetic pKa’s are higher than 7.4, meaning that some fraction of them exists in the protonated form (Barash: Clinical Anesthesia, ed 5, p 572; Brunton: Goodman & Gilman’s The Pharmacological Basis of Therapeutics, ed 12, pp 566, 572; Butterworth: Morgan & Mikhail’s Clinical Anesthesiology, ed 5, pp 271–272). Anatomy, Regional Anesthesia, and Pain Management 255
894. Which statement concerning local anesthetics is CORRECT? A. The un-ionized form of a local anesthetic binds to the nerve membrane to actually block conduction B. If one node of Ranvier is blocked, conduction will be reliably interrupted C. The presence of myelin enhances the ability of a local anesthetic to block nerve conduction D. Local anesthetics block transmission by inhibiting the voltage-gated potassium ion channels
894. (C) The un-ionized form of the local anesthetic traverses the nerve membrane, whereas the ionized form actually blocks conduction. About three nodes of Ranvier must be blocked to achieve anesthesia. The presence of myelin enhances the ability of a local anesthetic to block conduction, as does rapid firing. The local anesthetic blocks nerve transmission by inhibiting the voltage-gated sodium ion channels (Miller: Basics of Anesthesia, ed 6, pp 131–135).
895. Postdural puncture headaches A. Usually occur immediately following dural puncture B. Are relieved 8 to 12 hours after an epidural blood patch is performed C. Occur more frequently in nonpregnant patients compared with pregnant patients D. Can be associated with neurologic deficits 236 Part 2 Clinical Sciences
895. (D) PDPHs typically appear within 12 to 48 hours of a dural puncture but may be immediate and occasionally have become delayed for several days or months after a dural puncture. The headaches are characterized by dull or throbbing frontal or occipital pain, which worsens with sitting and improves with reclining. Postspinal headaches may be associated with neurologic symptoms such as diplopia, tinnitus, and reduced hearing acuity. Very rarely, a subdural hematoma will develop. The etiology of postspinal headaches is believed to be due to a reduction in CSF pressure and resulting tension on meningeal vessels and nerves (which results from leakage of CSF through the needle hole in the dura mater). Factors associated with an increased incidence of postspinal headaches include pregnancy, size (larger needles leave bigger holes than smaller needles), type of needle used to perform the block (cutting Quincke needles more commonly associated with PDPH than pencil-point Whitacre or bullet-shaped Sprotte needles), and the number of dural punctures. They occur more frequently in young adults compared with children and elderly persons. Conservative therapy for a postspinal headache includes bed rest, analgesics, and oral and intravenous hydration. If conservative therapy is not successful after 24 to 48 hours, an epidural “blood patch” with 10 to 20 mL of the patient’s blood can be performed. An epidural blood patch usually provides prompt relief of the postspinal headache (Barash: Clinical Anesthesia, ed 7, pp 926–927; Miller: Basics of Anesthesia, ed 6, pp 271–272).
896. Which of the following procedures for treatment of chronic pain requires localization of the epidural space with an epidural needle as part of technique? A. Intradiscal electrothermal therapy (IDET) B. Spinal cord stimulation C. Percutaneous disk decompression D. Vertebroplasty
896. (B) IDET is a procedure rarely used for intractable discogenic low back pain, in which a flexible thermal electrode is advanced through an introducer percutaneously into the posterolateral portion of a disk. The electrode is gradually heated to 90° C for 4 minutes (or 80-85° C for 5 minutes), which causes the collagen of the annulus fibrosus of the disk to contract and decreases intradiscal pressure. With percutaneous disk decompression or nucleoplasty, an electrode is passed through an introducer into the disk, the tissue is heated (40-70° C range), and a portion of the disk is removed. For spinal cord stimulation therapy, a trial is first performed, and, if it is successful, then permanent implantation is performed. When inserting a spinal cord stimulator, a Touhy epidural needle is advanced into the epidural space. After confirmation of proper needle placement with anteroposterior and lateral fluoroscopic views, the stimulation electrode is passed through the needle and threaded to the desired vertebral level. The needle is then removed and the leads attached to the external programmer. Vertebroplasty involves the injection of 2 to 6 mL of cement (polymethylmethacrylate) into a vertebral body to help treat vertebral compression fractures (Barash: Clinical Anesthesia, ed 7, pp 1663–1665).
897. Each of the following drugs has been used to treat neuropathic pain. Selective inhibition of serotonin and norepinephrine reuptake is the mechanism of which drug? A. Duloxetine B. Mexiletine C. Gabapentin D. Carbamazepine Skin Medial A C B D E Lateral
897. (A) Many drugs have been used to treat neuropathic pain, including analgesics (NSAIDs and opioids), first-generation antiepileptic drugs (e.g., carbamazepine and phenytoin), second-generation antiepileptic drugs (e.g., gabapentin, pregabalin), topical agents (e.g., lidocaine, capsaicin), antiarrhythmics (e.g., mexiletine), and tricyclic antidepressants (e.g., amitriptyline, nortriptyline, desipramine), as well as other antidepressants (e.g., duloxetine, venlafaxine). Duloxetine (Cymbalta) is a selective serotonin and norepinephrine reuptake inhibitor (SNRI) that is used for major depressive disorders, generalized anxiety disorders, fibromyalgia, and neuropathic pain. Mexiletine is an orally effective amine analog of lidocaine and may be effective in decreasing neuropathic pain when other drugs have failed. Gabapentin, a structural analog of γ-aminobutyric acid (GABA), works by increasing the synthesis of the inhibitory neurotransmitter GABA. Carbamazepine (Tegretol) is an anticonvulsant with specific analgesic properties for trigeminal neuralgia. Carbamazepine seems to reduce polysynaptic responses by an unknown mechanism (Butterworth: Morgan & Mikhail’s Clinical Anesthesiology, ed 5, pp 1037–1055; Cousins: Neural Blockade in Clinical Anesthesia and Pain Medicine, ed 4, p 1065; Hemmings: Pharmacology and Physiology for Anesthesia, ed 1, pp 280–281; Miller: Miller’s Anesthesia, ed 8, pp 1903–1910).
898. Musculocutaneous nerve A. C3-C5 B. T1-T4 C. T5-T12 D. T10-L1 E. S2-S4
898. (E)(Hebl: Mayo Clinic Atlas of Regional Anesthesia and Ultrasound-Guided Nerve Blockade, ed 1, pp 260–269.) Fat Ulnar n. Median n. Biceps m. Triceps m. Musculocutaneous n. Radial n. Coracobrachialis m. Intercostobrachial n. Axillary a. Axillary v. Skin
899. Axillary artery A. C3-C5 B. T1-T4 C. T5-T12 D. T10-L1 E. S2-S4
899. (B)(Hebl: Mayo Clinic Atlas of Regional Anesthesia and Ultrasound-Guided Nerve Blockade, ed 1, pp 260–269.) Fat Ulnar n. Median n. Biceps m. Triceps m. Musculocutaneous n. Radial n. Coracobrachialis m. Intercostobrachial n. Axillary a. Axillary v. Skin
900. Axillary vein A. C3-C5 B. T1-T4 C. T5-T12 D. T10-L1 E. S2-S4
900. (A) (Hebl: Mayo Clinic Atlas of Regional Anesthesia and Ultrasound-Guided Nerve Blockade, ed 1, pp 260–269.) Fat Ulnar n. Median n. Biceps m. Triceps m. Musculocutaneous n. Radial n. Coracobrachialis m. Intercostobrachial n. Axillary a. Axillary v. Skin
901. Ulnar nerve A. C3-C5 B. T1-T4 C. T5-T12 D. T10-L1 E. S2-S4
901. (C) (Hebl: Mayo Clinic Atlas of Regional Anesthesia and Ultrasound-Guided Nerve Blockade, ed 1, pp 260–269.) Fat Ulnar n. Median n. Biceps m. Triceps m. Musculocutaneous n. Radial n. Coracobrachialis m. Intercostobrachial n. Axillary a. Axillary v. Skin
902. Phrenic nerve A. C3-C5 B. T1-T4 C. T5-T12 D. T10-L1 E. S2-S4
902. (A)In the normal adult, breathing and coughing can be done exclusively by the diaphragm, which is innervated by the phrenic nerve (C3-C5). The heart rate is dependent upon intrinsic pacemaker activity of the sinoatrial node, which can be affected by the autonomic nervous system’s sympathetic nervous system’s cardiac accelerator fibers (T1-T4) as well as the parasympathetic nervous system’s vagus nerve (cranial nerve X). The first stage of labor pain is related to uterine contractions and dilation of the cervix (T10-L1). The second stage of labor is related to both uterine pain (T10-L1) and birth canal pain, which is conducted by the pudendal nerve (S2-S4). The greater splanchnic (T5-T9) and the lesser splanchnic (T10-T12) nerves supply sympathetic fibers to the celiac plexus, which inhibits much of the gastrointestinal tract (Barash: Clinical Anesthesia, ed 7, pp 364–367, 1149; Butterworth: Morgan & Mikhail’s Clinical Anesthesiology, ed 5, pp 492, 846, 1073; Miller: Miller’s Anesthesia, ed 8, pp 347–349, 1688, 2339).
903. Cardiac accelerator fibers A. C3-C5 B. T1-T4 C. T5-T12 D. T10-L1 E. S2-S4
903. (B)In the normal adult, breathing and coughing can be done exclusively by the diaphragm, which is innervated by the phrenic nerve (C3-C5). The heart rate is dependent upon intrinsic pacemaker activity of the sinoatrial node, which can be affected by the autonomic nervous system’s sympathetic nervous system’s cardiac accelerator fibers (T1-T4) as well as the parasympathetic nervous system’s vagus nerve (cranial nerve X). The first stage of labor pain is related to uterine contractions and dilation of the cervix (T10-L1). The second stage of labor is related to both uterine pain (T10-L1) and birth canal pain, which is conducted by the pudendal nerve (S2-S4). The greater splanchnic (T5-T9) and the lesser splanchnic (T10-T12) nerves supply sympathetic fibers to the celiac plexus, which inhibits much of the gastrointestinal tract (Barash: Clinical Anesthesia, ed 7, pp 364–367, 1149; Butterworth: Morgan & Mikhail’s Clinical Anesthesiology, ed 5, pp 492, 846, 1073; Miller: Miller’s Anesthesia, ed 8, pp 347–349, 1688, 2339).
904. Pudendal nerve A. C3-C5 B. T1-T4 C. T5-T12 D. T10-L1 E. S2-S4
904. (E)In the normal adult, breathing and coughing can be done exclusively by the diaphragm, which is innervated by the phrenic nerve (C3-C5). The heart rate is dependent upon intrinsic pacemaker activity of the sinoatrial node, which can be affected by the autonomic nervous system’s sympathetic nervous system’s cardiac accelerator fibers (T1-T4) as well as the parasympathetic nervous system’s vagus nerve (cranial nerve X). The first stage of labor pain is related to uterine contractions and dilation of the cervix (T10-L1). The second stage of labor is related to both uterine pain (T10-L1) and birth canal pain, which is conducted by the pudendal nerve (S2-S4). The greater splanchnic (T5-T9) and the lesser splanchnic (T10-T12) nerves supply sympathetic fibers to the celiac plexus, which inhibits much of the gastrointestinal tract (Barash: Clinical Anesthesia, ed 7, pp 364–367, 1149; Butterworth: Morgan & Mikhail’s Clinical Anesthesiology, ed 5, pp 492, 846, 1073; Miller: Miller’s Anesthesia, ed 8, pp 347–349, 1688, 2339).
905. Pain fibers to the uterus A. C3-C5 B. T1-T4 C. T5-T12 D. T10-L1 E. S2-S4
905. (D)In the normal adult, breathing and coughing can be done exclusively by the diaphragm, which is innervated by the phrenic nerve (C3-C5). The heart rate is dependent upon intrinsic pacemaker activity of the sinoatrial node, which can be affected by the autonomic nervous system’s sympathetic nervous system’s cardiac accelerator fibers (T1-T4) as well as the parasympathetic nervous system’s vagus nerve (cranial nerve X). The first stage of labor pain is related to uterine contractions and dilation of the cervix (T10-L1). The second stage of labor is related to both uterine pain (T10-L1) and birth canal pain, which is conducted by the pudendal nerve (S2-S4). The greater splanchnic (T5-T9) and the lesser splanchnic (T10-T12) nerves supply sympathetic fibers to the celiac plexus, which inhibits much of the gastrointestinal tract (Barash: Clinical Anesthesia, ed 7, pp 364–367, 1149; Butterworth: Morgan & Mikhail’s Clinical Anesthesiology, ed 5, pp 492, 846, 1073; Miller: Miller’s Anesthesia, ed 8, pp 347–349, 1688, 2339).
906. Inhibitory presynaptic fibers to the gastrointestinal tract A. C3-C5 B. T1-T4 C. T5-T12 D. T10-L1 E. S2-S4
906. (C) In the normal adult, breathing and coughing can be done exclusively by the diaphragm, which is innervated by the phrenic nerve (C3-C5). The heart rate is dependent upon intrinsic pacemaker activity of the sinoatrial node, which can be affected by the autonomic nervous system’s sympathetic nervous system’s cardiac accelerator fibers (T1-T4) as well as the parasympathetic nervous system’s vagus nerve (cranial nerve X). The first stage of labor pain is related to uterine contractions and dilation of the cervix (T10-L1). The second stage of labor is related to both uterine pain (T10-L1) and birth canal pain, which is conducted by the pudendal nerve (S2-S4). The greater splanchnic (T5-T9) and the lesser splanchnic (T10-T12) nerves supply sympathetic fibers to the celiac plexus, which inhibits much of the gastrointestinal tract (Barash: Clinical Anesthesia, ed 7, pp 364–367, 1149; Butterworth: Morgan & Mikhail’s Clinical Anesthesiology, ed 5, pp 492, 846, 1073; Miller: Miller’s Anesthesia, ed 8, pp 347–349, 1688, 2339).
907. Sensory innervation of the mucous membranes of the nose A. Trigeminal nerve B. Glossopharyngeal nerve C. Internal branch of the superior laryngeal nerve D. External branch of the superior laryngeal nerve E. Recurrent laryngeal nerve
907. (A) When an awake intubation is needed, local anesthetics can be applied topically or injected to anesthetize the airway. The sensory nerve supply to the upper airway is predominantly by three cranial nerves: the trigeminal nerve (cranial nerve V), the glossopharyngeal nerve (cranial nerve IX), and the vagus nerve (cranial nerve X). Branches from the trigeminal nerve provide sensory supply to the mucous membranes of the nose as well as the superior and inferior portions of the hard and soft palate. The glossopharyngeal nerve provides sensory innervation of the posterior third of the tongue, the vallecula, and the anterior surface of the epiglottis (lingual branch), the pharyngeal walls (pharyngeal branch), and the tonsils (tonsillar branch). The vagus nerve gives rise to the internal and external branches of superior laryngeal nerve as well as the recurrent laryngeal nerve. The sensory innervation of the mucosa of the larynx above the vocal cords comes from the internal branch of the superior laryngeal nerve, and the sensory innervation of the mucosa of the larynx below the vocal cords comes from the recurrent laryngeal nerve. With the exception of the cricothyroid muscle, the recurrent laryngeal nerve provides motor innervation of all the intrinsic muscles of the larynx. The cricothyroid muscle is supplied by the external branch of the superior laryngeal nerve. The muscles of the pharynx are supplied through the pharyngeal plexus from motor fibers from the spinal accessory nerve (cranial nerve XI) (Barash: Clinical Anesthesia, ed 7, pp 789–791; Butterworth: Morgan & Mikhail’s Clinical Anesthesiology, ed 7, pp 310–312). 258
908. Main sensory innervation to superior and inferior parts of the hard and soft palate A. Trigeminal nerve B. Glossopharyngeal nerve C. Internal branch of the superior laryngeal nerve D. External branch of the superior laryngeal nerve E. Recurrent laryngeal nerve
908. (A) When an awake intubation is needed, local anesthetics can be applied topically or injected to anesthetize the airway. The sensory nerve supply to the upper airway is predominantly by three cranial nerves: the trigeminal nerve (cranial nerve V), the glossopharyngeal nerve (cranial nerve IX), and the vagus nerve (cranial nerve X). Branches from the trigeminal nerve provide sensory supply to the mucous membranes of the nose as well as the superior and inferior portions of the hard and soft palate. The glossopharyngeal nerve provides sensory innervation of the posterior third of the tongue, the vallecula, and the anterior surface of the epiglottis (lingual branch), the pharyngeal walls (pharyngeal branch), and the tonsils (tonsillar branch). The vagus nerve gives rise to the internal and external branches of superior laryngeal nerve as well as the recurrent laryngeal nerve. The sensory innervation of the mucosa of the larynx above the vocal cords comes from the internal branch of the superior laryngeal nerve, and the sensory innervation of the mucosa of the larynx below the vocal cords comes from the recurrent laryngeal nerve. With the exception of the cricothyroid muscle, the recurrent laryngeal nerve provides motor innervation of all the intrinsic muscles of the larynx. The cricothyroid muscle is supplied by the external branch of the superior laryngeal nerve. The muscles of the pharynx are supplied through the pharyngeal plexus from motor fibers from the spinal accessory nerve (cranial nerve XI) (Barash: Clinical Anesthesia, ed 7, pp 789–791; Butterworth: Morgan & Mikhail’s Clinical Anesthesiology, ed 7, pp 310–312). 258
909. Sensory innervation of the larynx above the vocal cords A. Trigeminal nerve B. Glossopharyngeal nerve C. Internal branch of the superior laryngeal nerve D. External branch of the superior laryngeal nerve E. Recurrent laryngeal nerve
909. (C) When an awake intubation is needed, local anesthetics can be applied topically or injected to anesthetize the airway. The sensory nerve supply to the upper airway is predominantly by three cranial nerves: the trigeminal nerve (cranial nerve V), the glossopharyngeal nerve (cranial nerve IX), and the vagus nerve (cranial nerve X). Branches from the trigeminal nerve provide sensory supply to the mucous membranes of the nose as well as the superior and inferior portions of the hard and soft palate. The glossopharyngeal nerve provides sensory innervation of the posterior third of the tongue, the vallecula, and the anterior surface of the epiglottis (lingual branch), the pharyngeal walls (pharyngeal branch), and the tonsils (tonsillar branch). The vagus nerve gives rise to the internal and external branches of superior laryngeal nerve as well as the recurrent laryngeal nerve. The sensory innervation of the mucosa of the larynx above the vocal cords comes from the internal branch of the superior laryngeal nerve, and the sensory innervation of the mucosa of the larynx below the vocal cords comes from the recurrent laryngeal nerve. With the exception of the cricothyroid muscle, the recurrent laryngeal nerve provides motor innervation of all the intrinsic muscles of the larynx. The cricothyroid muscle is supplied by the external branch of the superior laryngeal nerve. The muscles of the pharynx are supplied through the pharyngeal plexus from motor fibers from the spinal accessory nerve (cranial nerve XI) (Barash: Clinical Anesthesia, ed 7, pp 789–791; Butterworth: Morgan & Mikhail’s Clinical Anesthesiology, ed 7, pp 310–312). 258
910. Sensory innervation below the vocal cords to the carina A. Trigeminal nerve B. Glossopharyngeal nerve C. Internal branch of the superior laryngeal nerve D. External branch of the superior laryngeal nerve E. Recurrent laryngeal nerve
910. (E) When an awake intubation is needed, local anesthetics can be applied topically or injected to anesthetize the airway. The sensory nerve supply to the upper airway is predominantly by three cranial nerves: the trigeminal nerve (cranial nerve V), the glossopharyngeal nerve (cranial nerve IX), and the vagus nerve (cranial nerve X). Branches from the trigeminal nerve provide sensory supply to the mucous membranes of the nose as well as the superior and inferior portions of the hard and soft palate. The glossopharyngeal nerve provides sensory innervation of the posterior third of the tongue, the vallecula, and the anterior surface of the epiglottis (lingual branch), the pharyngeal walls (pharyngeal branch), and the tonsils (tonsillar branch). The vagus nerve gives rise to the internal and external branches of superior laryngeal nerve as well as the recurrent laryngeal nerve. The sensory innervation of the mucosa of the larynx above the vocal cords comes from the internal branch of the superior laryngeal nerve, and the sensory innervation of the mucosa of the larynx below the vocal cords comes from the recurrent laryngeal nerve. With the exception of the cricothyroid muscle, the recurrent laryngeal nerve provides motor innervation of all the intrinsic muscles of the larynx. The cricothyroid muscle is supplied by the external branch of the superior laryngeal nerve. The muscles of the pharynx are supplied through the pharyngeal plexus from motor fibers from the spinal accessory nerve (cranial nerve XI) (Barash: Clinical Anesthesia, ed 7, pp 789–791; Butterworth: Morgan & Mikhail’s Clinical Anesthesiology, ed 7, pp 310–312). 258
911. Sensory innervation to posterior third of the tongue A. Trigeminal nerve B. Glossopharyngeal nerve C. Internal branch of the superior laryngeal nerve D. External branch of the superior laryngeal nerve E. Recurrent laryngeal nerve
911. (B) When an awake intubation is needed, local anesthetics can be applied topically or injected to anesthetize the airway. The sensory nerve supply to the upper airway is predominantly by three cranial nerves: the trigeminal nerve (cranial nerve V), the glossopharyngeal nerve (cranial nerve IX), and the vagus nerve (cranial nerve X). Branches from the trigeminal nerve provide sensory supply to the mucous membranes of the nose as well as the superior and inferior portions of the hard and soft palate. The glossopharyngeal nerve provides sensory innervation of the posterior third of the tongue, the vallecula, and the anterior surface of the epiglottis (lingual branch), the pharyngeal walls (pharyngeal branch), and the tonsils (tonsillar branch). The vagus nerve gives rise to the internal and external branches of superior laryngeal nerve as well as the recurrent laryngeal nerve. The sensory innervation of the mucosa of the larynx above the vocal cords comes from the internal branch of the superior laryngeal nerve, and the sensory innervation of the mucosa of the larynx below the vocal cords comes from the recurrent laryngeal nerve. With the exception of the cricothyroid muscle, the recurrent laryngeal nerve provides motor innervation of all the intrinsic muscles of the larynx. The cricothyroid muscle is supplied by the external branch of the superior laryngeal nerve. The muscles of the pharynx are supplied through the pharyngeal plexus from motor fibers from the spinal accessory nerve (cranial nerve XI) (Barash: Clinical Anesthesia, ed 7, pp 789–791; Butterworth: Morgan & Mikhail’s Clinical Anesthesiology, ed 7, pp 310–312). 258
912. Sensory innervation to the pharyngeal walls and the tonsils A. Trigeminal nerve B. Glossopharyngeal nerve C. Internal branch of the superior laryngeal nerve D. External branch of the superior laryngeal nerve E. Recurrent laryngeal nerve
912. (B) When an awake intubation is needed, local anesthetics can be applied topically or injected to anesthetize the airway. The sensory nerve supply to the upper airway is predominantly by three cranial nerves: the trigeminal nerve (cranial nerve V), the glossopharyngeal nerve (cranial nerve IX), and the vagus nerve (cranial nerve X). Branches from the trigeminal nerve provide sensory supply to the mucous membranes of the nose as well as the superior and inferior portions of the hard and soft palate. The glossopharyngeal nerve provides sensory innervation of the posterior third of the tongue, the vallecula, and the anterior surface of the epiglottis (lingual branch), the pharyngeal walls (pharyngeal branch), and the tonsils (tonsillar branch). The vagus nerve gives rise to the internal and external branches of superior laryngeal nerve as well as the recurrent laryngeal nerve. The sensory innervation of the mucosa of the larynx above the vocal cords comes from the internal branch of the superior laryngeal nerve, and the sensory innervation of the mucosa of the larynx below the vocal cords comes from the recurrent laryngeal nerve. With the exception of the cricothyroid muscle, the recurrent laryngeal nerve provides motor innervation of all the intrinsic muscles of the larynx. The cricothyroid muscle is supplied by the external branch of the superior laryngeal nerve. The muscles of the pharynx are supplied through the pharyngeal plexus from motor fibers from the spinal accessory nerve (cranial nerve XI) (Barash: Clinical Anesthesia, ed 7, pp 789–791; Butterworth: Morgan & Mikhail’s Clinical Anesthesiology, ed 7, pp 310–312). 258
913. Motor innervation to the intrinsic muscles of the larynx, except cricothyroid muscle A. Trigeminal nerve B. Glossopharyngeal nerve C. Internal branch of the superior laryngeal nerve D. External branch of the superior laryngeal nerve E. Recurrent laryngeal nerve
913. (E) When an awake intubation is needed, local anesthetics can be applied topically or injected to anesthetize the airway. The sensory nerve supply to the upper airway is predominantly by three cranial nerves: the trigeminal nerve (cranial nerve V), the glossopharyngeal nerve (cranial nerve IX), and the vagus nerve (cranial nerve X). Branches from the trigeminal nerve provide sensory supply to the mucous membranes of the nose as well as the superior and inferior portions of the hard and soft palate. The glossopharyngeal nerve provides sensory innervation of the posterior third of the tongue, the vallecula, and the anterior surface of the epiglottis (lingual branch), the pharyngeal walls (pharyngeal branch), and the tonsils (tonsillar branch). The vagus nerve gives rise to the internal and external branches of superior laryngeal nerve as well as the recurrent laryngeal nerve. The sensory innervation of the mucosa of the larynx above the vocal cords comes from the internal branch of the superior laryngeal nerve, and the sensory innervation of the mucosa of the larynx below the vocal cords comes from the recurrent laryngeal nerve. With the exception of the cricothyroid muscle, the recurrent laryngeal nerve provides motor innervation of all the intrinsic muscles of the larynx. The cricothyroid muscle is supplied by the external branch of the superior laryngeal nerve. The muscles of the pharynx are supplied through the pharyngeal plexus from motor fibers from the spinal accessory nerve (cranial nerve XI) (Barash: Clinical Anesthesia, ed 7, pp 789–791; Butterworth: Morgan & Mikhail’s Clinical Anesthesiology, ed 7, pp 310–312). 258
914. Motor innervation to the cricothyroid muscle A. Trigeminal nerve B. Glossopharyngeal nerve C. Internal branch of the superior laryngeal nerve D. External branch of the superior laryngeal nerve E. Recurrent laryngeal nerve
914. (D) When an awake intubation is needed, local anesthetics can be applied topically or injected to anesthetize the airway. The sensory nerve supply to the upper airway is predominantly by three cranial nerves: the trigeminal nerve (cranial nerve V), the glossopharyngeal nerve (cranial nerve IX), and the vagus nerve (cranial nerve X). Branches from the trigeminal nerve provide sensory supply to the mucous membranes of the nose as well as the superior and inferior portions of the hard and soft palate. The glossopharyngeal nerve provides sensory innervation of the posterior third of the tongue, the vallecula, and the anterior surface of the epiglottis (lingual branch), the pharyngeal walls (pharyngeal branch), and the tonsils (tonsillar branch). The vagus nerve gives rise to the internal and external branches of superior laryngeal nerve as well as the recurrent laryngeal nerve. The sensory innervation of the mucosa of the larynx above the vocal cords comes from the internal branch of the superior laryngeal nerve, and the sensory innervation of the mucosa of the larynx below the vocal cords comes from the recurrent laryngeal nerve. With the exception of the cricothyroid muscle, the recurrent laryngeal nerve provides motor innervation of all the intrinsic muscles of the larynx. The cricothyroid muscle is supplied by the external branch of the superior laryngeal nerve. The muscles of the pharynx are supplied through the pharyngeal plexus from motor fibers from the spinal accessory nerve (cranial nerve XI) (Barash: Clinical Anesthesia, ed 7, pp 789–791; Butterworth: Morgan & Mikhail’s Clinical Anesthesiology, ed 7, pp 310–312). 258
915. A 67-year-old man is to undergo a radical retropubic prostatectomy. He has aortic stenosis with a gradient of 37 mm Hg at rest. He has an allergy to penicillin. Which of the following is the best regimen for subacute bacterial endocarditis prophylaxis in this patient? A. Ampicillin and gentamicin B. Vancomycin and gentamicin C. Clindamycin and gentamicin D. None of the above
915. (D) In 2007, the American Heart Association revised the guidelines for prevention of infective endocarditis (IE). Presently, only patients with underlying cardiac conditions with the highest risk for an adverse outcome from IE should receive antibiotic prophylaxis for selected dental procedures. Prophylaxis is not recommended for patients undergoing elective genitourinary (GU) or gastrointestinal (GI) procedures. The cardiac conditions with the highest risk include: prosthetic cardiac valves, previous IE, several types of congenital heart disease (CHD), and cardiac transplantation recipients who develop cardiac valvulopathy. Any of the antibiotics listed in the question, cephalexin 2 g orally (or other firstor second-generation oral cephalosporin in equivalent dosage), or clindamycin 600 mg orally, IM, or IV should be administered 30 to 60 minutes before the procedure. This patient has aortic stenosis and does not need any prophylaxis (Wilson et al: Prevention of infective endocarditis—Guidelines from the American Heart Association, Circulation 115:1736–1754, 2007. http://circ.ahajournals.org).
916. A 64-year-old man develops heparin-induced thrombocytopenia (HIT), type II (antibody proven), after anticoagulation for aortic valve replacement with 25,000 units of heparin. The same patient requires an elective tricuspid valve replacement soon thereafter because of trauma from a transvenous pacemaker. The best option for cardiopulmonary bypass anticoagulation for this patient with the second operation would be A. Defer until disappearance of antibodies; use heparin B. Cardiopulmonary bypass with lepirudin in place of heparin C. Cardiopulmonary bypass with tirofiban in place of heparin D. Anticoagulation with fondaparinux
916. (A) Type II HIT is a serious, life-threatening condition. The clinical diagnosis is made by demonstrating a decrease in platelet count to 100,000/mm3 or half the preoperative value 5 to 10 days after administration of heparin. Patients with HIT are prone to paradoxical thrombosis and must be closely monitored. Serologically, patients demonstrate antibodies to the platelet factor 4 (PF4)/heparin antigen. If surgery involving cardiopulmonary bypass is contemplated, waiting until antibody titers become undetectable is the best choice. For emergency operations, various strategies for anticoagulation exist that include direct thrombin inhibitors, bivalirudin, and lepirudin. Other options are use of danaparoid (factor Xa inhibitor) or use of unfractionated heparin plus a drug to prevent thrombosis such as tirofiban (glycoprotein IIb/IIIa inhibitor), or epoprostenol (prostacyclin [PGI2]). Fondaparinux is not used for cardiopulmonary bypass anticoagulation. There is also the option of performing plasma phoresis to remove antiplatelet antibodies if time allows (Miller: Miller’s Anesthesia, ed 8, pp 2017–2022; Miller: Basics of Anesthesia, ed 5, pp 358–359).
917. Which of the following is the MOST sensitive indicator of left ventricular myocardial ischemia? A. Wall-motion abnormalities on the echocardiogram B. ST segment changes in lead V5 of the electrocardiogram (ECG) C. Appearance of V waves on the pulmonary capillary wedge pressure tracing D. Decrease in cardiac output as measured by the thermodilution technique
917. (A) All of the choices listed in this question occur during myocardial ischemia. However, of the choices listed, presence of left ventricular wall-motion abnormalities is the most sensitive indicator (Barash: Clinical Anesthesia, ed 7, p 744).
918. Oxygen consumption (Vo2) is measured in a 70-kg subject on a treadmill at 2500 mL per minute. This corresponds to: A. 1 metabolic equivalent (MET) B. 5 METs C. 10 METs D. 15 METs
918. (C) One MET is equal to the amount of energy expended during 1 minute at rest, which is roughly 3.5 mL of oxygen per kilogram of body weight per minute (3.5 mL/kg/min). For a 70-kg (150 lb) person, one MET would equal 250 mL O2 per minute. So 2500 mL would correspond to 10 METs (Barash: Clinical Anesthesia, ed 7, p 591).
919. Accidental injection of air into a peripheral vein would be LEAST likely to result in arterial air embolism in a patient with which of the following anatomic cardiac defects? A. Patent ductus arteriosus B. Eisenmenger syndrome C. Tetralogy of Fallot D. Tricuspid atresia
919. (A) The anesthetic management of patients with CHD requires thorough knowledge of the pathophysiology of the defect. In general, congenital heart defects can be categorized into those that result in left-to-right intracardiac shunting and into those that result in right-to-left shunting. The main feature in congenital heart defects that result in right-to-left intracardiac shunting is a reduction in pulmonary blood flow and arterial hypoxemia. The more common congenital heart defects that result in right-to-left intracardiac shunting include tetralogy of Fallot, Eisenmenger syndrome, Ebstein malformation of the tricuspid valve, pulmonary atresia with a ventricular septal defect, tricuspid atresia, and patent foramen ovale. Meticulous care must be taken to avoid infusion of air via intravenous solutions, because this can lead to arterial air embolism. Patients with congenital cardiac defects that result in left-to-right intracardiac shunting, such as patent ductus arteriosus, are at minimal risk for arterial air embolism, because blood flow through the shunt is primarily from the systemic vascular system to the pulmonary vascular system (Barash: Clinical Anesthesia, ed 7, pp 1106–1109).
920. Each of the following could be placed on the x-axis of the curve shown in the figure EXCEPT Function Normal Function A. Stroke volume B. Left ventricular end-diastolic pressure C. Left ventricular end-diastolic volume D. Left atrial pressure
920. (A) The Frank-Starling curve relates left ventricular filling pressure to left ventricular work. Left ventricular end-diastolic volume, left ventricular end-diastolic pressure, left atrial pressure, PA occlusion pressure, and, in some instances, central venous pressure can reflect left ventricular filling pressure. Cardiovascular Physiology and Anesthesia 269 Left ventricular work can be represented on the y-axis by left ventricular stroke work index, stroke volume, cardiac output, cardiac index, and arterial blood pressure (Miller: Miller’s Anesthesia, ed 8, pp 476–477).
921. The ECG rhythm strip below represents A. Atrial flutter B. Third-degree heart block C. Sinus tachycardia second-degree heart block D. Junctional rhythm
921. (A) The rhythm strip in the question depicts atrial flutter. The importance of examining more than one lead is emphasized in this question. The lower tracing looks like a junctional rhythm, but upon examination of the upper tracing, discrete P waves (actually F waves) corresponding to a rate of about 300/min are easily discerned. An atrial rate of 300 is common, often with 2:1 conduction, yielding a ventricular rate of 150/min. In the rhythm presented here, the ventricular rate is around 75/min, corresponding to a 4:1 conduction (Miller: Miller’s Anesthesia, ed 8, p 1441).
922. A 71-year-old man is undergoing revascularization of three coronary vessels on cardiopulmonary bypass at 28° C. After the last graft is sewn into the aorta, the arterial pressure measured from a left radial artery is 47 mm Hg and the pulmonary artery (PA) pressure is 6 mm Hg. Thirty minutes later, the arterial pressure is 52 mm Hg and PA pressure is 31 mm Hg. The MOST likely explanation for this is A. Malposition of the aortic cannula B. Malposition of the venous cannula C. Faulty ventricular venting D. PA catheter migration
922. (D) During cardiopulmonary bypass, it is common for a PA catheter to migrate distally 3 to 5 cm into the PA. In fact, PA catheter migration during cardiopulmonary bypass is so common that withdrawing the catheter 3 to 5 cm before the initiation of cardiopulmonary bypass may be routinely indicated. Distal catheter migration into a wedge position is often detected by noting an increase in the measured PA pressure. PA catheter migration during cardiopulmonary bypass has been implicated in cases of PA rupture. Although catheter migration is the most likely explanation for a rise in PA pressure during cardiopulmonary bypass, the anesthesiologist must also consider inadequate ventricular venting as a potential cause of increasing PA pressures during cardiopulmonary bypass, particularly if the PA pressure does not decline after withdrawal of the PA catheter from a presumed wedge position. Ventricular distention during cardiopulmonary bypass is detrimental because it can increase myocardial oxygen demand at a time when there is no coronary blood flow. Malposition of the aortic cannula may result in unilateral facial blanching. Malposition of the venous cannula may result in facial or scleral edema or may manifest as poor blood return to the cardiopulmonary bypass circuit (Barash: Clinical Anesthesia, ed 7, p 1095).
923. A 78-year-old patient is anesthetized for right hemicolectomy with isoflurane and nitrous oxide. Vecuronium is administered to facilitate muscle relaxation. At the end of the operation, the neuromuscular blockade is reversed with neostigmine 4 mg and glycopyrrolate 0.8 mg. The rhythm below is noted shortly after administration of these drugs. The patient’s blood pressure is 90/60. The MOST appropriate course of action at this point is A. DC cardioversion B. Isoproterenol drip C. Atropine D. Transcutaneous pacemaker
923. (C) Anticholinesterase drugs may have significant cholinergic side effects, including sinoatrial and atrioventricular node slowing, bronchoconstriction, and peristalsis. There is a high incidence of transient cardiac dysrhythmias after administration of these drugs. The cardiac effects vary from clinically unimportant atrial and junctional bradydysrhythmias, ectopic ventricular foci, to clinically important dysrhythmias such as high-grade heart block, including complete heart block and cardiac arrest. The rhythm strip in this question is that of a low-grade heart block with a junctional rhythm. The most appropriate treatment of this rhythm is administration of atropine (Butterworth: Morgan & Mikhail’s Clinical Anesthesiology, ed 5, pp 224–228).
924. While on cardiopulmonary bypass during elective coronary artery revascularization, the patient is noted to have bulging sclerae. Mean arterial pressure is 50 mm Hg, temperature is 28° C, and there is no ECG activity. The MOST appropriate action to take at this time is to A. Administer mannitol, 50 g IV B. Decrease the cardiac index C. Check the position of the aortic cannula D. Check the position of the venous return cannula 260 Part 2 Clinical Sciences
924. (D) Incorrect positioning of the aortic perfusion and venous return cannulae are possible complications associated with cardiopulmonary bypass. Improper positioning of the aortic cannula would tend to result in unilateral facial blanching, whereas facial edema (e.g., bulging sclerae) reflects venous congestion and may be caused by improper positioning of the venous return cannula. Incorrect positioning of the venous return cannula can occur when the cannula is inserted too far into the superior vena cava, which causes obstruction of the right innominate vein. If the venous cannula is inserted too far into the inferior vena cava, venous return from the lower regions of the body can be impaired and abdominal distention can occur. If this happens, the vena caval cannula should be withdrawn to a more proximal position, and the adequacy of the venous return from the patient to the cardiopulmonary bypass machine should be confirmed. A properly positioned venous return cannula will bleed back with nonpulsatile flow when the proximal end is lowered below the patient (Miller: Miller’s Anesthesia, ed 8, pp 2035–2036).
925. Which of the following correctly describes the effect of transposition of the great vessels on the rate of induction of anesthesia? A. Inhalation induction is faster than normal; intravenous induction is slower than normal B. Inhalation induction is slower than normal; intravenous induction is faster than normal C. Both inhalation and intravenous induction are faster than normal D. Both inhalation and intravenous induction are slower than normal
925. (B) Transposition of the great vessels is a congenital cardiac defect that results from failure of the truncus arteriosus to rotate during organogenesis such that the aorta arises from the right ventricle and the PA arises from the left ventricle. As a result, the left and right ventricles are not connected in series and the pulmonary and systemic circulations function independently. This results in profound arterial hypoxemia; survival is not possible unless there is a concomitant defect that allows for intermixing of blood between the two circulations. Induction of anesthesia with volatile anesthetics will be delayed because minimal portions of inhaled drugs will reach the systemic circulation. In contrast, anesthetic drugs that are administered intravenously will be distributed with minimal dilution to the brain; therefore, doses and rates of injection should be reduced in these patients (Butterworth: Morgan & Mikhail’s Clinical Anesthesiology, ed 5, p 427). 270 Part 2 Clinical Sciences
926. Anastomosis of the right atrium to the PA (Fontan procedure) is a useful surgical treatment for each of the following congenital cardiac defects EXCEPT A. Tricuspid atresia B. Hypoplastic left heart syndrome C. Pulmonary valve stenosis D. Truncus arteriosus
926. (D) The Fontan procedure (usually modified Fontan) is an anastomosis of the right atrial appendage to the PA. This procedure is most frequently performed to treat congenital cardiac defects, which decrease PA blood flow (e.g., pulmonary atresia and stenosis, and tricuspid atresia). The Fontan procedure is also used to increase pulmonary blood flow when it is necessary to surgically convert the right ventricle to a systemic ventricle (e.g., hypoplastic left heart syndrome). Truncus arteriosus occurs when a single arterial trunk, which overrides both ventricles (which are connected via a ventricular septal defect), gives rise to both the aorta and PA. Surgical treatment of this defect includes banding of the right and left pulmonary arteries and enclosure of the associated ventricular septal defect (Miller: Miller’s Anesthesia, ed 8, p 2809).
927. By what percentage is tissue metabolic rate reduced during cardiopulmonary bypass at 30° C? A. 10% B. 25% C. 50% D. 75%
927. (C) For each degree Celsius body temperature is lowered, tissue metabolic rate declines approximately 5% to 8%. A core temperature of 28° to 30° C would correspond roughly to a 50% reduction in metabolic rate (Barash: Clinical Anesthesia, ed 7, pp 1092–1093).
928. Effective inflation of an intra-aortic balloon catheter should occur at which of the following times? A. Immediately after P wave on ECG B. Immediately after closure of aortic valve C. During opening of the aortic valve D. During systolic upstroke on arterial tracing
928. (B) By deflating just before ventricular systole, an intra-aortic balloon pump (IABP) is designed to reduce aortic pressure and afterload, thereby enhancing left ventricular ejection and reducing wall tension and oxygen consumption. By inflating in diastole, just after closure of the aortic valve, diastolic aortic pressure and coronary blood flow are increased. Thus, proper timing of inflation and deflation is crucial to correct functioning of an IABP. The P wave on the ECG is a late diastolic event, and inflating the IABP just after the P wave would minimize augmentation of diastolic coronary blood flow. In addition, inflation of the device that late in diastole would risk having the balloon inflated during ventricular systole, which would dramatically increase ventricular afterload and worsen the myocardial oxygen supply and demand balance. Similarly, the midpoint of the QRS complex represents the electrical activation of the ventricles, which heralds the end of ventricular diastole, a time when the balloon should be deflating before ventricular ejection (Barash: Clinical Anesthesia, ed 7, pp 1102–1103).
929. Afterload reduction is beneficial during anesthesia for noncardiac surgery in patients with each of the following conditions EXCEPT A. Aortic insufficiency B. Patent ductus arteriosus C. Tetralogy of Fallot D. Congestive heart failure
929. (C) Afterload reduction during anesthesia is beneficial in all of the conditions listed in this question except tetralogy of Fallot. In tetralogy of Fallot, blood is shunted through a ventricular septal defect from the pulmonary circulation to the systemic circulation because of right ventricular outflow obstruction. A decrease in systemic vascular resistance would augment this right-to-left shunt through the ventricular septal defect, which would reduce pulmonary vascular blood flow and exacerbate systemic hypoxemia (Butterworth: Morgan & Mikhail’s Clinical Anesthesiology, ed 5, pp 426–427).
930. Administration of protamine to a patient who has not received heparin can result in A. Anticoagulation B. Hypercoagulation C. Profound bradycardia D. Hypertension
930. (A) Protamine is a basic compound isolated from the sperm of certain fish species and is a specific antagonist of heparin. The dose of protamine is 1.3 mg for each 100 units of heparin. If protamine is administered to a patient who has not received heparin, it can bind to platelets and soluble coagulation factors, producing an anticoagulant effect. There is no evidence that protamine has negative inotropic or chronotropic properties. Some persons (e.g., diabetics taking NPH insulin) may be allergic to protamine. Hypotension may occur when protamine is administered rapidly because it induces histamine release from mast cells (Kaplan: Kaplan’s Cardiac Anesthesia, ed 6, p 963).
931. The primary determinants of myocardial O2 consumption, from most to least important, are A. Preload > afterload > heart rate B. Heart rate > preload > afterload C. Afterload > preload > heart rate D. Heart rate > afterload > preload
931. (D) The primary goal in the anesthetic management of patients with coronary artery disease is to maintain the balance between myocardial O2 supply and demand. Myocardial O2 consumption (i.e., myocardial O2 demand) is determined by three factors: myocardial wall tension, heart rate, and myocardial contractile state. Myocardial wall tension is directly related to the end-diastolic ventricular pressure or volume (preload) and systemic vascular resistance (afterload). In general, myocardial work in the form of increased heart rate results in the greatest increase in myocardial O2 consumption. Also, for a given increase in myocardial work, the increase in myocardial O2 consumption is much less with volume work (preload) than with pressure work (afterload) (Stoelting: Pharmacology and Physiology in Anesthetic Practice, ed 4, p 754).
932. Cardiac tamponade is associated with A. Pulsus alternans B. Pulsus tardus C. Pulsus parvus D. Pulsus paradoxus
932. (D) Pulsus paradoxus describes an inspiratory fall in systolic arterial blood pressure of greater than 10 mm Hg often seen in cardiac tamponade. This inspiratory decline in systolic blood pressure represents an exaggeration of the normal small drop in blood pressure seen with inspiration in spontaneously breathing patients. In cardiac tamponade, ventricular filling is limited by the presence of blood, thrombus, or other material in the pericardial space. During inspiration in the spontaneously Cardiovascular Physiology and Anesthesia 271 breathing patient, negative intrathoracic pressure enhances filling of the right ventricle. Because total cardiac volume is limited by the pressurized pericardium in tamponade cases, as the right ventricle fills with inspiration, left ventricular preload and blood pressure decline. Pulsus paradoxus is occasionally seen in cases of severe airway obstruction and right ventricular infarction. Pulsus parvus and pulsus tardus describe, respectively, the diminished pulse wave and delayed upstroke in patients with aortic stenosis. Pulsus alternans describes alternating smaller and larger pulse waves, a condition sometimes seen in patients with severe left ventricular dysfunction. A bisferiens pulse is a pulse waveform with two systolic peaks seen in cases of significant aortic valvular regurgitation (Miller: Miller’s Anesthesia, ed 8, pp 2073–2074).
933. Which of the following drugs should NOT be administered via an endotracheal tube? A. Lidocaine B. NaHCO3 C. Atropine D. Naloxone
933. (B) The word ALONE is an acronym for five drugs that can be administered down the endotracheal tube: Atropine, Lidocaine, Oxygen, Naloxone, Epinephrine. In addition, vasopressin may be administered down the endotracheal tube. Although preoperatively clear antacids (e.g., Bicitra) have been administered orally to raise gastric pH in patients at high risk for aspiration with induction of general anesthesia to decrease the severity of acid aspiration, should aspiration occur, bicarbonate should not be instilled down the endotracheal tube because it would worsen the aspiration and might produce an alkaline burn to the lung (Barash: Clinical Anesthesia, ed 7, pp 1682–1683).
934. The mean arterial pressure in a patient with a blood pressure of 180/60 mm Hg is A. 90 mm Hg B. 100 mm Hg C. 110 mm Hg D. 120 mm Hg
934. (B) Mean arterial pressure can be calculated using the following formula: MAP = BPD + 1/3 (BPS − BPD) Where MAP (mm Hg) is the mean arterial pressure, BPD (mm Hg) is the diastolic blood pressure, and BPS (mm Hg) is the systolic blood pressure (Barash: Clinical Anesthesia, ed 7, p 708).
935. Hypothyroidism and hyperthyroidism could develop in patients receiving which of the following antidysrhythmic drugs? A. Amiodarone B. Verapamil C. Procainamide D. Lidocaine
935. (A) Amiodarone is a benzofurane derivative with a chemical structure similar to that of thyroxine, which accounts for its ability to cause either hypothyroidism or hyperthyroidism. Altered thyroid function occurs in 2% to 4% of patients when amiodarone is administered over a long period. Amiodarone prolongs the duration of the action potential of both atrial and ventricular muscle without altering the resting membrane potential. This accounts for its ability to depress sinoatrial and atrioventricular node function. Thus, amiodarone is effective pharmacologic therapy for both recurrent supraventricular and ventricular tachydysrhythmias. In patients with WPW syndrome, amiodarone increases the refractory period of the accessory pathway. Atropine-resistant bradycardia and hypotension may occur during general anesthesia because of the significant antiadrenergic effect of amiodarone. Should this occur, isoproterenol should be administered or a temporary artificial cardiac pacemaker should be inserted (Miller: Miller’s Anesthesia, ed 8, p 1175).
936. Calculate the systemic vascular resistance (in dyne-sec/ cm5) from the following data: cardiac output 5.0 L/ min, central venous pressure 8 mm Hg, mean arterial blood pressure 86 mm Hg, mean pulmonary arterial blood pressure 20 mm Hg, pulmonary capillary wedge pressure 9 mm Hg, heart rate 85 beats/min, patient weight 100 kg. A. 750 B. 1000 C. 1250 D. 1500
936. (C) Systemic vascular resistance can be calculated using the following formula: SVR = (MAP − CVP)/CO × 80 where SVR is the systemic vascular resistance, MAP (mm Hg) is the mean arterial pressure, CVP (mm Hg) is the central venous pressure, CO (L/min) is the cardiac output, and 80 is a factor to convert Wood units to dyne-sec/cm5. Calculation of SVR from the data in this question is as follows: SVR = (86 − 8) /5 × 80 = 1248 dyne-sec/cm5 (Miller: Miller’s Anesthesia, ed 8, p 1387)
937. Which of the following is NOT included in tetralogy of Fallot? A. Patent ductus arteriosus B. Right ventricular hypertrophy C. Ventricular septal defect D. Overriding aorta
937. (A) Tetralogy of Fallot is the most common congenital heart defect associated with a right-to-left intracardiac shunt. This congenital defect is characterized by a tetrad of congenital cardiac anomalies, including a ventricular septal defect, an aorta that overrides the ventricular septal defect, obstruction of the PA outflow tract, and right ventricular hypertrophy. The ventricular septal defect is typically large and single, an infundibular PA stenosis is usually prominent, and the distal PA may be hypoplastic or even absent. Although many patients with tetralogy of Fallot have a patent ductus arteriosus, this is not included in the definition (Hines: Stoelting’s Anesthesia and Co-Existing Disease, ed 6, pp 56–57).
938. A 65-year-old female patient with sepsis is undergoing an emergency exploratory laparotomy. After induction of anesthesia and tracheal intubation, the patient’s blood pressure is noted to be 65 systolic with a heart rate of 120 beats/min. Cardiac output determined by a thermodilution PA catheter is 13 L/min. Of the following vasopressors the LEAST appropriate choice would be A. Dobutamine B. Vasopressin C. Norepinephrine D. Phenylephrine Cardiovascular Physiology and Anesthesia 261
938. (A) The etiology of hypotension can be placed into two broad categories: decreased cardiac output and decreased systemic vascular resistance, or both. In this case, cardiac output is greater than normal, as one often sees in early sepsis. Treatment of this hypotension should be carried out with pharmacologic 272 Part 2 Clinical Sciences agents with strong α-agonist properties. Of the choices in this question, phenylephrine is the only drug that is a pure α-agonist. Dopamine in high doses has strong activity but significant β1 activity and some β2 activity as well. Norepinephrine likewise possesses strong α activity with some β1 activity. Vasopressin is a potent vasoconstrictor useful in the management of septic shock. Any of the aforementioned pharmacologic agents could be used to support pressure in patients with sepsis in conjunction with definitive treatment for the septic source. Because dobutamine is predominantly a β1 agonist, it would be an extremely poor choice for a patient with a high cardiac output in the face of a low systemic vascular resistance (Barash: Clinical Anesthesia, ed 7, p 1592).
939. A 61-year-old man develops this rhythm after thoracotomy and right upper lobe resection. Cardioversion is planned, the image below is taken from the biphasic defibrillator, and the device is set to deliver 200 J. The MOST appropriate step would be A. Select a different lead B. Deliver shock C. Reduce energy and deliver shock D. Set to asynchronous mode and shock
939. (A) The rhythm depicted is atrial flutter with 4:1 heart block. The atrial flutter waves (F waves) are occurring at approximately 300 per minute and the ventricular rate is approximately 75 per minute. The screen shows arrows indicating when the synchronous shock would be given. Ideally, the shock should occur during ventricular contraction (depolarization), that is, with QRS complex. This will effectively “reset” the heart and allow the normal P wave to be manifested. The current display shows the shock synchronized with the flutter waves. Shocking on a flutter wave that is not occurring during ventricular repolarization would not be a problem, but a shock during repolarization would be tantamount to an R on T phenomenon and might induce ventricular tachycardia or even ventricular fibrillation. It would be far preferable to change to a different lead in which the R wave is synchronized with the QRS and then apply the shock. Most atrial flutter can be terminated with a setting as low as 50 J. Delivering 200 J with the first attempt to convert to NSR is unwarranted in most cases. Delivering an asynchronous shock is ill advised since it too could induce an unstable rhythm through the R on T mechanism (Miller: Miller’s Anesthesia, ed 8, p 1441; Hines: Stoelting’s Anesthesia and Co-Existing Disease, ed 6, pp 79–81).
940. The MOST important pathophysiologic difference between pericardial effusion and cardiac tamponade is A. Type of fluid (e.g., transudate, exudate, blood) B. Quantity of fluid C. Pressure D. Inflammation
940. (C) Patients with pericardial disease may develop an increase in the amount of fluid (normally 15-30 mL) in the pericardial sac. Normally the pressure in the pericardial sac is 5 mm Hg less than the CVP and approximates pleural pressure. When the fluid pressure becomes elevated and impairs cardiac filling, cardiac tamponade is said to develop. If the amount of fluid increases acutely, as little as 100 mL may cause tamponade. If the increase in fluid develops slowly, an increase in volume of 2 L may develop before tamponade is produced. The type of fluid does not affect pressure. Inflammation may cause an increase in fluid, but it is the pressure that causes the tamponade (Hines: Stoelting’s Anesthesia and Co-Existing Disease, ed 6, pp 145–146; Miller: Miller’s Anesthesia, ed 8, pp 2073–2074).
941. A healthy 59-year-old, 60-kg woman with a normal preoperative ECG develops wide complex tachycardia under general anesthesia for breast biopsy. Blood pressure is 81/47 mm Hg, and heart rate is 220 beats/min and regular. The MOST appropriate therapy would be A. Electrical cardioversion B. Administration of lidocaine, 60 mg IV C. Administration of procainamide, 20 mg/min IV D. Administration of amiodarone, 300 mg IV
941. (A) An unstable patient with a wide complex tachycardia is presumed to be ventricular tachycardia (VT), and this rhythm represents a medical emergency that requires immediate synchronized cardioversion (ECC Committee: 2005 American Heart Association Guidelines for Cardiopulmonary Resuscitation and Emergency Cardiovascular Care, Circulation 112:IV69–IV73, 2005; Miller: Miller’s Anesthesia, ed 8, p 3191).
942. Although β-adrenergic receptor blockade is the best treatment for reentrant tachydysrhythmia associated with Romano-Ward syndrome, these dysrhythmias can also be effectively treated with A. Lidocaine B. Procainamide C. Left stellate ganglion blockade D. Right stellate ganglion blockade
942. (C) Romano-Ward syndrome is a rare congenital abnormality characterized by prolonged QT intervals on the ECG. Jervell-Lange-Nielsen syndrome is a congenital syndrome characterized by prolonged QT intervals on the ECG in association with congenital deafness. An imbalance between the right and left sides of the sympathetic nervous system may play a role in the etiology of these syndromes. This imbalance can be temporarily abolished with a left stellate ganglion block, which shortens the QT intervals. If this is successful, surgical ganglionectomy may be performed as permanent treatment (Hines: Stoelting’s Anesthesia and Co-Existing Disease, ed 6, p 86).
943. A 64-year-old patient with an axial flow left ventricular assist device (e.g., HeartMate II, Jarvik 2000) is scheduled for laparoscopic cholecystectomy under general anesthesia. Monitoring which of the following parameters is likely to be difficult in this patient? A. Blood pressure with blood pressure cuff B. Blood pressure with arterial line C. PA pressure with PA catheter D. Temperature with esophageal temperature probe
943. (A) The use of mechanical circulatory support is becoming more frequent because of advances in technology and a relative scarcity of organs available for transplant. Mechanical circulatory support can be used as bridge therapy for patients awaiting cardiac transplantation or as a bridge to recovery from a viral cardiomyopathy or from cardiogenic shock after myocardial infarction. In other patients, it can be destination therapy. Currently, the HeartMate VE (vented electrical) is the only mechanical device approved for destination therapy in the United States. Various versions of these devices can be used to support the right (not approved for destination therapy), the left, or both ventricles. Axial ( continuous) flow is nonpulsatile and nonphysiologic. These pumps are connected in parallel to the heart. Specifically, on the left side, blood is taken from the apex of the heart and returned to circulation via the aorta. In this configuration, little or no blood exits the aortic valve during systole. Measuring blood pressure with a cuff is not accurate in most patients and may be impossible. Pulse oximeters do Cardiovascular Physiology and Anesthesia 273 work with some patients, but this, too, requires pulsatile flow. Measurement of blood pressure with an arterial line is easily done, just as it is in patients on cardiopulmonary bypass undergoing open-heart operations (Miller: Miller’s Anesthesia, ed 8, pp 2066–2067).
944. In a normal person, what percentage of the cardiac output is dependent on the “atrial kick”? A. 25% B. 35% C. 45% D. 55%
944. (A) In a normal heart, approximately 15% to 20% of the cardiac output is produced by atrial systole “atrial kick.” In pathologic conditions, such as aortic stenosis, the “atrial kick” may contribute more substantially to cardiac output (Kaplan: Kaplan’s Cardiac Anesthesia, ed 6, p 578).
945. This arterial waveform is consistent with A. Aortic regurgitation B. Aortic stenosis C. Cardiac tamponade D. Hypovolemia
945. (A) The figure in this case shows a bisferiens pulse, recognized by its two systolic peaks. A bisferiens pulse can be seen in patients with significant aortic regurgitation. In aortic regurgitation, the left ventricle ejects a large volume of blood in systole with a rapid diastolic runoff as blood flows both to the periphery and back into the left ventricle. The first systolic peak of the bisferiens pulse represents the wave of blood ejected from the left ventricle. The second systolic peak represents a reflected pressure wave from the periphery. In contrast, patients with aortic stenosis display a delayed pulse wave with a diminished upstroke (pulsus tardus and pulsus parvus), whereas patients with cardiac tamponade show an exaggerated inspiratory decline in systolic blood pressure (pulsus paradoxus). Patients with hypovolemia may demonstrate systolic blood pressure variation, particularly during mechanical ventilation (Miller: Miller’s Anesthesia, ed 8, p 1358).
946. A 1-year-old child with tetralogy of Fallot is to undergo elective repair of a left inguinal hernia under general anesthesia. Which of the following anesthetics would provide the MOST stable hemodynamics in this patient? A. Sevoflurane and N2O B. Fentanyl and N2O C. Desflurane and oxygen D. Ketamine 262 Part 2 Clinical Sciences
946. (D) In patients with tetralogy of Fallot, it is important to maintain systemic vascular resistance to reduce the magnitude of the right-to-left intracardiac shunt. Therefore, induction of anesthesia in these patients is best accomplished with ketamine 3 to 4 mg/kg IM or 1 to 2 mg/kg IV. Remember that with right-to-left shunts, IV medications work more rapidly. Induction of anesthesia with a volatile anesthetic such as sevoflurane may be used, but careful monitoring of systemic oxygenation is needed because any decrease in systemic blood pressure would increase the right-to-left shunt (and would decrease the oxygen saturation). Ketamine will usually improve arterial oxygenation, which reflects increased pulmonary blood flow due to ketamine-induced increases in systemic vascular resistance (Butterworth: Morgan & Mikhail’s Clinical Anesthesiology, ed 5, pp 426–427).
947. The left ventricular pressure-volume loop shown in the figure depicts A. Mitral stenosis B. Mitral regurgitation C. Aortic stenosis D. Acute aortic insufficiency
947. (A) Mitral stenosis in adults occurs almost exclusively in individuals who had rheumatic fever during childhood. Mitral stenosis causes pathophysiologic changes both proximal and distal to the abnormal valve. In general, the left ventricle is “protected” or unloaded; that is, it is not exposed to excessive volume or pressure loads and therefore is rarely associated with abnormalities in left-sided myocardial contractility. In contrast, proximal to the valve, a diastolic pressure gradient develops between the left atrium and left ventricle in order to force blood across the stenotic valve orifice, which results in elevated left atrial pressures and decreased left atrial compliance and function. The elevated left atrial pressures are reflected back into the pulmonary vascular system, causing an increase in pulmonary vascular resistance and eventually poor right ventricular function. The left ventricular pressure-volume loop in patients with mitral stenosis demonstrates low-to-normal left ventricular end-diastolic volumes and pressures and a corresponding reduction in stroke volume (Miller: Miller’s Anesthesia, ed 8, pp 2050–2052).
948. A 54-year-old patient is undergoing a three-vessel coronary artery bypass graft under general anesthesia. After induction, the pulmonary capillary wedge pressure is 15 mm Hg and PA pressures are 26/13 mm Hg. Suddenly, new 30-mm Hg V waves appear on the monitor screen. Systemic blood pressure is 120/70 mm Hg, heart rate is 75 beats/min, and PA pressure is 50/35 mm Hg. Which of the following drugs should be administered to the patient? A. Nitroglycerin B. Nitroprusside C. Esmolol D. Dobutamine
948. (A) Ischemia of the posterior wall of the left ventricle and posterior leaflet of the mitral valve can cause prolapse of the posterior leaflet and retrograde blood flow into the left atrium during systole. This can be manifested as V (ventricular) waves on the pulmonary capillary wedge pressure tracing even before ST segment depression can be seen on the ECG (Miller: Miller’s Anesthesia, ed 8, p 1377).
949. A 62-year-old patient scheduled for elective repair of an abdominal aortic aneurysm develops a wide complex regular tachycardia (heart rate 150 beats/min) during induction of anesthesia. Blood pressure is 110/78 mm Hg. Which of the following drugs would be MOST useful in the management of this dysrhythmia? A. Esmolol, 35 mg IV B. Amiodarone, 150 mg IV over 10 minutes C. Adenosine, 6 mg rapidly over 3 seconds D. Verapamil, 5 to 10 mg IV
949. (B) The patient described in this question has a wide complex tachycardia of undetermined origin. As this patient appears to be hemodynamically stable and has an uncertain rhythm, amiodarone 150 mg IV over 10 minutes, repeated as needed to a maximum dose of 2.2 g IV over 24 hours is recommended (Miller: Miller’s Anesthesia, ed 8, pp 1391–1393).
950. Under maximum stress, how much cortisol is produced per day? A. 50 mg B. 150 mg C. 250 mg D. 350 mg
950. (B) The daily production of cortisol under normal circumstances is approximately 15 to 20 mg. Under maximum stress, daily cortisol production can increase to 75 to 150 mg/day yielding a plasma cortisol level of 30 to 50 μg/dL (Hemmings: Pharmacology and Physiology for Anesthesia, ed 1, p 548; Hines: Stoelting’s Anesthesia and Co-Existing Disease, ed 6, p 396). 274 Part 2 Clinical Sciences
951. With pacemakers, the concept of upper tracking rate (UTR) is relevant with which type(s) of device? A. VDD B. DDI C. AAI D. All of the above
951. (A) The generic pacemaker code NASPE/BPEG (North American Society of Pacing and Electrophysiology/ British Pacing and Electrophysiology Group) has five positions for pacemaker designation: I = paced chamber(s), II = sensed chamber(s), III = response(s) to sensing, IV = programmability, V = multisite pacing. UTR is applicable only to devices programmed to pace the ventricle based on depolarization (tracking) of the atrium, i.e., a triggering function. The purpose of UTR is to prevent a rapid (paced) ventricular rate in response to a rapid atrial rate such as paroxysmal supraventricular tachycardia (PSVT), atrial fibrillation, or atrial flutter. When the sensed atrial depolarization exceeds the UTR, the pacemaker (depending on model) will switch to the DDI mode (atrial tachy response). This would effectively stop the rapid supraventricular impulses from driving the ventricles unless these impulses could cross the native AV node. With other models, exceeding the UTR will result in the pacemaker creating a type II heart block. This would modulate the number of atrial contractions that ultimately drive the ventricle. UTR is applicable only to DDD and VDD pacemakers. AAI does not require UTR because it (1) does not pace the ventricle and (2) responds only with inhibition, not triggering (Miller: Miller’s Anesthesia, ed 8, pp 1467–1476).
952. Calculate the cardiac output from the following data: patient weight 70 kg, hemoglobin concentration 10 mg/dL, arterial blood gases on 100% O2: Pao2 450 mm Hg, Paco2 32 mm Hg, pH 7.46, Sao2 99%. Mixed venous blood gases are: Pvo2 30 mm Hg, Paco2 45 mm Hg, pH 7.32, Svo2 60%. A. 1.5 L/min B. 2.5 L/min C. 3.5 L/min D. 4.5 L/min
952. (D) The Fick equation can be used to calculate cardiac output (˙Q ) if the patient’s O2 consumption (˙V O2 ), arterial O2 content (Cao2), and mixed venous O2 content (CvO2) are determined. The downfalls of this type of ˙Q measurement are threefold: (1) sampling and analysis errors in vo2, (2) changes in Q while samples are being taken, and (3) accurate determination of vo2 may be difficult because of cumbersome equipment. The Fick equation is as follows: Q˙ = V˙ O2 (CaO2 − CvO2) × 10 ˙VO2 = 250 mL/min (� 4 mL/kg) CaO2 = 1.36 × hemoglobinconcentration × SaO2 + (0.003× PaO2 ) 1.36 × 10 mg/dL × 0.99 13.5 mL O2/dL of blood CvO2 = 1.36 × hemoglobinconcentration × Svo2 + (0.003× PvO2 ) 1.36 × 10 mg/dL × 0.60 8.16 mL O2/dL of blood Q˙ = 250 mL/min (13.5 mL/dL − 8.16 mL/dL)×10* = 250/53.4 = 4.68 L/min *The factor 10 converts O2 content to mL O2/L of blood (instead of mL O2/dL of blood) (Miller: Miller’s Anesthesia, ed 8, pp 478–479).
953. Normal resting myocardial O2 consumption is A. 2.0 mL/100 g/min B. 3.5 mL/100 g/min C. 8 mL/100 g/min D. 15 mL/100 g/min
953. (C) Myocardial preservation is achieved during cardiopulmonary bypass primarily by infusing cold (4° C) cardioplegia solutions containing potassium chloride 20 mEq/L. This rapidly produces hypothermia of the cardiac muscle and a flaccid myocardium. In the normal contracting muscle at 37° C, myocardial O2 consumption is approximately 8 to 10 mL/100 g/min. This is reduced in the fibrillating heart at 22° C to approximately 2 mL/100 g/min. Myocardial O2 consumption of the electromechanically quiescent heart at 22° C is less than 0.3 mL/100 g/min (Hemmings: Pharmacology and Physiology for Anesthesia, ed 1, p 383; Miller: Miller’s Anesthesia, ed 8, p 2038).
954. A 22-year-old man with hypertrophic cardiomyopathy (HOCM) is undergoing an elective cholecystectomy under general anesthesia. Immediately after induction with propofol, 2.5 mg/kg IV, the arterial blood pressure decreases from 140/82 to 70/40 mm Hg. What would be the most appropriate drug for treatment of hypotension in this patient? A. Ephedrine B. Epinephrine C. Isoproterenol D. Phenylephrine Cardiovascular Physiology and Anesthesia 263
954. (D) All of the drugs listed in this question except phenylephrine will increase the inotropic state of the myocardium, which can increase left ventricular outflow obstruction and decrease cardiac output. Phenylephrine, because it is a pure α-adrenergic receptor agonist, has minimal Cardiovascular Physiology and Anesthesia 275
955. A 65-year-old patient with moderate aortic stenosis develops a sudden increase in heart rate during an appendectomy under general anesthesia. The ventricular rate is 190 beats/min and is irregularly irregular, arterial blood pressure is 70/45 mm Hg, and there is 2-mm ST segment depression in lead V5 of the ECG. Which of the following would be the MOST appropriate treatment for myocardial ischemia in this patient? A. Electrical cardioversion B. Esmolol C. Phenylephrine D. Verapamil
955. (A) The classic signs and symptoms of critical aortic stenosis (angina, syncope, and congestive heart failure) are related primarily to an increase in left ventricular systolic pressure, which is necessary to maintain forward stroke volume. These elevated pressures cause concentric left ventricular hypertrophy. With severe disease, the left ventricular chamber becomes dilated and myocardial contractility diminishes. The primary goals in the anesthetic management of such patients undergoing noncardiac surgery are to maintain normal sinus rhythm and avoid prolonged alterations in heart rate (especially tachycardia), systemic vascular resistance, and intravascular fluid volume. Supraventricular tachycardia (especially new-onset atrial fibrillation) should be terminated promptly by electrical cardioversion in this patient because of concomitant hypotension and myocardial ischemia (Miller: Miller’s Anesthesia, ed 8, pp 3191–3193).
956. After emergency repair of a ruptured abdominal aortic aneurysm, a 68-year-old patient is mechanically ventilated in the intensive care unit with 20 cm H2O of positive end-expiratory pressure (PEEP) for 3 days. Sodium nitroprusside has been infused at a rate of 1.5 μg/kg/min for 48 hours to control hypertension. Suddenly, the systemic blood pressure falls from 130/70 to 50 mm Hg systolic and the Sao2 drops to 75%. The MOST likely cause of this scenario is A. Cyanide toxicity B. Acute myocardial infarction C. Tension pneumothorax D. Hyperventilation
956. (C) PEEP is produced by the application of positive pressure to the exhalation valve of the mechanical ventilator at the conclusion of the expiratory phase. It is often used to increase arterial oxygenation when Fio2 exceeds 0.50 to reduce the hazard of O2 toxicity. PEEP increases lung compliance and FRC by expanding previously collapsed but perfused alveoli, thus improving ventilation/perfusion matching and reducing the magnitude of the right-to-left transpulmonary shunt. There are, however, a number of potential hazards associated with the use of PEEP. These include decreased cardiac output, pulmonary barotrauma (i.e., tension pneumothorax), increased extravascular lung water, and redistribution of pulmonary blood flow. Barotrauma, such as pneumothorax, pneumomediastinum, and subcutaneous emphysema, occurs as a result of overdistention of alveoli by PEEP. Pulmonary barotrauma should be suspected when there is abrupt deterioration of arterial oxygenation and cardiovascular function during mechanical ventilation with PEEP. If barotrauma is suspected a chest x-ray film should be obtained, and if a tension pneumothorax is present a chest tube should be placed in the involved chest cavity (Butterworth: Morgan & Mikhail’s Clinical Anesthesiology, ed 5, pp 1298–1300).
957. Normal resting coronary artery blood flow is A. 10 mL/100 g/min B. 40 mL/100 g/min C. 75 mL/100 g/min D. 120 mL/100 g/min
957. (C) Resting coronary artery blood flow is approximately 225 to 250 mL/min or about 75 mL/100 g/min, or approximately 4% to 5% of the cardiac output. Resting myocardial O2 consumption is 8 to 10 mL/100 g/ min, or approximately 10% of the total body consumption of O2 (Barash: Clinical Anesthesia, ed 7, p 244).
958. Each of the following is associated with an increased incidence of PA rupture in patients with PA catheters EXCEPT A. Hypothermia B. Presence of PA atheromas C. Old age D. Anticoagulation
958. (B) PA rupture is a disastrous but fortunately rare complication associated with the use of PA catheters. The hallmark of PA rupture is hemoptysis, which may be minimal or copious. Efforts should be made to separate the lungs. This can be achieved by endobronchial intubation with a double-lumen endotracheal tube. The presence of atheromas in the PA is not associated with an increased risk of PA rupture. Atheromatous changes are usually minimal or absent in the middle and distal portions of the PA (i.e., in the segments where the tip of the PA catheter typically resides) (Miller: Miller’s Anesthesia, ed 8, pp 1372–1373).
959. Allergic reactions to protamine can occur with each of the following EXCEPT A. Diabetes treated with NPH insulin B. Diabetes treated with regular insulin C. Diabetes treated with PZI insulin D. Previous vasectomy
959. (B) Anaphylactic and anaphylactoid reactions to protamine occur in less than 5% of all allergic reactions during anesthesia, and when they occur, usually do so within 5 to 10 minutes of exposure. These reactions can occur in patients who have been exposed to protamine (e.g., diabetics taking NPH or PZI insulin, both of which contain protamine as a protein modifier; regular insulin does not contain protamine). Since protamine is derived from salmon sperm, patients with seafood allergies as well as men who have had a vasectomy (who may develop circulating antibodies to spermatozoa) may also develop a reaction. The likelihood of reactions may be reduced with prior administration of H1 blockers, H2 blockers, and corticosteroids. Protamine should be avoided in patients who have a history of previous anaphylactic reactions to protamine (Hines: Stoelting’s Anesthesia and Co-Existing Disease, ed 6, p 528).
960. A 66-year-old patient is undergoing a three-vessel coronary artery bypass operation. Anticoagulation is achieved with 20,000 units of heparin. How much protamine should be administered to this patient to completely reverse the heparin after cardiopulmonary bypass? A. 150 mg B. 250 mg C. 350 mg D. 450 mg
960. (B) Twenty thousand units of heparin are equal to 200 mg. Heparin is commonly neutralized by administration of 1.3 mg of protamine for each milligram of heparin. Protamine is a basic protein that combines to the acidic heparin molecule to produce an inactive complex that has no anticoagulant properties. The half-life of heparin is 1.5 hours at 37° C. At 25° C, metabolism of heparin is minimal (Miller: Miller’s Anesthesia, ed 8, p 2017).
961. The graph below represents A. Diastolic time (as percentage of cardiac cycle) as a function of heart rate B. Stroke volume as a function of end-diastolic pressure C. Cardiac index as a function of end-diastolic pressure D. Cardiac output as a function of ventricular end-diastolic volume
961. (A) Unlike most organs of the body where perfusion is continuous, coronary perfusion is somewhat intermittent. It is determined by the difference between aortic diastolic pressure and left and right ventricular end-diastolic pressures. During systole, left ventricular pressure increases to or above sys276 Part 2 Clinical Sciences temic arterial pressure, resulting in almost complete occlusion of the intramyocardial portions of the coronary arteries. Thus, perfusion of the left ventricular myocardium occurs almost entirely during diastole, resulting in a decrease in left ventricular coronary perfusion as heart rate increases. In contrast, the right ventricle is perfused during both systole and diastole, because right ventricular pressures remain less than that of the aorta. An increase in heart rate results in a relatively shorter diastolic period (Butterworth: Morgan & Mikhail’s Clinical Anesthesiology, ed 5, pp 362–365).
962. A 72-year-old woman is undergoing cardiopulmonary bypass for aortic and mitral valve replacement. The surgery is uneventful; however, in the intensive care unit, blood is noted to ooze from the PA catheter and venous access sites. Mediastinal chest tube output is 500 mL/hr. A thromboelastogram is obtained and shown in the figure. What is the MOST likely cause of profuse bleeding in this patient? A. Fibrinolysis B. Excess heparin C. Thrombocytopenia D. Factor VIII deficiency
962. (A) The thromboelastograph is a viscoelastometer that measures the viscoelastic properties of blood during clot formation. The coagulation variables measured from a thromboelastogram are (1) the R value (reaction time; normal value 7.5-15 minutes) and K value (normal 3-6 minutes), which reflects clot formation time; (2) MA (maximum amplitude; normal value 50-60 mm), which represents maximum clot strength; and (3) A60 (amplitude 60 minutes after the MA; normal value MA—5 mm), which represents the rate of clot destruction (i.e., fibrinolysis). The MA is determined by fibrinogen concentration, platelet count, and platelet function. The thromboelastogram depicted in the figure of this question is consistent with fibrinolysis (Miller: Miller’s Anesthesiology, ed 8, p 1878).
963. A 69-year-old man with an axial flow left ventricular assist device is anesthetized for kidney stone removal from the left ureter. The patient is “dry” and blood pressure falls precipitously to a mean pressure of 51 mm Hg with no pulsatility on the arterial tracing. In addition to a fluid bolus, each of the other interventions would be useful EXCEPT A. Increase pump speed from 7800 to 8500 rpm B. Ephedrine C. Phenylephrine D. Trendelenburg position 264 Part 2 Clinical Sciences
963. (A) Ventricular assist devices (VADs) are implanted in patients with end-stage heart failure in whom medical management has failed or is beginning to fail. VADs can be left sided only (LVAD), right sided only (RVAD), or biventricular (BiVAD). VADs may be implanted until the patient recovers (bridge to recovery), until the patient can receive a heart transplant (bridge to transplantation), or as the final method of treating heart failure (destination therapy). Patients can survive for long periods of time with LVAD therapy; the current record is just over 5 years. “Destination LVADs” have been implanted in patients ineligible for heart transplant, whose status improved to the extent they were subsequently reclassified and received heart transplantation. LVADs are in relatively widespread use, and patients are presenting to the operating room for other noncardiac-related operations. Treatment of hypotension may be a problem after induction of anesthesia. LVADs require adequate preload to function properly. The decrease in SVR as well as venodilation associated with induction and maintenance of general anesthesia can be treated in several ways. Phenylephrine and ephedrine are α1 agonists and increase SVR. Ephedrine may also increase inotropy and be beneficial on that basis in the face of right ventricular dysfunction. Fluids and Trendelenburg position are also likely to help raise the mean arterial pressure. An LVAD with inadequate preload will not perform better by increasing the rpm. Such an increase could simply make the device “suck down” and may actually worsen performance. The suck-down effect results in a completely empty left ventricle with myocardium being drawn over the inflow cannula. This greatly impairs preload to the LVAD and can result in hemodynamic collapse (Miller: Miller’s Anesthesia, ed 8, p 2067; Kaplan: Kaplan’s Cardiac Anesthesia, ed 6, pp 818–827).
964. The dose of adenosine necessary to convert paroxysmal supraventricular tachycardia to normal sinus rhythm should be initially reduced A. In patients receiving theophylline for chronic asthma B. In patients with a history of arterial thrombotic disease taking dipyridamole C. In patients with a history of chronic renal failure D. In chronic alcoholics
964. (B) Adenosine in doses of 6 mg IV (repeated if needed 1-2 minutes later with 12 mg) can be very effective in the treatment of supraventricular tachycardias, including those associated with WPW syndrome (unless atrial fibrillation [AF] with a wide complex WPW occurs, where adenosine may increase the heart rate [HR]). The drug is rapidly metabolized such that it is not influenced by liver or renal dysfunction. Its effects, however, can be markedly enhanced by drugs that interfere with nucleotide metabolism such as dipyridamole. Administration of the usual dose of adenosine to a patient receiving dipyridamole may result in asystole. If adenosine is used in patients receiving dipyridamole, or the patient has a central line, the initial dose is 3 mg. Methylxanthines, such as caffeine, theophylline, and amrinone, are competitive antagonists of this drug, and doses may need to be adjusted accordingly (Miller: Miller’s Anesthesia, ed 8, pp 3195–3197).
965. A 56-year-old male patient is anesthetized for elective coronary revascularization. A urinary catheter is placed after induction and coupled to a temperature transducer. A PA catheter is inserted, and the temperature probe on the distal portion of the catheter is also connected to a transducer. The reason for measuring the temperature of both the bladder and the blood in the pulmonary vasculature is A. Both are necessary for determining cardiac output by the thermodilution technique B. Bladder temperature is more accurate prebypass; PA catheter temperature is more accurate postbypass C. PA catheter temperature is more accurate prebypass; bladder temperature is more accurate postbypass D. It is helpful in determining the likelihood of recooling after discontinuation of cardiopulmonary bypass
965. (D) Temperature of the thermal compartment can be measured accurately in the PA, distal esophagus, tympanic membrane, or nasopharynx. These temperature monitoring sites are reliable, even during rapid thermal perturbations such as cardiopulmonary bypass. Other temperature sites, such as oral, axillary, rectal, and urinary bladder, will estimate core temperature reasonably accurately except during extreme thermal perturbations. During cardiac surgery, the temperature of the urinary bladder is usually equal to the PA when urine flow is high. However, it may be difficult to interpret urinary bladder temperature because it is strongly influenced by urine flow. The adequacy of rewarming after coronary artery bypass is thus best evaluated by considering both the core and urinary bladder temperatures (Stoelting: Pharmacology and Physiology in Anesthetic Practice, ed 4, p 694). Cardiovascular Physiology and Anesthesia 277
966. Which of the following would be the best intraoperative transesophageal echocardiograph (TEE) view to monitor for myocardial ischemia? A. Mid-esophageal four chamber view B. Transgastric mid-papillary left ventricular short axis view C. Mid-esophageal long axis view D. Mid-esophageal two chamber view
966. (B) The transgastric mid-papillary short axis view images the myocardium supplied by all three major coronary arteries: left anterior descending (LAD), left circumflex (CX), and right coronary (RCA) arteries. Thus, this view is preferred for the purpose of ischemia monitoring. The mid-esophageal four chamber view displays the anterolateral (LAD or CX) and inferoseptal (LAD or RCA) walls only, while the long axis view displays the anterior septal (LAD) and inferolateral (CX or RCA) walls. Two chamber views display the anterior (LAD) and inferior (RCA) walls (Kahn et al: Intraoperative echocardiography. In Kaplan: Essentials of Cardiac Anesthesia, ed 6, p 206).
967. Select the TRUE statement regarding cardiopulmonary resuscitation (CPR) and defibrillation by a health care provider in patients experiencing sudden cardiac arrest. A. Defibrillation times one should always precede CPR B. CPR should always be carried out for 2 minutes prior to defibrillation C. Two minutes of chest compressions alone (no ventilation) should be carried out prior to first shock D. If arrest less than 1 minute (witnessed), deliver one biphasic shock then five cycles of CPR
967. (D) The most frequent initial rhythm in a witnessed sudden cardiac arrest (SCA) is ventricular fibrillation (VF). Delays in either starting CPR or defibrillation reduce survival from SCA. Current recommendations for health care providers in any facility with an automated external defibrillator (AED) readily available is AED use within moments of the cardiac arrest. If an AED is not readily available, then CPR is started until the AED arrives at the scene. Recall one cycle of CPR is 30 compressions and two breaths. It is no longer recommended to deliver a three-shock sequence with biphasic defibrillators, because it is unlikely for the second or third shock to work after a failed first shock, and the second and third shocks may be harmful. After the shock, continue CPR for five cycles, then check for a pulse. If VF persists, repeat one shock and add epinephrine or vasopressin before or after a shock when an IV or intraosseous (IO) line is available. With monophasic defibrillators, it may be acceptable to deliver three-shock sequences, but all adult shocks should be 360 J. With out-of-hospital unwitnessed cardiac arrest by emergency medical service (EMS) personnel, five cycles of CPR (about 2 minutes) should be performed before checking the ECG and attempting defibrillation, especially when the response interval is greater than 4 minutes because shock effectiveness appears more successful after CPR (Part 1: Executive Summary: 2010 International Consensus on Cardiopulmonary Resuscitation and Emergency Cardiovascular Care Science with Treatment Recommendations, Circulation 122:S250–S275, 2010).
968. Which of the following medications blocks angiotensin at the receptor? A. Losartan (Cozaar) B. Terazosin (Hytrin) C. Lisinopril (Prinivil, Zestril) D. Spironolactone (Aldactone)
968. (A) The renin-angiotensin-aldosterone system is important in controlling blood pressure and blood volume. Renin helps to convert angiotensinogen to angiotensin I. Angiotensin-converting enzyme (ACE) helps to convert angiotensin I to angiotensin II. Angiotensin II has many pharmacologic actions including potent vasoconstriction action as well as stimulating aldosterone release from the adrenal gland. Losartan is an angiotensin receptor blocker (ARB) and is commonly used to treat hypertension. Patients taking ARBs, as well as patients who are on ACE inhibitors, are more prone to develop hypotension during anesthesia. In addition, the hypotension that develops may be more difficult to treat. That is why ARBs are commonly discontinued the day before surgery. Terazosin is an α1 blocker, lisinopril is an ACE inhibitor, spironolactone is a competitive antagonist to aldosterone, and amlodipine is a calcium channel blocker. Note: The endings of many generic drug names indicate the drug class (e.g., ARBs end in -sartan, α1 blockers end in -osin, ACE inhibitors end in -pril, and calcium channel blockers end in -dipine) (Miller: Miller’s Anesthesia, ed 8, p 377).
969. Untoward effects associated with administration of sodium bicarbonate during massive blood transfusion include each of the following EXCEPT A. Hyperkalemia B. Paradoxical cerebrospinal fluid acidosis C. Hypercarbia D. Hypernatremia
969. (A) Hemodynamically unstable cardiac dysrhythmias can result in hypoperfusion and metabolic acidosis. If severe metabolic acidosis is confirmed on arterial blood gases, intravenous sodium bicarbonate should be administered. Adverse effects associated with administration of sodium bicarbonate are well documented and include severe plasma hyperosmolality, paradoxic cerebrospinal fluid acidosis, hypernatremia, and hypercarbia, particularly in patients who are not adequately ventilated. Bicarbonate lowers potassium by lowering the extracellular hydrogen ion concentration, which results in lowering, not raising, the potassium concentration (Barash: Clinical Anesthesia, ed 7, p 1685).
970. Useful therapy for hypercyanotic “tet spells” in patients with tetralogy of Fallot might include any of the following EXCEPT A. Esmolol B. Morphine C. Phenylephrine D. Isoproterenol
970. (D) Hypercyanotic attacks primarily occur in infants 2 to 3 months of age and are frequently absent after 2 to 3 years of age. These attacks usually occur without provocation but can be associated with episodes of excitement, such as crying or exercise. The mechanism for these attacks is not known. It is believed, however, that hypercyanotic attacks occur as a result of spasm of the infundibular cardiac muscle or a decrease in systemic vascular resistance; both will exacerbate the right-to-left intracardiac shunt. Phenylephrine, an α-adrenergic receptor agonist, is the drug of choice for treatment of hypercyanotic attacks, because presumably phenylephrine increases systemic vascular resistance, which reduces the intracardiac right-to-left shunt and improves arterial oxygenation. Esmolol is also effective, presumably because it reduces spasm of the infundibular 278 Part 2 Clinical Sciences cardiac muscle. Isoproterenol with its β-mimetic effects reduces afterload and therefore increases right-to-left shunting and may exacerbate infundibular spasm. Because hypovolemia may increase sympathetic stimulation, adequate hydration with IV fluids may be helpful (Yao: Yao and Artusio’s Anesthesiology, ed 7, pp 910–912).
971. Sildenafil (Viagra) belongs to the same class of drugs as which of the following? A. Yohimbine B. Hydralazine C. Enalapril D. Milrinone
971. (D) Sildenafil (Viagra) is used for erectile dysfunction. Erection of the penis involves the local release of nitric oxide (NO), which increases cyclic guanine monophosphate (cGMP) in the corpus cavernosum. Sildenafil has no direct effects but inhibits phosphodiesterase type 5 (PDE5), which breaks down cGMP. The net effect is increasing cGMP. Yohimbine is an α-adrenergic blocker. Nitroglycerin and hydralazine are both direct-acting smooth muscle relaxants. Enalapril is an ACE inhibitor. Milrinone is an inhibitor of phosphodiesterase type 3 (PDE3) (Hemmings: Pharmacology and Physiology for Anesthesia, ed 1, p 413).
972. What is the minimal time after angioplasty and placement of a drug-eluting stent that dual antiplatelet therapy should be continued before considering stopping it for elective surgery? A. 3 months B. 6 months C. 1 year D. 18 months
972. (C) After a drug-eluting stent (DES) is placed, dual antiplatelet therapy (ASA + clopidogrel) is started to decrease the chance of stent thrombosis. Because stent thrombosis may develop months after a DES is placed, a minimum of 1 year of dual antiplatelet therapy is recommended before stopping the drugs prior to elective surgery. With newer generation (drug-eluting) stents with better pharmacologic platforms like everolimus, the ACC/AHA guidelines for DAPT (dual antiplatelet therapy) may be revised in the near future. If surgery is planned within 1 year of angioplasty and stent placement, consideration for using a bare-metal stent is recommended (where a minimum of 1 month of antiplatelet therapy is recommended) (Miller: Miller’s Anesthesia, ed 8, p 1185).
973. Bivalirudin is used as an anticoagulant for cardiopulmonary bypass primarily in patients with A. Heparin resistance B. Protamine allergy C. HIT type I D. HIT type II
973. (D) Heparin-induced thrombocytopenia (HIT) can be either nonimmune (type I) or immune (type II). HIT type I is a transient and clinically insignificant condition in which heparin binds to platelets causing a shortening of the platelet’s left span and a modest decrease in the platelet count. However, HIT type II can be a serious condition in which antibodies are formed (in 6%-15% of patients who are receiving unfractionated heparin for >5 days) to a complex of heparin and a platelet protein factor 4. This heparin-platelet factor 4 antibody complex binds to endothelial cells, which then stimulates thrombin production with a net result of both thrombocytopenia (>50% reduction in the platelet count) and venous and/or arterial thrombosis (<10% of cases). In patients with HIT, heparin should be avoided. In the setting of a thrombotic event or a patient with HIT needing anticoagulation (e.g., coronary artery bypass graft [CABG]) a direct thrombin inhibitor such as hirudin, lepirudin, bivalirudin, or argatroban should be used. Allergy to protamine can also be an indication for direct thrombin inhibitors, but HIT type II is a stronger reason for using bivalirudin (or other) than protamine allergy. Furthermore, heparinase, an enzyme derived from a gram-negative bacterium (Flavobacterium heparinum), can also be used to neutralize the effects of heparin. See also Answer 411 (Barash: Clinical Anesthesia, ed 7, p 416; Hines: Stoelting’s Anesthesia and Co-Existing Disease, ed 6, p 528).
974. Which of the following anatomic sites is associated with the LEAST incidence of central line infection? A. Internal jugular vein B. External jugular vein C. Subclavian vein D. Femoral vein
974. (C) Density of bacterial skin contamination and propensity to develop thrombosis in a cannulated vein are risk factors for the development of catheter-related bloodstream infections (CRBSIs). These risk factors are likely highest for femoral lines. In mostly observational studies, the risk of CRBSI was found to be lowest for subclavian central venous access. The Centers for Disease Control and Prevention recommends use of subclavian central lines when clinically possible (Miller: Miller’s Anesthesia, ed 8, p 1366).
975. The effects of clopidogrel (Plavix) can be reversed with A. Fresh frozen plasma B. Factor VIII concentrate C. Aprotinin D. None of the above Cardiovascular Physiology and Anesthesia 265
975. (D) Clopidogrel exerts its antithrombotic action by noncompetitively and irreversibly inhibiting the specific platelet adenosine diphosphate (ADP) receptor named P2Y12. Because the P2Y12 receptor is permanently affected, the duration of action of clopidogrel is for the life of the platelets. No drug reverses these effects, and only platelet transfusion can reverse the effects of clopidogrel (Miller: Miller’s Anesthesia, ed 8, p 1873).
976. A disadvantage of port access coronary artery bypass surgery utilizing the da Vinci robot versus “standard” coronary artery revascularization with cardiopulmonary bypass is A. Need for hypothermic cardiac arrest B. Greater incidence of intraoperative hypoxia C. Greater incidence of trauma to sternum D. Increased transfusion requirements
976. (B) Port access robotic surgery is a less invasive technique for coronary artery revascularization in selected patients. Access is gained to the heart through a left-sided minithoracotomy. This obviates the need for a sternotomy but does require one-lung ventilation and may result in hypoxia prior to initiation of cardiopulmonary bypass and after cessation. Hypothermic cardiac arrest is not required for robotic surgery (Miller: Miller’s Anesthesia, ed 8, pp 2586–2588). Cardiovascular Physiology and Anesthesia 279 Trachea Left mainstem bronchus Carina Right mainstem bronchus Upper lobe Lingula Lower lobe Lower lobe Middle lobe Upper lobe
977. A right-sided double-lumen tube will be used to separate ventilation of the right and left lungs for a left pneumonectomy. The plan for placement is to insert the distal tube into the trachea with a laryngoscope and then to advance the distal tube into the right mainstem bronchus under bronchoscopic guidance. After insertion of the tube with the laryngoscope, CO2 is seen on infrared spectrometer and the scope is passed through bronchial port until it exits the tube inside the lumen of the patient’s airway. A structure is seen that appears to be the carina. The scope is then passed into the right branch, and the structure in the picture below is visualized. The scope is located in the A. Right mainstem bronchus B. Left mainstem bronchus C. Lingular segment D. Right upper lobe
977. (D) One technique for placement of double-lumen tubes is to simply advance the tube such that the tip of the distal lumen is just above the carina, and then to place it exactly (the distal tube including cuff) into the right mainstem bronchus under direct vision using the bronchoscope. If the tube is initially advanced too far into the right mainstem bronchus (as it was in this question) a structure resembling the carina will be visualized. The “real” carina separates the left and right lungs, and if the bronchoscope is pushed into either the right or the left mainstem bronchi, a secondary “carina” will be visualized. In both cases, the secondary carina has only two branch points. On the left, the branches lead to the left upper lobe and left lower lobe. On the right, the branches lead to the right upper lobe and the right middle lobe. If the three lumens are seen after branching right from the “carina,” the “carina” in question is not the true carina but is, in fact, the branching point for the right upper and right middle lobes (see figure) (Barash: Clinical Anesthesia, ed 7, pp 1044–1046).
978. Which of the following maneuvers (after assuring proper tube placement) is LEAST likely to raise the Pao2 during one-lung ventilation with a double-lumen endotracheal tube? A. Continuous positive airway pressure (CPAP) to the nondependent lung B. PEEP to the dependent lung C. Continuous infusion of epoprostenol (Flolan) via central line D. Raising mean arterial pressure from 60 to 85 mm Hg
978. (C) During one-lung ventilation, ˙ V/ ˙Q abnormalities increase. After a few minutes of one-lung ventilation, hypoxic pulmonary vasoconstriction (HPV) develops, which helps decrease blood flow to the nondependent lung. Most patients will have adequate Pao2 when the dependent lung is ventilated with 100% oxygen, using a tidal volume (Vt) of 8 to 10 mL/kg and adjusting the respiratory rate to achieve a Paco2 of 40 mm Hg. In patients who develop hypoxemia with these settings, correction of poor hemodynamics as well as checking the position of the double-lumen tube is done first, then adding CPAP to the nondependent lung, adding PEEP to the dependent lung, or having the surgeon clamp the PA to the lung about to be removed, will help decrease the ˙ V/ ˙Q mismatch. Occasionally, intermittent inflation of the nondependent lung with 100% oxygen will be needed. Epoprostenol and nitric oxide (NO) would inhibit HPV and might lead to an increase in shunt and a decrease in Pao2 (Miller: Miller’s Anesthesia, ed 8, pp 1969–1970).
979. Which of the following drugs or interventions will cause the LEAST increase in heart rate in the transplanted denervated heart? A. Glucagon B. Atropine C. Isoproterenol D. Norepinephrine
979. (B) The transplanted heart is essentially denervated and initially has an intrinsic rate of about 110 beats/ min. About 25% of patients eventually develop a bradycardia that will require implantation of a permanent cardiac pacemaker. If bradycardia does develop, drugs that exert their effect by blocking the parasympathetic branches of the autonomic nervous system (e.g., atropine) will have no effect. Direct-acting drugs such as glucagon, isoproterenol, epinephrine, and norepinephrine will still be effective. Isoproterenol is commonly used for increasing heart rate in cardiac transplant recipients. Epinephrine and norepinephrine may have exaggerated β-mimetic effects on the heart rate because the increase in blood pressure will not lead to a reflex slowing of the heart rate via the baroreceptor 280 Part 2 Clinical Sciences reflexes (i.e., efferent vagus nerve). Drugs with both direct and indirect effects such as ephedrine evoke a less intense response. Implanted mechanical pacemakers work normally in heart transplant recipients since the cardiac leads are placed directly into the myocardium (Barash: Clinical Anesthesia, ed 7, p 1848; Miller: Miller’s Anesthesia, ed 8, p 2066).
980. A patient with known Wolff-Parkinson-White (WPW) syndrome develops a wide complex tachycardia during a hernia operation under general anesthesia. Vital signs are stable and pharmacologic treatment is desired. Which of the following drugs is MOST likely to be successful in controlling heart rate in this patient? A. Verapamil B. Esmolol C. Adenosine D. Procainamide
980. (D) Patients with WPW syndrome have an accessory pathway known as the bundle of Kent, which connects the atria with ventricles without passing through the atrioventricular (AV) node. AV nodal reentrant tachycardia (AVNRT) is the most common tachydysrhythmia associated with WPW syndrome and comprises 95% of arrhythmias associated with this syndrome. Greater than 90% of the time, conduction is orthodromic; that is, conduction passes through the AV node and the His-Purkinje system. Such conduction results in narrow, complex tachycardia, and any of the drugs mentioned in this question could be used to control rate. AVNRTs that travel through the accessory pathway (<10% of AVNRTs) are manifested as wide complex tachycardias (antidromic conduction) and are not amenable to treatment with β-blockers, calcium channel blockers, adenosine, or digoxin, and can, in fact, be made worse with these drugs. Intravenous procainamide, a class Ia antidysrhythmic agent, is the only useful pharmacologic agent among the drugs listed in the question. If pharmacologic therapy fails, electrical cardioversion is indicated to control rate (Fleisher: Anesthesia and Uncommon Diseases, ed 6, p 33).
981. A 63-year-old patient with a DDD-R pacemaker is scheduled for right hemicolectomy. The indication for pacemaker implantation was sick sinus syndrome, and the pacemaker has been reprogrammed to the asynchronous (DOO) mode at a rate of 70 for surgery. After induction, the patient’s native heart rate rises to 85 beats/min with blood pressure 130/90 mm Hg. Which of the following actions would be MOST appropriate? A. Turn off pacemaker for duration of case B. Administer lidocaine C. Administer esmolol D. Observe
981. (C) The DOO setting is the simplest dual chamber pacing mode. Because of concerns about electromagnetic interference from an electrical surgical unit (ESU) (i.e., the Bovie), pacemakers may be temporarily programmed into the asynchronous mode for surgery and then reprogrammed to the presurgical mode in the recovery room. With the VOO or DOO modes, the possibility of an R-on-T phenomenon exists if the native heart rate exceeds the programmed rate or when there are frequent premature ventricular contractions (PVCs) or premature atrial contractions (PACs). In the latter case, repolarization (from a PAC or PVC) may occur at the precise moment that the pacemaker is discharging (R wave). Turning off an implanted pacemaker would be extremely difficult in the middle of an operation. Furthermore, a slow rhythm could occur wherein pacing were again necessary. Intravenous lidocaine would be useless in this setting, as would switching the volatile agent from isoflurane to desflurane. At concentrations greater than 1 minimum alveolar concentration (MAC), desflurane can actually increase heart rate further. Administration of esmolol would slow the heart rate down below 70 so that the pacemaker could again “lead” (Miller: Miller’s Anesthesia, ed 8, pp 1464–1467).
982. The main advantage of milrinone is that it lacks which side effect, compared with amrinone, for long-term use? A. Tachycardia B. Hypothyroidism C. Thrombocytopenia D. Hyperglycemia
982. (C) Milrinone and amrinone (inamrinone) are phosphodiesterase type 3 (PDE3) inhibitors that increase cyclic adenosine monophosphate (cAMP) levels in cardiac and smooth muscle cells. They both produce positive inotropic effects and vasodilation (arterial and venous). Unlike milrinone, amrinone rapidly produces clinically significant thrombocytopenia especially after prolonged use (Hemmings: Pharmacology and Physiology for Anesthesia, ed 1, pp 390–391).
983. Systemic inflammatory response syndrome (SIRS) differs from sepsis in that patients with SIRS have A. A normal temperature B. A heart rate less than 90 beats/min C. A normal white blood cell count D. No documented infection
983. (D) SIRS can result from a variety of severe clinical insults, including cardiopulmonary bypass. The diagnosis of SIRS requires the presence of two or more of the following four conditions: temperature greater than 38° C or less than 36° C; heart rate greater than 90 beats/min; respiratory rate more than 20 breaths/min or a Paco2 of less than 32 mm Hg; a leukocyte count greater than 12,000 or less than 4000/mm3 or greater than 10% immature (band) forms. Sepsis is SIRS plus a documented infection (Barash: Clinical Anesthesia, ed 7, p 1590).
984. Arrange the percutaneous insertion sites from nearest to farthest for placement of a PA catheter. A. Left internal jugular, right internal jugular, antecubital, femoral B. Right internal jugular, left internal jugular, antecubital, femoral C. Right internal jugular, left internal jugular, femoral, antecubital D. Left internal jugular, right internal jugular, femoral, antecubital 266 Part 2 Clinical Sciences
984. (C) When a PA catheter is placed from the right internal jugular vein, the right atrium typically is reached at 20 to 25 cm, the right ventricle at 30 to 35 cm, the PA at about 40 to 45 cm, and the wedge position at 45 to 55 cm. Add about 5 to 10 cm from the left internal jugular vein and the left and right external jugular veins, 15 cm from the femoral veins, and 30 to 35 cm from the antecubital veins (Miller: Miller’s Anesthesia, ed 8, pp 1371–1372).
985. A pulmonary artery catheter capable of continuously monitoring SvO2 is placed in a patient for coronary artery bypass surgery. Just before instituting cardiopulmonary bypass, the SvO2 falls from 85% to 71%. Which of the following could account for this change in SvO2? A. Cooling the patient to 27° C B. Transfusion of two units packed red blood cells C. Epinephrine, 25 μg IV D. Myocardial ischemia
985. (D) The SvO2 reflects the overall ability of cardiac output to adequately meet metabolic needs and is thus a comprehensive measure of cardiac performance. There are several factors that can influence SvO2. These factors are easily understood by rearranging the Fick equation as follows: Cardiovascular Physiology and Anesthesia 281 See explanation to Question 106 for complete definition of O2 content. Thus, So2 can be reduced by a decrease in Sao2, CO, and hemoglobin, and an increase in O2. In the present case, labetalol reduces cardiac output through its negative inotropic effect. These factors must be accounted for when interpreting So2 measurements (Miller: Miller’s Anesthesia, ed 8, p 1387).
986. Which of the following terms refers to myocardial relaxation or diastole? A. Inotropy B. Chronotropy C. Dromotropy D. Lusitropy
986. (D) Inotropy refers to the force and velocity of ventricular contractions when preload and afterload are held constant. Chronotropy refers to the heart rate. Dromotropy refers to the conduction of impulses along conductive tissue. Bathmotropy refers to muscular excitation in response to a stimulus. Lusitropy refers to myocardial relaxation or diastole. A decrease in lusitropy is seen with the aging myocardium (Miller: Miller’s Anesthesia, ed 8, p 485).
987. A 31-year-old female with primary pulmonary hypertension is scheduled for a mastectomy. Pharmacologic agents that might be useful in reducing pulmonary vascular resistance include each of the following EXCEPT A. Prostaglandin I2 (epoprostenol) B. Oxygen C. Nitrous oxide D. Milrinone
987. (C) PA hypertension is defined as a mean PA pressure of greater than 25 mm Hg at rest or greater than 30 mm Hg with exercise. Epoprostenol, also called prostacyclin (PGI2), and Flolan, as well as alprostadil (PGE1), are usually administered by a continuous IV infusion centrally, producing both pulmonary and systemic vasodilation, but because systemic hypotension is common, their use is limited. Recently, inhaled epoprostenol and alprostadil have been described to reduce the systemic side effects. Because hypoxia produces pulmonary vasoconstriction, oxygen therapy is often administered to reduce the magnitude of pulmonary vasoconstriction that may develop. Inhaled nitric oxide (NO) in concentrations from 1 to 80 ppm (typically 20-40 ppm) produces smooth muscle relaxation and reduces PA pressures. Because NO is so rapidly metabolized, it has minimal systemic effects. Milrinone is a phosphodiesterase inhibitor that reduces pulmonary vascular resistance while having some inotropic effects. (If right ventricle failure is severe, norepinephrine or epinephrine may be preferred as an inotrope even though PA pressures will increase.) Milrinone is usually administered IV, but recently inhaled milrinone has been described to reduce systemic side effects. Inhaled volatile anesthetics tend to decrease PA resistance. On the other hand, NO tends to increase pulmonary vascular resistance and is not recommended to be used in patients with pulmonary hypertension (Miller: Basics of Anesthesia, ed 6, p 435; Hines: Stoelting’s Anesthesia and Co-Existing Disease, ed 6, pp 116–117).
988. Pulmonary vascular resistance as a function of lung volume is the LEAST at which volume? A. Total lung volume B. Residual volume C. Functional residual capacity (FRC) D. Expiratory reserve volume
988. (C) Pulmonary vascular resistance (PVR) is the sum of the resistance of small and large blood vessels and is least at the FRC. When the lung volume increases above FRC, PVR increases due to alveolar compression of the small intra-alveolar blood vessels. When the lung volume decreases below FRC, PVR increases due to the mechanical tortuosity or kinking of the large extra-alveolar blood vessels. Pulmonary vascular resistance also increases in areas of atelectasis when hypoxia causes pulmonary vasoconstriction (HPV) (Miller: Miller’s Anesthesia, ed 8, pp 681–687).
989. A 45-year-old patient with hypertrophic cardiomyopathy is anesthetized for skin grafting after suffering third-degree burns on his legs. As skin is harvested from his back, his heart rate rises and his systolic blood pressure falls to 85 mm Hg. Which of the following interventions is LEAST likely to improve this patient’s hemodynamics? A. Administration of esmolol B. Fluid bolus C. Dobutamine infusion D. Administration of sufentanil
989. (C) Hypertrophic cardiomyopathy is characterized by left ventricular outflow tract (LVOT) obstruction and is caused by asymmetric hypertrophy of the intraventricular septal muscle. The compensatory mechanism to maintain cardiac output is left ventricular hypertrophy. Events that increase outflow obstruction include increased myocardial contractility (e.g., β stimulation), decreased ventricular preload (e.g., hypovolemia, venodilation, tachycardia with reduced time to fill the ventricle, positivepressure ventilation), and decreased afterload (e.g., vasodilation). Perioperative management is aimed at preventing an increase in outflow obstruction. Hypotension often responds by increasing preload (fluid administration) and/or increasing afterload (α-adrenergic stimulation with phenylephrine). β-Blockade (e.g., esmolol) can help slow a fast heart rate and allow more time for ventricular filling as well as decreasing contractility. If the patient has a painful catecholamine response to surgery, narcotics may be helpful. Drugs with β-adrenergic activity such as ephedrine, dopamine, and dobutamine are contraindicated, because they increase myocardial contractility and heart rate, which causes more LVOT obstruction (Miller: Basics of Anesthesia, ed 6, p 403).
990. A 59-year-old patient is scheduled for right knee replacement. The patient has a long history of congestive heart failure (CHF) with 87% oxygen saturation while breathing room air in the holding area. Rales are audible throughout both lung fields with the patient upright. The MOST appropriate plan would be A. Arterial line and spinal with isobaric bupivacaine B. Arterial line, etomidate induction, sevoflurane, intraoperative TEE C. Arterial line, central venous pressure line (CVP), ketamine induction, N2O narcotic anesthetic, furosemide, milrinone D. Cancel the case
990. (D) CHF is one of the six major risk factors for patients undergoing elective major noncardiac surgery. The other major risk factors are high-risk surgery, ischemic heart disease, cerebrovascular disease, insulindependent diabetes mellitus, and preoperative serum creatinine of greater than 2 mg/dL. As this is an elective case, patients with CHF need to be optimally managed prior to surgery. This patient does not appear to be optimally managed, and surgery should be canceled (Miller: Basics of Anesthesia, ed 6, p 402).
991. Which of the following drugs is LEAST likely to cause unfavorable hemodynamic changes in patients with severe mitral stenosis? A. Ketamine B. Remifentanil C. Pancuronium D. Desflurane
991. (B) Symptoms of mitral stenosis develop when the mitral valve orifice (normally 4-6 cm2) is reduced 50% or more. Goals in management revolve around four main areas: preventing tachycardia (which 282 Part 2 Clinical Sciences decreases the diastolic time needed for LV filling); avoiding a marked increase in central blood volume (which may cause atrial fibrillation or CHF); preventing sudden drug-induced decreases in systemic vascular resistance (which may cause hypotension and reflex tachycardia); and avoiding hypoxia and hypercarbia (which may exacerbate pulmonary hypertension and cause right ventricular failure). Ketamine, pancuronium, and a rapid increase in the concentration of desflurane may all cause tachycardia, which results in a decrease in cardiac output. Nitrous oxide can be used in most cases; however, in severe cases where there is an increase in PA pressure, avoiding nitrous oxide may be beneficial. Remifentanil as well as fentanyl and sufentanil give good analgesia without increasing heart rate (HR) (Miller: Basics of Anesthesia, ed 6, pp 393–394).
992. You made an infusion of dopamine by mixing 200 mg of dopamine in 250 mL of sodium chloride (NS) or 5% dextrose injection (D5W). What is the infusion pump rate when infusing dopamine at a rate of 5 μg/kg/min for this 70-kg patient? A. 10 mL/hr B. 16 mL/hr C. 20 mL/hr D. 26 mL/hr
992. (D) Dopamine can be mixed in either D5W or normal saline (NS) solution. A mixture of 200 mg of dopamine in 250 mL of D5W would yield a concentration of 800 μg/mL (200 mg/250 mL = 0.8 mg/ mL = 800 μg/mL). At an infusion rate of 5 μg/70 kg/60 min, one would need 5 μg × 70 kg × 60 min = 21,000 μg/hr. 21,000 μg/hr ÷ 800 μg/mL = 26 mL/hr.
993. A79-year-old patient returns to the operating room with cardiac tamponade after three-vessel coronary artery grafting. In addition to gentle positive-pressure ventilation, which of the following permutation in hemodynamics would be MOST beneficial in this scenario? A. Increased preload, slow heart rate, increased afterload B. Normal preload, slow heart rate, decreased afterload C. Normal preload, fast heart rate, decreased afterload D. Increased preload, fast heart rate, increased afterload Cardiovascular Physiology and Anesthesia 267
993. (D) Patients with stenotic heart valves (mitral stenosis [MS], aortic stenosis [AS]) tend to do better with slow normal heart rates because it takes time for the heart chambers to fill during diastole (MS) or empty during systole (AS). Tachycardia in patients with AS may be especially harmful, because tachycardia leads to myocardial ischemia and ventricular dysfunction due to the thick ventricular walls. With cardiac valves that are insufficient (e.g., aortic insufficiency [AI]), faster heart rates are helpful because regurgitation occurs during diastole and faster rates decrease diastolic time. Patients with AI also benefit from a lower systemic vascular resistance (SVR), which promotes a better cardiac output (high SVR increases the amount of regurgitation during diastole). Too low of an SVR in these patients may lead to decreased coronary artery filling, because filling occurs during diastole. With hypertrophic cardiomyopathy, a high SVR helps to decrease the outflow obstruction, but a fast heart rate increases outflow obstruction. Patients with cardiac tamponade have a fixed ejection fraction that is very dependent upon high filling pressures, and the cardiac output is very much dependent upon the heart rate. A high SVR helps to maintain blood pressure in the face of the decreased cardiac output (Miller: Basics of Anesthesia, ed 6, pp 404–405).
994. Which of the following treatments would be the LEAST useful in treatment of the rhythm shown below? A. Procainamide B. Magnesium C. Overdrive pacing D. Unsynchronized cardioversion How long does the antiplatelet effect of each of the following medications last?
994. (A) The figure shows torsades de pointes (“twisting of the points”) in a patient who had a QTc interval of 450 msec and was having an acute myocardial infarction (MI). This condition can be induced by drugs (e.g., quinidine, procainamide, and phenothiazines such as droperidol), electrolyte abnormalities (e.g., hypokalemia, hypomagnesemia), and acute cardiac ischemia or infarction. If a prolonged QT interval is present, the shortening of the QT interval is performed as time permits (e.g., correction of electrolyte abnormalities). In the past, isoproterenol was used (shortens QT interval) but overdrive atrial or ventricular pacing is the more definitive treatment. Magnesium sulfate has also been used and is recommended by many as the first-line emergency drug. If the patient does not have a prolonged QT interval, standard drugs used for ventricular tachycardia can be used. If the patient becomes hemodynamically unstable, unsynchronized shocks (defibrillation doses) should be delivered (Miller: Miller’s Anesthesia, ed 8, pp 3197–3198).
995. P wave flattening, widening of the QRS complex, peaked T wave
995. (B) The ECG recording is a reflection of cardiac muscle electrical activity, and, although it is primarily used to diagnose arrhythmias or cardiac ischemia, the changes that occur may be related to electrolyte disturbances. Both hyperkalemia and hypokalemia are associated with impaired myocardial contractility, conduction disturbances, and cardiac arrhythmias. With hyperkalemia, the earliest changes are narrowing and peaking of the T wave (7-9 mEq/L). More severe degrees of hyperkalemia (>7 mEq/L) produce widening of the QRS complex that can merge with the T wave producing a sine wave pattern, decrease in P-wave amplitude, and an increase in the PR interval. The terminal event would be VF or asystole. The earliest changes with hypokalemia include T-wave flattening or inversion, appearance of U waves, and ST segment depression. With severe hypokalemia, the PR interval may become prolonged and the QRS complex may widen, then arrhythmias develop. Cardiovascular Physiology and Anesthesia 283 Hypocalcemia prolongs the QT interval (ST portion), whereas hypercalcemia shortens the QT interval. Hypernatremia and hyponatremia do not produce characteristic changes in the ECG (Barash: Clinical Anesthesia, ed 7, pp 1701–1720).
996. Depressed ST segments, flat T wave, U wave present
996. (A) The ECG recording is a reflection of cardiac muscle electrical activity, and, although it is primarily used to diagnose arrhythmias or cardiac ischemia, the changes that occur may be related to electrolyte disturbances. Both hyperkalemia and hypokalemia are associated with impaired myocardial contractility, conduction disturbances, and cardiac arrhythmias. With hyperkalemia, the earliest changes are narrowing and peaking of the T wave (7-9 mEq/L). More severe degrees of hyperkalemia (>7 mEq/L) produce widening of the QRS complex that can merge with the T wave producing a sine wave pattern, decrease in P-wave amplitude, and an increase in the PR interval. The terminal event would be VF or asystole. The earliest changes with hypokalemia include T-wave flattening or inversion, appearance of U waves, and ST segment depression. With severe hypokalemia, the PR interval may become prolonged and the QRS complex may widen, then arrhythmias develop. Cardiovascular Physiology and Anesthesia 283 Hypocalcemia prolongs the QT interval (ST portion), whereas hypercalcemia shortens the QT interval. Hypernatremia and hyponatremia do not produce characteristic changes in the ECG (Barash: Clinical Anesthesia, ed 7, pp 1701–1720).
997. Normal or increased PR interval, short QT interval A. Hypokalemia B. Hyperkalemia C. Hyponatremia D. Hypercalcemia 268 Cardiovascular Physiology and Anesthesia Answers, References, and Explanations
997. (D) The ECG recording is a reflection of cardiac muscle electrical activity, and, although it is primarily used to diagnose arrhythmias or cardiac ischemia, the changes that occur may be related to electrolyte disturbances. Both hyperkalemia and hypokalemia are associated with impaired myocardial contractility, conduction disturbances, and cardiac arrhythmias. With hyperkalemia, the earliest changes are narrowing and peaking of the T wave (7-9 mEq/L). More severe degrees of hyperkalemia (>7 mEq/L) produce widening of the QRS complex that can merge with the T wave producing a sine wave pattern, decrease in P-wave amplitude, and an increase in the PR interval. The terminal event would be VF or asystole. The earliest changes with hypokalemia include T-wave flattening or inversion, appearance of U waves, and ST segment depression. With severe hypokalemia, the PR interval may become prolonged and the QRS complex may widen, then arrhythmias develop. Cardiovascular Physiology and Anesthesia 283 Hypocalcemia prolongs the QT interval (ST portion), whereas hypercalcemia shortens the QT interval. Hypernatremia and hyponatremia do not produce characteristic changes in the ECG (Barash: Clinical Anesthesia, ed 7, pp 1701–1720).
998. Clopidogrel How long does the antiplatelet effect of each of the following medications last? A. 3 days B. 7 days C. 21 days D. Life of platelet
998. (B) Patients with cardiovascular disease often present for noncardiac surgery, both elective and emergent. Many patients receive antiplatelet therapy, and knowledge of the duration of action is important. Nonsteroidal anti-inflammatory drugs such as ibuprofen reversibly inhibit cyclooxygenase and prevent the synthesis of thromboxane A2, as well as PGI2, but the former effects predominate clinically. Aspirin antiplatelet effects last for the life of the platelet (7-10 days). Thienopyridine derivatives, which include clopidogrel (Plavex) and ticlopidine (Ticlid), inhibit platelet aggregation by interference with fibrinogen binding. The antiplatelet effects from Ticlopidine therapy last 14 to 21 days, while clopidogrel’s duration of action is shorter (7 days) (Miller: Basics of Anesthesia, ed 6, pp 358–359).
999. Ticlopidine How long does the antiplatelet effect of each of the following medications last? A. 3 days B. 7 days C. 21 days D. Life of platelet
999. (C) Patients with cardiovascular disease often present for noncardiac surgery, both elective and emergent. Many patients receive antiplatelet therapy, and knowledge of the duration of action is important. Nonsteroidal anti-inflammatory drugs such as ibuprofen reversibly inhibit cyclooxygenase and prevent the synthesis of thromboxane A2, as well as PGI2, but the former effects predominate clinically. Aspirin antiplatelet effects last for the life of the platelet (7-10 days). Thienopyridine derivatives, which include clopidogrel (Plavex) and ticlopidine (Ticlid), inhibit platelet aggregation by interference with fibrinogen binding. The antiplatelet effects from Ticlopidine therapy last 14 to 21 days, while clopidogrel’s duration of action is shorter (7 days) (Miller: Basics of Anesthesia, ed 6, pp 358–359).
1000. ASA How long does the antiplatelet effect of each of the following medications last? A. 3 days B. 7 days C. 21 days D. Life of platelet

1000. (D) Patients with cardiovascular disease often present for noncardiac surgery, both elective and emergent. Many patients receive antiplatelet therapy, and knowledge of the duration of action is important. Nonsteroidal anti-inflammatory drugs such as ibuprofen reversibly inhibit cyclooxygenase and prevent the synthesis of thromboxane A2, as well as PGI2, but the former effects predominate clinically. Aspirin antiplatelet effects last for the life of the platelet (7-10 days). Thienopyridine derivatives, which include clopidogrel (Plavex) and ticlopidine (Ticlid), inhibit platelet aggregation by interference with fibrinogen binding. The antiplatelet effects from Ticlopidine therapy last 14 to 21 days, while clopidogrel’s duration of action is shorter (7 days) (Miller: Basics of Anesthesia, ed 6, pp 358–359).

1001. Ibuprofen How long does the antiplatelet effect of each of the following medications last? A. 3 days B. 7 days C. 21 days D. Life of platelet
1001. (A) Patients with cardiovascular disease often present for noncardiac surgery, both elective and emergent. Many patients receive antiplatelet therapy, and knowledge of the duration of action is important. Nonsteroidal anti-inflammatory drugs such as ibuprofen reversibly inhibit cyclooxygenase and prevent the synthesis of thromboxane A2, as well as PGI2, but the former effects predominate clinically. Aspirin antiplatelet effects last for the life of the platelet (7-10 days). Thienopyridine derivatives, which include clopidogrel (Plavex) and ticlopidine (Ticlid), inhibit platelet aggregation by interference with fibrinogen binding. The antiplatelet effects from Ticlopidine therapy last 14 to 21 days, while clopidogrel’s duration of action is shorter (7 days) (Miller: Basics of Anesthesia, ed 6, pp 358–359).